CDS Mathematics (4 Pages Per Sheet)

You might also like

Download as pdf or txt
Download as pdf or txt
You are on page 1of 195

2020-21

EDITION Pathf nder

Pathf nder

ARIHANT PUBLICATIONS (INDIA) LIMITED


All Rights Reserved

© Publisher
No part of this publication may be re-produced, stored in a retrieval system or by any means,
electronic, mechanical, photocopying, recording, scanning, web or otherwise without the written
permission of the publisher. Arihant has obtained all the information in this book from the sources
More than believed to be reliable and true. However, Arihant or its editors or authors or illustrators don’t take
any responsibility for the absolute accuracy of any information published and the damage or loss
suffered thereupon.
All disputes subject to Meerut (UP) jurisdiction only.

Administrative & Production Offices


Regd. Office
‘Ramchhaya’ 4577/15, Agarwal Road, Darya Ganj, New Delhi -110002
Tele: 011- 47630600, 43518550

Head Office
Kalindi, TP Nagar, Meerut (UP) - 250002
Tel: 0121-7156203, 7156204

Sales & Support Offices


Agra, Ahmedabad, Bengaluru, Bareilly, Chennai, Delhi, Guwahati,
Hyderabad, Jaipur, Jhansi, Kolkata, Lucknow, Nagpur & Pune.
Compiled & Edited by
Arihant ‘Expert Team’ ISBN 978-93-24196-24-8

PO No : TXT-XX-XXXXXXX-X-XX
Published by Arihant Publications (India) Ltd.
For further information about the books published by Arihant, log on to
www.arihantbooks.com or e-mail at info@arihantbooks.com
Follow us on
ARIHANT PUBLICATIONS (INDIA) LIMITED
CONTENTS
CHEMISTRY BIOLOGY
Matter 632-634 Cell : The Unit of Life 682-686
CDS Solved Paper 2020 II 3-39 23. Triangles 248-271 Classification of Plants and Animals 686-690
Atomic Structure 634-636
CDS Solved Paper 2020 I 40-78 24. Quadrilateral and Polygon 272-286 Genetics and Molecular Biology and
Radioactivity 636-637
CDS Solved Paper 2019 II 1-40 25. Circle 287-310 Evolution of Life 691-697
CDS Solved Paper 2019 I 41-82 Chemical Bonding and Redox Reactions 638-640 Plant Morphology and Physiology 697-704
26. Area and Perimeter of Plane figures 311-335
CDS Solved Paper 2018 II 1-37 Gas Laws and Solutions 640-641 Animal Physiology 704-723
27. Surface Area and Volume of solids 336-362
Acids, Bases and Salts 642-643 Human Health and Diseases 723-726
28. Statistics 363-384
MATHEMATICS Chemical Thermodynamics and Applied Biology 726-731
Surface Chemistry 644-645 Practice Exercise 732-750
1. Number System 3-19 GENERAL ENGLISH
2. Sequence and Series 20-24 Electrochemistry 645-647
1. Spotting the Errors 387-434
3. HCF and LCM of Numbers 25-31 Inorganic Chemistry 647-654 GENERAL STUDIES
2. Vocabulary 435-451
4. Decimal Fractions 32-37 Organic Chemistry 654-656 1. History 753-852
3. Synonyms 452-462
2. Geography 853-946
5. Square Roots and Cube Roots 38-46 Man-Made Materials 656-658
4. Antonyms 463-474 3. Indian Polity 947-1005
6. Time and Distance 47-59 Environment and its Pollution 658-659
5. Idioms and Phrases 475-482 4. Indian Economy 1006-1049
7. Time and Work 60-68 Practice Exercise 660-681
6. Sentence Completion 483-499 5. General Knowledge 1050-1092
8. Percentage 69-76
7. Sentence Improvement 500-514
9. Simple Interest 77-82
8. Ordering of Words and Sentences 515-543
10. Compound Interest 83-91
9. Comprehension 544-576
11. Profit and Loss 92-100
12. Ratio and Proportion 101-112
GENERAL SCIENCE
13. Logarithm 113-119
14. Algebraic Operations 120-131
PHYSICS
15. HCF and LCM of Polynomials 132-138 Measurement, Motion, Work,
Energy and Power 579-586
16. Rational Expressions 139-143
Rotational Motion and Gravitation 587-589
17. Linear Equations 144-159
Properties of Matter 590-592
18. Quadratic Equations and Inequalities 160-182
Heat and Thermodynamics 593-596
19. Set Theory 183-193
Oscillations and Waves 596-599
20. Measurements of Angles and
Trigonometric Ratios 194-224 Optics 600-606

21. Height and Distance 225-236 Electric Current 606-609

22. Lines and Angles 237-247 Modern Physics 609-611


Practice Exercise 612-631
Pathf nder
SYLLABUS
PAPER I ENGLISH (only real roots to be considered). Simultaneous
linear equations in two unknowns-analytical and
The question paper will be designed to test the
graphical solutions. Simultaneous linear in-
candidates' understanding of English and
equations in two variables and their solutions.
years as on 2nd July in accordance with the year of workman-like use of words. Questions in English are
ABOUT THE EXAMINATION examination. (upper age relation of upto 26 years for from Synonyms, Antonyms, Reading
Practical problems leading to two simultaneous
A career in Defence services is the much sought after linear equations
candidates holding Commercial Pilot License issued by Comprehension, Para jumbles, Error spotting,
occupation today for those young and courageous youths of or in-equations in two variables or quadratic
DGCA). Jumbled sentences, Sentence connection and till in
the country who are willing to dedicate their lives to defend equations in one variable and their solutions. Set
(iii) For Officer’s Training Academy-male candidates (married the blanks.
the country and its people, the Combined Defence Services language and set notation, Rational expressions
Examination is the first test before they can join one of the or unmarried) and female candidates (unmarried and
issueless widows or divorces who have not remarried) and conditional identities, laws of indices.
best defence forces in the world i.e. the Indian Armed Forces.
having age not less than 18 years as on 1st July and not
PAPER II GENERAL KNOWLEDGE
UPSC conducts the Combined Defence Services Exam twice more than 24 years as on 2nd July in accordance with the
General Knowledge, including knowledge of Trigonometry
every year generally in February and August for recruiting year of examination are eligible. current events and such matters of everyday Sine x, cosine x, tangent x when 0° < x< 90°. Values
officers for the Indian Military Academy, Naval Academy, Air observation and experience in their scientific of sin x, cos x and tan x, for x = 0°, 30°, 45°, 60° and
Force Academy and Officers Training Academy. Male aspects as may be expected of an educated person 90°. Simple trigonometric identities. Use of
candidates can join IMA, Naval and Air Force but for female
EDUCATIONAL QUALIFICATION who has not made a special study of any scientific trigonometric tables. Simple cases of height and
candidates OTA in army is available. (i) For IMA/OTA a degree from a recognised university or subject. The paper will also include questions on
equivalent.
distance.
The examination comprises of two stages-the first stage history of India and geography of a nature which
consists of written test and those who qualify the written test (ii) For Naval Academy degree in engineering from a
candidates should be able to answer without Geometry
are called for interview by Service Selection Board (SSB) for recognised university.
special study. Lines and angles, Plane and plane figures
Intelligence and Personality Test. (iii) For Air Force Academy degree from a recognised
university (with Physics and Mathematics at 10+2 level) theorems on
or Bachelor of Engineering. PAPER III ELEMENTARY Properties of angles at a point
NATIONALITY Parallel lines
A candidate must be either MATHEMATICS
(i) Indian citizen, or
EXAMINATION PATTERN Arithmetic
Sides and angles of a triangle
(i) The written examination will be as follows: Congruency of triangles
(ii) A subject of Bhutan, or Number system-Natural numbers, Integers, Rational Similar triangles
(iii) A subject of Nepal, or No. of Maximum and real numbers. Concurrence of medians and altitudes
Subject Duration
(iv) A Tibetan refugee who came over to India before 1st Questions Marks Fundamental Operations-addition, subtraction, Properties of angles, sides and diagonals of a
January, 1962 with the intention of permanently setting English 2 hours 120 100 multiplication, division, square roots, decimal parallelogram, rectangle and square
in India, or fractions. Circle and its properties, including tangents and
General
(v) A person of Indian origin who has migrated from Knowledge 2 hours 120 100
normals
Pakistan, Burma, Sri Lanka and East African countries of
Elementary
Unitary Method Locus point
Kenya, Uganda, the United Republic of Tanzania, Zambia, 2 hours 100 100 Time and distance, time and work, percentages,
Mathematics
Malawi, Zaire and Ethiopia and Vietnam with the applications to simple and compound interest,
intention of permanently setting in India. Mensuration
profit and loss, ratio and proportion, variation. Areas of squares, rectangles, parallelograms,
Provided that a candidate belonging to categories (ii), Note- For admission in OTA, candidates are required to give
(iii), (iv) and (v) above shall be a person in whose favour a only English and General knowledge triangle and circle. Areas of figures,
certificate of eligibility has been issued by the papers of 200 marks. Elementary Number which can be split up into the figures (Field Book).
Government of India. Certificate of eligibility, will not , One third of the marks will be deducted for each wrong Theory Division algorithm, prime and composite Surface area and volume of cuboids,
however, be necessary in the case of candidate who are answer. numbers, Tests of divisibility by 2, 3, 4, 5, 9 and 11 lateral surface and volume of right circular cones
Gorkha subjects of Nepal. Multiples and factors, Factorisation Theorem, HCF and cylinders. Surface area and volume of spheres.
(ii) The SSB procedure consists of two stage selection
process. Only those candidates who clear the stage I are and LCM. Evaluation algorithm, logarithms to base
AGE LIMIT, SEX AND MARITAL STATUS permitted to appear for stage II. 10, laws of logarithm, use of logarithms tables. Statistics
(i) For IMA and Indian Naval Academy-unmarried male (a) Stage I comprises of Officer Intelligence Rating (OIR) Collection and tabulation of statistical data.
candidates having age not less than 18 years tests and Picture Perception, Description Test (PP and Algebra Graphical representation of frequency polygons,
on 1st July and not more than 23 years on 2nd July in DT) Basic operations, simple factors, Remainder histograms, bar charts, pie charts, etc. Measures of
accordance with the year of examination (b) Stage II comprises of Interview, Group Testing Officer Theorem, HCF, LCM, Theory of polynomials,
tasks, Psychology Tests and the Conference. These
Central Tendency.
are eligible. solutions of quadratic equations, relation between
(ii) For Air Force Academy-male candidates having age not tests are conducted over 4 days.
its roots and coefficients
less than 19 years as on 1st July and not more than 23
01
MATHEMATICS Quadratic Equations and Inequalities 3 4 CDS Pathfinder

6. Coprime numbers Two natural numbers x and y are The set of rational numbers is represented by ‘Q’.
said to be coprime, if they do not have any common 3 −3
divisor other than 1. e.g. , , 7, − 6 are rational numbers.
5 7
e.g. (9, 2), (5, 6), (11, 15) are the pairs of coprime • The decimal expansion of every rational number is
numbers. either terminating or non-terminating repeating.
• If x and y are any two coprimes, a number p is 1 1 8
divisible by x as well as by y, then the number is also e.g. = 0 . 2, = 0 . 333. . ., = 0 . 181818 . . . etc.
5 3 44
divisible by xy.
• Coprime are also called as relatively prime numbers. • The recurring decimal have been given a short notation

NUMBER SYSTEM • Twin primes Twin primes are pair of primes


which differ by 2. e.g. (3, 5), (7, 9), (11, 13) etc.
7. Composite numbers A composite number is any
number greater than one that is not a prime number.
as 0. 3333 = 0. 3, 0 . 181818 = 0. 18

Note • Zero is a rational number, since we can write 0 = 0 / 1.


• Every natural number, whole number and integer is a
rational number.
e.g. 4, 6, 8, 9, … all are composite numbers.
• ‘1’ is neither prime nor composite.
IRRATIONAL NUMBERS
The number which cannot be expressed in the form p/q,
Generally (10-12) questions have been asked from this chapter. Questions, from this section usually INTEGERS where p and q both are integers and q ≠ 0 are known as
test your basic knowledge of numbers and are mostly based on various properties of multiplication The collection of positive numbers, negative numbers irrational numbers. The irrational numbers when
and division. A good number of statement based questions have been asked from this chapter. and zero are called integers. The set of integers is expressed in decimal form are in non-terminating and
denoted by Z or I. non-repeating form. e.g. 2, 5 , 7 , 0.101005001, etc.
Thus, Z or I = {. . . , − 4, − 3, − 2, − 1, 0, 1, 2, 3, 4, . . . } is the 22
• Here, it is notable that exact value of π is not or
set of integers. Every natural number and whole number 7
is a part of integer. So, N ⊂ W ⊂ I. 22
3.14, as is a rational number while π is irrational.
7
NUMBER SYSTEM Types of Integers
Numbers are collection of certain symbols or figures called digits. The common number system in use Integers are of three types IMPORTANT FACTS
is decimal number system. In this system, we use ten symbols each representing a digit. These are 0, 1,
(i) Positive Integers It is a set of all positive numbers. 1. If a + b = x + y , where a and x are rational and
2, 3, 4, 5, 6, 7, 8, and 9. A combination of these figures representing a number is called a numeral.
It is denoted by I + , I + = {1, 2, 3, 4, . . . } b and y are irrational, then a = x and b = y .

Types of Numbers (ii) Negative Integers It is a set of all negative numbers. 2. The sum or difference of a rational and an irrational
number is irrational number.
1. Natural numbers Numbers which are used for counting i.e. 1, 2, 3, 4, ... are called natural numbers. It is denoted by I − , I − = {. . . , − 3, − 2, − 1}
3. The product of rational and irrational number is also
The set of natural numbers is denoted by ‘ N ’. Smallest natural number is 1 but we cannot find the
(iii) Non-negative Integers An integer that is either 0 or an irrational number.
largest natural number as successor of every natural number is again a natural number.
positive is called non-negative integer. {0, 1, 2, 3, . . . } 4. If we add, subtract, multiply or divide two irrational
2. Whole numbers Natural numbers including zero are known as whole numbers. The set of whole numbers, we may get an irrational number or rational
numbers is denoted by W. Note ‘0’ is neither positive nor negative. number.
• Every natural number is a whole number.
EXAMPLE 1. The smallest 3 digit prime number is EXAMPLE 2. The rational number lying between 2
• Zero (0) is the only whole number which is not a natural number.
a. 101 b. 103 c. 109 d. 113 and 3 is
3. Even numbers The numbers which are divisible by 2 are called as even numbers. e.g 2, 4, 6, 8, 10, Sol. a. The smallest 3 digit number is 100, which is divisible 49 56
… . In general these are represented by 2 m, where m ∈ N. a. b.
by 2. 28 35
4. Odd numbers The number which are not divisible by 2 are called as odd numbers. e.g. 1, 3, 5, 7, ∴ 100 is not a prime numbers. 63 85
c. d.
9, … . In general, these are represented by ( 2 m − 1) , where m ∈ N. 101 < 11 and 101is not divisible by 2, 3, 5 and 7. 45 68
5. Prime numbers Those numbers which are divisible by 1 and the number itself are known as prime ∴ 101is a prime number. Sol. b. We have, 2 = 1. 414 ... and 3 = 1. 732...
numbers. e.g. 2, 3, 5, 7,..., etc. are prime numbers. Hence, 101is the smallest 3 digit prime number. 49 7 63 7
• If a number is not divisible by any of the prime numbers upto square root of that number, then it is a prime
= = 1. 75, = = 1. 4
28 4 45 5
number. RATIONAL NUMBERS 56 8
= = 1. 6,
85 5
= = 1. 25
• 2 is the only even number which is prime. 35 5 68 4
The numbers which are expressed in the form of p / q
• The prime numbers upto 100 are : 2, 3, 5, 7, 11, 13, 17, 19, 23, 29, 31, 37, 41, 43, 47, 53, 59, 61, 67, 71, Clearly, 1. 6 lies between 2 and 3.
where p and q are integers and are coprimes, q ≠ 0 are
73, 79, 83, 89 and 97 i.e. there are 25 prime numbers upto 100. 56
called rational numbers. Hence, lies between 2 and 3.
35
MATHEMATICS Number System 5 6 CDS Pathfinder

2. | x − 2 | = x − 2, if x > 2, | x − 2 | = 0 , if x = 2 To find the Unit’s Place Digit of a given EXAMPLE 7. When a positive integer n is divided by
REAL NUMBERS | x − 2 | = 2 − x, if x < 2 5, the remainder is 2. What is the remainder when the
The collection of all rational and all irrational numbers Expression number 3n is divided by 5?
together forms the set of real numbers and is denoted by
Some Properties of Absolute Values When Number is in the form of Product a. 1 b. 2 c. 3 d. 4
‘R’. Thus, all natural numbers, whole numbers, integers, To find the unit digit in the product of two or more
1. | x | ≥ 0 for all real x. Sol. a. Let n = 5q + 2 and 3n = 3( 5q + 2)
rational and irrational numbers are real numbers. number we take unit digit of every number and then
⇒ 3n = 15q + 6 = 15q + ( 5 + 1) = 5( 3q + 1) + 1
2. | x | = a means x = a or x = − a multiply them. Then, the unit digit of the resultant when 3n is divided by 5, then remainder is 1.
Properties of Real Numbers 3. | x | > a means x > a or x < − a product is the unit digit of the product of original
Properties of real numbers are as follows numbers. Divisibility Test
4. If n = x 2 ; n = x 2 = | x | = x, if x > 0 = − x , if x < 0
General Properties of R When Number is in the form of Index Divisor Condition with Example
1. If x and y are two real numbers, then either EXAMPLE 3. Find the value of x which satisfy the Let the exponential number of the form be a n and n ∈ I. 2 If the unit place of a number is ‘0’ or divisible by 2 i.e. unit
x > y, y > x or x = y. inequalities | x | ≥ x and 2x − 1 > 3. 1. In case, if a is any of (0, 1, 5, 6), then the unit’s place
place is even. e.g. 17980, 314782, 6148, 316 etc.

2. If x and y are two real numbers, then 3 If the sum of the digits of the given number is divisible by
a. All positive number digit is 0, 1, 5 and 6, respectively. 3. e.g. 24375, here 2 + 4 + 3 + 7 + 5 = 21 ÷ 3 = 7
1 1 b. All positive number greater than 2
(i ) x > y ⇒ < (ii ) x > y ⇒ − x < − y 2. In case, if a is any of (4 and 9) Hence, 24375 is divisible by 3.
x y c. All negative number less than −2
(i) and if power is odd, then the unit’s place digit is 4 4 If last two digits of number is divisible by 4.
(iii ) x > y ⇒ x + a > y + a d. All negative number
and 9, respectively. e.g. 589372, 72 is divisible by 4.
(iv ) x > y ⇒ xa > ya, when a > 0 Sol. b. | x | ≥ x is true for all real values of x. Now, consider (ii) and if power is even, then the unit’s place digit is 6 Hence, 589372 is also divisible by 4.
3. If xy = 0 ⇒ x=0 or y=0 2x − 1 > 3 or 2x > 4 or x > 2. and 1, respectively. 5 If digit at unit place is 5 or 0.

Properties of operations on R So, the solution set is all positive number greater than 2. 3. In case, if a is any of (2, 3, 7, 8), then see the e.g. 895, 700 etc.
following steps 6 If given number is divisibly by both 2 and 3.
Let ‘∗’ be any operation defined on R.
1. Closure Property If a ∈ R and b ∈ R, then a * b ∈ R.
FACTORS IN SET Step I First, divide the exponent of a by 4. e.g. 759312. Here the last digit is divisible by 2. And
(7 + 5 + 9 + 3 + 1 + 2 ) = 27, is divisible by 3.
Step II If any remainder comes on division. Put it as the
2. Associative Property If a, b, c ∈ R, then OF INTEGERS power of a and get the result.
Hence, 759312 is divisible by 6.
a *( b * c ) = ( a * b)* c. Let a, b ∈ I, we say that a is a factor of b, if there exists Step III If any remainder does not come on division. Put
7 If twice the number at units place is subtracted from rest of
the digits and the remainder is divisible by 7.
3. Commutative Property If a, b ∈ R, then a * b = b * a. an integer p such that b = ap; in short we write a / b read 4 as the power of a and get the result. e.g. (a) 875 = 87 − 2( 5) = 87 − 10 = 77 ÷ 7 = 11
4. Identity Let ‘I’ be the identity, then it ‘a’ divides ‘b’.
EXAMPLE 5. Find the unit digit of 207 × 781 × 39 × 94. Hence, 875 is divisible by 7.
I * a = a * I = a ∀ a ∈ R. If a is factor of b, then b is called a multiple of a. (b) 5103 = 510 − 2( 3) = 510 − 6 = 504 ÷ 7 = 72
a. 4 b. 2 c. 1 d. 5
5. Inverse Let a ′ be the inverse of a, then Hence, 5103 is divisible by 7.
a ′ * a = a * a ′ = I ∀ a ∈ R. Properties of factor and multiples Sol. b. Taking unit digit of every number and then Note Trick is applicable for number greater than 99.
multiplying them = 7 × 1× 9 × 4 = 7 × 36 8 If last three digits are divisible by 8.
• The above properties hold for addition and For real numbers a and b, if b = ac then ‘a’ is a factor
multiplication on R. Again taking unit digit and then multiplying = 7 × 6 = 42 e.g. (a) 96432 → 432 ÷ 8 = 54
‘b’ and we write a / b.
• The additive identity of R is O and additive inverse of ∴Required unit digit = 2 (b) 16000 → 000 ÷ 8 = 0
(i) a / b, b / c ⇒ a / c (This law is known as transitivity)
a ∈ R is −a. EXAMPLE 6. What is the last digit in 7 402 + 3 402 ?
9 If sum of all the digits are divisible by 9.
(ii) a / a, ∀a ∈ R (This law is known as reflexivity) e.g. 317349 ⇒ ( 3 + 1 + 7 + 3 + 4 + 9) = 27 ÷ 9 = 3
• The multiplicative identity of R is 1 and multiplicative a. 0 b. 4 c. 8 d. None of these Hence, 317349 is divisible by 9.
inverse of a ∈ R is 1 / a. (iii) a / b and a / c ⇒ a / b + c and a / b − c
Sol. c. On division of 402 by 4, we get 2 as remainder. 10 If last digit of a number is ‘0’. e.g. 130, 36980, etc.
• R is closed for subtraction, if a, b ∈ R, then a − b ∈ R. (iv) If p is prime number and p divides ab where a,b are 11 If the difference between sum of digits at even places and
• If a, b, c ∈ R, then a × ( b + c ) = ( a × b) + ( a × c ), is the ∴ Last digit of 7402 = Last digit of 72 = Last digit of 49 = 9
integers, then p divides ‘a’ or p divides ‘b’. Thus sum of digits at odd places is divisible by 11.
distributive property on R. Last digit of 3402 = Last digit of 32 e.g. 10615.
p / ab ⇒ p / a or p / b. = Last digit of 9 = 9 Sum of digits at odd place = 1 + 6 + 5 = 12
Absolute Value of a Real Number Sum of digits at even place = 0 + 1 = 1
EXAMPLE 4. How many factors of 2 5 × 3 6 are perfect ∴ Last digit of 7402 + 3402 = Last digit of (9 + 9) = 8
The absolute value of a real number x is denoted by | x |. Difference = 12 − 1 = 11 ÷ 11 = 1
Thus, | 3 | = 3 and | − 4 | = 4. squares? So, 10615 is divisible by 11.
 x, when x > 0 a. 9 b. 12 DIVISION ON NUMBERS
 EXAMPLE 8. What is the remainder when 41000 is
If x is any real number, then | x | =  0, when x = 0
c. 18 d. 4 (DIVISION ALGORITHM) divided by 7? e 2014 I
 − x, when x < 0 Sol. b. Any factor of this number should be of the form Let ‘a’ and ‘b’ be two integers such that b ≠ 0. On
 2a × 3 b .
a. 1 b. 2 c. 4 d. 5
dividing ‘a’ by ‘b’, ‘q’ will be the quotient and ‘r’ will be
e.g. For the factor to be a perfect square a, b have to be even Sol. c. On division of 41, 42 , 43 , 44 , 45 , 46 , and 47 by 7,
the remainder, then the relationship between a, b, q and r
1. If x = 5, | 5 | = 5, if x = 0, | 0 | = 0 a can take values 0, 2, 4 and b can take values 0, 2, 4, 6. is a = bq + r , where 0 ≤ r < b. Or in general, we have we get remainders 4, 2, 1, 4, 2, 1and 4 respectively.
If x = − 5, | − 5 | = − ( −5) = 5 ∴Total number of perfect squares = 3 × 4 = 12 Now, 44 gives us same remainder as 41, so the cyclicity is
Dividend = Divisor × Quotient + Remainder of 3.
MATHEMATICS Number System 7 8 CDS Pathfinder

So, any power of 3 or a multiple of 3 will give a 2. Multiplication of a Given The operations have to be carried out in the order, in
remainder of 1.
Number by a Power of 5
SIMPLIFICATION •
which they appear in the word VBODMAS.
So, 4999 will give a remainder of 1. The word simplification refers to a procedure of • The order of brackets to be simplied is (), {}, [ ].
Method Put as many zeros to the right of the converting a complex arithmetical expression into a
41000
∴ Remainder of =4 multiplicand as is the power of 5 in the multiplier.
7 simple expression. EXAMPLE 11. Simply the following expression 52 − 4
Divide the number so formed by 2 raised to the same of (17 − 12) + 4 × 7.
For simplifying an expression, follow the rule
Theorem of Divisibility power as is the power of 5.
‘VBODMAS’. The letter of word means a. 38 b. 85 c. 60 d. 72
1. If N is a composite number of the form e.g. Multiply 6798 by 125
V — Under line portion i.e. bar Sol. c. 52 − 4 of (17 − 12) + 4 × 7
N = a p ⋅ b q ⋅ c r . . . , where a, b and c are primes, then 6798000
Here, 6798 × 125 = 6798 × 5 3 = B — Brackets = 52 − 4 of 5 + 4 × 7 (simplifying parenthesis)
the number of divisors of N, represented by mis 23 O — Orders or powers = 52 − 4 × 5 + 4 × 7 (simplifying of)
given by m = ( p + 1) ( q + 1) (r + 1) . . .
6798000 D — Division = 52 − 20 + 4 × 7 (simplifying multiplication)
2. The sum of the divisors of N, represented by S is = = 849750
8 M — Multiplication = 52 − 20 + 28 (simplifying multiplication)
given by
A — Addition = 32 + 28 (simplifying subtraction)
( a p + 1 − 1) ( b q + 1 − 1) ( c r + 1 − 1) Important Identities
S= ⋅ ⋅ S — Subtraction = 60 (simplifying addition)
( a − 1) ( b − 1) ( c − 1) The identities given below are very useful for quick
multiplication.
SOME IMPORTANT RESULTS ON DIVISION
Here, if a, b and c are real numbers, then
1. If p divides q and r, then p also divides their sum and
difference also.
3
2. For any natural number n , ( n − n ) is divisible by 6.
1. (i) ( a + b)2 = ( a + b)(a + b) = a 2 + b2 + 2ab
(ii) ( a + b)3 = a 3 + b3 + 3ab ( a + b)
PRACTICE EXERCISE
3. The product of three consecutive natural numbers is 2. (i) ( a − b)2 = ( a − b)( a − b) = a 2 + b2 − 2ab
always divisible by 6. 1. Let ‘a’ and ‘b’ be natural number, not necessarily 8. In a division operation, the divisor is 5 times the
4. ( x m m
− a ) is divisible by ( x + a ) for even values of m.
(ii) ( a − b)3 = a 3 − b3 − 3ab ( a − b) distinct. For all values of ‘a’ and ‘b’ the natural quotient and twice the remainder. If the
number would be remainder is 15, then what is the dividend?
5. ( x m + a m ) is divisible by ( x + a ) for odd values of m. 3. (i) ( a + b) = ( a − b)2 + 4ab (a) (a + b ) (b) a / b (c) a − b (d) log (ab ) (a) 175 (b) 185 (c) 195 (d) 250
6. ( x m − a m ) is divisible by ( x − a ) for all values of m. 9. If 42 ∗ 8 is a multiple of 9, then the digit
(ii) ( a − b) = ( a + b)2 − 4ab 2. x and y are two natural numbers such that x is
less than y,q is the quotient and r is the represented by ∗ is
EXAMPLE 9. 19 5 + 21 5 is divisible by 4. (i) ( a + b)2 + ( a − b)2 = 2 ( a 2 + b2 ) remainder when y is divided by x. Therefore, (a) 0 (b) 1 (c) 2 (d) 4
a. Only 10 b. Only 20 2
(ii) ( a + b) − ( a − b) = 4ab2 (a) r = 0 (b) r < 0 (c) r > x (d) 0 ≤ r < x 16 16 ∗ 9 9 9
c. Both 10 and 20 d. Neither 10 nor 20 10. The value at ∗ in × − × + × = 1 is
2 2 3. If p is a prime number and p divides ab i.e. p/ ab, 7 7 7 7 7 7
 a + b  a − b
Sol. c. We can check divisibility of 195 + 215 by 10 by 5. ab =   −  where ‘a’ and ‘b’ are integers, then (a) 33 (b) −33 (c) −11 (d) 32
adding the unit digits of 95 and 15 which is equal to  2   2 
(a) p/ a or p/ b (b) p/ a and p/ b 11. The value of 10 ÷ 4 + 6 × 4 is
9 + 1 = 10. So, it must be divisible by 10. 2 2
6. a − b = ( a − b)( a + b) (c) p/ a − b (d) None of these
(a) 4 (b) 1/4 (c) 5 (d) None of these
Now, for divisibility by 20 we add 19 and 21 which is 4. Which one of the following is a prime number? a b c a+ b+ c
equal to 40. EXAMPLE 10. The value of (a) 161 (b) 171 12. If = = , then the value of is
4 5 6 b
So, it is clear that it is also divisible by 20. 1.073 × 1.073 − 0.927 × 0.927 (3 4 ) 4 × 9 6 (c) 173 (d) 221
+ is (a) 3 (b) 2 (c) 6 (d) 4
So, 195 + 215 is divisible by both 10 and 20. 1.073 − 0.927 (27)7 × 3 9 5. If n is a natural number, then n is
(a) always a whole number
13. The value of [a − b + ( b − a )] − [2a − 2b + ( b − 2a )] is
1 1
Shortcut Methods for Multiplication a. 2
3
b. 2
5 (b) always a natural number (a) b − 2 a (b) a − 2 b (c) b + 2 a (d) b
1. Multiplication of a Given c. 2
1
d. 3
(c) sometimes a natural number and sometimes an 14. If 3.325 × 10k = 0.0003325, the value of ‘k’ is
irrational number
Number by 9, 99 etc. 9 (d) always an irrational number (a) 4 (b) −4 (c) −3 (d) −2
Method Place as many zeros at the right of the .
1073 × 1073
. − 0.927 × 0.927 ( 34 ) 4 + (9)6 6. The product of a rational number and an 15. If [0.04 × 0.4 × x ] = 0.4 × 0.04 × y, then the
Sol. c. +
multiplicand as is the number of nines in the multiplier .
1073 − 0.927 ( 27)7 × ( 3) 9 irrational number is value of x / y is
and subtract from the number, so formed from the (a) natural number (b) an irrational number (a) 0.0016 (b) 0.16 (c) 0.016 (d) 0.160
. ) 2 − (0.927) 2 ( 34 ) 4 × ( 32)6
(1073
multiplicand, to get the result. e.g. = + 37 (c) a composite number (d) a rational number
1073
. − 0.927 ( 3 ) × ( 3) 9 16. The unit digit in the product (127)170 is
(i) Multiply 8886 by 9999
(1073
. + 0.927) (1073
. − 0.927) 328
7. What is the value of x for which x, x + 1, x + 3 are (a) 3 (b) 9 (c) 7 (d) 3
So, 8886 × 9999 = 88860000 − 8886 = 88851114 = + 30 all prime numbers?
1073
. − 0.927 3 (a) 0 (b) 1 17. The unit digit in the product ( 771 × 659 × 365 ) is
(ii) Multiply 56985 by 999 1 1 1 (c) 2 (d) 101 (a) 6 (b) 2 (c) 4 (d) 1
= 2+ 2 = 2+ = 2
So, 56985 × 999 = 56985000 − 56985 = 56928015 3 9 9
MATHEMATICS Number System 9 10 CDS Pathfinder

18. When n is divided by 4, the remainder is 3. 30. If k is any even positive integer, then ( k2 + 2k) is 42. If p is an integer, then every square integer is of 54. What will be the remainder when 19100 is
What is the remainder when 2n is divided by (a) divisible by 24 divided by 20?
the form
4? (b) divisible by 8 but may not be divisible by 24 (a) 2 p or (4 p − 1) (b) 4p or (4 p − 1) (a) 19 (b) 20 (c) 3 (d) 1
(a) 0 (b) 2 (c) 6 (d) 3 (c) divisible by 4 but may not be divisible by 8 (c) 3p or (3 p + 1) (d) 4p or (4 p + 1) 55. 712 − 412 is exactly divisible by which of the
1 1 1 1 1 1 (d) divisible by 2 but may not be divisible by 4
− − × − 43. If r and s are any real numbers such that 0 ≤ s ≤ 1 following number?
19. If 4 6 48 ÷ 4 6 48 = x, the value of 31. The number 2784936 is divisible by which one of and r + s = 1, then what is the maximum value of
1 1 1  1 1 1 (a) 34 (b) 33 (c) 36 (d) 35
− −  × −  the following numbers?
4  6 48 4  6 48 (a) 86 (b) 87 (c) 88 (d) 89
the product?
3 1 1
56. If 2 p + 3q = 17 and 2 p + 2 − 3q + 1 = 5, then find the
x is (a) 1 (b) (c) (d) value of p and q.
32. Which among the following is the largest four-digit 4 2 4
20 21 21 20 (a) −2, 3 (b) 2, − 3
(a) (b) − (c) (d) − number that is divisible by 88?
(c) 3, 2 (d) 2, 3
21 20 20 21 44. A number when divided by 2, 3 or 5 gives
49 1
(a) 9988 (b) 9966 (c) 9944 (d) 8888 remainder 1. The number is 57. If = 3+ , where x , y and z are natural
20. If x is negative real number, then 15 1
1 1 33. The pair of rational numbers that lies between
1 (a) 31 (b) 47 (c) 49 (d) 53 x+
(a)| x| = x (b)| x| = − x (c)| x| = (d)| x| = − 1
x x 4 45. The largest integer that divides product of any y+
3 z
x 3 x− y and is four consecutive integers is
21. If = , then the value of is 4 (a) 4 (b) 6 (c) 12 (d) 24
numbers, then what is z equal to?
y 5 x+ y 262 752 24 74 (a) 1 (b) 2 (c) 3
1 1 1 (a) , (b) , 46. Which one of the following three digit numbers
(a) − (b) (c) (d) −6 1000 1000 100 100 (d) Cannot be determined due to insufficient data
4 4 6 9 31 252 748 divides 9238 and 7091 with the same remainder
(c) , (d) ,
in each case? 58. The least number which is a perfect square and
7− x 40 40 1000 1000
22. If x is real, then < 2, if and only if (a) 113 (b) 209 (c) 317 (d) 191
has 540 as a factor is
3 34. By adding x to 1254934, the resulting number (a) 8100 (b) 6400 (c) 4900 (d) 3600
(a) 1 < x < 13 (b) −1 < x < 13 (c) x < 13 (d) x > 13 becomes divisible by 11, while adding y to 1254934 47. For a positive integer n, define d( n ) = The
59. If A is real and 1 + A + A2 + A3 = 40, then A is
23. A number is divisible by 25 only, if makes the resulting number divisible by 3. Which number of positive divisors of n. What is the equal to
one of the following is the set of values for x and y? value of d( d( d(12)))?
(a) the last digit of the number is zero (a) −3 (b) −1 (c) 1 (d) 3
(b) the last digit of the number is 5 (a) x = 1, y = 1 (b) x = 1, y = − 1 (a) 1 (b) 2 (c) 4 (d) None of these
(c) x = − 1, y = 1 (d) x = − 1, y = − 1 60. How many factors of 1080 are perfect squares?
(c) the last two digit of the number is divisible by 15 48. If three sides of a right angled triangle are (a) 4 (b) 6 (c) 8 (d) 5
(d) the last two digit of the number is divisible by 25 35. What is the last digit in the expansion of 34798 ? integers in their lowest form, then one of its
2.48 × 2.48 − 1.52 × 1.52 (a) 1 (b) 3 (c) 7 (d) 9 sides is always divisible by 61. Consider the following statements:
24. The value of is (a) 6 (b) 5 (c) 7 (d) None of these I. In a given whole number, if the sum of the odd
0.96 36. What least value must be given to ⊗, so that the numbered digit is equal to the sum of even
(a) 4 (b) 0.96 (c) 16 (d) 15.04 number 84705 ⊗ 2 is divisible by 9? 49. If −1 ≤ x ≤ 3 and 1 ≤ y ≤ 3, then the maximum numbered digits, then the number is divisible
25. Which one of the following is correct regarding (a) 0 (b) 1 (c) 2 (d) 3 value of ( 3 y − 4x ) is by 11.
the number 222222? 37. What is the total number of three digit numbers (a) 18 (b) 13 (c) 5 (d) −6 II. In a given whole number, if the difference of
(a) It is divisible by 3 but not divisible by 7 with unit digit 7 and divisible by 11? sum of odd numbered digits and even numbered
50. If ‘ p’ is an integer greater than 3, then on dividing
(b) It is divisible by 3 and 7 but not divisible by 11 digits is divisible by 11, then the number is
(a) 6 (b) 7 (c) 8 (d) 9 p11 + 1 by p − 1, we would get the remainder as
(c) It is divisible by 2 and 7 but not divisible by 11 divisible by 11.
7 3 (c) −2 (d) −1
(d) It is divisible by 3, 7 and 11 38. Find the value of ‘a’ and ‘b’ 3 ×b = 8. (a) 2 (b) 0
Which of the statement(s) given above is/are
a 15
26. What is the sum of positive integers less than (a) 2, 11 (b) 11, 2 (c) 1, 1 (d) 2, 1
51. If A is the set of squares of natural numbers and correct?
100 which leave a remainder 1 when divided x and y are any two element of A, then the (a) Only I (b) Both I and II
by 3 and leave a remainder 2 when divided by 39. If a , b and c are real numbers such that correct statement is (c) Only II (d) None of these
4? a < b and c < 0, then which of the statements is (a) x + y belongs to A (b) x − y belongs to A
x 62. Consider the following statements:
(a) 416 (b) 620 (c) 1250 (d) 1314 true? (c) belongs to A (d) x y belongs to A
(a) (a/c ) < (b/c ) (b) ac < bc y A number a1a2a3a4 a5 is divisible by 9, if
27. What is the last digit in the expansion of I. a1 + a2 + a3 + a4 + a5 is divisible by 9.
(c) (c/a) > (c/b ) (d) ac > bc
( 2457)754 ? 52. A ten-digit number is divisible by 4 as well as by
II. a1 − a2 + a3 − a4 + a5 is divisible by 9.
(a) 3 (b) 7 (c) 8 (d) 9 40. If we divide a positive integer by another positive 5. What could be the possible digit at the ten’s
integer, what is the resulting number? place in the given number? Which of the statement(s) given above is/are
28. A three-digit number is divisible by 11 and (a) 0, 1, 2, 4, or 6 (b) 1, 2, 4, 6 or 8 correct?
(a) It is always a natural number
has its digit in the unit’s place equal to 1. The (a) Only I (b) Only II
(b) It is always an integer (c) 2, 3, 4, 6 or 8 (d) 0, 2, 4, 6 or 8
number is 297 more than the number obtained (c) Both I and II (d) Neither I nor II
by reversing the digits. What is the number? (c) It is a rational number 53. If x < 0 < y, then which one of the following
(a) 121 (b) 231 (c) 561 (d) 451 (d) It is an irrational number 63. Consider the following statements:
relations is correct?
29. The remainder on dividing given integers a 41. What can be said about the expansion of 212n − 64 n , (a)
1
<
1
<
1
(b)
11
<>
1 If p is a prime such that p + 2 is also a prime,
and b by 7 are, respectively 5 and 4. What is where n is a positive integer? x2 xy y2 x2 xy y2 then
the remainder when ab is divided by 7? (a) Last digit is 4 (b) Last digit is 8 1 1
(c) <
1 1
(d) > I. p( p + 2) + 1 is a perfect square.
(a) 3 (b) 4 (c) 5 (d) 6 (c) Last digit is 2 (d) Last two digits are zero x y x y II. 12 is a divisor of p + ( p + 2), if p > 3
MATHEMATICS Number System 11 12 CDS Pathfinder

Which of the statement(s) given above is/are PREVIOUS YEARS QUESTIONS 79. If k is a positive integer, then every square 88. What is the remainder when (1235 × 4523 × 2451)
correct? integer is of the form e 2013 II
(a) Only I (b) Only II is divided by 12? e 2014 II
69. What number should be added to 231228 to (a) 4k (b) 4k or 4k + 3 (a) 1 (b) 3 (c) 5 (d) 7
(c) Both I and II (d) Neither I nor II make it exactly divisible by 33? e 2012 I (c) 4k + 1 or 4k + 3 (d) 4k or 4k + 1
64. Consider the following statements: (a) 1 (b) 2 (c) 3 (d) 4 89. What is the number of divisors of 360? e 2014 II
80. If b is the largest square divisor of c and a2 (a) 12 (b) 18
I. If x and y are composite integers, so also is x + y. 70. If a positive integer leaves remainder 28 when divides c, then which one of the following is (c) 24 (d) None of these
II. If x and y are composite integers and x > y, then divided by 143, then what is the remainder correct (where, a, b and c are integers)? e 2013 II
x − y is also a composite integer. obtained on dividing the same number by 13? (a) b divides a (b) a does not divide b 90. The multiplication of a three-digit number XY 5
III. If x and y are composite integers, so also in xy. e 2012 I
(c) a divides b (d) a and b are coprime with digit Z yields X 215. What is X + Y + Z
(a) 0 (b) 2 (c) 9 (d) 10 equal to? e 2014 II
Which of the statement(s) given above is/are 81. Every prime number of the form 3k + 1 can be (a) 13 (b) 15 (c) 17 (d) 18
correct? 71. Which one of the following is neither prime
represented in the form 6m + 1 (where, k and m
(a) I, II and III (b) I and II number nor composite number? e 2012 II 91. How many pairs of X and Y are possible in the
(a) 1 (b) 2 (c) 3 (d) None of these
are integers), when e 2013 II
(c) Only III (d) None of these number 763X 4Y 2, if the number is divisible
(a) k is odd
72. Which one of the following has least number of by 9? e 2014 II
65. Consider the following statements: (b) k is even (a) 8 (b) 9 (c) 10 (d) 11
I. The product of any three consecutive integers is divisors? e 2012 II (c) k can be both odd and even
divisible by 6. (a) 88 (b) 91 (c) 96 (d) 99
(d) No such form is possible 92. If an = 3 − 4n , then what is a1 + a2 + a3 +…+ an
II. Any integer can be expressed in one of the 73. How many numbers between −11 and 11 are 82. Consider the following statements
equal to? e 2014 II
three forms 3k, 3k + 1, 3k + 2, where k is an multiples of 2 or 3? e 2012 II (a) − n (4n − 3) (b) − n (2 n − 1) (c) −n2 (d) − n (2 n + 1)
integer. (a) 11 (b) 14 (c) 15 (d) None of these I. 7710312401 is divisible by 11.
II. 173 is a prime number. 93. Consider all those two-digit positive integers less
Which of the statement(s) given above is/are 74. Consider the following statements than 50, which when divided by 4 yield unity as
correct? Which of the statement(s) given above is/are
I. If n is a prime number greater than 5, then remainder. What is their sum? e 2014 II
correct? e 2013 II
(a) Only I (b) Only II n4 − 1 is divisible by 2400. (a) 310 (b) 314 (c) 218 (d) 323
(c) Both I and II (d) Neither I nor II (a) Only I (b) Only II
II. Every square number is of the form 5n or (5n − 1) 94. p, q and r are prime numbers such that
(c) Both I and II (d) Neither I nor II
66. Consider the following statements: or (5n + 1), where n is a whole number.
p < q < r < 13. In how many cases would
I. A natural number is divisible by 2, if its last Which of the statement(s) given above is/are 83. Consider the following statements ( p + q + r ) also be a prime number? e 2014 II
digit is divisible by 2. correct? e 2012 II I. To obtain prime numbers less than 121, we are (a) 1 (b) 2 (c) 3 (d) None of these
II. A natural number is divisible by 2, if its last (a) Only I (b) Only II to reject all the multiples of 2, 3, 5 and 7.
digit is either zero or 2. (c) Both I and II (d) Neither I nor II II. Every composite number less than 121 is 95. The digit in the units place of the product
III. A natural number is divisible by 2, if its last divisible by a prime number less than 11. 81 × 82 × 83 × 84 ×…× 99 is e 2015 I
digit is even. 75. If N , ( N + 2) and ( N + 4) are prime numbers, (a) 0 (b) 4 (c) 6 (d) 8
then the number of possible solutions for N are Which of the statement(s) given above is/are
Which of the statements given above are correct? correct? e 2013 II 96. What is the remainder obtained when
(a) 1 (b) 2 e 2013 I 1421 × 1423 × 1425 is divided by 12? e 2015 I
(a) I and II (b) I and III (c) II and III (d) All of these (c) 3 (d) None of these (a) Only I (b) Only II
(a) 1 (b) 2 (c) 3 (d) 4
67. Consider the following statements for natural (c) Both I and II (d) Neither I nor II
76. The two-digit number, which when divided by 97. What is the remainder when 496 is divided by 6?
numbers a, b and c : sum of the digits and product of the digits, 84. Consider the following statements
I. If ‘a’ is divisible by ‘b’ and ‘b’ is divisible by ‘c’, respectively leaves the same remainder the I. No integer of the form 4k + 3, where k an integer, e 2015 I
then a must be divisible by ‘c’. can be expressed as the sum of two squares. (a) 4 (b) 3
number is and the difference of quotients is one.
II. If ‘a’ is a factor of both ‘b’ and ‘c’, then ‘a’ must (c) 2 (d) 1
(a) 14 (b) 23 (c) 32 (d) 41 e 2013 I II. Square of an odd integer can expressed in the
be a factor of ‘b + c’. form 8k + 1, where k is an integer. 98. What is the maximum value of m, if the number
77. If x is positive even integer and y is negative
III. If ‘a’ is a factor of both ‘b’ and ‘c’, then ‘a’ must N = 35 × 45 × 55 × 60 × 124 × 75 is divisible by
factor of ‘b − c’. odd integer, then x y is e 2013 I Which of the statement(s) given above is/are
be a
(a) odd integer (b) even integer correct? e 2014 I 5m ? e 2015 I
Which of the statements given above are correct? (a) Only I (b) Only II (a) 4 (b) 5
(c) rational number (d) None of these (c) 6 (d) 7
(a) I, II and III (b) I and II (c) Both I and II (d) Neither I nor II
(c) II and III (d) None of these 78. Consider the following statements 2 2
99. If a − b = 4 and a + b = 40, where a and b are
I. There is a finite number of rational numbers 85. What is 262 + 972 equal to? e 2014 I
68. Consider the following statements: (a) 27 2 + 932 (b) 342 + 932 (c) 82 2 + 412 (d) 792 + 62 2 positive integers, then a3 + b6 is equal to e 2015 I
between any two rational numbers. (a) 264 (b) 280
I. Set of positive powers of 2 is closed under II. There is an infinite number of rational numbers
multiplication. 86. If n is a whole number greater than 1, then (c) 300 (d) 324
between any two rational numbers. n2( n2 − 1) is always divisible by
II. The set {1, 0, − 1} is closed under multiplication. III. There is a finite number of irrational number
e 2014 I 100. If n is a natural number and n = p1x1 p2x2 p3x3 ,
(a) 12 (b) 24 (c) 48 (d) 60
III. The number 35 has exactly four divisors. between any two rational numbers. where p1, p2 , p3 are distinct prime factors, then
IV. The set {1, 0, − 1} is closed under addition. Which of the statement(s) given above is/are
87. What is the remainder when (1723 + 2323 + 2923 ) the number of prime factors for n is e 2015 I
Which of the statements given above are correct? correct? e 2013 I
is divided by 23? e 2014 II (a) x1 + x2 + x3 (b) x1x2x3
(a) Only I (b) Only II (a) 0 (b) 1 (c) 2 (d) 3 (c) (x1 + 1)(x2 + 1)(x3 + 1) (d) None of these
(a) I, II and III are true (b) Only III
(c) Only IV (d) All of these (c) Only III (d) Both II and III
MATHEMATICS Number System 13 14 CDS Pathfinder

37 1 111. What is the maximum value of m, if the number 119. If a and b are negative real numbers and c is a 120. If m and n are distinct natural numbers, then
101. If = 2+ ,where x , y and z are natural
13 1 N = 90 × 42 × 324 × 55 is divisible by 3m ? e 2016 I positive real number, then which of the following which of the following is/are integer/integers?
x+
1 is/are correct? m n m n
y+ (a) 8 (b) 7 (c) 6 (d) 5 I. + II. mn  +  (m2 + n2 )−1
z I. a − b < a − c n m  n m
numbers, then what is z equal to? e 2015 I 112. 710 − 510 is divisible by e 2016 I II. If a < b, then
a b
< mn
(a) 5 (b) 7 (c) 10 (d) 11 c c III.
(a) 1 (b) 2 (c) 3 1 1 m2 + n2
3 3 III. <
(d) Cannot be determined due to insufficient data 113. If a = 117 + b and a = 3 + b, then the value of b c
Select the correct answer using the code given
102. A student was asked to multiply a number by 25. a + b is (given that a > 0 and b > 0) e 2016 I Select the correct answer using the code given
below e 2016 (I)
He instead multiplied the number by 52 and got (a) 7 (b) 9 (c) 11 (d) 13 below e 2016 I
(a) I and II (b) Only II
the answer 324 more than the correct answer. (a) Only I (b) Only II
114. Let m be a non-zero integer and n be a positive (c) Only III (d) II and III (c) II and III (d) Only III
The number to be multiplied was e 2015 I
(a) 12 (b) 15 (c) 25 (d) 32 integer. Let R be the remainder obtained on
dividing the polynomial x n + m n by ( x − m ).
103. The number of pairs ( x , y ), where x , y are integers
Then, e 2016 I
ANSWERS
satisfying the equation 21x + 48 y = 5, is e 2015 II (a) R is a non-zero even integer
(a) Zero (b) One (c) Two (d) Infinity 1 2 3 4 5 6 7 8 9 10
(b) R is odd, if m is odd
11 12 13 14 15 16 17 18 19 20
104. The largest natural number which divides every (c) R = s 2 for some integer s, if n is even
natural number of the form ( n3 − n ) ( n − 2), 3
21 22 23 24 25 26 27 28 29 30
(d) R = t for some integer t, if 3 divides n
where n is a natural number greater than 2, is 31 32 33 34 35 36 37 38 39 40
61 1
(a) 6 (b) 12 e 2015 II 115. If = 3+ where x , y and z are natural 41 42 43 44 45 46 47 48 49 50
(c) 24 (d) 48 19 1
x+ 51 52 53 54 55 56 57 58 59 60
105. The digit in the units place of the resulting 1
y+ 61 62 63 64 65 66 67 68 69 70
number of the expression ( 234)100 + ( 234)101, is x
numbers, then what z is equal to? e 2016 I
71 72 73 74 75 76 77 78 79 80
(a) 6 (b) 4 e 2015 II
(c) 2 (d) 0 (a) 1 (b) 2 (c) 3 (d) 4 81 82 83 84 85 86 87 88 89 90
91 92 93 94 95 96 97 98 99 100
106. A number when divided by 7 leaves a remainder 116. Let x and y be positive integers such that x is
3 and the resulting quotient, when divided by 11 101 102 103 104 105 106 107 108 109 110
prime and y is composite. Which of the following
leaves a remainder 6. If the same number when statements are correct? 111 112 113 114 115 116 117 118 119 120
divided by 11 leaves a remainder m and the I. ( y − x) can be an even integer.
resulting quotient when divided by 7 leaves a
II. xy can be an even integer.
remainder n. What are the values of m and n,
respectively? e 2015 II III. 0.5 (x + y) can be an even integer.
(a) 1 and 4 (b) 4 and 1 (c) 3 and 6 (d) 6 and 3
107. The seven digit number 876 p37q is divisible by
Select the correct answer using the code given
below e 2016 I
HINTS AND SOLUTIONS
(a) I and II (b) II and III 256 9x 81
225. The values of p and q can be respectively 1. (a) ( a + b ) always represent a natural 6. (b) We know that, the product of a ⇒ − + =1
(c) I and III (d) All of these
(a) 9, 0 (b) 0, 0 e 2015 II number ∀ a , b ∈ N . rational number and an irrational 49 49 49
(c) 0, 5 (d) 5,9 117. Consider the following statements: 2. (d) As, y = qx + r, so 0 ≤ r < x. number is an irrational number. ⇒ 256 + 81 − 9x = 49
108. Let x and y be positive integers such that x > y. I. Every natural number is a real number. 3. (a) As p is prime, so p/a or p/b. 7. (c) If x = 2, then, x, x + 1 and x + 3 are ⇒ 9x = 288 ⇒ x = 32
all prime numbers.
The expressions 3x + 2 y and 2x + 3 y, when II. Every real number is a rational number. 4. (c) Consider the given options ⇒ 9x = 288 ⇒ x = 32
divided by 5 leave remainders 2 and 3, III. Every integer is real number. 8. (c) Q Dividend = D × Q + R
(a) 161 is divisible by 7. Hence, 161 is 11. (a) By BODMAS, 10 ÷ 4 + 6 × 4
respectively. What is the remainder when ( x − y ), IV. Every rational number is a real number. not a prime number. Given, D = 5Q and D = 2R
is divided by 5? e 2015 II = 10 ÷ ( 4 + 6) × 4
Which of the above statements are correct? (b) Since, 1 + 7 + 1 = 9. Therefore, 171 When R = 15, D = 2 × 15 = 30
(a) 4 (b) 2 (c) 1 (d) 0 (a) I, II and III (b) I, III and IV e 2016 I is divisible by 3. Hence, 171 is not a D 30 = 10 ÷ 10 × 4 = 1 × 4 = 4
(c) II and III (d) III and IV prime number. ∴ Q= = =6 a b c
109. The sum of first 47 terms of the series 5 5 12. (a) = = = k (say)
(c) 173 < ( 14)2 4 5 6
1 1 1 1 1 1 1 1 1 118. Consider the following statements in respect of ∴ Dividend = 30 × 6 + 15 = 195
+ − − − + + + − − ... , is 173 is not divisible by any of the ⇒ a = 4k , b = 5k and c = 6k
two different non-zero integers p and q. 9. (d) As, 4 + 2 + * + 8 = 14 + * ; 42 * 8 is
4 5 6 4 5 6 4 5 6 e 2015 II numbers 2, 3, 5, 7, 11, 13. Hence,
I. For ( p + q) to be less than ( p − q), q must be divisible by 9, if 14 + * divisible by 9. a + b + c 4k + 5k + 6k 15k
1 1 9 173 is a prime number. So, = = =3
(a) 0 (b) − (c) (d) negative. b 5k 5k
6 6 20 (d) 221 is divisible by 13. Hence, 221 is So, 14 + ∗ = 18 (nearest multiple of 9)
II. For ( p + q) to be greater than ( p − q), both p and not a prime number. ⇒ ∗ = 18 − 14 = 4 13. (d) [ a − b + ( b − a )]
110. The value of the expression q must be positive. − [ 2a − 2b + ( b − 2a )]
5. (c) Consider, 2, 4 ∈ N 16 16 * 9 9 9
( 243 + 647)2 + ( 243 − 647)2 10. (d) Here, × − × + × =1
is equal to Which of the above statements is/are correct? So, 4 = 2, a natural number 7 7 7 7 7 7 ⇒ [ a − b + b − a ] − [ 2a − 2b + b − 2a ]
243 × 243 + 647 × 647 e 2016 I (a) Only I (b) Only II e 2016 I
and 2 = irrational number. Put * = x ⇒ 0 − [ 2a − 2a − b ] = 0 + b = b
(a) 0 (b) 1 (c) 2 (d) 3 (c) Both I and II (d) Neither I nor II
MATHEMATICS Number System 15 16 CDS Pathfinder

0.0003325 7 − x 49 1
= 22. (a) As, <2 30. (b) If k is any even positive integer, then 38. (b) Apply hit and trial method from the 48. (b) Given, by pythagoras theorem, =3+
k
14. (b) 10 (given) 57. (c)
3. 325  3  k 2 + 2k is divisible by 8 but may not be given option. As, here when (3 ) 2 + ( 4 ) 2 = ( 5 ) 2 15 x+
1
3.325 × 10−4 7−x 7 − x a = 11, b = 2, then 1
= = 10 −4
⇒ < 2 or −  <2 divisible by 24. Let the sides of a right triangle be 3, 4, y+
3.325 3  3  7
3 ×b
3
=3
7
×2
3
5.
z
Let k = 2m,m ∈ N , then 1 49 4
⇒ 10k = 10−4 [Q | x | < a ⇒ x < a or − x < a] a 15 11 15 ⇒ = −3 =
k + k ⋅ 2 = 4m + 4m = 4m (m + 1)
2 2 Hence, one of its sides is always divisible 1
x−7 40 33 x+ 15 15
∴ k = −4 ⇒ 7 − x < 6 or <2 = × =8 by 5. yz + 1
3 which is divisible by 8. 11 15
15. (c) 0.04 × 0.4 × x = 0.4 × 0.04 × y 49. (b)Q1 ≤ y ≤ 3 ⇒ 3 ≤ 3 y ≤ 9 …(i) z
⇒ − x < − 1 or x − 7 < 6 31. (c) Given number is 2784936. 39. (d) Since, a < b ⇒ a − b < 0. Also, c < 0
(given) and −1 ≤ x ≤ 3 ⇒ − 4 ≤ 4x ≤ 12 1 4
⇒ x > 1 or x < 13 Sum of digits at odd places= 25 and sum ∴ ( a − b ) c > 0 ⇒ ac − bc > 0 ⇒ ac > bc ⇒ =
x 0.4 × 0.04 of digits at even places = 14 ⇒ −12 ≤ −4x ≤ 4 …(ii) x+
z 15
∴ = ⇒ 1 < x < 13
y 0.04 × 0.4 40. (c) When we divide a positive integer by yz + 1
∴ Difference = 25 − 14 = 11 From Eqs. (i) and (ii), we get
23. (d) A number is divisible by 25 when its another positive integer, the resultant yz + 1 4
x 0.4 × 0.4 × 0.04 × 0.04 So, number is divisisble by 11. −9 ≤ 3 y − 4x ≤ 13 ⇒ =
= = 0.016 last 2 digits are either zero or divisible by will be a rational number i.e. in the form
y 0.04 × 0.4 Also last three digits of 2784936 i.e. 936 xyz + x + z 15
25. of p/q, where p and q are positive integers Maximum value is 13.
(squaring both sides) ( 2.48)2 − ( 1. 52)2 is divisible by 8. and q ≠ 0. 50. (a) When p11 + 1 is divided by ( p − 1), ⇒ 15( yz + 1) = 4 ( xyz + x + z )
24. (a) Hence, 2784936 is divisible by both 8
16. (b) Since, unit digit in 74 is 1. 0.96 41. (d) 212 n − 64 n = ( 212 )n − ( 64 ) n then the remainder is ⇒ 15 yz + 15 = 4xyz + 4x + 4z
( 2.48 − 1.52)( 2.48 + 152
. ) and 11 i.e. 88.
∴ 7 168
= (7 ) 4 42
give unit digit 1. = = ( 4096) − ( 1296)
n n
( 1)11
+ 1= 1+ 1= 2 ⇒ 15 − 4x = ( 4xy + 4 − 15 y )z
0.96 32. (c) A number divisible by 88, if it is
∴ 7 = 7 × 7 gives the unit digit
170 168 2 ∴ 2 12 n
−6 4n
= ( 4096 − 1296) k 51. (d) Let x = a and y = b for some
2 2 15 − 4x
[Q a − b = ( a − b )( a + b )]
2 2 divisible by 8 and 11. z=
as 1 × 7 × 7 = 9 As x − y is always divisible by ( x − m )
n m a , b ∈ N , then xy = a 2 b 2 = ( ab )2 , where ( 4xy + 4 − 15 y )
0.96 × 4 In the given options 9944 and 8888 are ab ∈ N .
17. (c) Unit place in 7 4 = 1, so unit place in = =4
0.96 divisible by 88. Hence, maximum = 2800 ( k ) As x, y and z are natural numbers.
768 is 1. number is 9944. So, xy ∈ A .
25. (d) Given, number is 222222. Hence, last two digits are always be zero.
x a2  a 
2 Again, let x = 3 and y = 1
∴ Unit place in 7 68 × 7 3 = 3. Similarly, 1 3 But =
a
=   ∉ A as ∉ N .
33. (d) Q = 0.25 and = 0.75 15 − 4(3)
unit place in 6 59 is 6 and unit place in
Here, sum of digits
4 4
42. (c) We know that y b2  b  b Then, z =
= 2 + 2 + 2 + 2 + 2 + 2 = 12, 4 = 3 p + 1, for p = 1, 9 = 3 p, for p = 3 4(3)( 1) + 4 − 15( 1)
34 is 1, so unit place in 365 = 364 × 3 = 3. Only option (d) lies between 0.25 and 52. (d) Since, a ten-digit number is divisible
which is divisible by 3. So, given number 16 = 3 p + 1, for p = 5, 15 − 12 3
∴ Unit place in 771 × 6 59 × 365 is the 0.75. Since
252
= 0. 252 by 4 as well as by 5, then this number = =
is divisible by 3.
1000 25 = 3 p + 1, for p = 8 must be divisible by 20. 12 + 4 − 15 1
unit place of 3 × 6 × 3 = 4
Now, sum of odd terms of digits − Sum ⇒ z = 3, which is a natural number.
of even terms of digits = 6 − 6 = 0, 748 36 = 3 p, for p = 12 We known that any number is divisible
18. (b) As, n is divided by 4 the remainder is and = 0.748
1000 Hence, every square integer is of form by 20, if last two digits is divisible by 20. 58. (a) Required number is
3, so it is divisible by 11.
either 3 p or 3 p + 1. It means unit place will be zero and ten’s
n = 4q + 3, where q is quotient. Also, in a number if a digit is repeated 34. (b) Difference of sums of even and odd x 2 = 540 × q = 3 × 3 × 3 × 2 × 2 × 5 × q
place may be 0, 2, 4, 6 or 8.
⇒ 2n = 8q + 6 six times, then the number is divisible by places digit of 1254934 43. (d) Given, r + s = 1 In order to make x 2 a perfect square, the
1 53. (c) As, x < 0 < y [given]
⇒ 2n = (8k + 4) + 2 = 4 ( 2k + 1) + 2
7, 11 and 13. = ( 1 + 5 + 9 + 4) − ( 2 + 4 + 3) For maximum product, r = s = least number we must have to put is
2 ⇒ x < 0 and y > 0 q = 3 × 5 = 15
So, if 2n is divided by 4 the quotient
Hence, the given number is divisible by = 19 − 9 = 10 1 1 1
3, 7 and 11. ∴ rs = × = ∴
1
< 0 and
1 1 1
>0 ∴ < ∴ x 2 = 540 × 15 = 8100
is 2k + 1 and remainder is 2. This number will be divisible by 11, 2 2 4
26. (a) Required numbers are of the form of x y x y
1 1 1 1 1 1 after adding x, if x = 1. 59. (d) We have, 1 + A + A 2 + A 3 = 40
− − × − 12q − 2 44. (a) Required number
Also, the sum of digits of 1254934 54. (d) On division of ( 19)n by 20, we get ⇒ ( 1 + A ) + A 2 ( 1 + A ) = 40
19. (c) Given, 4 6 48 ÷ 4 6 48 i.e. 10, 22, 34, 46, 58, 70, 82, 94 = [LCM of 2, 3 and 5] +1
1 1 1 1 1 1 = 1 + 2 + 5 + 4 + 9 + 3 + 4 = 28 remainder either 19 or 1.
− −  × −  = 30 + 1 = 31 ⇒ ( 1 + A )( 1 + A 2 ) = 40
4  6 48  4  6 48  ∴ Total sum = 10 + 22 + 34 + 46 + 58 Since, last digit of ( 19)100 is 1.
1254934 will be divisible by 3, after Only A = 3 satisfies the above equation,
+ 70 + 82 + 94 = 416 adding y , if y = − 1 45. (d) The largest integer that divides ( 19)100
=x ∴ Remainder of is 1. so, A = 3.
product of any four consecutive integers
12 − 8 − 1 1 1 27. (d) The last digit in the expansion of 35. (d) Last digit in the expansion of 34798 20
− 754 754
is 24. 60. (a) We have, 1080 = 23 × 33 × 5
⇒ x = 48 ÷ 24 48 ( 2457) is equal to last digit of ( 7) . = Last digit in the expansion of e.g. 1, 2, 3, 4 are four consecutive 55. (b) We know that ( xn − y n ) is divisible
12 − (8 − 1) 1  8 − 1 ( 7)754 = ( 74 )188 × 72 = 1 × 72 by ( x − y ) for all ‘n’ and is divisible by For any perfect square, all the powers of
×  (34 )1199 ⋅ 32
integers.
48 4  48  = ( 7)2 = 49 Multiplication = 1 × 2 × 3 × 4 = 24
( x + y ) for even ‘n’. the primes have to be even numbers.
3 2−1 = Last digit in the expansion of 3 = 9 2
∴( 7 − 4 ) is divisible by ( 7 + 4) = 11
12 12 So, if the factor is of the form
Hence, last digit is 9. which divided by 24.
36. (b) The given number is divisible by 9, if and ( 7 − 4) = 3 2a × 3b × 5c
⇒ x = 48 ÷ 48 28. (d) On taking option (d),
5 7 46. (a) When we divide the number 9238
sum of the digits is divisible by 9. Thus, ( 712 − 412 ) is divisible by 33. The values a can be 0 and 2, b can be 0
48 4 × 48 The reverse digit of 451 is 154. and 7091 by 113, we get the same
Now, 154 + 297 = 451 is equal to the
Here, sum of digits remainder 85. 56. (c) Here, 2 p + 3q = 17 ...(i) and 2 and c can take the value 0.
3 4 21
⇒ x = ÷ ⇒x = original number. = 8 + 4 + 7 + 0 + 5 + 2 + ⊗ = 26 + ⊗ 47. (d) d ( d ( d ( 12))) = d ( d ( 6)) 2p + 2
− 3q +1
=5 Totally, there are 4 possibilities.
5 7 20
29. (d) Let a = 7 p + 5 and b = 7q + 4 If ⊗ = 1 , then 26 + 1 = 27 is divisible by [Q positive integer divisor of or 4 ⋅ 2 − 3 ⋅ 3 = 5
p q
...(ii) 61. (b) Clearly, both statements satisfies
20. (b) Clearly, absolute value is defined by 9. 12 = 1, 2, 3, 4, 6, 12]
where, p and q are natural numbers. On multiplying Eq. (i), by 3 and adding divisibility rule of 11.
| x | = − x.
d ( d ( 6)) = d ( 4) it with Eq. (ii), we get
x− y x/ y − 1 3/5− 1 ∴ ab = ( 7 p + 5) ( 7q + 4) 37. (c) The total number of three-digit 62. (a) As, we know that a number
21. (a) = = numbers with unit digit 7 and divisible [Q positive integer divisor of 7 ⋅ 2 p = 56, 2 p = 8 = 23 , ⇒ p = 3 a a a a a is divisible by 9, if sum of
x + y x/y + 1 3/5 + 1 ab = 49 pq + ( 4 p + 5 q ) 7 + 20 6 = 1, 2, 3, 6] 1 2 3 4 5
by 11 are 187, 297, 407, 517, 627, 737,
= 7 ( 7 pq + 4 p + 5 q ) + 7 × 2 + 6 Put p = 3 in Eq. (i), we get the digits, i.e. a + a + a + a + a
(3 − 5 ) / 5 −2 1 847, 957. and d( 4) = 3 1 2 3 4 5
= = =− when ab is divided by 7, we get the [Q positive integer divisor of 23 + 3q = 17, 3q = 17 − 8 = 9 = 32 is divisible by 9. Hence, only statement I
(3 + 5 ) / 5 8 4 ∴ Total numbers = 8
remainder 6. 4 = 1, 2, 4] ∴ q=2 is true.
MATHEMATICS Number System 17 18 CDS Pathfinder

63. (c) Taking p = 11 72. (b)Here, 88 = 2 × 2 × 2 × 11 = ( 2)3 × ( 11)1 So, a 2 will divide bx or a will divide b. n2 ( n2 − 1) = ( 2)2 ( 22 − 1) = 4 × 3 = 12 10 103. (a) 21x + 48 y = 5 ⇒ 3( 7x + 16 y ) = 5
= [ 2 × 13 + ( 10 − 1)4]
p + 2 = 13 [a prime number] 91 = ( 7)1 × ( 13)1 [since, it cannot divide x as it is not a Illustration 2 Put n = 3, then 2 If x , y are integer, then LHS of the
whole square] n2 ( n2 − 1) = (3)2 (32 − 1) = 9 × 8 = 72 10 10 × 62
I. 11 × 13 + 1 = 144 [a square number] 96 = 2 × 2 × 2 × 2 × 2 × 3 = [ 26 + 36] = = 310 above equation is multiple of 3, but the
81. (b) Every prime number of the form 2 2 RHS of above equation is not multiple
II. 11 + 13 = 24 [12 is a divisor of 24] = ( 2 ) × (3 )
5 1 1723 + 2923 2323
3k + 1 can be represented in the form 87. (a) We have, + of 3.
23 23 94. (b) The prime numbers less than 13 are
Hence, both statements I and II are and 99 = 3 × 3 × 11 = (3)2 × ( 11)1 6m + 1 only, when k is even. ∴ There is no any integral values of x
2, 3, 5, 7, 11.
correct. So, 91 has least number of divisors. ( 1723 + 2923 ) is divisible by
82. (c) I. In 7710312401, difference between Also, p < q < r < 13 and y exist.
64. (c) I. If x = 15 and y = 14, then sum of even place digits and the sum of ( 17 + 29) i.e. 46
73. (c) Following are the numbers between and p + q + r is a prime number. 104. (c) Let x = ( n3 − n)( n − 2), where n > 2
x + y = 15 + 14 = 29, which is a − 11and 11 which are multiples of 2 or 3. odd place digits 0. So, it is divisible by So, it is divisible by 23.
prime number. So, if x and y are 11. 23
Also, 23 is always divisible by 23. Hence, only two possible pairs exist i.e. Take n = 3, we get
composite, then x + y is not always − 10, − 9, − 8, − 6, − 4, − 3, − 2, (3, 5, 11) and (5, 7, 11). x = (33 − 3)(3 − 2) = ( 27 − 3)( 1) = 24
II. Since, 173 < ( 14)2 and it is not ∴ Remainder = 0
composite. 0, 2, 3, 4, 6, 8, 9, 10 divisible by 2, 3, 5, 7, 11 and 13. So, 88. (b) Let E = ( 1235 × 4523 × 2451) 95. (a) Product of unit digits [which is divisible by 6, 12 and 24]
II. If x = 15 and y = 14, then So, the numbers of multiples 2 or 3, it is a prime number.
= ( 12 × 102 + 11)( 12 × 376 + 11) = 1× 2 × 3 × 4 × 5 × 6 × 7 × 8 × 9 Take n = 4, we get x = ( 43 − 4)( 4 − 2)
x − y = 15 − 14 = 1 which is neither between –11 and 11 are 15. Hence, both statements I and II are
prime nor composite, hence again correct. × ( 12 × 204 + 3 ) × 1 × 2 × 3 × ........ × 8 × 9 = ( 64 − 4) × 2 = 120
x − y is not always composite. 74. (b) I. Given, n is a prime number greater
83. (c) Both the statements given are When we divide E by 12, then =0 [which is again divisible by 6, 12 and 24]
than 5.
III. Third condition is satisfied for all
Now, n4 − 1 = ( n2 − 1)( n2 + 1) correct. As 121 is the square of 11. So, to Remainder = Remainder when ∴ Required digit in the unit place is 0. Now, take n = 5, we get
measure. obtain prime numbers less than 121, we 11 × 11 × 3 or 363 is divided by 12 = 3
Hence, only III is correct. = ( n − 1)( n + 1)( n2 + 1) 96. (c) See question 88. x = (53 − 5)(5 − 2) = ( 125 − 5) × 3
reject all the multiples of prime numbers 89. (c) Q 360 = 23 × 32 × 51 = 120 × 3 = 360
65. (c) I. The product of any three Put n = 11, less than 11 i.e. 2, 3, 5 and 7. Similarly, ∴ Number of divisors 97. (d) See example 8.
consecutive integers is divisible by 6. n4 − 1 = ( 11 − 1)( 11 + 1)( 121 + 1) every composite number less than 121 is = (3 + 1)( 2 + 1)( 1 + 1) 98. (c) N = 35 × 45 × 55 × 60 × 124 × 75
[which is again divisible by 6, 12 and 24]
[prime number greater than 5] divisible by a prime number less than 11 = 4 × 3 × 2 = 24
II. Here, 3k = {. . . − 6, − 3, 0, 3, 6, . . .} = 5 × 7 × 5 × 9 × 5 × 11 × 5 × 12 × 124
Hence, 24 is the largest natural number.
= 10 × 12 × 122 i.e. 2, 3, 5 or 7.
90. (a) Given, three-digit number = XY 5
3k + 1 = {. . . − 5, − 2, 1, 4, 7, . . .} 105. (d) We have, ( 234)100 + ( 234)101
= 14640 which is not divisible by 2400. 84. (a) I. f ( k ) = 4k + 3 × 52 × 3
and 3k + 2 = {. . . − 4, − 1, 2, 5, 8, . . .} XY 5 ⇒ ( 234)100 ( 1 + 234) = 235( 234)100
and =Z = 56 × 7 × 9 × 11 × 12 × 124 × 3
So, statement I is not true. For k = 1, f ( 1) = 4 × 1 + 3 = 7 X 215
∴ {3k , 3k + 1, 3k + 2} For k = 2, f ( 2) = 4 × 2 + 3 = 11 We know that square of any number
II. Every square number can be of the Here, Z can take values 1, 3, 5, 7 and 9. Hence, the maximum value of m is 6.
= {. . . − 6, − 5, − 4, − 3, − 2, − 1 having 4 at unit place is a number in
form 5n or (5n ± 1) or (5n ± 4). For k = 3, f (3) = 4 × 3 + 3 = 15 99. (b) Given a − b = 4 …(i)
But only 9 satisfies it, which 6 at unit place.
0, 1, 2, 3, 4, 5, 6 . . . } So, statement II is true. Values of f ( k ) for k = 1, 2 ,... cannot 135
On squaring both sides, we get Any exponent of a number 6 at unit
Hence, it is true. be expressed as sum of two squares, Q ×9
75. (a) When N is a natural number, then since 12 + 22 = 5, 12 + 32 = 10, 1215 ( a − b )2 = ( 4)2 ⇒ a 2 + b 2 − 2ab = 16 place is always 6 at unit place.
66. (d) All statements are true. there is only one possible case that N, (N 22 + 32 = 13. ( 235)(…6) = … 30
then X = 1, Y = 3 and Z = 9 ⇒ 40 − 2ab = 16 [Qa + b = 40]
2 2 Q
+ 2), (N + 4) are prime numbers.
67. (a) (i) If b / a and c / b, then a = bx and II. f ( k ) = 8k + 1 Now, X + Y + Z = 1 + 3 + 9 = 13 Resulting number have 0 at unit place.
When N = 3, then N, (N + 2), (N + 4) ⇒ 2ab = 24 ⇒ ab = 12
b = cy for x, y ∈ N For k = 1, f ( 1) = (8 × 1) + 1 = 9 91. (d) Given number is 763X 4Y 2.
a+ b = a 2 + b 2 + 2ab 106. (a) Let the number be y.
⇒ a = bx = cy ( x) = cxy ⇒ c / a = 3, 5, 7 all are primes numbers. For k = 2, f ( 2) = (8 × 2) + 1 = 17 Q
Since, given number is divisible by 9. ∴ y = 7q + 3 and q = 11p + 6
(ii) If a / b and a / c, then b = ax and 76. (c) From options, For k = 3, f (3) = (8 × 3) + 1 = 25 = 40 + 2 × 12 = 8
∴ 7 + 6 + 3 + X + 4 + Y + 2 = 9k ∴ y = 7( 11p + 6) + 3
c = ay for xy ∈ N , (c)
32
=
32
=2 (remainder) For k = 4, f ( 4) = (8 × 4) + 1 = 33 ∴ a+ b =8 … (ii)
⇒ 22 + X + Y = 9k ⇒ y = 77 p + 45
⇒ ( b + c ) = ax + ay = a ( x + y ) (3 + 2 ) 5 For k = 5, f (5) = (8 × 5) + 1 = 41
32 32 It is clear that LHS is divisible by 9, if On solving Eq. (i) and Eq. (ii), we get
So, a / ( b + c ) and = =2 (remainder) f ( k ) = 8k + 1 is square of an odd a = 6 and b = 2
When divided by 11 remainder is 1 and
X + Y = 5,14
(iii) From (ii), we have ( b − c ) = a( x − y ) (3 × 2 ) 6 integer only for some values of k. quotient is 7 p + 4 and when the
When sum of X and Y is 5, then Now, a 3 + b 6 = 63 + 26 = 23 × 33 + 26 quotient is divided by 7 remainder is 4.
⇒ a / ( b − c) ∴ Remainder are same. So, only statement I is correct.
possible pairs are (1, 4), (4, 1), (2, 3),
and difference of quotients = 6 − 5 = 1 85. (d) To check which option is equal to = 2 3 (3 3 + 2 3 ) 107. (c) Seven digits number 876 p37q is
68. (d) All are true. (3, 2), (0, 5), (5, 0). When sum of X and
26 2 + 972 , we take the sum of unit Y is 14, then possible pairs are = 8 ( 27 + 8) = 8 × 35 = 280 divisible by 225, if this number is
69. (c) Given, 33 ) 231228 ( 7006 77. (c) If x is a positive even integer and y is digit’s square of both number of the divisible by 9 and 5.
negative odd integer, then x y is a (5, 9),(9, 5),(6, 8),(8, 6) and (7, 7). Hence, the value of a + b is 280.
3 6
231 question as well as the answer options. If this number is divisible by 9.
rational number. Hence, the possible pairs are 11.
228 Whichever answer option shows the 100. (c) If factor of given number is of the Then, sum of its digits is divisible by 9.
198 78. (b) We know that, between any two same result will be the answer. 92. (b) Given, an = 3 − 4n
form of p α , p α ... p α n , then number Now, sum of digits
30 rational numbers, there are an infinite Here, in 262 + 972 , 62 = 36 ∴ Σan = Σ(3 − 4n) 1 1 2 2 n

number of rational and irrational numbers. [ n × ( n + 1)] of prime factors are =8+ 7+ 6+ p+3+ 7+ q
Now, 33 − 30 = 3 and 72 = 49 = 3n − 4
Hence, only statement II is correct. 2 (α + 1) (α + 1) … (α n + 1) = 31 + p + q
So, on adding 3 to 231228, it will be 36 + 49 = 85 1 2
So,
= 3 n − 2 n2 − 2 n Hence, the prime factor of n are ∴ p + q = 5 or p + q = 14
completely divisible by 33. 79. (d) If k is a positive integer, then every
square integer is of the form 4k or 4k + 1, For option (a), 72 + 32 = 49 + 9 = 58 ∴ Given number is divisible by 225
= n − 2n2 = − n( 2n − 1) ( x + 1)( x + 1) and ( x + 1)
70. (b) Given, N = 143k + 28 as every square number is either a For option (b), 42 + 32 = 16 + 9 = 25 1 2 3
when q = 5
⇒ N = 143k + 26 + 2 93. (a) Let the two-digit numbers less than, 101. (b) See question 57.
multiple of 4 or exceeds multiple of 4 by For option (c), 22 + 12 = 4 + 1 = 5 50 which when divided by 4 yield unity if q = 5, p = 0 or 9.
⇒ N = 13 ( 11k + 2) + 2 unity. 102. (a) Let x be the required number.
For option (d), 92 + 22 = 81 + 4 = 85 as remainder be 13, 17, … , 49. 108. (a) We have, when 3x + 2 y is divided by
∴When the number is divided by 13 the 80. (c) Since, b is largest square divisor of c. ∴ 52x − 25x = 324 ⇒ 27x = 324 5, remainder is 2.
Only option (d) satisfies the condition. Here, first term, a = 13, common 324
remainder is 2. So, c = bx ⇒ x= = 12
86. (a) If n is greater than 1, then n2 ( n2 − 1) difference, d = 4 and n = 10 ∴ 3x + 2 y = 5q + 2 …(i)
71. (a) 1 is neither prime number nor n 27
[where, x is not a whole square number] is always divisible by 12. ∴ Required sum = [ 2a + ( n − 1)d ] and when 2x + 3 y is divided by 5, the
composite number. Also, a 2 divides c. 2 Hence, the required number is 12.
Illustration 1 Put n = 2, then remainder is 3.
02
MATHEMATICS Number System 19 20 CDS Pathfinder

∴ 2x + 3 y = 5m + 3 …(ii) Also, a 3 − b 3 = 117 I. Q p + q< p − q


Subtract Eq. (ii) from Eq.(i), we get ⇒ ( a − b )( a 2 + b 2 + ab ) = 117 ⇒ q + q< 0
x − y = 5( q − m ) − 1 117 ⇒ 2q < 0
⇒ a 2 + b 2 + ab = = 39 …(ii)
x − y = 5( q − m ) − 5 + 4 3 ⇒ q< 0
x − y = 5( q − m − 1) + 4 On subtracting Eq. (i) from Eq. (ii) ∴ q must be negative.
∴ x − y is divided by 5 remainder is 4. 3ab = 30 ⇒ ab = 10 Hence, statement I is correct.
109. (b) The sum of first 47 terms of the Now, a + b = ( a − b ) + 4ab
2
II. Q p + q> p − q
series
= 9 + 40 = 49 = 7 ⇒ q + q> 0
1 1 1 1 1 1 1 ∴ 2q > 0

SEQUENCE
+ − − − + + 114. (a) If R is the remainder obtained by
4 5 6 4 5 6 4
dividing the polynomial xn + m n by ∴ q must be positive irrespective of p.
1 1
+ − … 47 term ( x − m ), then ( xn + m n − R) is divisible Hence, statement II is incorrect.
5 6 by ( x − m ).
119. (d) I. a − b < a − c
It is clear that sum of first 6 term is zero. Let f ( x) = xn + m n − R ⇒b − c> 0

AND SERIES
...(i)
Similarly, sum of first 42 terms is zero. Q f ( x) is divisible by ( x − m ).
As, b is negative real number and c is
Sum of last 5 terms ∴ f (m ) = 0 positive real number, then Eq. (i) is
1 1 1 1 1 −1 ⇒ (m ) n + (m ) n − R = 0 not true.
= + − − − =
4 5 6 4 5 6 R = 2(m )n II. If a < b, when a and b are negative
( 243 + 647)2 + ( 243 − 647)2 As, m is a non-zero integer and n is a real numbers and c is a positive
110. (c) a b
243 × 243 + 647 × 647 positive integer, then R is a non-zero real number, then < is always
even integer. c c Usually (1-2) questions have been asked from this chapter. Questions are mostly based on
2[( 243)2 + ( 647)2 ]
= =2 true for b > a.
( 243)2 + ( 647) 2 115. (c) See question 57. 1 1 relation between arithmatic geometric and harmonic mean.
III. < is always true, as c is a positive
[∴( a + b )2 + ( a − b )2 = 2 ( a 2 + b 2 )] 116. (d) x is prime and y is composite b c
111. (b) We have, N = 90 × 42 × 324 × 55 number. real number and b is a negative real
Since, x is prime number. number.
= 32 × 10 × 3 × 14 × 34 × 4 × 55
∴ x may be {2, 3, 5, 7, 11, …} and y may 120. (d) I. If m and n are distinct natural
= 37 × 10 × 14 × 4 × 55
Hence, N is divisible by 37 .
be {4, 6, 8, 9, 10, 12, 14, 15, …} numbers, then
m
+
n m
n
is integer if and SEQUENCE
Here, y may be even or odd number and
So, the maximum value of m is 7 when x is odd number except 2. only if m = n. Hence statement I is A set of numbers arranged in a definite order according to some definite rule is called a sequence.
N is divisible by 3m . incorrect. e.g. 2, 4, 6, 8, ... is a sequence.
∴ ( y − x) can be even number.
II. mn  +  (m 2 + n2 )−1
m n
112. (d) 710 − 510 = ( 75 )2 − (55 )2 Also, xy can be even number.  n m
= ( 75 + 55 ) ( 75 − 55 ) If y is odd integer and x ≥ 3.
Then, 0.5( x + y ) is an even integer. = mn 
 m 2 + n2  

1 
 =1
SERIES
= ( 16807 + 3125) ( 75 − 55 )  mn   m 2 + n2 
Hence, statements I, II and III are If a1 , a 2 , a 3 ,... a n is a sequence, then the expression a1 + a 2 + a 3 + ... + a n is called the series.
= 19932 × ( 75 − 55 ) Hence, statement II is correct for all
correct. The series is said to be finite or infinite depending upon the last term is given or not. e.g.
Since, 19932 is divisible by 11. values of m and n.
117. (b) A real number is collection of all mn
Hence, 710 − 510 is divisible by 11. III. Now, 2
m + n2
is a fraction. (i) a1 , a 2 , a 3 , . . . , a n is a finite sequence and is denoted as { a k } n .
rational and irrational numbers. k=1
113. (a) We have, a − b = 3
So, statement III is incorrect.
by { a n } ∞
So, statement II is false.
Squaring both sides, we get (ii) a1 , a 2 , a 3 , a 4 , . . is an infinite sequence and is denoted n=1 or simply { a n }.
118. (a) Given, p and q are non-zero integers.
a 2 + b 2 − 2ab = 9 …(i)
Here, a1 is called the first term and in case of finite sequence a n is called the last term.
e.g. 2, 4, 6, 8, … , 100 is a finite sequence.
1, 2, 3, 4, … is an infinite sequence.

PROGRESSION
Sequence following certain patterns are called progressions.
e.g. 2, 3, 4, 5, … is a progression, here each term is increasing by 1.

Arithmetical Progression (AP)


An arithmetic progression is a sequence in which the difference between any term and its preceding
term is constant throughout. The constant ‘d’ is called the common difference. The first term of an AP
is represented by ‘a’.
MATHEMATICS Sequence and Series 21 22 CDS Pathfinder

If an AP has first term = a and common difference = d, a(r n − 1)  a + c  4 p r  = apcr


2 2 2
Let a and b be two numbers and H be the HM between
Sum of the n terms of a GP is S n = , ⇒   [from Eqs. (i) and (ii)]
 2   ( p + r) 2 

then the general form of an AP is r −1 them.
a, a + d , a + 2d , a + 3d , . . . , a + ( n − 1) d a(1 − r n ) Then, a, b and c are in HP 2
( a + c) pr
if r > 1 and S n = , if r < 1. ⇒ = ac
• nth term of an AP is Tn = a + ( n − 1) d . 1−r ∴ H=
2ab ( p + r) 2
Sum of the first n terms of an AP is a a+b
• • Sum of infinite terms of a GP is S n = . ( a + c) 2 ( p + r) 2
n n 1−r ⇒ =
S n = [ 2a + ( n − 1) d ] or S n = [ a + l ] Relation between Arithmetic, Geometric ac pr
2 2
Geometric Mean (GM) and Harmonic Mean p2 r 2 a2 c2 p r a c
where, l = last term ⇒ + + 2= + +2 ⇒ + = +
If three terms are in GP, then the middle term is called Let A, G and H be the arithmetic, geometric and pr pr ac ac r p c a
EXAMPLE 1. Find the sum of 11 terms of the geometric mean of the other two. If a, b and c are in harmonic means between a and b, then
−7, − 2, 3, 8 K . Sum to n Terms of Special Series
GP, then b is the GM of a and c. (i) A ≥ G ≥ H (ii ) G 2 = AH
a. 200 b. 198 c. 326 d. 137 The sum of first n terms of some special series is given
Let a and b be two numbers and G be the GM between
below
Sol. d. Here, a = −7, l = −7 + (10 × 5) = 43 them. Then, a, G, b are in GP, G = ab, a > 0, b > 0. EXAMPLE 4. If a, b, c are in AP, p, q, r are in HP and
 a + l  −7 + 43 p r 1. The sum of first n natural numbers
∴ S 11 = n  = 11×   ap, bq, cr are in GP, then + is equal to
 2  2  EXAMPLE 3. If the 4th, 10th and 16th terms of a GP r p n ( n + 1)
= ∑ n = 1 + 2 + 3+…+ n =
36 are x, y and z respectively, then x, y , z are in a c a c 2
= 11× = 198 a. − b. +
2 a. AP b. GP c a c a 2. The sum of square of the first n natural numbers
c. AGP d. HP b q b q
d. + d. −
Arithmetic Mean (AM) Sol. b. Let the first term and common ratio be a and r, q b q b n ( n + 1)( 2n + 1)
= ∑ n 2 = 12 + 22 +…+ n 2 =
When three terms are in AP, then the middle term is respectively. a+ c 6
4−1
Sol. b. Since, a, b, c are in AP ⇒ b= ...(i)
called arithmetic mean of the other two. Given, T4 = x ⇒ ar 3
= x ⇒ ar = x …(i) 2 3. The sum of cubes of the first n natural numbers
2pr 2
If a, b and c are in AP, then b is AM of a and c. T10 = y ⇒ ar 10 − 1 = y ⇒ ar 9 = y …(ii) p, q, r are in HP ⇒ q= …(ii)  n ( n + 1)
p+ r = ∑ n 3 = 13 + 23 +…+ n 3 = 
Let a and b be two numbers and M be the AM between T16 = z ⇒ ar 16 − 1 = z ⇒ ar 15 = z …(iii)  2 
and ap, bq, cr are in GP, b2q2 = apcr
a+b
them. Then, a, M, b are in AP, M = . On multiplying Eq. (i), by Eq. (iii), we get
2
ar 3 × ar 15 = xz ⇒ a2r 18 = xz
EXAMPLE 2. Find the arithmetic mean (AM) ⇒ ( ar 9 ) 2 = xz ⇒ y 2 = xz [from Eq. (i)]

PRACTICE EXERCISE
between 3 and 9. Hence, x, y and z are in GP.
a. 4 b. 6
c. 8 d. None of these Harmonic Progression (HP)
3 + 9 12 A sequence is said to be harmonic progression (HP). If
Sol. b. The arithmetic mean is = = 6.
2 2 the reciprocals of its terms are in arithmetic progression
So, 3, 6, 9 are in AP. 3 + 5 + 7 +L+ n 5. An AP consists of n (odd terms) and its middle
(AP). 1. If = 7, then the value of n
5 + 8 + 11 + L + 10 terms term is m. Then, the sum of the AP is
1 1 1
Geometric Progression e.g. The sequence, 1, , , … is an HP because the
3 5 7
is (a) 2mn (b)
1
mn (c) mn (d) mn2
A geometric progression is a progression of numbers, (a) 35 (b) 36 (c) 37 (d) 40 2
sequence 1, 3, 5, 7 … is an AP.
whose first term is non-zero and each of the term is 2. If sum of n terms of an AP is 3n2 + 5n and 2 3 4
1 1 1 1 6. The sum of 1 + + + + L ∞ upto n terms is
obtained by multiplying its preceding term by a If a1 , a 2 , a 3 , . . ., a n are in HP, then , , ,..., Tm = 164, then m is equal to 5 52 53
constant quantity. This constant quantity is called the a1 a 2 a 3 an 25 15
(a) 26 (b) 27 (c) 28 (d) None of these (a) (b)
common ratio of the GP. are in AP. 16 16
1
3. In a GP, if the ( m + n )th term be p and ( m − n )th 5 3
Thus, if t 1 ,t 2 and t 3 are in GP, then common ratio • nth term of an HP is T n = term be q, then its mth term is (c) (d)
Second term t 2 1  1 1 p+ q
16 2
r= = + ( n − 1) −  (a) pq (b) p/q (c) q/p (d)
First term t1 a1  a 2 a1  7. The sum of n terms of an AP is an ( n − 1). The
4. The sum of the first ‘n’ terms of the series sum of the squares of these terms is equal to
If ‘a’ is the first term and ‘r’ is the common ratio, then 1 3 7 15
+ + + + L is a2
GP can be written as a, ar , ar 2 , ar 3 , …, ar n−1 = ( a ≠ 0 )
Harmonic Mean (HM) 2 4 8 16 (a) a2n2 (n − 1)2 (b)
6
n (n − 1) (2 n − 1)
If three terms are in HP, then the middle term is called n
(a) 2 − n − 1 (b) 1 − 2 −n 2
2 a2
n th term of a GP is Tn = ar n−1 = l.
2a
• the harmonic mean of the other two. If a, b and c are (c) n (n − 1) (2 n − 1) (d) n (n + 1) (2 n + 1)
(c) n + 2 − n − 1 (d) 2 n − 1 3 3
[where, l = last term] in HM, then b is the HM of a and c.
MATHEMATICS Sequence and Series 23 24 CDS Pathfinder

5. (c) Middle term = T ( c − b + a) ( b − c − a) ∴mnth term of HP = 1


8. If S be the sum to infinity of a GP, whose first 14. If the sum of first ‘n’ natural numbers is n +1 ⇒ =
c( b − a ) a( b − c )
term is a, then the sum of first n terms is n( n + 1) 2 13. (c) a , 2a + 2, 3a + 3 are in GP.
n+ 1
n
2
. Then, what will be the sum of first ‘n’ ∴ a +  − 1 d = m [given] ⇒
1
=−
1
⇒ ( 2 a + 2 ) 2 = (3 a + 3 )a
(a) S  1 − 
a  2  c( b − a ) a( b − c )
 S terms of the series of alternate positive and ⇒ 4a 2 + 4 + 8a = 3a 2 + 3a
2a + ( n − 1) d = 2m …(i) ⇒ ba − ca = − cb + ac
 negative numbers when ‘n’ is even? ⇒ a 2 + 5a + 4 = 0
a 
n n
(b) S 1 −  1 −   Now, Sn = [ 2a + ( n − 1) d ] = nm ⇒ ab + bc = 2ac ⇒ a = −1, − 4
 S  12 − 22 + 32 − 42 + 52 − K 2
 2ac
∴ b= Now, a = −1 does not satisfy the given
 a 
n n (n + 1) n2 (n + 1) − n (n + 1) 6. (a) The given sequence is arithmetic a+ c
(c) a 1 −  1 −   I.
2
II.
2
III.
2
1
geometric series, where r = and d = 1
series.
 S  Hence, a , b , c are in HP. ∴ −4, − 6, − 9 are in GP.
 5
Which of the above statement(s) is/are correct? a dr 3
S∞ = +
∴ t = −4   = −13.5
(d) None of the above 11. (a) Q a, x, y, z, b are in AP. 3
(a) Only I (b) Only III (c) Only II (d) None of these 1 − r ( 1 − r )2 a + b  2
∴ x + y + z = 3 
4
9. If the non-zero numbers a , b, c are in AP and 1 
1×  2 
1
tan− 1 a , tan− 1 b, tan− 1 c are also in AP, then = + 5
+ b
14. (b) When ‘n’ is even.
PREVIOUS YEARS QUESTIONS 15 = 3
1 2 a
1−  1 − 1 ⇒  Let n = 2m, then
(a) a = b = c (b) b 2 = 2 ac 5  5
  2 
(c) a2 = bc (d) c 2 = ab 15. If A, G and H are the arithmetic, geometric and = 12 − 22 + 32 − 42 + 52 − K
= +
5 5
=
25 ⇒ a + b = 10 …(i)
= ( 12 − 22 ) + (32 − 42 ) + (52 − 62 )
1 1 1 1 harmonic means between a and b respectively, 4 16 16 Also, a , x , y , z , b are in HP.
10. + = + , then a , b, c are in then which one of the following relations is + K + ( 2m − 1)2 − ( 2m )2
b− a b− c a c 7. (c) Let Sn = an( n − 1), then ⇒
1 1 1 1 1
, , , , are in AP.
correct? = ( 1 + 2)( 1 − 1) + (3 + 4)( − 1)
(a) AP (b) GP S = a ( n − 1)( n − 2) a x y z b
n −1 + (5 + 6)( − 1) + K
(a) G is the geometric mean between A and H e 2015 I ∴ Tn = Sn − S = 2a ( n − 1) a + b
+ + = 3
(c) HP (d) None of these 1 1 1
(b) A is the arithmetic mean between G and H n −1 ⇒  + ( 2m − 1 + 2m )( − 1)
Tn 2 = 4a 2 ( n − 1)2 x y z  2ab 
11. The value of x + y + z is 15, if a , x , y , z , b are in (c) H is the harmonic mean between A and G 2 2 ( n − 1)( n)( 2n − 1)
= − ( 1 + 2 + 3 + 4 + ... + 2m )
1 1 1 5 ∴ Sum = ΣTn = 4a ⇒ − n ( n + 1)
AP while the value of + + is , if a , x , y , z , b (d) None of the above 6 5 3 × 10 = −2m( 2m + 1) =
x y z 3 = [Q a + b = 10]
16. Consider the following statements in respect of 2a 2 n( n − 1)( 2n − 1) 3 2ab 2 2
=
are in HP. Then, a and b are
n( n + 1) 3 ⇒ ab = 9 15. (a) Given, A, G and H are the arithmetic,
(a) 1, 9 (b) 3, 7 the expression S n = , where ‘n’ is an geometric and harmonic means between
2 8. (b) Let r be the common ratio of GP, On solving Eqs. (i) and (ii), we get
(c) 7, 3 (d) None of these then a and b, respectively.
integer. a = 1, b = 9 or b = 1, a = 9
12. If the mth and nth term of a HP are n and m S=
a
, r = 1−
a a+ b 2ab
I. There are exactly two values of n for which ∴A = , G = ab and H =
respectively, then the mnth term is 1− r S 12. (b) Let a be the first term and d the 2 a+ b
S n = 861. a( 1 − r )
n
a common difference of corresponding ( i) ( ii ) ( iii )
(a) 0 (b) 1 (c) 2 (d)
1
II. S n = S − ( n +1) and hence for any integer m we have ∴ Sn = = (1 − rn ) AP.
2 1− r 1− r On multiplying Eq. (i) and (iii), we get
two values of n for which S n = m . n 1
 a 
= S 1 −  1 − 
So, m th and nth term of AP are and a+ b 2ab
13. If a , 2a + 2, 3a + 3 are in GP, then what is the Which of these statement(s) is/are correct? e 2016 (I)   S  
n ∴ AH = × = ab = ( ab )2
1 2 a+ b
fourth term of the GP? (a) Only I (b) Only II .
(a) − 13.5 (b) 13.5 (c) − 27 (d) 27 (c) Both I and II (d) Neither I nor II 9. (a) Since, 2b = a + c …(i) m [from Eq. (ii)]
and 2 tan − 1 b = tan − 1 a + tan − 1 c ∴
1
= a + (m − 1) d …(i) AH = G 2
2b a+ c n
⇒ = Hence, the option (a) is correct.
ANSWERS ⇒
1 − b2 1 − ac
b 2 = ac
and
1
m
= a + ( n − 1) d …(ii) n ( n + 1)
[from Eq. (i)] 16. (a) I. Sn = = 861
2 2
1 2 3 4 5 6 7 8 9 10 ⇒ 4b = 4ac On solving Eqs. (i) and (ii), we get,
2
⇒ ( a + c ) − 4ac = 0 [from Eq. (i)] 1 1 ⇒ n2 + n − 861 × 2 = 0
11 12 13 14 15 16 d = and a =
⇒ ( a − c )2 = 0 mn mn ⇒ ( n + 42) ( n − 41) = 0
⇒ a=c=b ∴mnth term of AP ⇒ n = − 42, 41
1 1 1 1 1 1 Hence, statement I is correct.
+ = + = + (mn − 1) ×
HINTS AND SOLUTIONS 10. (c) Given,


1
b−a b−c a
1 1
− = −
1
c
=
mn
1 + mn − 1
=1
mn II. Given, Sn = S
− ( n + 1)
If Sn = m, then we have two values of
n 1 3 7 15 b−a c a b−c mn n if and only if m is positive integer.
[ 2 × 3 + ( n − 1) 2] 2. (b) Q Tm = Sm − S 4. (c) + + + +... + n terms
2 m −1 Hence, statement II is incorrect.
1. (a) Q =7 ⇒ 164 = 3 ( 2m − 1) + 5 ⋅ 1 2 4 8 16
10
=  1 −  +  1 −  +  1 − 
[ 2 × 5 + ( 10 − 1) × 3] 1 1 1
2 ⇒ 6m = 162  2  4  8
n( n + 2) ∴ m = 27
⇒ =7 + ... +  1 − n 
1
5 × 37 3. (a) Tm + n = arm + n − 1 = p,  2 
2
⇒ n + 2n − 1295 = 0 Tm − n = arm − n − 1 = q  1 1 1 1
= n −  + + + ... + n 
⇒ n2 + 37n − 35n − 1295 = 0 On multiplying, we get a 2 r 2 m − 2 = pq 2 4 8 2 
⇒ ( n + 37) ( n − 35) = 0 1  1 − ( 1 / 2)n 
∴ Tm = arm − 1 = pq = n−  −n
 = n+ 2 − 1
∴ n = 35 2  1− 1/ 2 
03
25 26 CDS Pathfinder

EXAMPLE 1. The LCM of 30, 250, 490 is Sol. b. LCM (5, 6, 7, 8) = 840.
a. 46750 b. 36750 Here, R = 3 ⇒ Number is of the form 840 k + 3.
c. 26750 d. None of these Least value of k for which (840 k + 3) when divided by 9
leaves no remainder is 2.
Sol. b. Here, 30 = 2 × 3 × 5
250 = 5 × 5 × 2 × 5 = 2 × 53 ∴Required number = 840 × 2 + 3 = 1683
and 490 = 7 × 7 × 2 × 5 = 2 × 5 × 72
∴ LCM of 30, 250 and 490 = 2 × 53 × 72 × 3 = 36750 Highest Common Factor (HCF)
2. Division Method Write the given number in a row and 1. Common Factor A common factor of two or more
divide them with the common prime divisor. On numbers is a number which divides each of them exactly.
division. Write the quotient in each case below the

HCF AND LCM OF number. If any number is not divisible by the


respective divisor, then write it as such in the next row.
Keep on dividing the quotients until you get 1.
e.g. 2 is common factor of 2, 10, 20.
2. Highest Common Factor The Highest Common
Factor (HCF) of two or more numbers is the largest
number that divides all the given numbers exactly. It is

NUMBERS
Multiply all the divisors to get the required LCM. also known as Greatest Common Divisor (GCD).
HCF is always a factor of LCM.
EXAMPLE 2. What is the LCM of 120, 144, 160 and
e.g. HCF of the numbers 18 and 24 is 6.
180.
a. 1450 b. 1620 c. 1440 d. 1380 Methods of Finding HCF
Sol. c. 1. HCF by Prime Factorization Write the given number
2 120, 144, 160, 180
as product of prime factors and then find the product
2 60, 72, 80, 90
of least powers of common prime factors. This
2 30, 36, 40, 45
product is the required HCF of given numbers.
Usually (2-3) questions have been asked from this chapter. Generally questions based on LCM and HCF 3 15, 18, 20, 45
are related to traffic lights, racitracks, largest size of tile etc. This concept is also useful in the chapters 3 5, 3, 10, 15 EXAMPLE 5. The HCF of 65, 75 and 105 is
5 5, 1, 10, 5
of time and distance, time and work, pipes and cristerns etc. a. 4 b. 5 c. 6 d. 8
2 1, 1, 2, 1
1, 1, 1 1 Sol. b. Here, 65 = 13 × 5 , 75 = 5 × 5 × 3
and 105 = 7 × 3 × 5
∴LCM of 120, 144, 160 and 180 = 2 × 2 × 2 × 3 × 3 × 5 × 2 ∴HCF of 65, 75 and 105 = 5
= 1440
FACTORS 2. HCF by Division Method Suppose we have to find
the HCF of two given numbers, divide the large
IMPORTANT POINTS
If number ‘a’ divides the number b without leaving a remainder, then ‘a’ is said to be the factor of ‘b’. number by the smaller one. Now, you will get the
1. The least number which when divided by x, y and z
e.g. (i) 4 is a factor of 16 as 16 = 4 × 4 (ii) 9 is a factor of 729 as 729 = 9 × 9 × 9 remainder. Divide the divisor by the remainder. Repeat
leaving the remainders a , b and c , respectively is
given by [LCM of ( x, y , z ) − p ], where this process until no remainder is left, the last divisor
Prime Factors p = ( x − a ) = ( y − b ) = ( z − c ). used in this process is the desired greatest common
The factors that cannot be again factorized i.e. the factors which are prime numbers are called prime factors. 2. The least number which when divided by x, y and z divisor i.e. HCF. In order to find the HCF of three
e.g. (i) 9 is a factor of 729, but 3 is a prime factor of 729. leaving the same remainder R in each case is given numbers, then, HCF of [(HCF of any two) and (third
by [LCM of ( x, y , z ) + R ]. number)] gives the HCF of given three numbers.
(ii) 8 and 9 are factors of 72, but 2 and 3 are prime factors of 72.

Least Common Multiple (LCM) EXAMPLE 3. What is the least number which when EXAMPLE 6. The HCF of 204, 1190 and 1445 is
divided by 42, 72 and 84 leaves the remainders 25, 55 a. 85 b. 15 c. 17 d. 75
1. Common Multiple A common multiple of two or more numbers is a number which is exactly divisible
and 67, respectively?
(without leaving remainder) by each of them. Sol. c. Here, 1190)1445 (1
a. 521 b. 512 c. 504 d. 487
e.g. 30 is a common multiple of 2, 3, 5, 6, 10 and 15 because 30 is exactly divisible by each number. 1190
Sol. d. Here, difference = ( 42 − 25) = (72 − 55) = (84 − 67) = 17
Similarly, 72 is a common multiple of 2, 3, 4, 6, 9, 12 and 18. 255)1190 (4
Now, LCM (42, 72, 84) = 504
2. Least Common Multiple The least common multiple of two or more given numbers is the least number 1020
∴ Required number = 504 − 17 = 487
which is exactly divisible by all the given numbers. 170) 255 (1
e.g. 84, 162, 252 are the common multiples of 2, 3, 4, 7. But, 84 is the LCM of 2, 3, 4, 7. EXAMPLE 4. Find the least number which when 170
divided by 5, 6, 7 and 8 leaves a remainder 3 but 85)170 (2
Methods of Finding LCM when divided by 9, leaves no remainder. 170
1. Prime Factorization Method Write down the given numbers as the product of prime factors. Then, the a. 1620 b. 1683 ×
LCM is the product of the highest powers of all the prime factors. c. 1635 d. 1672 So, HCF of 1190 and 1445 is 85.
MATHEMATICS HCF and L CM of Numbers 27 28 CDS Pathfinder

Now, 85) 204 (2


170
34) 85 (2
Sol. a. HCF of numerators i.e. 14, 21 and 7 is 7 and
LCM of denominators i.e. 3, 9 and 15 is 45. So, HCF of
7
given fractions = .
45
PRACTICE EXERCISE
68
17) 34 (2
34
Relation between LCM and 1. If x = 23 × 32 × 54 and y = 22 × 32 × 5 × 7, then 13. What is the HCF of a 2b4 + 2a 2b2 and ( ab)7 − 4a 2b9?
× HCF of Two Numbers HCF of x and y is (a) ab (b) a2 b 2 (c) a2 b 3 (d) a3 b 2
∴ HCF of 85 and 204 is 17. (a) 180 (b) 360 (c) 540 (d) 35
14. If a number is exactly divisible by 11 and 13,
Product of two numbers = (Their HCF) × (Their LCM)
Hence, HCF of 204,1190 and 1145 is 17. 2. LCM of 23 × 3 × 5 and 24 × 5 × 7 is which of the following types the number must be?
EXAMPLE 10. The LCM of two numbers is 90 times (a) Divisible by (11+13) (b) Divisible by (13 −11)
(a) 2 12
× 3× 5 ×7
2
(b) 2 × 5 × 7 × 9
4

IMPORTANT POINTS their HCF. The sum of LCM and HCF is 1456. If one of (c) Divisible by (11×13) (d) Divisible by (13 ÷11)
(c) 2 4 × 3 × 5 × 7 (d) 2 3 × 3 × 5 × 7
the numbers is 160, then what is the other number? 15. What is the sum of the digits of the least number
1. For integers x, y and z , if HCF (x, y) = 1 and e 2014 II 4 3 7
HCF (x, z) = 1, then HCF of ( x, y , z ) is always 1. 3. LCM of , and is which when divided by 52, leaves 33 as
a. 120 b. 136 c. 144 d. 184 5 10 15 remainder, when divided by 78 leaves 59 and
2. The greatest number that will divide x, y and z leaving 2 8 4
Sol. c. Let the HCF of two numbers be x. (a) 8 (b) (c) 20 (d) 16 when divided by 117, leaves 98 as remainder ?
remainders a , b and c , respectively is given by HCF of 3 15 5
LCM of two numbers be 90x. (a) 17 (b) 18 (c) 19 (d) 21
( x − a ), ( y − b ), ( z − c ).
According to the question, 3 9 15
3. The greatest number that will divide x, y and z leaving 4. What is the HCF of , and ? 16. For any integer n, what is the HCF of integers
the same remainder in each case is given by [HCF of LCM + HCF = 1456 2 7 14 m = 2n + 1 and k = 9n + 4 ?
3 3 3
| x − y | , | y − z | , | z − x |] ⇒ 90x + x = 1456 (a) (b) (c) (d) 3 (a) 3 (b) 1 (c) 2 (d) 4
7 14 2
⇒ 91 x = 1456 17. For any three natural numbers a , b and c, if HCF
EXAMPLE 7. Find the greatest number which will ⇒ x = 16 5. The least number divisible by 12, 15, 20 and is a
 a b
divide 400, 435 and 541 leaving 9,10 and 14 as ∴ HCF of two numbers = 16 perfect square is ( a , b) = c, then HCF  ,  is
 c c
remainders respectively. and LCM of two numbers = 90 × 16 = 1440 (a) 900 (b) 400 (c) 36 (d) 256
(a) a / c (b) b / c (c) c (d) Always 1
a. 19 b. 17 c. 13 d. 9 e 2014 I We know that, 6. The least number which when divided by 5, 6, 7
Sol. b Required number = HCF of (400-9, 435-10, 541-14) LCM × HCF = Product of two numbers and 8 leaves a remainder 3 is 18. Raj, Rachit and Asha begin to tag around a circular
= HCF of (391, 425, 527) = 17 stadium. They complete their revolutions in 42 s,
⇒ 1440 × 16 = 160 × Second number (a) 423 (b) 843 (c) 1683 (d) 2523
56 s and 63 s, respectively. After how many seconds
EXAMPLE 8. For any integers ‘a’ and ‘b’ with HCF 1440 × 16 7. The HCF of two numbers is 1/5th of their LCM.
∴Second number = = 144 will they be together at the starting point?
(a, b) = 1, what is HCF (a + b, a − b) equal to? 160 If the product of the two numbers is 720, then (a) 366 (b) 252 (c) 504 (d) 605
a. It is always 1 b. It is always 2 e 2014 I the HCF of the numbers is
c. Either 1 or 2 d. None of these EXAMPLE 11. What is the greatest number that (a) 13 (b) 12 (c) 14 (d) 18
19. Find the side of the largest possible square slabs
divides 13850 and 17030 and leaves a remainder 17? which can be paved on the floor of a room
Sol. c. Put arbitrary values of a and b. 8. The LCM of two numbers is 39780 and their
a. 477 b. 159 c. 107 d. 87 e 2012 II 2m 50cm long and 1m 50cm broad. Also, find the
Illustration 1 Let a = 9 and b = 8.
ratio is 13 : 15. Then, the numbers are number of such slabs to pave the floor.
∴ HCF (8 + 9, 9 − 8) ⇒ HCF (17, 1) = 1 Sol. b. Required number = HCF of (13850 − 17), (17030 − 17)
(a) 273, 315 (b) 2652, 3060 (a) 25,20 (b) 30,15 (c) 50,15 (d) 55,10
Illustration 2 Let a = 23 and b = 17. = HCF of (13833, 17013) = 159
(c) 516, 685 (d) None of these
20. Four bells begin to toll together and toll,
∴ HCF (17 + 23, 23 − 17) ⇒ HCF ( 40, 6) = 2 EXAMPLE 12. There are three drums with 1653 litre 9. If the highest common factor of two positive respectively at intervals of 5, 6, 8 and 12 s. How
Hence, HCF (a + b, a − b) can either be 1 or 2. 2261 litre and 2527 litre of petrol. The greatest integers is 24, then their least common multiple many times will they toll together in an hour
possible size of the measuring vessel with which we cannot be excluding the one at the start?
How to Calculate LCM and can measure the petrol of any drum while every time (a) 72 (b) 216 (c) 372 (d) 600 (a) 10 (b) 19 (c) 13 (d) 9
HCF of Fractions the vessel must be completely filled is
10. If the HCF of three numbers 144, x and 192 is 21. 21 mango trees, 42 apple trees and 56 orange
a. 31 b. 27 c. 19 d. 41 12, then the number x cannot be trees have to be planted in rows such that
The LCM and HCF of fractions can be obtained from Sol. c. The maximum capacity of the vessel = HCF of 1653, (a) 180 (b) 84 (c) 60 (d) 48 each row contains the same number of trees of
the following formula 2261 and 2527 = 19 one variety only. What is the minimum number
HCF of numerators 11. Consider those numbers between 300 and 400
1. HCF of fractions = of rows in which the above trees may be planted?
LCM of denominators EXAMPLE 13. John, Kate and smith at same time, such that when each number is divided by 6, 9 (a) 3 (b) 15 (c) 17 (d) 20
same point and in same direction to run around a and 12, it leaves 4 as remainder in each case.
LCM of numerators 22. A person has four iron bars whose lengths are
2. LCM of fractions = circular in 150 seconds. Find after what time will they What is the sum of the numbers?
HCF of denominators meet again? (a) 692 (b) 764 (c) 1080 (d) 1092 24 m, 36 m, 48 m and 72 m, respectively. This
person wants to cut pieces of same length
14 21 7 a. 30 min b. 25 min c. 20 min d. 15 min 12. What is the smallest positive integer which when
EXAMPLE 9. The HCF of , , is from each of four bars. What is the least number
3 9 15 Sol. b. LCM of 250, 300 and 150 = 1500 sec divided by 4, 5, 8 and 9 leaves remainder 3, 4, 7 of total pieces, if he is to cut without any
7 2 7 = 25 min. and 8, respectively? wastage?
a. b. 3 c. d. None of these Hence, john, kate and smith meet after 25 min.
45 5 30 (a) 119 (b) 319 (c) 359 (d) 719 (a) 10 b) 15 (c) 20 (d) 25
MATHEMATICS HCF and L CM of Numbers 29 30 CDS Pathfinder

23. For two natural numbers m and n, let gmn 33. For any integer n, HCF of (22n + 7, 33n + 10) is
denote the greatest common factor of m and n. equal to e 2014 I
ANSWERS
Consider the following in respect of three natural (a) n (b) 1 (c) 11 (d) None of these 1 2 3 4 5 6 7 8 9 10
numbers k, m and n.
34. In a fire range, 4 shooters are firing at their 11 12 13 14 15 16 17 18 19 20
I. gm ( nk) = g(mn) k II. gmn gnk = gmk
respective targets. The first, the second, the 21 22 23 24 25 26 27 28 29 30
Which of the above statement(s) is/are correct?
third and the fourth shooter hit the target once 31 32 33 34 35 36 37 38 39 40
(a) Only I (b) Only II
in every 5 s, 6 s, 7 s and 8 s, respectively. If all 41 42
(c) Both I and II (d) Neither I nor II
of them hit their target at 9 : 00 am, when will
24. Consider the following in respect of integers a they hit their target together again? e 2014 I
and b
I. HCF (a , b) = HCF (a + b, b)
(a) 9 : 04 am
(c) 9 : 14 am
(b) 9 : 08 am
(d) None of these HINTS AND SOLUTIONS
II. HCF (a , b) = HCF (a , b − a ) for b > a 35. If a and b be positive integers, then HCF of
Which of the above statement(s) is/are correct?  a b 
 ,  equal to? e 2014 I 1. (a) Given, x = 23 × 32 × 54 9. (c) In the given options, only 372 is not 16. (b) Since, m = 2n + 1 is an odd integer,
(a) Only I (b) Only II  HCF ( a,b) HCF ( a,b) and y = 22 × 32 × 5 × 7 divisible by 24. Therefore, LCM of so its factors may be 1 or 3 and
(c) Both I and II (d) None of these
(a) a (b) b (c) 1 (d)
a
∴ HCF = 2 × 3 × 5 = 4 × 9 × 5 = 180
2 2 numbers cannot be 372. k = 9 n + 4 its factors may be 1, 2 and 4.
HCF (a, b ) 10. (d) Here, we know that Hence, HCF of (m , k ) is 1.
2. (c) Here, say a = 23 × 3 × 5 and
PREVIOUS YEARS’ QUESTIONS 36. The HCF of two natural numbers m and n is 24 144 = 12 × 2 × 2 × 3 17. (d) It is always 1
and their product is 552. How many sets of b = 24 × 5 × 7, then and 192 = 12 × 2 × 2 × 2 × 2
25. The HCF and LCM of two natural numbers are Illustrations Let a = 21 and b = 35
values of m and n are possible? e 2014 II LCM = 24 × 3 × 5 × 7 By taking option (d), 48 = 12 × 2 × 2
12 and 72, respectively. What is the difference (a) 1 (b) 2 (c) 4 Then, HCF (21, 35) = 7
3. (d) Here, LCM Hence, the value of x will not be 48
∴ HCF  ,  = HCF (3, 5) = 1
between the two numbers, if one of the numbers (d) No set of m and n is possible satisfying the given 21 35
LCM of 4, 3, 7 84 4 otherwise the HCF of given numbers  7 7
is 24? e 2012 I conditions = = = 16 becomes 48.
(a) 12 (b) 18 (c) 21 (d) 24 HCF of 5, 10, 15 5 5
37. The LCM of two integers is 1237. What is their 11. (a) LCM of 6, 9 and 12 = 36 18. (c) Required time = LCM of 42,56 and
26. The sum of two numbers is 232 and their HCF is 4. (b) HCF  , ,  =
HCF? e 2014 II 3 9 15 HCF (3, 9, 15) 63 s LCM of 42, 56 and 63 is
So, number is the form of 36 p + 4.
29. What is the number of such pairs of numbers (a) 37 (b) 19  2 7 14  LCM (2, 7, 14) 2 42, 56, 63
Since, the required numbers are between
satisfying the above condition? e 2012 I (c) 1 (d) Cannot be determined 3 3 21, 28, 63
(a) One (b) Two (c) Four (d) None of these = 300 and 400.
38. There are 48 cricket balls, 72 hockey balls and 14 ∴ p = 9 and 10 7 7, 28, 21
27. The product of HCF and LCM of 18 and 15 is 84 tennis balls and they have to be arranged in 5. (a) 2 1, 4, 3
12, 15, 20 ∴ Required sum = 328 + 364 = 692
e 2012 II several rows in such a way that every row
2 6, 15, 10 ∴ Required time
(a) 120 (b) 150 (c) 175 (d) 270 contains the same number of balls of one type. 12. (c) Here,
3 3, 15, 5 = 2 × 3 × 7 × 4 × 3 = 504 s.
28. Three planets revolve round the Sun once in 200, What is the minimum number of rows required 4−3=5− 4=8− 7= 9−8= 1
for this to happen? e 2014 II 5 1, 5, 5 19. (c) Side of largest possible square slab is
250 and 300 days, respectively in their own Now, 4 = 2 × 2, 5 = 5 the HCF of 250 cm and 150 cm
(a) 12 (b) 16 (c) 17 (d) 19 1, 1, 1
orbits. When do they all come relatively to the 8 = 2 × 2 × 2, 9 = 3 × 3 250 = 5 × 5 × 5 × 2
same position at a certain point of time in their 39. Consider all positive two digit numbers each of LCM of 12, 15 and 20 = 2 × 2 × 3 × 5
∴ LCM = 5 × 2 × 2 × 2 × 3 × 3 = 360 150 = 5 × 5 × 3 × 2
orbits? e 2012 II which when divided by 7 leaves a remainder 3. ∴ Required perfect square
Required number = 360 − 1 = 359 ∴ HCF is 50. Then, number of slabs
(a) After 3000 days (b) After 2000 days What is their sum? e 2015 II = 2 × 2 × 3 × 3 × 5 × 5 = 900
(a) 661 (b) 666 (c) 676 (d) 777 13. (b) a b + 2a b = a b ( b + 2)
2 4 2 2 2 2 2
…(i) Area of floor 250 × 150
(c) After 1500 days (d) After 1200 days 6. (b) To find out the least number, firstly = = = 15
and ( ab )7 − 4a 2 b 9 = a 7 b 7 − 4a 2 b 9 Area of slab 50 × 50
29. The LCM of two numbers is 2376 while their 40. What is the sum of digits of the least multiple of we find out the LCM of given numbers.
13, which when divided by 6, 8 and 12 leaves 5, 7 ∴ LCM (5, 6, 7, 8) = a b ( a b − 4b )
2 2 5 5 7
…(ii) 20. (a) Here, LCM of 5, 6, 8 and 12 is 360, so
HCF is 33. If one of the numbers is 297, then the
and 11, respectively, as the remainders? e 2015 II = LCM (5, 2 × 3, 7, 23 ) = 840 From Eqs. (i) and (ii), we get the bells will toll after 360 s.
other number is e 2013 I
(a) 5 (b) 6 (c) 7 (d) 8 ∴ Required number = 840 + 3 = 843 HCF = a b 2 2 So, in an hour they will toll together
(a) 216 (b) 264 (c) 642 (d) 792
60 × 60
30. The HCF of two numbers is 98 and their LCM is 41. The LCM of two numbers is 12 times their HCF. 7. (b) Let LCM = 5x, then HCF = x 14. (c) LCM of 11 and 13 will be ( 11 × 13). = = 10 times
The sum of HCF and LCM is 403. If one of the Now, product of numbers = 720 Hence, if a number is exactly divisible 360
2352. The sum of the numbers may be e 2013 II
numbers is 93, then the other number is e 2015 II So, 5x × x = 720 by 11 and 13, then the same number 21. (c) The HCF of (21, 42, 56) is 7.
(a) 1372 (b) 1398 (c) 1426 (d) 1484 (a) 124 (b) 128 (c) 134 (d) 138 ⇒ 5x2 = 720 ⇒ x = 12 must be exactly divisible by their LCM ∴ The minimum number of rows
31. If for integers a, b and c, HCF ( a , b) = 1 and HCF i.e. ( 11 × 13). 21 42 56
42. Consider the following in respect of natural 8. (b) The numbers are 13x and 15x. = + +
( a , c) = 1 , then which one of the following is numbers a , b and c e 2016 I So, x is the HCF. Now, 15. (a) Here, 52 − 33 = 78 − 59 7 7 7
= 117 − 98 = 19 = 3 + 6 + 8 = 17
correct? e 2013 II I. LCM (ab, ac) = a LCM (b, c) HCF × LCM = Product of numbers
(a) HCF (a, bc ) = 1 (b) HCF (a, bc ) = a Now, 52 = 13 × 2 × 2 22. (b) Here, 24 = 12 × 2, 36 = 12 × 3
II. HCF (ab, ac) = a HCF (b, c) x × 39780 = 13x × 15x ⇒ 78 = 13 × 2 × 3
(c) HCF (a, bc ) = b (d) None of these 48 = 12 × 4, 72 = 12 × 6
III. HCF (a , b) < LCM (a , b) ⇒ x × 39780 = 13 × 15 × x2 ⇒ 117 = 13 × 3 × 3
32. What is the number of integral solutions of the IV. HCF (a , b) divides LCM (a , b). 39780 ∴ LCM = 13 × 2 × 2 × 3 × 3 = 468 ∴ HCF ( 24, 36, 48, 72) = 12
⇒ x= = 204. 24 36 48 72
equations HCF (a, b) = 5 and a + b = 65 ? e 2014 I Which of the above statement(s) is/are correct? 13 × 15 ∴ Required number = 468 − 19 = 449 Total piece = + + +
(a) Less than 65 (b) Infinitely many 12 12 12 12
(a) III and II (b) III and IV ∴ Numbers are 13 × 204 = 2652 and Hence, the sum of the digits is 17.
(c) Exactly one (d) None of these (c) I, II and IV (d) All of these 15 × 204 = 3060 = 2 + 3 + 4 + 6 = 15
04
MATHEMATICS HCF and L CM of Numbers 31 32 CDS Pathfinder

23. (d) I. Let three natural numbers are 30. (a) It is given that, the HCF of two 38. (c) Given, number of cricket balls
m = 16, n = 15, k = 20 numbers is 98. This means that both the = 48 = 24 × 3
∴ gm = g =4 numbers are multiples of 98. Therefore,
( nk ) 16 ( 300 ) the sum of these two numbers must also Number of hockey balls = 72 = 23 × 32
and g = g = 20 be a multiple of 98. Among all the four and number of tennis balls
(m n ) k 240 ( 20 )
∴ g ≠ g options given, only option (a) satisfies = 84 = 22 × 3 × 7
m ( nk ) ( mn ) k
this condition.
II. g mn g = g g = 1×5 =5 ∴ HCF of 48, 72 and 84 = 22 × 3 = 12
nk 16 , 15 15 , 20
and g = g =4 31. (a) For integers a, b and c, if HCF (a, b) = Now, minimum number of rows
mk 16 , 20 1 and HCF (a, c) = 1 then, HCF (a, b c)
∴ g mn g ≠ g 48 72 84
nk mk =1 = + +

DECIMAL FRACTIONS
So, neither I nor II is correct. 12 12 12
32. (a) HCF ( a , b ) = 5
24. (c) I. Let a = 4, and b = 10 = 4 + 6 + 7 = 17
Let a = 5x and b = 5 y
∴ a + b = 14 39. (c) The required numbers are 10, 17,
∴ 5x + 5 y = 65 ⇒ x + y = 13
24,…, 94.
HCF ( 4, 10) = 2 ∴ Number of pairs of ( x, y ) = ( 1, 12),
Total number of numbers is 13.
and HCF (14, 10) = 2 (2, 11), (3, 10), (4, 9), (5, 8), (6, 7)
Sum of these numbers
∴ HCF ( a , b ) = HCF ( a + b , b ) Hence, number of solutions is less than
13 13
II. Let a = 6 and b = 15 65. = [ 10 + 94] = × 104
2 2
∴ b − a = 15 − 6 = 9 33. (b) HCF of (22n + 7, 33n + 10) is
always 1. = 13 × 52 = 676
HCF (6, 15) = 3
Illustration For n = 1, HCF (29, 43) 40. (d) Here, 6 − 5 = 1, 8 − 7 = 1 Regularly (1-2) questions have been asked from this chapter. It is one of the most common chapters which
HCF (6, 9) = 3
=1 12 − 11 = 1
∴ HCF ( a , b ) = HCF ( a , b − a ) we are studying from the very starting age.
LCM × HCF For n = 2, HCF (51, 76) = 1 LCM of 6,8 and 12 = 24
25. (a) Second number =
First number For n = 3, HCF (73, 109)= 1 Required number
72 × 12 34. (c) Time after which they will hit the = 24 k − 1, k is any natural number
= = 36 target together again
24 For k = 6, the number = 144 − 1 = 143
∴ Difference between two numbers = LCM of 5, 6, 7 and 8 which is multiple of 13 FRACTION
= 36 − 24 = 12 = 5 × 3 × 7 × 2 × 2 × 2 = 840 s So, sum of digits = 1 + 4 + 3 = 8 If any unit is divided into some parts, then ...... each of these parts is called fraction of that unit. A
26. (b) Let two numbers be 29x and 29 y. Duration after which they will hit target
together
41. (a) Let other number be b and HCF be x. fraction is represented as p/q, where q ≠ 0 and here q is called as denominator and p is called as
29 x + 29 y = 232 ⇒ x + y = 8 ⇒ LCM = 12x numerator.
840
∴ ( x, y ) = ( 1, 7), (3, 5) = = 14 min.
60 We have, x + 12x = 403 ⇒ 13x = 403 e.g. 1/ 2, 3/ 4, 6 /7 etc. are fractions.
Hence, one such pair is 87 and 145. So, they will hit the target together after ∴ x = 31
and the other pair is 203 and 29. 14 min. Product of two numbers Simple fraction
27. (d) Here, 18 = 2 × 3 × 3 and 15 = 3 × 5 Hence, they will hit together again at = LCM × HCF 6 2
9 : 14 am. The fraction which has denominator other than power of 10, is called simple fraction. e.g. , etc.
HCF of 18 and 15 = 3 ∴ 93 × b = x × 12x 5 9
 a b  ⇒ 93 × b = 12 × 31 × 31
LCM of 18 and 15 = 2 × 3 × 3 × 5 = 90 35. (c) HCF  ,  is
 HCF a, b HCF a, b  Note Simple fraction is also known as vulgar fraction.
∴ b = 124
∴ Product of HCF and LCM of both always equal to 1.
numbers = 3 × 90 = 270 42. (d) a , b and c are natural numbers.
28. (a) Given that, three planets revolves the
Illustration Let the two positive
integers be a = 24 and b = 36.
I. LCM of ( ab , ac ) = abc DECIMAL FRACTION
Sun once in 200, 250 and 300 days, a × LCM of ( b , c ) = abc Fraction that has powers of 10 in the denominator are called decimal fraction. e.g.
 
∴HCF  
respectively in their own orbits. 24 36 Hence, statement I is correct. 1 1
,
∴ Required time = LCM of 200, 250  HCF ( 24, 36) HCF ( 24, 36)  II. HCF ( ab , ac ) = a HCF ( b , c ) (i) is the tenth part of 1 is written as 0.1. (ii) is the hundredth part of 1 is written as 0.01.
  10 100
and 300 = 3000 days HCF of ( ab , ac ) = Common factor of
HCF  ,  ⇒ HCF (2, 3) = 1
24 36
( ab , ac )
Hence, after 3000 days they all come
relatively to the same position at a
 12 12 
and a × HCF ( b , c ) = a × common
Types of Decimals
certain point of time in their orbits. 36. (d) LCM of two natural numbers factor of ( b , c ) 1. Recurring Decimals A decimal number in which a digit or set of digits repeats regularly is called
29. (b) Given, LCM of two numbers = 2376 =
Product of m and n 552
= = 23 Hence, statement II is correct. non-terminating repeating decimals or recurring decimals. To represent these fractions, a line is
HCF of two numbers = 33 HCF of m and n 24 III. We know that HCF is always less drawn on repeating digits.
Here, no set of m and n is possible than LCM. 1
One of the number = 297 e.g. = 0 .1428571428571 . . . = 0. 142857
satisfying the given conditions as LCM Hence, statement III is correct. 7
(HCF of two numbers) × (LCM of is always a multiple of HCF. IV. HCF ( a , b ) divides LCM ( a , b ) 2
two numbers) because a common factor = 0 .66666 . . . = 0 .6
37. (c) Given, LCM of two integers is 1237, 3
= (First number) × (Second number) between a , b always divides ( a × b ).
which is a prime number.
33 × 2376 4
∴ Second number = = 264 So, their HCF is 1.
Hence, statement IV is correct. = 1.33333 . . . = 1.3
297 3
MATHEMATICS Decimal Fractions 33 34 CDS Pathfinder

2. Pure recurring decimal A decimal fraction in Sol. c. Here, To Convert a Pure Recurring Decimal 2 5 16 3
EXAMPLE 10. Arrange , , , in ascending
which all the digits after the decimal points are 51.300
into a Simple Fraction 3 6 25 7
repeated, is called a pure recurring decimal. 7.078 order.
In order to convert a pure recurring decimal fraction
e.g. 0.786786786 . . . = 0. 786 1.380
into simple (vulgar) fraction, we write the repeated a.
2 5 16
, , and
3
b.
16 5 3
, , and
2
+ 0.900
0.77777 . . . = 0.7 figures only once in the numerator without decimal 3 6 25 7 25 6 7 3
60.658
point and write as many nines in the denominator as the 3 16 2 5
EXAMPLE 1. Which one of the following is a c. , , and d. None of these
number of repeating figure. 7 25 3 6
non-terminating repeating decimal? Note Adding/Annexing zeroes to the extreme right of a decimal
5 45
13 3 3 137
fraction does not change its value. e.g. 0. 5 = or 0. 45 = Sol. c. Firstly, we convert each of the fraction in decimal
a. b. c. d. 9 99 form.
8 16 11 25
Multiplication 2 5
= 0. 666, = 0.833,
16 3
= 0.64, = 0. 428571
3
= 0 ⋅ 272727...
EXAMPLE 7. 1.27 in the form p/q is equal to 3 6 25 7
Sol. c. Q While multiplying two or more decimal fractions,
11 127 73 14 11
consider them without decimal point and multiply them a. b. c. d. Here, 0. 428571 < 0.64 < 0. 666 < 0.8333
= 0 ⋅ 27 100 100 11 14
as usual. In the product so obtained the decimal point is Hence,
3 16 2 5
< < < and so ,
3 16 2 5
, and are in
13 13 × 125 1625 marked off as many places from the right as in the sum 99 + 27 126 14 7 25 3 6 7 25 3 6
= = = 1.625 Sol. c. 1. 27 = 1 + 0. 27 = 1 + 27 / 99 = = =
8 8 × 125 1000 of the decimal places in the given numbers. 99 99 11 ascending order.
3 3 × 625 1875 To Convert Mixed Recurring Decimal 7 5 6
= = = 0.1875 EXAMPLE 4. 5 × 0 . 25 × 6.301 × 0.00394 = ? EXAMPLE 11. Arrange , , in descending order
16 16 × 625 10000 8 6 7
137 137 × 4 548
a. 0.0310324220 b. 0.0310324210 into a Simple Fraction
= = = 5.48 a. 6/7, 5/6, 7/8 b. 7/8, 5/6, 6/7
c. 0.0310324250 d. None of these The numerator is obtained by taking the difference
25 25 × 4 100 c. 7/8, 6/7, 5/6 d. 6/7, 7/8, 5/6
Sol. c. For the product of 5 × 0.25 × 6.301 × 0.00394 between the number formed by all the digits after decimal
From above it is clear that all of these are terminating point. (Here, repeated digits are taken only once) and the Sol. c. LCM of 8, 6, 7 = 168
3 Here, 5 × 25 × 6301 × 394 = 310324250 number formed by non-repeating digits. The
decimals. Hence, is a non-terminating repeating
11 Total decimal places ( 2 + 3 + 5) = 10 7 7 × 21 147 5 5 × 28 140
denominator is obtained by placing, as many nines as in ∴ = = ⇒ = =
decimal. ∴ 5 × 0. 25 × 6. 301 × 0. 00394 = 0.031032425 repeating digits followed by as many zeroes as the 8 8 × 21 168 6 6 × 28 168
3. Mixed recurring decimal A decimal fraction in number of non-repeating digits. 6 6 × 24 144
and = =
which some digits after the decimal point are not Division 7 7 × 24 168
repeated while some are repeated, is called a mixed While dividing the given decimal fraction by a natural EXAMPLE 8. The vulgar fraction of 0.1236 is
140 144 147
recurring decimal. number, divide it without the decimal point to get 120 102 93 Q < <
a. b. c. d. None of these 168 168 168
e.g. 0.179999 . . . = 0.179 quotient. Here, in the quotient so obtained, place the 823 825 825
7 6 5
0.053939 . . . = 0.0539 decimal point after as many places from the right as are 1236 − 12 1224 102 So, descending order of fractions is , , .
Sol. b. 0.1236 = = = 8 7 6
there in the dividend. 9900 9900 825
Note The decimal expansion of a rational number is either
terminating or non-terminating recurring. In other words, a For dividing a decimal fraction by a decimal fraction, EXAMPLE 9. What is the value of 1. 34 + 4.12 ? HCF and LCM of
number whose decimal expansion is terminating or multiply the dividend and the divisor by a suitable power
non-terminating recurring is a rational number. of 10 to make the divisor a whole number and then a.
133
b.
371
c. 5
219
d. 5
461 Decimal Fractions
90 90 990 990
proceed as in the previous rule. For finding the HCF or LCM of decimal fractions, first
Operations on Decimal Fractions, 134 − 1 133 412 − 41 371 make the given fractions to have same number of
Sol. d. Q 1. 34 = = and 4.12 = =
Addition and Subtraction EXAMPLE 5. 0.01834 ÷ 13 = ? 99 99 90 90 decimal places by annexing zeroes, if needed. Now, find
a. 141.077 b. 1.41077 133 371 1330 + 4081 5411 461 the HCF or LCM of the numbers without considering
∴134
. + 4.12 = + = = =5
While making addition or subtraction of decimal c. 0.00141077 d. None of these 99 90 990 990 990 decimal. Finally, in the result mark as many decimal
fractions the numbers are placed in such a way that the places as are there in each of the given numbers.
Sol. c. Here, for 0.01834 ÷ 13
decimal point lie in one column. Then, the numbers Comparison of Fractions By EXAMPLE 12. The HCF and LCM of 1.600, 8 and 2.4 is
can be added or subtracted as usual.

1834
= 141077
. Converting in Decimal Form a. 0.800 and 24 b. 0.6000 and 38
EXAMPLE 2. 7.093 − 3.57 = ? 13
To compare the fraction using decimal, convert each one
So, 0 . 01834 ÷ 13 = 0 . 00141077 c. 0.900 and 42 d. None of these
a. 3.523 b. 3.513 of the given fractions in decimal form and now arrange
c. 3.143 d. 3.532 them in ascending or descending order as per the Sol. a. We can write the given numbers as 1.600, 8.000,
EXAMPLE 6. 0 .0066 ÷ 0 .22 = ? 2.400.
Sol. a. Here, 7.093 requirement.
– 3.570 a. 0.03 b. 0.3 c. 3 d. None of these So, without decimal, the numbers are 1600, 8000 and
3.523 Sol. a. 0 . 0066 ÷ 0 . 22 By Equating Denominators 2400.
Now, HCF of 1600, 8000 and 2400 is 800.
0.0066 × 10000
For comparison of fractions, take LCM of the
EXAMPLE 3. 51.3 + 7 .078 + 1.38 + 0 .9 = ? So, =
66
=
3
= 0.03 denominator of all fractions. So, that their denominators ∴ HCF of 1.600, 8.000 and 2.400 is 0.800.
a. 61.668 b. 59.238 0.22 × 10000 22 × 100 100 And LCM of 1600, 8000, 2400 is 24000.
are same. Now, the fractions having largest numerator is
c. 60.658 d. None of these the largest fraction. ∴ LCM of 1.600, 8.000 and 2.400 is 24.000.
MATHEMATICS Decimal Fractions 35 36 CDS Pathfinder

PRACTICE EXERCISE 25. Consider the following decimal numbers:


I. 1.16666666…
III. 2.010010001…
II.
IV.
1.181181118…
1.454545…
28. Representation of 0. 2341 in the form
and q are integers, q ≠ 0, is
p
q
, where p

e 2013 I
781 1171 2341 2339
(a) (b) (c) (d)
1. If ( 15 . 9273 − x ) = 11. 0049, then the value of x is p Which of the above numbers represent(s) rational 3330 4995 9990 9990
15. If 2 . 5252525 . . . =(in the lowest form), then number(s)?
(a) 4.9224 (b) 0.4922 (c) 0.4294 (d) 6.932 q
q 29. Let p be a prime number other than 2 or 5. One
what is the value of ? (a) Only IV (b) II and III
2. If ( 15. 39 + 0 . 236 + 5 . 290 + 0 . 0002 ) = x , then the p (c) I and IV (d) None of these
would like to express the vulgar fraction 1/ p in
value of x is the form of a recurring decimal. Then, the
(a) 0.4 (b) 0.42525 (c) 0.0396 (d) 0.396
(a) 0.20916 (b) 2.0916 (c) 209.16 (d) 20.9162
decimal will be e 2015 I
16. What decimal of an hour is a second? PREVIOUS YEARS’ QUESTIONS (a) a pure recurring decimal and its period will be
3. If 175 × 1. 24 = 2.17, then the value of 1.75 × 124 is (a) 0.25 (b) 0.0256 (c) 0.00027 (d) 0.0125
necessarily ( p − 1)
(a) 217 (b) 0.0217 26. What is the value of 0.007 + 17. 83 + 310. 0202 ? (b) a mixed recurring decimal and its period will be
3 5 necessarily ( p − 1)
(c) 2.17 (d) None of these 17. If 5 = 2.24, then the value of is (a) 327.86638 (b) 327.86638 e 2012 I
(c) a pure recurring decimal and its period will be some
4. If 111.744 ÷ 28.8 = 3.88, then the value of 2 5 − 0.48 (c) 327.86683 (d) 327.8668 factor of ( p − 1)
(a) 1.68 (b) 16.8 (c) 168 (d) 0.168 (d) a mixed recurring decimal and its period will be
1117.44 ÷ 288 is 27. What is the value of 0.242424…? e 2012 II
some factor of ( p − 1)
(a) 3.88 (b) 0.388 (c) 388.0 (d) 38.8  ( 0. 1)2 − ( 0. 01)2  23 8 7 47
 
18. The value of  + 1 is equal to (a) (b) (c) (d)
( 0. 5)4
− ( 0.4)
4
 0. 0001 
99 33 33 198 30. The value of ( 0. 63 + 0. 37) is e 2015 II
5. The value of is equal to 100 100 1000
( 0. 5) + ( 0.4)
2 2 (a) 1001 (b) 11 (c) 101 (d) 100 (a) 1 (b) (c) (d)
91 99 999
(a) 0.9 (b) 0.09 (c) 9 (d) 0.009 3
19. The value of is equal to
0.004 × 0.0008 0. 3 − 3.03
3+
6. The value of
0.02
equals to
3 × 0. 91 ANSWERS
(a) 0.000016 (b) 0.00016 (a) 0.75 (b) 1.5 (c) 15 (d) 0.15
1 2 3 4 5 6 7 8 9 10
(c) 0.0016 (d) None of these
20. What should be subtracted from the 11 12 13 14 15 16 17 18 19 20
7. Which of the following sets of the fractions is in multiplication of 0.527 and 2.013 to get 1?
21 22 23 24 25 26 27 28 29 30
ascending order? (a) 0.939085 (b) 0.060851 (c) 1.91984 (d) 2.16085
6 7 5 11 5 6 7 11
(a) , , , (b) , , ,
8 9 6 13 6 8 9 13 21. The value of 0. 3467 + 0. 1333 is equal to
11 5 7 6 11 7 6 5
(c) , , ,
13 6 9 8
(d) , , ,
13 9 8 6
(a) 0.48
(c) 0. 48
(b) 0.4801
(d) 0.48 HINTS AND SOLUTIONS
8. When 0.232323…is converted into a fraction, 22. The greatest fraction out of
2 5 11
, ,
7
and is
then the result is 5 6 12 8 1. (a) x = 15.9273 − 11.0049 = 4.9224 =
. )2 − ( 0.4)2 ][( 05
[( 05 . )2 + ( 0.4)2 ] 10k = 10−4
1 2 23 23 . )2 + ( 0.4)2 ]
(a) (b) (c) (d) (a)
7
(b)
5
(c)
11
(d)
2 2. (d) 15.3900 [( 05 So, k = − 4
5 9 99 100 8 6 12 5 = ( 05
. )2 − ( 0.4)2 = ( 05
. − 0.4)( 05
. + 0.4) 6 8 7
0.2360 10. (c) 0.6 + 0.8 + 0.7 = + +
9. If 3. 245 × 10 = 0.0003245, then the value of k is
k
3 5.2900 [Rule 1] 9 9 9
23. Which of the following fractions is greater than = 0.1 × 0.9 = 0.09 21 7 1
(a) 4 (b) –4 (c) 3 (d) 5 4 + 0.0002 [Rule 2] = = =2
5 9 3 3
20.9162 0.004 × 0.0008 0.0000032
10. The value of ( 0.6 + 0.8 + 0.7 ) is and less than . 6. (b) = 11. (a) Here, 6.88 − 2.58
6 x = 20.9162 0.002 0.02
1 1 1 1
=  6 +
(a) 2 (b) 2 (c) 2 (d) 1 1 2 4 9 88   58 
(a) (b) (c) (d) = 0.00016  − 2 + 
8 9 3 9 2 3 5 10 3. (a) 1.75 × 1.24 = 2.17  99   99 
6 7 5
11. The value of ( 6. 88 − 2. 58) is ⇒ 1.75 × 124 = 217 7. (a) Here, = 0.75, = 0.7 ,  88 58 
24. Consider the following statements: 8 9 6 = ( 6 − 2) +  − 
(a) 4. 30 (b) 4.29 (c) 3.22 (d) 4. 38 111.744 11  99 99 
I.
1
cannot be written as a terminating decimal. 4. (a) Since, = 3.88 = 083
. , = 0846
.
28.8 88 − 58 30
12. The expression (11.98 × 11.98 + 11.98 × x + 0.02 22 13 =4 + =4 + = 4.30
2 111744 So, 0.75 < 0.7 < 083
. < 0846
. 99 99
× 0.02) will be a perfect square for x equal to II. can be written as a terminating decimal. ⇒ = 3.88 [Rule 6]
15 28800 12. (c) Given expression
(a) 0.02 (b) 0.2 (c) 0.04 (d) 0.4 
1 1117.44 11744
= = 3.88 [Rule 4] 8. (c) Given, 0.232323 …= 0. 23 = ( 1198
. )2 + ( 0.02)2 + 1198
. ×x
x+ y III. can be written as a terminating decimal. Now,
13. If 2. 5x = 0. 5 y, then the value of is 16 288 28800 (which is a recurring decimal) =
23
For the given expression to be a perfect
x− y 99 square, we must have
Which of the statement(s) given above is/are . )2 ] 2 − [( 0. 4)2 ] 2
[( 05 9. (b) Here, 10k =
0.0003245
(a) −1.3 (b) −1.5 (c) 1.5 (d) 1.3 5. (b) . × x = 2 × 1198
1198 . × 0.02
correct? 3.245
. )2 + ( 0. 4)2
( 05
14. 7.2 exceeds its one-tenth by (a) Only I (b) Only II 3.245 × 10−4 ⇒ x = 0.04
(a) 8.48 (b) 5.48 (c) 6.48 (d) 5.28 (c) I and III (d) II and III [( a 2 − b 2 ) = ( a + b ) ( a − b )] = [by using ( a + b )2 = a 2 + b 2 + 2ab]
3.245
05
MATHEMATICS Decimal Fractions 37 38 CDS Pathfinder

13. (b) 2.5x = 0.5 y 3 3 25. (c) Since, 1.16666… and 1.454545… are
19. (b) =
x .
05 3 + 0.3 − 3.03 3 − 273 recurring numbers and we know that,
⇒ = = 0. 20
y 2⋅5 3 × 0.91 3 × 91 recurring numbers represent rational
numbers.
Now, the expression is [dividing
3 3
numerator and denominator by y] = = = 1.5 Hence, statements I and IV are rational
x + y x / y + 1 0.20 + 1 120
. 3−1 2 numbers.
= = =
x− y x / y − 1 0.20 − 1 −080 . 26. (b) 0.007 = 0.007777777
20. (b) Let x should be subtracted
= −15
. ( 0.527 × 2.013) − x > 1 ⇒ 17.83 = 1783838383
.
Shortcut Method 1.060851 − x > 1 310.0202 = 310. 020222222
x 05 . x + y 05. + 25
.


=
y 2.5

x− y
=
. − 25
05 .

[using componendo and dividendo]


x+ y
=
3
= − 1.5
x = 0.060851
21. (b) 0.34 67 + 0.1333

=
3467 − 34 1333 − 13
+
= 3278663838
.
= 32786638
.
on adding

27. (b) Given, 0.242424... = 0. 24


SQUARE ROOTS
AND CUBE ROOTS
x − y −2 9900 9900
3433 + 1320 4753 [which is a recurring decimal number]
7.2 = = 24 8
14. (c) 7. 2 − 9900 9900 = =
10 99 33
4801 − 48
⇒ 7. 2 − 0. 72 = 6.48 = = 0. 4801
2341 − 2 2339
p 9900 28. (d) 0. 2341 = =
15. (d) Given, = 2.52 …(i) 9990 9990
2 5 11
q 22. (c) = 0.4, = 083
. , = 0.916
5 6 12 29. (c) Value p may be 3, 7, 11, 13.
Now, 100 multiply both sides, we get
7 1
100 p and = 0875. = 0.3 , Period = 1 Generally (1-3) questions have been asked from this chapter. This section will be useful in solving
= 252.52 …(ii) 8 3
q simplification questions and will save lots of time while doing fuzzy calculations.
On subtracting Eq. (i) from Eq. (ii), we
Clearly, the greatest fraction is 0.916 i.e. Here, p − 1 = 2 and 1 is a factor of 2.
11 1
get . = 0. 142857, Period = 6
99 p q 99 12 7
= 250 ⇒ = = 0.396
q p 250 3 5
23. (c) = 0.75, = 0833
. Here, p − 1 = 6 and 6 is a factor of 6.
4 6
16. (c) Required decimal fraction =
1
60 × 60 1 2 4 9
1
= 0. 09, Period = 2 SQUARE
= 05
. , = 0.66, = 08
. and = 0.9 11
1 2 3 5 10 The square of a number is obtained by multiplying the number by itself.
= = 0.00027 Here, p − 1 = 10 and 2 is a factor of 10.
3600 Clearly, 0.8 lies between 0.75 and  
e.g. Square of 5 = 5 × 5 = 25, square of 6 = 6 × 6 = 36
3 5 3 × 2. 24 0.8333. 30. (c) We have, 0. 63 + 0.37 Perfect square A natural number n is called a perfect square or a square number, if there exists a
17. (a) =
2 5 − 0.48 2 × 2. 24 − 0.48 4 3 5 natural number m such that n = m2 .
∴ lies between and . Let x = 0.63 6363 63 …
5 4 6 e.g. The numbers 1, 4, 9, 16, 25, ... are perfect square.
[Q 5 = 2.24] 100x = 63.636363 …, 99x = 63
1 2
=
6.72
=
6.72
= 168
.
24. (c) In
22
and
15
, 22 and 15 are not in
and x =
63 Properties of Perfect Square
4. 48 − 0. 48 4 1 99
the form of 2 × 5n but in .
m
 The properties of perfect squares are discussed below.
16 Similarly, y = 0.37 = 0373737
. (i) A number having 2, 3, 7 or 8 at unit’s place is never a perfect square.
 ( 0.1) − ( 0.01)
2 2

18. (d) Here, + 1
  16 in the form of 24 × 50 . So,
1 37 e.g. 172, 2783
 0.0001  can be Q y=
16 (ii) The squares of odd numbers are odd and the squares of even numbers are even.
0.01 − 0.0001 99
+ 1 =  + 1
0.0099
= written as a terminating decimal. (iii) The difference of squares of two consecutive natural numbers is equal to their sum.
0.0001  0.0001  ∴ x+ y =
63 37 100
+ =
= ( 99 + 1) = 100 Hence, statements I and III are correct. 99 99 99 e.g. 7 2 − 6 2 = 7 + 6 = 13
(iv) The product of four consecutive natural numbers plus one is a always square.
(v) A number ending in an odd number of zeroes is never a perfect square.
Square Root
The square root of a number x is that number which when multiplied by itself gives x as the product.
It is denoted by .
In general, y 2 = x ⇒ y = x
Here, y is the square root of x, if and only if x is the square of y.
e.g. 4 = 2× 2 = 2
Thus, 2 is the square root of 4.
MATHEMATICS Square Roots and Cube Roots 39 40 CDS Pathfinder

x 4 Step III Put the quotient above the period and write the
EXAMPLE 1. If = , then the value of x is
64 8 product of divisor and quotient just below the CUBE POWERS OR EXPONENTS
first period. If a number is multiplied three times with itself, then the An expression that represents repeated multiplication of
a. 16 b. 12 c. 8 d. 4
Step IV Subtract the product of divisor and quotient from result of this multiplication is called the cube of that number. the same factor is called a power. It is written as ‘x n ’
x 4 and is read as ‘x’ raised to the power n.
Sol. a. We have, = , the first period and bring down the next period to e.g. 8 = 2 × 2 × 2 = 23
64 8 the right of the remainder. This becomes the new Here, x is called the ‘base’.
On squaring both sides, we get
2
dividend. Properties of Cube and n is called the ‘exponent’.
 x  4
2
x 16 Step V Double the quotient and enter it with a blank on
  =   ⇒ = ⇒ x = 16 1. Cubes of all even natural numbers are even and all e.g. 65 = 6 × 6 × 6 × 6 × 6 = 7776.
 64  8 64 64 its right. odd natural numbers are odd.
Step VI Guess a largest possible digit to fill the blank 2. Cubes of the numbers ending in digit 0, 1, 4, 5, 6 and Laws of Exponents
Methods to Find Square Root which will also become the next digit in the 9 are the numbers ending in the same digit. x
quotient, such that when the new divisor is Let be any rational number and m, n be any integers.
The square root of a given number can be determined 3. Cubes of negative integers are negative. y
by using any of the following two methods multiplied to the new quotient the product is less
than or equal to the dividend. 4. Cubes of numbers ending in digits 3 and 7 ends in Then,
1. Prime Factorization Method m n m+ n
Step VII Repeat the above steps till all the periods have digit 7 and 3, respectively.  x  x  x
2. General Method/Division Method (i)   ×   =  
been taken up.  y  y  y
Prime Factorization Now, the quotient so obtained is the required CUBE ROOT m n m− n n
 x  m mn
 x  x  x  x
In this method, we express the given number as the square root of the given number. The cube root of a given number is the number whose (ii)   ÷   =  (iii)    =  
product of prime factors. Now, for finding square root, cube is the given number.  y  y  y  y    y
we take the product of these prime factors choosing one EXAMPLE 4. The square root of 1522756 is In general, we can say that a 3 = b, then cube root of ‘b’ −n n 0
 x  y  x
out of every pair of same prime factors. a. 1182 b. 1222 is ‘a’. Cube root of a number ‘a’ is denoted as 3 a. (iv)   =  (v)   =1
c. 1234 d. 1334  y  x  y
EXAMPLE 2. The square root of 213444 is e.g. 3 64 = 4 × 4 × 4 = 4. Thus, 4 is the cube root of 64.
Sol. c
a. 332 b. 368 c. 432 d. 462 EXAMPLE 8. If 2m + 21 + m = 24, then the value of m is
1234 x 5
Sol. d. The prime factorisation of 213444 is
1 1 52 27 56 EXAMPLE 6. If 3 = , then the value of x is a. 1 b. 2 c. 3 d. 5
1 27 3
213444 = 2 × 2 × 3 × 3 × 7 × 7 × 11 × 11
22 52 a. 125 b.25 c. 27 d. 9 Sol. c. Given, 2m + 21 + m = 24 ⇒ 2m + 21 × 2m = 24
∴ 213444 = 2 × 2 × 3 × 3 × 7 × 7 × 11× 11
×2 44 x 5 ⇒ 2m(1 + 2) = 24 ⇒ 2m × 3 = 24 ⇒ 2m = 8 = 23
Now, taking one number from each pair and multiplying Sol. a. We have 3 =
them, we get 213444 = 2 × 3 × 7 × 11 = 462 243 0827 27 3 Now, on comparing both sides, we get
×3 729 On cubing both sides, we get ∴ m=3
EXAMPLE 3. What is the square root of 2464 9856  x
3
 5
3
x 125 Hence, the value of m is 3.
0. 324 × 0.64 × 129.6 3  =   ⇒ =
? ×4 9856  27  3 27 27
0 . 729 × 1. 024 × 36 × SURDS OR RADICALS
∴ x = 125
a. 4 b. 3 Hence, the square root of 1522756 is 1234. Let x be a rational number and n be a positive integer,
c. 2 d. 1 Method to Find Cube Root such that n x is irrational, then n x is called a radical or
EXAMPLE 5. The least number of four digits which
0.324 × 0.64 × 129.6 324 × 64 × 1296 Method to calculate the cube root of a number is as follow surd of order n and here x is called the radicand.
Sol. d. = is a perfect square is e 2012 II
• A surd of order 2 is called a quadratic surd i.e. 2, 3.
0.729 × 1024
. × 36 729 × 1024 × 36 a. 1204 b. 1024 Prime Factorization Method • A surd of order 3 is called a cubic surd. i.e. 3 2, 3 3.
( 22 × 34) × ( 26) × ( 24 × 34) ( 2 × 32) × 23 × ( 22 × 32) c. 1402 d. 1420
In this method, we express the given number as the
= = • A surd of order 4 is called a biquadratic surd i.e. 4 5 , 4 7 .
( 36) × ( 210) × ( 22 × 32) 33 × 25 × ( 2 × 3) Sol. b. Here, the greatest three-digit number = 999 product of prime factors.
18 × 8 × 36 Now, for finding cube root, we take the product of Laws of Radicals
= =1 31
27 × 32 × 6 these prime factors choosing one out of every three,
3 9 99 The laws of exponents which are applicable to the surds
from the factors. are
Division Method ×3 9
To find the square root of a given number using this 61 99 EXAMPLE 7. Find the cube root of 373248. (i) ( n x ) n = x
method, the following steps are to be followed. 61 a. 42 b. 52 c. 62 d. 72 (ii) n xy = n x ⋅ n y
Step I In the given number, place bars over every pair of 38 Sol. d. Here, 373248 = 8 × 8 × 8 × 9 × 9 × 9 x n
x
digits starting with the unit’s digit. Each pair and Here, the greatest number of three digits which is (iii) n =
the remaining one digit (if any) on the extreme left 373248 = 2 × 2 × 2 × 2 × 2 × 2 y n y
perfect square = 999 − 38 = 961 = ( 31) 2
is called a period. × 2× 2× 2× 3× 3× 3× 3× 3× 3
So, the smallest four-digit number which is perfect (iv) ( n x ) m = ( n x m )
Step II Think of the largest number whose square is less ⇒ 373248 = 2 × 2 × 2 × 3 × 3 = 72
3
square = ( 31+ 1) 2 = ( 32) 2 = 1024
than or equal to the first period. Hence, the cube root of 373248 is 72. (v) m n
x = mn x = n m x
MATHEMATICS Square Roots and Cube Roots 41 42 CDS Pathfinder

Types of Surds
1. Pure surds A surd which has only 1 as a rational
Multiplication and Division of Surds
p
Suppose given surds are a, b and r c , then first find
the LCM of p, q and r to make the radial powers of the
q PRACTICE EXERCISE
factor is called a pure surd.
e.g. 2, 3 2, 4 3 all are pure surds. surds same and then their multiplication and division
can be done using following rules. x 4 14. Find the value of
2. Mixed surds A surd which is not a pure surd or has 1. If = , then the value of x is
n
x x 49 7
factor other than unity is called a mixed surd. (i) n x × n y = n xy (ii) =n 343 + 307 + 273 + 241 + 225
(a) 9 (b) 25 (c) 16 (d) 8
4 n y y
e.g. 2 3, 5 3 12, 3 35 are mixed surds. (a) 18 (b) 19 (c) 19 (d) 18 + 3
3 2. Which of the following cannot be a digit in the
1 1 1 1
3. Like and Unlike surds When the radicands of two EXAMPLE 11. Simplify 6 12 ÷ 3 ⋅ 3 2. unit place of a perfect square? 15. What is − + −
(a) 1 (b) 5 (c) 7 (d) 0 9− 8 8− 7 7− 6 6− 5
surds are same, then those are known as like surds. a. 3 3 b. 3
1
e.g. 6 3 and 22 ⋅ 3. 1 3. The number 0.0001 is + equal to?
c. 3 2 d. 3 5− 4
When radicands of two surds are different, then they 3 (a) a rational number less than 0.01
1
are called unlike surds. e.g. 4 7 and 6 5. Sol. d. Here, the order of 3, 3
2 is 2 and 3, respectively. (b) a rational number (a) 0 (b) 1 (c) 5 (d)
3
(c) an irrational number
So, LCM 2, 3 and 6 = 6
Comparison of Two Surds (d) Neither a rational number nor an irrational number 16. What is one of the square roots of 9 − 2 14?
3 = 31/ 2 = ( 36/ 2)1/ 6 = ( 33)1/ 6 = ( 27)1/ 6 = 6 27 (a) 7 − (b) 6 − (c) 7 − (d) 7 − 2
1. If two surds are of the same order, then the one 4. If m and n are natural numbers, then m n is 3 3 5
whose radicand is larger, is the larger of the two.
3
2 = ( 2) 1/ 3
= (2 6 / 3 1/ 6
) = (2 )2 1/ 6
= 4
6
(a) always irrational 17. If px = r y = m and r w = pz = n , then which one of
e.g. 17 > 13, 3 21 > 3 16 6
12 6
12 6
12 12 1 (b) irrational unless n is the mth power of an integer
the following is correct?
So, = = =6 =6
2. If any two surds have different orders are to be 3⋅ 3 2 6
27 ⋅ 6 4 6 27 × 4 27 × 4 9 (c) irrational unless m is the nth power of an integer
(a) x w = yz (b) x z = yw
(d) irrational unless m and n are coprime
compared, then we will first reduce them to the same 2 1/ 6 (c) x + y = w + z (d) x − y = w − z
 1  2/ 6 1/ 3
=    =  = 
1 1 1
but smallest order and then compare them. =3 5. What is that fraction which when multiplied by
 
 3   3  3 3 18. If a x = b y = cz and abc = 1, then what is
itself gives 227.798649?
Note Let n x be a surd of order n. (a) 15.093 (b) 15.099 (c) 14.093 (d) 9.0019 xy + yz + zx equal to
m
Then, n x = x m/ n i.e. x 1/ n = ( x m/ n )1/ m is a surd of order m. Rationalisation of Surds (a) xyz (b) x + y + z (c) 0 (d) 1
6. If x = 3018 + 36 + 169 , then the value of x is 5+1 5−1
Process of converting surd into rational number is called 19. If a = and b = , then the value of
EXAMPLE 9. Convert 2 into a surd of order 4.
as rationalisation of surds. (a) 55 (b) 44 (c) 63 (d) 42 5−1 5+1
a. 4 2 b. 4 8 c. 4 4 d. None of these  a 2 + ab + b2 
When the product of two surds is a rational number, then 7. If 3 x = x / 5, then x is equal to  2  is
Sol. c. 2=2 1/ 2
= (2 4 / 2 1/ 4
) = (2 )
2 1/ 4
= 4
4
each surd is called a rationalising factor of each other. (a) 5 5
6
(b) 55
5
(c) 56 (d) 5  a − ab + b2 
EXAMPLE 10. Consider the following in respect of e.g. 3
25 × 3 5 = 3 25 × 5 = 3 125 = 3 5 3 = 5 8. What is the value of (a) 3/4 (b) 4/3 (c) 3/5 (d) 5/3
the numbers 2, 3 3, and 6 6 . e 2014 (I)
So, 3 25 and 3 5 are rationalising factor of each other. 20. If 3 5 + 125 = 17.88, then what will be the value
29.16 + 0.2916 + 0.002916 + 0. 00002916 ?
I. 6 6 is the greatest number.
(a) 5.9949 (b) 5.9894 (c) 5.9984 (d) 5.9994 of 80 + 6 5
II. 2 is the smallest number. Rationalising factors of a given surd − 3/ 2 (a) 13.41 (b) 20.46 (c) 21.66 (d) 22.35
Which of these statements is/are correct? 1 9. ( 5 )−5/ 2 × ( 5)
• Rationalising factor of = a 21. If N = 20.15 and N b = 16, then b is equal to
a. Only I b. Only II c. Both I and II d. Neither I nor II a 1 1 1
(a) (b) (c) (d) None of these
625 25 125 (a) 80 / 3 (b) 5 / 3 (c) 4 (d) 3 / 5
Sol. d. Taking LCM of 2, 3 and 6 = 12 1
• Rationalising factor of = am b 10. If a = 3, b = 9 and c = 10, then the value of
Now, 2 = 21/ 2 = 26/12 = 12 26 = 12 64 a± b 22. If 6A = 2, 6B = 5 and 6Q = 15, then Q equals to
1 2 13 + a + 112 + b + c − 1 is (a) B + 3 (b) 5A + B (c) B2 − 2 B (d) B − A + 1
6 = (6) 6 = (6)12 = 12 62 = 12 36
6 5+ 3 5− 3
EXAMPLE 12. + is equal to (a) 15 (b) 18 (c) 16 (d) 10 4 6 12
23. Arrange 3 , 10 , 25 in descending order.
3
3 = 31/ 3 = 34/12 = 12 81 5− 3 5+ 3
So, neither I nor II is correct. 11. If 0. 9 × 0.09 × x = 0. 9 × 0.09 × z, then the value (a) 6 10 > 4 3 > 12 25 (b) 12 25 > 4 3 > 6 10
a. 16 b. 8 c. 4 d. 15
x (c) 6 10 > 12 25 > 4 3 (d) 4 3 > 12 25 > 6 10
of is
Operations on Surds Sol. b.
5+ 3
+
5− 3 z
5− 3 5+ 3 24. If ( 3.7)x = ( 0.037) y = 10000, then what is the value
(a) 0.081 (b) 0.810 (c) 0.81 (d) 8.09
Addition and Subtraction of Surds ( 5 + 3)( 5 + 3) ( 5 − 3) ( 5 − 3) of
1 1
− ?
= + × x y
Addition or subtraction takes place only between like ( 5 − 3)( 5 + 3) ( 5 + 3) ( 5 − 3) 12. 2 2 2 2 2 is equal to
1 1
surds, using laws of numbers. e.g. ( 5 + 3) 2 ( 5 − 3) 2 ( 5 + 3) 2 + ( 5 − 3) 2 (a) 0 (b) 2 (c) 1 (d) 2 31/ 32
(a) 1 (b) 2 (c) (d)
(i) 2 3 + 4 3 + 5 = 6 3 + 5 = + = 2 4
5 − 3
2 2
5 − 3
2 2 ( 5) 2 − ( 3) 2 7+ 5
(ii) 6 5 − 3 2 + 7 5 + 4 2 = (6 5 + 7 5 ) + ( 4 2 − 3 2 ) 13. If 35 = 5. 9160, then the value of is 25. The greatest six digit number which is a perfect
2[( 5) 2 + ( 3) 2 ] 2( 5 + 3) 7− 5 square is
= = =8
= 13 5 + 2 5− 3 2 (a) 9.1060 (b) 10.9160 (c) 11.9160 (d) 12 (a) 998004 (b) 998006 (c) 998049 (d) 998001
MATHEMATICS Square Roots and Cube Roots 43 44 CDS Pathfinder

26. What is the smallest number by which 26244 37. If 16 × 8n + 2 = 2m , then m is equal to e 2013 I ANSWERS
must be divided to get a perfect cube? (a) n + 8 (b) 2 n + 10
(a) 4 (b) 6 (c) 36 (d) 16 (c) 3 n + 2 (d) 3 n + 10 1 2 3 4 5 6 7 8 9 10
27. What is the smallest number that must be added 2 11 12 13 14 15 16 17 18 19 20
to 1780 to make it a perfect square?
38. When a ball bounces, it rises to of the height
3 21 22 23 24 25 26 27 28 29 30
(a) 39 (b) 49 (c) 59 (d) 69 from which it fell. If the ball is dropped from a 31 32 33 34 35 36 37 38 39 40
28. A gardener plants 17956 trees in such a way height of 36 m, how high will it rise at the third
41 42 43 44 45 46 47 48
that there are as many rows as there are trees in bounce? e 2013 I
a row. The number of trees in a row are 1 2 1 2
(a) 10 m (b) 10 m (c) 10 m (d) 12 m
(a) 136 (b) 164 (c) 134 (d) 166 3 3 3 3
29. A group of student decided to collect as many
paise from each member of the group as is the
39. The product of four consecutive natural numbers
plus one is e 2014 I
HINTS AND SOLUTIONS
number of members. If the total collection (a) a non-square
amounts to ` 32.49 the number of members in the (b) always sum of two square numbers x 4 x 4 4
1. (c) Given, = or = = ( 5 )− 4 =  
1 = 343 + 307 + 17
group, is (c) a square 49 7 7 7  5
(a) 37 (b) 47 (c) 57 (d) 27 (d) None of the above = 343 + 18 = 361 = 19
Now, on squaring both sides, we get (1 / 2 ) × 4 2
=   =   =
1 1 1 1 1 1
30. A general arranges his soldiers in rows to form a 15. (c) − +
40. The difference of two consecutive cubes e 2014 I ( x )2 = ( 4)2 ⇒ x = 16 5 5 25 9− 8 8− 7 7− 6
perfect square. He find that in doing, so (a) is odd or even (b) is never divisible by 2 Hence, the value of x is 16. 1 1
10. (b) Given, a = 3, b = 9 and c = 10 − +
60 soldiers are left out. If the total number of (c) is always even (d) None of these 6− 5 5− 4
soldiers be 8160. Then, the number of soldiers in 2. (c) The digits 2, 3, 7 and 8 cannot be at ∴ 13 + a + 112 + b + c−1
( 0.75) 3 the unit place of a perfect square =( 9 + 8) − ( 8 + 7)
each row is 41. The square root of + [0.75 + ( 0.75)2 + 1] is = 13 + 3 + 112 + 9 + 10 − 1
number. +( 7+ 6) − ( 6 + 5)
(a) 90 (b) 91 (c) 92 (d) 80 1 − 0.75 = 16 + 121 + 9 = 4 + 11 + 3 = 18
e 2015 I 3. (b) 0.0001 = 0.01 which is a rational +( 5+ 4)
31. A ball is dropped from a height 64 m above the number. x 0.9 × 0.09
(a) 1 (b) 2 (c) 3 (d) 4 11. (a) Here, = [on rationalisation]
ground and every time it hits the ground it rises z 0.9 × 0.09
to a height equal to half of the previous. What is 5 + 10 4. (b) If m and n are natural numbers, then = 9+ 4 =3+ 2=5
42. What is equal to? m n is irrational unless n is mth power of 0.9 × 0.09 × 0.9 × 0.09
the height attained after it hits the ground for =
5 5 − 2 20 − 32 + 50 an integer. 0.9 × 0.09 16. (d) 9 − 2 14 = 7+ 2−2× 7 × 2
the 16th time?
e 2015 I = 0.9 × 0.09 = ( 7− 2 )2 = 7− 2
(a) 2 −12 m (b) 2 −11 m (c) 2 −10 m (d) 2 −9 m 5. (a) Let fraction be x, then
(a) 5 (b) 5 2 (c) 5 5 (d) 5 2
⇒ 
x2 = 227.798649 x
 = ( 0.9 × 0.09 )
2
a + 3b + a − 3b 91 1  z 17. (a) Given, p = r = m and
x y
32. If x = , then 3bx 2 − 2ax + 3b is 43. If x = , then the value of 3 − is ⇒ x= 227.798649 = 15.093
a + 3b − a − 3b rw = pz = n
216 (1 −x )1/ 3 e 2015 II ⇒
x
= 0.081
equal to (given that, b ≠ 0). 6. (a) Given, x = 3018 + 36 + 169 z Now, px = r y
9 5 4 4
(a) (b) (c) (d) On multiply power with w on both
(a) 1 (b) 0 (c) ab (d) 2ab 5 9 9 5 = 3018 + 36 + 13
12. (d) Here, 2 2 2 2 2 sides, we get
44. Which one of the following is correct? e 2015 II = 3018 + 49 = 3018 + 7 ( p x )w = ( r y )w ⇒ p xw = r yw
PREVIOUS YEARS’ QUESTIONS (a) 2 < 4 6 < 3 4 (b) 2 > 4 6 > 3
4 = 2 2 2 2 × 21 / 2 = 2 2 2 × 23 / 4 ⇒ px w = ( rw ) y ...(i)
(c) 4 6 < 2 < 3 4 (d) 4 6 > 2 > 3
4 = 3025 = 55
33. If a x = b, b y = c and xyz = 1, then what is the 1 1 Hence, the value of x is 55.
On putting r w = p z in Eq. (i), we get
45. If x = 3 + 2, then the value of x + x + + 3 , is 3 = 2 2 × 27 / 8 = 2 × 215 / 16 = 231 / 32
value of cz ? e 2012 I p xw = ( p z ) y ⇒ p x w = p zy
x x 7. (c) 3 x = x /5 7+ 5
(a) a (b) b (c) ab (d) a/b 13. (c) Here, On comparing both sides, we get
e 2015 II On cubing both sides, we get 7− 5
34. If 196 x 4 = x 6, then x 3 is equal to which one of ∴ xw = zy
(a) 10 3 (b) 20 3 (c) 10 2 (d) 20 2 x = x 3 / 53 7+ 5 7+ 5 ( 7+ 5 )2
the following? e 2012 I − 1/ 3 On squaring both sides, we get = × = 18. (c) Given, a x = b y = c z = k [Let]
46. If x = 2 1/ 3
+2 , then the value of 2x − 6x − 5 is
3
7− 5 7+ 5 7−5
x6 x2 x6 ⇒ a = k1 / x , b = k1 / y
(a) (b) 14x4 (c) (d) 14x2 equal to e 2016 I x = x 6 / 56 ⇒ 56 = = x5
14 14 x 7 + 5 + 2 35 12 + 2 35 and c=k 1/z
(a) 0 (b) 1 (c) 2 (d) 3 = = 1 1 1
⇒ x 5 = 56 ⇒ x = 5 56 + +
35. If 10 + 3 x = 4, then what is the value of x? 47. If 4x 2 y = 128 and 33 x 32 y − 9xy = 0, then the value
2 2 ∴ abc = k x y z

e 2012 I 8. (d) 29.16 + 0.2916 + 0.002916 [Q ( a + b )2 = a 2 + b 2 + 2ab ]


of x + y can be equal to
1 1 1
e 2016 I + +
(a) 150 (b) 216 (c) 316 (d) 450 + 0.00002916 = 6 + 35 = 6 + 5.9160 = 119160
. ⇒ 1= kx y z = k0
(a) 7 (b) 5 (c) 3 (d) 1
2 2 = 5.4 + 054
. + 0.054 + 0.0054 [Q abc = 1, given]
36. The expression [( 2 ) ] gives e 2013 I a + 2b + a − 2b 14. (b) 343 + 307 + 273 + 241 + 225
48. If x = , then bx 2 − ax + b is = 5.9994 On comparing both sides, we get
(a) a natural number
a + 2b − a − 2b 1 1 1
(b) an integer and not a natural number 9. (b) ( 5 )− 5 / 2 × ( 5 )− 3 / 2 = 343 + 307 + 273 + 241 + 15 + + =0
equal to (given that b ≠ 0) e 2016 I x y z
(c) a rational number but not an integer = ( 5 )− 5 / 2 − 3 / 2 = ( 5 )− 8 / 2
= 343 + 307 + 273 + 16
(d) a real number but not a rational number (a) 0 (b) 1 (c) ab (d) 2ab ⇒ xy + yz + zx = 0
MATHEMATICS Square Roots and Cube Roots 45 46 CDS Pathfinder

5+1 5+1 ⇒
4 4
− =3 − 1⇒
1 1 1
− = a + 3b + a − 3b 38. (b) After first bounce, height of ball 5 + 10 1
+ 3 x +  = 24 3
x3 +
1
19. (b) a = × 32. (b) Given, x = =
5−1 5+1 x y x y 2 a + 3b − a − 3b 1
5 5−4 5−4 2+5 2 x3  x
=   × 36
2
1 1 1 3
Hence, the value of − is ⋅ (5 + 10 ) ( 5 − 2 ) Q x3 + 3 + x + = 24 3 − 2  x + 
( 5 + 1)2 3 + 5 By rationalising, we get 1 1 1
= = = × 
5−1 2
x y 2 ( a + 3b )2 + ( a − 3b )2 ( 5 + 2) ( 5 − 2) x x x
x= and after third bounce, height of ball
5−1 5−1 25. (d) Here, the greatest six-digit number ( a + 3b )2 − ( a − 3b )2 3 = 24 3 − 2 × 2 3
=   × 36
b= × 2 [by rationalisation]
= 999999 = 24 3 − 4 3 = 20 3
5+1 5−1 2 ( a + 3b )( a − 3b ) 3 5 5 − 5 2 + 50 − 20
999 + =
( 5 − 1)2 3 − 5 ( a + 3b )2 − ( a − 3b )2 8 8 × 4 32 2 46. (a) Given, x = 21 / 3 + 2−1 / 3
= = 9 99 99 99 = × 36 = = m = 10 m 52 − 22
5−1 2 a + 3b + a − 3b + 2 a 2 − 9b 2 27 3 3 3 ∴ 2x3 − 6x − 5 = 2 ( 21 / 3 + 2− 1 / 3 )3
81 x= [Q ( a − b )( a + b ) = a 2 − b 2 ]
− 6 ( 21 / 3 + 2−1 / 3 ) − 5
2 2
3 + 5  3 − 5  189 1899 a + 3b − a + 3b Hence, the required height at third
5 5 −5 2 + 5 2 − 2 5 3 5
∴ a2 + b2 =   +  2 = = = 5
 2   2  ×9 1701 2a + 2 a 2 − 9b 2 bounce is 10 m. = 2[ 2 + 2−1 + 3 ( 21 / 3 + 2−1 / 3 )]
x= 3 5−2 3
2( 9 + 5) 1989 19899 a + 3b − a + 3b − 6 ( 21 / 3 + 2−1 / 3 ) − 5
= =7 39. (c) Product of four consecutive numbers 43. (a) We have, x =
91
1
×9 17901
4 (a + a 2 − 9b 2 ) plus one is always a square. 216 =4+ 2× + 6 ( 21 / 3 + 2−1 / 3 )
5+1 5−1 1998 = 2
3
Also, ab = × =1 (3 b ) Illustration 1 Let four consecutive 91 125  5  − 6 ( 21 / 3 + 2−1 / 3 ) − 5
5−1 5+1 Here, the greatest number of six digit ⇒ 1− x = 1− = = 
which is perfect square 3bx2 − 2ax + 3b = 0
numbers be 3, 4, 5 and 6. 216 216  6  = 4 + 1− 5 = 0
a + ab + b
2 2
( a + b 2 ) + ab
2
∴ (3 × 4 × 5 × 6) + 1 = 361 = ( 19)2
∴ 2 = = 999999 − 1998 = 998001
1
5 1 6 47. (b) We have, 4x 2 y = 128
a − ab + b 2 ( a 2 + b 2 ) − ab 33. (a) Given, a x = b , b y = c and x yz = 1 ⇒ ( 1 − x) 3 = ⇒ =
Illustration 2 Let four consecutive 6 1
5
7+ 1 4 26. (c) We have, Now, a x = b numbers be 9, 10, 11 and 12. (1 − x) 3 ⇒ 2 2 x 2 y = ( 2)7
= =
7−1 3 26244 = 2 × 2 × 3 × 3 × 3 × 3 × 3 × 3 × 3 × 3 On multiplying both sides by y in ∴ ( 9 × 10 × 11 × 12) + 1= 11881= ( 109)2 ⇒ 2 2 x + y = ( 2)7
1 6 9
So, to get a perfect cube, power, we get ⇒ 3− =3− = ⇒ 2x + y = 7
20. (d) 3 5 + 125 = 1788
. 40. (b) The difference of two consecutive 1
5 5
26244 must be divided by 2 × 2 × 3 × 3 ( a ) = ( b)
x y y
⇒ a xy
=b y
cubes is never divisible by 2. (1 − x) 3 ⇒ y = 7 − 2x …(i)
⇒ 3 5 + 5 5 = 1788
. i.e., 36.
Here, b y = c ⇒ ax y = c Illustration 1 Let the two consecutive Now, 33 x ⋅ 32 y = 9xy
⇒ 8 5 = 1788
. 44. (b) We have, 2, 61 / 4 , 41 / 3
27. (d) We know that, ( 42)2 = 1764 and numbers be 4 and 5.
1788
. Again, multiply with z in power, we get ⇒ 33 x+2y = 32 xy
⇒ 5= = 2.235 ( 43)2 = 1849 The LCM of 2, 4, 3 are 12.
8 ( a x y )z = c z ⇒ a x y z = cz ∴ (5)3 − ( 4)3 = 125 − 64 = 61 1 1 1 ⇒ 3x + 2 y = 2xy …(ii)
Since, 1764 < 1780 < 1849
∴ 80 + 6 5 = 4 5 + 6 5 Given, xyz = 1 ⇒ a = c z Illustration 2 Let the two consecutive ( 26 ) 12 ; ( 63 ) 12 ; ( 44 ) 12 ⇒ 3x + 2 ( 7 − 2x) = 2x ( 7 − 2x)
Hence, the smallest number that must be
= 10 5 = 10 × 2.235 = 22.35 numbers be 9 and 10.
added to 1780 is ( 1849 − 1780), i.e. 69. ∴ c =az 1 1 1
[from Eq. (i)]
21. (a) N = 20.15 ∴ ( 10)3 − ( 9)3 = 1000 − 729 = 271 ( 64) 12 ; ( 216) 12 ; ( 256) 12
28. (c) Given, total number of tress = 17956 Hence, the value of c z is a. ⇒ 3x + 14 − 4x = 14x − 4x2
⇒ N = ( 2) 3 / 20
⇒ ( N ) = ( 2)
b 3 b / 20
∴ Number of trees in each row x 6 3
( 0.75) 1 1
⇒ 4x − 15x + 14 = 0
2
34. (d) 196x4 = x6 , 196 = = x2 41. (b) + [ 0.75 + ( 0.75)2 + 1] 2<6 4
<4 3
or 2> 4
6> 3
4
But, N b = 16 = 17956 = 134 x4 1 − 0.75 ⇒ 4x2 − 8x − 7x + 14 = 0
∴ 16 = ( 2) ⇒ 2 = ( 2)
3 b / 20 4 3 b / 20
29. (c) Total rupees collected = ` 32.49 ⇒ x= 196 = 14 3
( 0.75) ⇒ 4x ( x − 2) − 7 ( x − 2) = 0
= + [ 175
. + ( 0.75)2 ] 45. (b) We have, x = 3 + 2
3b 80 = 32.49 × 100 paise = 3249 paise ⇒ x3 = 14x2
⇒ 4= ⇒ b= 0.25 1 1 3− 2 ⇒ ( 4x − 7) ( x − 2) = 0
20 3 Q = ×
∴ Number of member in the group 35. (b) Given, 10 + x = 4 3
= 3 × ( 0.75)2 + 175
. + ( 0.75)2 3+ 2 3− 2 7
x ⇒ x = or x = 2
22. (d) Since 15 = 3 × 5 = 3 × 6B = 3249 = 57 4
On squaring both sides, we get 3− 2
6 61 = 4 × ( 0.75)2 + 175
. = 4=2 = = 3− 2 7
As, 3 = = A = 61 − A 30. (a) Here, number of soldier arranged 10 + 3 x = 16 3−2 ∴ y = or y = 3
2 6 Hence, the required square root is 2. 1 2
= 8160 − 60 = 8100 ⇒ 3 x = 16 − 10 = 6 ∴ x+ =2 3
∴ 15 = 3 × 6B = 61 − A × 6B 5 + 10 x ∴ x + y=
21
or x + y = 5
As number of soldier in each row is Now, on cubing both sides, we get 42. (d) 4
⇒ 6Q = 61 − A × 6B ⇒ 6Q = 61 − A + B equal to number of row. 5 5 − 2 20 − 32 + 50 On cubing both sides, we get
x = ( 6)3 = 216 48. (a) Refer to question 32.
⇒ Q = 1− A + B So, number of soldier in each row Hence, the value of x is 216.
23. (a) LCM of 4, 6, 12 = 12 = 8100 = 90 36. (d) Expression = [( 2 ) 2
] 2
4
3 = 12 33 = 12 27 1
31. (c) After Ist hit, height of the ball 2
6
10 = 12 102 = 12 100 1 = ( 2 )(2 ) 2
= ( 2 )(2 ) 2

= ( 64)
25 = 12 25
12 1  1 − 1
2 1
×2 2 
 2


= ( 2) 2 = ( 2)(2 )
Clearly, 12 100 > 12 27 > 12 25 After IInd hit, height of the ball
Thus, 6 10 > 4 3 > 12 25 2 which denotes a real number but not a
=   ( 64)
1
rational number.
24. (c) Given, (3.7)x = ( 0.037) y = 10000  2
37. (d) Given, 16 × 8n + 2
= 2m
⇒ (3.7)x = 104 and ( 0.037) y = 104 After 16th hit, height of the ball
⇒ ( 2)4 × 2 3 (n + 2)
= 2m
37 = 104 / x + 1 and 37 = 104 / y + 3
16

=   ( 64)
1
 2 ⇒ 2(4 + 3 n + 6)
= 2m ⇒ 2(3 n + 10 ) = 2m
⇒ 104 / x + 1 = 104 / y + 3
On comparing, we get
1
On comparing both sides, we get = 16 ( 26 ) = 2−10 m 3n + 10 = m
4 4 2
+ 1= + 3 ⇒ m = 3n + 10
x y
06
MATHEMATICS Area and Perimeter of Plane Figures 47 48 CDS Pathfinder

(ii) To convert speed of an object from m/s to km/h,


18 Important Rules and Formulae
multiply it by .
5 Rule 1 If A travels with speed x km/h for t 1 h and with
18 x t 1 + yt 2
e.g. 5 m/s = 5 × km/h = 18 km/h speed y km/h for t 2 h, then average speed =
5 t1 + t 2
Average Speed EXAMPLE 4. A man walks at the rate of 5 km/h for
The average speed of an object is defined as total 6 h and at 4 km/h for 12 h. Find out the average
distance travelled divided by total time taken. speed (in km/h) of the man.

TIME AND DISTANCE i.e. Average speed =


Total distance
Total time
EXAMPLE 2. A man completes 30 km of a journey
a. 2
1
3
b. 5
1
3
c. 7
1
3
d. None of these

Sol. d. Here, x = 5 km/h, y = 4 km/h, t1 = 6 h and t 2 = 12 h


at 6 km/h and the remaining 40 km of the journey in xt1 + yt 2 5 × 6 + 4 × 12 30 + 48
∴ Average speed = = =
5 h. Then, his average speed for the whole journey is t1 + t 2 6 + 12 18
a. 5 km/h b. 7 km/h 78 1
= = 4 km/h
c. 7.5 km/h d. None of these 18 3
Total distance travelled 70
Regularly (4-7) questions have been asked from this chapter. Generally the type of questions vary from Sol. b. Average speed = = = 7 km/h Rule 2 If a man/vehicle covers two equal distances with
Total time taken 10
simple to quite complex. But if you know the basics, then this section becomes easy to score area. the speed of x km/h and y km/h respectively, then the
Relative Speed average speed of the man/vehicle for complete journey
The speed of an object with respect to other is called 2xy
will be .
relative speed. Suppose two bodies are moving with x+ y
speeds of x km/h and y km/h respectively, then their
Distance relative speed will be EXAMPLE 5. A man covers half of his journey at
6 km/h and the remaining half at 3 km/h. Then, his
The length of the path travelled by any object or a person between two places is known as distance. (i) ( x + y) km/h, if both the bodies are moving in average speed is
The unit of distance is m or km. opposite directions.
a. 1 km/h b. 2 km/h c. 3 km/h d. 4 km/h
(ii) ( x − y) km/h, if both the bodies are moving in same
Sol. d. Here, x = 6 km/h and y = 3 km/h
Time direction.
2xy 2 × 6 × 3 36
The duration in hours, minutes or seconds spent to cover a certain distance is called time. EXAMPLE 3. Two persons 27 km apart setting out at ∴ Average speed = = = = 4 km/h
x+ y 6+ 3 9
the same time are together in 9 h, if they walk in the
Speed same direction but in 3 h, if they walk in opposite Rule 3 If a man changes his speed to  x  of his usual
directions. Then, their rates of walking (speeds) are  y
The distance travelled by any object or a person per unit time is known as speed of that object or a
person. The unit of speed is m/s or km/h. a. 2 km/h and 4 km/h b. 3 km/h and 5 km/h
speed and gets late by t min, then the usual time taken by
distance c. 4 km/h and 8 km/h d. None of these t ×x
So, speed = Sol. d. Let the first person be walking faster with speed him = .
time ( y − x)
x km/h and second walking with speed y km/h.
EXAMPLE 1. There are 20 poles with a constant distance between each pole. A car takes 24 s to Case I Both walking in same directions.  x
Note If a person changes his speed to   of his usual and
reach the 12th pole. How much time will it take to reach the last pole? ∴ Distance travelled by first person in 9 h = 9x km  y
t ×x
a. 25.25 s b. 17.45 s c. 35.75 s d. 41.45 s and distance travelled by second person in 9 h = 9 y km reaches early by t min, then usual time taken by him = .
( x − y)
Sol. d. Suppose distance between each pole is 1m. As both are 27 km apart
∴ Total distance = 19 m ∴ 9x − 9y = 27 ⇒ x − y = 3 ...(i) EXAMPLE 6. If a man travels with a speed of 2/5
It takes 24 s to cover 11 m. Case II Both walking in opposite directions. times of his original speed and he reached his office
24 ∴ Distance travelled by first person in 3 h = 3x 15 min late to fixed time, then the time taken with his
∴ To cover 19 m, it will take × 19 = 4145
. . original speed, is
11 and distance travelled by second person in 3 h = 3y
So, by condition, 3x + 3y = 27 ⇒ x + y = 9 ...(ii) a. 10 min b. 15 min c. 20 min d. 25 min
Conversion of Unit On adding Eqs. (i) and (ii), we get Sol. a. Here, x = 2 , y = 5 and t = 15 min
5 ⇒ 2x = 12 ⇒ x = 6 km/h
(i) To convert speed of an object from km/h to m/s, multiply it by . x × t 2 × 15
18 ∴ Required time = =
Put the value of x in Eq. (ii), we get y − x 5− 2
5 6 + y = 9 ⇒ y = 3 km/h
e.g. 36 km/h = 36 × m/s = 10 m/s =
2 × 15
= 2 × 5 = 10 min
18 So, their speeds are 6 km/h and 3 km/h. 3
MATHEMATICS Time and Distance 49 50 CDS Pathfinder

(ii) Time taken by a train x m long in passing EXAMPLE 11. Two trains, one from Howrah to Patna
Rule 4 When a man travels from A to B with a speed Rule 10 Let the speed of boat in downstream = u km/h
an object of length y m = Time taken by the and the other from Patna to Howrah, start
x km/h and reaches t 1 h later after the fixed time and when train to cover ( x + y) m. simultaneously. After they meet, the trains reach their and speed of boat in upstream = v km/h, then
he travels with a speed y km/h from A to B, he reaches his destinations after 9 h and 16 h, respectively. What is 1
the ratio of their speeds? (i) Speed of boat in still water = (u + v ) km/h
destination t 2 h before the fixed time, then distance EXAMPLE 9. A 100 m long train is moving at a speed 2
xy (t 1 + t 2 ) of 60 km/h. In what time will it cross a signal pole? a. 4:3 b. 3:1 1
between A and B = km. (ii) Speed of stream (current) = (u − v ) km/h
y−x a. 6 s b. 12 s c. 15 s d. None of these c. 4:5 d. 3:2 2
5 50 Sol. a. Let us name the trains as A and B. Then, A’s speed :
Sol. a. Here, speed of train = 60 km/h = 60 × = m/s EXAMPLE 13. The speed of boat upstream and
EXAMPLE 7. Walking at 3 km/h, Rajeev reaches his 18 3 B’s speed = t 2 : t1 = 16 : 9 = 4 : 3.
school 5 min late, if he walks at 4 km/h, he will be and distance covered by train in passing a pole speed of boat downstream are 7 km/h and 13 km/h,
5 min early. The distance of Rajeev’s school from his = Length of the train Problems Based on Boat and Stream respectively. Then, the speed of stream and speed of
house is Q Length of train = 100 m Boats and streams is an application of speed, time and
boat in still water is
1 1 Distance travelled by train distance. Some of the important terms are explained a. 10 km/h and 3 km/h b. 15 km/h and 9 km/h
a. 1 km b. 2 km c. 2 km d. 5 km ∴Time taken to pass the pole = = 6s
2 2 Speed of train below c. 20 km/h and 6 km/h d. 40 km/h and 12 km/h
5 5
Sol. b. Here, x = 3 km/h, y = 4 km/h, t1 = h and t 2 = h • Still water If the speed of water is zero, then water is Sol. a. Let the speed of boat in downstream and upstream
60 60 Rule 7 Time taken by a train of length l1 metre moving
considered to be still water. be u km/h and v km/h, respectively.
3 × 4 5 5
+ 
xy (t1 + t 2)  60 60 at a speed of s 1 m/s to cross another train of length l 2 • Stream water If the water of a river is moving at a Given, v = 7 km/h and u = 13 km/h
∴ Required distance = = moving at a speed of s 2 m/s in same/opposite direction is 1 1
y−x 4−3 certain speed, then it is called stream water. Speed of boat in still water = (7 + 13) = ( 20) = 10 km/h
l1 + l 2 2 2
10 • Downstream motion In water, the direction along the 1 1
= 3× 4 × = 2 km s1 ± s 2 Speed of the stream = (13 − 7) = (6) = 3 km/h
60 stream is called downstream. 2 2
Rule 5 A person travels from A to B with a speed Use ‘+’ for opposite direction. • Upstream motion In water, the direction against the
Use ‘–’ for same direction. stream is called upstream.
x km/h and reaches t 1 h late. Later he increases his speed
Some rules and problems related to boat and stream are
RACES
by y km/h to cover the same distance and he still gets late EXAMPLE 10. Two trains of lengths 110 m and A contest of speed in running, driving, riding, sailing or
130 m travel on parallel track. If they move in the given below
by t 2 h, then the distance between A and B rowing over a specified distance is called a race.
x same direction, the first one which is faster takes one Rule 9 If the speed of a boat in still water be x
= (t 1 − t 2 )( x + y) . minute to pass the other one completely. If they move Suppose A and B are two contestants in a race. If A
y km/h and speed of stream be y km/h, then beats B by x m and length of the track is dm.
in opposite directions then they pass each other in 3s,
EXAMPLE 8. A boy walking at a speed of 20 km/h then the speed of the trains is (i) Speed of boat downstream = ( x + y) km/h Then, Distance travelled by B when A finishes the race
reaches his school 30 min late. Next time he a. 41 m/s and 39 m/s b. 32 m/s and 43 m/s (ii) Speed of boat upstream = ( x − y) km/h Speed of A d
= d − x and =
increased his speed by 5 km/h but still he is late by c. 42 m/s and 38 m/s d. None of these Speed of B d − x
10 min. What is the distance of the school from his EXAMPLE 12. A sailor goes 8 km downstream in
Sol. c. Let v1 be the velocity of faster train and v 2 be the
home? 40 min and returns back in 1 h. Then, the speed of Note (i) A gives B a start of x m means when A starts at starting
velocity of slower train.
100 50 100 2 the sailor in still water and the speed of the current is point, B starts x m ahead from starting point.
a. km b. km c. km d. km Case I If they move in the same direction, then
3 3 7 5 110 + 130 a. 5 km/h and 3 km/h b. 10 km/h and 2 km/h (ii) A gives B a start of t s means A starts t s after B starts
= 60 s ⇒ v1 − v 2 = 4 …(i) c. 7 km/h and 10 km/h d. None of these from the same point.
1 v1 − v 2
Sol. a. Given, x = 20 km/h, y = 5 km/h, t1 = 30 min = h
2 Case II If they move in opposite directions, Sol. b. Let the speed of the sailor in still water = x km/h EXAMPLE 14. In a 200 m race, A can beat B by 50m
1 110 + 130 and speed of the current (stream) = y km/h and B can beat C by 8m. In the same race, A can beat
and t 2 = 10 min = h = 3 ⇒ v1 + v 2 = 80 …(ii)
6 v1 + v 2 Then, speed of the sailor downstream = ( x + y) km/h C by what distance?
x
∴ Required distance = (t1 − t 2) ( x + y) On adding Eqs. (i) and (ii), we get 2v1 = 84 ⇒ v1 = 42 m/s and speed of the sailor upstream = ( x − y) km/h a. 60 m b. 72 m c. 56 m d. 66 m
y
Now, putting the value of v1 in Eq. (ii), we get 40 2 Sol. c. A can beat B by 50 m.
=  −  ( 20 + 5) × ∴ Time to travel 8 km downstream = 40 min = h= h
1 1 20 100
= km 10
 2 6 5 3 42 + v 2 = 80, v 2 = 38 m/s = 3 × 4 × = 2 km 60 3 speed of A 200 200
60 ∴ = =
8 2 24  distance  speed of B 200 − 50 150
⇒ = ⇒x+ y = Q = time
Problem Based on Trains Rule 8 If two trains start at the same time from points x+ y 3 2  speed  Also, B can beat C by 8m,
Problems based on trains are same as the problems A and B towards each other and after crossing each other, ⇒ x + y = 12 …(i) speed of B 200 200
∴ = =
related to speed, Time and Distance. The only difference they take t 1 and t 2 time in reaching points B and A 8 speed of C 200 − 8 192
and time to return = 1h ⇒ =1
is that the length of the moving object (train) is taken t x−y speed of A speed of B speed of A
respectively, then ratio of their speed = 2 Now, = ×
into consideration in these types of problems. t1 ⇒ x− y =8 …(ii) speed of C speed of C speed of B
Rule 6 (i) Time taken by a train x m long in passing a S1 t On adding Eqs. (i) and (ii), we get x = 10 and y = 2 200 200 200
or = 2 = × =
single post or pole or standing man = Time S2 t1 Hence, speed of sailor in still water = 10 km/h 150 192 144
taken by the train to cover x m. Speed of current (stream) = 2 km/h So, A beats C by (200 − 144) = 56 m
MATHEMATICS Time and Distance 51 52 CDS Pathfinder

PRACTICE EXERCISE 18. A bullock cart has to cover a distance of 80 km


in 10 h. If it covers half of the journey in 3/5th
time, what should be its speed to cover the
28. A person x started 5 min earlier at 60 km/h from
a place P, then another person y followed him at
48 km/h, started his journey at 3:05 pm. Which
remaining distance in the time left? of the following is/are correct.
from B to A at a speed of 60 km/h. The time of (a) 5 km/h (b) 10 km/h (c) 15 km/h (d) 18 km/h I. At 3 : 15 pm, x and y are 7 km apart.
1. A car completes a journey in 6 h with a speed of
their meeting is 3 II. At 3 : 25 pm, y will overtake x
50 km/h. At what speed must it travel to 19. By walking at of his usual speed, a man reaches
(a) 8 : 30 am (b) 8 : 00 am (c) 9 : 00 am (d) 9 : 15 am
complete the journey in 5 h? 4 (a) Only I (b) Only II
(a) 60 km/h (b) 55 km/h (c) 45 km/h (d) 61 km/h 11. A train T1 leaves a place P at 5:00 am and his office 25 min later than usual. His usual time is (c) Both I and II (d) Neither I nor II
reaches another place Q at 9 : 00 am another (a) 60 min (b) 70 min (c) 75 min (d) 80 min
2. Kiran covers a certain distance at 80 km/h and 29. A boy walking at a speed of 20 km/h reaches his
returns back to the same point at 20 km/h. Then, train T2 leaves the place Q at 7 : 00 am and 20. A train covers a distance in 50 min, if it runs at school 30 min late. Next time he increases his
the average speed during the whole journey is reaches the place P at 10 : 30 am. The time at a speed of 48 km/h on an average. The speed at speed by 4 km/h but still he is late by 10 min.
(a) 35 km/h (b) 32 km/h (c) 30 km/h (d) 28 km/h which the two trains cross each other is which the train must run to reduce the time of
Which of the following statement is/are correct?
(a) 8 : 26 am (b) 7 : 56 am (c) 8 : 15 am (d) 8 : 00 am journey to 40 min, will be
3. Normally Sarita takes 3 h to travel between two I. The distance of the school from his home is
(a) 10 km/h (b) 20 km/h (c) 40 km/h (d) 60 km/h
stations with a constant speed. One day her 12. A certain distance is covered at a certain speed. 50 km/h.
speed was reduced by 12 km/h and she took If half of the distance is covered in double time, 21. Two trains travel in the same direction at II. If he increases his speed by 10 km/h but still
45 min more to complete the journey. Then, the then the ratio of the two speeds is 50 km/h and 32 km/h, respectively. A man in the late by 10 min, then distance of the school will
distance between the two stations is (a) 4 : 1 (b) 1 : 4 (c) 2 : 1 (d) 1 : 2 slower train observes that 15 s elapse before the be 20 km.
faster train completely passes him. What is the (a) Both I and II are correct (b) Only I is correct
(a) 60 km (b) 120 km (c) 180 km (d) 95 km 13. Two trains start running at the same time from length of the faster train? (c) Only II is correct (d) Neither I nor II are correct.
4. Rani goes to school at 10 km/h and reaches the two stations which are 210 km apart and going 625
(a) 75 m (b) 125 m (c) 150 m (d) m
school 6 min late. Next day, she covers this in opposite directions cross each other at a 3
distance of 100 km from one of the station. The Directions (Q. Nos.30-31) Ramesh and Prateek start
distance at 12 km/h and reaches the school 22. A father and his son start at a point A with running at same time in opposite directions from
9 min earlier than the scheduled time. What is ratio of their speed is speeds of 12 km/h and 18 km/h, respectively and
(a) 11 : 9 (b) 10 : 11 (c) 11 : 10 (d) 9 : 11 two points and after passing each other they
the distance of her school from her house? reach another point B. If his son starts 60 min
complete their journeys in ‘x’ and ‘y’ h, respectively.
(a) 16 km (b) 12 km (c) 10 km (d) 15 km 14. A police car is ordered to chase a speeding car after his father at A and reaches B, 60 min
that is 5 km ahead. The thief car is travelling at before his father, what is the distance between A Then, Speed of Ramesh : Speed of Prateek = y : x
5. A man travels first 50 km at 25 km/h next 40 km and B ?
at 20 km/h and then 90 km at 15 km/h. His an average speed of 80 km/h and the police car 30. Find the ratio of the speeds of Ramesh and
pursues it at an average speed of 100 km/h. How (a) 90 km (b) 72 km (c) 36 km (d) None of these
average speed (in m/s) for the whole journey is Prateek, if Ramesh and Prateek completed their
(a) 18 (b) 5 (c) 10 (d) 36 long does it take for the police car to overtake 23. Two trains of lengths 100 m and 150 m are journeys in 16 h and 25 h after passing each other.
the other car? travelling in opposite directions at speeds of
6. A boy is running at a speed of p km/h to cover a (a) 5 : 4 (b) 5 : 3 (c) 4 : 5 (d) 3 : 5
(a) 17 min (b) 19 min (c) 13 min (d) 15 min 75 km /h and 50 km/h, respectively. What is the
distance of 1 km. But due to the slippery ground, time taken by them to cross each other? 31. What if two persons completed their journey
his speed is reduced by q km/h ( p > q ). If he 15. Points A and B are 70 km apart on a highway. A (a) 7.4 s (b) 7.2 s (c) 7 s (d) 6.8 s with speeds in the ratio 7 : 9, then how much
takes r h to cover the distance, then car starts from A and another car starts from B time they should have taken to complete their
1 pq 1 1 24. A man can walk uphill at the rate of 2.5 km/h
(a) = (b) = p + q (c) r = p − q (d) = p − q at the same time. If they travel in the same and downhill at the rate of 3.25 km/h. If the journey after they meet each other.
r p+ q r r direction they meet in 7 h, but they travel (a) 81 : 49 (b) 7: 9 (c) 49 : 81 (d) 14 : 18
total time required to walk a certain distance up
towards each other they meet in 1 h. What are the hill and return to the starting position is 4 h
7. A train passes telegraph post in 40 s moving at a
the speeds of the cars? 36 min, what is the distance he walked up the hill?
rate of 36 km/h. Then, the length of the train is
(a) 30 km/h, 40 km/h (b) 36 km/h, 40 km/h
PREVIOUS YEARS’ QUESTIONS
(a) 400 m (b) 500 m (c) 450 m (d) 395 m (a) 3.5 km (b) 4.5 km (c) 5.5 km (d) 6.5 km
(c) 19 km/h, 20 km/h (d) 40 km/h, 50 km/h 32. A train 280 m long is moving at a speed of
8. A person can run around a circular path of 25. A boat goes 30 km upstream and 44 km
16. Assume that the distance that a car runs on 1 L downstream in 10 h. In 13 h it can go 40 km 60 km/h. What is the time taken by the train to
radius 21 m in 44 s. In what time will the same cross a platform 220 m long? e 2012 I
person run a distance of 3 km? of petrol varies inversely as the square of the upstream and 55 km downstream. The speed of
speed at which it is driven. It gives a run of the boat in still water is (a) 45 s (b) 40 s (c) 35 s (d) 30 s
(a) 18 min 40 s (b) 16 min 30 s
25 km/L at a speed of 30 km/h. At what speed (a) 9 km/h (b) 8 km/h (c) 4 km/h (d) 3 km/h 33. A car travels along the four sides of a square at
(c) 18 min 30 s (d) 16 min 40 s
should it be driven to get a run of 36 km/L? 26. A motorboat takes 2 h to travel a distance of speeds v , 2 v , 3 v and 4 v , respectively. If u is the
9. A car is ahead of a scooter by 30 km. Car goes at (a) 12.5 km/h (b) 25 km/h (c) 30 km/h (d) 40 km/h 9 km down the current and it takes 6 h to travel average speed of the car in its travel around the
the rate of 50 km/h and the scooter goes at the the same distance against the current. What is square, then which one of the following is
rate of 60 km/h. The scooter overtakes the car 17. A man standing on a railway platform observes correct? e 2012 I
the speed of the boat in still water (in km/h)?
after that a train going in one direction takes 4 s to
(a) 3 (b) 2 (c) 1.5 (d) 10 (a) u = 2 .25 v (b) u = 3 v
1 pass him. Another train of same length going in (c) v < u < 2 v (d) 3 v < u < 4 v
(a) 3 h (b) 3.5 h (c) 4 h (d) 3 h
4 the opposite direction takes 5 s to pass him. The 27. Ram travels from P to Q at 10 km/h and return at
time taken (in seconds) by the two trains to cross 15 km/h. Shyam travels from P to Q and return 34. A car is travelling at a constant rate of 45 km/h.
10. Two towns A and B are 250 km apart. A bus each other will be at 12.5 km/h. If he takes 12 min less than Ram, The distance travelled by car from 10 : 40 am to
starts from A to B at 6 : 00 am at a speed of 32 33 40 49 then what is the distance between P and Q? 1 : 00 pm is e 2012 II
40 km/h. At the same time another bus starts (a) (b) (c) (d)
9 7 9 9 (a) 60 km (b) 45 km (c) 36 km (d) 30 km (a) 165 km (b) 150 km (c) 120 km (d) 105 km
MATHEMATICS Time and Distance 53 54 CDS Pathfinder

35. A person travels a certain distance at 3 km/h 45. A man rides one-third of the distance from A to 53. By increasing the speed of his car by 15 km/h, a 60. A bike consumes 20 mL of petrol per kilometre,
and reaches 15 min late. If he travels at 4 km/h, B at the rate of x km/h and the remainder at the person covers 300 km distance by taking an hour if it is driven at a speed in the range of
he reaches 15 min earlier. The distance he has to rate of 2y km/h. If he had travelled at a uniform less than before. The original speed of the car 25-50 km/h and consumes 40 mL of petrol per
travel is e 2013 I rate of 6z km/h, then he could have ridden from was e 2015 II kilometre at any other speed. How much
(a) 4.5 km (b) 6 km (c) 7.2 km (d) 12 km A to B and back again in the same time. Which (a) 45 km/h (b) 50 km/h (c) 60 km/h (d) 75 km/h petrol is consumed by the bike in travelling a
36. If a body covers a distance at the rate of x km/h one of the following is correct? e 2014 II distance of 50 km, if the bike is driven at a
54. Two trains, one is of 121 m in length at the
and another equal distance at the rate of y km/h, (a) z = x + y (b) 3z = x + y speed of 40 km/h for the first 10 km, at a speed
speed of 40 km/h and the other is of 99 m in of 60 km/h for the next 30 km and at a speed of
1 1 1 1 1 1
then the average speed (in km/h) is e 2013 I (c) = + (d) = + length at the speed of 32 km/h are running in
x+ y 2x y x+ y z x y 2z x y 30 km/h for the last 10 km? e 2015 II
(a) (b) x y (c) (d) opposite directions. In how much time will they (a) 1 L (b) 1.2 L (c) 1.4 L (d) 1.6 L
2 x+ y xy 46. In a flight of 600 km, an aircraft was slowed be completely clear from each other from the
37. A sailor sails a distance of 48 km along the flow down due to bad weather. Its average speed for moment they meet ? e 2015 II 61. A passenger train takes 1 h less for a journey of
the trip was reduced by 200 km/h and the time (a) 10 s (b) 11 s (c) 16 s (d) 21 s 120 km, if its speed is increased by 10 km/h
of a river in 8 h. If it takes 12 h to return the
of flight increased by 30 min. The duration of the from its usual speed. What is its usual speed?
same distance, then the speed of the flow of the 55. The speeds of three buses are in the ratio
river is e 2013 I flight is e 2015 I e 2016 I
2 : 3 : 4. The time taken by these buses to travel (a) 50 km/h (b) 40 km/h (c) 35 km/h (d) 30 km/h
(a) 0.5 km/h (b) 1 km/h (c) 1.5 km/h (d) 2 km/h (a) 1 h (b) 2 h (c) 3 h (d) 4 h the same distance will be in the ratio. e 2015 II
38. A train running at the speed of 72 km/h goes 47. With a uniform speed, a car covers a distance in (a) 2 : 3 : 4 (b) 4 : 3 : 2 62. A man walking at 5 km/h noticed that a 225 m
(c) 4 : 3 : 6 (d) 6 : 4 : 3 long train coming in the opposite direction crossed
past a pole in 15 s. What is the length of the 8 h. Had the speed been increased by 4 km/h,
him in 9 s. The speed of the train is e 2016 I
train? e 2013 II the same distance could have been covered in 7 h 56. Two trains are moving in the same direction (a) 75 km/h (b) 80 km/h (c) 85 km/h (d) 90 km/h
(a) 150 m (b) 200 m (c) 300 m (d) 350 m and 30 min. What is the distance covered? at 1.5 km/min and 60 km/h, respectively. A man
e 2015 I
in the faster train observes that it takes 27 s to
63. A cyclist moves non-stop from A to B, a distance
39. Two cars A and B start simultaneously from a (a) 420 km (b) 480 km (c) 520 km (d) 640 km of 14 km, at a certain average speed. If his
certain place at the speed of 30 km/h and cross the slower train. The length of the slower
average speed reduces by 1 km/h, then he takes
45 km/h, respectively. The car B reaches the 48. A car travels the first one-third of a certain train is e 2015 II
20 min more to cover the same distance. The
destination 2 h earlier than A. What is the distance with a speed of 10 km/h, the next (a) 225 m (b) 230 m (c) 240 m (d) 250 m
original average speed of the cyclist is e 2016 I
distance between the starting point and one-third distance with a speed of 20 km/h and 57. In a race of 100 m, A beats B by 4 m and A beats (a) 5 km/h (b) 6 km/h
destination? e 2013 II the last one-third distance with a speed of C by 2 m. By how many metres (approximately) (c) 7 km/h (d) None of these
(a) 90 km (b) 180 km (c) 270 km (d) 360 km 60 km/h. The average speed of the car for the would C beat B in another 100 m race assuming
whole journey is e 2015 I 64. In a race of 1000 m, A beats B by 100 m or 10 s.
40. A man cycles with a speed of 10 km/h and C and B run with their respective speeds as in If they start a race of 1000 m simultaneously
reaches his office at 1 : 00 pm. However, when (a) 18 km/h (b) 24 km/h (c) 30 km/h (d) 36 km/h the earlier race? e 2015 II
from the same point and if B gets injured after
he cycles with a speed of 15 km/h, he reaches his 49. A man rows 32 km downstream and 14 km (a) 2 (b) 2.04 (c) 2.08 (d) 3.2 running 50 m less than half the race length and
office at 11 : 00 am. At what speed should he upstream, and he takes 6 h to cover each distance. due to which his speed gets halved, then by how
cycle, so that he reaches his office at 12 noon?
58. Three athletes run a 4 km race. Their speeds are in
What is the speed of the current? e 2015 I
the ratio 16 : 15 : 11. When the winner wins the much time will A beat B ? e 2016 I
e 2013 II (a) 0.5 km/h (b) 1 km/h (c) 1.5 km/h (d) 2 km/h (a) 65 s (b) 60 s (c) 50 s (d) 45 s
race, then the distance between the athlete in the
(a) 12.5 km/h (b) 12 km/h (c) 13 km/h (d) 13.5 km/h 2 second position to the athlete in the third
50. A runs 1 times as fast as B. If A gives B a start 65. In a race A, B and C take part. A beats B by
41. A train takes 9 s to cross a pole. If the speed of the 3 position is e 2015 II 30 m, B beats C by 20 m and A beats C by 48 m.
train is 48 km/h, the length of the train is e 2014 I of 80 m, how far must the winning post from the (a) 1000 m (b) 800 m (c) 750 m (d) 600 m
starting point be so that A and B might reach it Which of the following is/are correct?
(a) 150 m (b) 120 m (c) 90 m (d) 80 m
at the same time? e 2015 I
59. A motorboat, whose speed is 15 km/h in still I. The length of the race is 300 m.
42. A train takes 10 s to cross a pole and 20 s to (a) 200 m (b) 300 m (c) 270 m (d) 160 m
water goes 30 km downstream and comes back in II. The speeds of A, B and C are in the ratio
cross a platform of length 200 m. What is the a total of 4h and 30 min. The speed of the 50 : 45 : 42.
length of the train? e 2014 II 51. A thief is noticed by a policeman from a distance stream is e 2015 II
Select the correct answer using codes given below.
(a) 50 m (b) 100 m (c) 150 m (d) 200 m of 200 m. The thief starts running and the (a) 4 km/h (b) 5 km/h
(a) Only I (b) Only II e 2016 I
policeman chases him. The thief and the (c) 6 km/h (d) 10 km/h
(c) Both I and II (d) Neither I nor II
43. A train travels at a speed of 40 km/h and policeman run at the speed of 10 km/h and
another train at a speed of 20 m/s. What is the 11 km/h, respectively. What is the distance
ratio of speed of the first train to that of the between them after 6 min?
second train? e 2014 II (a) 100 m (b) 120 m
e 2015 I
(c) 150 m (d) 160 m
ANSWERS
(a) 2 : 1 (b) 5 : 9 (c) 5 : 3 (d) 9 : 5 1 a 2 b 3 c 4 d 5 b 6 d 7 a 8 d 9 a 10 a
52. Two persons A and B start simultaneously from
44. The distance between two points (A and B ) is two places c km apart and walk in the same 11 b 12 a 13 c 14 d 15 a 16 b 17 c 18 b 19 c 20 d
110 km. X starts running from point A at a direction. If A travels at the rate of p km/h and B 21 a 22 b 23 b 24 d 25 b 26 a 27 d 28 a 29 c 30 a
speed of 60 km/h and Y starts running from travels at the rate of q km/h, then A has
point B at a speed of 40 km/h at the same time. travelled before he overtakes B a distance of 31 a 32 d 33 c 34 d 35 b 36 c 37 b 38 c 39 b 40 b
They meet at a point C, somewhere on the line e 2015 I 41 b 42 d 43 b 44 a 45 c 46 a 47 b 48 a 49 c 50 a
AB. What is the ratio of AC to BC ? e 2014 II qc pc qc pc 51 52 53 54 55 56 57 58 59 60
(a) km (b) km (c) km (d) km a b c b d a b a b d
(a) 3 : 2 (b) 2 : 3 (c) 3 : 4 (d) 4 : 3 p+ q p−q p−q p+ q
MATHEMATICS Time and Distance 55 56 CDS Pathfinder

When A = 36 km/L 22. (b) Let distance between A and On solving the above equations, we get
HINTS AND SOLUTIONS v2 =
22500
36
⇒v =
22500 150
36
=
6
B be x km.
By given condition,
x = 8, y = 3
Thus, the speed of boat in still water
x x = 8 km/h
Hence, the speed of a car is 25 km/h. − = 2 ⇒ 6x = 2 × 18 × 12
1. (a) Let speed be v km/h. 7. (a) Length of train = Distance covered 12. (a) Let x km distance be covered in y h. 12 18
17. (c) Let the length of each train be I m. 26. (a) Let the speed of motorboat be
2 × 18 × 12
⇒ Speed of first train =   m/s ∴ x= = 72 km
We know that, in 40 s at the rate of 36 km/h. Then, speed of object in first case I x km/h and the speed of water be y km/h.
[Rule 6 (i)] x  4 6
Distance = Speed × Time = km/h Now, speed of boat downstream
and speed of second train =   m/s
I Hence, the required distance is 72 km.
6 × 50 = 5 × v ∴ Length of train y = ( x + y ) km/h [Rule 9]
5 5 23. (b) Relative speed when trains are in
6 × 50 = 40 × 36 × = 400 m As, half of this distance is covered in
⇒ v= = 60 km/h As, both trains are moving in opposite and speed of boat upstream
18 double time. opposite direction
5 direction. = ( x − y ) km/h
2xy 8. (d) Distance travelled in 44 s = 2 πr Then, speed of object in second case =V + V
2. (b) Average speed = [Rule 2] Time taken to cross each other 1 2 By given condition,
x + y =2×
22
× 21 = 132 m
x
= ÷ 2y = ×
x 1
=
x
km/h = 75 + 50 = 125 km/h
I+I 9
7 2 2 2y 4y = [Rule 7] 125 × 5 = 2 ⇒ 2x + 2 y = 9 …(i)
2 × 80 × 20 I I = m/s x+ y
= = 32 km/h 132 ∴ Ratio of first and second speeds + 18
∴ Speed = = 3 m/s
80 + 20 x x 1
4 5 and total distance covered 9
44
= : = 1: = 4 : 1   and = 6 ⇒ 6x − 6 y = 9
Q speed = distance  y 4y 4  2I   20 × 2  40 = ( 100 + 150) = 250 m x− y
3. (c) Let Sarita’s speed be u and distance is  time  =  s=  = s ⇒ 2x − 2 y = 3
constant.  9I   9  9 ∴ Time taken to cross each other …(ii)
3000 13. (c) Let the speed of two trains be x km/h  
Then, u × 3 = (u − 12)
15
Time taken to travel 3 km =  20  Total covered distance On solving Eqs. (i) and (ii), we get
4 3
and y km/h, respectively. Then, the time =
taken by first train to cover Relative speed x = 3 km/h and y = 1.5 km/h
Q 3 h + 45 min = 15 h  = 1000 s =
1000
min = 16 min 40 s
18. (b) Given, total distance to cover in 10 h
110 km = Time taken by second train to 250 × 18 ∴ Speed of boat = 3 km/h
 4  60 = 80 km = = 7.2 s
45 × 4 cover 100 km . 3 125 × 5 27. (d) Let the distance between P and
⇒ u= = 60 km/h 9. (a) Distance between car and scooter 110 100 x 110 If it covers 40 km in th of time i.e.
= 30 km Thus, = ⇒ = 5 24. (d) Let he walked up the hill at a Q = d km. Total time taken by Ram
3
x y y 100 40 km in 6 h. d d 25d
∴ Distance = Speed × Time Relative speed distance of x km. = + =
∴ x : y = 11 : 10 ∴ Remaining time = 10 − 6 = 4 h 10 15 150
= 60 × 3 = 180 km y = 60 − 50 = 10 km/h By given condition, 2d 4d
14. (d) Distance travelled by thief car in one Remaining distance = 40 km x x 36 Total time taken by Shyam = =
6 So, the time taken by scooter to overtake + = 4 12.5 25
4. (d) Here, x = 10, y = 12, t = h and 30 hour = 80 km 25. 3.25 60 According to question,
1
60 the car = =3h ∴ Required speed = ( 40 ÷ 4) km/h
⇒ x 
10 Distance travelled in one hour by police 1 1  276 25d 4d 12
t =
9
=
40
= 10 km/h +  = − =
2
h car = 100 km  25
. 3.25  60 150 25 60
60 10. (a) Let the buses meet x h after 6:00 am. 4
So, police travels extra 20 km in 1 h. 25 − 24  1
xy( t + t ) Then, the distance covered by the two 19. (c) Here, x = 3 m/s, y = 4 m/s ⇒ x  +
2 4  276
 = ⇒ d  =
∴ Required distance =
1 2 So, to overtake thief, police car has to  5 13  60  150  5
y−x buses is 250 km. travel 5 km extra. and t = 25 min
276 65 d 1
∴ 40x + 60x = 250 ∴ Time =
5 1
= h=
60
min = 15 min x ×t ∴ x = × = 6.5 km ⇒ = ⇒ d = 30 km
∴Required time = [Rule 3] 60 46 150 5
[Rule 4] 250 20 4 4 y−x
⇒ x= = 2.5 h = 2h and 30 min
10 × 12 × 15 100 25. (b) Let speed of boat in still water 28. (a) Distance travelled by x in 15 min
= 15. (a) Let the speed of car A = x km/h 3 × 25
So, they will meet at 8 : 30 am. = = 75 min 15
60 × 2 Let the speed of car B = y km/h 4 −3 = x km/h = 60 × = 15 km
11. (b) Let the distance between P and Q be 60
Total distance covered by b other the 20. (d) Distance travelled in 1h = 48 km and speed of boat in current water
= 15 km x km and let the two trains meet y h Distance travelled by y in 10 min
cars in 1h = 70 km ∴ Distance travelled in 50 min = y km/h 10
5. (b) Total distance covered after 7 am. = 48 × = 8 km
Then,T covers x km in 4h and T covers x ( 1) + y( 1) = 70 48 ∴ Downstream speed = ( x + y ) km/h 60
= 50 + 40 + 90 = 180 km 1 2 = × 50 = 40 km
1 x + y = 70 …(i) 60 Upstream speed = ( x − y ) km/h Difference = ( 15 − 8) km = 7 km
x km in 3 h. 40
Total time taken 2 [in opposite direction] Time to be reduced = h [Rule 9] Hence, at 3:15 pm they are 7 km apart.
60
= 
50 40 90  x ∴ Required speed
+ +  = 10 h ∴ Speed of train T = km/h Distance covered by both the cars in According to the question, So, statement I is true. As speed of x is
 25 20 15  1
4 7 h = 70 km 40 × 60 30 44
40 + = 10 …(i) greater than y. So, y will never overtake x.
∴ Average speed for the whole journey 2x = = = 60 km/h x− y x+ y
and speed of train T = km/h But in same direction 40 / 60 40 Thus, statement II is false.
2
7 7x − 7 y = 70 ⇒ x − y = 10 ...(ii) 40 55
Total distance travelled 29. (c) I. Here, x = 20 km/h, y = 4 km/h,
= 21. (a) Let the length of faster train be x km. and + = 13 …(ii)
Total time taken According to the question, added Eq. (i) and (ii), we get x− y x+ y t = 30 min, t = 10 min
x ( y + 2) 2xy Q Trains travel in the same direction.
180 + =x 2x = 80, x = 40 km/h, y = 30 km/h 1
= u and
1
=v
1 2
= = 18 km/h ∴ Relative speed = (50 − 32) = 18 km/h
Let …(iii) According to formula,
10 4 7
16. (b) Let the speed of car be v and distance x− y x+ y
18 × 5 ( y + 2) 2 y 15 ∴ Required distance
⇒ + =1 covered by car in one litre be A. Elapsed time = 15 s = h 30u + 44v = 10
∴ 18 km/h = m/s = 5 m/s Then, x
18 4 7 1 K 3600 = (t − t ) (x + y) [Rule 5]
7( y + 2) + 8 y ∴ A∝ 2 ⇒ A= 2 distance 40u + 55v = 13 1 2
y
6. (d) Actual speed of boy = ( p − q ) km/h ⇒ =1 v v Now, time =
(30 − 10)
( 20 + 4)  
speed On solving the above equations, we get 20
1 28 Then, A = 25 km/h and v = 30 km/h =
Time taken to cover 1 km = ⇒ 7 y + 14 + 8 y = 28 K 15 x 1
u = ,v=
1 60  4
p−q So, 25 = ⇒ K = 900 × 25 ⇒ = ⇒ x × 3600 = 18 × 15 5 11
14 14 20 20
∴ y= h= × 60 min = 56 min (30)2 3600 18 = × 24 × = 5 × 8 = 40 km
1 1
∴ =r ⇒ = p−q 15 15 22500 18 × 15 From Eq. (iii), we get 60 4
p−q r ⇒ K = 22500 ⇒ A = ∴x= = 0.075km = 75 m So, I is incorrect.
So, trains will meet at 7 : 56 am. v2 3600 x − y = 5 and x + y = 11
MATHEMATICS Time and Distance 57 58 CDS Pathfinder

II. Here, x = 20 km/h, y = 10 km/h 140 40. (b) Let t h be the time taken by the man 44. (a) Given, distance between two points 1 1 1 = ( x − y ) km / h
= 140 min = h − =
t = 30 min, t = 10 min 60 to reach his office at speed of 15 km/h. (A and B) = 110 km x − 200 x 600 × 2 According to the question,
1 2
According to formula, So, required distance travelled by car Then, time taken to reach office at the Their relative speed x − ( x − 200) 32
=6 Q speed = distance 
= speed × time speed of 10 km/h = ( t + 2) h 1
∴ Required distance 140 = 60 + 40 = 100 km/h ⇒ = x+ y  time 
= 45 × = 105 km s ×t = s ×t x ( x − 200) 1200
 30 − 10  60 Now, Time after which they meet 14
 ( 20 + 10)  20  =6
1 1 2 2
= and
 60   10  35. (b) Given, x = 3 km/h, y = 4 km/h, ⇒ 10 × ( t + 2) = 15 × t =
Total distance ⇒ x2 − 200x − 240000 = 0 x− y
 
15 ⇒ 10 t + 20 = 15 t ⇒ 5 t = 20 Relative speed ⇒ ( x − 600)( x + 400) = 0 ⇒ 6x + 6 y = 32 … (i)
[Rule 5] t = 15 min = h
1
60 ∴ t =4h 110 ⇒ x = 600 km / h and 6x − 6 y = 14 … (ii)
=
20
× 30 ×
20
= 20 km =
60 10 and t = 15 min =
15
h Now, distance covered to reach office 100 or x ≠ −400 km/h On subtracting Eq. (ii) from Eq. (i), we
2
60 =s × t C 600 get
So, II is correct. xy ( t + t ) 1 1 A B ∴ Required time = = 1 h.
∴ Required distance = 1 2 110 km 600 12 y = 18
30. (a) x = 16, y = 25 = 10 × ( 4 + 2) = 10 × 6 = 60 km
y−x = 1. 10 h y=
18
= 1.5 km/h
Now, speed required to reach office at 12 47. (b) Let the distance between A and B be
According to formula, [Rule 4] Distance covered by A in 1.1 h 12
noon in 5 h. x km and speed be V km/h.
Ramesh’ s Speed y 25 5 = AC = 60 × 1.1 = 66 km
3 × 4  +
∴ = = = 15 15  50. (a) Let the speed of B be x m/s.
 ∴ Speed =
60
= 12 km/h x km
Prateek’ s Speed x 16 4  60 60  Remaining distance 2 5x
= 5 A B ∴ Speed of A = 1 x = m/s
or 5:4 4 −3 = BC = 110 − 66 = 44 km 3 3
41. (b) Speed of train = 48 km/h Case I Given, distance = x km, speed
31. (a) Here, we know the relation = 12 ×
30 1
= 12 × = 6 km ∴ Required ratio = AC : BC Ratio of speed of rates of A and B
=  48 ×
5 = V km/h and time = 8 h.
Speed of Ramesh y 7 y 60 2  m/s = 66 : 44 = 3 : 2 5x
= ⇒ =  18  Distance x = : x = 5 :3
Speed of Prateek x 9 x 36. (c) If a body covers a distance at the rate 45. (c) Let the total distance be d. ∴ Speed = ⇒ V = … (i) 3
of x km/h and another equal distance at Let the length of train be x m 1 Time 8
49 y Time taken to cover rd distance, Q 2 m are gained in a race of 5 m.
On squaring both sides, = = 81 : 49 the rate of y km/h, then =
2xy 5 Case II If speed = (V + 4) km/h
81 x x = 48 × ×9 3 5
x+ y 18 1 1 15 ∴ 1 m are gained in a race of m.
d and time = 7 h = h 2
32. (d) To cover a distance by train [Rule 2] x = 120 m d  distance 
t =3 = Q time = 
2 2
= 280 + 220 = 500 m 1
x 3x  speed  then, V +4=
x
⇒ V +4=
2x So, 80 m are gained in a race of
37. (b) Let speed of the flow of water Length of the train is 120 m. 5
Speed of train = 60 km/h 1 2 15 / 2 15 × 80 m = 200 m
5
be v km/h and rate of sailing of sailer be
42. (d) Let the speed of a train be x m/s and Remaining distance = d − d = d
= 60 × m/s 3 3 x 2x 2
u km/h.
length be y m. 2 ⇒ +4= [from Eq .(i)]
18 Time taken to cover rd distance, 8 15 51. (a) Given, speed of thief = 10 km/h
50 Speed of sailor downstream
= m/s Condition I When t = 10 s 2 3 2x x x
= (u + v ) km/h [Rule 9] d ⇒ − =4 ⇒ =4 10 × 1000 500
3 distance 2d d = m/min = m/min
time = t = 3 = = 15 8 120 60 3
∴ Time taken by train Speed of sailor upstream = (u − v ) km/h speed 2
2y 6y 3y
total distance 48 ⇒ x = 480 km and speed of policeman = 11 km/h
= Condition I u + v = y
⇒ 10 = ∴ y = 10x ...(i) Time taken to cover distance from A to 48. (a) x/3 km x/3 km x/3 km
speed 8 x 11 × 1000 550
B and B to A, t1 t2 t3 = m/min = m/min
500 ⇒ u+ v=6 ...(i) A C D B
= = 30 s Condition II When t = 20 s 2d d x km 60 3
50 / 3 48 ∴ t= =
Condition II u − v = and total distance = y + 200 m 6z 3z Now, distance travelled by thief in 6 min
12 Let total distance of AB be x km.
Total distance x/3 x 500
33. (c) Let side of a square be x. ⇒ u−v=4 ...(ii) ∴ Time = According to the question,
For distance AC, t = = h = × 6 = 1000 m
x Speed d d d 1
10 30 3
D C On subtracting Eqs. (ii) from (i), we get t + t =t ⇒ + =
3v y + 200 1 2
3x 3 y 3z x/3 x and distance travelled by policeman in
v = 1 km/h 20 = ⇒ 20x = y + 200 For distance CD, t = = h.
x 2
20 60 550
38. (c) Speed of train = 72 km/h ⇒
1
+
1
=
1

1 1 1
+ = 6 min = × 6 = 1100 m
x 4v 2v x 20 ×
y
= y + 200 3 x 3 y 3z x y z x/3 x 3
5 For distance BD, t = = h.
= 72 × 10 3
60 180 ∴ Difference = ( 1100 − 1000) = 100 m
v 18 46. (a) Let the original speed of an aircraft
A B ⇒ 2 y − y = 200 ⇒ y = 200 ∴ Total time taken Hence, the distance between them after
x be x km/h and its reduced speed
= 20 m/s Hence, the length of train is 200 m. 6 min is 100 m.
total distance = ( x − 200) km/h. =t +t +t =
x
+
x
+
x
Q Average speed ( u ) = Length of train = distance covered
total time 43. (b) Given, speed of a train = 40 km/h Condition I
1 2 3
30 60 180 52. (b) Let A and B will meet after t h at
(x + x + x + x) x = 20 × 15 10x
==
x point E.
= x x x x = 40 ×
5
m/s Time taken by aircraft to cover 600 km
+ + + x = 300 m 600 180 18
18 = h. Total distance A E
v 2v 3v 4v ∴ Average speed = B
4x Length of the train is 300 m. Speed of another train = 20 m/s x
= Total time taken Distance travelled by A = pt h
Condition II Time taken by aircraft to
x  +
1 1 1 1 39. (b) Let the distance between two points ∴ Required ratio x
+ +  cover = x = 18 km / h and distance travelled by B = qt h
 v 2v 3v 4v  be x km Speed of first train 600
= 600 km = h 18 According to the question,
4×v 48v Time taken by A − Time taken by B = 2 Speed of second train ( x − 200)
= = = 1.92 v x x pt = qt + c ⇒ pt − qt = c
1 1 1 25 ⇒ − =2 49. (c) Let speed of a boat in still water be
1+ + + 40 ×
5 According to the question, c
2 3 4 30 45
18 = 40 × 5 = 2 × 5
x km/h and speed of the stream be y km/h. ⇒ t( p − q ) = c ⇒ t = …(i)
− = 600

600 1 
= Q 30 min = h 
1 p−q
which lies in the interval v < u < 2v. x =2
3 2
⇒ 20 20 × 18 18 x − 200 2  2 
Then, speed of the boat downstream
 90  Distance travelled by A = pt
x Q
34. (d) Given, speed of a car = 45 km/h 10 5 = ( x + y ) km / h [Rule 9]
= = or 5 : 9 =
pc
[from Eq. (i)]
Time taken by the car ⇒ x = 90 × 2 = 180 km 18 9 On dividing both sides by 600, we get and speed of the boat upstream p−q
07
MATHEMATICS Time and Distance 59 60 CDS Pathfinder

53. (c) Let the original speed of car be 57. (b) When A covers 100 m, then B covers ⇒ x = 85 km/h
x km/h. 96 m and C covers 98 m. Hence, the speed of the train is 85 km/h.
300
Time taken to cover 300 km = h i.e. when C covers 98 m, then B covers
x 63. (c) Refer to question 46.
96 m.
If the speed of car is increased by 64. (a) Since, either A beats B by 100 m or
∴ when C covers 100 m, then B covers
15 km/h. 10 s. It means that B runs 100 m in 10 s.
100 × 96
300 300 100
Then, = −1 m, i.e. 97.96 ∴ Speed of B = = 10 m / s
x + 15 x 98 10
300 300
⇒ 1= − So,C beats B by approximately 2.04 m. B A
x x + 15

TIME AND WORK


58. (a) Ratio of speeds are 16 : 15 : 11. 900 m 100 m
  Let speed of winner athletic be 16x km/h. 1000 m
x + 15 − x 
= 300  Similarly, speed of athletic in second
 x( x + 15)  Q B gets injured at a distance of 450 m
  and third position be 15x and 11x, and his speed gets halved.
4500 respectively.
⇒ 1= 2 So, time taken by B to cover 1000 m
x + 15x Total distance travelled by winner is 4 km. 450 550
⇒ x2 + 15x − 4500 = 0 4 = + = 155
∴ Time = h 10 5
⇒ ( x + 75)( x − 60) = 0 16x Q Ratio of speed of A and B is equal to
∴ x = 60 km/h Distance travelled by second athletic in ratio of distance covered by A and B.
4 4 15 Usually (2-4) questions have been asked from this chapter. Generally questions are asked on Man-
54. (b) Total length of trains h= × 15x = km A : B = 1000 : 900 = 10 : 9
16x 16x 4 hours formulae, total wages paid for a work, number of days to complete a certain work and pipes
= 121 + 99 = 220 m 10 100
Similarly, distance travelled by third Now, speed of A = × 10 = m/s
9 9 and cisterns.
Relative speed of trains = ( 40 + 32) athletic in
km/h 4 4 11 Time taken by A to cover
h= × 11x = km 1000
= 72 km/h 16x 16x 4 1000 m = × 9 = 90 s
5 ∴ Difference between their distances 100
= 72 × m/s = 20 m/s
= 
18 15 11 Hence, A beat B by length of time In this chapter, we will study the relationship among the quantity of work given, wages given,
−  km
distance 220  4 4 = ( 155 − 90) = 65 s stipulated time, number of persons etc., and after it, we will be able to calculate the work in stipulated
∴ Time = = = 11 s
speed 20 = 1 km = 1000 m 65. (c) I. Let the length of race be x m. time by arranging some persons according to the work but before that lets discuss some basic rules.
55. (d) Let the distance be x. 59. (b) Refer to example 12. Then, distance covered by A = x m.
Now, ratio of time taken to travel the 60. (d) Petrol consumed by the bike Distance covered by B when A IMPORTANT RULES AND FORMULAE
distance by each bus is = ( 10 × 20) + (30 × 40) reaches the destination = x − 30
Rule 1 1
x x x
: : ⇒
12 12 12
: : + ( 10 × 20) mL Distance covered by C when A If a person can do a piece of work in ‘n’ days, then he will do
of the work in one day and if a
2 3 4 2 3 4 reaches the destination = x − 48 n
= ( 200 + 1200 + 200) mL x − 30
1
= 6: 4:3 Ratio of speed of B to A = person can do th of work in one day, then he will complete the work in n days.
= 1600 mL = 1. 6 L x n
56. (a) Speed of faster train is 1.5 km/min.
61. (d) Refer to question 53. x − 48 e.g. If Raj can do a piece of work in 20 days, then he will do 1/20th of the work in one day.
i.e. 90 km/h or 25 m/s Ratio of speed of C to A =
62. (c) Let the speed of the train = x km/h x
and speed of slower train = 60 km/h Note In the problems related to time and work it is always considered that a man/woman works at uniform rate.
Then, relative speed of train and distance covered by C when B
50
= m/s reaches the destination = x − 20 Rule 2 A and B can do a piece of work in x days and y days, respectively. Then, time taken by ( A + B) to
3 = ( x + 5) km/h x
Since, these trains are moving in same Ratio of speed B to C = xy
and length of the train x − 20 complete the work is equal to reciprocal of ( A + B)’s one day’s work, i.e. .
direction.
= 225 m = 0. 225 km [given] x − 30 x − 48  B C B  x+ y
= = ×
So, relative speed of train
Time taken by train to cross the man x x − 20  A A C 
=  25 −  m/s =
50 25 Rule 3 If A and B can complete a work in x days and A alone can finish that work in y days, then
m/s 0.225 ⇒ x2 − 50x + 600 = x2 − 48x
 3 3 = h
x+ 5 ∴ x = 300 m xy
Time taken by crossing slower from number of days required to complete the work by B = days
= 27 s
According to the question, II. The speeds of A,B and C are in the y−x
0.225 9 ratio
∴ Distance = speed × time = Rule 4 If A and B can do a piece of work in x days. B and C can do same work in y days, C and A can
x + 5 3600 300 : 270 : 252 = 50 : 45 : 42
25
= × 27 = 225 m ⇒ x + 5 = 90 2xyz
3 do same work in z days. Then, they will complete the same work in days by working
xy + yz + zx
together.
61 62 CDS Pathfinder

Note If A and B can do a piece of work in x days and y days,


EXAMPLE 1. Raj can do a piece of work in 20 days
and Rohan can do it in 12 days. How long will they
EXAMPLE 4. 15 men complete a work in 16 days. If
24 men are employed, then the time required to
respectively and they received ` k as wages by working
yk xk
PIPES AND CISTERNS
together, then share of A = ` and share of B = ` . Problems on pipes and cisterns are based on the basic
take if both work together? complete that work will be e 2014 I x+ y x+ y
concept of time and work. Pipes are connected to a tank or
1
a. 5 days
1
b. 7 days
1
c. 3 days
1
d. 9 days
a. 7 days b. 8 days c. 10 days d. 12 days Wages are directly proportional to the work done and cistern and are used t4o fill or empty the tank or cistern.
2 2 2 2 Sol. c. Let the work done be 1. indirectly proportional to the time taken by the individual.
Sol. b. Raj’s one day’s work =
1 Here, M1 = 15, D1 = 16, W1 = W2 = 1, M 2 = 24 and D2 = ? EXAMPLE 7. X completes a job in 2 days and Y Important Rules and Formulae
20 Now, according to the formula, completes it in 3 days and Z takes 4 days to complete Rule 9 If there are two pipes A and B takes ‘a’ and ‘b’ h
1
Rohan’s one day’s work = M1D1W2 = M 2D2W1 ⇒ 15 × 16 × 1 = 24 × D2 × 1 it. If they work together and get ` 3900 for the job, respectively to fill a tanker, then the two pipes together
12 15 × 16 240 then how much amount does Y get?
1 3+ 5 8 ⇒ D2 = ⇒ = = 10 days  1 1
∴ (Raj + Rohan)’s one day’s work =
1
+ = = 24 24 a. ` 1800 b. ` 1200 c. ` 900 d. ` 800 fill  +  part of the tank in 1h and time taken to fill
20 12 60 60
 a b
Therefore, 10 days are required to complete the work. Sol. b. Here, a = 2 days, b = 3 days and c = 4 days and  ab 
∴ Number of day’s taken by Raj and Rohan together to the tank will be   h. If a pipe fills a tank in ‘a’ h, then
Rule 6 If ‘m’ men or ‘n’ women can do a piece of k = ` 3900  a + b
60 1
complete the work = days or 7 days work in ‘a’ days, then x men and y women can do the Now, amount of y =
ac
×k [Rule 8]
8 2 1
1 ab + bc + ca in 1 h only of the tank is filled.
Here, x = 20 days and y = 12 days same work in days. 2× 4 a
x y = × 3900
xy + 2× 3+ 3× 4 + 4 × 2
∴Required time = [Rule 2]
x+ y m× a n × a 8 8 × 3900 EXAMPLE 10. Pipe A can fill a tank in 45 h and pipe
= × 3900 = = ` 1200
6 + 12 + 8 26 B can fill it in 36 h. If both the pipes are opened in the
20 × 12 20 × 12 15 1
= = = = 7 days EXAMPLE 5. If 3 men or 4 women can reap a field empty tank. In how many hours will it be full?
20 + 12 32 2 2 SOME OTHER FORMULAS
in 43 days. How long will 7 men and 5 women take to a. 10 h b. 15 h c. 20 h d. 28 h
EXAMPLE 2. A and B together can do a piece of reap it? 1. If A and B can do a piece of work in x and y days, Sol. c. Here, a = 45 h and b = 36 h.
a. 3 days b. 7 days c. 12 days d. 15 days respectively. A and B started working together but A left ab 45 × 36 1620
work in 12 days and A alone can do it 18 days. In how the work t days before completing the work, then time ∴ Required time = = = = 20 h
a + b 45 + 36 81
many days can B alone do it? Sol. c. Here, m = 3, n = 4, a = 43, x = 7 and y = 5 taken to complete the work will be
(x + t)y
days.
1 1 (x + y) Rule10 (i) If a pipe can fill a tank in ‘a’ h and another
a. 14 days b. 24 days c. 36 days d. 28 days ∴ Required days = = [Rule 6]
x y 7 5 can fill the tank in ‘b’ h but a third pipe
Sol. c. Here, x = 12 and y = 18 + + 2. A and B do a piece of work in ‘a’ and ‘b’ days,
m × a n × a 3 × 43 4 × 43 respectively. Both begin together but after some days, empties the filled tank in ‘ c’ h, then in one
xy 12 × 18
∴Time taken by B = = = 36 days [Rule 3]
=
1
=
1
= 12days
A leaves the work and the remaining work is  1 1 1
y − x 18 − 12 1  7 5 1 43 completed by B in x days. Then, the time after which A hour  + −  part of the tank will be
 +  × (b − x ) a  a b c

43 3 4  43 12 left is given by T =
EXAMPLE 3. A and B can do a piece of work in a +b filled (when all the three pipes are open) and
3 days. B and C in 9 days and A and C in 12 days. Find Rule 7 If A can do a work in x days and B can do y% fast 3. If A and B can do a piece of work in x and y days, time taken to fill the tank will be
the time in which A, B and C can finish the work, 100 x respectively. They start working together and after t abc
h.
working together. than A, then B will complete the work in days. days B leaves the work, then time taken to finish the bc + ac − ab
(100 + y) x
1 whole work will be × ( y − t ) days.
a. 3 days b. 3 days y (ii) If a pipe can fill a tank in a1 h and another
4 EXAMPLE 6. x can do a work in 16 days. In how pipe can empty the filled tank in a 2 h, then
15 many days will the work be completed by y, if the EXAMPLE 8. Akshu can do a piece of work in 10 1 1
c. 3 days d. None of these days and Harshal can do same work in 12 days. They in one hour  −  part of the tank will be
19 efficiency of y is 60% more than that of x? e 2013 II  a1 a 2 
started working together but Akshu left the work 2
Sol. c. Here, x = 3, y = 9 and z = 12 a. 10 days b. 12 days c. 25 days d. 30 days
days before completion of work, then time taken to filled. (when both pipes are open) and time
According to rule, Sol. a. Here, x = 16 days and y = 60% faster complete the work?  a a 
2xyz 100 x 100 × 16 1600 taken to fill the tank will be  1 2  h.
Required time taken by, A, B and C = [Rule 4] ∴ Required days = = = = 10 days 6
a. 6
3 3
days b. 5 days c. 4 days
2
d. 7 days  a 2 − a1 
xy + yz + zx 100 + y 100 + 60 160 11 10 2 5
2 × 3 × 9 × 12 6 × 12 72 15 [Rule 7] 6 12 × (10 + 2) 12 × 12 6 EXAMPLE 11. Two pipes ‘A’ and ‘B’ can fill a tank in
= = = =3 days Sol. a. 6 days = = =6 days
3 × 9 + 9 × 12 + 3 × 12 19 19 19 Rule 8 If A, B and C can do a piece of work in x, y and 11 10 + 12 22 11 36 min and 45 min, respectively. A waste pipe ‘C’ can
Rule 5 If two groups, M 1 persons of the first group can z days, respectively and they received ` k as wages by empty the tank in 30 min. In how much time the tank
EXAMPLE 9. A can do a piece of work in 10 days is full if all three pipes are opened?
do ‘W1 ’, work in ‘D1 ’ days working T1 h in a day earning working together, then and B can do same work in 15 days. They started
yz a. 60 min b. 90 min c. 115 min d. None of these
a sum of ` R1 and M 2 persons of the second group can do share of A = ` ×k working together but after 2 days, A left work. The
Sol. a. Here, a = 36, b = 45 and c = 30.
W2 work in D2 days working T2 h in a day earning a sum xy + yz + zx remaining work was completed by B alone, then time
taken to complete the work? So, time taken to fill the tank will be
of ` R2 . If each person of both group has the same xz
share of B = ` ×k 3 2 3 5 =
abc
[Rule 8]
efficiency of work, then xy + yz + zx a. 8 days b. 8 days c. 5 days d. 2 days ac + bc − ab
11 3 4 3
M 1 D1T1 M 2 D2T2 xy 2 10 26 2 36 × 45 × 30
= and share of C = ` ×k Sol. b. 8 days = (15 − 2) = = 8 days = = 60 min
W1 R1 W2 R2 xy + yz + zx 3 15 3 3 36 × 30 + 45 × 30 − 36 × 45
63 64 CDS Pathfinder

PRACTICE EXERCISE 17. If 6 men and 8 boys can do a piece of work in


10 days while 26 men and 48 boys can do the
same in 2 days, what is the time taken by 15
PREVIOUS YEARS’ QUESTIONS
25. 45 people take 18 days to dig a pond. If the pond
would have to be dig in 15 days, then the
men and 20 boys in doing the same type of work?
number of people to be employed will be e 2012 I
3 9. A and B can do a piece of work in 40 days and 50 (a) 4 days (b) 5 days (c) 6 days (d) 7 days
1. X can do of a work in 12 days. In how many (a) 50 (b) 54 (c) 60 (d) 72
4 days, respectively. Both begin together but after 18. Four taps can individually fill a cistern of water
days X can finish the 1/2 work? a certain time, A leaves off. In this case B 26. A and B can do a piece of work in 10 h. B and C
in 1h, 2h, 3h and 6h, respectively. If all the four can do it in 15 h, while A and C take 12 h to
(a) 8 days (b) 16 days (c) 12 days (d) 24 days finishes the remaining work in 20 days. After taps are opened simultaneously, the cistern can
how may days did A leave? complete the work. B independently can
2. A can do a piece of work in 10 days and B can do be filled in how many minutes? complete the work in e 2012 I
1
the same work in 12 days. How long will they (a) 14 days (b) 13 days (c) 13 days (d) 15 days (a) 20 (b) 30 (c) 35 (d) 40 (a) 12 h (b) 16 h (c) 20 h (d) 24 h
3
take to finish the work, if both work together? 19. 76 ladies complete a job in 33 days. Due to some
5 5 10. P and Q can do a job in 2 days, Q and R can do it 27. X can do a piece of work in 25 days. Y is 25%
(a) 5 days (b) 6 days reason some ladies did not join the work and more efficient than X. The number of days taken
11 11 in 4 days and P and R in 12/5 days. What is the therefore, it was completed in 44 days. The by Y is e 2012 II
1
(c) 5 days (d) None of these
number of days required for P alone to do the number of ladies who did not report for the work is (a) 15 days (b) 20 days (c) 21 days (d) 30 days
5 job?
(a) 17 (b) 18 (c) 19 (d) 20
(a) 5/2 (b) 3 (c) 14/5 (d) 6 28. A mason can build a tank in 12 h. After working
3. A, B and C working together take 30 min to 20. 9 men finish one-third work in 10 days. The for 6 h, he took the help of a boy and finished
address a pile of envelopes. A and B together 1
11. of a work is completed in half a day by number of additional men required for finishing the work in another 5 h. The time that the boy
would take 40 min, A and C together would take 48 the remaining work in 2 more days will be will take alone to complete the work is e 2013 I
45 min. How long would each take working 1
5 persons. Then, of the work can be completed (a) 78 (b) 81 (c) 55 (d) 30 (a) 30 h (b) 45 h (c) 60 h (d) 64 h
alone? 40
(a) A : 72 min, B : 90 min, C : 120 min by 6 persons in how many days? 21. Ravi and Sneha working separately can finish a 29. X can complete a job in 12 days. If X and Y work
job in 8 and 12 h, respectively. If they work for 2
(b) A : 42 min, B : 90 min, C : 120 min (a) 1 (b) 2 (c) 3 (d)
1 together, they can complete the job in 6 days. Y
2 an hour alternately, Ravi beginning at 9:00 am. 3
(c) A : 72 min, B : 90 min, C : 100 min alone can complete the job in e 2013 I
12. A garrison of ‘n’ men had enough food to last for When will the job be finished?
(d) A : 72 min, B : 80 min, C : 120 min (a) 10 days (b) 12 days (c) 15 days (d) 18 days
30 days. After 10 days, 50 more men joined (a) 7 : 30 pm (b) 7 : 00 pm (c) 6 : 30 pm (d) 6 : 00 pm
4. A and B together can do a piece of work in 12 them. If the food now lasted for 16 days, what is 22. Consider the following statements: 30. Pipe A can fill a tank in 10 min and pipe B can
days and A alone can do it in 36 days. In how the value of n? empty it in 15 min. If both the pipes are opened
many days can B alone do it? I. If 18 men can earn ` 1440 in 5 days, then in an empty tank, the time taken to make it
(a) 200 (b) 240 (c) 280 (d) 320 10 men can earn ` 1280 in 6 days.
(a) 18 days (b) 12 days full is e 2013 I
13. Two pipes A and B can fill a tank in 12 and II. If 16 men can earn ` 1120 in 7 days, then (a) 20 min (b) 25 min
(c) 15 days (d) 20 days
16 min, respectively. If both the pipes are opened 21 men can earn ` 800 in 4 days. (c) 30 min (d) None of these
5. Ram can do a piece of work in 6 days and Shyam simultaneously, after how much time should B be Which of the statement(s) given above is/are
can finish the same work in 12 days. How much closed so that the tank is full in 9 min? 31. 4 goats and 6 sheeps can graze a field in 50
correct? days. 2 goats and 9 sheeps can graze the field in
work will be finished, if both work together for (a) 3 min (b) 5 min (c) 4 min (d) 2 min (a) Only I (b) Only II e 2013 II
2 days?
14. 2 men undertake to do a job for ` 1400. One can (c) Both I and II (d) Neither I nor II (a) 100 days (b) 75 days (c) 50 days (d) 25 days
(a) One-fourth of the work (b) One-third of the work
do it alone in 7 days and the other in 8 days.
(c) Half of the work (d) Whole of the work Directions (Q. Nos. 23-24) If a1 men and b1 boys can
32. A can finish a work in 15 days, B in 20 days and
With the assistance of a boy they finish the work
C in 25 days. All these three worked together
6. A and B can do given work in 8 days; B and C can in 3 days. How should the money be divided? complete a work in x days, while a2 men and b 2
and earned ` 4700. The share of C is e 2013 II
do the same work in 12 days and A, B, C complete (a) ` 600, ` 525, ` 275 (b) ` 550, ` 500, ` 350 boys can complete the same work in y days, then
(a) ` 1200 (b) ` 1500 (c) ` 1800 (d) ` 2000
it in 6 days. In how many days can A and C finish (c) ` 650, ` 470, ` 280 (d) None of these One day work of 1 man (yb 2 − xb1)
= 33. 20 workers working for 5 h per day complete a
it?
15. A, B and C can do a piece of work individually One day work of 1 boy ( xa1 − ya2)
work in 10 days. If 25 workers are employed to
(a) 12 (b) 8 (c) 14 (d) 16
in 8, 10 and 15 days, respectively. A and B start 23. If 14 men and 12 boys can finish a work in 4 work 10 h per day, what is the time required to
7. A can finish a work in 8 days and B can do it in working but A quits after working for 2 days. days, while 8 men and 16 boys can finish the complete the work? e 2013 II
12 days. After A had worked for 3 days, B also After this, C joins B till the completion of same work is 5 days. Compare the 1 day work of (a) 4 days (b) 5 days (c) 6 days (d) 8 days
joins A to finish the remaining work. In how work. In how many days will the work be 1 man and 1 boy.
many days will the remaining work be finished? completed? 34. 18 men can earn ` 360 in 5 days. How much
1 1 money will 15 men earn in 9 days? e 2013 II
(a) 53/9 days (b) 34/7 days (a) 2 (b) 1 (c) (d) 3
(a) 2 days (b) 3 days 2 2 (a) ` 600 (b) ` 540 (c) ` 480 (d) ` 360
(c) 4 days (d) 5 days (c) 85/13 days (d) 53/10 days
24. 28 men and ‘m’ boys can finish a work in 4 days, 35. 2 men and 1 woman can complete a piece of
8. Two taps can fill a tub in 5 min and 7 min, 16. A can do a piece of work in ‘x ’ days and B can while 20 men and 6 boys can finish the same work in 14 days, while 4 women and 2 men can
respectively. A pipe can empty it in 3 min. If all do the same work 3x days. To finish the work in 5 days. Now, if the ratio of 1 day work of do the same work in 8 days. If a man gets ` 90
the three are kept open simultaneously, when work together they take 12 days. What is the 1 man and 1 boy is 1 : 2, what will be the value per day, what should be the wages per day of a
will the tub be full? value of ‘x’? of ‘m’? woman? e 2013 II
(a) 60 min (b) 85 min (c) 90 min (d) 105 min (a) 8 (b) 10 (c) 12 (d) 16 (a) 7 (b) 6 (c) 8 (d) 12 (a) ` 48 (b) ` 60 (c) ` 72 (d) ` 135
65 66 CDS Pathfinder

4. (a) Here, x = 12 and y = 36 1 1 1 14. (a) Let the boy completes the work in
36. A can do a piece of work in 4 days and B can 40. A and B are two taps which can fill a tank ∴ 1 min work of all taps = + −
xy 12 × 36 5 7 3 x days.
complete the same work in 12 days. What is the individually in 10 min and 20 min, respectively. ∴ Time taken by B = =
y − x 36 − 12 21 + 15 − 35 1 According to the condition,
number of days required to do the same work However, there is a leakage at the bottom, which = =
105 105 1 1 1 1 1 1 1 1
together? e 2013 II can empty a filled tank in 40 min. If the tank is [Rule 3] + + = ⇒ = − −
Hence, tap will be filled in 105 min, if 7 8 x 3 x 3 8 7
empty initially, then how much time will both 12 × 36
(a) 2 days (b) 3 days (c) 4 days (d) 5 days = = 18 days they work together. 1 56 − 21 − 24 11
the taps take to fill the tank with leakage? 24 ⇒ = =
37. A, B and C can do a piece of work individually in e 2015 II 9. (b) Here, a = 40 days, b = 50 days, x 168 168
1
8, 12 and 15 days, respectively. A and B start 5. (c) ∴ One day work of Ram = x = 20 and T = ? 168
(a) 2 min (b) 4 min (c) 5 min (d) 8 min 6 ∴ x= days
working but A quits after working for 2 days. ( b − x )a 11
1 ∴ Required time =
After this, C joins B till the completion of work. 41. If 4 men working 4 h per day for 4 days complete ∴ One day work of Shyam = a+ b So, money is to be shared in the ratio
4 units of work, then how many units of work 12 (50 − 20) × 40 30 × 40
In how many days will the work be completed? = = 1 1 11
Hence, one day work of, Ram and : : or 24 : 21 : 11
e 2014 II will be completed by 2 men working for 2 h per
Shyam ( 40 + 50) 90 7 8 168
8 6 7 3 day in 2 days? e 2015 II 40 1
(a) 5 days (b) 4 days (c) 6 days (d) 3 days 1 1 1 1 2+ 1 3 1 = = 13 days Thus,
9 7 13 4 (a) 2 (b) 1 (c) (d) = + = = = 3 3 A ’s amount =
24
× 1400 = ` 600
2 8 6 12 12 12 4 12 56
38. A is thrice as efficient as B and hence completes 1 10. (b) Here, x = 2, y = 4 and z =
a work in 40 days less than the number of days 42. If m persons can paint a house in d days, then ∴ Two day’s work = 5 B ’s amount =
21
× 1400 = ` 525
2
taken by B. What will be the number of days how many days will it take for ( m + 2) persons to ∴ Time taken by ( P + Q + R) 56
paint the same house? e 2015 II Thus, if they work together for 2 day’s, 2xyz 11
taken by both of them when working together? = [Rule 4] Boy’s amount = × 1400 = ` 275
m+ 2 md then half of the work will be complete.
xy + yz + zx 56
e 2014 II (a) md + 2 (b) md − 2 (c) (d)
md m+ 2 6. (b) Here, x = 8, y = 12, z = 12 1
(a) 22.5 days (b) 15 days (c) 20 days (d) 18 days 12 15. (d) A’s work in one day = , B’s work in
Now, time taken by ( A + B + C ) 2×2×4× 8
39. The efficiency of P is twice that of Q, whereas 43. Two pipes A and B can fill a tank in 60 min and = 5 1
2xyz one day =
the efficiency of P and Q together is three times 75 min, respectively. There is also an outlet C. If = [Rule 4] 2×4+ 4×
12 12
+ ×2 10
A, B and C are opened together, then the tank is xy + yz + zx 5 5
that of R. If P , Q and R work together on a job, C’ work in one day =
1
full in 50 min. How much time will be taken by 2 × 8 × 12 × z 192
in what ratio should they share their earnings? = 15
e 2015 I C to empty the full tank? e 2016 I ⇒ 6= 40 + 48 + 24
8 × 12 + 12 × z + z × 8 ( A + B )’s work in one day
(a) 2 : 1 : 1 (b) 4 : 2 : 1 (c) 4 : 3 : 2 (d) 4 : 2 : 3 (a) 100 min (b) 110 min (c) 120 min (d) 125 min 192 12 1 1 5+ 4 9
= = days = + = =
⇒ 96 + 20z = 32z 112 7 8 10 40 40
⇒ 96 = 12z 2×9 9
Now, P one days work ( A + B )’s work in two days = =
ANSWERS ⇒ z=8
=
7 1 7−3
− = 9
40
11
20
So, A and C can do the work in 8 days. 12 4 12 Remaining work = 1 − =
1 20 20
1 a 2 a 3 a 4 a 5 c 6 b 7 b 8 d 9 b 10 b 7. (b) One day’s work of A = 4 1
= = ( B + C )’s work in one day
8 12 3
11 d 12 a 13 c 14 a 15 d 16 d 17 a 18 b 19 c 20 b 1 1 1 3+ 2 5 1
One day’s work of B = Hence, P alone can do in 3 days. = + = = =
21 c 22 d 23 a 24 b 25 b 26 d 27 b 28 c 29 c 30 c 12 10 15 30 30 6
MD MD
3 11. (d) Using the formula 1 1 = 2 2 Since, ( B + C ) complete the work in 6
31 d 32 a 33 a 34 b 35 b 36 b 37 a 38 b 39 a 40 d 3 day’s work of A = W W
8 1 2 days.
3 11
41 c 42 d 43 a Remaining work of A = 1 − 12. (a) By given condition, ∴ work will be completed in
8 20
n × 30 = n × 10 + ( n + 50 ) × 16 11 11 × 3 33
5 6× = =
= ⇒ 20 n = 16 n + 800
days
8 20 10 10
800 ∴ Total number of days
HINTS AND SOLUTIONS One day’s work of A and B together

= +
1 1 3+ 2
= =
5
∴ n=
4
= 200
= 2+
33 20 + 33 53
= = days
13. (c) Here, x = 12, y = 16 and t = 9 10 10 10
8 12 24 24
3 5 1 1 4 −3 1 Two pipes A and B can fill a tank in x h 16. (d) 1 day work of A =
1
1. (a) Since, X can do of work in =5 days = − = = Number of days to finish the work
4 11 30 40 120 120 and y h, respectively. If both the pipes ax x
5 5
12 days. [Rule 1] A’ one min’s work = ( A + B )’s one min = ÷ = 3 days opened simultaneously, then the time 1
3. (a) B’s one min’s work 8 24 1 day work of B =
12 × 4 work − B’s one min work after which B should be closed, so that 3x
So, X can do 1 work in days. = ( A + B + C )’s one min’s work
3 1 1 9−4 1 the tank is filled in
= − = 8. (d) 1 min work of first tap = ∴ 1 day work of both A and B
1 12 × 4 × 1 − ( A + C )’s one min’s work 40 90 360 5 th = [ y( 1 − t / x )]n
∴X can do work in = 8 days 1
= +
1
=
4
2 3×2 1 1 6−4 2 1 5 1 1 Required time after which B should be
= − = = = = = 1 min work of second tap = x 3x 3x
closed = y  1 −  = 16 1 − 
2. (a) Here, x = 10 and y = 12 30 45 180 180 90 360 72 7 t 9
given, one day work of both A and B
∴ Number of days taken by A and B Hence, A, B and C alone can finish the 1  x  12 
C ’s one min’s work = ( A + B + C )’s one and 1 min work of third = − =
1
xy 12 × 10 work in 72 min, 90 min and 120 min, 3 3
= = [Rule 2] min work − ( A + B )’s one min work = 16 × = 4 min 12
respectively.
x + y 12 + 10 12
67 68 CDS Pathfinder

4 1 1 3600 32. (a) Here, x = 15 days, y = 20 days and 1 x x−2 Part of tank emptied by leakage in 1 min
⇒ = ⇒ 3x = 48 ⇒ x = 16 21. (c) Work done by Ravi in one hour = = =8h ⇒ + + =1
3x 12 8 450 z = 25 days and K = `4700 4 12 15 1
=
Hence, the value of x is 16. 1 1 1   15 + 5x + 4( x − 2) 40
Work done by Sneha in one hour = So, 1 h work of B = − Share of C = ` 
kxy
 ⇒ =1
12 8 12  xy + yz + zx  60
17. (a) Given, 6 M + 8 B = 10 days ...(i) Part of tank filled by both taps and
3−2 1 ⇒ 15 + 5x + 4x − 8 = 60 ⇒ 9x + 7 = 60
leakage is 
Total work done by them in 2 h = = 4700 × 15 × 20 1 1 1 1
and 26 M + 48 B = 2 days ... (ii) =` 53 8 + −  =
1 1 5 24 24 ⇒ 9x = 53 ⇒ x = =5  10 20 40  8
15 M + 20 B = ? = + = 15 × 20 + 20 × 25 + 25 × 15
8 12 24 Hence, B alone can do in 24 h. 9 9
4700 × 15 × 20 Hence, total time taken to fill the tank is
Here, M D = M D =` = ` 1200 Hence, the work will completed in
1 1 2 2 Work done is 4 pairs of hours 27. (b) Here, x = 25 days and y = 25 1175 8 8 min.
⇒ ( 6 M + 8 B ) × 10 = ( 26 M + 48 B ) × 2 5 5 100x 100 × 25 5 days.
= ×4= ∴ Required days = = 33. (a) Refer to Question No 11 9 41. (c) Here, M = 4,T = 4,W = 4,D = 4
⇒ 60 M + 80 B = 52 M + 96 B 24 6 100 + y 100 + 25 1 1 1 1
[Using Rule 5] 38. (b) Let efficiency of A be x days. and M = 2, T = 2, D = 2, W = ?
⇒ 8M = 16B Remaining work after 8 hours 2500 2 2 2 2
= = 20 days 34. (b) Refer to Question No 22 Then, efficiency of B = 3x days. Using the formula,
∴ M = 2B 5 1 125
= 1− = 1 M ×T ×D M ×T ×D
Then, 15 M + 20 B = 15 × 2 B + 20 B 6 6 1 35. (b)Q 14 days work of 2 men and 1 women ∴ B’s one day work = 1 1 1 = 2 2 2 [Rule 5]
28. (c) Mason work for 1 h = 3x W W
= 30 B + 20 B =50 B Now, its Ravi’s turn 12 = 8 days work of 4 women and 2 men 1 2
1
On putting the value of M in Eq. (i), we 1 1 1 6 1 ⇒ 1 day work of 28 men and 14 women and A’ s one day work = 4×4×4 2×2×2
So, work left after 9 hours = − = Mason Work for 6 h = = x ⇒ =
get 6 8 24 12 2 = 1day work of 32 women and 16 men Again, let time taken by B = t days 4 W
⇒ 6 × 2B + 8 B = 10 days 1 Work left = 1 −
1
=
1 2

⇒ 12 B + 8 B = 10 days work will be done by sneha in


2 2 ⇒ 28 men + 14 women Then, time taken by A = ( t − 40) days 8 1
24 = 32 women + 16 men ⇒ W = =
∴ 20 boys finish the work in 10 days Now, let the boy can finish the work t t − 40 2
16 2
12 ×
1
h = 30 min ⇒ 28 m − 16 m = 32 w − 14 w Now, = ⇒ t = 3( t − 40)
One boy finish the work in 24 in x h. 3x x 42. (d) m persons paint a house in d days.
10 × 20 days = 200 days 1 1 x + 12 m 18 3
Total time taken = 9 h and 30 min Then, their 1 h work = + = ⇒ 12 m = 18 w ⇒ = = ⇒ 3t − t = 120, t =
120
= 60 days ∴ 1 person paints a house in
200 w 12 2
and 50 boys finish the work in days 12 x 12x 2 (m × d ) days.
Required time = 6 : 30 pm
50 x + 12 1 5x + 60 1 So, efficiency of 1 man and 1 woman is So, B complete the work in 60 days and
∴ ×5= ⇒ = and m + 2 persons paint a house in
= 4 days 22. (d) I. 18 men can earn in 5 days = ` 1440 12x 2 12x 2 3 : 2. A complete the work in 20 days.
1440 So, their wages must be in the same ratio  
18. (b) Part filled by first tap in 1h = 1 1 man can earn in 1 day = ` ⇒ 10x + 120 = 12x ⇒ 120 = 2x ( A + B )'s 1 day work  md  days.
18 × 5 90 3 m + 2
1 ∴ x = 60 h ∴ = 1 1 4 1  
Part filled by second tap in 1h = x 2 = + = =
2 ∴ 10 men can earn in 6 days 60 20 60 15
1
1 1440 29. (c) X ’s one day work = and ( X + Y )’s [where, x = wages per day of a woman] 43. (a) Since, two pipes A and B fill a tank in
Part filled by third tap in 1h = = × 6 × 10 12 Hence, they will complete the work in
18 × 5 3 90 × 2 60 min and 75 min, respectively.
3 ∴ x= = ` 60 15 days when they work together.
one day work = ∴ Part of tank filled by pipe A in 1 min
Part filled by fourth tap in 1h =
1 = ` 960 ≠ ` 1280 20 3 39. (a) Let efficiency of P be x days.
1 1
6 II. 16 men can earn in 7 days = ` 1120 ∴ Y ’s one day work 36. (b) A’s one day work = Then, efficiency of Q = 2x days =
Total tank filled by all taps in 1h 1120 3 1 4 1 4 60
1 man can earn in 1 day = = − = = 1 ∴ Q’s one day work =
1
1 1 1 16 × 7 20 12 60 15 and B’s one day work = and part of tank filled by pipe B in 1 min
= 1+ + + 12 2x 1
2 3 6 ∴ 21 men can earn in 4 days ∴ Number of day’s taken by Y to 1 =
One day work of A and B together and P ’s one day work = 75
∴ Required time =
1 1120 complete the work = 15 days
= × 21 × 4 = ` 840 ≠ ` 800 1 1 3+ 1 4 1 x
1 1 1
1+ + + 16 × 7 1 = + = = = Now, part of tank filled by A and B
30. (c) Part filled by pipe A in 1 min = 4 12 12 12 3 Now, ( P + Q )’s one day work together in 1 min
2 3 6 So, neither statement I nor II is correct. 10 1 1 3
6 6 6 Days required by A and B together to do = + = =
1
+
1
=
9
=
3
= = h= × 60 min 23. (a) Here, a = 14, b = 12, x = 4, and part emptied by pipe B in 1 min x 2x 2x
6 + 3 + 2 + 1 12 12 1 1 1 the work 60 75 300 100
a = 8, b = 16 and y = 5 = =
1 ∴( P + Q ) will complete the whole work
2 2 15 2x Let part of tank emptied by pipe C in 1
= 30 min One day work of A and B together
One day work of 1 man ( yb2 − xb1 ) ∴ Total tank filled in minutes
in days.
min = .
1
= 3
19. (c) Given, M = 76, D = 33 One day work of 1 boy ( xa − ya ) 1 1 3−2 1 = 3 days C
1 1 1 2 = − = = 1 According to the question,
Let number of ladies who did not 10 15 30 30 37. (a) Work done by A in one day = So, net part of tank filled by pipes A, B
report for the work = x 5 × 16 − 4 × 12 R will complete this work = 2x days
= =2 Hence, the tank will be filled in 30 min.
8 and C together in 1 min
4 × 14 − 5 × 8 1 1
By given condition, Work done by B in one day = ∴ R’ s one day work = =
3
− =
1 1
[Given]
24. (b) Refer to Question No 23. 31. (d) Here, m = 4, n = 6, a = 50 and x = 2, 12 2x
M = 76 − x and D = 44 y=9 1 100 C 50
2 2
25. (b) Using the formula, M D = M D and work done by C in one day = 1 1 1
Required ratio = : :
1 1
= 1: : 1 3 1 1
We know that, 1 1 2 2
∴ Required days = x
1 15 x 2x 2x 2 2 ⇒ = − =
MD =MD 26. (d) Here, x = 10, y = 15, z = 12. y C 100 50 100
1 1 2 2
[Rule 5] + Let the work will be completed in = 2 : 1: 1
∴ 76 × 33 = ( 76 − x) × 44 ∴ Time taken by ( A + B + C ) m× a n× a x days. ∴ C = 100
1
76 × 3 2xyz =
1 2 x x−2 40. (d) Part of tank filled by A in 1 min = Hence, the time taken by pipe C to
⇒ 76 − x = = 19 × 3 ⇒ x = 19 = [Rule 4] 2 9
[Rule 6] Then, + + =1 10
4 xy + yz + zx + 8 12 15
1
empty the tank is 100 min.
4 × 50 6 × 50 Part of tank filled by B in 1 min =
Hence, the number of ladies is 19. 2 × 10 × 15 × 12 1 1 100 20
= = = = = 25 days
20. (b) Refer to Question No 11 1 3 4
10 × 15 + 15 × 12 + 12 × 10 + 4
[Using Rule 5] 100 100 100
08
69 70 CDS Pathfinder

EXAMPLE 2. A person’s salary has increased from


Rule 8 If the value of an object is first changed
` 7200 to ` 8100. What is the percentage increase in
his salary? e 2013 II
(increased or decreased) by x% and then changed (increased
2 1 or decreased) by y% , then
a. 25% b. 18% c. 16 % d. 12 % ( ± x)( ± y)

3 2 Net effect =  ± x ± y + %
8100 − 7200  100 
Sol. d. Percentage increase in salary = × 100%
7200 Here, + sign is used in case of increment and − sign is used
900 1
= × 100% = 12.5% = 12 % in case of decrement.
7200 2

PERCENTAGE
Rule 5 If A is x% more than ‘B’, then EXAMPLE 5 The price of an article is first increased by
 x  20% and later on the price were decreased by 25% due
B would be  × 100 % less than ‘A’. to reduction in sales. Find the net percentage change in
(100 + x)  final price of article.
Rule 6 If A is x% less than ‘B’, then B would be a. 20% b. 10% c. 30% d. 15%
Sol. b. Here, a = 20 % , b = 25 %
 x 
(100 − x) × 100 % more than A.
10
So, required percentage = × 100 = 10%
  100
Generally (2-3) questions have been asked from this chapter. Generally questions which are asked from this  ( ± a) ( ± b) 
Required change = ( ± a) + ( ± b) + %
chapter are tricky but you can easily solve them by using the short-tricks formulae in very less time. EXAMPLE 3. Raj get 10% less marks than Rohit in  100 
an examination. What percentage of marks does
Rohit gets more than Raj?  20 × ( −25)   positive sign for increase 
= 20 − 25 + %
1 1 1  100  negative sign for decrease
a. 11 % b. 11 % c. 12 % d. None of these
6 9 9 = [ −5 − 5]%
PER CENT Sol. b. Given, x = 10%
10
= −10 %
∴ Rohit more percentage than Raj = × 100 ∴Net percentage change is a decrease of 10 % because
Per cent is a fraction whose denominator is 100 and numerator of the fraction is called the rate per 100 − 10 final result is negative.
cent. Per cent is denoted by the symbol ‘%’. 10 × 100 1
= = 11 % Rule 9 If the population of a town is P and it increases
Important Rules and Formulae 90 9 (or decreases) at the rate of R% per annum, then
Rule 7 If the price of a commodity is increased
n
a
(i) Population after n yr = P 1 ±
Rule 1 For expressing a% as a fraction, we can write a % = , i.e. divide ‘a’ by 100 and reduce it to (or decreased) by x%, then the decrease (or increase) in R

100 consumption, so as not to increase or (decrease) the  100 
25 1
the lowest form, e.g. 25% = =  x  P
100 4 expenditure is  × 100 %. (ii) Population, n yr ago =
 100 ± x  n
1 ± R 
a a a
=  × 100 %  
Rule 2 For expressing a fraction as a per cent, we can write  100 
b b b  EXAMPLE 4. If the price of the cooking gas rises
2 2 200
by 15%, by what per cent should a family reduce its
e.g. = × 100% = % consumption so as not to exceed the budget on EXAMPLE 6 The population of a town is 352800. If it
3 3 3 cooking gas? increases at the rate of 5% per annum, then what will
1 1 1 be its population 2 yr hence, also find the population
Rule 3 To find how much per cent one quantity is of another quantity, we can write a. 12 % b. 13 % c. 14 % d. None of these 2 yr ago.
23 23 23
The quantity to be expressed in per cent a. 315000 b. 316500 c. 200045 d. 320000
Required percentage = × 100% Sol. b. Let initial price of cooking gas be ` 100.
2nd quantity (in respect of which the per cent has to be obtained) Price after increase = ` 115
Sol d. Given that, p = 352800, R = 5% and n = 2
According to the formula,
On ` 115 he should reduce ` 15 on ` 100, he should
EXAMPLE 1. 20g is what per cent of 1000g? R n 5 2
reduce Population after 2 yr = P 1+ 
 = 352800 × 1+ 
a. 2% b. 5% c. 7% d. 9% 15 1  100  100
20 = × 100 = 13 %
 100 + 5 = 352800 ×  21 × 21 = 388962
2
Sol. a. Required percentage = × 100% = 2% 115 23
1000 = 352800 ×    
Shortcut Method  100   20 20
Rule 4 Percentage increase or decrease in a quantity when it increases or decreases by some value. Here, x = 15% Population 2 yr ago
 x 
Increase or decrease in the value ∴Reduction in consumption =  × 100 % P 352800 20 20
 100 + x  = n = = 352800 × × = 320000
Percentage = × 100 % 1 R  1 5 2 21 21
Initial value 15 1  +   + 
= × 100% = 13 %  100  100
115 23
71 72 CDS Pathfinder

Rule 10 If the present population of a city is P and there an increment of 45%, then find the population of city 27. Two numbers are less than a third number by
16. If after 24% of wastage the net output of a
is a increment or decrement of R1 %, R2 % and R3 % in at the end of year 2007. coal-mine is 68400 quintals. Then, the total 30% and 40%, respectively. How much per cent
first, second and third year respectively, then a. 1083875 b. 1083000 c. 1089000 d. 1135000 output of the coal-mine in quintals is is the second number less than the first?
2
Population of city after 3 yr Sol a. Given that, P = 1000000 , R1 = 15 %, R2 = 35% (a) 70000 (b) 90000 (c) 80000 (d) 89000 (a) 35% (b) 36% (c) 14% (d) 14 %
7
(decrease) and R3 = 45% 17. A rise of 25% in the price of grapes compels a
 R  R  R  Population of city at the end of year 2007. 28. A sample of 5 L of glycerine is formed to be
= P 1 ± 1  1 ± 2  1 ± 3  person to buy 1.5 kg of grapes less for ` 240.
 100   100   100   R   R   R  Then,the original price of grapes per kg is adulterated to the extent of 20%. Find how
= P 1+ 1  1− 2  1+ 3  glycerine should be added to bring down
 100  100  100 (a) ` 40 (b) ` 32 (c) ` 30 (d) ` 28
Note Use ‘+’ sign for increases and ‘–’ sign for decreases. percentage of impurity to 5%?
= 1000000 1+
5   35   45 
 1−  1+  18. The price of an item is increased by 20% and (a) 10 L (b) 25 L (c) 15 L (d) 20 L
 100  100  100 then decreased by 20% the final price as
EXAMPLE 7 Population of a city in 2004 was 29. The daily wages of a worker increase by 20% but
1000000. If in 2005 there is an increment of 15%, in 115 65 145 compared to original price is
= 1000000 × × × the number of hours worked by him also dropped
2006 there is a decrement of 35% and in 2007 there is 100 100 100 (a) 4% less (b) 4% more (c) 20% less (d) 20% more by 20%. If originally he was getting ` 200 per
= 1083875 19. Sohan saves 14% of his salary while George week, his wages per week now is
saves 22%. If both gets the same salary and (a) ` 160 (b) ` 192 (c) ` 210 (d) ` 198
George saves ` 1540. Then, the salary of each of 30. 140 L of a liquid contains 90% of acid and the
PRACTICE EXERCISE them is
(a) ` 9500 (b) ` 17000 (c) ` 7000 (d) ` 7500
20. A’s salary is half that of B. If A got a 50% rise in
rest water. How much water must be added to
make the water 12.5% of the resulting mixture?
(a) 4 L (b) 10 L (c) 12 L (d) 3 L
his salary and B got a 25% rise in his salary, 31. A person spends 30% of monthly salary on rent,
1. If 90% of A = 30% of B and B = x % of A, the value 10. To pass an examination, a candidate needs 40% then the percentage increase in combined 25% on food, 20% on children’s education and
of x is marks. All questions carry equal marks. A salaries of both is 12% on electricity and the balance of ` 1040 on
(a) 700 (b) 600 (c) 300 (d) 1100 candidate just passed by getting 10 answers (a) 13%
1
(b) 33 % (c) 33% (d) 45% the remaining items. What is the monthly salary
correct by attempting 15 of the total questions. 3 of the person?
2. When 40% of a number is added to 42, the result How many questions are there in the
is the number itself. The number is examination?
21. A man donated 4% of his salary to a charity and (a) ` 8000 (b) ` 9000 (c) ` 9600 (d) ` 10600
(a) 70 (b) 90 (c) 82 (d) 72 deposited 10% of the rest in the bank. If now he 32. In an office, 40% of the staff is female and rest is
(a) 25 (b) 30
(c) 40 (d) 45
has ` 10800, then his income was male. 60% of the male and 40% of female voted for
3. If x% of y is 13x, then the value of y is (a) ` 13500 (b) ` 14500 Ramesh. The percentage of votes Ramesh got was
(a) 880 (b) 1300 (c) 1200 (d) 700 11. If salary of X is 20% more than salary of Y, then by (c) ` 40000 (d) ` 12500 (a) 24% (b) 42% (c) 50% (d) 52%
how much percentage is salary of Y less than X ?
4. What is the number when 20% of number is 30% 22. Rohit saves 30% of his salary. When his 33. In a class-X of 30 students, 24 passed in first
(a) 25 (b) 20
of 40? expenses increased by 30%, he is able to save class; in another class-Y of 35 students, 28
50 65
(a) 90 (b) 80 (c) 60 (d) 50 (c) (d) ` 1215 per month. His monthly salary is passed in first class. In which class was the
3 4
5. What is the number when increased by 20% (a) ` 13500 (b) ` 14500 percentage of students passed first class more?
2
becomes 300? 12. Water contains 14 % of hydrogen and the rest is (c) ` 30000 (d) ` 12500 (a) Class-X (b) Class-Y
7 (c) Both X and Y (d) None of these
(a) 250 (b) 200 (c) 180 (d) 280 23. A man spends 75% of his income. If his income is
oxygen. In 350 g of water, oxygen will be
6. If 50% of ( x − y ) = 40% of ( x + y ), then what (a) 300 g (b) 250 g increased by 20% and he increased his expenditure
Directions (Q. Nos. 34-36) The population of the
percent of x is y? (c) 200 g (d) None of these by 10%. His savings percentage is increased by
town is 126800. It increases by 15% in the 1st year
1 1 1 1 (a) 25% (b) 50% (c) 75% (d) 10%
(a) 10 % (b) 11 % (c) 13 % (d) 21 % 13. The income of ‘A’ is 20% higher than that of ‘B’. due to increase crime in the city.
9 9 9 9
The income of ‘B’ is 25% less than of C. What per 24. The price of wheat has increased by 60%. In
order to restore to the original price, the new 34. What is the population of the town at the end of
7. Two candidates fought an election one get 65% of cent less is A ’s income from ‘C’s income? 2nd year if the population decreases by 20% in
the votes and won by 300 votes. The total (a) 7% (b) 8% price must be reduced by
(a) 37.5% (b) 33% (c) 34% (d) 40% the second year?
number of votes polled in the election is (c) 10% (d) 12.5%
(a) 174984 (b) 135996 (c) 116656 (d) 145820
(a) 700 (b) 950 (c) 1000 (d) 900 14. 38 L of milk was poured into a tub and the tub 25. If the numerator of a fraction increased by 20%
1 and its denominator be diminished by 10%. The 35. What is the population of the town, if the population
8. A man spends ` 3500 and saves 12 % of his was found to be 5% empty. To completely fill the 16 decrease by 15% at the end of 2nd year?
2 tub, what amount of additional milk must be value of the fraction is , then the fraction is
27 (a) 123749 (b) 123479
income. His monthly income (in `) is poured?
(a) 4/9 (b) 3/2 (c) 3/8 (d) 9/4 (c) 123947 (d) None of these
(a) 4000 (b) 3800 (c) 4200 (d) 4500 (a) 1 L (b) 2 L (c) 3 L (d) 4 L
26. 1 L of water is evaporated from 6 L of a solution 36. What is the population of the town, if the
9. In an examination 52% of the candidates failed 15. 10% of the inhabitants of a certain city left that having 4% of sugar. The percentage of sugar in population increases by 20% at the end of 2nd
in English, 42% in Mathematics and 17% in city. Later on 10% of the remaining inhabitants
the remaining solution is year?
both. The number of those who passed in both of that city again left the city. What is the 4 4
(a) 4% (b) 5% (c) 4 % (d) % (a) 194784 (b) 174984 (c) 179484 (d) 178494
the subjects is remaining percentage of population of that city? 5 5
(a) 23% (b) 40% (c) 53% (d) 33% (a) 80% (b) 80.4% (c) 80.4% (d) 81%
MATHEMATICS Percentage 73 74 CDS Pathfinder

PREVIOUS YEARS’ QUESTIONS


37. A man losses 20% of his money. After spending
44. A person could save 10% of his income. But 2 yr
later, when his income increased by 20%, he HINTS AND SOLUTIONS
could save the same amount only as before. By
25% of the remainder, he has ` 480 left. What is how much percentage has his expenditure
the amount of money he originally had? e 2012 I 1. (c) Given, 90% of A = 30% of B 8. (a) Let monthly income be ` x. Now, difference between C’ s salary and
increased? e 2015 I 90 A 30B 1 A ’ s salary
(a) ` 600 (b) ` 720 (c) ` 800 (d) 840 2 1 = [Rule 1] ⇒ 87 % of x = ` 3500
(a) 22 % (b) 23 % 100 100 2 9x x
= x− =
38. The price of an article is ` 25. After two 9 3 A 3 175 10 10
2 2 ⇒ = ⇒ B = 3A ⇒ x = ` 3500
successive cuts by the same percentage, the price (c) 24 % (d) 25 % B 9 2 × 100 ∴ The required percentage
9 9 xA
becomes ` 20.25. If each time the cut was x%, Now, B = x % of A, 3 A = [Rule 1] x
100 3500 × 2 × 100 = × 100% = 10%
then e 2012 II 45. 20% of a number when added to 20 becomes the ∴ x= = ` 4000 10 × x
(a) x = 9 (b) x = 10 (c) x = 11 (d) x = 115
. number itself, then the number is e 2015 II
∴ x = 300 175
Hence, the value of x is 300. 9. (a) Number of candidates who passed in 14. (b) Amount of milk in tub = 38 L
(a) 20 (b) 25 (c) 50 (d) 80
39. What is 5% of 50% of 500? e 2012 II
2. (a) Let the number be x, both the subject = 100 − ( x + y − z )% Q Tub found to be 5% empty
(a) 12.5 (b) 25 (c) 1.25 (d) 6.25 46. A’s salary was increased by 40% and then then, 42 + 40% of x = x = 100 − (52 + 42 − 17)% ∴ Total quantity of milk in per cent
40. X , Y and Z had taken a dinner together. The cost decreased by 20%. On the whole A’s salary is 40 = 95%
increased by e 2015 II ⇒ x = 42 + x [Rule 1] = 23%
of the meal of Z was 20% more than that of Y 100 Now, completely fill the tub, total
(a) 60% (b) 40% (c) 20% (d) 12% 10. (a) Let the total number of questions in
and the cost of the meal of X was 5/6 as much as ⇒
3
x = 42 amount of additional milk
examination be x.
the cost of the meal of Z. If Y paid ` 100, then 47. In an election 10% of the voters on the voter list 5 38 × 5
what was the total amount that all the three of 42 × 5 By given condition, 40% of x = 10 = = 2L
did not cast their vote and 60 voters cast their ∴ x= = 70 95
x × 40 1000
them had paid? e 2013 II ballot papers blank. There were only two 3 ⇒ = 10 ⇒ x = = 25
(a) ` 285 (b) ` 300 100 40 15. (d) Remaining percentage
candidates. The winner was supported by 47% of Hence, the required number is 70.
=  1 −
10   10 
(c) ` 355 (d) None of these total voters in the voter list and he got 308 votes 3. (b) Given, x% of y = 13x
11. (c) Let the salary of y be ` x .  1− 
 100   100 
more than his rival. The number of voters on the x 120x 6x
41. A water pipe is cut into two pieces. The longer ⇒ y = 13x [Rule 1] ∴ Salary of x = ` =` 90 90
piece is 70% of the length of the pipe. By how voter list is e 2015 II 100 100 5 = × = 81%
(a) 3600 (b) 6200 (c) 6028 (d) 6400 ∴ y = 13 × 100 = 1300 100 100
much percentage is the longer piece longer than Difference of their salaries
16. (b) Let total output be x quintal.
= `  − x = `
the shorter piece? e 2013 II 48. The salary of a person is increased by 10% of his 4. (c) Let the number be x. 6x x
(a) 140% (b)
400
% original salary. But he received the same amount By given condition, 20% of x  5  5 ∴ Useful output = 100 − 24 = 76%
3 76
even after increment. What is the percentage of = 30% of 40 ∴ Required percentage ⇒ x = 68400
(c) 40% (d) None of these 20x 30 × 40 100
his salary he did not receive? e 2016 I
⇒ = ⇒ 20x = 30 × 40 =
x/5 50
× 100% = %
42. If m% of m + n % of n = 2% of ( m × n ), then what 68400 × 100
(a) 11% (b) 10% (c)
100
% (d)
90
% 100 100 6x / 5 3 ⇒ x=
percentage of m is n? e 2014 II 11 11 ∴ x = 60 76
[Rule 3]
(a) 50% (b) 75% Hence, the required number is 60. ∴ x = 90000 quintal
49. The expenditure of a household for a certain Shortcut Method
(c) 100% (d) Cannot be determined 5. (a) Let the number be x. Here, x = 20% 17. (b) Let original price of grapes be ` x.
month is ` 20000, out of which ` 8000 is spent 20x
43. The price of a commodity increased by 5% from ∴ x+ = 300 ∴ Required percentage 25
on education, ` 5900 on food, ` 2800 on ∴ Increased price = x + x
100
=  × 100 % [Rule 5]
2010 to 2011, 8% from 2011 to 2012 and 77% from shopping and the rest on personal care. What x 100
300 × 100
2012 to 2013. What is the average price increase percentage of expenditure is spent on personal ⇒ x= = 250  100 + x  125
=` x …(i)
120  × 100 %
(approximate) from 2010 to 2013? e 2014 II care? e 2016 I =
20 100
(a) 26% (b) 32% (c) 24% (d) 30% (a) 12% (b) 16.5% (c) 18% (d) 21.8% 6. (b) Given, 50% of ( x − y ) = 40% of ( x + y )  100 + 20  Increased price of 1.5 kg of grapes
50 40 2000 50
× (x − y) = × (x + y) = %= % 25
100 100 = × 240 = ` 60
120 3 100
⇒ 5x − 5 y = 4x + 4 y [Rule 1] 12. (a) Percentage of oxygen in water 60
ANSWERS ⇒ x = 9y
r
… (i) 2
= 100 − 14 = 100 −
100 5
= 85 %
∴ Increased price of 1 kg =
1.5
Let r % of x = y ⇒ ×x= y 7 7 7
1 c 2 a 3 b 4 c 5 a 6 b 7 c 8 a 9 a 10 a
100 = ` 40 …(ii)
r
⇒ × 9y = y [from Eq. (i)] Percentage of oxygen in 350 g of water
100 So, from Eqs. (i) and (ii), we have
11 c 12 a 13 c 14 b 15 d 16 b 17 b 18 a 19 c 20 b 5 600 350
100 1 = 85 % of 350 = × = 300 g 125
∴ r= = 11 % 7 7 100 x = ` 40
21 d 22 a 23 b 24 a 25 a 26 c 27 d 28 c 29 b 30 a 9 9 100
13. (c) Let the income of C be ` x.
7. (c) Let total number of votes polled be x. 40 × 100
31 a 32 d 33 c 34 c 35 c 36 b 37 c 38 b 39 a 40 d 25x ∴ x= = ` 32
∴Income of B = x − x × 25% = x − 125
∴ 65% of x − 35% of x = 300 100
41 b 42 c 43 d 44 a 45 b 46 d 47 b 48 c 49 b 65 35 18. (a) The price of item first increased by
x− x = 300 = x− =
x 3x
100 100 20% and then decreased by 20%.
4 4
30 300 × 100 20 × ( −20) 
⇒ x = 300 ⇒ x =
and, the income of A =
3x 3x
+ × 20% ∴ Net effect =  20 − 20 + 
100 30  100 
4 4
∴ x = 1000 votes 3x 3x 1 9x −400
= + × = = = −4% [Rule 8]
Hence, the total number of votes is 1000. 4 4 5 10 100
MATHEMATICS Percentage 75 76 CDS Pathfinder

= 126800  1 +
19. (c) Let salary of each of them be ` x. 24. (a) New price reduced by 30. (a) Water in the mixture = 10% of 140 L 3  1 = ( 100 + 20) = ` 120 Winner was supported by 47% of total
 1− 
 20   5 voter. i.e. 47% of x.
=   × 100%
George saves 22% of x and his saving x 10 5
[Rule 7] = × 140 = 14 L and, the cost of the meal of X =
as
 100 + x  Hence, rival got vote = (90% of x − 60)
= 126800     = 116656
amount is ` 1540. 100 23 4 6

22
x = 1540  20   5  much as the cost of the meal of − 47% of x.
= 
60  Let x L of water added in the mixture,
100  × 100% = 37.5% 14 + x 
5
Z = × 120 = ` 100 = 43% of x − 60
1540 × 100
 100 + 60  then   × 100 = 125
. 35. (c) Given, R = 15%, R = 15% 6
⇒ x= = ` 7000 x  140 + x  1 2
It is given that difference between their
25. (a) Let fraction be .  R  R 
22 ∴ Population = P  1 + 1   1 − 2  ∴ Total amount that all the three of vote is 308.
y ⇒ 1400 + 100x = 1750 + 125
. x  100   100 
20. (b) Let A ’s salary = x. Then, them has paid Then, 47% of x − 43% of x + 60 = 308
120% ofx 4x ⇒ . x = 350
875
B’s salary = 2x New fraction = =
= 126800  1 +
15   15  = 100 + 100 + 120 = ` 320 ⇒ 4% of x = 308 − 60
90% of y 3 y 350 1− 
New salary of A = 150% of x =
3 ⇒ x = = 4L  100   100  4
x 875. 41. (b) (Pipe) ⇒ x = 248
2 According to question, 115 85 100
= 126800 × × = 123947
Total salary of B = 125% of 2x = x
5 [by given condition] 31. (a) Let the monthly salary of the person 248 × 100
be ` x.
100 100
70% 30% ∴ x= = 6200
2 4x 16 x 16 3 4 4
= ⇒ = × = 36. (b) Given, R = 15% and R = 20%
Total combined salary 3 y 27 y 27 4 9 By given condition, 1 2 ∴ Increase in percentage of longer piece 48. (c) Let the original salary be x.
( 100 − 30 − 25 − 20 − 12) × x ∴ Required population
=  x + x = 4x
3 5 compared to shorter piece
= 1040 Then, increased salary
= P  1 +
2 2  26. (c) Amount of sugar in 6 L of solution 15   20  70 − 30
100  1+  = × 100% [Rule 4]
= 
4  100   100  110  11x
∴ Required increment in salary = × 6 = 0. 24 L 13x 1040 × 100 30 x= `
⇒ = ` 1040 ⇒ x =  100  10
100
4x − 3x = 126800     = 174984
x 100 1 100 13 23 6 40 400
= = × 100 = =33 %  20   5  = × 100% = %
3x 3x 3 3 After evaporation, sugar in 5 L = 0. 24 L ⇒ x = ` 8000 30 3 Q He received the same salary even after
∴ Percentage of sugar increment.
[Rule 4] Hence, the monthly salary of the person 37. (c) Refer to question 21. 42. (c) Refer to question 6.
 0. 24  is ` 8000. Amount of salary he did not receive
21. (d) Let total income be ` x. 4 38. (b) Refer to question 34.
= × 100 = 4 % 43. (d) Average price increase 11x x
 5  = − x =`
Income deposited = 10% of  x −
4    5 32. (d) Let total number of staff be 100. 5 + 8 + 77 
= 
x 39. (a) 5% of 50% of 500 90 10 10
 100  Female staff = 40 % = % = 30%
27. (d) Let the third number be z. =
5
×
50
× 500 = 12.5 [Rule 1]  3  3 ∴ Amount of salary in percentage
10  4  96 Male staff = ( 100 − 40) = 60
= x − x = x ( 100 − 30) 100 100
 
7z x
100  100  1000 ∴ First number x = ×z = 44. (a) Refer to question 23.
 
100 10 Votes casted by females 40. (d) Given, the cost of meal of Y = ` 100 = 10 × 100 %
Remaining income = ` 10800 40 45. (b) Refer to question 2.
 11x 
4 96x Second number = × 40 = 16 Now, according to the question,  
∴ x+ + 10800 = x ( 100 − 40) 100 46. (d) Refer to question 18.  10 
100 1000 y= ×z =
6z Cost of the meal of Z = 20% more than
60 x 100
136x 100 10 Votes casted by males = × 60 = 36 that of Y 47. (b) Let the number of voters on the voter = × 100% = %
⇒ 10800 = x − 100
=  100 + × 100
Difference between first and second 20 list be x. 11x 11
1000
864x 10800 × 1000 number Votes casted by both males and females  100  Total cast vote = 90% of x − 60 49. (b) Refer to question 31.
⇒ 10800 = ⇒x = = 16 + 36 = 52
7z 6z z
1000 864 = (x − y) = − =
∴ x = ` 12500 10 10 10 ∴ Percentage votes obtained = 52%.
22. (a) Let the salary of Rohit be ` 100, then Hence, the required percent [Rule 4] 33. (c) For class-X, let the student passed in
saving = ` 30 z first class = a%.
100 2
Expenses = ` 70 = 10 × 100% = % = 14 % Then, by condition given in question,
7z 7 7
New expenses = ( 100 + 30)% of ` 70 a × 30
10 a% of 30 = 24 ⇒ = 24
= ` 91 100
28. (c) Glycerine in the given sample = 80%
New saving = ` ( 100 − 91) = ` 9 ∴ a = 80%
of 5 L
He saves ` 9, his salary = ` 100 80 Now, for class-Y let the student passed
If he saves ` 1215. = ×5= 4L in first class = b%.
100
Then, his salary = `  × 1215
100 According to the question,
 9  Let x L of glycerine be added, then
4+ x b% of 35 = 28
= ` 13500 × 100 = 95 b
(5 + x ) ⇒ × 35 = 28
23. (b) Let income of man be ` 100. 100
⇒ 80 + 20x = 95 + 19x
Then, his expenditure = ` 75 ∴ b = 80%
∴ x = 15 L
and savings = ` 25 Hence, both classes have equal
New income = ` ( 100 + 20) = ` 120 29. (b) Increased wages of the worker percentage of students getting first class.
= 200 + 20% of 200 = ` 240
New expenditure = ` ( 75 + 7.5) 34. (c) Given, R = 15% and R = 20% .
Also, let he worked for x h. 1 2
= ` 82.50 ∴ Required population
New saving = ` ( 120 − 825. ) = ` 37.50 ∴ Reduced working hours
 R   R 
=x−
20
x = 080 = P 1+ 1  1− 2  [Rule 10]
Saving difference = 37.50 − 25.0 = 12.5 . x
 100   100 
100
∴ Percentage increase saving
= 126800  1 +
240 15   20 
12.5 Required wages = × 08
. x = ` 192  1− 
= × 100 = 50% x  100   100 
25
09
78 CDS Pathfinder

1
Sol. d. Given, SI = ` 400, R = 10% and T = 1 month = yr Rule 2 If a certain Principal Amounts to ` A1 in t 1 yr
12
and to ` A 2 in t 2 yr, then the sum ( P ) is given by
P ×R ×T 100 × SI 100 × 400 × 12
SI = or P = = = ` 48000  A t − A1 t 2 
100 R ×T 10 × 1 ` 2 1  and the rate per cent ( R) per annum is
 t1 − t 2 
EXAMPLE 2. In what time will the simple interest on 100 ( A 2 − A1 )
` 400 at 10% per annum be the same as the simple given by  %.
interest on ` 1000 for 4 yr at 4% per annum?  A1 t 2 − A 2 t 1 
a. 2 yr b. 3 yr c. 4 yr d. 6 yr
EXAMPLE 5. A certain sum amounts to ` 1586 in

SIMPLE INTEREST Sol. c. Here, P = `1000, T = 4 yr, and R = 4%

∴ Simple interest =
P × R × T 1000 × 4 × 4
100
=
100
= ` 160

Now, simple interest = ` 160, P = ` 400, R = 10%,


2 yr and ` 1729 in 3 yr. Find the rate and the sum.
a. 8%, ` 1200
b. 9%, ` 1300
c. 10%, ` 1000
100 × SI 100 × 160 d. 11%, ` 1300
then, T = = = 4 yr
P ×R 400 × 10 Sol. d. Here, A1 = ` 1586, t1 = 2 yr, A2 = 1729 and t 2 = 3 yr
A2t1 − A1t 2 1729 × 2 − 1586 × 3
EXAMPLE 3. A sum at simple interest of 4% per ∴ Required principle = =
annum amounts to ` 3120 in 5 yr. Find the sum. t1 − t 2 2− 3
Regularly (1-2) questions have been asked from this chapter. Generally direct formula based
a. ` 2500 b. ` 1300 c. ` 4000 d. ` 2600 = ` 1300
questions are asked from this chapter and hence making it easy to score area. ( A − A1) × 100 
Sol. d. Here, T = 5 yr, R = 4%, A = ` 3120 and required rate R =  2 %
100 × A 100 × 3120 100 × 3120  A1t 2 − A2t1 
We know that, P = = = (1729 − 1586) × 100 143 × 100
100 + RT 100 + 4 × 5 120 = = = 11%
= ` 2600 1586 × 3 − 1729 × 2 1300
INTEREST Rule 3 At the same rate of simple interest, if a sum of
When money is borrowed by a person, then customarily the money lender used to charge some extra Important Rules and Formulae money becomes n1 times of itself in t 1 yr and n2 times in
money in lieu of the money lent by him. This extra money earned by the money lender is called Rule 1 If a sum of money becomes n times in ‘T ’ yr at ( n − 1)
t 2 years, then t 2 = 2 t 1 yr.
interest.
simple interest, then rate of interest will be, ( n1 − 1)
Some terms related to interest are given below
 100( n − 1)
Principal (P) The money which is borrowed from a money lender, is called Principal. R=  % EXAMPLE 6. A sum of money becomes 3 times in 5 yr
 T 
Amount (A) The sum of the principal and the interest is called Amount i.e. Amount (A) = Principal ( P ) + at simple interest. In how many years, will the same sum
Interest ( I ) . become 6 times at the same rate of simple interest?
EXAMPLE 4. At what rate per cent per annum will a
sum of money double in 8 yr? a. 10 yr
Rate (R) It is the rate at which the interest is charged on Principal.
1 b. 12 yr
Time (T) The time period, for which the money is lent or deposited or borrowed, is called Time. a. 12% b. 12 %
2 c. 12.5 yr
c. 13% d. 15% d. 10.5 yr
SIMPLE INTEREST Sol. b. Here, n = 2 and T = 8 yr Sol. c. Here, n1 = 3, t1 = 5yr and n2 = 6, t 2 = ?
If the interest is calculated on the original Principal for any length of time, then it is called simple 100(n − 1) 100 × ( 2 − 1) 100 1 (n2 − 1) (6 − 1) 25
interest. ∴ Rate = = = = 12 % ∴ Required time (t 2) = t1 ×5= = 12.5 yr
T 8 8 2 (n1 − 1) ( 3 − 1) 2
Principal × Rate × Time P × R×T
Simple Interest (SI) = or SI =
100 100
Amount ( A ) = Principal ( P ) + Interest ( I ) and P =
100 × A
100 + RT PRACTICE EXERCISE
Where, A → Amount
R → Rate of simple Interest 1. Find the amount on a sum of ` 400 for 3 yr at 3. If a certain sum is doubled in 8 yr on simple
simple interest at 5% per annum. interest, in how many years will it be four times?
T → Time
(a) ` 460 (b) ` 415 (c) ` 435 (d) ` 412 (a) 24 yr (b) 16 yr (c) 32 yr (d) 12 yr
EXAMPLE 1. The sum required to earn a monthly interest of `400 at 10% per annum at simple 2. Find what sum of money will amount to ` 900 in 4. A sum of money at simple interest amount to
interest is 4 yr at 5% per annum on simple interest? ` 1260 in 2 yr and `1350 in 5 yr, then the rate
a. ` 2000 b. ` 12000 c. ` 24000 d. ` 48000 (a) ` 750 (b) ` 650 per cent per annum is
(c) ` 500 (d) ` 550 (a) 30% (b) 10% (c) 2.5% (d) 5%
MATHEMATICS Simple Interest 79 80 CDS Pathfinder

the same rate of interest. Which of the following Directions (Q. Nos. 25-27) If SI for a certain sum P1 PREVIOUS YEARS’ QUESTIONS
5. The difference of 13% per annum and 12% of a
statement is true in this regard? for time T1 and rate of interest R1 is I1 and SI for
sum in 1 yr is ` 110. Then, the sum is
(a) A will get twice the amount of interest that B would get another sum P2 for time T2 and rate of interest R2 is 28. The principal on which a simple interest of ` 55
(a) ` 12000 (b) ` 13000 (c) ` 11000 (d) ` 16000
(b) B will get twice the amount of interest that A would get. P R T − PR
1 1T1 will be obtained after 9 months at the rate of
I2, then difference of SI = I2 − I1 = 2 2 2 .
6. The simple interest on a sum of money at 10% per (c) A and B will get the same amount as interest. 100 3( 2 / 3)% per annum is e 2013 I
annum for 6 yr is half the sum. Then, the sum is (d) B will get four times the amount of interest that A (a) ` 1000 (b) ` 1500
(a) ` 5000 (b) not possible (c) ` 4000 (d) ` 6000
would get 25. Simple interest for the sum of ` 1500 is ` 50 in
(c) ` 2000 (d) ` 2500
18. Out of a sum of ` 625 a part was lent at 5% and 4 yr and ` 80 in 8 yr, the rate of SI is
7. The sum which amounts to ` 840 in 5 yr at the the other at 10% simple interest. If the interest (a) 0.6% (b) 5% (c) 0.05% (d) 0.5% 29. In how much time would the simple interest on a
rate of 8% per annum simple interest is on the first part after 2 yr is equal to the principal amount be 0.125 times the principal
100 × 8 × 5 + 100 100 + 840 interest on the second part after 4 yr, then the
26. Simple interest for the sum of ` 1230 for 2 yr is amount at 10% per annum? e 2015 I
(a) ` (b) `
840 100 + 5 × 8 second sum (in `) is ` 10 more than the simple interest for ` 1130 for
1 3 1 3
100 × 840 840 × 5 × 8 the same duration. Find the rate of interest. (a) 1 yr (b) 1 yr (c) 2 yr (d) 2 yr
(c) ` (d) ` (a) ` 125 (b) ` 200 (c) ` 250 (d) ` 300 4 4 4 4
100 + 5 × 8 100 (a) 5% (b) 6% (c) 8% (d) 2%
19. A man invests an amount of ` 15860 in the 30. If a sum of money at a certain rate of simple
8. A certain sum at simple interest amounts to names of his three sons A, B and C in such a way 27. If the annual payment oa ` A will discharge a interest per year doubles in 5 yr and at a
` 1040 in 3 yr and to ` 1360 in 7 yr. Then, the sum is that they get the same interest after 2, 3 and 4 debt of ` 1092 due in 2 yr at 12% simple interest, different rate of simple interest per year becomes
(a) ` 750 (b) ` 800 (c) ` 900 (d) ` 1000 yr, respectively. If the rate of simple interest is then three times in 12 yr, then the difference in
5%, then the ratio of the amounts invested I. A will be ` 515 (approx.). the two rates of simple interest per year is
9. A sum becomes 6 fold at 5% per annum. At what
rate, the sum becomes 12 fold? among A, B and C will be II. A will be ` 530 (approx.), if interest rate is 6%. e 2016 I
1 1 1 (a) Only I (b) Both I and II 1 1
(a) 10% (b) 12% (c) 9% (d) 11% (a) 10 : 15 : 20 (b) : : (a) 2% (b) 3% (c) 3 % (d) 4 %
10 15 20 (c) Only II (d) Neither I nor II 3 3
10. A man borrowed ` 40000 at 8% simple interest (c) 110 : 115 : 120 (d)
1
:
1
:
1
per year. At the end of second year he paid back 110 115 120
certain amount and at the end of fifth year he
paid back ` 35960 and cleared the debt. What is
20. A sum was put at simple interest at a certain
the amount did he pay back after the second year?
rate for 2 yr. Had it been put at 3% higher rate, ANSWERS
it would have fetched ` 72 more. The sum is
(a) ` 16200 (b) ` 17400
(a) ` 1200 (b) ` 1600 (c) ` 1900 (d) ` 1400 1 2 a 3 a 4 c 5 c 6 b 7 c 8 b 9 d 10 b
(c) ` 18600 (d) None of these
21. Harsha makes a fixed deposit of ` 20000 in Bank 11 b 12 d 13 a 14 a 15 c 16 c 17 c 18 a 19 b 20 a
11. A sum of ` 1550 was lent partly at 5% and partly
of India for a period of 3 yr. If the rate of 21 c 22 b 23 c 24 d 25 d 26 a 27 b 28 c 29 a 30 c
at 8% simple interest. The total interest received
after 3 yr was ` 300. The ratio of money lent at interest be 13% SI per annum charged half
5% to 8% is yearly, what amount will he get after 42 months?
(a) ` 27800 (b) ` 28100 (c) ` 29100 (d) ` 30000
(a) 11 : 12 (b) 16 : 15 (c) 12 : 21
12. Rahim buys a house and pays ` 8000 cash
(d) 11 : 13
22. A sum of money lent on simple interest triples HINTS AND SOLUTIONS
and ` 9600 at 5 yr credit at 4% per annum simple itself in 15 yr and 6 months. In how many year
interest. Then, the cash price of the house still it be doubled? 1. (a) Given, P = ` 400, R = 5% Q Sum 4. (c) Simple interest in 3 yr
(a) ` 10000 (b) ` 9600 (c) ` 17000 (d) ` 16000 (a) 6 yr and 3 months (b) 7 yr and 9 months and T = 3 yr 100 × A 100 × 900 = ` ( 1350 − 1260) = ` 90
= = = ` 750
(c) 8 yr and 3 months (d) 9 yr and 6 months P × R×T 100 + RT 100 + 5 × 4
13. At what rate per cent per annum simple interest, Simple interest = ∴ Simple interest for 2 yr
23. Mr Pawan invests an amount of ` 24200 at the 100 3. (a) Let the sum be ` x, so amount
will a sum of money triple itself in 25 yr? 2
400 × 3 × 5 = × 90 = ` 60
(a) 8% (b) 9% (c) 10% (d) 12% rate of 4% per annum for 6 yr to obtain a simple SI = = ` 60 = ` 2x 3
interest, later he invests the principal amount as 100 ∴ SI = ` x
14. A man invested ` 1000 on a simple interest at a Q Principal = ` ( 1260 − 60) = ` 1200
well as the amount obtained as simple interest ∴Amount = P + SI = 400 + 60 = ` 460 Let R be rate of interest.
certain rate and ` 1500 at 2% higher rate. The 100 × SI 100 × x ∴ Rate,
for another 4 yr at the same rate of interest. 2. (a) Let the sum of money be ` x. ∴ R= = = 125
. % 100 × SI 100 × 60 60 5
total interest in 3 yr is ` 390. What is the rate of P ×T x ×8 R= = = =
What amount of simple interest will be obtained Amount = Sum + SI
interest for ` 1000? Q Now, the needed amount = ` 4x P ×T 1200 × 2 24 2
at the end of the last 4 yr? x ×5 × 4
(a) 4% (b) 5% (c) 6% (d) 8%
(a) ` 4800 (b) ` 4850.32 (c) ` 4801.28 (d) ` 4700 ∴ Amount = x + ∴ SI = ` ( 4x − x) = ` 3x R = 25
. %
100
15. In what time the simple interest on a sum of 100 × SI 100 × 3x Shortcut Method
24. A person invested some amount at the rate of Q SI = P × R × T  ∴T= = = 24 yr
money be 3 / 8 of the principal with rate of  P×R x × 125
. Here, A = ` 1260, t = 2 yr,
12% simple interest and the remaining at 10%. 100  1 1
interest 3(1 / 8)%? Shortcut Method A = ` 1350 and t = 5yr
He received yearly an interest of ` 130. Had he But amount = ` 900 2 2
(a) 9 yr (b) 6 yr (c) 12 yr (d) 15 yr Here, n = 2, n = 4 and t = 8 yr
interchanged the amounts invested, he would 20x 1 2 1 ∴ Required rate
∴ 900 = x + ( n − 1)
16. If the rate of simple interest is 12% per annum have received an interest of ` 134. How much 100  (A − A ) 
∴ Required time = 2 ×t
the amount that would fetch interest of ` 6000 money did he invest at different rates? 6x 900 × 5 ( n − 1) 1 = 2 1
 × 100% [Rule 2]
per annum is (a) ` 500 at the rate of 10%, ` 800 at the rate of 12%
⇒ 900 = ⇒x= = ` 750 1  A1 t 2 − A2 t1 
5 6 [Rule 3]
(a) ` 7200 (b) ` 72000 (c) ` 50000 (d) ` 48543.69 (b) ` 700 at the rate of 10%, ` 600 at the rate of 12% Shortcut Method ( 4 − 1) 3 ( 1350 − 1260)
= × 8 = × 8 = 24 yr = × 100% = 25
. %
17. A lends a sum of money for 10 yr at 5% simple (c) ` 800 at the rate of 10%, ` 400 at the rate of 12% Here, A = ` 900, R = 5% and T = 4yr ( 2 − 1) 1 1260 × 5 − 1350 × 2
interest, B lends double that amount for 5 yr at (d) ` 700 at the rate of 10%, ` 500 at the rate of 12%
MATHEMATICS Simple Interest 81 82 CDS Pathfinder

5. (c) Let the sum be ` x. Shortcut Method 13. (a) Let the principal be ` P 19. (b) Let the amount of A = ` a, Let amount doubled be t yr. 27. (b) Case I Annual payment
1
x × 13 × 1 x × 12 × 1 As, R = 5%, n = 6, n = 12. As, amount = 3P and T = 25 yr time = 2 yr and rate = 5% Now, SI = 2P − P = ` P 100P
Then, − = 110 =
100 100
1 1 2
∴ SI = 3P − P = 2P ∴ Simple Interest of SI × 100 RT (T − 1)
x According to the formula, ∴ t = 100T +
⇒ = 110 n −1 100 × SI a × 2 × 5 10a 1
P×R 2
100 12 − 1 Q Rate = A= =
R = 2 ×R = × 5 [Rule 3] principal × T 100 100 1092 × 100
⇒ x = 110 × 100 = ` 11000 2
n −1 1
6−1 P × 100 × 31 =
⇒ t = 24( 2 − 1)
1 100 × 2P Let the amount of B = ` b, rate = 5% 1
P × 400
Shortcut Method = = 8% 100 × 2 +
=
11
× 5 = 11% P × 25 and time = 3 yr. 31 2
Here, P = ` 110 , r = 13%, r = 12% 5 ∴ Simple interest of = = 7 yr and 9 months
1 2 1092 × 100
and t = t = 1yr 14. (a) Let a man invest ` 1000 at a R%.
b × 3 × 5 15b
4 = = ` 515.09 ≈ ` 515
1 2 10. (b) Total borrowed money = ` 40000 B= = 212
∴ Short trick Now, rate is increased by 2%. P × R×T
and rate of interest = 8% 100 100 23. (c) Case I SI = Case II Annual payment
110 × 100 ∴ New rate = ( R + 2)% Let the amount of C = ` c, time = 4 yr 100
= 40000 × 8 × 2 1092 × 100
13 − 12 The interest for 2 yr = By given condition, and rate = 5% 24200 × 4 × 6 =
100 R= = ` 5808 12( 2 − 1)
= ` 11000 1000 × R × 3 1500 × ( R + 2) × 3 c × 4 × 5 20c 100 100 × 2 +
= ` 6400 + = 390 ∴Simple interest of C = = 2
100 100 100 100 ∴ Amount = Principal + SI
6. (b) Let the sum (principal) be ` x.
a × 10 b × 15 c × 20 1092 × 100
x Let he paid ` x at the end of second
= = = = ` 530.09 ≈ ` 530
∴ Simple interest = ` year. ⇒ 30R + 45R + 90 = 390 But SI = 24200 + 5808 = 30008 206
2 100 100 100
∴ Interest will be calculated on ⇒ 75R = 300 ⇒ R = 4% 30008 × 4 × 4
and T = 6 yr, R = 10% per annum ⇒ 10 a = 15 b = 20 c = k Case II SI = = ` 4801.28 28. (c) Let P be the principal amount.
1 25
P × R×T ` ( 40000 − x + 6400) 15. (c) Here, rate of interest = 3 % = % k k k 100
SI = ` 55, time, t = 9 months =
9
∴ SI = 8 8 So, a= , b= , c= yr
100 ( 46400 − x) × 3 × 8 10 15 20 12
Interest for 3 yr = Let principal be ` x. 24. (d) Let the person invest amount x and y
x x × 10 × 6 100 1 1 1 2 11
⇒ = ⇒
1
=
6
and simple interest = ` x
3 ∴ a : b :c = : : into two different rates of interest. and rate, r = 3 % = %
6 10 15 20 3 3
2 100 2 10 = ` ( 46400 − x) 8 x × 12 × 1 y × 10 × 1
Which is not true, so it is not a possible 25 25 ∴ + = 130 P × R×T
x× × T 20. (a) Let the sum be ` x and the original 100 100 Q SI =
case. 6 3 8 100
∴ ( 46400 − x) + 46400 − x = 35960 ∴ x= rate r%, then
Q SI = PRT 
8 100 x×r×2 SI × 100 55 × 100
7. (c) Let the sum be ` 100. 25 Simple interest =  100  ⇒ P= = × 3 × 12
300
21576 × 25 ⇒ = T ⇒ T = 12 yr 100 R×T 11 × 9
Then, amount = (Sum + SI) ⇒ x= = ` 17400
31
25 Now, rate is increased by 3%. ⇒ 12x + 10 y = 13000 …(i)
100 × 8 × 5 
=  100 +  = ( 100 + 8 × 5) 16. (c) Given, rate of interest = 12% per ∴ New rate = ( r + 3)% y × 12 × 1 x × 10 × 1 = 5 × 100 × 4 = ` 2000
 100  11. (b) Let sum lent at rate 5% be ` x. and + = 134
annum x × ( r + 3) × 2 ∴ Principal ( P ) = ` 2000
So, when the amount is ( 100 + 8 × 5), Q Simple interest = 100 100
Then, sum lent at rate 8% Simple interest = ` 6000 per annum 100
then sum = 100 ⇒ 12 y + 10x = 13400 …(ii) 29. (a) Let the principal be ` x and the time
= ` ( 1550 − x) Let principal is ` P. x × ( r + 3) × 2 x × r × 2
When the amount is ` 840, then sum ∴ − = 72 be t yr.
x ×5 ×3 P × 1 × 12 100 100 On solving Eqs. (i) and (ii), we get
Rate = 10%
100 × 840  ∴ Simple interest at rate 5% = ∴ 6000 =
= `   100 100 ⇒
( xr + 3x)2 2xr
− = 72 x = ` 500 and y = ` 700 P × R×T
 100 + 8 × 5  6000 × 100 100 100 Q Simple interest =
Simple interest at rate 8% Q P= = ` 50000 2xr + 6x − 2xr 25. (d) Here, I = 50, I = 80, T = 4 yr, 100
8. (b) Simple interest for 4 yr ( 1550 − x) × 8 × 3 12 ⇒ = 72 1 2 1
= 100 According to the question,
= ` ( 1360 − 1040) = ` 320 100 Hence, the required Principal amount is T = 8 yr, and P = ` 1500
` 50000. ∴ x = ` 1200 2 0.125 × Principal = Simple interest
Simple interest for 3 yr x × 15 ( 1550 − x) × 24 According to formula,
∴ + = 300 P × 10 × T
= `  × 3 = ` 240
320 17. (c) For A Let the amount be ` x. 21. (c) As rate of interest is charged half P × R × (T − T ) ∴ 0.125 P =
100 100
 4  15x 37200 24x Rate of interest = 5% and Time = 10 yr yearly, I −I = 2 1 100
2 1
⇒ + − = 300 x × 5 × 10 x 13 100 125P 10 × P × T
∴ Sum or principle 100 100 100 ∴ Simple interest = = So, rate = % half yearly 1500 × R × (8 − 4) ⇒ =
= ` ( 1040 − 240) = ` 800 100 2 2 ⇒ 80 − 50 = 1000 100
⇒ x = ` 800  42  100 125
Shortcut Method Amount lent at rate 8% For B The amount be ` 2x. time  × 2 half yearly = 7 half yearly ⇒ =T
 12  ⇒ 30 = 15 × R × 4 100
Here, A = ` 1040 , t = 3 yr, = ` ( 1550 − 800) = ` 750 Rate of interest = 5% and Time = 5 yr
1 1
2x × 5 × 5 x 20000 × 13 × 7 ⇒ 2R = 1 5 1
A = ` 1360 and t = 7yr 800 16 SI = = SI = = ` 9100 ⇒ T = = 1 yr
2 2 ∴ Required ratio = = = 16 : 15 100 2 100 × 2 ⇒
1
R = = 0.5% 4 4
∴ Required principle (sum) 750 15
∴ Amount ( A ) = 20000 + 9100 2
So, A and B both will get the same 30. (c) Let principal = ` P, then amount of
A t −At  = ` 29100 money = ` 2P
= ` 2 1 1 2 [Rule 2]
12. (d) Let the amount remaining to pay be amount as interest. 26. (a) Here, I − I = ` 10, P = ` 1230,
1 2 1
 t −t  ` x.
∴ SI = 2P − P = `P
 1 2  18. (a) Let the first part and the second part 22. (b) Let initial amount be ` P, then P = ` 1130 and T = 2 yr
∴ Price of house = ` ( x + 8000) be ` x and ` ( 625 − x), respectively. A = ` 3P and 2
P × r ×5
1360 × 3 − 1040 × 7 x × 4 ×5 According to formula, Now, P= ⇒ r = 20%
= = `800 ⇒ 9600 − =x Q The simple interest on both part in T = 15 yr and 6 months =
31
yr
3−7 100
100 same. 2 T × R × (P − P )
9. (d) SI at 5% = 6 P − P = 5P x I −I = 2 1 Amount of money after 12 yr = `3P
⇒ 9600 − = x x × 5 × 2 ( 625 − x) × 10 × 4 Q SI = A − P = 3P − P = ` 2P 2 1
100
P ×5 ×T 5 So, = ∴ SI = 3P − P = 2P
∴ 5P = ⇒ T = 100 yr 100 100 ⇒ P×
31
×
R
= 2P R × 2( 1130 − 1230)
6x 9600 × 5 −10 = P × R × 12 50
100 ⇒ 9600 = ⇒ =x ⇒ 10x = ( 625 − x)40 2 100 100 Now, 2P = ⇒ R= %
Now, for new rate ( R), 5 6 2 × 2 × 100 100 3
⇒ x = ( 625 − x)4 ⇒ R= % −200R
P × R × 100 ⇒ x = ` 8000 −10 =
11P = ∴ x = ` 500 31 100 ∴ Difference between two interest rates
100 ∴ Cash price of the house
=  20 −  % =
∴ Second part 400 50 10 1
∴ R = 11% = % ∴ R = 5% %=3 %
= ` (8000 + 8000) = ` 16000 = ` ( 625 − 500) = ` 125 31  3 3 3
10
83 84 CDS Pathfinder

EXAMPLE 2. The compound interest on ` 24000


1 Rule 4 When interest is compounded annually but time
yr at the rate of
compounded semi-annually for 1 a
2 is given in fraction (say, t yr), then
b
10% per annum is t
 R  1 + ( a/b)R
a. ` 3783 b. ` 2783 c. ` 2763 d. ` 3763 Amount = P 1 +  × 
 100   100 
Sol. a. Given, P = ` 24000
Rate of interest = 10% per annum EXAMPLE 4. The amount and compound interest on
and
1
time = 1 yr = yr
3 ` 5000 compounded annually for 2 yr 6 months at the
2 2 rate of 10% per annum is

COMPOUND INTEREST
Since, the interest is compounded half-yearly. a. ` 6252.5 and ` 1342.5 b. ` 6000 and ` 1300
10
Then, rate (R) = % half-yearly c. ` 6250 and ` 1340 d. ` 6352.5 and ` 1352.5
2
3 Sol. d. Here, time = 2 yr and 6 months
and time (n) = 2 × = 3 half-yearly a 1
2 ∴ t = 2 yr and = yr
 b 2
5 3 R  n
∴ A = 24000 1 +  
Q A = P 1+ 100   1+ a R 
 100 R t 
  Amount = P 1+  × b 
Q
 100 
1 3 21 3  100 
= 24000 1 +  = 24000   [Rule 1]  
Usually (1-2) questions have been asked from this chapter. Questions from this chapter are based on  20  20
21 21 21  1
⋅ 10
the direct application of compound interest formula. = 24000 × × × = ` 27783 2 
A = 5000 1 +
10 
20 20 20 ∴  1 + 2 
 
100  100 
Now, compound interest = A − P  
= ` 27783 − ` 24000 = ` 3783
110 2  105
Rule 3 When interest is compounded quarterly = 5000    
 100  100
COMPOUND INTEREST (every 3 months), then
= 5000 ×
11 11 21
× × = ` 6352.50
When the interest is calculated on the amount of previous year, then it is known as compound interest. 4n 10 10 20
 R 
It will make a deposit or loan grow at a faster rate than simple interest. (i) Amount = P 1 +  ∴ Compound interest = A − P
 100 × 4 CI = 6352.50 − 5000 = ` 1352.50
 R 
4n 
Important Rules and Formulae (ii) CI = P 1 +  − 1 Rule 5 When rate of interest for n1 , n2 and n3 yr are
Consider P = Principal (amount borrowed) , A = Amount (Principal + Interest)  100 × 4 
R1 , R2 and R3 respectively, then
R = Rate of interest per annum and n = Number of years n1 n2 n3
EXAMPLE 3. The amount and the compound  R   R2   R3 
Rule 1 Amount = P 1 + 1  1 +  1 + 
When interest is compounded annually, then interest on ` 100000 compounded quarterly for 9  100   100   100 
n  n  months at the rate of 4% per annum is
 R R
(i) Amount = P 1 +  (ii) CI = A – P = P 1 +  − 1 a. ` 106060 and ` 6060.10 b. ` 103030 and ` 3030.10 EXAMPLE 5. The compound interest on ` 5000 for
 100   100 
 c. ` 103030.10 and ` 3030.10 d. ` 106060.10 and ` 6060.10 4 yr if the rate of interest is 10% per annum for the
Sol. c. Given, P = ` 100000, rate of interest = 4% per annum first two years and 15% for the next two years is
EXAMPLE 1. The amount and the compound interest on ` 24000 compounded annually for 2 yr at a. ` 3000 b. ` 3001 c. ` 3002 d. None of these
and time = 9 months
the rate of 10% per annum is
Since, the interest is compounded quarterly. Sol. b. Here, R1 = 10%, n = n 1 + n2 = 4 yr
a. ` 39040 and ` 4040 b. ` 29040 and ` 5040 c. ` 19040 and ` 3040 d. None of these
4 n 1 = 2 yr, R2 = 15% , n2 = 2 yr
Then, rate (R) = = 1% quarterly
Sol. b. Here, P = ` 24000, R = 10% per annum and n = 2 yr 4
and principal = ` 5000
9
R n 10  2 110 2 and time (n) = 9 months = × 4 = 3 quarter yearly R n1 R n2
Q A = P 1 +  = 24000 1 +  = 24000   = 24000 ×
110 110
× = ` 29040 12 Q Amount = P 1 + 1  1+ 2 
 100  100  100 100 100  100  100
 R n
Q Amount, A = P 1+  10  2  15  2
Q Compound interest = Amount − Principal = ` 29040 − ` 24000 = ` 5040  100 ∴ A = 5000 1 +  1 + 
 100  100
So, amount = ` 29040 and compound interest = ` 5040 1 3 101 3
∴ A = 100000 1 +  = 100000   110 2  115 2
 100  100
Rule 2 = 5000 ×    
When interest is compounded half-yearly (every 6 months), then  100  100

 r 
2n  R 
2n  101 101 101
= 100000 × × × = ` 103030.10 = 5000 ×
11 11 23 23
× × × = ` 8001.125
(i) Amount = P 1 +  (ii) CI = 1 +  − 1 100 100 100 10 10 20 20
 100 × 2  100 × 2  ∴Compound interest = A − P = ` 103030.10 − ` 100000 ∴ Compound interest
= ` 3030.10 = A − P = ` 8001125
. − ` 5000 ~
− ` 3001
MATHEMATICS Compound Interest 85 86 CDS Pathfinder

EXAMPLE 7. A sum of money on compound interest x = value of each instalment  100 (100) 2 (100) 3 
Rule 6 The difference D between simple and compound ⇒ P =x + 2 + 
amount to ` 9680 in 2 yr and to ` 10648 in 3 yr. What  R
n
(100 + R) (100 + R) (100 + R) 3 
interest accrued on ` P at the rate of interest of R% is is the rate of interest per annum? Total amount paid in instalments, A = P 1 + 
given by  100   100   100  2
 100 
3

a. 5% b. 10% c. 15% d. 20% = 486680   +  +  
PR2 n = number of instalments  100 + 15  100 + 15  100 + 15 
(i) For 2 yr, D = 2 Sol. b. Given, A1 = ` 10648 and A2 = ` 9680
(100 )
( A2 − A1) EXAMPLE 9. Sapna borrowed some money on  20  20 2  20 3 
PR2 ( 300 + r ) ∴ Rate of compound interest = × 100% = 486680 ×  +  +  
(ii) For 3 yr, D = A1 compound interest and returned it in 3 yr in equal
 23
 23  23

(100 )3 10648 − 9680 annual instalments. If rate of interest is 15 % per
= × 100% = 10% 20  20 400
9680 annum and annual instalment is ` 486680, then the = 486680 × 1 + + 
EXAMPLE 6. The difference between the compound 23  23 529 
interest and the simple interest on a certain sum at sum borrowedes.
= 1111200
12% per annum for 2 yr is ` 90. Then, sum is DEPRECIATION a. 111300
c. 1111200
b. 122400
d. 154320 ∴ Principal borrowed = ` 1111200
a. ` 6250 b. ` 6350 c. ` 6520 d. ` 6950
The value of certain things like the machine, vehicle etc.,
Sol. a. Let the sum be ` 100. decreases over a period of time. The decrement in the Sol. c. Given that, rate of interest, R = 15% pa
value things is called depreciation. The depreciation per SOME IMPORTANT RESULTS
Case I Then, simple interest on ` 100 for 2 yr Annual instalment, x = ` 486680
unit time is called the rate of depreciation. Total number of instalments, n = 3 1. Simple interest and compound interest are equal for first
 100 × 12 × 2
=   = ` 24 Rule 8 If the present value of a article is P and it year period on the same sum and at the same rate.
 100   
Case II Amount when ` 100 is borrowed for 2 yr depreciate at the rate of R% per annum. Then,  x x x  2. If A and B are the amounts of a certain sum for two
∴ P= + 2 + 3
consecutive years, then simple interest for 1 yr = B − A
12  2  R n  n  1 + R
 1 + R  1 + R  
= 100 × 1 +  
A = P 1+ 100 ,Rule 1 (i) 

(i) Value of article after n yr = P × 1 −
R   100 
3. If certain sum at compound interest becomes x times in
 100   100  100 
   100  n 1 yr and y times in n 2 yr, then x 1 /n 1 = y 1 /n 2
= ` 
28 28 3136
= 100 × ×  P
25 25  25  (ii) Value of article n yr ago = n
 R 
∴ Compound interest for 2yr =  − 100 = ` 
3136 636
 1 − 
 25   25   100 
Difference between CI and SI =   636 
− 24 = `
36
 25  25 EXAMPLE 8. Given that carbon − 14 (C 14 ) decays at a
36 constant rate in such a way that it reduces to 50% in
If the difference between CI and SI is ` , then the 5568 yr. Then, the age of an old wooden piece in
25
sum = ` 100 which the carbon is only 12.5% of the original is
If the difference between CI and SI is ` 1, a. 16000 yr b. 16244 yr c. 16702 yr d. None of these
 100 × 25
then the sum = `   Sol. d. Let the rate of decay be R% and the age of the
 36  wooden piece be n yr.
If the difference between CI and SI is ` 90, then the sum
R  5568
Then, 
50  
⇒ 1 −
R 
 = 1 −   = (0.5)1/ 5568 …(i)
= 100 × × 90 = ` 6250
25
 100  100  100
 36 
12.5  R n R n
Here, the sum is ` 6250. ⇒ = 1 −  ⇒ (0.125) = 1 − 
100  100  100
Shortcut Method
Let the required sum be ` P. ⇒ (0.125) = 1 −
1/ n  R 
 ⇒ (0. 5) = 1 −
3/ n  R 
 …(ii)
 100  100
Here, difference = ` 90 and R = 12% 1
∴ Difference between CI and SI From Eqs. (i) and (ii), (0.5) 3/ n = (0.5) 5568
PR 2 (P)(12) 2 P × 144
90 = = ⇒ 90 = On comparing both sides, we get
(100) 2 (100) 2 100 × 100 3 1
= ⇒ n = 3 × 5568 = 16704 yr
∴ P = ` 6250 n 5568
Required sum = ` 6250
Rule 7 If a certain sum at compound interest becomes INSTALMENTS
A1 , in n yr and A 2 in ( n + 1) yr, then When a borrower pays the sum in parts, then we say
that he/she is paying in instalments.
(A 2 − A1 )
(i) Rate of compound interest = × 100% x x x
n A1 ∴ P= + +…+
A   R  2 n
(ii) Sum = A1  1  1 +  1 + R  
1 +
R

 A2   100   100   100 
MATHEMATICS Compound Interest 87 88 CDS Pathfinder

PRACTICE EXERCISE 21. A sum of money doubles itself at compound


interest in 15 yr. It will becomes 8 times in
(a) 30 yr (b) 40 yr
26. From the given statements, identify which of the
following or both are correct and then select the
appropriate option.
(c) 60 yr (d) 45 yr I. The Present worth of ` 169 due in 2 yr at 4% Pa
1. Kiran purchased a scooter for ` 24000. The value 11. A saving bank gives interest which compounds compound interest is 156.25.
of scooter is depreciating at the rate of 5% per annually. Raju deposited ` 100 and received 22. A man borrows ` 4000 at 8% per annum on
compound interest. At the end of every year he II. If the simple interest on a certain sum for 2 yr is
annum. Then, its value after 3 yr is ` 121 at the end of second year. Rate of ` 120 and compound interest is ` 129, then the
(a) ` 20577 (b) ` 20977 (c) ` 20677 (d) ` 20877 compound interest per annum is pays ` 1500 as part payment of loan and
rate of interest must be 15%.
(a) 10% (b) 15% (c) 11.5% (d) 20.5% interest. How much does he still owe to the bank
2. If P be the principal amount and the rate of after three such annual payments? Which statement(s) is/are correct?
interest be r% per annum and the compound 12. The amount of a certain sum at compound (a) ` 1799 (b) ` 2000 (a) Only II (b) Only I
interest is calculated k times in a year, then interest for 4 yr at 10% in ` 4410. The sum is (c) Neither I nor II (d) Both I and II
(c) ` 169.25 (d) None of these
what is the amount at the end of n yr? (a) 3012.08 (b) 3015 (c) 3020.16 (d) 3016.9
nk nk
23. A sum of ` 10000 deposited at compound interest
(a) P  1 +
r 
(b) P  1 +
kr  PREVIOUS YEARS’ QUESTIONS
  13. The compound interest on ` 5000 for 3 yr at 8% becomes double after 5 yr. After 20 yr, the
 100k   100  for first year, 10% for second year and 12% for
n/ k n/ k amount will be 27. What is the compound interest on ` 1600 at 25%
(c) P  1 +
kr 
(d) P  1 +
kr  third year will be
  (a) ` 160000 (b) ` 40000 per annum of 2 yr compound annually? e 2012 II
 100   100k  (a) ` 1560.40 (b) ` 1500 (c) ` 1565.60 (d) ` 1652.80
(c) ` 50000 (d) ` 60000 (a) ` 700 (b) ` 750 (c) ` 800 (d) ` 900
3. The amount of a certain sum at compound 14. An amount of ` x at compound interest at 20%
interest for 2 yr at 5% is ` 4410. The sum is per annum for 3 yr becomes y. What is y : x? 24. Vinod and Karan each invested ` 15000 for 3 yr 28. The difference between compound interest and
(a) 3 : 1 (b) 36 : 25 at the same rate of interest but Vinod's simple interest for 2 yr at the rate of 10% over
(a) ` 4000 (b) ` 4200 (c) ` 3900 (d) ` 3800
(c) 216 : 125 (d) 125 : 216 investment is compounded annually while principal amount of ` X is ` 10. What is the
4. A person borrowed ` 7500 at 16% compound Karan’s investment is charged on simple value of X? e 2014 II
interest. How much does he have to pay at the 15. The compound interest on ` 2000 for 1 yr at the interest. (a) ` 100 (b) ` 1000 (c) ` 500 (d) ` 5000
end of 2 yr to clear the loan? rate of 8% per annum, when the interest is
I. Vinod receive more interest than karan. 29. The difference between compound interest and
(a) ` 9900 (b) ` 10092 (c) ` 11000 (d) ` 11052 compounded semi-annually
II. Data is insufficient to calculate the interest. simple interest at the same rate of interest R per
(a) ` 163.20 (b) ` 2163.20 (c) ` 3153.20 (d) ` 1163
5. If the rate of interest is 10% per annum and Which one is correct? cent per annum on an amount of ` 15000 for
is compounded half-yearly, then the principal of 16. ` 16000 invested at 10% per annum compounded 2 yr is ` 96. What is the value of R? e 2015 I
semi-annually amounts to ` 18522. Then, the (a) Only I (b) Only II
` 400 in 3/2 yr will amount to (a) 8% (b) 10%
(a) ` 463.00 (b) ` 463.05 period of investment is (c) Neither I nor II (d) Both I and II
(c) 12% (d) Cannot be determined
1 5
(c) ` 463.15 (d) ` 463.20 (a) 1 yr (b) 3 yr (c) 2 yr (d) yr 25. An amount is invested in a bank at compound
2 2 rate of interest. The total amount, including 30. There is 60% increase in an amount in 6 yr at
6. At compound interest, if a certain sum of money 25 simple interest. What will be the compound
interest, after first and third year is ` 1200 and
doubles in n yr, then the amount will be four 17. A sum compounded annually becomes times interest on ` 12000 after 3 yr at the same rate of
fold in 16 ` 1587, respectively.
interest? e 2015 I
(a) 2 n2 yr (b) n2 yr (c) 2n yr (d) 4n yr of itself in 2 yr. Then, the rate of interest per I. The principal amount will be ` 1403. (a) ` 2160 (b) ` 3120 (c) ` 3972 (d) ` 6240
annum is II. The rate of interest is 16%
7. The simple interest on a certain sum of money (a) 25% (b) 20% (c) 15%
1
(d) 7 % 31. A sum of ` 10000, is deposited for 1 yr at the
for 3 yr at 8% per annum is half the compound 2 Which statement(s) is/are correct? rate of interest 10% compounded half yearly.
interest on ` 4000 for 2 yr at 10% per annum. (a) Both I and II (b) Only I What will be the interest at the end of one year?
What is the sum placed on simple interest? 18. A sum of ` 3200 invested at 10% per annum
(c) Only II (d) Neither I nor II e 2015 II
(a) ` 1550 (b) ` 1650 (c) ` 1750 (d) ` 2000
compounded quarterly amounts to ` 3362, then
(a) ` 1000 (b) ` 1025 (c) ` 1050 (d) ` 1100
the time period is
8. What is the least number of complete years in 1
(a) 1 yr
1
(b) yr (c) 2 yr (d) 1 yr
which a sum of money at 20% compound interest 2 2
will be more than doubled?
19. A sum amount to ` 9680 in 2 yr and to ` 10648 ANSWERS
(a) 7 (b) 6 (c) 5 (d) 4
in 3 yr compounded annually. Then, the sum and
9. The difference between the simple interest and rate of interest, respectively are 1 a 2 a 3 a 4 b 5 b 6 c 7 c 8 d 9 c 10 b
the compound interest (compounded annually) on (a) ` 8000, 10% (b) ` 8500, 10% 11 a 12 a 13 d 14 c 15 a 16 a 17 a 18 b 19 a 20 a
` 1250 for 2 yr at 8% per annum will be (c) ` 8500, 9% (d) ` 8000, 9% 21 d 22 c 23 a 24 b 25 d 26 d 27 d 28 b 29 a 30 c
(a) ` 18 (b) ` 2 (c) ` 8 (d) ` 4
20. If the value of a machine depreciates by 10% of 31 b
10. The compound interest on a sum for 2 yr is ` 832 its value at the beginning of the year and its
and the simple interest on the same sum at the present value is estimated as ` 10935, then what
same rate for the same period is ` 800. What is was its value three years back?
the rate of interest? (a) ` 15000 (b) ` 7000
(a) 6% (b) 8% (c) 10% (d) 12% (c) ` 8050 (d) None of these
MATHEMATICS Compound Interest 89 90 CDS Pathfinder

n
11. (a) Given, principal (P) = ` 100 and time ( n) = 1 × 2 = 2 half-yearly  
3362 = 3200  1 +
10 
HINTS AND SOLUTIONS Amount (A) received after 2 yr = ` 121
Q

CI = P   1 +
R 
n
 − 1
 ⇒


4 × 100 

Let rate of interest = R% per annum  100   n 2 n
n 3362  410   41   41 
Q A = P  1 +
R  [Rule 1. (ii)] ⇒ =  ⇒  =  
n
9. (c) Given, P = ` 1250, R = 8% per  [Rule 1. (i)]
3200  400   40   40 
x  1 +
1. (a) Given, P = ` 24000, R = 5% per R   100  2
∴  = 2x CI = 2000  1 +
4 
 100  annum and n = 2 yr  − 2000 ⇒ n = 2 ⇒ 4t = 2 yr [as n = 4t ]
2
 100 
121 = 100  1 +
annum and n = 3 yr P × R×T R 
n
Q Simple interest = ∴  1
n
⇒ 1 + R  = 2  100   26 
2
∴ t = yr
∴ A = P  1 −
R    …(i) 100 = 2000 ×   − 2000
 [Rule 8 (i)]  100   25  2
 100  1250 × 8 × 2 121  100 + R 
2
∴ Simple interest = ⇒ =  = 2163. 20 − 2000 = ` 163.20 19. (a) Given, A = ` 9680, A = ` 10648
3 Let it becomes four fold in N yr. 100  100  1 2
= 24000  1 −
5  100
 N
16. (a) Given, principal (P) = ` 16000 and n = 2 yr
x  1 +
 100  R  = ` 200
2
100 + R 
2
= 4x ⇒   = 
Then,  11
 100   Amount received at the end of period ∴ Rate of compound interest
3
Compound interest  10   100  A −A
= 24000 × 
95  Q (A) = ` 18522
 1 + R 
N
11 100 + R =
2 1
× 100% [Rule 7 (i)]
 100  ⇒ =4  n

= P   1 +
  R  ⇒ = Let time = t yr
 100   − 1 [Rule 1. (ii)] 10 100
A
 100 
1
= ` 20577  ∴ n = 2t
N 11 × 100 10648 − 9680
⇒ 100 + R = = × 100% = 10%
22 =  1 +
R 
2. (a) Given, principal amount = ` P ⇒ 
2
Rate = 10% per annum i.e 5% half-yearly
Q CI = 1250  1 +
8  10 9680
r  100   − 1250 n
A = P  1 +
Rate of interest, R = % per annum and  100  R  A 
n
⇒ 100 + R = 110 Q  [Rule 1. (i)]
 100  ∴ Sum = A  1 
k 2n N
⇒  1 +
R 
=  1 +
R  2
1
[Rule 7 (ii)]
∴ R = 110 − 100 = 10 
= 1250 × 
Time, T = nk   108 
 100   100   − 1250 n  A2 
18522 = 16000  1 +
 100  5 
T ∴ Rate of interest = 10% ∴ 
A = P  1 +
R  
2
 [Rule 1. (i)] [from Eq. (i)] 100  = 9680 
9680 

Q
 100  = 1458 − 1250 = ` 208 12. (a) Refer to example 3. n  10648 
⇒ 
∴ N = 2n yr 18522   21 
nk
∴ Difference in SI and CI  =  2 2
  13. (d) Given, P = ` 5000,  16000   20 
= 9680 
110   10 
∴ A = P 1 +
r  7. (c) Let the principal amount be ` P.  = 9680  
 = 208 − 200 = ` 8 R = 8%, R = 10%, R = 12% 3  121   11 
100 k 
n n
⇒ 
1 9261  21   21   21 
  By given condition, SI = CI 1 2 3
 =   ⇒  =   9680 × 100
2 Shortcut Method and n = n = n = 1 yr  8000   20   20   20  = = ` 8000
1 2 3
3. (a) Let the principal be = ` x. 121
P ×8 ×3 1 Here, p = ` 1250 and R = 8% Q Amount ⇒ n=3
⇒ = 20. (a) Let the value of machine 3 yr ago be
Given, n = 2 yr, R = 5% , A = ` 4410 100 2 PR2 
n
R  1
n
R  2 R  3
n
n = 2t ⇒ 2t = 3
∴ Required difference = = P1 + 1  1+ 2  1+ 3 
Q
` x.
n
 2
 ( 100)2 3 1
A = P  1 +
R   10   100   100   100  ∴ t = yr = 1 yr
Q  [Rule 1. (i)]  4000  1 +  − 4000 and given, P = ` 10935, R = 10% and
 100  100  2 2
  [Rule 6. (i)] [Rule 5] n = 3 yr
2 1250 × 64 17. (a) Let sum/ principal be ` x and the rate n
x = P  1 −
R 
4410 = x  1 +
5  = = `8
∴  [Rule 1 (ii)]  12    [Rule 8 (i)]
= 5000 ×  1 +
100 × 100 8  10   be R% per annum. Q

 100  1+ 1+  100 

24P 1 
= 4000 ×
121
− 4000   100   100   100   ∴ Amount, A =
25
x and n = 2 yr
10. (b) Given, CI = ` 832, SI = ` 800, 3
∴ x  1 −
2
100 2   10 
⇒ 4410 = x  
21  27 11 28 
100
n = 2 yr =  5000 × × ×  = ` 6652.80
16  = 10935
 20   100 
 25 10 25 
n
A = P  1 + 
1 R
= [ 4840 − 4000] 
n

CI = P  1 +
4410 × 400 R  Q  [Rule 1. (i)] 3
x 
 − 1  100  90 
⇒ x= = ` 4000 2 ∴Compound interest = 665280
. − 5000 ⇒  = 10935
441 24P  100   2  100 
⇒ = 420 = ` 1652.80 ∴
25
x = x  1 +
R 
4. (b) Given, P = ` 7500, R = 16% and [Rule 1. (ii)]  10935 × 10 × 10 × 10
100 16  100  ∴ x= = ` 15000
n = 2 yr 420 × 100 14. (c) Given, P = ` x , R = 20% per annum, 9×9×9
 2

832 = P  1 +
∴ P= = ` 1750 R  n = 3 yr 2
n ∴  − 1 …(i) 25  R 
⇒ = 1 +
Q A = P  1 +
R  
 100 
24 21. (d) Let the sum be ` x and rate be R %
 [Rule 1. (i)]  and A=` y 16  100 
 100  8. (d) Let the sum of money be ` P.
per annum.
P × R×T n
A = P  1 +
2 Also, SI = R  5
2
 R 
2
∴ Amount, A = ` 2x
 [Rule 1. (i)] ⇒   = 1+
= 7500  1 +
16  ∴ Amount = ` 2P
Q 
 100  100   4  100  and time, n = 15 yr
 100  n
P × R× 2
A = P  1 +
R  3 n
y = x  1 +
20 
A = P  1 +
⇒  [Rule 1. (i)] ⇒ 800 = 5  R  R 1 R 
2
 ∴  ⇒ = 1 +  ⇒ =  [Rule 1. (i)]
= 7500 
116  100  100  100  4  100 
Q
 100 
 100 4
 100  n 40000
2P = P  1 +
20  ⇒ P=
3 15
y = x   ⇒ x  1 +
R 
⇒ 6 y 216 ∴ R = 25%
29 29  ⇒ = Then,  = 2x
= 7500 × ×  100  R 5 x 125  100 
25 25 n From Eq. (i), 18. (b) Here, P = ` 3200,
2P  6  ⇒ y : x = 216 : 125 1 + R 
15
= 12 × 841 = ` 10092 ⇒ =  A = ` 3362 ⇒ =2
5 40000  R2 2R    …(i)
P 832 =  +  15. (a) Given, principal ( P ) = ` 2000  100 
5. (b) Refer to example 2. n R  10000 100  [since, amount is payable quarterly]
2 =  
6 Time = 1 yr
⇒ 10 Suppose the sum becomes eight times in
6. (c) Let the principal be ` x. 5 40000  R ∴ R = 10% per annum = % quarterly
+ 2 Rate of interest = 8% per annum ‘n’ yr, then
⇒ 832 =  4
∴ Rate = ` R, Amount = ` 2x 100  100 
n
x  1 +
On putting n = 4, we get and let time = t R 
Since, the interest is compounded  = 8x [by given condition]
and Time = n yr  100 
4
 6  = 1296 = 2 (approx) ⇒ 832 = 4R + 800 semi-annually. ⇒ n = 4t
 
n
A = P  1 +
R  n n
A = P  1 +
R 
⇒  1 +
5 8 R 
Q  [Rule 1. (i)] 625 ∴ R=
32
= 8% Then, rate ( R) = % = 4 half-yearly Q  [Rule 1. (i)]  =8= 2
3
…(ii)
 100  2  100   100 
∴ Least number of years = 4 4
11
MATHEMATICS Compound Interest 91 92 C D S Pathfinder

3
 R  
15 Amount after 3rd yr = ` 1587 27. (d) P = ` 1600, r = 25% and n = 2 yr
[but 23 =   1 +   from Eq. (i)] n
∴ A = P 1+
R 
  100  
3
x  1 +
R  [Rule 1. (i)]
 = ` 1587 …(ii)  100 
n 45  100 
⇒  R   R 
1 +  = 2 = 1 +
3
 2
= 1600  1 +
 100   100  On dividing Eq. (ii) by Eq. (i), we get 25 
[on comparing] 2
 100 
 1 + R  = 1587 = 529
∴ n = 45 yr   5 5
 100  1200 400 = 1600 × × = ` 2500
Shortcut Method 4 4
R 23 R 3
1+ = ⇒ =
Since, a sum of money at compound 100 20 100 20 ∴ Compound interest
interest doubles itself in 15 yr. = ` 2500 − `1600 = ` 900
∴ R = 15%
Hence, it will becomes eight times
(23 times) in = 15 × 3 = 45 yr
22. (c) Amount due at the end of 1 yr
Put R = 15% in Eq. (i),
x  1 +

15 
 = ` 1200 ⇒ x =
100 
1200 × 100
115
28. (b) Refer to example 6.
29. (a) Given, principal = ` 15000

Rate, R = ?
PROFIT AND LOSS
= 4000  1 +
8  ∴ x = ` 1043.478
 = ` 4320
 100  Difference, D = ` 96
26. (d) I. Given, R = 4%, n = 2 yr and
∴ Amount due after 1 yr PR2
A = ` 169, P = ? For 2 yr Difference, D =
= ( 4320 − 1500) = ` 2820 n ( 100)2
A = P  1 +
R 
 [Rule 1. (i)]
Amount due at the end of second year  100  [Rule 6 (i)] Generally (2-4) questions have been asked from this chapter. Generally questions are based on basic
= 2820  1 +
8  15000 × R2
 = ` 3045.60
2
169 = P  1 +
4  ⇒ 96 = formulae and tricks explained in this chapter which are helpful to solve the problems more easily
 100   100 × 100
 100  and quickly.
∴ Amount due after second year 2 ⇒ R2 = 64
⇒ 169 = P  
26
= (3045.60 − 1500) = ` 1545.60  25  ∴ R = 8%
Amount due at the end of third year
169 × 25 × 25 30. (c) ∴ Let the principal be `x.
= 1545.60  1 +
8  P= = ` 156.25
 = ` 1669.25 26 × 26 Then, SI = `
60x
 100 
Amount due after the payment of third II. Given, SI = ` 120 , n = 2 yr
100
Principal × Rate × Time
Cost Price (CP)
instalment Q SI = The price, at which an article is bought is called its cost price. All the overhead expenses in the
and CI = ` 129. 100
= ( 1669. 25 − 1500) = ` 169.25 P × R×T 60x x × Rate × 6 transection like freight, damage etc., are added to the cost price.
SI = ⇒ = [QTime = 6 yr]
23. (a) Let the sum be ` x, then 100 100 100
5 5 P × R× 2 60 = Rate × 6 Selling Price (SP)
x  1 +
R   R  120 = ⇒ PR = ` 6000
 = 2x ⇒  1 +  =2
 100   100  100
∴ Rate = 10%
6000 The price at which an article sold is called its selling price.
…(i) ∴ P= …(i) Again principal = ` 12000, Time = 3 yr
The amount after 20 yr, R
Time
Profit (SP>CP) When an article is sold at a price more than its cost price, then profit is earned.
Amount = Principal  1 +

4  R 
n
 Rate 
R   Profit = Selling price − Cost price
20 5
CI = P   1 +  − 1 [Rule 1. (ii)] 
x  1 +
R 
= x   1 +

    100 
 100   100  
  100   3 • Loss = Cost price − Selling price
= 12000  1 +
 2
 10 
129 = P   1 +
= 24 x = 16x [from Eq. (i)] R  
 − 1 .  100  Loss (CP>SP) When an article is sold at a price lower than its cost price, then loss is incurred.
= 16 × 10000 = ` 160000  100   3
Profit (SP − CP)
= 12000  
11
[put x = ` 10000] 1290000 = P [( 100 + R)2 − 1002 ]. • Profit % = × 100% = × 100%
 10  CP CP
24. (b) Data is insufficient as the rate of = P [ R + R × 200].
2
11 × 11 × 11 Loss (CP − SP)
interest is not given to calculate further. 6000 2 = 12000 ×
= [ R + R × 200] 1000 • Loss % = × 100 % = × 100%
25. (d) Let amount be ` x and rate of R CP CP
interest is R % annually. [from Eq. (i)] = 12 × 121 × 11 = ` 15972
1290000 = 6000R + 1200000 ∴ CI = Amount − Principal Note Profit and loss percentage is always calculated as the percentage of CP unless otherwise specified.
According to the questions,
90000
Amount after 1st yr = ` 1200 R= = 15% = 15972 − 12000 = ` 3972
6000
x  1 +
R 
 = 1200 …(i) Hence, both statement are correct. 31. (b) Refer to example 2. List Price/Marked Price
 100  The price of an article which is displayed on the price tag is known as marked price. It is the normal
price of the thing/products without a discount. Sometimes the shopkeeper increases or decreases the
cost price, then this price is taken as the list price of the article.
MATHEMATICS Profit and Loss 93 94 CDS Pathfinder

Sol. b. Here, x = 10 EXAMPLE 9. If the cost price of 18 chairs be equal


Important Rules and Formulae Rule 4 When two different articles are sold at the 2 to selling price of 16 chairs. The gain percent is
Overall loss % =   %
x
same selling price, getting gain/loss of x% on the first and ∴
Rule 1 If there is a profit of r%, then  10 a. 12% b. 12.5%
gain/loss of y% on the second, then the overall % gain or 2 c. 14% d. 15.5%
(100 + r ) × CP 100 × SP % loss in the transection is given by =   % = 1%
10
(i) SP = (ii) CP =  10 Sol. b. Given, n = 18, m = 16 [Q n > m]
100 (100 + r )
 100( x + y) + 2xy 
(100 + x) + (100 + x)
%. The above expression represent Rule 5 A person buys two items for ` S. One is sold at  18 − 16 Q n − m × 100
  ∴ Profit percentage =   × 100
EXAMPLE 1. If cost price of a fan is ` 720. If there is  16   m 
2 a loss of r% and the other at a gain of R%, if each item
a profit of 16
%. Then, selling price is overall gain or loss accordingly as its sign is positive or 2 100
3 was sold at the same price, then = × 100 = = 12.5%
negative.
S × (100 + R) 16 8
a. ` 840 b. ` 940 Cost price of item sold at loss =
c. ` 1050 d. None of these When two different articles are sold at the same selling (100 − r ) + (100 + R)
2
Sol. a. Given, r = 16 % and cost price = ` 720
price getting a gain of x% on the first and loss of x% on
Cost price of item sold at gain =
S × (100 − r ) DISCOUNT
3
the second, then the overall % loss in the transection is (100 − r ) + (100 + R)
2 The reduction allowed on the marked price of an article
given by   %.
x
720 × 100 + 16 
2 is called as discount. Discount is always reckoned on the
CP × (100 + r)  3  10  EXAMPLE 7. Sudhir bought two boxes for ` 1300. marked price.
∴Selling price (SP) = =
100 100 He sold one box at a profit of 20% and the other at a
loss of 12%. If the selling price of both boxes is the Selling price = Marked price − Discount
720 × 100 +
50 Note That in such questions there is always a loss.

 3  = 720 × 350 = ` 840 same, then the cost price of each box is Rule 8 If discount allowed is r%, then
= EXAMPLE 4. A trader sells two cycles at ` 1188 each
100 300 a. ` 700 and ` 500 b. ` 750 and ` 550 (100 − r )
and gains 10% on the first and loses 12% on the c. ` 800 and ` 600 d. None of these Selling price = × Marked price
Rule 2 If there is a loss of r%, then second. What is the profit or loss percent on the 100
whole? Sol. b. Given, cost of two boxes S = ` 1300, profit on first
(100 − r ) 100 × SP box R = 20% EXAMPLE 10. If the marked price of a fan is ` 700
(i) SP = × CP (ii) CP = a. 1% loss b. 1% gain c. No loss no gain d. 3.2% loss
100 (100 − r ) and loss on second box r = 12% and a discount of 10% is given on it, then what is the
Sol. d. When there is a profit of x% and loss of y%, then selling price of the fan?
the resultant Cost price of box sold at loss
EXAMPLE 2. When Selling price is ` 75 and there is 1300 × (100 + 20) 1300 × 120 a. ` 500 b. ` 575
Profit/Loss percentage =  x − y −
a loss of 12%. Then, cost price is xy  = = = ` 750
% (100 − 12) + (100 + 20) 88 + 120 c. ` 610 d. ` 630
 100
a. ` 22 b. ` 44
Cost price of box sold at gain Sol. d. Given, r = 10%
c. ` 55 d. ` 85.22 Here, x = 10% and y = 12
1300 × (100 − 12) 1300 × 88 (100 − 10)
10 × 12 = = = ` 550
Sol. d. Given, r = 12% and selling price = ` 75 ∴Profit/Loss percentage = 10 − 12 −  % = −3.2% (100 − 12) + (100 + 20) 88 + 120 Selling price = × 700
 100  100
100 × SP 100 × 75 100 × 75 Rule 6 If a trader professer to sell his goods at cost 90 × 700
∴ Cost price = = = = 85.22 = = ` 630
(100 − r) (100 − 12) 88 EXAMPLE 5. Harish sold two scooters, each for price, but uses false weights, then gain % 100
` 24000. If he makes 20% profit on the first and 15% true weight − false weight
Rule 3 (i) When there are two successive profits or losses Rule 9 If a shopkeeper marks his items at x% above the
loss on the second. What is his gain or loss percent in = × 100
of x% and y%, then the resultant profit or the transection? false weight cost price and allows customers a discount of y% then
loss percent is given by 20 20 32 32  xy 
 ( ± x)( ± y) a. loss % b. gain % c. loss % d. gain % EXAMPLE 8. A dishonest dealer sell his goods at there is  x − y −  % profit or loss according to
± x ± y +  %. 41 41 73 73
cost price, but he uses a weight of 960 gm for the kg  100 
 100  Sol. a. Here, x = 20 and y = −15 weight, then the percentage of gain is positive or negative sign, respectively.
1 1 1 1
Note l Take positive for profit and negative for loss.  100( x + y) + 2xy  a. 2 % b. 4 % c. 6 % d. 3 %
∴ Overall gain/loss % =  % 3 6 9 2 EXAMPLE 11. A dealer marked his goods 20% above
(100 + x) + (100 + y) 
l If the sign of result is positive, then there is a total gain.
If the sign of result is negative, then there is a total loss. true weight − false weight the cost price and allows a discount of 10%. Then, his
Sol. b. Gain % = × 100
l

100( 20 − 15) + 2 × 20 × −15 gain percent is


=
false weight
EXAMPLE 3. By selling a watch for ` 132 a trademan %
 (100 + 20) + (100 − 15)  1000 − 960 40 1 a. 2% b. 4%
got two successive profits of 10% and 20%, then the = × 100 = × 100 = 4 % c. 6% d. 8%
−100 −20 960 960 6
resultant profit percentage is = %= %
205 41 Rule 7 If cost price of ‘n’ articles is equal to the selling Sol. d. Given, x = 20% and y = 10%
a. 22% b. 30%
Profit/Loss percentage =  x − y −
Which represents loss. price of ‘m’ articles, then xy 
c. 32% d. 34% %
 100
 n − m
Sol. c. Given, the successive profits are 10%, 20%. EXAMPLE 6. Vikram sold two horses for ` 990 each Profit percentage =   × 100% ( n > m) 20 × 10
 20 × 10 gaining 10% on the one and lossing 10% on the
 m  =  20 − 10 −  % = 8%
So, resultant profit percentage = 10 + 20 + %  100 
 100  other. Find his gain or loss percent.  m − n
and loss percentage =   × 100% ( m > n)
= 32% a. 1% gain b. 1% loss c. 4 % gain d. 4 % loss  m  Hence, positive sign shows profit i.e. profit of 8%.
MATHEMATICS Profit and Loss 95 96 CDS Pathfinder

Successive Discounts Rule 11 If marked price of an item is ` x and the 13. A discount series of 10%, 20% and 40% is equal 23. A fruit-seller buys lemons at 2 for a rupee and
to a single discount of
(Discount Series) successive discount rates are r1 %, r 2 %, r 3 % and so on, (a) 50% (b) 60% (c) 56.8% (d) 70.28%
sells them at 5 for three rupees. What is his gain
per cent?
then selling price of the item 1 1
Rule 10 Single discount equivalent to three successive 14. Successive discounts of 12 % and 7 % are (a) 10% (b) 15% (c) 20% (d) 25%
 r  r  r  2 2 24. A trader marks 10% higher than the cost price.
discounts r1 %, r 2 % and r 3 % SP = x × 1 − 1  1 − 2  1 − 3 
 100   100   100  given on the marked price of a cupboard. If the He gives a discount of 10% on the marked price.
  r  r  r  customer pays ` 2590, then what is the marked
= 1 − 1 − 1  1 − 2  1 − 3  × 100% In this kind of sales how much per cent does the
  100   100   100   price?
  EXAMPLE 13. The marked price of a watch is ` 1600. trader gain or loss?
The shopkeeper gives successive discount of 10%, r% (a) ` 3108 (b) ` 3148 (c) ` 3200 (d) ` 3600 (a) 5% gain (b) 2% gain (c) 1% loss (d) 3% loss
Note This formula also can be apply for more than three
successive discounts. to the customer. If the customer pays ` 1224 for the 15. The difference between a discount of 40% on 25. One saree was purchased for ` 564 after getting
watch, then the value of r is ` 1000 and two successive discounts of 35% and a discount of 6% and another saree was
EXAMPLE 12. A single discount which is equivalent a. 20% b. 35% c. 37% d. 15% 5% on the same amount is purchased for ` 396 after getting a discount of
to two successive discounts of 20% and 5% is (a) ` 15.50 (b) ` 16.50 (c) ` 17.50 (d) ` 18.00 1%. Taking both the items as a single
Sol. d. Given, cost price x = ` 1600, r1 = 10% transaction, what is the percentage of discount?
a. 20% b. 22% c. 24% d. 25% 16. A dealer buys an article listed at ` 100 and gets
and r2 = r % (a) 3.5% (b) 4% (c) 7% (d) 7.5%
Sol. c. Given, r1 = 20%, r2 = 5% two successive discounts of 10% and 20%. He
∴ Selling price = Cost price 1− 1  1− 2 
r r spends 10% of the cost price on transport etc. At 26. A dishonest dealer professes to sell his good at
∴Single discount equivalent to two succesive discount  100  100
20% and 5% what price should he sell the article to earn a cost price but uses a false weight and thus gains
25%. For a kilogram he uses a weight of
1224 = 16001−
10   r  profit of 15%?
= 1− 1−
20   5   1− 
 1−  × 100%  100  100
  100  100  (a) ` 90 (b) ` 91 (c) ` 91.08 (d) ` 91.10 (a) 700 g (b) 750 g (c) 800 g (d) 850 g
r 1224 r 1224 17. An item costing ` 200 is being sold at 10% loss. 27. A person bought two old scooters for ` 9000. By
= 1− ×  × 100% = 1−  × 100%
4 19 19 ⇒ 1− = ⇒ = 1−
 5 20  25 100 160 × 9 100 1440 If the price is further reduced by 5%, then the selling one at a profit of 25% and the other at a
6 ∴ r = 15% selling price will be loss of 20%, to neither gain nor loses. The cost of
= × 100% = 24%
25 (a) ` 170 (b) ` 171 (c) ` 175 (d) ` 179 each scooter is
18. A man bought a number of oranges at 3 for a (a) ` 3500, ` 500 (b) ` 4500, ` 4500
rupee and an equal number at 2 for a rupee. At (c) ` 4000, ` 5000 (d) ` 5300, ` 3700

PRACTICE EXERCISE what price per dozen should he sell them to


make a profit of 20%?
(a) ` 4 (b) ` 5 (c) ` 6 (d) ` 7
28. The manufacturer of a certain item can sell all
he can produce at the selling price of ` 60 each.
It costs him ` 40 in materials and labour to
19. A milk vendor bought 28 L of milk at the cost of produce each item and he has overhead expenses
7. Sneha gains 10% on selling a pen. If she sells it ` 8.50 per L. After adding some water, he sold of ` 3000 per week in order to operate the plant.
1. By selling an article for ` 247.50, Sonu get a The number of items he should produce and sell in
at double the price, the profit per cent is the mixture at the same price. If he gains 12.5%,
profit of 12.5%. The cost of the article is order to make a profit of atleast ` 1000 per week is
(a) 120% (b) 60% (c) 100% (d) 200% then how much water did he add?
(a) ` 220 (b) ` 205 (c) ` 210 (d) ` 200
(a) 5.5 L (b) 4.5 L (a) 400 (b) 300 (c) 250 (d) 200
2. If cost price of a fan is ` 720 and its SP is ` 840. 8. On selling an article for ` 240, a trader loses (c) 3.5 L (d) 2.5 L
4%. In order to gain 10%, he must sell the article 1
Find the gain percent. 29. A person sold a table at a profit of 6 %. If he
2 1 7
for 20. A man sold two watches, each for ` 495. If he 2
(a) 16% (b) 16 % (c) 16 % (d) 16 % (a) ` 275 (b) ` 280 (c) ` 285 (d) ` 300 gained 10% on one watch and suffered a loss of had sold it for ` 1250 more, he would have
3 3 3
9. By selling 8 dozen pencils, a shopkeeper gains 10% on the other, then what is the loss or gain gained 19%
3. By selling an article for ` 110, a man losses 12%. the selling price of 1 dozen pencils. What is the percentage in the transaction? I. The CP of the table is ` 10000.
For how much should he sell it to gain 8%? gain? (a) 1% gain (b) 1% loss II. CP will ` 19000, if he had sold it for ` 2400 more
(a) ` 120 (b) ` 125 (c) ` 135 (d) ` 140 1 1 2 1 (c) 100/99% loss (d) No gain no loss and gained same profit
(a) 12 % (b) 13 % (c) 14 % (d) 87 %
2 7 7 2
4. A man buys 4 tables and 5 chairs for ` 1000. If 21. Jyoti bought a computer system for ` 40000. She Which one is correct?
he sells the tables at 10% profit and chairs 20% 10. A man purchased a watch for ` 400 and sold it at sold it to Brajesh at a loss of 4%. If Brajesh sells (a) Only II (b) Only I
profit, he earns a profit of ` 120. Then, what is a gain of 20% of the selling price. The selling it for ` 40320 to Yash, then the profit per cent (c) Neither I nor II (d) Both I and II
the cost of one table? price of the watch is earned by Brajesh is
(a) ` 300 (b) ` 320 (c) ` 440 (d) ` 500 30. A bookseller sells a book at a gain of 10%. If he
(a) ` 200 (b) ` 220 (c) ` 240 (d) ` 260 (a) 3% (b) 5%
had bought it at 4% less and sold it for ` 6 more,
11. A person A sells a table costing ` 2000 to a (c) 7% (d) 10% 3
5. If selling price of 8 articles is equal to the cost person B and earns a profit of 6%. The person B he would have gained 18 %
price of 10 articles, then per cent gain or loss is sells it to another person C at a loss of 5%. At 22. List price of a video cassette is ` 100. A dealer 4
(a) 20% (b) 25% (c) 30% (d) 35% sells three video cassettes for ` 274.50 after I. The SP of the book is ` 165.
what price did B sell the table?
(a) ` 2054 (b) ` 2050 (c) ` 2024 (d) ` 2014 allowing discount at certain rate. The rate of II. The CP of the book is ` 150.
6. A man sells fans at the same price on one he discount allowed is
gain 20% and losses 20% on the other. His gain 12. What price did the seller mark at the printed Which one is correct?
(a) 7% (b) 7.5% (a) Neither I nor II (b) Both I and II
or loss is price of a watch purchased at ` 380, so that after
(c) 8% (d) 8.5% (c) Only I (d) Only II
(a) 4% loss (b) 4% gain giving 5% discount, there is 25% profit?
(c) Neither gain nor loss (d) 1% loss (a) ` 400 (b) ` 450 (c) ` 500 (d) ` 600
MATHEMATICS Profit and Loss 97 98 CDS Pathfinder

e 2013 II
PREVIOUS YEARS’ QUESTIONS
31. A person bought 8 quintal of rice for certain
(a) 10% loss
(c) 20% profit
(b) 10% profit
(d) 20% loss
HINTS AND SOLUTIONS
rupees. After a week, he sold 3 quintal of rice at
10% profit, 3 quintal of rice with neither profit 39. On a 20% discount sale, an article costs ` 596.
1. (a) Given, selling price of article Selling price of an article for a profit of ⇒ Selling price
nor loss and 2 quintal at 5% loss. In this What was the original price of the article? 25
= ` 247.50 and gain = 10%
= 1000 ×  1 −
% 35   5 
transaction, what is the profit? e 2012 I e 2014 I
2 240 × 100 110 240 × 110 1− 
(a) ` 720 (b) ` 735 (c) ` 745 (d) ` 775 = × =  100   100 
(a) 10% (b) 20% (c) 25% (d) None of these ∴ Cost price 96 100 96
40. A man buys 200 oranges for ` 1000. How many   = ` 275 [Rule 11]
32. The cost of two articles are in the ratio 3 : 5. If   65 95
oranges for ` 100 can be sold, so that his profit 100 = × × 1000 = ` 617.50
there is 30% loss on the first article and 20% = ` × 247.50 [Rule 1] 9. (c) Let selling price of 1 dozen pencil be
100 100
gain on the second article, then what is overall percentage is 25%? e 2014 II   100 + 25   ` x.
percentage of loss or gain? e 2012 I (a) 10 (b) 14 (c) 16 (d) 20   2   ∴ Selling price of 8 dozen pencils = ` 8x ∴Difference = ( 617. 50 − 600) = ` 17.50
(a) 2.25% gain (b) 5.25% loss 100 × 2 × 24750. and profit = `x 16. (c) List price of article be ` 100.
41. When an article is sold at 20% discount, the =` = ` 220
(c) 2% loss (d) None of these selling price is ` 24. What will be the selling 225 Q Cost price of 8 dozen pencils Cost price for dealer
price when the discount is 30%. 2. (b) Total gain = SP – CP = 8x − x = ` 7x ( 100 − 20) ( 100 − 10)
33. A cloth store is offering Buy 3, get 1 free. What e 2014 II
x 2 = × × 100
is the net percentage discount being offered by (a) ` 25 (b) ` 23 (c) ` 21 (d) ` 20 = (840 − 720) = ` 120 ∴ Gain, percent = × 100% = 14 % 100 100
120 2 7x 7
the store? e 2012 II ∴ Gain percent = × 100 = 16 % 80 × 90 × 100
42. A shopkeeper sells his articles at their cost price 720 3 10. (d) Given, CP = ` 400, let selling price be = = ` 72
1 but uses a faulty balance which reads 1000 g for 100 × 100
(a) 20% (b) 25% (c) 30% (d) 33 % 3. (c) Given, SP = ` 110 and loss = 12% ` x.
3 800 g. What is his actual profit percentage? Money spent on transport
Then, 400 + 20% of x = x
∴ Costprice = `  × 110 = ` 125
100 10
34. A person sold an article for ` 136 and got 15% (a) 25% (b) 20% e 2014 II
 88  x 4x = × 72 = ` 7.20
⇒ 400 + = x ⇒ = 400 100
loss. Had he sold it for ` x, he would have got a (c) 40% (d) 30% 5 5
profit of 15%. Which one of the following is Now, CP = ` 125, gain required = 8% 400 × 5 ∴Total cost price = 72 + 7. 20 = ` 79.20
43. A person selling an article for ` 96 finds that his ⇒ x= ⇒ x = ` 500
( 100 + 8)
correct? e 2012 II
loss per cent is one-fourth of the amount of ∴ SP = `  × 125 = ` 135 4 ∴ Selling price
(a) 190 < x < 200 (b) 180 < x < 190  100  ∴ Selling price = ` 500 ( 100 + 15)
rupees that he had paid for the article. What can = × 79. 20
(c) 170 < x < 180 (d) 160 < x < 170 4. (a) Let cost of one table and one chair 11. (d) The cost price of table for person B 100
be the cost price? e 2014 II
35. A man buys a television set which lists for be ` x and ` y, respectively. 2000 115 × 79.20
(a) Only ` 160 (b) Only ` 240 = 2000 + 6 × = = ` 91.08
` 5000 at 10% discount. He gets an additional (c) Either ` 160 or ` 240 (d) Neither ` 160 nor ` 240 ∴ Cost of 4 tables and 5 chairs 100 100
2% discount (after the first discount) for paying = ` 4x + 5 y = 2000 + 120 = ` 2120 17. (b) Given, cost of article = ` 200
cash. What does he actually pay for the set? 44. A milkman claims to sell milk at its cost price
but he is making a profit of 20% since, he has ∴ 4x + 5 y = 1000 ...(i) Selling price for person B ∴ Selling price of article = 95% of
e 2012 II
mixed some amount of water in the milk. What ∴ Now, profit on 4 tables 2120 × 5 ( 90% of 200)
(a) ` 4410 (b) ` 4400 (c) ` 4000 (d) ` 4500 4x = 2120 −
is the percentage of milk in the mixture? e 2015 I = 4x × 10% = ` 100
=
95
×
90
× 200 = ` 171
36. A merchant earns a profit of 20% by selling a 10 = 2120 − 106 = ` 2014 100 100
250 200
basket containing 80 apples which costs ` 240 (a) 80% (b) % (c) 75% (d) % Profit on 5 chairs = 5 y × 20% = ` y
3 3 4x 12. (c) Let marked price be ` x. 18. (c) Given, CP of 3 oranges of Ist variety
but he gives one-fourth of it to his friend at cost ∴ + y = 120 ...(ii)
10 Selling price after 5% discount =`1
price and sells the remaining apples. In order to 45. The value of a single discount on some amount
5 19 1
earn the same profit, at what price must he sell which is equivalent to a series of discounts Now, on solving Eqs. (i) and (ii), we get =x− x = x CP of 1 orange of Ist variety = `
each apple? e 2012 II of 10%, 20% and 40% on the same amount, is x = ` 200 100 20 3
(a) ` 3.00 (b) ` 3.60 (c) ` 3.80 (d) ` 4.80 equal to e 2015 II 5. (b) Refer to example 9. Profit = SP − CP =
19
x − 380 CP of 2 oranges of IInd variety = ` 1
20 1
37. A person sold an article for ` 3600 and got a (a) 43.2% (b) 50% (c) 56.8% (d) 70% 6. (a) Refer to example 4. CP of 1 orange of IInd variety = `
19
profit of 20%. Had he sold the article for ` 3150, 7. (a) Let the selling price be ` 100. x − 380 2
46. A cloth merchant buys cloth from a weaver and Profit % = 20 × 100
how much profit would he have got? e 2013 II cheats him by using a scale which is 10 cm longer SP × 100 Total CP of 2 oranges of different
∴ Cost price = [Rule 1] 380 1 1 5
(a) 4% (b) 5% (c) 6% (d) 10% than a normal metre scale. He claims to sell 100 + 10 19 variety = + = `
100 × 100 1000 x − 380 3 2 6
38. Two lots of onions with equal quantity, one cloth at the cost price to his customers, but while = =` 25 = 20 × 100
selling uses a scale which is 10 cm shorter than 110 11 380 Profit on 2 oranges of different variety
costing ` 10 per kg and the other costing ` 15 per
Now, if SP is ` 200. 20 5 20 5 1
kg, are mixed together and whole lot is sold at a normal metre scale. What is his gain? e 2016 I x = 475 × = ` 500 = 20% of = × =`
∴ Gain = `  200 −  = `
2 1 1000 1200 19 6 100 6 6
` 15 per kg. What is the profit or loss? (a) 20% (b) 21% (c) 22 % (d) 23 %
9 3  11  11 ∴ SP of 2 oranges of different variety
13. (c) Refer to example 12.
1200 / 11
Gain percent = × 100 = 120% 5
= + =`1
1
ANSWERS 1000 / 11 14. (c) Refer to example 13.
15. (c) Case I Discount = 40 %
6 6
8. (a) Given, selling price of an article Hence, SP of 12 oranges is ` 6.
1 a 2 b 3 c 4 a 5 b 6 a 7 a 8 a 9 c 10 d ⇒ Selling price = 60% of 1000 19. (c) Given, cost price of 1L = ` 8.50
= ` 240
11 d 12 c 13 c 14 c 15 c 16 c 17 b 18 c 19 c 20 b 60
Loss = 4% = × 1000 = ` 600 ∴Total CP of milk = 28 × 850
. = ` 238
21 b 22 d 23 c 24 c 25 b 26 c 27 c 28 d 29 b 30 b 100
∴Cost price of an article for the loss of 4% ∴ Profit = 125. % of 238
31 d 32 d 33 b 34 b 35 a 36 c 37 b 38 c 39 c 40 c Case II Two successive discounts are
240 × 100 125
.
41 c 42 a 43 c 44 b 45 c 46 a =` [Rule 2] of 35% and 5%. = × 238 = ` 29.75
96 100
MATHEMATICS Profit and Loss 99 100 CDS Pathfinder

Let he added x L of water. Total amount after discount 19x 213x 32. (d) Let the CP of two articles be 3x and 1 20 x
⇒ − = 1250 But he sold of his apples i.e. 20 apples ⇒ x = 24 + x ⇒ x = 24 +
∴ Profit = x × 85
. ⇒ 29. 75 = x × 85
. = 564 + 396 = ` 960 100 200 5 x, respectively. 4 100 5
⇒ x = 1250 × 8 = 10000 3x × 70 21x for ` 60. x 4x
∴ x = 3 .5 L ∴ Discount per cent ∴ SP of first article = = ⇒ x− = 24 ⇒ = 24
II. If SP =
213x
+ 2400 and gain 100 10 ∴ SP of remaining 60 apples 5 5
20. (b) Refer to example 6. 1000 − 960 40
= × 100 = % = 4% 200 = ( 288 − 60) = ` 228 24 × 5
1000 10
[Rule 2] ⇒ x= = 30
21. (b) Cost price of computer for Brajesh = 19%, then and SP of second article 228 4
4 26. (c) Here, true weight = 1000 213x ∴ SP of 1 apple = = ` 3.80
= 40000 − ( 40000) = ` 38400 + 2400 − x 5x × 120 60 ∴ Cost price of an article = ` 30
100 and gain = 25% = = 6x
19 = 200 × 100 100 37. (b) Let the cost price of the article be ` x. Now, selling price after 30% discount
Selling price of computer for Brajesh 1000 − false weight x
30
= ` 40320 ⇒ 25 = × 100 19x 13x ∴ Total SP Given, profit percentage = 20% = 30 − 30% of 30 = 30 − × 30
false weight ⇒ − = 2400 100
∴ Profit amount = (40320 − 38400) 100 200 = 6x +
21x 60x + 21x 81x
= = Now, x +
20x
= 3600 ⇒
120x
= 3600
= ` 1920 false weight ⇒ x = 2400 × 8 = 19200 10 10 10 100 100 = 30 − 9 = ` 21
⇒ = 1000 − false weight
∴ Profit percentage earned by Brajesh 4 Hence, I is correct but II is incorrect. ∴ Total CP = 3 x + 5 x = 8x ∴ x = ` 3000 42. (a) Q Actual profit percentage
1920 4 81x 81x − 80x x Fair weight − Unfair weight
= × 100 = 5% ⇒ false weight = 1000 × = 800 30. (b) Let CP = x ∴ Profit = − 8x = = Now, profit percentage when the article = × 100 %
38400 5 10 10 10 is sold for ` 3150 Unfair weight
110x 11x
22. (d) Given, list price of a video cassette then SP = = ∴Overall percentage of gain percentage 3150 − 3000 150 1000 − 800
Hence, he uses the weight 800 g. × 100 = × 100 = 5%
100 10
x = × 100% = 25%
= ` 100 27. (c) Let cost price of one scooter be ` x. 96x 24x × 100 3000 3000 800
Now, CP = 96% of x = =` x × 100
Let the rate of discount be r%. ∴ Cost price of other scooter 100 25 = 10 = = 1. 25% 38. (c) Let each lot of onion contains x kg. 43. (c) Let the cost price of an article be ` x.x
8x 10 × 8x Then, total cost price of these two lots
Selling price of 3 video cassette According to the questions and selling price of an article = ` 96
= ` ( 9000 − x) 33. (b) We know that, together = 10 x + 15x = 25x
= ` 274.50 SP = `  11x
+ 6
∴ Selling price of Ist scooter [given]
∴ Selling price of 1 video cassette  10  Net percentage discount Selling price of whole lot
25x 125x x − 96 1
=x+ = Discount 1 = 15 × ( x + x) = 15 × 2x = 30 x × 100 = x
=`
27450
.
= ` 91.50 100 100 ∴ 
11x
+ 6 = 118 % of
3 24x = × 100% = × 100% = 25% x 4
3  10  4 25 Cost price 4 30x − 25x
r Also, selling price of 2nd scooter ∴ Profit percentage = × 100 ⇒ x2 − 400 x + 38400 = 0
∴ 100 − × 100 = ` 91.50 11x + 60 475 24x 57x 34. (b) We know that, 25x
⇒ = × = ⇒ x2 − 160 x − 240 x + 38400 = 0
= ( 9000 − x)  1 −
100 20  5x
 10 400 25 50 = × 100 = 20%
Cost price =   × SP
100
⇒ 100 − 91.50 = r ⇒ 8.50 = r  100   ⇒ ( x − 160) ( x − 240) = 0
⇒ 570x = 550x + 3000  100 − Loss%  25x
= ( 9000 − x)  
80
∴ Rate of discount = 85
. % 3000 Hence, the profit is 20%. ∴ x = 160 or 240
 100  ⇒ x= = 150 [Rule 2]
1 Hence, the cost price of the article is
23. (c) CP of 1 lemon = 20 39. (c) Let the original price be ` x.
= 
Total selling price of both scooters 100 
2 ∴ CP = ` 150  × 136 either
3 =
125x
+ ( 9000 − x)
80  100 − 15  Since, at discount of 20% article costs
SP of 1 lemon = 11 ` 160 or ` 240.
100 100 ∴ SP = × 150 = ` 165 136 × 100 ` 596.
5 10 = = ` 160 80 596 × 100
125x 80 Then, 596 = ×x ⇒x= 44. (b) Let CP of 1 L of milk be ` x.x
∴ Gain % ∴ + ( 9000 − x) = 9000 Hence, I and II both are correct. 85
100 100 100 80 ∴
3 1
− 31. (d) Let CP of 8 quintal be ` x. Q Given, profit percentage = 15% SP of 1 L of milk
6 −5 [given] = ` 745 = xx × 120% = ` 1. 2x
= 5 2 × 100 = × 2 × 100 = 20% 45x x ∴ Selling price (x)
1 10 ⇒ + 7200 = 9000 ∴ CP of 1 quintal = ` Hence, the original price of an article is Now, as in ` 12. x,x the quantity of milk
100 8 160 × ( 100 + 15) 160 × 115 sold = 1 L
2 = = = ` 184 ` 745.
∴ x = 4000 ∴ SP of 3 quintal rice at 10% profit 100 100
40. (c) Given, cost price of 200 oranges ∴ In ` x,x quantity of milk sold
24. (c) Let the cost price be ` x. ∴ Cost price of 1st scooter = ` 4000 So, option (b) is correct because
3x 3x 1 3x 3x 33x = ` 1000 1 5
x × 110 11x Hence, cost price of 2nd scooter = + × = + = 180 < x < 190. = ×x= L
Marked price = =`f 8 8 10 8 80 80 1000 12
. x 6
100 10 = ( 9000 − x) = ` 5000 SP of 3 quintal rice without profit or 35. (a) Given, list price of television = ` 5000 ∴ Cost price of 1 orange = = `5
200 According to the question,
11x 90 99x 3x
∴ SP = × = 28. (d) Let the number of items be x. loss = ` and discount = 10%  100 + 25  CP of milk and SP of mixture are same,
10 100 100 8 Selling price of 1 orange = 5 
Then, selling price of items = 60 x ∴ After discount television cost  100  therefore in mixture, quantity of milk
∴ Required gain/loss per cent SP of 2 quintal rice at 5% loss 5
Cost of material of items = 40 x = 5000 − 5000 × 10% 125 must be L.
99x
−x 2x 2x 5 x x =5× = ` 6.25 6
Overhead expenses = ` 3000 = − × = − 5000 × 10 100
= 100 × 100 = − 1% [Rule 8] 8 8 100 4 4 × 20 = 5000 − = ` 4500 Hence, the required percentage
x ∴ 60 x − ( 40 x + 3000) = 1000 100 Now, in ` 6.25 number of oranges can be
19x 19x 5 250
⇒ 20 x = 4000 = = Additional discount = 2% sold = 1 = × 100% = %
[negative sign i.e. loss] 4 × 20 80
4000 6 3
25. (b) Let marked price of two sarees be ` x ∴ x= = 200 ∴ Total SP =
33x 3x 19x
+ + ∴ Actually price of television In ` 100, number of oranges can be sold
20 80 8 80 4500 × 2 1 45. (c) Refer to example 12.
and ` y, respectively. = 4500 − = ` 4410 = × 100 = 16
6x 94x 29. (b) I. Let CP = x 33x + 30x + 19x 82x 100 6.25 46. (a) Let the amount of cloth = 100 cm
∴ x− = 564 ⇒ = 564 = =
100 100 213 80 80 36. (c) Q CP of 80 apples = ` 240 Hence, 16 oranges can be sold in ` 100 Then, amount of cloth purchased
Then, SP = x.
y SP − CP for profit 25%. = 110 cm
⇒ x = ` 600 and y − = 396 200 ∴ Profit percentage = × 100% ∴ CP of 1 apple =
240
=`3
100 213x CP 41. (c) Let the cost price of article be ` x. and amount of cloth sold = 90 cm
If SP = + 1250, then gain = 19% 82x 80

99 y
= 396 ⇒ y = ` 400 200 −x Discount = 20% and selling price = 24 110 − 90
(82 − 80)x ∴ CP of 20 apples = `3 × 20 = ` 60 ∴ Gain % × 100%
100 213x = 80 × 100% = × 100%
+ 1250 − x x 80x To earn a profit of 20%, SP = 120% of [given] 100
∴ Total MP amount = 600 + 400 ⇒ 19 = 200 × 100 2 20
= × 100 = 2.5% 240 = ` 288 ∴ x = 24 + 20% of x = × 100% = 20%
= ` 1000 x 80 100
MATHEMATICS Simple Interest 101 102 CDS Pathfinder

12
RATIO AND
Types of Ratios
1. Compound Ratio When two or more ratios are
multiplied together, they are said to be compound
ratio.
a c e
If , ,
b d f
g
and are all ratios, then their compound
h
ratio is aceg : bdfh.
2 4
e.g. Compound ratio of , and =
3 7
1 2× 4×1 8

2. Duplicate Ratio When a ratio is compounded with


= .
3 3 × 7 × 3 63

itself, the resulting ratio is called the duplicate ratio.


So, a 2 : b 2 is duplicate ratio of a : b.
6. Reciprocal Ratio

a. 1 : 2 b. 2 : 3
If a : b is a ratio, then
reciprocal ratio, i.e. b : a .
e.g. Reciprocal ratio of 3 : 7 is

Sol. c. The duplicate ratio of 5 : 6 is


1 1

c. 1 : 1

The reciprocal ratio of 25 : 42 is


5 5 25
× =
6 6 36
1 1
:
25 42
: i.e. 7 : 3.
3 7
EXAMPLE 4. Compound ratio of the duplicate ratio
of 5 : 6, the reciprocal ratio of 25:42 and the
subduplicate ratio of 36:49 is
d. 4 : 9

i.e.
42
25
1 1
: is its
a b

PROPORTION EXAMPLE 2. The duplicate ratio of the ratio 2 2 : 3 5 36 36 6


and subduplicate ratio of is =
is 49 49 7
a. 4 : 9 b. 8 : 45 c. 2 : 3 d. 6 : 45 25 42 6 1
∴ Compound ratio is × × = = 1: 1
Sol. b. The duplicate ratio of 2 2 : 3 5 is 36 25 7 1
2 2 2 2 4 4 8
× = = = 8 : 45 IMPORTANT POINTS
3 5 3 5 9 25 45
3. Triplicate Ratio If a ratio is compounded three l Usually, the ratio is expressed in its lowest terms.

Regularly (1-2) questions have been asked from this chapter. Generally both the topics i.e. ratio and times with itself, then resulting ratio is called l Ratio exists only between quantities of the same kind.
triplicate ratio. l Ratio is a fraction, so it has no units.
proportion have same importance but more emphasis is given on proportion. The concepts of this
So, a 3 : b 3 is the triplicate ratio of a : b. l Ratio is taken only between positive quantities.
chapter are extremely used in other chapters like-age, average. etc.
EXAMPLE 3. If 4x + 3 : 9x + 10 is the triplicate ratio
of 3 : 4. Then, the value of x is PROPORTION
a. 2 b. 6 c. 8 d. 12 The equality of two ratios is called proportion.
RATIO Sol. b. Since, 4x + 3 : 9x + 10 is the triplicate ratio of 3 : 4. ‘: :’ is the sign of proportion.
4x + 3 3 3 3 4x + 3 27 So, if a : b = c : d , we write it as
A ratio is the comparison of two or more quantities of the same type (or kind) by division. i.e. if a and Therefore, = × × ⇒ =
b are two quantities of same kind (or same unit), then the fraction a / b is called the ratio a to b and 9x + 10 4 4 4 9x + 10 64 a : b : : c : d and say that a, b, c and d are in proportion.
we write it as a : b. Here, a is called antecedent and b is called the consequent. ⇒ 64( 4x + 3) = 27(9x + 10) The quantities a, b, c and d are called the first, second,
e.g. If a fruit box contains 8 oranges and 7 lemons then the ratio of oranges to lemons is 8 to 7 or 8:7. ⇒ 256x + 192 = 243x + 270
third and fourth terms of proportion respectively. First
and fourth terms are called extreme terms and second
Note If both terms a and b of a ratio are multiplied or divided by the same quantity, then ratio remains unchanged. ⇒ 256x − 243x = 270 − 192 ⇒ 13x = 78 and third terms are called means or middle terms.
78
a na a b a a/n ∴ x= =6 If quantities a, b, c and d are in proportion, then
i.e. a : b is same as na : nb ⇒ = and a : b is same as : ⇒ = . 13
b nb n n b b/n a c
Hence, the value of x is 6. = ⇒ ad = bc
b d
EXAMPLE 1. If the ratio of 90 cm to 1.5 m is same as 3 : x, then x is 4. Subduplicate Ratio If two numbers are in ratio, i.e. product of extreme terms = product of middle terms.
a. 4 b. 5 c. 2 d. 1 then the ratio of their square roots is called sub This is also called cross-product rule.
Sol. b. 1.5 m = 150 cm (units must be same) duplicate ratio.
Ratio of 90 cm to 1.5 m =
90
=
90 / 30 3
= . If a : b is ratio, then the subduplicate ratio is a : b. Types of Proportion
150 150 / 30 5 1. Continued Proportion The non-zero quantities of
e.g. Subduplicate ratio of 9 : 16 is 9 : 16 = 3 : 4.
As this ratio is same as 3 : x same kind, a, b, c, d, e, f, … are said to be continued
3 3
∴ = 5. Subtriplicate Ratio If two numbers are in ratio, a b c e
x 5 proportion, if = = = = …
then the ratio of their cube roots is called b c d f
⇒ 3x = 5 × 3 subtriplicate ratio.
⇒ x=5 EXAMPLE 5. If a, b, c, d and e are in continued
If a : b is ratio, then the subtriplicate ratio is 3 a : 3 b.
proportion, then a/e is equal to
e.g. Subtriplicate ratio of 64 : 27 is 64 : 27 = 4 : 3
3 3
a. a3 /b3 b. a 4 /b 4 c. b3 /a3 d. b 4 /a 4
MATHEMATICS Ratio and Proportion 103 104 CDS Pathfinder

Sol. b. Since, , , , and are in continued proportion. EXAMPLE 8. The fourth proportional to
2
p 2 − pq + q 2 , p 3 + q 3 , p − q is
MIXTURE OR ALLIGATION VARIATIONS
Q = = = ⇒ = = = ⇒ = If two or more quantities are mixed together in a certain When a change in a certain quantity leads to a certain
2 4 3 2 6 4 a. p + q b. p − q c. p + q
2 2
d. p − q
2 2
ratio, then the product is called mixture. change in another quantity then it is called variations.
1
= = 2 ⋅ = 2 , = = 4 ⋅ 2 = 3 Sol. d. Let the fourth proportional be x.
4
Then, 2
− + 2
: 3
+ 3
= − :x
Mean Price Direct Variation
∴ = = Cost price of a unit quantity of the mixture is called the If x varies directly with y, then as x increses (decrease) y
( 4
/ 3
) 4 2
− + 2
− ( − )( 3
+ 3
)
⇒ = ⇒x= mean price. also increases (decreases)
2. Mean Proportional If a, b and c are in continued
3
+ 3
x 2
− + 2

proportion, then b is called the mean proportional of ( − )( + i.e. x ∝ y


a and c. ∴ x=
3 3
)
=( − )( + ) =( 2
− 2
) Rule of Mixture or Alligation
2
− + 2
It is the rule to find the ratio in which two or more ⇒ x = ky. where, k is the constant of variation.
a b
i.e. = ⇒ b 2 = ac ⇒ b = ac ingredients are mixed or to be mixed.
b c
Properties of Proportion If two quantities are mixed in the ratio n1 : n2 , then
Inverse Variation
EXAMPLE 6. What is the mean proportional between If four non zero quantities a, b, c and d are in proportion, n1 P2 − Pm If x varies inversely with y, then an increase is the value
(15 + 200 )and (27 − 648)? then properties of proportion are as follows. = of x causes a decreases in the value of y.
e 2012 I n2 Pm − P1
a. 4 b. 14 7 c. 3 5 d. 5 3 1. Invertendo If a : b : : c : d, then b : a : : d : c 1
where P1 , P2 and Pm are cheaper price, dearer price and i.e. x ∝
a c b d mean price respectively. y
Sol. c. Let mean proportional between (15 + 200) and i.e. = ⇒ =
( 27 − 648) is x b d a c k′
It can also be expressed as ⇒ x=
Then, x = (15 + 200)( 27 − 648) 2. Alternendo If a : b :: c : d, then a : c :: b : d Cheaper price Dearer price y
a c a b where, k ′ is the constant of variation.
= (15 + 10 2)( 27 − 18 2) i.e. = ⇒ = P1 P2
b d c d
Mean price
= 5 ( 3 + 2 2).9( 3 − 2 2) = 45[( 3) 2 − ( 2 2) 2 ] 3. Componendo (Adding the Denominator) EXAMPLE 11. If a quantity y varies as the sum of
Pm three quantities of which the first varies as x, the
= 45 (9 − 8) = 45 = 3 5 If a : b :: c : d, then ( a + b): b : : ( c + d ): d second varies as − x + x 2 , the third varies as x 3 − x 2 ,
3. Third Proportional If a : b : : b : c, then c is called a c a + b c +d P2 – Pm Pm– P1
i.e. = ⇒ = then what is y equal to?
the 3rd proportional to a and b. Now, c will be b d b d Cheaper quantity n1 P2 − Pm
calculated as below: ∴ = = a. kx3 , where k is a constant
4. Dividendo (Subtracting the Denominator) Dearer quantity n2 Pm − P1 b. kx + lx2 + mx3 , where k , l and m are constants
b2 If a : b :: c : d, then a : ( a − b): : c : ( c − d )
a : b :: b : c ⇒ a : b = b : c ⇒ c = c. kx2 , where k is a constant
a a c a c Note P1 < Pm < P2
i.e. = ⇒ = d. kx, where k is a constant
EXAMPLE 7. The two numbers such that their mean b d a − b c −d EXAMPLE 10. In what proportion, must wheat at Sol. b. Since, first term ∝ x
proportional is 24 and the third proportional is 1536, are 5. Componendo and Dividendo ` 6.20 per kg be mixed with wheat at ` 7.20 per kg, so
⇒ First term= 1x ,
a. 3 and 98 b. 6 and 96 c. 4 and 92 d. None of these If a : b :: c : d, then ( a + b): ( a − b): : ( c + d ): ( c − d ) that the mixture be worth ` 6.50 per kg?
a. 6 : 4 b. 7 : 4 c. 7 : 3 d. 5 : 2 ⇒ Second term ∝ ( − x + x2)
Sol. b. Let and be the required numbers, then mean a c a + b c +d
proportional between and is 24. i.e. = ⇒ = ⇒ Second term = 2( − x + x2 )
b d a − b c −d Sol. c. Given, cost price of cheaper quantity = ` 6.20 per kg
∴ : 24 = 24 : ⇒ = 242, = 576 …(i) Cost price of dearer quantity ` 7.20 per kg and third term ∝ ( x3 − x2) ⇒ Third term = 3 (x − x )
3 2

EXAMPLE 9. If (a − b) : (a + b) = 1 : 5, then what is ∝ [ 1x + ( − x + x2 ) + 3 ( x − x )] (given)


3 2
and third proportional to and is 1536. Mean price = ` 6.50 per kg Also, 2
⇒ : = : 1536 (a 2 − b 2 ) : (a 2 + b 2 ) equal to? ⇒ = − +( − 3) x + 3x ]
2 3
According to the rule of alligation, 4[( 1 2) x 2
2
⇒ 2
= 1536 ⇒ = …(ii)
a.
13
b.
4
c.
5
d. None of these CP of cheaper CP of dearer = 4 ( 1 − 2) x+( 2 − 3) 4x + 3 4x
2 3
1536 5 13 13 (620 paise) (720 paise)
On putting the value of in Eq. (i), we get = x + x2 + x3
1 − + 5 + + − 5+ 1 Mean price
2
Sol. c. Given, ⇒ = ⇒ = = (650 paise) where, = 4 ( 1 − 2 ),
⇒ ⋅ = 576 ⇒ 3
= 576 × 1536 5 + − 1 + − + 5−1
1536 (650 – 620) =( 2 − 3) 4,
(using componendo and dividendo rule) (720 – 650)
⇒ = (96 × 6) × (96 × 16) = 963 ⇒
3
= 96 2 = 70 paise
= 30 paise and =
2 6 3 3 9 3 4
576 576 ⇒ = = ⇒ = ⇒ 2 =
∴ = = =6 [from Eq. (i)] 2 4 2 2 4
96 ∴ Required ratio = 70 : 30 = 7 : 3
Again, using componendo and dividendo rule
Hence, the required numbers are 6 and 96. 2
+ 2
9 + 4 13
4. Fourth Proportional If four numbers or quantities ⇒ = =
2
− 2
9−4 5
a, b, c and d are in proportion, then d is known as
fourth proportional.
2
− 2 5
⇒ 2 = ∴( 2
− 2
) :( 2
+ 2
) = 5 : 13
2
+ 13
i.e. a:b::c:d
MATHEMATICS Ratio and Proportion 105 106 CDS Pathfinder

PRACTICE EXERCISE 26. If q is the mean proportional between p and r,


then
p2 − q 2 + r 2
is equal to
36. A mixture contains milk and water in the ratio
5 : 1. On adding 5 L of water, the ratio of milk
and water becomes 5 : 2. What is the quantity of
p− 2 − q − 2 + r − 2 milk in the original mixture?
(a) p2q 3 (b) q 3 (c) q 4 (d) p2 r 2q 4 (a) 5 L (b) 25 L
1. The compounded ratio of 2a : 6b, 7a : 49b and 15. Divide 1870 into three parts in such a way that 27. The monthly incomes of A and B are in the ratio (c) 27.5 L (d) 32.5 L
3a : 12b is half of the first part, one-third of the second part 4 : 3. Each of them saves ` 600. If the ratio of the 37. If x varies as the mth power of y , y varies as the
(a) a : 84 b (b) a3 : 84 b 3 (c) a : 24 b (d) 2 a : 3 b and one-sixth of the third part are equal. Then, expenditure is 3 : 2, then what is the monthly
the third part is nth power of z and x varies as the p th power of
income of A ?
z, then which one of the following is correct?
2. The subduplicate ratio of 16x 4 : 625 y 6 is (a) 340 (b) 510 (c) 1020 (d) 1320 (a) ` 2400 (b) ` 1800 (c) ` 2000 (d) ` 3600
(a) p = m + n (b) p = m − n
(a) 4x2 : 25 y3 (b) 4x2 : 25 y2 (c) 4x : 25 y (d) 8x2 : 125 y3 16. Out of the ratios 7 : 20, 13 : 25, 17 : 30 and 28. Three numbers are in the ratio 3 : 2 : 5 and the (c) p = mn (d) None of these
11 : 15, the smallest one is sum of their squares is 1862. What are the three
3. If x + 5 : 3x + 4 is the duplicate ratio of 5 : 8 , then (a) 11 : 15 (b) 7 : 20 (c) 11 : 16 (d) 17 : 30 numbers?
38. The wages of labourers in a factory has increased
the value of x is in the ratio 22 : 25 and their number decreased in
(a) 18, 12, 30 (b) 24, 16, 40
17. What is the ratio of the numbers of 0.5 of a the ratio 3 : 2. What was the original wage bill of
(a) 16 (b) 18 (c) 20 (d) 22 number is equal to 0.07 of another ? (c) 15, 10, 25 (d) 21, 14, 35
the factory, if the present bill is ` 5000?
4. If x : 6 : : 5 : 3 , then the value of x is (a) 50 : 7 (b) 5 : 7 (c) 1 : 14 (d) 7 : 50 29. The speeds of three cars are in the ratio 4 : 3 : 2. (a) ` 4000 (b) ` 6000
18. What is the ratio whose terms differ by 40 and What is the ratio between the times taken by the (c) ` 8000 (d) None of these
(a) 8 (b) 10 (c) 12 (d) 13
2 cars to cover the same distance?
5. The third proportional to 9 and 12 is the measure of which is ? (a) 2 : 3 : 4 (b) 3 : 4 : 6
39. The ratio of A to B is x : 8 and the ratio of B to C
7
(c) 1 : 2 : 3 (d) 4 : 3 : 2 is 12 : z. If the ratio of A to C is 2 : 1, then what
(a) 12 (b) 14 (c) 16 (d) 18 (a) 14 : 56 (b) 15 : 56 (c) 16 : 56 (d) 16 : 72
is the ratio of x : z ?
3 5 3 2 30. The ratio between the ages of A and B is 2 : 5.
19. If P : Q = : and Q : R = : , then what is (a) 2 : 3 (b) 3 : 2 (c) 4 : 3 (d) 3 : 4
6. The mean proportional between ( 2 + 3 ) and 5 7 4 5 After 8 yr, their ages will be in the ratio 1 : 2.
What is the difference between their present 40. A bag contains ` 112 in the form of ` 1, 50 paise
( 8 − 48 ) is P : Q : R equal to?
and 10 paise coins in the ratio 3 : 8 : 10. What is
3 5 2 9 15 2 3 3 2 3 5 3 ages?
(a) 2 (b) 3 (c) 4 (d) 5 (a) : : (b) : : (c) : : (d) : : (a) 20 yr (b) 22 yr (c) 24 yr (d) 25 yr the number of 50 paise coins?
5 7 5 20 28 7 5 4 5 5 7 4
7. The fourth proportional to 7, 11, 14 is 31. Let y is equal to the sum of two quantities of which
(a) 112 (b) 108 (c) 96 (d) 84
(a) 16 (b) 18 (c) 20 (d) 22 20. Two numbers are in the ratio 3 : 5. If 9 is
subtracted from each number, then they are in one varies directly as x and the other inversely as x. 41. In a class, the number of boys is more than
8. In a ratio which is equal to 7 : 8, if the the ratio of 12 : 23. What is the second number? If y = 6 when x = 4 and y = 10/ 3, when x = 3, then the number of girls by 12% of the total students.
antecedent is 35, then the consequent is what is the relation between x and y? What is the ratio of number of boys to that of
(a) 44 (b) 55 (c) 66 (d) 77
(a) 30 (b) 32 (c) 36 (d) 40 (a) y = x + (4/ x) (b) y = − 2 x + (4/ x) girls?
21. The mean proportional between two numbers is (c) y = 2 x + (8/ x) (d) y = 2 x − (8/ x) (a) 11 : 14 (b) 14 : 11
9. If x : y = 1 : 3, y : z = 5 : k, z : t = 2 : 5 and t : x = 3 : 4, 28 and their third proportional to them is 224. (c) 28 : 25 (d) 25 : 28
then what is the value of k? The two numbers are 32. If ` 8400 is divided among A, B and C in the
(a) 1/2 (b) 1/3 (c) 2 (d) 3 (a) 7 and 112 (b) 14 and 56 (c) 28 and 28 (d) 21 and 36 1 1 1 42. A sum of ` 53 is divided among A, B, C in such a
ratio : : , what is the share of A?
10. If the ratio of x to y is 25 times the ratio of y to 22. ` 770 have been divided among A, B, C in such a 5 6 10 way that A gets ` 7 more than what B gets and B
2 (a) ` 1800 (b) ` 3000 (c) ` 3600 (d) ` 4000 gets ` 8 more than what C gets. The ratio of
x, then what is the ratio of x to y? way that A receives th of what B and C together their shares is
(a) 1 : 5 (b) 5 : 1 (c) 25 : 1 (d) 1 : 25 9 33. A certain amount of money has to be divided (a) 25 : 18 : 10 (b) 6 : 7 : 8
receive. What is A’s share? between two persons P and Q in the ratio 3 : 5. (c) 12 : 14 : 9 (d) 15 : 8 : 30
11. If p% of ` x is equal to t times q% of ` y, then But it was divided in the ratio of 2 : 3 and there
(a) ` 154 (b) ` 140 (c) ` 250 (d) ` 254
what is the ratio of x to y ? by Q loses ` 10. What was the amount? 43. A person P started a business with a capital of
(a) pt : q (b) p : qt (c) qt : p (d) q : pt 23. x varies directly as y and inversely as square of ` 2525 and another person Q joined P after some
z. When y = 4 and z = 14, then x = 10. If y = 16 (a) ` 250 (b) ` 300 (c) ` 350 (d) ` 400
12. If a : b = 2 : 3 and x : y = 3 : 4 , then the value of months with a capital of ` 1200. Out of the total
and z = 7, what is x? 34. 20 L of a mixture contains milk and water in the annual profit of ` 1644, P ′s share was ` 1212.
4ay − 3bx
is (a) 180 (b) 160 (c) 154 (d) 140 ratio 4 : 3. If 6 L of this mixture are replaced by When did Q join as partner?
5ax − 2by 6 L of milk, the ratio of milk to water in the new
m + 3n + m − 3n (a) After 2 months (b) After 3 months
(a)
5
(b)
5
(c)
4
(d)
5 24. If x = , then mixture will become (c) After 4 months (d) After 5 months
3 6 5 4 m + 3n − m − 3n
(a) 7 : 3 (b) 8 : 3 (c) 9 : 7 (d) 4 : 6
(a) 3 nx2 − 2 mx + 3 n = 0 (b) 2 nx2 − 2 mx + 3 n = 0 44. Seats for Mathematics, Physics and Biology in a
x3 + 3x 341
13. If = , then the value of x is 35. Two vessels are full with milk and water in the school are in the ratio 5 : 7 : 8. There is a proposal
3x 2 + 1 91 (c) 3 nx2 − 2 mx − 3 n = 0 (d) 3 nx2 + 3mx + 3 n = 0
ratios 1 : 3 and 3 : 5, respectively. If both are to increase these seats by 40%, 50% and 75%,
(a) 16 (b) 14 (c) 13 (d) 11 1 1 2 mixed in the ratio 3 : 2, what is the ratio of milk respectively What will be the ratio of increased
25. If p + r = 2q and + = , then
q s r and water in the new mixture? seats?
14. What is the number which has to be added to each
term of the ratio 49 : 68, so that it becomes 3 : 4? (a) p : s = r : q (b) p2 : r 2 = q : s (a) 4 : 15 (b) 3 : 7 (a) 2 : 3 : 4 (b) 6 : 7: 8
(a) 3 (b) 5 (c) 8 (d) 9 (c) p : q = r : s (d) p2 = 2 r + s (c) 6 : 7 (d) None of these (c) 6 : 8: 9 (d) 5 : 7 : 12
MATHEMATICS Ratio and Proportion 107 108 CDS Pathfinder

45. The ratio between the number of passengers PREVIOUS YEARS’ QUESTIONS 61. If a : b = 3 : 5 and b : c = 7 : 8 , then 2a : 3b : 7c is 64. The annual incomes of two persons are in
travelling by I and II class between the two equal to e 2015 II the ratio 9 : 7 and their expenses are in the ratio
railway stations is 1:50, whereas the ratio of I 51. If x : y = 7 : 5, then what is the value of (a) 42 : 105 : 320 (b) 15 : 21 : 40 4 : 3. If each of them saves ` 2000 per year, what
and II class fares between the same station is ( 5x − 2 y ) : ( 3x + 2 y )? e 2012 I (c) 6 : 15 : 40 (d) 30 : 21 : 350 is the difference in their annual incomes?
3:1, If on a particular day, ` 1325 were collected (a) 5/4 (b) 6/5 (c) 25/31 (d) 31/42 e 2016 I
from the passengers travelling between these 62. The speeds of three buses are in the ratio 2 : 3 : (a) ` 4000 (b) ` 4500 (c) ` 5000 (d) ` 5500
stations, then what was the amount collected 52. Two numbers are in the ratio 2 : 3. If 9 is added 4. The time taken by these buses to travel the
a b c
from the II class passengers? to each number, they will be in the ratio 3 : 4. same distance will be in the ratio. e 2015 II 65. If = = , then which of the following is/are
What is the product of the two numbers? e 2012 I (a) 2 : 3 : 4 (b) 4 : 3 : 2 b c d
(a) ` 750 (b) ` 1000 (c) 4 : 3 : 6 (d) 6 : 4 : 3 correct?
(a) 360 (b) 480 (c) 486 (d) 512
(c) ` 850 (d) ` 1250 b3 + c3 + d3 d a 2 + b2 + c2 a
63. In a mixture of milk and water of volume 30 L, I. = II. =
46. If x varies inversely as the square of y in such a 53. A milkman bought 15 kg of milk and mixed 3 kg a3 + b3 + c3 a b2 + c2 + d 2 d
of water in it. If the price per kg of the mixture the ratio of milk and water is 7 : 3. The quantity
way that, if x = 1, then y = 6. of water to be added to the mixture to make the Select the correct answer using the code given
becomes ` 22, what is cost price of the milk below. e 2016 I
I. If y = 3, then x = 4 II. If y = 6, then x = 1 per kg? e 2012 II
ratio of milk and water 1 : 2 is e 2015 II
(a) 30 (b) 32 (a) Only I (b) Only II
Which of the following options is correct? (a) ` 28.00 (b) ` 26.40 (c) ` 24.00 (d) ` 22.60 (c) Both I and II (d) Neither I nor II
(c) 33 (d) 35
(a) Only II (b) Both I and II
54. Sex ratio is defined as the number of females per
(c) Only I (d) Neither I nor II
1000 males. In a place, the total inhabitants are
47. If a : b = c : d = e : f = 1 : 2, then 1935000, out of which 935000 are females. What ANSWERS
3a + 5c + 7e 1 a 2 + c2 + e2 1 is the sex ratio for the place? e 2012 II
I. = II. 2 = (a) 935 (b) 1000 1 b 2 a 3 c 4 b 5 c 6 a 7 d 8 d 9 a 10 b
3b + 5d + 7 f 3 b + d2 + f 2 2
(c) 1935 (d) 9350 11 c 12 b 13 d 14 c 15 c 16 b 17 d 18 c 19 b 20 b
Which of the following option is correct?
55. In a certain school, the ratio of boys to girls is 21 b 22 b 23 b 24 a 25 c 26 c 27 a 28 d 29 b 30 c
(a) Only I (b) Only II
(c) Both I and II (d) Neither I nor II 7 : 5. If there are 2400 students in the school, 31 d 32 c 33 d 34 a 35 b 36 b 37 c 38 d 39 c 40 a
then how many girls are there? e 2013 I 41 b 42 a 43 b 44 a 45 d 46 b 47 d 48 c 49 a 50 c
48. A cat takes 5 leaps for every 4 leaps of a dog but 51 c 52 c 53 b 54 a 55 d 56 a 57 d 58 a 59 a 60 b
(a) 500 (b) 700 (c) 800 (d) 1000
3 leaps of the dog are equal to 4 leaps of the cat.
Now, the 56. If A : B = 2 : 3, B : C = 5 : 7 and C : D = 3 : 10, then 61 c 62 d 63 c 64 a 65 a

I. Ratio of the speeds of the cat to that of the dog what is A : D equal to? e 2014 I
is 15 : 16.
II. Ratio of the distance of the cat to that of the dog
(a) 1 : 7 (b) 2 : 7 (c) 1 : 5 (d) 5 : 1
57. ( x + y ) : ( x − y ) = 3 : 5 and xy = positive imply that
HINTS AND SOLUTIONS
is 15:16, covered in 30 min.
(a) x and y are both positive e 2014 II
Which of the following option is correct? (b) x and y are both negative 1. (b) The required compounded ratio is 5. (c) Let the third proportional to 9 and 8. (d) Let the consequent be x.
(c) one of them is positive and one of them is negative 2a 7a 3a a3 12 be x. ∴ 35: x = 7:8
(a) Only I (b) Only II × × = = a 3 : 84b 3
(d) no real solution for x and y exists 6 b 49 b 12 b 84 b 3 ∴ 9 : 12 : : 12 : x 35 7
(c) Both I and II (d) Neither I nor II Then, = [by given condition]
9 12 x 8
58. The ratio of ages of A and B is 2 : 5 and the ratio ⇒ = ⇒ 9x = 144
49. Determine the ratio of the number of people 2. (a) The subduplicate ratio of 12 x 35 × 8
⇒ 7x = 35 × 8 ∴ x = = 40
having characteristic X to the number of people of ages of B and C is 3 : 4. What is the ratio of 16x4 : 625 y 6 is 144 7
∴ x= = 16
having characteristic Y in a population of 100 ages of A, B and C? e 2014 II 4 2 9 9. (a) Given, x : y = 1 : 3, y : z = 5 : k,
16x 4x
subjects from the following table: (a) 6 : 15 : 20 (b) 8 : 5 : 3 = = 4x2 : 25 y 3
25 y 3 6. (a) Mean proportional between z : t = 2 : 5 and t : x = 3 : 4
Having X and Y 10 (c) 6 : 5 : 4 (d) 2 : 15 : 4 625 y 6
( 2 + 3 ) and (8 − 48 ) x y z t
Having X but not Y 30 52 × × × =1
3. (c) Duplicate ratio of 5 : 8 = 2 = (2 + 3 ) (8 − 48 ) y z t x
Having Y but not X 20 59. The height of a tree varies as the square root of 8
52 x+5 x+5 1 5 2 3 1
Having neither X nor Y 40 ⇒ = ⇒
25
= = (2 + 3 ) 4( 2 − 3 ) ⇒ × × × = 1⇒ k =
its age (between 5 to 17 yr). When the age of the 3 k 5 4 2
(a) 4:3 (b) 3:2 (c) 1:2 (d) 2:3 8 2 3x + 4 64 3x + 4
tree is 9 yr, its height is 4 ft. What will be the
10. (b) Given, = 25  
x y
height of the tree at the age of 16 yr? e 2014 II ⇒ 25(3x + 4) = 64 ( x + 5) = 2 (2 + 3) (2 − 3)
50. Fresh grapes contain 90 percent water by weight y  x
while dried grapes contain 20 percent water by (a) 5 ft 4 inch (b) 5 ft 5 inch ⇒ 75x + 100 = 64x + 320
= 2 ( 4 − 3) = 2 × 1 = 2 x2 25 x 5
weight. What is the weight of dry grapes contain (c) 4 ft 4 inch (d) 4 ft 5 inch ⇒ 75x − 64x = 320 − 100 ⇒ = ⇒ = or , 5 : 1
[Q ( a − b )( a + b ) = a 2 − b 2 ] y2 1 y 1
20 percent water by weight. What is the weight 60. 16 L of a mixture contains milk and water in the ⇒ 11x = 220
of dry grapes available from 20 kg of fresh 220 7. (d) Let fourth proportional be x, then 11. (c) According to the question,
ratio 5 : 3. If 4 L of milk is added to this mixture, ∴ x= = 20
grapes? the ratio of milk to water in the new mixture 11 7 : 11 : : 14 : x p % of x = t ( q% of y )
(a) 2 kg (b) 2.4 kg 7 14 xp yq
would be e 2015 I 4. (b) Given, x : 6 : : 5 : 3 ⇒ = ⇒ 7x = 11 × 14 ⇒ = ×t
(c) 2.5 kg (d) None of these (a) 2 : 1 (b) 7 : 3 (c) 4 : 3 (d) 8 : 3 x 5 30 11 x 100 100
∴ = ⇒ 3x = 30 ⇒ x = = 10 ⇒ x = 2 × 11
6 3 3 ∴ x : y = qt : p
∴ x = 22
MATHEMATICS Ratio and Proportion 109 110 CDS Pathfinder

4ay − 3bx 17 17 10 170 ⇒ b 3 = 28 × 28 × 224 26. (c) As, given q 2 = pr 10 37. (c) Given, x ∝ y m ...(i)
12. (b) ⇒ = × = when y = and x = 3 ,
5ax − 2by [dividing numerator and 30 30 10 300 ⇒ b = 56 p2 − q2 + r2 p2 − q2 + r2 3 y ∝ zn ...(ii)
= 10 m
denominator by by] 11 11 20 220
= × = then = 3l + ⇒ 9 l + m = 10…(iii) x ∝z p
and Again from Eq. (i) say, p −2 − q −2 + r −2 1

1
+
1 3 3 ...(iii)
4ay 3bx 4  a  − 3  x  15 15 20 300 28 × 28 p2 pr r 2
−     105 7 a= = 14 On solving Eqs. (ii) and (iii), On putting the values of x and y from
by by  b  y As, is the smallest, so or, 7 : 20 is 56 Eqs. (ii) and (iii) in Eq. (i), we get
= = 300 20 p −q + r
2 2 2
( p r )( p − q + r )
2 2 2 2 2
l = 2 and m = −8
5ax 2by a  x = = z p ∝ (z n )m ⇒ z p ∝ z mn
− 
5    −2 the smallest ratio. 22. (b) As, A : ( B + C ) = 2 : 9 [given] r 2 − pr + p 2 ( r 2 − pr + p 2 ) 8
by by  b  y From Eq. (i), y = 2x −
17. (d) Let the numbers be A and B. Sum of ratios = 2 + 9 = 11 p2 r2 x ∴ p = mn
4  − 3  8 − 9
2 3 2 1 1 1
∴ 0.5 A = 0.07B [by given condition] So, A ’s part = 770 × = ` 140 ( p 2 r 2 )( p 2 − pr + r 2 ) 32. (c) Given, A : B : C = : : 38. (d) Let the initial and final salary be
   4 3 4 11 = 5 6 10 ` 22x and ` 25x, respectively.
= 3 = ⇒
A 0.07
= =
7 ( r 2 − pr + p 2 )
i.e. 7 : 50 = 6:5:3
5    − 2
2 3 5 1
−2 B 0.5 50 23. (b) Given, x ∝ y and x ∝ 2 = p 2 r 2 = ( pr )2 = ( q 2 )2 = q 4 6 and let initial and final number of
 3   4 2 z ∴ Share of A = × 8400 employees be 3y and 2y, respectively.
6+5+3
32 − 27 5 18. (c) Let the ratio be x and ( x + 40). y
Now, x ∝ 2 ⇒ x = 2
ky 27. (a) Let monthly incomes of A ’s and B’s ∴ Present bill = Final salary × Final
5 2 5 z z be 4x and 3x respectively 6
= 12 = 12 = × = Then, by given condition,
x
=
2 = × 8400 = ` 3600 number of employees
5−4 1 12 1 6 x + 40 7 Q x = 10 , y = 4 and z = 14 Each saving = ` 600 14 5000 = 25x × 2 y
2 2 k⋅4 1960 Q Income − saving = Expenditure 33. (d) Let the amount be ` x. 5000
x3 + 3x 341 ⇒ 7x = 2x + 80 ⇒ x = 16 ∴ 10 = ⇒k = = 490 ⇒ = xy ⇒ xy = 100
= 4x − 600 3 5x 5 50
13. (d) Given, 2 196 4 ∴ = In first condition,Q’s part = = x
3x + 1 91 So, the required ratio is 16 : 56. Now, z = 7 and y = 16, then 3x − 600 2 5+3 8 ∴ Original bill = Initial salary × Initial
By componendo and dividendo rule, 3 5 number of employees
19. (b) Given, P : Q = : …(i) 490 × 16 ⇒ 2 ( 4x − 600) = 3 (3x − 600) In second condition, Q’s part
x3 + 3x + 3x2 + 1 341 + 91 432 5 7 x= = 160 = 22x × 3 y
= = 3 2 7×7 8x − 1200 = 9x − 1800 3x 3
and Q:R= : …(ii) = = x = 66 xy = 66 × 100 = ` 6600
x3 + 3x − 3x2 − 1 341 − 91 250 m + 3n + m − 3n ⇒ 1800 − 1200 = 9x − 8x ⇒ x = 600 2+3 5
4 5 x
( x + 1)3 216 3 3 5 3 24. (a) = ∴ A ’s income = 4x = 4 × 600 = ` 2400
39. (c) Given,
⇒ = From Eq. (i), P : Q = × : × 1 m + 3n − m − 3n 5 3
By given condition, x − x = 10 A : B = x : 8 and B : C = 12 : z
( x − 1)3 125 5 4 7 4 8 5
28. (d) Let three numbers be 3x, 2x and 5x, A A B x 12 3x
[Q ( a + b )3 = a 3 + b 3 + 3ab( a + b )] 9 15 By componendo and dividendo rule, ⇒ A :C = = × = × =
= : …(iii) respectively. ⇒
x
= 10 ⇒ x = ` 400 C B C 8 z 2z
On cube roots both sides, we get 20 28 x+ 1 m + 3n + m − 3n According to the question, 40
3 5 2 5 = But A :C = 2: 1
x+ 1 6 From Eq. (ii), Q : R = × : × x−1 m + 3n + m − 3n 34. (a) Remaining mixture = 20 − 6 = 14 L
⇒ = ⇒ 5( x + 1) = 6( x − 1) 4 7 5 7 (3x)2 + ( 2x)2 + ( 5x )2 = 1862 ⇒
3x 2
= ⇒ x:z = 4:3
x−1 5 2z 1
15 2 4
∴ x = 11 = : …(iv) + m + 3n − m − 3n ⇒ 9x2 + 4x2 + 25x2 = 1862 Quantity of milk = 14 × = 8 L
28 7 1862 7 40. (a) Let the number of coins of ` 1, 50
14. (c) Let the number be x − m + 3n + m − 3n ⇒ x2 = = 49 ∴ x =7 ∴ Water contents = 14 − 8 = 6 L paise and 10 paise be 3x, 8x and 10x,
From Eqs. (iii) and (iv), 38
49 + x 3 respectively
∴ = 9 15 2 x+ 1 m + 3n Q Milk in new mixture
68 + x 4 P :Q: R = : : ⇒ = Hence, the numbers are 3 × 7, 2 × 7 and
x−1 m − 3n = 8 + 6 = 14 L and water = 6 L According to the question,
20 28 7 5 × 7 i.e. 21, 14 and 35.
⇒ 196 + 4x = 204 + 3x 3x 8x 10x
20. (b) Let two numbers be 3x and 5x, On squaring both sides, we get 1 ∴ Ratio of milk and water in new + + = 112
∴ x=8 29. (b) Speed ∝ 1 2 10
respectively. ( x + 1)2 m + 3n Time 14 7 ⇒ 3x + 4x + x = 112
15. (c) Let the three parts be A , B and C . = mixture = =
According to the question, ( x − 1)2 m − 3n 1 1 1
By given condition, ∴ Required ratio = : : = 3 : 4 : 6 6 3 112
⇒ x= = 14
1 1
A= B= C =x
1
[say] (3x − 9) : ( 5x − 9 ) :: 12 : 23 Again applying componendo and 4 3 2 Hence, the required ratio 7 : 3. 8
2 3 6 3x − 9 12 dividendo rule, 30. (c) Let the ages of A and B be 2x yr and ∴ Number of 50 paise coins
1 ⇒ = 35. (b) Let x be the part of milk in new
∴ A = x ⇒ A = 2x 5x − 9 23 ( x + 1)2 + ( x − 1)2 m + 3n + m − 3n 5x yr, respectively. mixture. = 14 × 8 = 112
2 =
⇒ 69x − 207 = 60x − 108 ( x + 1)2 − ( x − 1)2 m + 3n − m + 3n After 8 yr age of A = ( 2x + 8) yr By law of mixture, 41. (b) Let the number of boys and girls be x
1
B = x ⇒ B = 3x 69x − 60x = 207 − 108 ⇒ 9x = 99 2( x2 + 1) 2m and age of B = ( 5x + 8) yr and y , respectively.
⇒ = 3
3 2x + 8 1 −x
1 ∴ x = 11 2 ( 2x) 2(3n) According to the question, = 8 3 3 3 ∴ Total number of students = ( x + y )
C = x ⇒ C = 6x 5x + 8 2 ∴ = ⇒ − 2x = 3x −
1 2 4 4 According to the question,
6 ∴ Second number = 5x = 5 × 11 = 55 x2 + 1 m x−
As, A + B + C = 1870 ⇒ = ⇒ 3n( x2 + 1) = 2mx ⇒ 4x + 16 = 5x + 8 4 Number of boys = Number of girls
21. (b) Let the numbers are a and b and its 2x 3n ⇒ x=8 6 3 3 + Total Number of students × 12%
[by given condition] ⇒ 5x = = ∴ x=
mean proportional is x ∴ 3nx2 − 2mx + 3n = 0 ∴ Difference between their present ages 4 2 10 x = y + ( x + y ) × 12%
∴ 2x + 3x + 6x = 1870
∴ a : x :: x : b p+ r = 5x − 2x = 3x = 3 × 8 = 24 yr 3 3 ( x + y ) × 12
⇒ 11x = 1870 25. (c) Given, p + r = 2q ⇒ 2 = Required ratio = = or, 3 : 7 x− y =
a : 28 : : 28 : b (Q x = 28) q 31. (d) By given condition, 10 − 3 7 100
1870
∴ x= = 170 ⇒ 282 = ab 1 1 2 ⇒ 25x − 25 y = 3x + 3 y
11 …(i) and + = y = lx +
m
…(i) 36. (b) Let quantities of milk and water be
q s r x 5x and x L, respectively. ⇒ 22x = 28 y
∴ Third part i.e. C = 6 × 170 = 1020 Again, a : b : : b : 224
1 1 p+ r s + q p+ r 5x 5
b2 ⇒ + = ⇒ = where, l and m are proportionality According to the question, = ∴ x : y = 14 : 11
16. (b) LCM of 20, 25, 30, 15 is 300. ⇒ b 2 = a × 224 ⇒ a = …(ii) q s qr sq qr constants. x+5 2
224 42. (a) Let C gets ` x .
7 7 15 105 ⇒ r ( s + q ) = s( p + r ) y = 6 and x = 4,
So, = × = From Eq. (i) say,
when ⇒ 10x = 5x + 25 Now, by given conditions,
20 20 15 300 p r m
⇒ rq = sp ⇒ = then 6 = 4l + ∴ x=5
13 13 12 156 b2 4 Amount of B = ` ( x + 8)
⇒ = × = 282 = ×b q s Hence, the quantity of milk in the
25 25 12 300 224 ∴ p: q = r: s ⇒ 16 l + m = 24 …(ii) original mixture = 5 × 5 = 25 L Amount of A = ` ( x + 15)
MATHEMATICS Ratio and Proportion 111 112 CDS Pathfinder

∴ x + ( x + 8) + ( x + 15) = 53 II. On putting y = 6 in Eq. (ii), we get 53. (b) Let cost price of milk be ` x per kg. If age = 9 yr, then height = 4 ft 40 a ⇒ 9 + y = 42
From Eq. (i), we get 4 = k 9 Now, 2a : 3b : 7c = 2a : 5a :
⇒ x = 10 36 Then, according to the rule of mixture, ∴ y = 33 L
x = =1 4 3
∴ A : B : C = ( 10 + 15) : ( 10 + 8) : 10 36 22 : ( x − 22) = 15 : 3 ⇒ 4 = k ×3 ∴ k = = 2:5:
40
= 6 : 15 : 40
3 64. (a) Refer to question 27.
x =1 22 15 3
= 25 : 18 : 10 ⇒ = If age = 16 yr, then h = k 16 = × 4
4 a b
65. (a) = = = k
c
(say)
Both statements I and II are correct. x − 22 3 3 62. (d) Let the distance be x.
43. (b) Let Q join for x months. b c d
a
= =
c e
=
1 22 Q k = 4  Now, ratio of time taken to travel the
∴ Ratio of capital 47. (d) Given that, ⇒ =5 ⇒ a = bk , b = ck, c = dk
b d f 2 x − 22  3  distance by each bus is
= 2525 × 12 : 1200 × x ∴ a = dk 3 , b = dk 2 , c = dk
b d f ∴ x = ` 26.40 16 1 x x x 12 12 12
= 2525 : 100x = 101 : 4x ⇒ a= , c= , e= ∴ h= ft = 5 = 5 ft 4 inch : : ⇒ : : b 3 + c3 + d 3
2 2 2 3 3 2 3 4 2 3 4 I.
101 54. (a) Total number of inhabitants = 1935000 a 3 + b 3 + c3
∴ P’s profit = × 1644  3 b 5d 7 f  ⇒ 6:4:3
101 + 4x  + +  Total number of females = 935000 60. (b) Refer to question 34.
3a + 5c + 7e b 3 + c3 + d 3
I. = 2 2 2  a 3 63. (c) Since, milk = 7x and water = 3x =
101 × 1644 3b + 5d + 7 f  3b + 5d + 7 f  ∴ Total number of males 61. (c) We have, = ( bk )3 + ( ck )3 + ( dk )3
⇒ 1212 =   b 5 Then, 7x + 3x = 30 ⇒ 10x = 30
101 + 4x = 1935000 − 935000 = 1000000 1 ( b 3 + c3 + d 3 ) 1 d
1 b=
5a
and
b 7
= ∴ x =3 = = =
(3b + 5d + 7 f ) 1 935000 k 3 ( b 3 + c3 + d 3 ) k 3
1 1 Q
⇒ = ∴ sex ratio = × 1000 = 935 3 c 8 a
137 101 + 4x = 2 = 1000000 So, milk = 21 L and water = 9 L
3b + 5d + 7 f 2 8 8 5a 40 a Hence, statement I is correct.
⇒ 4x = 36 ⇒ x=9 55. (d) Given, ratio of boys to girls = 7 : 5 ⇒ c= b= × = Let y L of water to be added.
a 2 + b 2 + c2 b
b2 d 2 f2 7 7 3 21 21 1 II. Similarly = k2 =
Hence, Q joined for 9 months i.e. he + + and total number of students = 2400 Then, = b 2 + c2 + d 2
a +c +e
2 2 2
4 = 1 5a 40 a 9+ y 2
d
joined after 3 months. II. = 42 4
Sum of ratio = 7 + 5 = 12 Q a:b:c= a: :
b2 + d 2 + f 2
b +d + f2
2
4 3 21 Hence, statement II is incorrect.
44. (a) Originally, let the numbers of seats 5
∴ Number of girls = × 2400 = 1000
for Mathematics, Physics and Biology ∴ Neither I nor II correct. 12
be 5x, 7x and 8x, respectively. 48. (c) I. 4 leaps of cat = 3 leaps of dog 56. (a) Given, A : B = 2 : 3, B : C = 5 : 7
Number of increased seats are (140% of 3
⇒ 1 leap of cat = leap of dog and C : D = 3 : 10
5x) (150% of 7x) and (175% of 8x). 4 A A B C
Required ratio can be obtained as ∴ = × ×
Cat takes 5 leap for every 4 leaps of dog. D B C D
⇒  × 5x  :  × 7x 
140 150 ∴ Required Ratio 2 5 3 1
 100   100  = × × =
= (5 × cat’s leap) : (4 × dog’s leap) 3 7 10 7
:  × 8x 
175
=  5 × dog' s leap : (4 × dog’s leap) ∴ A : D = 1: 7
3
 100   4 
 21  57. (d) Given that, ( x + y ) : ( x − y ) = 3 : 5
⇒  7x : x : 14x  × 2 = 15 : 16 x+ y 3
 2  ∴ =
⇒ ( 14x : 21x : 28x ) ÷ 7x Thus,
Speed of cat
=
15 x− y 5
Speed of dog 16 On applying componendo and dividendo
∴ 2:3: 4
Distance (cat) s (cat) × t rule, we get
II. =
45. (d) Let the number of passengers
Distance (dog) s (dog) × t (x + y) + (x − y) 3 + 5
travelling by class I and class II be x and =
(x + y) − (x − y) 3 − 5
50x respectively. s (cat) × 30  s (cat) 15 
= = =  2x 8 x
Then, amount collected from class I and s (dog) × 30  s (dog) 16  ⇒ = ⇒ = −4
II will be ` 3 × x and ` 50x respectively. 2 y −2 y
Thus, both statements I and II are ∴ x = − 4y
Given, 3x + 50x = 1325 correct.
But it is given , xy = positive
53x = 1325 ⇒ x = 25 49. (a) Number of people having ∴ − 4 y × y = positive
∴ Amount collected from class II characteristic X
⇒ − 4 y 2 = positive, which is not possible.
= 50 × 25 = ` 1250 = 10 + 30 = 40
Hence, no real solution for x and y exists.
1 k Number of people having characteristic Y
46. (b) As x ∝ 2 ⇒ x = 2 …(i) 58. (a) Given, ratio of ages of A and B i.e.
y y = 10 + 20 = 30 A : B = 2:5
If x = 1 Required ratio = 40:30 = 4:3. Ratio of ages of B and C i.e.
and y=6 50. (c) Fresh grapes contain 10% pulp. B :C = 3 : 4
2:5
k ∴20 kg fresh grapes contain 2 kg pulp. ∴ Ratio of ages of A, B and C =
1= 2 ⇒ k = 36
6 3:4
Dry grapes contain 80% pulp.
On putting the value of k in Eq. (i), we
2 kg pulp would contain = 2 × 3 : 3 × 5 : 5 × 4 = 6 : 15 : 20
get
36 2 20 59. (a) Let height of tree be h ft and age be
x = 2 …(ii) = = 25
. kg dry grapes.
08
. 8 a yr.
y
36 51. (c) Refer to question 12 Then, according to the question,
I. On putting y = 3 in Eq. (ii), x =
9 h∝ a ⇒ h=k a ..(i)
52. (c) Refer to question 20
x =4 where, k is a constant.
∴ Their product = 18 × 27 = 486
13
MATHEMATICS > Logarithm 113 114 CDS Pathfinder

EXAMPLE 3. The value of log10 (15) is Sol. b. log 3 4 ⋅ log 4 5 ⋅ log 5 6 ⋅ log 6 7 ⋅ log 7 8 ⋅ log 8 9
a. log10 2 − log10 5 b. log10 5
log 4 log 5 log 6 log 7 log 8 log 9
c. log10 3 + log10 5 d. log10 2 + log10 5 = × × × × ×
log 3 log 4 log 5 log 6 log 7 log 8
Sol. c. log10 15 = log 10 ( 3 × 5) = log10 3 + log10 5
log 9 log 32 2 log 3
= = = =2
Rule 2 If m and n are positive rational numbers, then log 3 log 3 log 3

 m
log a   = log a m − log a n
 n SOME USEFUL RESULTS

LOGARITHM
logarithm of 1 with any base is always zero i.e.
EXAMPLE 4. The value of x in
l

loga 1 = 0
log x − log (x − 1) = log 3 is
l logarithm of base to itself is always 1 i.e. loga a = 1
a. 2/3 b. 3/2 c. 1/2 d. 1/4
l x > y ⇒ loga x > loga y for a > 1
Sol. b. log x − log ( x − 1) = log 3
l x > y ⇒ loga x < loga y for 0 < a < 1
x x 1
⇒ log = log 3 ⇒ =3 for x > 0, 1 ≠ a > 0, loga n ( x ) = loga x
x−1 x−1
l

n
⇒ x = 3 ( x − 1) ⇒ x = 3x − 3 ⇒ −2x = − 3 l for 1 ≠ a ,b > 0, x > 0, a logb x = x logb a
Usually (2-3) questions have been asked from this chapter. Generally questions are based on
If ‘a’ is a positive real number and n is a positive
fundamental rules of logarithm. It is especially used to solve complicated mathematical calculations. ∴ x = 3/ 2 l

rational number, then


Applications of square, cube, surds and indices are extremely used here. Rule 3 If m and n are positive rational numbers, then, x
(i) log y n x = loga n .
log a ( mn ) = n log a m a
y
(ii) a log a n = n
EXAMPLE 5. The value of x in
(iii) loga n = log n 2 = log n 3 = loga m n m
log x 4 + log x 16 + log x 64 = 12 is a2 a3
DEFINITION a. 1 b. 2 c. 3 d. 4
If ‘a’ is a positive real number, other than 1 and ‘b’ is a rational number such that a b = N, then we say Sol. b. log x 4 + log x 16 + log x 64 = 12
that logarithm of N to base ‘a’ is b or ‘b’ is the logarithm of N to base ‘a’, written as log a N = b
Common Logarithm and Natural
So, a = N ⇔ log a N = b ,
b
e.g. 7 = 1 ⇔ log 7 1 = 0, 81
0 1/4
= 3 ⇔ log 81 3 = 1 / 4
⇒ log x 22 + log x 24 + log x 26 = 12 Logarithm
⇒ 2 log x 2 + 4 log x 2 + 6 log x 2 = 12 Logarithm can have any positive base other than 0. But
EXAMPLE 1. Which one of the following has the value of x = 4? ⇒ 12 log x 2 = 12 ⇒ log x 2 = 1
there are two most important base which are generally used.
3 1. Common Logarithm Logarithm to the base ‘10’ is
a. log2 x = 3 b. log5 x = 3 c. log81 x = d. log 2
x=4 ∴ x=2
2 called common logarithm. It is also called Brigg’s
Sol. d. a. log 2 x = 3 ⇔ x = 2 = 8 3
b. log 5 x = 3 ⇔ x = 5 = 1253
[by definition] log 256 logarithm.
EXAMPLE 6. The value of x in = log x is
c. log 81 x = 3 / 2 ⇔ x = (81) 3/ 2 = ( 34) 3/ 2, ⇔ x = 36 ⇔ x = 729 log 16 e.g. log 100 = log 10 100 = 2
d. log 2
x = 4 ⇔ x = ( 2) , x = ( 2 ) = 2 = 4
4 1/ 2 4 2
a. 1 b. 10 c. 100 d. 0 log 1000 = log 10 1000 = 3
EXAMPLE 2. If log5 y = x and log 2 z = x, then the value of 20 x in terms of y and z is log 256 log 162 2 log 16 2. Natural Logarithm Logarithm to the base e is
Sol. c. = = =2
log 16 log 16 log 16 called natural logarithm. It is also called Napier
a. zy b. z2 y c. xy 2 d. z/y
⇒ log x = 2 ⇒ log10 x = 2 [Q log x = log10 x] logarithm. log e x is usually denoted by ln x. e is
Sol. b. Given, log 5 y = x and log 2 z = x ⇒ y = 5x and z = 2x …(i)
∴ x = 102 = 100 irrational number between 2 and 3.
Now, 20x = ( 22 × 5) x = 22x × 5x = ( 2x ) 2 ⋅ 5x = z 2 ⋅ y [from Eq. (i)]
Rule 4 If m is a positive rational number and a, b are ln x = y ⇒ e y = x
∴ 20 = z ⋅ y
x 2

positive real numbers i.e. a ≠ 1 and b ≠ 1, then


Fundamental Rules of Logarithms log a m =
log b m ANTILOGARITHM
Rule 1 If m and n are positive rational number, then log a ( mn) = log a m + log a n log b a The positive number ‘a’ is called the antilogarithm of a
EXAMPLE 7. The value of number b, if log a = b. If ‘a’ is antilogarithm of b, it is
If m1 , m2 , m3 , . . . , mn are positive rational numbers, then
log3 4 ⋅ log4 5 ⋅ log5 6 ⋅ log6 7 ⋅ log 7 8 ⋅ log8 9 is written as a = antilog b.
log a ( m1 m2 m3 . . . mn ) = log a m1 + log a m2 + . . . + log a mn
So, a = antilog b ⇔ log a = b
a. 1 b. 2 c. −1 d. None of these
MATHEMATICS > Logarithm 115 116 CDS Pathfinder

Characteristic and Mantissa


of a Logarithm
first significant figures is at nth place, where n is the
characteristic. PRACTICE EXERCISE
The logarithm of positive real number ‘n’ consists of EXAMPLE 8. The characteristic of the logarithm of
two parts the number 0.00000014 is 1 1 1
1. What is the value of log100 0. 1 ? 16. The value of + + is
1. The integral part is known as the characteristic. It is a. 1 b. 7 logxy ( xyz ) log yz ( xyz ) logzx ( xyz )
(a) 1/2 (b) –1/2 (c) –2 (d) 2
always an integer (positive, negative or zero). c. −7 d. None of these (a) xyz (b) 2 (c) 0 (d) 1
1 2. The value of 3 log 3 + 2 log 2 is
2. The decimal part is called as the mantissa. The Sol. c. 0.00000014 = 1.4 × = 1.4 × 10−7 (a) log 108 (b) log 106 (c) log 109 (d) None of these 17. The value of
mantissa is never negative and is always less than 10000000 1 1 1
∴ The characteristic of log 0.00000014 = − 7 1 + + is
one. 3. If loga 2 = , then the value of a is 1 + logx ( yz ) 1 + log y ( xz ) 1 + logz ( xy )
6
1
To Find the Characteristic EXAMPLE 9. The characteristic of the logarithm of (a) ( 2 )6 (b) (6)1/ 2 (c) 3 (d) −6 (a) 1 (b) (c) x = yz (d) 0
the number 8.188 is xy2
Case I The characteristic of the log of a number 4. If log3 x = − 2, then the value of x is
a. 2 b. 1 18. If log4 ( x + x ) − log4 ( x + 1) = 2, then the value of x
2
greater than 1 is positive and numerically one less than 1 1 1 1
c. −1 d. 0 (a) (b) − (c) (d) − is
the number of digits to the left of the decimal part. 9 9 8 8 (a) 4 (b) 8 (c) 16 (d) 1
e.g. Sol. d. 8.188 = 8.188 × 1 = 8.188 × 100 19. If log4 x + log2 x = 6, then the value of x is
Value Characteristic 5. Find the logarithm of 1728 to the base 2 3.
∴ The characteristic of log 8.188 = 0 (a) 3.124 (b) 3.1732 (c) 6 (d) 5 (a) 16 (b) 4 (c) 2 (d) 1
log 3.257 1− 1 = 0
log 32.57 2 −1= 1 EXAMPLE 10. The characteristic of the logarithm of 6. What is the value of 20. Given log10 2 = 0.3010, the value of log10 5 is
log 3257
. 3−1= 2 the number 566.37 is (log1/ 2 2)(log1/ 3 3)(log1/ 4 4) . . .(log1/ 1000 1000)? (a) 0.6990 (b) 0.6919 (c) 0.6119 (d) 0.7525
a. 2 b. −2 (a) 1 (b) –1 (c) 1 or –1 (d) 0 x y
21. If log + log = log ( x + y ), then
Case II The characteristic of log of a number less than c. 4 d. None of these 7. What is the value of y x
1 is negative and numerically one more than the number Sol. a. 566.37 = 56637
. × 102 1 (a) x + y = 1 (b) x − y = 0 (c) x − y = 1 (d) x = y
of zeroes immediately after the decimal point. It is log 10 25 − 2log 10 3 + log 10 18?
represented by bar over the digit. ∴ The characteristic of log 566. 37 = 2
2 22. The characteristic in log 6.7482 × 10−5 is
(a) 2 (b) 3 (c) 1 (d) 0
(a) 6 (b) − 4 (c) 5 (d) − 5
The bar over the characteristic indicates that it is negative. EXAMPLE 11. The characteristic of the logarithm of 8. What is the value of [log 10 (5 log 10 100)] 2 ?
the number 313 is 23. If 10x = 1.73 and log10 1730 = 3. 2380, then x
e.g. (a) 4 (b) 3 (c) 2 (d) 1
Value Characteristic equals to
a. 3 b. 2
log 0.3257 − ( 0 + 1) = −1 i.e. 1 9. The value of log y x ⋅ logz y ⋅ logx z is (a) 2.380 (b) 0.2380 (c) 2.2380 (d) 1.380
c. 1 d. 0
log 0.03257 − (1 + 1) = −2 i.e. 2 (a) log xyz (b) xyz (c) 1 (d) 0 24. If 22x + 3 = 6x − 1 , then x equals
Sol. b. 313 = 313
. × 102
log 0.003257 − (2 + 1) = −3 i.e. 3 10. The value of log3 ( 27 × 4 9 × 3 9 ) is 4 log 2 + log 3 3 log 2 + 2 log 3
(a) (b)
∴ The characteristic of log 313 = 2 1 1 1 log 3 − log 2 log 3 − 2 log 2
(a) 4 (b) 4 (c) 8 (d) 4 log 48
Rules for Inserting Decimal Point 3 3 6 (c)
log 7
(d) None of these
Two rules are used for inserting a decimal point. SOME IMPORTANT POINTS 11. The value of log2 [log2 log2 log2 ( 65536)] is log10 m + 2 log10 n + 3 log10 p
25. The value of 10 is
Rule 5 When the characteristic of the logarithm is 1. The base of a logarithm is never taken as zero and (a) 8 (b) 16 (c) 4 (d) 1
(a) m2 np3 (b) mn2 p3 (c) m3 np2 (d) None of these
negative.
positive, we insert the decimal point after the ( n + 1) th 2. Log of negative integers are not defined and also
12. What is the value of [log 13(10)]/[log 169(10)] ?
26. Given that log10 2 = 0.3010, log10 3 = 0.4771 and
digit, where n is the characteristic. 1
loge 0 is not defined. (a) (b) 2 (c) 1 (d) log10 13 108
2 log10 7 = 0.8491, then log10 is equal to
3. Logarithmic function is positive as well as negative 7
Rule 6 When the characteristic of the logarithm is
but exponential function is always positive. 13. What is the value of (a) 2.6123 (b) 1.6088 (c) 1.6320 (d) 2.4558
negative, we insert the decimal point such that the 1 
 log 10 125 − 2log 10 4 + log 10 32 + log 10 1 ? 27. If a , b and c are three consecutive integers, then
3 
1 2 log ( ac + 1) is equal to
(a) 0 (b) (c) 1 (d)
5 5 (a) log (2b ) (b) (log b )2 (c) 2 log b (d) None of these
14. If log r 6 = m and log r 3 = n , then what is 28. If logr p = 2, logr q = 3, then the value of log p q is
log r (r / 2) equal to? 1 2 3
(a) m − n + 1 (b) m + n − 1 (c) 1 − m − n (d) 1 − m + n (a) (b) (c) (d) 6
3 3 2
3 4 5 x log x
15. What is log 10   + log 10   + log 10   + L 29. If log x y = a and log = b, then
2 2
is equal to
2 3 4 y log y
upto 8 terms equal to? a − 3b a + 3b a + 2b a − 2b
(a) (b) (c) (d)
(a) 0 (b) 1 (c) log10 5 (d) None of these a + 2b a − 2b a − 2b a + 3b
MATHEMATICS > Logarithm 117 118 CDS Pathfinder

 log 2   log 3   log 4  = log 5 − 4 log 2 + 5 log 2 19. (a) Given, log x + log x = 6
30. If log10 5 = 0.70, then log5 10 is equal to 37. Consider the following statements =      10 10 10 4 2
 − log 2   − log 3   − log 4  = log 5 + log 2 = log 5 × 2 log x log x
(a) 1.35 (b) 1.40 (c) 1.43143 (d) 1.56 I. (log10 0.1)2 + log10 10 ⋅ log10 100 = 3 10 10 10 ⇒ + =6
 log 1000  = log 10 = 1 [Rule 1] log 4 log 2
II. log10log10 10 = 1 III. log10 10 + log10 10 = 1 ...   10
31. The value of  − log 1000  14. (d) Given, log r 6 = m and log r 3 = n log x log x
 1  1  1 Which of the statements given above are correct? ⇒ + =6
log3 1 +  + log3 1 +  + log3 1 +  = ( −1) × ( −1) × ( −1) × . . . × ( −1) Q log r 6 = log r ( 2 × 3) = log r 2 + log r 3 2 log 2 log 2
 3  4  5 (a) I and III (b) II and III
 1 (c) I and II (d) All are correct [Q number of terms is odd] ∴ log r 3 + log r 2 = m ⇒ 3 log x = 12 log 2
+ . . . + log3 1 +  is
(a) −1 + 2 log 5
 24 x
) ... ∞ = −1 ⇒ n + log r 2 = m ⇒ log x = 4 log 2 ⇒ log x = log 24
3 (b) 2 38. If y = ( a x )( a , then which one of the following
⇒ log x = log 16
(c) 3 (d) 4 1 ⇒ log r 2 = m − n
is correct? 7. (c) log 25 − 2 log 3 + log 18
10 10 10 On comparing both sides, we get
2
∴ log r   = log r r − log r 2
r
32. If log ( x + y ) = log x + log y and x = 1.1568, then y (a) log y = xy log a (b) log y = x + y log a
 2
[Rule 2]
∴ x = 16
(c) log y = y + x log a (d) log y = ( y + x) log a = log 251 / 2 − log 32 + log 18
is equal to 10 10 10
[Rule 3] = 1−m + n 20. (a) log 5 = log
10
(a) 7.3776 (b) 7 .3776 (c) 5.3776 (d) 5.3116 10 10
15. (c) log   + log   + log  
= log 5 − log 9 + log 18 3 4 5 2
33. If log8 x + log4 x + log2 x = 11. Then, the value of x PREVIOUS YEARS’ QUESTIONS 10 10 10 10  
2 10  
3 10  
4 = log 10 − log 2 = 1 − 0.3010
5 × 18 90 10 10
is 39. What is the logarithm of 0.0001 with respect to = log = log + L + 8th term
10
9 10
9 = 0.6990
(a) 128 (b) 16 (c) 32 (d) 64 base 10? e 2012 II  n + 2 x y
(a) 4 (b) 3 (c) − 4 (d) − 3
= log 10 = 1 Q Tn = log   21. (a) Given, log + log = log ( x + y )
10 10  
34. What is the value of log y x5 logx y 2 logz z3 ? 8. (d) [log (5 log 100)]2  n + 1
y x
x y
40. If log10 a = p and log10 b = q , then what is the 10 10 ⇒ log ⋅ = log ( x + y )
(a) 10 (b) 30 (c) 20 (d) 60 = [log (5 log 102 )]2  10  y x
value of log10( a pbq )? 10 10 ⇒ T = log  
35. If (log3 x )(log x 2x )(log2x y ) = logx x 2, then what is e 2012 II = [log (10 log 10)]2 8 10  9 ⇒ log 1 = log ( x + y )
p2 10 10
the value of y? (a) p2 + q 2 (b) p2 − q 2 (c) p2 q 2 (d) 2
= [log 10]2 [Q log 10 = 1] ∴ log   + log   + log  
3 4 5 ∴ x+ y =1
10 10
q 10   10   10  
(a)
9
(b) 9 (c) 18 (d) 27 =1 =1 2 2 3 4 22. (d) The characteristic in
41. What are the possible solutions for x of the log 6.7482 × 10−5 is −5.
+ L + log  
2 9. (c) log y x ⋅ log z y ⋅ log x z 10
n
equation x x
= x x , where x and n are positive 10 
9 23. (b) Given, 10x = 1. 73, x = log
36. If log10 2, log10( 2x − 1), log10 ( 2x + 3) are three =
log x log y log z
× × =1 10
1. 73
integers?
= log  × × × L ×  [Rule 1]
e 2015 I 3 4 5 10
consecutive terms of AP, then which one of the 2 2
log y log z log x = log 1730 − log 1000
10 
(a) 0, n (b) 1, n (c) n, n (d) 1, n 9
10 10
following is correct?  log b  2 3 4 = log 1730 − log 103
Q log a b = log a  10 10
= log   = log 5
I. x = 1 II. x = log 2 5
1   10
= 3.2380 − 3 = 0.2380
42. The value of log10 3125 − 4 log10 2 + log10 32 is 10 
5 2 10
(a) Both I and II (b) Only II e 2016 I 10. (d) Let log ( 27 × 4
9× 3
9) = x 24. (a) 22 x + 3
= 6x − 1
3
(c) Only I (d) None of these (a) 0 (b) 1 (c) 2 (d) 3 16. (b) The given expression is
⇒ 3 = 27 ×
x 4
9× 3
9 Taking log on both sides, we get
= log xyz ( xy ) + log xyz ( yz ) + log xyz (zx)
⇒ 3x = 33 × 32 / 4 × 32 / 3 ( 2x + 3) log 2 = ( x − 1) log 6
= log xyz ( xy ⋅ yz ⋅ zx)
ANSWERS 3 =3x 25 / 6

= log xyz ( xyz )2 = 2 log xyz xyz = 2


[Rule 1]
⇒ 2x log 2 + 3 log 2
On comparing both sides, we get = ( x − 1) (log 2 + log 3)
1 b 2 a 3 a 4 a 5 c 6 b 7 c 8 d 9 c 10 d 25 1 ⇒ 2x log 2 + 3 log 2 = x (log 2 + log 3)
⇒ x= =4 17. (a) The given expression is
11 d 12 b 13 c 14 d 15 c 16 b 17 a 18 c 19 a 20 a 6 6 1 − log 2 − log 3
=
21 a 22 d 23 b 24 a 25 b 26 b 27 c 28 c 29 c 30 c 11. (d) log log log log 216 log x ( yz ) + log x x ⇒ x (log 2 − log 3) = − 4 log 2 − log 3
2 2 2 2
= log log log ( 16) [Q log 2 = 1] 1 4 log 2 + log 3
31 a 32 a 33 d 34 b 35 b 36 b 37 a 38 a 39 c 40 a 2 2 2 2
+ ∴ x=
= log log log ( 24 ) = log log ( 4) log y ( xz ) + log y y log 3 − log 2
41 a 42 b 2 2 2 2 2

= log log ( 2 ) = log ( 2) = 1 2 1


2 2 2 + 25. (b) 10 log 10 m + 2 log
10
n + 3 log
10
p
log z ( xy ) + log z z
log (10) log (10) m + log n 2 + log p3
= 10 log 10
HINTS AND SOLUTIONS 12. (b)
log
13

169
(10)
=
log
13

13 2
(10) =
1
log x xyz
+
1
log y xyz
+
1
log z xyz
⇒ 10 log 10 mn 2 p 3
= mn2 p 3
10 10

[Q
log 10  a log a= p]p
Q log c = log a c 
 1 1 On comparing both sides, we get x = 6 1
1. (b) log 0.1 = log   3. (a) Q log a 2= ⇒ a1/6
= 2 = 13
= log xyz x + log xyz y + log xyz z
10 2  10 
 
log 10 
100 6 1 ab b 108
6. (b) (log 2)(log 3)(log 4) = log xyz xyz = 1 [Rule 1] 26. (b) log = log 108 − log 7
∴ a = ( 2 )6 1/2 1/3 1/4 2 13 10
7 10 10
= log   = log ( 10)−1
1 1 1
10 
. . . (log 1000) 1
2 10  2 10
4. (a) log x = − 2 ⇒ x = 3 −2
1
= 2 =
1 1 /1000 = =2 18. (c) log ( x2 + x) − log ( x + 1) = 2 = log 2 × 3 − log
2 3
71/2
 log 2   log 3   log 4  1/ 2 4 4 10 10
1 1 3
3 9 =       x2 + x  x2 + x
= − log 10 = −
⇒ log   = 2 ⇒ 42 =
[Rule 3]  log 1 / 2   log 1 / 3   log 1 / 4  1 1
2 10
2 13. (c) We have, log 125 − 2 log 4 = 2 log 2 + 3 log 3− log 7
1728 = x 4  
 x+ 1 x+ 1
5. (c) Let log 10 10 10 10
2 10
2 3   3
2. (a) 3 log 3 + 2 log 2 = log 33 + log 22 ⇒ ( 2 3 )x = 1728  log 1000   log a  + log 32 + log 1
... Q log b a = log b  ⇒ 16x + 16 = x2 + x = 2 × (0. 3010) + 3(0.4771)
 log 1/ 1000   
10 10
= log 27 + log 4 [Rule 3] 1728 = 26 ( 3 )6 = ( 2 3 )6   = log (125)1 / 3 − 2 log (2)2 1
Q
10 10 ⇒ x2 − 15x − 16 = 0 − (0.8491)
= log ( 27 × 4) = log 108 [Rule 1] ( 2 3 )x = ( 2 3 )6 [Rule 4] 2
+ log (2)5 + 0 [Q log 1 = 0] ∴ x = 16 or x = − 1 [not possible]
10 10 = 0.6020 + 1.4313 − 0.4245 = 1.6088
14
MATHEMATICS > Logarithm 119 120 CDS Pathfinder

x ...∞
27. (c) Let first integer be a. 11log x 38. (a) Given, y = ( a x )(a )
⇒ = 11 ⇔ log x = 6 log 2
Then, b = a + 1 and c = a + 2 6 log 2
⇒ x = 26 , ∴ x = 64 ∴ y = (a ) x y
∴ ac + 1 = a ( a + 2) + 1
34. (b) ∴ log y x5 log x y 2 log z z 3 On taking log both sides, we get
= a 2 + 2a + 1 = ( a + 1 )2
= 5 log y x ⋅ 2 log x y ⋅ 3 log z z log y = y log a x
ac + 1 = b 2
[Qlog a b n = n log a b] ∴ log y = xy log a
So, log ( ac + 1) = log b = 2 log b 2

28. (c) Given, log r p = 2 and log r q = 3 = 5 log y x ⋅ 2 log x y ⋅ 3 × 1 39. (c) Let, log 0.0001 = x
10
log r q 3 5 log x log y  log b  1
= × 2⋅ × 3 Q log a b = Then, x = log
By relation, log q =
p
= log y log x  log a  10
( 10)4

ALGEBRAIC
log r p 2
= 5 × 2 × 3 = 30
= log 1 − log ( 10)4
29. (c) Given, log x2 y 2 = a 35. (b) (log x) (log x 2x) (log y ) = log x x2
10 10
3 2x =0−4=−4
[Q log (mn) = log m + log n] log x log 2x log y log x2
⇒ × × = 40. (a) Given, log a = p and log b = q
∴ log x2 + log y 2 = a

OPERATIONS
log 3 log x log 2x log x 10 10
⇒ 2 log x + 2 log y = a …(i) log ( a p b q ) = log a p + log b q
 log a 
Q log b a = log b 
10 10 10
Also, log x − log y = b …(ii)
  ⇒ log ( a p b q ) = p log a + q log b
On solving Eqs. (i) and (ii), we get 10 10 10
log y 2 log x ∴ log ( a b ) = p × p + q × q
p q
a + 2b a − 2b ⇒ = [Q log a b = b log a ] 10
log x = and log y = log 3 log x
4 4 = p2 + q2
log x a + 2b ⇒ log y = 2 log 3 ⇒ log y = log 32
∴ = [Q log m = log n ⇒ m = n]
log y a − 2b 41. (a) Given, x x
= n
xx , where x and n
⇒ log y = log 9 are positive integers.
log
10
10 1 Generally (3-4) questions have been asked from this chapter. Questions which are asked from
30. (c) log 10 = = = 143143
. ∴ y=9 On taking log both sides, we get
5 log 5 0.70 this chapter are mostly based on direct identities and factor theorem.
10 36. (b) Given that, log 2, log ( 2 − 1) and x
[Q log 10 = 1] 10 10 log x x
= log[ n xx ]
10 log ( 2x + 3) are in AP.
10
⇒ x log x = log( xx )1 / n = log xx/n
31. (a) log  1 +  + log  1 + 
1 1 then,
3
3 3 
4 2 log ( 2x − 1) = log 2 + log ( 2x + 3) ⇒
x
x log x = log x
10 10 10
+ log  1 +  + . . . + log  1 +
1 1 ⇒ log ( 2x − 1)2 = log ( 2 x + 1 + 6)
n
Algebraic Expressions
3 

5 3 
24 
10 10 x
⇒ ( 2x − 1)2 = ( 2 + 6) x +1 ⇒ x log x − log x = 0
4 5 6 n A combination of constants and variables connected by the four fundamental operations +, −, × and ÷ is
= log + log + log + ⇒ 22 x + 1 − 2.2x = 2 ⋅ 2x + 6 called an algebraic expression.
log x  x −  = 0
3 3 3 x
3 4 5 ⇒ 2 − 4.2 − 5 = 0
2x x ⇒
25  n 
. . . + log ⇒ ( 2x )2 − 4 ( 2x ) − 5 = 0
3
24
⇒ y2 − 4y − 5 = 0
Q log x ≠ 0 POLYNOMIALS
x A polynomial is an algebraic expression consisting of variables and coefficients, having non-negative
= log 4 − log 3 + log 5 − log 4 ∴ y = 5, − 1 ∴ x−
=0
3 3 3 3
n
+ log 6 − log 5 Hence, 2x = −1 or 2x = 5 x integral powers. e.g. x 3 + 5x 2 − 1, 3x 3 + 4x 2 y + 17 etc.
3 3
⇒ x=
+ . . . + log 25 − log 24 ∴ x = log ( −1) is not possible n (i) 3x 2 + 9x − 1 + 7 / x is not a polynomial as it contains a term, namely 7 / x, having negative integral
3 3 2
or x = log (5) ∴
x
=n power of variable x.
= − log 3 + log 25 = − 1 + 2 log 5 2
3 3 3 Then, x = log ( 5 ) is answer. x
32. (a) Given, log ( x + y ) = log x + log y 2 (ii) 5x 2 − 7 x 7/2 + x − 1 is not a polynomial as the term −7 x 7/2 contains rational power of variable x.
Hence, II is correct. On squaring both sides, we get
⇒ log ( x + y ) = log xy x2
37. (a) I. [log (0.1)]2 + log 10 ⋅ log 100 =n Degree of a Polynomial
10 10 10
⇒ x + y = xy x
= [ − log 10]2 + log 10 ⋅ log 102 The exponent of the highest degree term in a polynomial, is known as degree of polynomial.
⇒y=
x
=
11568
.
=
11568
. 10 10 10
⇒ x 2 − nx = 0
= ( −1) + (1) ⋅ 2 log 10 e.g. (i) 4x 3 − 9x 2 + 7 x + 9 is a polynomial with variable x of degree 3.
2
x − 1 11568
. − 1 0.1568 10 ⇒ x( x − n) = 0
= + 1 + (1) ⋅ 2 = 1 + 2 = 3 ∴ x = 0, x = n (ii) 4x 3 y − 3x 2 + 2xy + 1 is a polynomial of degree 4.
= 7.37755 = 7.3776
Hence, statement I is correct.
33. (d) Given, log x + log x + log x = 11 Hence, the possible solution of x is 0, n.
8 4 2 II. log log 10 = log (1) = 0 ≠ 1 Various Types of Polynomials
log x log x log x 10 10 10
1
⇒ + + = 11 Hence, statement II is incorrect. 42. (b) log 3125 − 4 log 2 + log 32
log 8 log 4 log 2 5 10 10 10 Polynomials are classified on the basis of degree of polynomial and number of terms which are given
III. log 10 + log 10
10
1
10
1 = log [(5)5 ]1 / 5 − log ( 2)4 + log 32
below.
log x log x log x = log 10 + log 10
⇒ + + = 11 2 10
2 10
10 10 10
log 23 log 22 log 2
1 1 = log 5 − log 16 + log 32
Based on Degree of Polynomial
= + =1 10 10 10
log x log x log x
2 2 (i) Constant polynomial A polynomial of degree zero, is called constant polynomial. e.g. f ( x) = 8.
⇒ + + = 11 5 × 32 
3 log 2 2 log 2 log 2 Hence, statement III is correct. = log   = log10 = 1 Note The polynomial f( x) = 0 is called the zero polynomial. The degree of zero polynomial is not defined.
10 
16 
MATHEMATICS Algebraic Operations 121 122 CDS Pathfinder

(ii) Linear polynomial A polynomial of degree 1, is • ( a 3 − b 3 ) = ( a − b) ( a 2 + b 2 + ab) Factorisation of Perfect Sol. c. 4 3x2 + 5x − 2 3 = 4 3x2 + 8x − 3x − 2 3
called linear polynomial. e.g. 9x + 7, x − 9, x + 2 etc. • ( a + b + c ) 2 = a 2 + b 2 + c 2 + 2 ( ab + bc + ac ) Square Polynomials [Q 4 3 × ( − 2 3) = − 24 = 8 × ( − 3)]
(iii) Quadratic polynomial A polynomial of degree 2, is • a 3 + b 3 + c 3 − 3abc = ( a + b + c ) ( a 2 + b 2 + c 2 = 4x ( 3x + 2) − 3( 3x + 2)
If the polynomial is given as the perfect square
called quadratic polynomial. − ab − ac − bc) = ( 3x + 2)( 4x − 3)
quadratic polynomial, then use the following Identities
The general form of quadratic polynomial in x is • If a + b + c = 0, then a 3 + b 3 + c 3 = 3abc to factorise it.
ax 2 + bx + c, where a, b and c are real and a ≠ 0.
Factorisation of Sum and
• a 4 + a 2 b 2 + b 4 = ( a 2 + ab + b 2 ) ( a 2 − ab + b 2 ) • a 2 + 2ab + b 2 = ( a + b) 2 • a 2 − 2ab + b 2 = ( a − b) 2 Difference of Cubes
e.g. x 2 − 7 x + 12, 5 y 2 − 7, z 2 − 4 etc., are all quadratic
In case, the polynomial is given in the form of a 3 + b 3 or
polynomials. Methods of Factorisation EXAMPLE 4. Factorise 16 x 2 − 8x + 1
a 3 − b 3 . To factorise it, following identities can be
(iv) Cubic polynomial A polynomial of degree 3, is These methods are given below a. ( 2x − 1) 2
b. ( x − 4)2
c. ( x − 2)
2
d. ( 4 x − 1)
2
applied.
called cubic polynomial. Sol. d. 16x2 − 8x + 1 = ( 4x) 2 − 2( 4x) + 12 = ( 4x − 1) 2 • a 3 + b 3 = ( a + b)( a 2 − ab + b 2 )
Factorisation by Taking Out the
The general form of a cubic polynomial in x is = ( 4x − 1)( 4x − 1) • a 3 − b 3 = ( a − b)( a 2 + ab + b 2 )
Common Factor
ax 3 + bx 2 + cx + d , where a, b, c and d are real and a ≠ 0. EXAMPLE 5. Factorise x 2 − 2 5x + 5 EXAMPLE 9. Factorise (2x + 3y ) 3 − (2x − 3y ) 3
If each term of an expression has a common factor, take
e.g. 3x 3 + 2x 2 + 7 x − 1 , 8 y 3 + 5 y + 2, x 3 − 8 etc., are out the common factors. a. 18( 4 x2 + 3y 2 ) b. 18 y ( 3x2 + 4 y 2 )
a. ( x 5 − 1)2 b. ( x − 5)2 c. ( x + 5)( x − 5) d. ( x − 5)2
all cubic polynomials. c. xy ( 4 x2 + 3y 2 ) d. 18 y ( 4 x2 + 3y 2 )
EXAMPLE 1. Factorise value of 4(3a −2b) − 5(3a −2b) is Sol. d. x − 2 5x + 5 = x − 2 5x + ( 5) = ( x − 5)
2 2 2 2 2
(v) Biquadratic polynomial A polynomial of degree 4,
a. ( 3a − 2b)(12a − 8b − 5) b. ( 2a − 3b)(12a − 8b) = ( x − 5)( x − 5) Sol. d. ( 2x + 3y) − ( 2x − 3y)
3 3
is called a biquadratic polynomial.
c. ( 2a − 3b)( 8 a − 12b − 5) d. None of these Put 2x + 3y = a and 2x − 3y = b
The general form of a biquadratic polynomial in x is Factorising the Difference of Two Squares
⇒ a3 − b3 = ( a − b)( a2 + ab + b2) = ( 2x + 3y − 2x + 3y)
ax 4 + bx 3 + cx 2 + dx + e, where a, b, c, d and e are Sol. a. 4 ( 3a − 2b) 2 − 5( 3a − 2b) = ( 3a − 2b)[ 4( 3a − 2b) − 5] In case, the polynomial is given in the form of a 2 − b 2 , [( 2x + 3y) 2 + ( 2x + 3y)( 2x − 3y) + ( 2x − 3y) 2 ]
real numbers and a ≠ 0. = ( 3a − 2b)[12a − 8b − 5] to evaluate it, following identity is applied. = 6y[ 4x2 + 9y 2 + 12xy + 4x2 − 9y 2 + 4x2
e.g. 6x 4 + 3x 3 + 2x 2 + x + 2, x 4 − 8 etc., • a 2 − b 2 = ( a − b)( a + b) + 9y 2 − 12xy ]
Factorisation by Grouping = 6y[12x2 + 9y 2 ] = 18y[ 4x2 + 3y 2 ]
are all biquadratic polynomials.
Sometimes in a given polynomial, it is not possible to EXAMPLE 6. Factorise 2a 5 − 32a
Hence, ( 2x + 3y) 3 − ( 2x − 3y) 3 = 18y[ 4x2 + 3y 2 ]
Based on Number of Terms take out a common factor directly. However, on
a. ( a2 + 4)( a − 2)( a + 2) b. 2a ( a2 + 4)( a − 2)( a + 2)
rearranging the terms of the polynomial and grouping Factorisation of Polynomial of form
(i) Monomial A polynomial containing only one term them such that all the terms have a common factor, the c. 2a ( a + 4)
2
d. None of these
is called a monomial. e.g. 6x 2 polynomial can be easily factorised. a3 + b3 + c 3 − 3abc
Sol. b. 2a5 − 32a = 2a( a4 − 16)
(ii) Binomial A polynomial containing two terms is Here, it is easy to use.
EXAMPLE 2. Factorise x 2 + y − xy − x = 2a[( a2) 2 − ( 4) 2 ] = 2a( a2 + 4)( a2 − 4)
called a binomial. e.g. 5x 3 + 7 xy • a 3 + b 3 + c 3 − 3abc
= 2a( a2 + 4)( a2 − 22) = 2a( a2 + 4)( a − 2)( a + 2) = ( a + b + c )( a 2 + b 2 + c 2 − ab − bc − ac )
(iii) Trinomial A polynomial containing three terms is a. ( x − y )( x + 1) b. ( x − y )( x 2 − 1)
• If a + b + c = 0, then a 3 + b 3 + c 3 = 3abc
called a trinomial. e.g. 7 x 2 y + 5x + 6 c. ( x + y )( x2 + 1) d. ( x − y )( x − 1) Factorisation of Quadratic Polynomials
Sol. d. Method I x + y − xy − x = x − x + y − xy
2 2 Quadratic polynomials of the type ax 2 + bx + c, where EXAMPLE 10. Factorise x 3 + 27 y 3 + 8x 3 − 18xyz
FACTORISATION a ≠ 0, a and b are coefficients of x 2 and x respectively a. ( x + 3y + 2z)( x2 + 9 y 2 + 4 z2 + 3xy + 6 yz + 2xz)
= x ( x − 1) + y(1 − x) = x ( x − 1) − y( x − 1) = ( x − 1)( x − y)
and c is constant, can be factorised by splitting the
To express a polynomial as the product of other Method II x2 + y − xy − x = x2 − xy + y − x b. ( x + 3y + 2z)( x2 + 9 y 2 + 4 z2 − 3xy − 6 yz − 2xz)
middle term. We find two numbers p and q such that
polynomials of degree less than that of the given = x ( x − y) − 1( x − y) = ( x − y)( x − 1) c. ( x − 3y − 2z)( x2 + 9 y 2 − 4 z2 − 3xy + 6 yz + 2xz)
p + q = b and pq = ac, then
polynomial is called as factorisation. d. None of the above
4 4 ax 2 + bx + c = ax 2 + ( p + q)x + c = ax 2 + px + qx + c
e.g. x − 49 = x − 7 = ( x − 7 )( x + 7 )
2 2 2
EXAMPLE 3. Factorise x 2 + + 4 − 2x − Sol. b. x3 + 27y 3 + 8z 3 − 18xyz
x2 x
 2  2   2  2  EXAMPLE 7. Factorise x 2 + 9x + 14 = x3 + ( 3y) 3 + ( 2z) 3 − 3( x)( 3y)( 2z)
Important Identities a.  x −   x + + 2 b.  x −   x + − 2 = ( x + 3y + 2z)[ x2 + 9y 2 + 4z 2 − x ( 3y)
 x  x   x  x 
( a − b ) = ( a + b)( a − b)
2 2 a. ( x + 2)( x + 7) b. ( x − 2)( x − 7) − 3y( 2z) − x ( 2z)]
•  2  2 
= ( x + 3y + 2z)([ x2 + 9y 2 + 4z 2 − 3xy − 6yz − 2xz ])
c.  x +   x + − 2 d. None of these c. ( x + 2)( x − 7) d. ( x − 2)( x + 7)
• ( a + b) 2 = a 2 + b 2 + 2ab and ( a − b) 2 = a 2 + b 2 − 2ab  x  x 
( a + b) 2 − ( a − b) 2 = 4ab Sol. a. x2 + 9x + 14 = x2 + (7 + 2) x + 14 = x2 + 7x + 2x + 14 EXAMPLE 11. Factorise
Sol. c.  x2 + 2 + 4 − 2x −  =  x2 + 2 + 4 − 2  x + 
• 4 4 4 2
 (2x − 3y ) 3 + (3y − 5z) 3 + (5z − 2x) 3
• ( a + b) 2 + ( a − b) 2 = 2 ( a 2 + b 2 ) x x  x   x = x ( x + 7) + 2( x + 7) = ( x + 7)( x + 2)
( a + b) 3 = a 3 + b 3 + 3ab ( a + b) 2 2
a. 3( 2x − 3y )( 3y − 5z)( 5z − 2x)
=  x +  − 2  x +  =  x +   x + − 2
• 2 2 2 EXAMPLE 8. Factorise 4 3x 2 + 5x − 2 3
 x  x  x   b. ( 2x − 3y )( 3y − 5z)( 5z − 2x)
• ( a − b) 3 = a 3 − b 3 − 3ab ( a − b) x
a. ( 2x + 3)( 4 x − 3) b. ( 2x − 3)( 4 x + 3) c. 3( 2x − 3y )( 3y + 5z)( 5z − 2x)
• ( a 3 + b 3 ) = ( a + b) ( a 2 + b 2 − ab)
c. ( 3x + 2)( 4 x − 3) d. None of these d. None of the above
MATHEMATICS Algebraic Operations 123 124 CDS Pathfinder

Sol. a. Here, 2x − 3y = a, 3y − 5z = b and 5z − 2x = c


Q a + b + c = 0 ⇒ a 3 + b 3 + c 3 = 3abc
∴ ( 2x − 3y) 3 + ( 3y − 5z) 3 + ( 5z − 2x) 3
EXAMPLE 14. If two factors of a 4 − 2a 3 − 9a 2 + 2a + 8
are (a +1) and (a −1), then what are the other two
factors?
PRACTICE EXERCISE
= 3( 2x − 3y)( 3y − 5z)( 5z − 2x) a. ( a − 2) and ( a + 4) b. ( a + 2) and ( a + 4)
c. ( a + 2) and ( a − 4) d. ( a − 2) and ( a − 4)
x = 5, y = 3 and z = 2, then 10. What are the factors of  x 2 − 2x − 9 ?
Remainder Theorem 1. If the value of 1
Sol. c. Let f ( a) = a4 − 2a3 − 9a2 + 2a + 8 x 2 + y 2 + z 2 − 2xy + 2 yz − 2zx is 3 
Let p( x) be a polynomial of degree n greater than or and ( a + 1) and ( a − 1) are two factors of f ( a). 1 1
equal to 1 (i.e. n ≥ 1) and a be any real number. If p( x) is (a) 125 (b) 0 (c) −25 (d) 10 (a) (x − 9)(x + 3) (b) (x − 9)(x − 3)
Now, by division method, 3 3
divided by the linear polynomial ( x − a), then the a2 –1) a4 – 2a3 – 9a2 + 2a + 8 (a2 – 2a – 8 2. The factors of x 2 − 2 3x + 3 are 1 1
(c) (x + 9)(x + 3) (d) (x + 9)(x − 3)
remainder is p( a). Remainder can be evaluated by a4 – a2 (a) (x + 3 )2 (b) (x − 3 )2 3 3
substituting x − a = 0, i.e. x = a in p( x) . (c) (x + 3 )(x − 3) (d) (x + 2 )(x + 3)
– 2a3 – 8a2 + 2a + 8 11. What are the factors of ( 6 3x 2 − 47x + 5 3 )?
EXAMPLE 12. The remainder when 3. The factors of ( a 4b4 − 16c4 ) are
– 2a3 2a (a) (3 3x + 5 3 )(2 x − 5 3 )
12x 3 − 13x 2 − 5x + 9 is divided by (3x + 2) is (a) 4(a2 b 2 + c 2 )(ab − 2c )(ab + 2c )
– 8a2 +8 (b) (3 3x − 5 3 )(x 3x + 1)
a. 1 b. 2 c. 3 d. 0 – 8a2 +8 (b) (a2 b 2 − 4 c 2 )(ab + 2c )2 (c) (2 x + 5 3 )(3 3x + 1)
Sol. c. Let p( x) = 12x3 − 13x2 − 5x + 9 and q( x) = 3x + 2 × (c) (a2 b 2 + 4 c 2 )(ab + 2c )(ab − 2 c ) (d) (2 x − 5 3 )(3 3x − 1)
−2
When p ( x) is divided by ( 3x + 2), then remainder is p   . So, the required factor of f ( a) is ( a2 − 2a − 8). (d) (a2 b 2 − 4 c 2 )2 (ab + 2c )(ab + 4 c ) 12. The factors of ( 8a3 + 125b3 − 64c3 + 120abc) are
 3 Further factorise it by splitting the middle term, i.e.
−2 −2 3
−2 2
−2 (a) (2 a + 5b − 4 c ) (2 a + 5b + 4c )
Now, p   = 12   − 13   − 5   + 9 a − 2a − 8 = a − 4a + 2a − 8 4. The factors of ( x 8 − y 8 ) are
2 2
 3  3  3  3 (b) (2 a − 5b − 4 c ) (2 a + 5b + 4c )
= a( a − 4) + 2( a − 4) = ( a + 2)( a − 4) (a) (x4 + y4 )(x2 + y2 )(x + y)(x − y)

= 12 ×   − 13 × +
8 4 10 Hence, ( a + 2) and ( a − 4) are other two factors of f ( a). (c) (2 a + 5b − 4c ) (4a2 + 25b 2 + 16c 2 − 10ab + 20bc + 8ac )
+9=3 (b) (x2 + y2 )2 (x + y)(x − y)
 27 9 3 (d) (2 a + 5b + 4c ) (4a2 + 25b 2 + 16c 2 − 10ab + 20bc + 8ac )
(c) (x4 + y4 )(x2 + y2 )2
Hence, the required remainder is 3. Factorisation of Polynomials
(d) (x + y )(x − y)
2 2 2 13. If x1/ 3 + y1/ 3 + z1/ 3 = 0, then
Factor Theorem Using Factor Theorem
5. The factors of  a 2 + a +  are (a) x + y + z = 0 (b) ( x + y + z)3 = 27 xyz
1
Let p( x) be a polynomial, if degree n ≥ 1 and a be any To factorise the polynomial, follow the following steps  4 (c) x + y + z = 3 xyz (d) x3 + y3 + z3 = 0
(i) Let p ( x) be a given polynomial. 2
real number. Then, (a)  a +  (b)  a +  (a + 2 )
1 1 1 1
 2  2 14. If x + = 5, then the value of x3 + 3 is
(i) If p( a) = 0, then ( x − a) is a factor of p( x) . (ii) find all the possible factors of constant term in p ( x). x x
2
(ii) If ( x − a) is a factor of p( x), then p( a) = 0. (iii) Take any one of the factors, say (α ) and find p (α ). If (c)  a +   a −  (d)  a − 
1 1 1 (a) 8 5 (b) 2 5 (c) 5 5 (d) 7 5
 2  2  2
p (α ) = 0 the ( x − α ) is a factor of p ( x).
15. If  x +  = 6, then  x 2 + 2  is equal to
1 1
EXAMPLE 13. For what values of k will 6. The factors of 8 − 4x − 2x3 + x 4 are
4x 5 + 9x 4 − 7 x 3 − 5x 2 − 4kx + 3k 2 (iv) Divide the polynomial p ( x) by ( x − α ) to find its all  x  x 
contain x −1 as a factor? other factors. (a) (2 − x)(4 − x3 ) (b) (2 + x)(4 − x3 ) (a) 32 (b) 38 (c) 34 (d) 44
1 1 1 (c) (2 + x)(3 − x3 ) (d) (2 − x)(x3 − 4)
a. 3, − b. 3, −1 c. 0, d. 1, EXAMPLE 15. Factorise 2x − 5x − 19x + 42 . 3 2
16. If x −
1 1 9
= , then what is 9x 2 + 2 equal to?
2 3 3
7. The factors of ( a 2 − b2 − 4ac + 4c2 ) are x 3 x
Sol. d. Given, P( x) = 4x5 + 9x4 − 7x3 − 5x2 − 4kx + 3k 2 a. ( x − 2) ( 2x − 7) ( x + 3) b. ( x + 2) ( 2x + 7) ( x − 3)
(a) (a + 2c + b )(a − 2c − b ) (b) (a − 2c + b )(a − 2c − b ) (a) 18 (b) 19 (c) 20 (d) 21
Since, ( x − 1) is a factor of P( x). ∴ P(1) = 0 c. ( x − 1) ( 3x − 7) ( x − 4) d. ( 2x + 3) ( 4 x − 7) ( x − 1) (c) (a − 2 b + c )(a + b + 2c ) (d) (a − 2 b )(a + 2 b + 2c )
17. If a + b + c = 6 and a 2 + b2 + c2 = 26, then what is
⇒ 4 × (1) 5 + 9 × (1) 4 − 7 × (1) 3 − 5 × (1) 2 −4 × k × (1) + 3 × k 2 = 0 Sol. a. Let p ( x) = 2x3 − 5x2 − 19x + 42 8. What are the factors of x 2+ 4 y 2 + 4 y − 4xy ab + bc + ca equal to?
⇒ 3k 2 − 4k + 1 = 0 ⇒ 3k 2 − 3k − k + 1 = 0 Hence, constant term is 42 and its all factors are = ± 1, −2x − 8? (a) 0 (b) 2 (c) 4 (d) 5
1 ± 2, ± 3, ± 6, ± 7, ± 14, ± 21 and ± 42. (x − 2 y − 4) and (x − 2 y + 2 )
⇒ ( 3k − 1)( k − 1) = 0 ⇒ k = , 1 (a)
3
At x = 1, p (1) = 2(1) 3 − 5(1) 2 − 19 + 42 (b) (x − y + 2 ) and (x − 4 y + 4) 18. If ( x 4 + x − 4 ) = 322, then what is one of the value
= 2 − 5 − 19 + 42 = 20 ≠ 0 (c) (x − y + 2 ) and (x − 4 y − 4) of ( x − x − 1 )?
Division Synthetic Algorithm (x + 2 y − 4) and (x + 2 y + 2 )
So, ( x − 1) is not a factor of p ( x). (d) (a) 18 (b) 16 (c) 8 (d) 4
If p( x) and g( x) any two polynomials with g( x) ≠ 0, then
At x = 2, p ( 2) = 2( 2) 3 − 5( 2) 2 − 19( 2) + 42 = 58 − 58 = 0 9. Factorise a3 −
1
− 2a +
2
we can find polynomials q( x) and r ( x) such that 3 19. If a + b + c = 0, then what is the value of
So, ( x − 2) is a factor of p ( x). a a
p( x) = g( x) × q( x) + r ( x) 1  2 1 a 2 b2 c2
(a)  a + 
Now, on dividing p ( x) by ( x − 2), we get ( 2x 2 − x − 2) as 1 + + ?
 a + 2  − 2 a − 
i.e. Dividend = (Divisor × Quotient) + Remainder quotient i.e. other factor of p( x).  a  a   a bc ca ab
1  2 
(b)  a − 
1 1 (a) − 3 (b) 0 (c) 1 (d) 3
where, r ( x) = 0 or degree of r ( x) < degree of g( x) , then So, p ( x) = ( x − 2)[ 2x2 − x − 21]   a + 2 + 1 + 2  a − 
 a  a   a
we say that p( x) divided by g( x), gives q ( x) as quotient = ( x − 2) ( 2x2 − 7x + 6x − 21) 20. If a + b + c = 10 and ab + bc + ca = 31, then the
(c)  a +
1  1
and r ( x) as remainder.  a −  value of a 2 + b2 + c2 is
= ( x − 2)[ x ( 2x − 7) + 3 ( 2x − 7)]  a   a 
If the remainder r ( x) is zero, we say that divisor g ( x) is ∴ p ( x) = ( x − 2)( 2x − 7)( x + 3) (d) None of the above (a) 48 (b) 38 (c) 28 (d) 18
a factor of p ( x).
MATHEMATICS Algebraic Operations 125 126 CDS Pathfinder

21. The factors of x 2 +


1
+ 1 − 2x −
1
are 32. If the remainder of the polynomial PREVIOUS YEARS’ QUESTIONS 52. x( y 2 − z 2 ) + y( z 2 − x 2 ) + z( x 2 − y 2 ) is divisible by
4x 2 x a0 + a1x + a2x 2 + L + an x n when divided by ( x − 1) (a) Only ( y − z) e 2013 II
41. What is the value of k that ( 2x − 1) may be a
(a)  x −
1 
+ 2  (b)  x +
1 
− 2  (b) Only ( z − x)
1 1
 x −  x + is 1, then which one of the following is correct?
 2x  2x   2x  2x  (a) a0 + a2 + L = a1 + a3 + L
factor of 4x 4 − ( k − 1)x3 + kx 2 − 6x + 1? e 2012 I (c) Both (y − z) and ( z − x)
(c)  x −   − 2  (d)  x +
1 
− 2  (d) Neither ( y − z) nor ( z − x)
1 1 1 a0 + a2 + L = 1 + a1 + a3 + L (a) 8 (b) 9 (c) 12 (d) 13
 x −  x − (b)
 2x  2x   2x  2x 
(c) 1 + a0 + a2 + L = − (a1 + a3 + L ) 42. x 4 + xy3 + x3 y + xz3 + y 4 + yz3 is divisible by 53. Consider the following statements
22. The factors of ( a 2 − b2 ) ( c2 − d 2 ) − 4abcd are (d) 1 − a0 − a2 − L = a1 + a3 + L (a) Only (x − y) e 2012 I I. x + 3 is the factor of x3 + 2x2 + 3x + 8.
(a) (ac + bd + bc + ad ) (ac − bd − bc − ad ) (b) Only (x 3 + y 3 + z 3) II. x − 2 is the factor of x3 + 2x2 + 3x + 8.
33. If u , v and w are real numbers such that
(b) (ac − bd − bc + ad ) (ac + bd + dc + ab ) (c) Both (x + y) and (x 3 + y 3 + z 3 )
u3 − 8v3 − 27w3 = 18 uvw, then which one of the Which of the statement(s) given above is/are
(c) (ac − bd + bc + ad ) (ac − bd − bc − ad ) (d) None of the above correct? e 2013 II
following is correct?
(d) (ac − bd − bc − ad ) (ac + bd + dc + ab ) 43. If x + 5x + 10k leaves remainder −2x when
3 2 (a) Only I (b) Only II
(a) u − v + w = 0 (b) u = − v = − w
23. If ‘a’ is an integer such that a +
1 17
= , then the (c) u − 2 v = 3w (d) u + 2 v = − 3w divided by x 2 + 2, then what is the value of k? (c) Both I and II (d) Neither I nor II
a 4
34. The value of ( a1/ 8 + a − 1/ 8 ) ( a1/ 8 − a − 1/ 8 ) ( a1/ 4 + a − 1/ 4 ) e 2012 I 54. x3 + 6x 2 + 11x + 6 is divisible by e 2014 I
 1
value of  a −  is (a) –2 (b) –1 (c) 1 (d) 2 (a) Only (x + 1) (b) Only (x + 2 )
 a ( a1/ 2 + a − 1/ 2 ) is
1 (c) Only (x + 3) (d) All of these
(a) 4 (b)
13
(c)
17
(d)
15 (a) (a + a− 1 ) (b) (a − a− 1 ) (c) (a2 + a− 2 ) (d) (a2 − a− 2 ) 44. If x + = a , then what is the value of
4 4 4 x 55. What should be added to the x ( x + a ) ( x + 2a )
24. x + 4 y is divisible by which one of the following?
4 4 35. The polynomial f ( x ) = x 4 − 2x3 + 3x 2 − ax + b, 1 1
x3 + x 2 + 3 + 2 ? ( x + 3a ), so that the sum be a perfect square?
(a) (x + 2 xy + 2 y )
2 2
(b) (x + 2 y )
2 2 when divided by ( x − 1) and ( x + 1) leaves the x x e 2012 I
(a) 9 a2 (b) 4a2 e 2014 I
remainders 5 and 19, respectively. The values of (a) a3 + a2 (b) a3 + a2 − 5a
(c) (x − 2 y )
2 2 (c) a4 (d) None of these
(d) None of these a and b are (c) a3 + a2 − 3a − 2 (d) a3 + a2 − 4a − 2
56. If  x 2 +
25. If x ( x + y + z ) = 9, y ( x + y + z ) = 16 and (a) a = 4, b = 3 (b) a = 3, b = 4
1  17  3 1
45. If p ( x ) is a common multiple of degree 6 of the  = , then what is  x − 3  equal to?
(c) a = 5, b = 8 (d) a = 9, b = 7  x2  4  x 
z ( x + y + z ) = 144, then x equal to polynomials f ( x ) = x + x − x − 1 and
3 2 e 2014 I
9 9 9 16 75 63 95
(a) (b) (c) (d) 36. If the expression ax 2 + bx + c is equal to 4 when g ( x ) = x3 − x 2 + x − 1, then which one of the (a) (b) (c) (d) None of these
5 7 13 13
x = 0, leaves a remainder 4 when divided by x + 1 following is correct?
16 8 8
26. If x = ( b − c) ( a − d ), y = ( c − a ) ( b − d ) and z = ( a − b) and a remainder 6 when divided by x + 2., then
e 2012 I
57. What is the remainder when x5 − 5x 2 + 125 is
(a) p(x) = (x − 1)2 (x + 1)2 (x2 + 1)
( c − d ), then what is x3 + y3 + z3 equal to? the values of a , b and c are respectively:
(b) p(x) = (x − 1) (x + 1) (x2 + 1)2 divided by x + 5? e 2014 II
(a) xyz (b) 2xyz (c) 3xyz (d) − 3xyz (a) 1, 1, 4 (b) 2, 2, 4 (c) 3, 3, 4 (d) 4, 4, 4
(c) − 3125
(c) p(x) = (x − 1)3 (x + 1) (x2 + 1) (a) 0 (b) 125 (d) 3125
27. If the expression ( px + x − 2x − q ) is divisible by
3 2
37. If ( x − xy + x y − y
3/ 2 1/ 2 1/ 2 3/ 2
) is divided by (d) p(x) = (x − 1)2 (x4 + 1) 58. If ax + by − 2 = 0 and axby = 1, where a ≠ 0, b ≠ 0,
( x − 1) and ( x + 1), what are the values of p and q x1/ 2 − y1/ 2 the quotient is 46. What is the value of k which will make the then what is ( a 2x + b2 y ) equal to? e 2014 II
respectively? (a) x − y (b) x + y (c) x 1/ 2
+ y
1/ 2
(d) x − y
2 2
expression 4k2 + 12k + k a perfect square?e 2012 II
(a) 2, − 1 (b) − 2, 1 (c) − 2, − 1 (d) 2, 1 (a) a + b (b) 2ab (c) a 3 + b 3 (d) a 4 + b 4
38. If, 9x + 3 px + 6q when divided by 3x + 1 leaves a
2 (a) 5 (b) 7 (c) 8 (d) 9
28. If the expression px3 + 3x 2 − 3 and 2x3 − 5x + p 3 47. The factors of 5 px − 10qy + 2rpx − 4 qry is/are 59. If ( x + k) is the common factor of x 2 + ax + b and
when divided by x − 4 leave the same remainder,
remainder −
and qx 2 + 4 px + 7 is exactly
4 (a) Only (5 + 2 r ) e 2013 I x 2 + cx + d, then what is k equal to? e 2014 II
then what is the value of p? divisible by x + 1, then the values of p and q (b) Only ( px − 2qy) (a) (d − b ) / (c − a ) (b) (d − b ) / (a − c )
(a) − 1 (b) 1 (c) − 2 (d) 2 respectively are (c) Both (5 + 2 r ) and ( px − 2qy) (c) (d + b ) / (c + a ) (d) (d − b ) / (c + a )
29. Let f ( x ) = a0x n + a1x n −1 + a2x n − 2 +...+ an −1x + an , (a) 0, 7 / 4 (b) −7 / 4, 0 (d) Neither (5 + 2 r ) nor ( px − 2qy)
7
(c) , 0 (d) None of these 60. Consider the following statements:
where a0 , a1 , a2... an are constants. If f ( x ) is
4 48. ( a + 1)4 − a 4 is divisible by e 2013 I I. (a − b − c) is one of the factors of
divided by ax − b, the remainder is (a) − 2 a 2 + 2 a − 1 (b) 2 a 3 − 2 a − 1
−b  −a 39. If a + b + c = 1, then which of the following 3abc + b3 + c3 − a3 .
(a) f   (b) f  (c) f   (d) f  
b a
 statements are true? (c) 2 a 3 − 2 a + 1 (d) 2 a 2 + 2 a + 1 II. (b + c − 1) is one of the factors of 3bc + b + c − 1.
3 3
 a  a   b  b
I. (a + b)(b + c)(c + a ) = bc + ac + ab − abc
30. Which one of the following statements is correct? 49. One of the factors of the polynomial Which of the statement(s) given above is/are
(a) Remainder theorem is a special case of factor II. a 2 + b2 − c2 + 2ab = a + b − c x 4 − 7x3 + 5x 2 − 6x + 81 is correct? e 2014 II
e 2013 I
theorem (a) Only I (b) Only II (a) x + 2 (b) x − 2 (c) x + 3 (d) x − 3 (a) Only I (b) Only II
(b) Factor theorem is a special case of remainder (c) Both I and II (d) Neither I nor II (c) Both I and II (d) Neither I nor II
theorem
1
50. If the expression x3 + 3x 2 + 4x + k has a factor
(c) Factor theorem and remainder theorem are two 40. If x + = 4, then the value of expression x + 5, then what is the value of k? e 2013 II
61. For what value of k is ( x − 5) a factor of
independent results x x3 − 3x 2 + kx − 10 ?
(d) None of the above (a) −70 (b) 70 (c) 48 (d) −48 e 2015 I
1 (a) −8
I. x3 + = 52 II. x = 2 + 3 (b) 4 (c) 2 (d) 1
31. What is x ( y − z ) ( y + z ) + y ( z − x ) ( z + x ) + z( x − y ) x3 51. The quantity which must be added to
(1 − x )(1 + x 2 ) to obtain x3 is 62. If x + y + z = 0, then x + y + z + 3xyz is equal to
3 3 3
( x + y ) equal to? Which of the following is correct? e 2013 II
(a) (x + y) ( y + z) ( z + x) (b) (x − y) (x − z) ( z − y) (a) Only I (b) Only II (a) 2 x3 + 3x2 + x + 1 (b) 2 x3 + x2 + x − 1 e 2015 I
(c) 2 x3 − x2 + x − 1 (d) − x3 + x − 1 (a) 0 (b) 6xyz (c) 12xyz (d) xyz
(c) (x + y) ( z − y) (x − z) (d) ( y − x) ( z − y) (x − z) (c) Neither I nor II (d) Both I and II
MATHEMATICS Algebraic Operations 127 128 CDS Pathfinder

11. (d) 6 3 x2 − 47x + 5 3 1 25. (c) Given, x ( x + y + z ) = 9,


66. If the polynomial x 6 + px5 + qx 4 − x 2 − x − 3 is 18. (d) Given, x4 + = 322
63. The expression x3 q 2 − x3 pt + 4x 2 pt −4x 2q 2 = 6 3 x2 − 45x − 2x + 5 3 x4 y ( x + y + z ) = 16
+3xq − 3xpt is divisible by
2 divisible by ( x 4 − 1), then the value of 2
⇒  x2 + 2  − 2 = 322
e 2015 I 1 and z ( x + y + z ) = 144
= 3 3 x ( 2x − 5 3 ) − 1 ( 2x − 5 3 )
(a) Only (x − 1) (b) Only (x − 3) p2 + q 2 is e 2016 I  x 
= ( 2x − 5 3 ) (3 3 x − 1) 2 On adding all three equations, we get
(c) Both (x − 1) and (x − 3) (d) Neither (x − 1) nor (x − 3) (a) 1 (b) 9 (c) 10 (d) 13 ⇒  1
 x + 2  = 324 = 18
2 2
x( x + y + z ) + y( x + y + z ) + z( x + y + z )
12. (c) 8a 3 + 125b 3 − 64c 3 + 120abc  x 
64. Which one of the following is correct? e 2015 II 67. If the linear factors of ax 2 − ( a 2 + 1) x + a are p 1 = 9 + 16 + 144
= ( 2a )3 + (5b )3 − ( 4c )3 + 120abc ⇒ x2 + 2 = 18
(a) (x + 2 ) is a factor of x4 − 6x3 + 12 x2 − 24x + 32 and q, then p + q is equal to e 2016 I x ⇒ ( x + y + z ) ( x + y + z ) = 9 + 16 + 144
= ( 2a )3 + (5b )3 + ( − 4c )3 − 3( 2a ) 2
(a) (x − 1) (a + 1) (b) (x + 1) (a + 1) ⇒ ( x + y + z )2 = 169

⇒  x −  + 2 = 18 ⇒ x − = 4
(b) (x + 2 ) is a factor of x4 + 6x3 − 12 x2 + 24x − 32 1 1
(c) (x − 1) (a − 1) (d) (x + 1) (a − 1) (5 b ) ( − 4 c )  ⇒ x + y + z = 13
x x
(c) (x − 2 ) is a factor of x4 − 6x3 + 12 x2 − 24x + 32 = ( 2a + 5b − 4c ) ( 4a 2 + 25b 2 + 16 c 2 Q x ( x + y + z) = 9
(d) (x − 2 ) is a factor of x4 + 6x3 − 12 x2 + 24x − 32 x 10 y 19. (d) Given, a + b + c = 0 9
68. If = − and x − y = 8, then the value of − 10ab + 20bc + 8ac) ⇒ x ( 13) = 9 ⇒ x =
y 3 x ⇒ a 3 + b 3 + c 3 = 3 abc 13
65. For what value of k ( x + 1) is a factor of [Q a 3 + b 3 + c 3 − 3abc = ( a + b + c )
xy is equal to e 2016 I
On dividing both sides by abc, we get 26. (c) Given, x = ( b − c ) ( a − d ),
x3 + kx 2 − x + 2 ? e 2016 I ( a + b + c − ab − bc − ac )]
2 2 2
a3 b3 c3 y = ( c − a) ( b − d )
(d) −2 (a) 36 (b) 24 (c) 16 (d) 9 ⇒ + + =3
(a) 4 (b) 3 (c) 1 13. (b) Since, x 1 / 3 + y 1 / 3 + z1 / 3 = 0 abc abc abc and z = ( a − b) ( c − d )
∴( x 1 / 3 )3 + ( y 1 / 3 )3 + (z1 / 3 )3 a2 b2 c2 Q x + y + z = ( b − c) ( a − d )
⇒ + + =3
−3 x 1 / 3 y 1 / 3z1 / 3 = 0 bc ac ab + ( c − a) ( b − d )
ANSWERS ⇒ x + y + z − 3( xyz )1 / 3 = 0 20. (b) Q ( a + b + c )2 = a 2 + b 2 + c 2
=0
+ ( a − b) ( c − d )
⇒ x + y + z = 3( xyz )1 / 3 + 2ab + 2bc + 2ac
1 b 2 b 3 c 4 a 5 a 6 a 7 b 8 a 9 d 10 a Hence, x3 + y 3 + z 3 = 3xyz
⇒ ( x + y + z )3 = 27xyz ( 10)2 = ( a 2 + b 2 + c 2 ) + 2 (31)
11 d 12 c 13 b 14 b 15 c 16 b 17 d 18 d 19 d 20 b 3 27. (d) Here, px3 + x2 − 2x − q is divisible
a 2 + b 2 + c 2 = 100 − 62
14. (b) Q  x +  = x3 + 3 + 3  x + 
1 1 1 by ( x − 1) and ( x + 1).
21 b 22 c 23 d 24 a 25 c 26 c 27 d 28 b 29 a 30 b  x x  x ⇒ a 2 + b 2 + c 2 = 38
1 ∴ p ( 1)3 + ( 1)2 − 2 ( 1) − q = 0
31 b 32 d 33 c 34 b 35 c 36 a 37 b 38 c 39 c 40 d ( 5 )3 = x3 + 3 + 3 ( 5 ) 1 1 ⇒ p−q=1 …(i)
x 21. (b) x2 + + 1 − 2x −
41 d 42 c 43 c 44 c 45 a 46 d 47 c 48 d 49 d 50 b 4x 2 x and p ( − 1)3 + ( − 1)2 − 2 ( − 1) − q = 0
Q x + 1 = 5 
⇒ p + q =3
=  x2 + + 1 − 2  x +
51 c 52 c 53 d 54 d 55 c 56 b 57 c 58 a 59 a 60 c 1 1 …(ii)
 x    

61 a 62 b 63 c 64 c 65 d 66 c 67 a 68 d 4x 2 2x  On solving Eqs. (i) and (ii), we get
1
⇒ x3 + =5 5 −3 5 = 2 5 2 p = 2 and q = 1
=  x +
x3 1  1
 − 2 x + 
 2x   2x  28. (b) Let f ( x) = px3 + 3x2 − 3
15. (c) Given,  x +  = 6
1
and g ( x) = 2x3 − 5x + p.
=  x +
 x 1  1 
HINTS AND SOLUTIONS On squaring both sides, we get

 
2x  
x +
2x
− 2 
 On dividing f ( x ) by ( x − 4), remainder is
2
 x + 1  = 62 22. (c) ( a 2 − b 2 )( c 2 − d 2 ) − 4abcd f ( 4) = p ( 4)3 + 3 ( 4)2 − 3
  = 64 p + 48 − 3 = 64 p + 45
1. (b) x2 + y 2 + z 2 − 2xy + 2 yz − 2zx 5. (a)  a 2 + a +  = a 2 + a + a + ∴ z 2 − 2z − 8 = z 2 − 4z + 2z − 8  x
1 1 1 1 = a 2 c 2 − a 2 d 2 − b 2 c 2 + b 2 d 2 − 4abcd
 4 2 2 4 = z(z − 4) + 2(z − 4) 1 = ( a 2 c 2 + b 2 d 2 − 2abcd ) Similarly g( 4) = 2( 4)3 − 5 ( 4) + p
= ( x − y − z ) 2 = (5 − 3 − 2 ) 2 = 0 ⇒ x2 + 2 + 2 = 36
= (z + 2)(z − 4)
= a  a +  +  a +  − ( b 2 c 2 + a 2 d 2 + 2abcd )
1 1 1 x = 128 − 20 + p = 108 + p
[Q x = 5, y = 3 and z = 2] 1
 2 2  2 Now, put z = x − 2 y ⇒ x + 2 = 34
2
= ( ac − bd ) − ( bc + ad )2
2
But f ( 4) = g ( 4) [given]
2. (b) x2 − 2 3 x + 3 ∴ ( x − 2 y + 2) x
2
= ( ac − bd + bc + ad )( ac − bd − bc − ad ) ∴ 64 p + 45 = 108 + p
=  a +   a +  =  a + 
1 1 1
= x2 − 3 x − 3 x + 3 and ( x − 2 y − 4) are required factors. 1 1
 2  2  2 16. (b) Given, x − = 2 ⇒ 63 p = 63 ⇒ p = 1
23. (d) Q  a −  =  a + 1 − 4
x 3 1
= x( x − 3 ) − 3 ( x − 3) 1
9. (d) a 3 − 3 − 2a +
2   29. (a) ax − b = 0 ⇒ x = b / a
6. (a) 8 − 4x − 2x3 + x4 3  a  a
= (x − 3) (x − 3) = (x − 3) 2 a a ⇒ 3x − = 1
= 4 ( 2 − x) − x 3 ( 2 − x)
3 x  17 
2 By remainder theorem, if f ( x ) is divisible
= a 3 −   − 2  a − 
1 1 =   −4 by ax − b, the remainder is f ( b / a ).
3. (c) ( a 4 b 4 − 16c 4 ) = [( a 2 b 2 )2 − ( 4c 2 )2 ] = ( 2 − x) ( 4 − x 3 )  a  a On squaring both sides, we get  4
2
 17  30. (b) Factor theorem is a special case of
=  a −   a 2 + 2 + 1 − 2  a −   3x − 3  = ( 1)2 Q given  a +  =
1 1 1 1
= [ a 2 b 2 + 4c 2 ] [ a 2 b 2 − 4c 2 ] 7. (b) ( a 2 − b 2 − 4ac + 4c 2 )    remainder theorem.
 a  a   a  x  a 4 
[Q a 2 − b 2 = ( a − b ) ( a + b )] = ( a 2 − 4ac + 4c 2 ) − b 2  1  2 1  31. (b) x ( y − z )( y + z ) + y (z − x) (z + x)
=  a −   a + 2 − 1 9 289 − 64
[( a )2 − 2( 2c )( a ) + ( 2c )2 ] − b 2  9x 2 + 2 − 2 × 9 = 1 = =
225 15
= + z (x − y) (x + y)
= [ a 2 b 2 + 4c 2 ] [( ab )2 − ( 2c )2 ] a  a  x 16 16 4
= ( a − 2c ) − b
2 2
( x2 − 6x − 27) = x ( y 2 − z 2 ) + y (z 2 − x 2 )
10. (a)  x2 − 2x − 9 =
= [ a 2 b 2 + 4c 2 ] [( ab + 2c ) ( ab − 2c )] 1 9
⇒ 9x2 + 2 = 19 24. (a) x4 + 4 y 4 + z ( x2 − y 2 )
= ( a − 2c − b ) ( a − 2c + b ) 3  3 x
4. (a) x8 − y 8 = [( x4 )2 − ( y 4 )2 ] 1 = x 4 + 4 y 4 + 4x 2 y 2 − 4x 2 y 2 = x ( y 2 − z 2 ) + yz 2 − yx2
8. (a) x2 + 4 y 2 + 4 y − 4xy − 2x − 8 = [ x2 − 9x + 3x − 27] 17. (d) Q ( a + b + c )2 = a 2 + b 2 + c 2 + 2 + zx2 − zy 2
= [ x4 + y 4 ][ x4 − y 4 ] 3 = ( x 2 + 2 y 2 ) 2 − ( 2 xy ) 2
= x2 − 4xy + 4 y 2 − 2x + 4 y − 8 ( ab + bc + ca ) = ( y − z ) ( xy + xz − x2 − yz )
1
= [ x( x − 9) + 3 ( x − 9)] = ( x2 + 2 y 2 − 2xy )
= [ x4 + y 4 ][( x2 )2 − ( y 2 )2 ] = ( y − z ) [ y ( x − z ) + x (z − x)]
= ( x − 2 y )2 − 2 ( x − 2 y ) − 8 3 ∴ ( 6)2 = 26 + 2 ( ab + bc + ca ) ( x 2 + 2 y 2 + 2 xy )
= ( x4 + y 4 )( x2 + y 2 )( x2 − y 2 ) 1 = ( y − z ) (z − x ) ( x − y )
Let x − 2 y = z = ( x + 3) ( x − 9) ⇒ 2 ( ab + bc + ca ) = 10 From above it is clear that x4 + 4 y 4 is
= ( x4 + y 4 ) ( x2 + y 2 ) ( x + y ) ( x − y ) 3 divisible by x + 2 y + 2xy
2 2 = ( x − y ) ( x − z ) (z − y )
⇒ ab + bc + ca = 5
MATHEMATICS Algebraic Operations 129 130 CDS Pathfinder

− 3 × . x x −  =
4 3
32. (d) Let f ( x) = a + a x + a x2 37. (b) x 3 / 2 − xy 1 / 2 + x 1 / 2 y − y 3 / 2 = x y 2 − x 3 + x 3 − x y2 = 0 1 1 1 27
⇒ 4 ×   − ( k − 1) ×  
1 1 46. (d) Put the value of all options in ⇒ x3 −
0 1 2
+ L + an x n = x ( x 1 / 2 − y 1 / 2 ) + y( x 1 / 2 − y 1 / 2 )  2  2 4k 2 + 12k + k So, (z − x) is also a factor, so it is also x3 x  x 8
divisible by (z − x).
+ 3 ×  
Since, ( x − 1) is a factor of f ( x).
2
when, k = 9, 1 27 3
= ( x 1 / 2 − y 1 / 2 )( x + y ) + k ×   − 6 × + 1 = 0 ⇒ x3 − 3 =
1 1
Put x = 1 in f ( x), then  2 53. (d) Put x = − 3 in p ( x ), we get x 8  2
⇒ Quotient 2 then, 4( 9)2 + 12( 9) + 9 = 441= ( 21)2
f ( 1) = a + a + a + L + an ( x 1 / 2 − y 1 / 2 )( x + y ) 1 ( k − 1) k 47. (c) Given expression Let p ( x ) = x + 2x + 3x + 8
3 2 [From Eq. (i)]
0 1 2
= =x+ y ⇒ − + −2=0 1 27 9
⇒ 1 = a + a + a + L + an ( x 1 / 2 − y1 / 2 ) 4 8 4 = 5 px − 10 qy + 2rpx − 4 qry p( −3) = ( − 3) 3 + 2 ( − 3)2 + 3 ( − 3) + 8 ⇒ x3 −
= +
0 1 2
x3 8 2
∴ 1− a − a −L= a + a + L k ( k − 1) 1 = (5 px + 2rpx) − ( 10 qy + 4 qry ) = −27 + 18 − 9 + 8
0 2 1 3 38. (c) Let f ( x ) = 9x 2 + 3 px + 6q ⇒ − =2−  1  63
∴ x − 3  =
33. (c) Given, (u )3 + ( − 2v )3 + ( − 3w )3 = px (5 + 2r ) − 2 qy (5 + 2r ) = − 10 ≠ 0
3

Given, f ( −1 / 3) = −3 / 4 4 8 4
= (5 + 2r ) ( px − 2 qy )  x  8
= 3 × ( − 2) ( − 3) uvw 2 2k − ( k − 1 ) 8−1 7 So, ( x + 3) is not the factor of
⇒ 9 −  + 3 p  −  + 6q = −3 / 4 ⇒ = =
1 1 So, factors of given expression are both x3 + 2x2 + 3 x + 8. 57. (c) Let f ( x ) = x5 − 5x2 + 125
∴ u + ( − 2v ) + ( − 3w ) = 0
 3  3 8 4 4 (5 + 2r ) and ( px − 2qy ). put x = −5,
[Q a + b 3 + c 3 = 3abc , then
3 Similarly, put x = 2 in p ( x ), we get Now,
⇒ 1 − p + 6 q = −3 / 4 ⇒ 8k − 4k + 4 = 56 48. (d) Given, ( a + 1)4 − a 4
a + b + c = 0] p( 2) = ( 2)3 + 2 ( 2)2 + 3 ( 2) + 8 ∴ Required remainder = f ( − 5)
⇒ u − 2v − 3w = 0 ⇒ 24q − 4 p + 7 = 0 ⇒ 4k = 52 ⇒ k = 13 = {( a + 1)2 − a 2 } {( a + 1)2 + a 2 } = 30 ≠ 0 So, x − 2 is also not the factor
…(i)
= {( a + 1) + a } {( a + 1) − a } = ( −5)5 − 5 ( − 52 ) + 125
∴ u − 2v = 3w Let g( x ) = qx 2 + 4 px + 7 Hence, the value of k is 13. of x3 + 2x2 + 3x + 8.
{ a 2 + 1 + 2a + a 2 } = − 3125 − 125 + 125 = − 3125
34. (b) We know that, ( a + b ) ( a − b ) Since, ( x + 1) is a factor of g( x ) 42. (c) Given, x4 + xy 3 + x3 y + xz 3 = ( 2a + 1) ( 1) ( 2a 2 + 2a + 1) 54. (d) Let f ( x) = x3 + 6x2 + 11x + 6
58. (a) Given, ax + by − 2 = 0
= a2 − b2 ∴ g( −1) = 0 ⇒ q − 4 p + 7 = 0…(ii) + y 4 + yz 3 = ( 2a + 1) ( 2a + 2a + 1)
2
Put x = − 1 in f ( x), we get
So, ( a + 1)4 − a 4 is divisible by ⇒ a x + by = 2
So, [( a1 / 8 + a − 1 / 8 ) ( a1 / 8 − a −1 / 8 ) On solving Eq. (i) and Eq. (ii), we get = ( x4 + xy 3 + xz 3 ) + ( x3 y + y 4 + yz 3 ) f ( −1) = ( −1)3 + 6( −1)2 + 11( −1) + 6
( 2a 2 + 2a + 1) Squaring on both sides, we get
( a1 / 4 + a − 1 / 4 ) ( a1 / 2 + a − 1 / 2 ) ] q = 0 and p = 7 / 4 = −1 + 6 − 11 + 6
= x( x + y + z ) + y( x + y + z )
3 3 3 3 3 3
( ax + by )2 = ( 2)2
= [( a1 / 8 )2 − ( a −1 / 8 )2 ( a1 / 4 + a −1 / 4 )] 49. (d) Let f ( x) = x4 − 7x3 + 5x2 − 6x + 81
39. (c) I. ( a + b )( b + c )( c + a ) = ( x + y ) ( x3 + y 3 + z 3 ) = −12 + 12 = 0
−1 / 4 −1 / 4 f (3) = (3)4 − 7(3)3 + 5(3)2 − 6(3) + 81 ⇒ a 2 x2 + b 2 y 2 + 2axby = 4
= (a1/4
−a )( a 1/4
+a ) = ( 1 − c )( 1 − a )( 1 − b ) Hence, ( x + 1) is a factor of f ( x).
Hence, given polynomial is divisible by = 81 − 189 + 45 − 18 + 81 = 0
( a1 / 2 + a −1 / 2 ) [Q a + b + c = 1] ⇒ a 2 x2 + b 2 y 2 + 2 = 4 [Q axby = 1 ]
both ∴ f ( x ) = ( x + 1)( x2 + 5x + 6 )
= [a −a − 1/2
+ a − 1/2 ∴( x − 3) is a factor of f ( x )
1/2
] [a 1/2
] = ( 1 − c ) [ 1 − b − a + ab ] ( x + y ) and ( x3 + y 3 + z 3 ). = ( x + 1)( x2 + 3x + 2x + 6 ) ⇒ a 2 x2 + b 2 y 2 = 2
= ( a − a− 1 ) = 1 − b − a + ab − c + bc + ac − abc 50. (b) Let f ( x) = x3 + 3x2 + 4x + k = ( x + 1)( x + 2 )( x + 3 ) ⇒ a 2 x2 = 1 and b 2 y 2 = 1
43. (c) x2 + 2 ) x3 + 5x2 + 10k ( x + 5
35. (c) f ( x) = x4 − 2x3 + 3x2 − ax + b = 1 − ( a + b + c ) + ab + bc + ac − abc Q ( x + 5) is a factor of f ( x) Hence, ( x + 1), ( x + 2 ) and ( x + 3 ) are ⇒ ax = 1 and by = 1
x3 + 2x
= ab + bc + ac − abc − − ∴ f ( −5 ) = 0
Put x = 1 as f ( 1) = 5, f ( − 1) = 19 the factors of f ( x ). 1 1
II. a 2 + b 2 − c 2 + 2ab 5x2 – 2x + 10k ⇒ x = and y =
f ( 1) = 1 − 2 ( 1) + 3 ( 1) − a + b
4 3 2 ⇒ ( −5) + 3( −5) + 4( −5) + k = 0
3 2
55. (c) x( x + a ) ( x + 2a ) ( x + 3a ) a b
= a 2 + b 2 + 2ab − c 2 5x2 + 10 ⇒ −125 + 75 − 20 + k = 0
⇒1− 2 + 3 − a + b =5 − − = ( x + a )( x + 2a )x ( x + 3a ) ∴
1 1
a2 x + b2 y = a2 ⋅ + b2 ⋅ = a + b
= ( a + b )2 − c 2
⇒ − a + b =3 …(i) - 2x - 10 + 10k = Remainder ⇒ −70 + k = 0 ⇒ k = 70 a b
= ( a + b + c )( a + b − c ) = a + b − c = ( x 2 + 3ax + 2a 2 )( x 2 + 3ax )
and f ( −1) = ( −1) − 2( −1) + 3 4 3 But given that, remainder = − 2x Hence, the value of k is 70. 59. (a) Given, x + k is the common factor of
[Q a + b + c = 1] = ( x 2 + 3ax + a 2 + a 2 )
( − 1)2 − a ( − 1) + b ∴ − 2x − 10 + 10 k = − 2x 51. (c) Let p( x) be added to obtain x3 . x2 + ax + b and x2 + cx + d. Then, put
So, both statements are correct. ( x 2 + 3ax + a 2 − a 2 )
⇒ 1 + 2 + 3 + a + b = 19 1 ⇒ −10 + 10 k = 0 Then, ( 1 − x)( 1 + x ) + p( x) = x
2 3 x = − k in given polynomial.
40. (d) We have, x + = 4 = ( x + 3ax + a ) − a
2 2 2 4
⇒ a + b = 13 …(ii) x ⇒ 10 k = 10 ⇒ k = 1 ⇒ 1 + x2 − x − x3 + p( x) = x3 ∴ k 2 − ka + b = 0
3 So, a 4 must be added to k 2 − kc + d = 0
I.  x +  = x3 + 3 + 3 ( x)
1 1 1 ⇒ p( x) = x3 − 1 − x2 + x + x3 and
On solving Eqs. (i) and (ii), we get 44. (c) Given that, x + =a ...(i) x ( x + a )( x + 2a )( x + 3a ) to make it a
a = 5, b = 8  x x x Then, k 2 − ka + b = k 2 − kc + d
∴ p( x) = 2x3 − x2 + x − 1 perfect square.
×    x +  d −b
1 1 1 1
36. (a) Let f ( x ) = ax 2 + bx + c Then, x3 + x2 + 3 + 2 So, 2x3 − x2 + x − 1 is added to  x2 + 1  = 17 ⇒ k ( c − a) = d − b ⇒ k =
 x  x x x 56. (b) Given,   c−a
Given f ( 0) = 4 1 ( 1 − x)( 1 + x2 ) to obtain x3 .  x2  4
( 4) = x + 3 + 3 ( 4)
3 3  1  2
= x + 3  + x + 2 
3 1
60. (c) I. We have, 3abc + b 3 + c 3 − a 3
⇒0 + 0 + c = 4 ⇒a − b + 4 = 4 ⇒c = 4  1 17
x x   x  52. (c) Let f(x) = x( y 2 − z 2 ) + y(z 2 − x 2 ) ⇒ x2 + 2 − 2 + 2 =
On dividing f ( x ) by x + 1, remainder is 4 1 1 x 4 = − ( a 3 − b 3 − c 3 − 3abc )
64 − 12 = x3 + 3 ⇒ x3 + 3 = 52 3 2
+ z( x − y )
2 2
=  x +  − 3  x +  +  x +  − 2
1 1 1
∴ f ( −1) = 4 x x  1
2
17 = − [ a 3 + ( − b )3 + ( − c )3 − 3( a )( − b )( − c )]
1  x  x  x ⇒ x −  + 2 =
⇒ a−b+ c= 4 ⇒a−b+ 4= 4 II. x + = 4 ⇒ x − 4x + 1 = 0 2 If it is divisible by ( y − z ), then y − z is  x 4 = − ( a − b − c )( a 2 + b 2 + c 2 + ab
⇒ a−b = 0 …(i) x = a 3 − 3a + a 2 − 2 = a 3 + a 2 − 3a − 2 a factor of f ( x ). 2 − bc + ac)
4 ± 16 − 4 × 1 × 1  1 17
On dividing f ( x ) by x + 2, remainder is 6 x= 45. (a) Given, f ( x) = x3 + x2 − x − 1 ∴ y −z = 0 ⇒ y =z ⇒ x −  = −2 So, ( a − b − c ) is a factor of
2×1  x 4
∴ f ( −2 ) = 6 and g( x) = x3 − x2 + x − 1 On putting y = z, we get 3abc + b 3 + c 3 − a 3 .
4± 2 3  1 9
2
⇒ 4a − 2b + c = 6 = = 2± 3 x(z 2 − z 2 ) + z(z 2 − x 2 ) + z( x 2 − z 2 ) ⇒ x −  =
2 p( x ) = f ( x) ⋅ g ( x)  x 4
Hence, statement I is correct.
⇒ 4a − 2b = 2 …(ii) = ( x3 + x2 − x − 1) ( x3 − x2 + x − 1) = z 3 − zx 2 + zx 2 − z 3 = 0 II. Now, 3bc + b 3 + c 3 − 1
⇒ a−b+ 4= 4
Hence, both statements are correct.
⇒  x − 1 = 3
= [ x 2 ( x + 1) − 1( x + 1)] Hence, y − z is a factor, so it is divisible   …(i)
= b 3 + c 3 − ( 1)3 − 3bc( −1)
41. (d) Let f ( x ) = 4x4 − ( k − 1)x3 + kx2  x 2
On solving Eq. (i) and eq. (ii), we get by ( y − z ).
−6x + 1 …(i) [ x 2 ( x − 1) + 1( x − 1)] = ( b + c − 1)[ b 2 + c 2 + 12 − bc + c + b ]
a = 1, b = 1 On putting z = x, we get On cubing both sides, we get
Since, ( 2x − 1) is a factor of f ( x ) = [( x + 1)( x 2 − 1)][( x − 1)( x 2 + 1)] 3 3 So, ( b + c − 1) is a factor of
Hence, x ( y2 − x 2 ) + y(x 2 − x 2 )  x − 1 =  3 
= ( x + 1)( x + 1)( x − 1)( x − 1)( x 2 + 1)     3bc + b 3 + c 3 − 1 .
f   = 0  x  2
1
a = 1, b = 1, c = 4. ∴ + x (x 2 − y 2 )
 2 = ( x + 1)2 ( x − 1)2 ( x 2 + 1)
Hence, statement II is also correct.
15
MATHEMATICS Algebraic Operations 131 132 CDS Pathfinder

61. (a) Let p( x) = x3 − 3x2 + kx − 10 64. (c) In option (c), we have = x( ax − 1) − a ( ax − 1)


Then, ( x − 5) is a factor of p( x). ( 2)4 − 6 ( 2)3 + 12( 2)2 − 24 × 2 + 32 = ( x − a )( a x − 1)
∴ p(5) = 0 = 16 − 6(8) + 12( 4) − 48 + 32 If p and q are linear factors of f ( x), then
= 16 − 48 + 48 − 48 + 32 = 0 p = ( x − a ) and q = ( ax − 1)
⇒ 5 − 3(5) + k × 5 − 10 = 0
3 2

Hence, ( x − 2) is a factor of ∴ p + q = ( x − a ) + ( ax − 1)
⇒ 125 − 75 + 5k − 10 = 0
x 4 − 6x 3 + 12x 2 − 24x + 32 = ( x − 1)( a + 1)
⇒ 40 + 5k = 0 ⇒ k = −8.
65. (d) Let f ( x) = x3 + kx2 − x + 2 68. (d) We have, x − y = 8 …(i)
62. (b) Q x3 + y 3 + z 3 − 3xyz = ( x + y + z ) x 10 y
If ( x + 1) is a factor of f ( x). and = −
( x + y + z − xy − yz − zx)
2 2 2

HCF AND LCM OF


Then, f ( −1) = 0 y 3 x
Given, x+ y +z = 0 ⇒ ( −1)3 + k( −1)2 − ( −1) + 2 = 0 x y 10
⇒ + =
∴ x + y + z 3 − 3xyz = 0
3 3
⇒ −1 + k + 1 + 2 = 0 ⇒ k = − 2 y x 3
⇒ x + y + z 3 + 3xyz = 6xyz
3 3
66. (c) Let f ( x ) = x6 + px5 + qx4 ( x )2 + ( y )2 10
⇒ =
63. (c) x3 q 2 − x3 pt + 4x2 pt − 4x2 q 2

= x3 q 2 − x3 pt − 4x2 q 2
+ 3xq 2 − 3xpt
Then, f ( 1) = f ( − 1) = 0
− x2 − x − 3
Q f ( x ) is divisible by ( x4 − 1).

Now, f ( 1) = 1 + p + q − 1 − 1 − 3 = 0

xy

x+ y =
3
3
10
xy

On squaring both sides, we have


POLYNOMIALS
+ 4x2 pt + 3xq 2 − 3xpt 100
⇒ p+ q=4 …(i) ( x + y )2 = xy
= x3 ( q 2 − pt ) − 4x2 ( q 2 − pt ) 9
and f ( − 1) = 1 − p + q − 1 + 1 − 3 100
+3x( q 2 − pt ) ⇒ ( x − y ) + 4 xy =
2
xy
= ( x − 4x + 3x)( q − pt )
3 2 2 ⇒ q− p=2 …(ii) 9
On solving Eqs. (i) and (ii), we get q = 3 [Q ( x + y )2 = ( x − y )2 + 4xy ] Regularly (1-2) questions have been asked from this chapter. Generally questions are based on your
= x( x2 − 4x + 3)( q 2 − pt )
and p = 1 ⇒ (8)2 =  − 4 xy [from Eq. (i)]
100
= x ( x2 − 3x − x + 3)( q 2 − pt )  9 
prior knowledge of GCD and LCM of numbers and expressions.
∴ p 2 + q 2 = 32 + 12 = 10
= x [ x ( x − 3) − 1 ( x − 3)]( q 2 − pt ) 64
67. (a) Let f ( x) = ax2 − ( a 2 + 1) x + a ⇒ 64 = xy ⇒ xy = 9
= x ( x − 3)( x − 1)( q 2 − pt ) 9
= a x2 − x − a 2 x + a

DIVISOR
A polynomial d ( x) is said to be a divisor of polynomial p( x) if d ( x) is a factor of p( x) i.e., p( x) can be
written as p( x) = d ( x) ⋅ q( x), where q( x) is a quotient polynomial e.g. ( x − 3) is a divisor of ( x − 3) 2 ( x + 1) .

HCF (GCD) of Polynomials


A polynomial h ( x) is called the HCF or GCD of two or more given polynomials, if h( x) is a
polynomial of highest degree dividing each one of the given polynomial without leaving any remainder.

Note The coefficient of highest degree term is always taken as positive.

EXAMPLE 1. The HCF of polynomials (x + 2) 2 (x − 3) 2 and (x + 1) (x + 2) 2 (x − 3) is


a. ( x + 2) ( x − 3) b. ( x + 2)2 ( x − 3) c. ( x + 1) ( x + 2) ( x − 3) d. ( x + 2)2 ( x − 3)2
Sol. b. Let p( x) = ( x + 2) 2 ( x − 3) 2 and q( x) = ( x + 1) ( x + 2) 2( x − 3)
The common factor between two polynomials is ( x + 2) 2 ( x − 3) , ∴ HCF = ( x + 2) 2 ( x − 3)

HCF by Factorisation Method


Following are the steps for calculating HCF through factorisation method
Step I Resolve the given polynomials in the complete factored form.
Step II Find the HCF of the numerical factors (if any) of given polynomial.
Step III Find the factors of highest degree which is common to all given polynomials.
Step IV The product of all such common factors and HCF of the numerical factors is the HCF of given
polynomials.
MATHEMATICS HCF and LCM of Polynomials 133 134 CDS Pathfinder

EXAMPLE 2. The HCF of p(x) = 24(6 x 4 − x 3 − 2x 2 ) and −39 is taken out as common factor from remainder. EXAMPLE 5. The LCM of x 3 − 2x 2 − x + 2 and EXAMPLE 6. The HCF of two polynomials is x + 3
q(x) = 20(2x 6 + 3x 5 + x 4 ) is x3 − 11x − 6) x4 − 39x − 22 (x x 3 − x 2 − 4x + 4 is and their LCM is x 3 − 7 x + 6. If one of the polynomials
x4 − 11x2 − 6x is x 2 + 2x − 3. Then, the other is
a. 4( 2x + 1) b. x2 ( 2x + 1) c. 4 x2 d. None of these a. ( x 2 − 1)( x 2 − 4) b. ( x 2 + 2)( x 2 − 4)
– + + a. x2 − x − 6 b. x2 − x + 6
Sol. d. Here, P( x) = 24 (6x − x − 2x ) = 2 ⋅ 3⋅ x ⋅ (6x − x − 2)
4 3 2 3 2 2 c. ( x 2 + 1)( x 2 + 2) d. None of these
11x2 − 33x − 22 c. x2 + x + 6 d. None of these
= 23 ⋅ 3⋅ x2 ⋅ (6x2 − 4x + 3x − 2) = 23 ⋅ 3⋅ x2( 2x + 1) ⋅ ( 3x − 2) Sol. a. Here, x − 2x − x + 2 = x ( x − 2) − 1( x − 2)
3 2 2
Again 11 is taken out as common factor from remainder Sol. d. Here, HCF = x + 3 and LCM = x3 − 7x + 6
and q( x) = 20 ( 2x6 + 3x5 + x4) = 22 ⋅ 5⋅ x4( 2x2 + 3x + 1) ⇒ ( x − 2)( x2 − 1) = ( x − 2)( x − 1)( x + 1)
x2 − 3x − 2) x3 − 11x − 6 (x + 3
= 22 ⋅ 5⋅ x4 ⋅ ( 2x2 + 2x + x + 1) = 22. 5. x4( 2x + 1)( x + 1) = ( x − 1)( x + 1)( x − 2) Also, one polynomial = x2 + 2x − 3
x3 − 3x2 − 2x
HCF of numerical factor = 22 = 4 , Highest degree common – + + and x3 − x2 − 4x + 4 = x2( x − 1) − 4( x − 1) As, LCM × HCF = Product of two polynomials
factor = x2 ( 2x + 1) 3x2 − 9x − 6 ⇒ ( x − 1)( x2 − 4) = ( x − 1)( x − 2)( x + 2) LCM × HCF
Other polynomial =
∴ Required HCF = 4x ( 2x + 1) 2
3x − 9x − 6
2
One polynomial
∴ Required LCM = ( x − 1)( x + 1)( x − 2)( x + 2)
− + +
HCF by Division Method = ( x2 − 1)( x2 − 22 ) ( x + 3) × ( x3 − 7x + 6)
× =
To find the HCF of polynomials which cannot be [Q ( a − b )( a + b) = a2 − b2] x2 + 2x − 3
∴ HCF of 11x − 39x − 8
4 3

factorised easily, we use successive division method. = ( x 2 − 1)( x 2 − 4 ) ( x + 3)( x3 − 7x + 6)


and x4 − 39x − 22 is x2 − 3x − 2 . =
Step I Arrange the given polynomials in descending order 2
Also, HCF of 2x and 2x is 2x. ( x + 3)( x − 1)
For any two polynomials p( x) and q( x)
of powers of its variables.
∴ Required HCF = 2x ( x2 − 3x − 2) p ( x) × q ( x) = (Their HCF) × (Their LCM) ( x + 3)( x − 1)( x2 + x − 6)
Step II If any common factor is present in the terms of i.e. Product of two polynomials
= = x2 + x − 6
( x + 3)( x − 1)
each polynomial, it should be taken out. LCM of Polynomials = Product of their HCF and LCM
Step III Divide the polynomial of highest degree by the So, other polynomial is x2 + x − 6.
A polynomial h( x) is called the LCM of two or more
polynomial of lowest degree. polynomials, if it is a polynomial of smallest degree
or If both the polynomials are of the same degree then which is divided by each one of the given polynomials
any one of them can be taken as divisor or dividend. without leaving any remainder.
Step IV After the first division take the remainder as the

Step V
new divisor and first divisor as new dividend.
Continue this process of dividing the last divisor
by the last remainder until the remainder becomes
EXAMPLE 4. The LCM of 12x 2 y 3 z 2 and
18x 4 y 2 z 3 is
a. x4 y 3 z3
PRACTICE EXERCISE
zero.
b. 9 x2 y 2 z3
Step VI The product of common factors obtained from step 1. The LCM of ( x − 1)( x − 2) and x 2( x − 2)( x + 3) is 6. What is the HCF of ( x4 − x2 − 6) and ( x4 − 4x2 + 3)?
c. 36 x3 y 3 z 4
II and last divisor is the HCF of given polynomials. (a) (x − 1) (b) (x − 1)(x − 2 )(x + 3) (a) x2 − 3 (b) x + 2 (c) x + 3 (d) x2 + 3
4 3 3
d. 36 x y z (c) x2 (x − 1)(x − 2 )(x + 3) (d) None of these
Note If the first term of a remainder is negative at any stage, the 7. The HCF of the polynomial A and B where
Sol. d. Here, 12x2y 3z 2 = 22 × 31 × x2 × y 3 × z 2
l

sign of all of its term must be changed. 2. The LCM of 2 ( a 2 − b2 ), 3 ( a3 − b3 ), 4 ( a 4 − b4 ) is A = ( x + 3)2( x − 2)( x + 1)2
If at any stage, the remainder contains common factor it 18x4 y 2z 3 = 21 × 32 × x4 × y 2 × z 3 (a) 6(a − b )(a + b )(a2 + b 2 ) (b) 12(a4 − b 4 )(a2 + ab + b 2 ) and B = ( x + 1)2( x + 3)( x + 4) is given by
l

should be taken out.


∴ Required LCM = 22 × 32 × x4 × y 3 × z 3 (c) a − b
3 3
(d) 12(a − b )
4 4
(a) (x + 1) 2 (x + 3) (b) (x + 1)(x + 3)2
EXAMPLE 3. The HCF of 22x 6 − 78x 5 − 16 x 2 and = 36x4 y 3z 3
3. The HCF of two expressions a and b is 1. Their (c) (x + 1)(x + 3) (d) (x + 3)2 (x + 1)2
2x 5 − 78x 2 − 44x is LCM is
a. ( x 2 − 3x − 2) b. 2x( x 2 − 3x − 2) LCM by Factorisation Method 1 8. The HCF of 22x( x + 1) and 36x 2( 2x 2 + 3x + 1) is
2
(a) (a + b ) (b) a − b (c) ab (d)
c. 22x ( x − 3x − 2)
2
d. None of these Following are the steps for calculating LCM through ab (a) 2 x(x + 1) (b) x(x + 1) (c) 2(x + 1) (d) 2 (x + 1)2
factorisation method 4. The LCM of the polynomials
Sol. b. Let p ( x) = 22x6 − 78x5 − 16x2 = 2x2(11x4 − 39x3 − 8) ( x + 3)2( x − 2)( x + 1)2; ( x + 1)3 ( x + 3)( x + 4) is 9. The LCM of a 2 − b2 − c2 − 2bc,
Step I Resolve the given polynomials in the b2 − c2 − a 2 − 2ac and c2 − a 2 − b2 − 2ab is
q ( x) = 2x5 − 78x2 − 44x = 2x ( x4 − 39x − 22) (a) (x − 2 )(x + 1)3 (x + 3)2 (x + 4)
complete factored form (a) (a + b + c )
Let us divide (11x − 39x − 8) by ( x − 39x − 22)
4 3 4 (b) (x − 2 )(x + 1)3 (x + 3)(x + 4) (b) (a − b − c ) (a + b + c )
Step II Find the LCM of the numerical factors
(c) (x − 2 )(x + 3)(x + 4) (c) (a + b + c ) (c − a − b )
x4 − 39x − 22) 11x4 − 39x3 − 8 (11 (if any) of given polynomials
(d) (x − 2 )2 (x + 1)(x + 3)2 (x + 4) (d) (a + b + c ) (a − b − c ) (b − c − a) (c − a − b )
11x4 − 242 − 429x Step III The required LCM is the product of LCM
– + + of numerical factors and each factor raised to 5. HCF of 4 y 4x − 9 y 2x3 and 4 y 2x 2 + 6 yx3 is 10. LCM of [( x + 3) ( x − 2)]2 and [( x − 2) ( x − 6)] is
− 39x3 + 429x + 234 the highest power. (a) y2 x (2 y + 3x) (b) yx (3x + 2 y) (a) (x + 3) (x − 2 )3 (x − 6) (b) (x + 3) (x − 2 )2 (x − 6)
(c) yx2 (x + 3) (d) None of these (c) (x + 3) (x − 2 ) (x − 6) (d) (x + 3) (x − 6)
MATHEMATICS HCF and LCM of Polynomials 135 136 CDS Pathfinder

11. If ( z − 1) is the HCF of ( z 2 − 1) and pz 2 − q( z + 1), 21. If f( x ) and g( x ) are two polynomials with integral PREVIOUS YEARS’ QUESTIONS 37. What is the HCF of 8 ( x5 − x3 + x ) and 28 ( x 6 + 1) ?
then coefficients which vanish at x = 1 / 2, then what is 30. What is the LCM of a3 b − ab3 , a3 b2 + a 2b3 and (a) 4(x4 − x2 + 1) (b) 2(x4 − x2 + 1) e 2014 I
(a) 2 p = q (b) p = 2q (c) 3 p = 2q (d) 3 p = 2 p the factor of HCF of f( x) and g( x) ? (c) (x4 − x2 + 1) (d) None of these
ab ( a + b) ? e 2012 I
12. The HCF of two expressions is 3x 2 + 4x − 4 and (a) x − 1 (b) x − 2 (c) 2 x − 1 (d) 2 x + 1 (a) a2 b 2 (a2 − b 2 ) (b) ab (a2 − b 2 )
38. What is the highest common factor of
their LCM is 3x 4 + 4x3 − 7x 2 − 4x + 4. The 22. The HCF of two polynomials p( x ) and q( x ) is (c) a2 b 2 + ab 3 (d) a3 b 3 (a2 − b 2 ) 2x3 + x 2 − x − 2 and 3x3 − 2x 2 + x − 2 ? e 2014 II
expressions are 2x ( x + 2) and LCM is 24x ( x + 2) ( x − 2). If 2
31. What is the HCF of 36 ( 3x4 + 5x3 − 2x2 ), (a) x − 1 (b) x + 1 (c) 2 x + 1 (d) 2 x − 1
(a) (x − 1)(3x2 + 4x − 4) and (3x2 + 4x − 4) p( x) = 8x3 + 32x2 + 32x, then what is q ( x ) equal to? 9 ( 6x3 + 4x2 − 2x) and 54 ( 27x4 − x) ? e 2012 I 39. The HCF and LCM of two polynomials are ( x + y )
(b) (x + 1)(3x + 4x − 4) and (x + 2 )(3x + 4x − 4)
2 2
(a) 4x3 − 16x (b) 6x3 − 24x (c) 12 x3 + 24x (d) 12 x3 − 24x (a) 9x (x + 1) (b) 9x (3x − 1) (c) 18x (3x − 1) (d) 18x (x + 1) and ( 3x5 + 5x 4 y + 2x3 y 2 − 3x 2 y3 − 5xy 4 − 2 y5 ),
(c) (x + 2 )(3x2 + 4x + 4) and (x − 1) 32. What is the HCF of the polynomials respectively. If one of the polynomials is ( x 2 − y 2 ),
23. If ( x − 2) is the HCF of ( ax 2 + bx + c) and
(d) (x + 1)(3x2 + 4x − 4) and (x − 1)(3x2 + 4x − 4) x3 + 8, x2 + 5x + 6 and x3 + 2x2 + 4x + 8 ? e 2013 II then the other polynomial is e 2015 I
( bx 2 + ax + c), then value of c is
(a) x + 2 (b) x + 3 (c) (x + 2 )2 (d) None of these (a) 3x4 − 8x3 y + 10x2 y2 + 7 x y3 − 2 y4
13. The LCM and HCF of two polynomials p ( x ) and (a) 2 (a + b ) (b) (a + b ) (c) −3 (a + b ) (d) − (a + b )
33. The LCM of ( x3 − x2 − 2x) and ( x3 + x2 ) is e 2013 II (b) 3x4 − 8x3 y − 10x2 y2 + 7 x y3 + 2 y4
q( x ) are 36x 2( x + a )( x3 − a3 ) and x 2( x − a ), 24. We have three polynomials A = 8 p + p2 + 12,
(a) x 3 − x 2 − 2 x (b) x 2 + x (c) x 4 − x 3 − 2 x 2 (d) x − 2 (c) 3x4 + 8x3 y + 10x2 y2 + 7 x y3 + 2 y4
respectively. If p ( x ) = ( 4x 2 )( x 2 − a 2 ), then q( x ) B = p2 + 2 p − 24 and C = p2 + 15 p + 54
(d) 3x4 + 8x3 y − 10x2 y2 + 7 x y3 + 2 y4
(a) 4x3 (x3 − a3 ) (b) 12 x3 (x2 − a2 )
I. Their LCM is ( p + 6) ( p − 4) (p + 2) ( p + 9) 34. The HCF of ( x4 − y 4 ) and ( x6 − y 6 ) is e 2013 II
(c) 9x3 (x3 − a3 ) (d) 36x3 (x3 − a3 ) (a) x 2 − y 2 (b) x − y (c) x 3 − y 3 (d) x4 − y4 40. If ( x + 1) is the HCF of Ax 2 + Bx + C and
II. Their HCF is ( p + 6)( p − 2)
35. What is the LCM of x + 2x − 8, x − 4x 2 + 4x and
2 3 Bx 2 + Ax + C where A ≠ B, then the value of C is
14. The LCM of two polynomials p ( x ) and q ( x ) is Then, which of the following codes is/are correct
x3 − 7x + 6. If p ( x ) = x 2 + 2x − 3 and (a) Only I (b) Only II x 2 + 4x ? e 2013 II e 2015 II
(a) x(x + 4)(x − 2 ) 2
(b) x(x + 4)(x − 2 ) (a) A (b) B (c) A − B (d) 0
q( x ) = x 2 + x − 6, then the HCF is (c) Neither I nor II (d) Both I and II
(c) x(x + 4)(x + 2 )2 (d) x(x + 4)2 (x − 2 ) 41. The sum and difference of two expressions are
(a) (x + 3) (b) (x − 3) 25. Which of the following statements are true?
36. What is the HCF of a 2b4 + 2a 2b2 and ( ab)7 − 4a 2b9 ? 5x 2 − x − 4 and x 2 + 9x − 10 respectively. The HCF
(c) (x + 3)(x − 2 ) (d) (x − 1) I. HCF of x2 − 6x + 9 and x3 − 27 is (x − 3). of the two expressions will be e 2016 I
e 2013 II
15. If ( x + k) is the HCF of ( x + ax + b) and 2 II. LCM of 10x2yz, 15xyz , 20xy2z 2 is 120x2y2z 2. (a) x + 1 (b) (x − 1) (c) (3x + 7 ) (d) (2 x − 3)
(a) ab (b) a2 b 3 (c) a2 b 2 (d) a3 b 2
( x 2 + px + q ), then the value of k is III. HCF of (6x2 − 7x − 3) and (2x2 + 11x − 21) is
(2x − 3).
b + q b −q  a + b  a − b
(a)  
 a + p
(b)  
 a − p
(c) 
 p+ q
 (d) 
 p−q
 Select the correct answer using the codes given below ANSWERS
(a) I and III (b) I, II and III
(c) II and III (d) None of these 1 c 2 b 3 c 4 a 5 b 6 a 7 a 8 a 9 d 10 b
16. What is the value of k for which the HCF of
11 b 12 d 13 c 14 a 15 b 16 a 17 b 18 c 19 d 20 a
2x 2 + kx − 12 and x 2 + x − 2k − 2 is ( x + 4)? 26. Consider the following statements :
21 c 22 b 23 c 24 a 25 a 26 c 27 c 28 b 29 c 30 a
(a) 5 (b) 7 (c) 10 (d) −4 I. The HCF of x+ y and x10 − y10 is x + y.
II. The HCF of x+ y and x10 + y10 is x + y. 31 c 32 a 33 c 34 a 35 a 36 c 37 a 38 a 39 c 40 d
17. If the HCF of ( x 2 + x − 12) and ( 2x 2 − kx − 9) is 41 b
III. The HCF of x− y and x10 + y10 is x − y.
( x − k), then what is the value of k?
(a) − 3 (b) 3 (c) − 4 (d) 4 IV. The HCF of x− y and x10 − y10 is x − y.
Which of the statement(s) given above is/are
18. If GCD of the polynomials ( x3 − 2x 2 + px + 6) and
( x 2 − 5x + q ) is ( x − 3). Then, the value of 5q + 6 p is
correct?
(a) I and II (b) II and III (c) I and IV (d) II and IV
HINTS AND SOLUTIONS
(a) −1 (b) 1
1. (c) LCM = product of the largest power Product of expressions 6. (a) Let p( x ) = x 4 − x 2 − 6
(c) 0 (d) None of these Directions (Q. Nos. 27-29) A student wrote five 3. (c) LCM =
polynomials such as A = pq − np, B = pq − mq, of each factor HCF
= x 4 − 3x 2 + 2x 2 − 6
19. The sum and the difference of two expressions is = x2 ( x − 1)( x − 2)( x + 3) a×b
C = q 2 − 3nq + 2n2, D = pq − 2 pn − mq + 2mn, = = ab = x 2 ( x 2 − 3) + 2( x 2 − 3)
5x − x − 4 and x + 9x − 10 respectively, then
2 2
1
E = pq − np − mq + mn. Now, he divide the 2.(b) Here, 2 ( a 2 − b 2 ) = 2 ( a + b ) ( a − b ) = ( x 2 + 2)( x 2 − 3)
their LCM would be equal to 4. (a) Given, ( x + 3)2 ( x − 2)( x + 1)2
polynomials into groups and calculate the HCF and 3 ( a 3 − b 3 ) = 3 ( a − b ) ( a 2 + ab + b 2 )
(a) (x − 1) (b) (2 x + 3)(3x + 7 ) q( x ) = x 4 − 4x 2 + 3
LCM. and 4 ( a 4 − b 4 ) = 4 ( a + b ) ( a − b ) and ( x + 1) 3 ( x + 3)( x + 4)
(c) (2 x − 3)(3x + 7 ) (d) (x − 1)(2 x − 3)(3x + 7 ) = x 4 − 3x 2 − x 2 + 3
27. Calculate the HCF of A, C and E ( a2 + b2 ) LCM
20. Find the values of a and b so that the polynomials = ( x − 2)( x + 1) 3 ( x + 3)2 ( x + 4) = x 2 ( x 2 − 3) − 1( x 2 − 3)
(a) (q − 2 n) (b) ( p − n) (c) (q − n) (d) (q − n)(q − 2 n) LCM of numerical coefficients = 12
p ( x ) and q( x ) have ( x + 1)( x + 3) as their HCF and LCM of algebraic expressions = ( x 2 − 3)( x 2 − 1)
5. (b) 4 y 4 x − 9 y 2 x3 = y 2 x( 4 y 2 − 9x2 )
28. Calculate the LCM of D and E = ( a − b )( a + b )( a 2 + b 2 ) HCF of p( x ), q( x ) = x 2 − 3
p ( x ) = ( x 2 + 3x + 2)( x 2 + 2x + a ) (a) ( p − m)(q − 2 n) (b) ( p − m)(q − n)(q − 2 n) = y 2 x( 2 y − 3x)( 2 y + 3x)
( a 2 + ab + b 2 ) 7. (a) A = ( x + 3)2 ( x − 2)( x + 1)2 and
and q( x ) = ( x 2 + 7x + 12)( x 2 + 7x + b) (c) (q − n)(q − 2 n)(m − p) (d) (q − n)( p − m) 4 y 2 x2 + 6 yx3 = 2 yx2 ( 2 y + 3x)
= ( a 4 − b 4 )( a 2 + ab + b 2 ) B = ( x + 1)2 ( x + 3)( x + 4)
(a) −3, 6 (b) 3, − 6 29. The HCF of all five polynomials together is ∴ Required HCF = xy( 2 y + 3x)
∴ LCM of polynomials ∴ HCF of polynomials
(c) 6, − 3 (d) None of these (a) (q − n) (b) (q − 2 n) (c) 1 (d) ( p − m) = 12( a 4 − b 4 )( a 2 + ab + b 2 ) = ( x + 3)( x + 1)2
MATHEMATICS HCF and LCM of Polynomials 137 138 CDS Pathfinder

8. (a) Here, HCF of 22 and 36 is 2. 14. (a) Given, Subtracting Eqs. (i) and (ii), we have 25. (a) I. x2 − 6x + 9 = ( x − 3)( x − 3) 31. (c) Let P (x) = 36(3x4 + 5x3 − 2x2 ) Now, LCM of ( x2 + 2x − 8),
Now, x ( x + 1)2 = x ( x + 1) ( x + 1) p( x) = x2 + 2x − 3 = ( x + 3)( x − 1) 2q( x) = 4x2 − 10x + 6 and x3 − 27 = x3 − (3)3 = 36x2 (3x2 + 5x − 2) ( x3 − 4x2 + 4x) and ( x2 + 4x)
x 2 ( 2x 2 + 3x + 1) = x 2 ( 2x + 1) ( x + 1) q( x) = x2 + x − 6 = ( x + 3)( x − 2) ⇒ q( x) = 2x2 − 5x + 3 = ( x − 3)( x2 + 3x + 9)
= 36x 2 ( x + 2) (3x − 1) = x ( x − 2)2 ( x + 4)
∴ HCF = x − 3.
Common factors of x ( x + 1)2 and LCM = 2x 2 − 3x − 2x + 3 Q (x) = 9( 6x3 + 4x2 − 2x) 36. (c) a 2 b 4 + 2a 2 b 2 = a 2 b 2 ( b 2 + 2) ...(i)
Hence, it is true.
and x 2 ( 2x 2 + 3x + 1) are x ( x + 1). = x3 − 7x + 6 = ( x − 1)( x2 + x − 6) = x( 2x − 3) − 1( 2x − 3)
II. LCM of 10x2 yz , 15xyz = 18x(3x2 + 2x − 1) and ( ab )7 − 4a 2 b 9 = a 7 b 7 − 4a 2 b 9
Hence, required HCF = 2x ( x + 1) = ( x − 1)( x + 3)( x − 2) = ( x − 1)( 2x − 3)
p ( x) × q ( x) and 20xy 2z 2 is 60x2 y 2z 2 . = 18 x(3x − 1)( x + 1) = a 2 b 2 ( a 5 b 5 − 4b 7 ) ...(ii)
∴ HCF = ∴ LCM of p( x) and q( x)
9. (d) a 2 − b 2 − c 2 − 2bc = a 2 − ( b 2 + c 2 + 2bc ) Hence, it is false. R (x) = 54( 27x4 − x) = 54x( 27x3 − 1)
LCM = ( x − 1)( 2x − 3)(3x + 7) From Eqs. (i) and (ii), HCF = a 2 b 2
= a 2 − ( b + c )2 = ( a + b + c ) ( a − b − c ) ( x + 3)( x − 1) × ( x + 3)( x − 2) III. 6x2 − 7x − 3 = ( 2x − 3)(3x + 1) = 54x (3x − 1)( 9x + 1 + 3x )
2
= 20. (a) p ( x) = ( x + 2)( x + 1)( x2 + 2x + a ) 37. (a) Let p( x) = 8( x5 − x3 + x)
b 2 − c 2 − a 2 − 2ac = b 2 − ( c 2 + a 2 + 2ac ) ( x − 1)( x + 3)( x − 2) and 2x2 + 11x − 21 HCF of [36, 18, 54] = 18
q ( x) = ( x + 3)( x + 4)( x2 + 7x + b ) = ( x + 7)( 2x − 3) = 4 × 2 × x( x4 − x2 + 1)
= b 2 − ( c + a )2 ∴ HCF of [P (x ), Q (x ), R (x )]
= ( x + 3) As HCF is ( x + 1)( x + 3), then both
= ( b − c − a) ( b + c + a) Hence, HCF = ( 2x − 3), it is also and q( x) = 28( x6 + 1)
15. (b) Since, ( x + k ) is the HCF, it will ( x + 1) and ( x + 3) must be factors of true. = 18x (3x − 1)
= ( a + b + c) ( b − c − a ) = 7 × 4[( x2 )3 + ( 1) 3 ]
divide both the polynomials without p( x) and q( x). Hence, the statement I and III are 32. (a) Let f ( x) = x3 + 8 = x3 + 23 = 4 × 7 × ( x2 + 1)( x4 − x2 + 1)
and c 2 − a 2 − b 2 − 2ab leaving any remainder, thus x = − k will For p(x), ( x + 1) is already a factor, so correct.
= c 2 − ( a 2 + b 2 + 2ab ) = c 2 − ( a + b )2 make both of them zero. = ( x + 2) ( x 2 − 2x + 4) ∴ HCF of p( x) and q( x)
( x + 3) must be a factor of x2 + 2x + a. 26. (c) We know that,( x + y ) and ( x − y ) are
= ( c − a − b) ( c + a + b) ∴ k 2 − pk + q = k 2 − ak + b = ( x + 2) ( x − 2) ( x − 2) = 4( x4 − x2 + 1)
So, ( −3)2 + 2( −3) + a = 0 the factors of ( x 10 − y 10 ).
= ( a + b + c) ( c − a − b ) or − ak + b = − pk + q g( x) = x2 + 5x + 6
⇒ 9−6+ a=0 Hence, statements I and IV are true. 38. (a) Let f ( x) = 2x3 + x2 − x − 2
∴ Required LCM ⇒ ak − pk = b − q = x + 3x + 2x + 6
2

= ( a + b + c) ( a − b − c) b−q ∴ a = −3 27. (c) We have, A = pq − np = p ( q − n) …(i) = ( x − 1)( 2x2 + 3x + 2)


∴ k= = ( x + 3) ( x + 2)
( b − c − a) ( c − a − b) a− p For q (x), ( x + 3) is already factor. C = q 2 − 3nq + 2n2 and g( x) = 3x3 − 2x2 + x − 2
∴( x + 1) must be a factor of = q 2 − 2nq − nq + 2n2 and h( x) = x3 + 2x2 + 4x + 8
10. (b) Given, [( x + 3) ( x − 2)2 ] 16. (a) Since, ( x + 4) is HCF, so it will = ( x − 1)(3x2 + x + 2)
x2 + 7x + b. = q ( q − 2n) − n( q − 2n) = ( x + 2) ( x 2 + 4)
and [( x − 2) ( x − 6)] divide both the expressions i.e. x = − 4 Hence, the highest factor of f ( x) and
will make each one zero. ∴( −1)2 + 7( −1) + b = 0 = ( q − n)( q − 2n) …(ii) ∴ HCF of { f ( x), g ( x), h ( x)} = ( x + 2) g( x) is ( x − 1).
∴ LCM = ( x + 3) ( x − 2)2 ( x − 6)
∴ 2( −4)2 + k( −4) − 12 = 0 ∴ b=6 E = pq − np − mq + mn 33. (c) Let f (x) = x3 − x2 − 2x 39. (c) Given, HCF = ( x + y ) and
11. (b) Since, (z − 1) is the HCF, so it will
divide each one of the given polynomials. ⇒ 32 − 12 = 4k So, a = − 3 and b = 6 is solution. = p( q − n) − m( q − n) = x ( x 2 − x − 2) LCM = 3x5 + 5x4 y + 2x3 y 2
So, z = 1 will make each one zero. ∴ 20 = 4k ⇒ k =5 1 = ( p − m )( q − n) …(iii) = x ( x + 1) ( x − 2)
21. (c) Given, f ( x) and g( x) vanish at x = − 3 x 2 y 3 − 5 xy 4 − 2 y 5
∴ p ( 1)2 − q ( 1 + 1) = 0 ⇒ p = 2q 17. (b) Since, HCF of x2 + x − 12 and 2 ∴ HCF of A, C and E = ( q − n)
and g (x ) = x3 + x 2 = x 3 (3x 2 + 5xy + 2 y 2 ) − y 3
12. (d) Let p( x ) and q( x ) be two polynomials 2x2 − kx − 9 is ( x − k ), then ( x − k ) will So, ( 2x − 1) is a factor of f ( x) and g( x) 28. (b) We have, E = ( p − m )( q − n) ... (i)
= x2 ( x + 1) = x ⋅ x ( x + 1) (3x 2 + 5xy + 2 y 2 )
and p( x ) ÷ HCF = a be the factor of 2x2 − kx − 9. both. D = pq − 2np − mq + 2mn
∴ 2k 2 − k 2 − 9 = 0 ∴ LCM of [ f ( x), g( x)] = (3x2 + 5xy + 2 y 2 )( x3 − y 3 )
and q( x ) ÷ HCF = b Hence, HCF of f ( x) and = p( q − 2n) − m ( q − 2n)
⇒ k2 − 9 = 0 g( x) = 2x − 1 . = ( p − m )( q − 2n) …(ii) = x2 ( x + 1) ( x − 2) We know that,
∴ p( x ) × q( x ) = LCM × HCF
⇒ k = ±3 ∴ LCM of D and E = x 2 ( x 2 − x − 2) Product of two polynomials
⇒ a × HCF × b × HCF = LCM × HCF 22. (b) Refer to question 13.
and factor of x2 + x − 12 are = ( p − m )( q − n)( q − 2n) = HCF × LCM
⇒ a × b × HCF = LCM 23. (c) As ( x − 2) is the HCF of = x 4 − x 3 − 2x 2
( x + 4)( x − 3). ∴ Required polynomial
LCM ( ax 2 + bx + c ) and ( bx 2 + ax + c ) 29. (c) The factors of given polynomials are
ab = Hence, value of k is 3. 34. (a) Let f ( x) = ( x 4 − y 4 )
as follows ( x + y )( x3 − y 3 )(3x2 + 5xy + 2 y 2 )
HCF So, it will divide both the expressions,
= ( x2 − y 2 ) ( x2 + y 2 ) =
3x 4 + 4x 3 − 7x 2 − 4x + 4 18. (c) Here, ( x − 3) is GCD, so is a factor of ( x − y )( x + y )
∴ a ( 2)2 + b( 2) + c = 0 A = pq − np = p ( q − n) …(i)
= both of them. = (x − y) (x + y) (x + y ) 2 2
3x + 4x − 4
2
⇒ 4a + 2b + c = 0 …(i) B = pq − mq = q ( p − m ) ( x − y ) ( x 2 + y 2 + xy )
∴ Putting x = 3, in both makes each …(ii) and g( x) = ( x6 − y 6 )
= ( x + 1)( x − 1) polynomial zero. and b ( 2)2 + a ( 2) + c = 0 (3x 2 + 5xy + 2 y 2 )
C = ( q − n)( q − 2n) …(iii) = (x + y ) (x − y )
3 3 3 3
Let a = ( x + 1) and b = ( x − 1), 3 3 − 2(3)2 + p(3) + 6 = 0 ⇒ p = −5 =
⇒ 4b + 2a + c = 0 …(ii) (x − y)
D = ( p − m )( q − 2n) …(iv) = ( x + y ) ( x2 − x y + y 2 ) ( x − y )
then the required expression are 3 − 5(3) + q = 0 ⇒ q = 6
2
adding Eqs. (i) and (ii), we get
( x + 1) (3x2 + 4x − 4) ∴ 5 q + 6 p = 5 ( 6) + ( −5)( 6) E = ( p − m )( q − n) …(v) ( x2 + x y + y 2 ) = ( x2 + y 2 + xy )(3x2 + 5xy + 2 y 2 )
⇒ 6a + 6b + 2c = 0
and ( x − 1) (3x2 + 4x − 4). = 30 − 30 = 0 There is no such factor which is = ( x − y ) ( x + y ) ( x2 − x y + y 2 ) = 3x4 + 8x3 y + 10x2 y 2 + 7xy 3 + 2 y 4
⇒ 2 c = −6 a − 6 b
13. (c) Here, LCM = 36x3 ( x + a )( x3 − a 3 ) common to all given five polynomials. 40. (d) ( x + 1) is the HCF of
19. (d) Let the expressions be p( x) and q( x), ⇒ c = −3 ( a + b ) ( x2 + x y + y 2 )
and HCF = x2 ( x − a ) Thus, HCF (A, B, C , D, E) = 1
then Ax 2 + Bx + C and Bx 2 + Ax + C
24. (a) A = p 2 + 8 p + 12 = ( p + 2)( p + 6) ∴ HCF of
p( x) = 4x ( x − a )
2 2 2 p ( x) + q ( x) = 5x2 − x − 4 …(i) 30. (a) Here, a 3 b − ab 3 = ab ( a 2 − b 2 ) ∴ A( −1)2 + B( − 1) + C = 0
B = p 2 + 2 p − 24 = ( p − 4)( p + 6) [ f ( x), g( x)] = ( x − y ) ( x + y )
But p( x) × q ( x) = HCF × LCM p ( x) − q( x) = x2 + 9x − 10 …(ii) = ab ( a − b ) ( a + b ) ⇒ A−B+C =0
HCF × LCM C = p + 15 p + 54 = ( p + 9)( p + 6)
2 =x − y
2 2
⇒ C =B− A
q ( x) = On adding Eqs. (i) and (ii), we have a3b2 + a2b3 = a2b2 ( a + b)
35. (a) x 2 + 2 x − 8 = ( x − 2) ( x + 4) and B( − 1)2 + A ( −1) + C = 0
p ( x) 2 p( x) = 6x2 + 8x − 14 I. LCM of A, B ∴ LCM [( a 3 b − ab 3 ), ( a 3 b 2 + a 2 b 3 ), ⇒ B− A+C =0
x2 ( x − a ) 36x3 ( x + a )( x3 − a 3 ) ⇒ p( x) = 3x2 + 4x − 7 and C = ( p + 2)( p + 6)( p − 4)( p + 9) ab ( a + b )] x 3 − 4x 2 + 4x = x [ x 2 − 4x + 4] ⇒ C = A−B
=
4x 2 ( x 2 − a 2 ) = 3x 2 + 7x − 3x − 7 Thus, I is correct = a2b2 ( a + b) ( a − b) = x ( x − 2)2 ∴ C =0
= 9x 3 ( x 3 − a 3 ) = x(3x + 7) − 1(3x + 7) II. HCF of A, B and C = ( p + 6) = a2b2 ( a2 − b2 ) x 2 + 4x = x ( x + 4) 41. (b) Refer to question 19.
∴ p( x) = (3x + 7)( x − 1) Hence, II is incorrect.
16
MATHEMATICS Rational Expressions 139 140 CDS Pathfinder

p ( x)
Operations on Rational 3. Multiplication of Rational Expressions If
q ( x)
and
Expressions g ( x)
are two rational expressions, then their product
1. Addition or Subtraction of Rational Expressions h ( x)
with like denominators p ( x) g ( x) p ( x) ⋅ g ( x)
is given by × =
P ( x) h ( x) q ( x) h ( x) q ( x) ⋅ h ( x)
It and are two rational expressions then
q ( x) q ( x) p ( x) q ( x)
• Multiplicative inverse of is .
p ( x) h ( x) p ( x) ± h ( x) q ( x) p ( x)

RATIONAL EXPRESSIONS ±
q ( x) q ( x)
=
q ( x)
2. Addition or subtraction of Rational Expressions
with unlike denominators
• 1 is the multiplicative identity.

EXAMPLE 6. The product of


x2 −1
and
x +2
is
To add or subtract rational expressions with unlike x2 +1 x +1
denominators, follow the steps given below ( x − 1) ( x − 2)
a. b.
Step I Write each denominator in the factor form ( x2 + 1) ( x2 + 1)
Usually (1-2) questions have been asked from this chapter. Generally questions are asked from this Step II Find the LCM of the denominators ( x − 1)( x + 2) x+ 2
chapter are based on simplification of rational expressions. c. d.
Step III Rewrite each rational expression with LCM as ( x2 + 1) ( x2 − 1)
the denominator
x2 − 1 ( x + 2) ( x − 1) ( x + 1) ( x + 2)
Step IV Add or subtract the numerators. Sol. c. Here, product = × =
x2 + 1 ( x + 1) ( x2 + 1) ( x + 1)

EXAMPLE 1. The lowest term of an expression x +1 x −1 [Q a2 − b2 = ( a − b)( a + b)]


RATIONAL EXPRESSIONS a3 −b3
EXAMPLE 4. The sum of
x −1
and
x +1
is
=
( x − 1) ( x + 2)
p ( x) is ( x2 + 1)
An expression in the form of , where p ( x) and q ( x) a 2 + ab + b 2 x +1
2
1 2x + 2
2
x +2
2
q ( x) a a. b. c. d. p ( x) g ( x)
a. a + b b. a − b c. ab d. x2 − 1 x +1
2
x2 − 1 x2 − 1 4. Division of Rational Expressions If and
are polynomials and q ( x) ≠ 0 is called a rational b q ( x) h ( x)
expression. Sol. b. Rational expression x + 1 x − 1 ( x + 1) ( x + 1) ( x − 1) ( x − 1)
Sol. c. + = × + × are two rational expressions, then their division is
p ( x) a3 − b 3 ( a − b) ( a2 + ab + b2) x − 1 x + 1 ( x − 1) ( x + 1) ( x + 1) ( x − 1)
• In the rational expression , p ( x) is called the = = =a−b p ( x) g ( x) p ( x) h ( x)
q ( x) a2 + ab + b2 ( a2 + ab + b2) ( x + 1) 2 + ( x − 1) 2
given by ÷ = ×
= q ( x) h ( x) q ( x) g ( x)
numerator and q ( x) is called the denominator of the [Q( a3 − b3) = ( a − b)( a2 + ab + b2)] ( x2 − 1)
p ( x) g ( x)
rational expression. = Product of and the reciprocal of
EXAMPLE 2. The lowest term of an expression x + 1+ 2x + x2 + 1− 2x 2x + 2
2 2
= = 2 q ( x) h ( x)
• Every polynomial is a rational expression. Since, p ( x) 3x 2 − 11x − 4 x2 − 1 x −1 i.e.
h ( x)
.
p ( x) is g ( x)
can always be written as and a constant function 1 6x 2 − 7x − 3 [Q ( a + b) 2 = a2 + b2 + 2ab and ( a − b) 2 = a2 + b2 − 2ab]
1
x+ 4 x+ 4 x−4 x−4 EXAMPLE 7. The lowest term of an expression
is a polynomial of degree 0. a. b. c. d. EXAMPLE 5. What should be subtracted from
2x + 3 2x − 3 2x + 3 2x − 3 7x 4 x 2 + 8x + 12 x 2 + 4x − 12
• Every rational expression need not be a polynomial. to get ? ÷ is
3x − 11x − 4
2
3x − 12x + x − 4
2
x 2 + x − 12 x+4 x 2 − 7 x + 12 x −4
Sol. d. =
Working Rule to Reduce the Given 6x2 − 7x − 3 6x2 − 9x + 2x − 3 1 3 1 3 ( x − 2) x−2
a. b. c. d. a. b.
3x ( x − 4) + 1( x − 4) ( 3x + 1) ( x − 4) x−4 x−3 x−3 x+ 3 x+ 3 ( x − 3)( x + 2) x+ 2
Rational Expression in its Lowest Term = = =
3x ( 2x − 3) + 1( 2x − 3) ( 3x + 1) ( 2x − 3) 2x − 3 ( x + 2) x+ 3
1. Firstly, factorize both the polynomials p ( x) and q ( x). Sol. b. Let p( x) is to be subtracted, then c. d.
7x 4 ( x − 3)( x − 2) x−2
EXAMPLE 3. The lowest term of an expression − p( x) =
2. Find the HCF of p ( x) and q ( x). If HCF of p ( x) and 12x 3 y 5 z 4 x2 + x − 12 x+ 4 x2 + 8x + 12 x2 + 4x − 12
p ( x) is 7x 4 7x 4 Sol. c. Here, ÷
q ( x) is one, then rational expression is in its 18x 2 y 6 z 5 ⇒ p( x) = 2 − = − x2 − 7x + 12 x−4
q ( x) x + x − 12 x + 4 ( x + 4) ( x − 3) ( x + 4)
2x xy 2x 3x x2 + 6x + 2x + 12 x−4
lowest terms. a. b. c. d. 7x 4( x − 3) 7x − 4 ( x − 3) = ×
yz z 3yz 2yz = − = x2 − 4x − 3x + 12 ( x2 + 6x − 2x − 12)
3. If HCF is not equal to 1. Then, divide both p ( x) and ( x + 4)( x − 3) ( x + 4)( x − 3) ( x + 4) ( x − 3)
12x y z 3 5 4
6 × 2× x × x × y × z
2 5
2x 4
( x + 6) ( x + 2) x−4 x+ 2
q ( x) by their HCF and the rational expression is Sol. c. = = 3x + 12 3 ( x + 4) 3 = × =
18x2y 6z 5 6 × 3 × x2 × y × y 5 × z × z 4 3yz = = = ( x − 4) ( x − 3) ( x + 6) ( x − 2) ( x − 3) ( x − 2)
obtained in the lowest terms. ( x + 4) ( x − 3) ( x + 4) ( x − 3) x − 3
MATHEMATICS Rational Expressions 141 142 CDS Pathfinder

PRACTICE EXERCISE 15. If


1
+
2
+
1009
x + 1 y + 2 z + 1009
x y
= 1, then what is the
z 18. What is
PREVIOUS YEARS’ QUESTIONS
( x 2 + y 2 )( x − y ) − ( x − y )3
equal to?
value of + + ?
x + 1 y + 2 z + 1009 x 2 y − xy 2 e 2013 II
(a) 0 (b) 2 (c) 3 (d) 4 (a) 1 (b) 2 (c) 4 (d) − 2
1. Which of the following are rational expressions?  x − 1  x + 1
8. If A =   and B =  x − 1 , then ( A + B ) is
2
1 1 2b 4b3 8b7
x3 − 3x2 + 2 z3 − 3z 2  x + 1   16. What is
x 2 − 3x + 2
÷
x 2 − 5x + 4 19. What is − − − −
I. II.
2z + 3
equal to? a − b a + b a 2 + b2 a 4 + b4 a 8 − b8
x2 + 1 4x + 8x − 4
4 2
4x + 8x + 4
4 2
x − 5x + 6
2
x − 7x + 12
2
(a) (b) x +3 x+ 1 equal to? e 2014 II
x2 − x + 2 x3 + 3x2 − 1 x4 − 2 x2 + 1 x4 − 2 x2 + 1 (a) (b) 1 (c) (d) 2
III. IV. x−3 x+ 3 (a) a + b (b) a − b (c) 1 (d) 0
x+3 x2 + x − 1 4x + 8x + 4
4 2
(c) (d) None of these
Select the correct answer using the codes given below x + 2x + 1
4
17. If x + y + z = 0, then what is
xyz 20. If a 2 − by − cz = 0, ax − b2 + cz = 0 and
( x + y )( y + z )( z + x ) ax + by − c2 = 0, then the value of
(a) I, II and III (b) II, III and IV
9. The value of x y z
(c) All of these (d) None of these 2 2
equal to (where, x ≠ − y , y ≠ − z , z ≠ − x ) ? + + will be
a + 2 ( a + 1)
1
+
a

b
is (a) −1 (b) 1 a+ x b+ y c+ z e 2016 I
2. The simplified form of − . (1 − a ) (1 − b) (1 − a ) ( b − a ) ( b − 1) ( a − b) (c) x y + yz + zx (d) None of these (a) a + b + c (b) 3
a + 3 ( a + 2) (c) 1 (d) 0
1 a+2 (a) 1 + a (b) 1 − a2
(a) (b)
a2 + 5a + 6 a2 + 5a + 6 (c) 1 − b 2 (d) None of these ANSWERS
(a + 2 ) 2
(c) 0 (d)  x y  y z   z x
a + 5a + 6
2
 −  −  −  1 a 2 a 3 b 4 c 5 b 6 b 7 b 8 b 9 d 10 b
10. The value of  y x   z y  x z  is
 x + 1 2 11 a 12 b 13 d 14 c 15 b 16 b 17 a 18 b 19 d 20 c
1 11 11 1
3.  2 −  expressed as a rational expression is  2 − 2  2 − 2  2 − 2
 x − 1 x x y  y z  z x 
x2 − 2 − (x − 2 ) (a) x2 y2 z2 (b) − x2 y2 z2
(a)
x (x2 − 1)
2 (x + 1)
(b)
x (x − 1)
1
(c) 1 (d) None of these HINTS AND SOLUTIONS
(c) (d) 11. What is the simplified form of a−c b−c −a
(x3 − 1) x (x + 1) x 3 + 3x 2 − 1 5. (b) Here, + = −
b
 x 2 − 3x + 2  x 2 − 9   x3 + 2x 2 + 4x 
1. (a) Here,
x2 + x − 1
is not a rational
( a − b )( x − a ) ( b − a )( x − b ) ( a − b )( c − a ) ( b − c )( a − b )
( x − 1) ( x − 2) ( x 2 − 9x + 14)   ÷   ×   ? c
4. The expression in  x3 − 8   x 2 + 7x + 12  x 2 + 3x − 4  expression, since the denominator is not ( a − c )( x − b ) − ( b − c )( x − a ) −
= ( c − a )( b − c )
( x − 7) ( x 2 − 3x + 2) x x −2 a polynomial. ( a − b )( x − a )( x − b )
(a) (b)
a + 2 ( a + 1) ( − a )( b − c ) − b( c − a ) − c( a − b )
lowest terms is x−3 x−3 − ax − ab − xc + bc − ( bx − ab − cx + ac ) =
2. (a) We know = ( a − b )( b − c )( c − a )
(a)
1
(b) (x − 7 ) (c) (x − 2 ) (d)
1 x x+ 3 a + 3 ( a + 2) ( a − b )( x − a )( x − b )
(c) (d) − ab + ac − bc + ab − ac + bc
x−7 x−2 x+ 3 x+ 4 ( a + 2) − ( a + 1)( a + 3)
2
=
= ax + bc − bx − ac
( a + 3)( a + 2) = ( a − b )( b − c )( c − a )
a−c b− c 12. If pq + qr + rp = 0, then what is the value of ( a − b )( x − a )( x − b )
5. + is equal to =
0
=0
( a − b) ( x − a ) ( b − a ) ( x − b) p2 q2 r2 a + 4 + 4a − ( a + 4a + 3)
2 2
x( a − b ) − c ( − b + a )
= = ( a − b )( b − c )( c − a )
b−a x−c + + ? a 2 + 5a + 6 ( a − b )( x − a )( x − b )
(a) (b) p2 − qr q 2 − rp r 2 − pq 1 8. (b) Here,
( x − b ) ( x − a) ( x − a) ( x − b ) = ( x − c )( a − b ) ( x − c)
a + 5a + 6
2 = = x − 1   x + 1 
2
( A + B )2 =  
(a) 0 (b) 1
b−a ( a − b )( x − a )( x − b ) ( x − a )( x − b ) + 
(c) (d) None of these (c) −1 (d) 3 x + 1 2   x + 1  x − 1  
3. (b) Here,  2
(x − b) (x − c ) 1 2
−  = − x −3 x +1 2
13. If x + y + z = 0, then what is the value of  x − 1 x x − 1 x 6. (b) Here, reciprocal of is
2
 (x − 1)2 + (x + 1)2   2 (x2 + 1) 
2
x−3 x 2 + 1 ( x − 3) =  = 2 
6. The sum of the rational expression and its 1 1 1 [Q x2 − 1 = ( x + 1)( x − 1)]  (x + 1) (x − 1)   x − 1 
x +1
2
+ + ? So,
x2 + y2 − z 2 y2 + z 2 − x2 z 2 + x2 − y2 x − 2x + 2 −( x − 2)
reciprocal is = = x −3 x2 + 1 ( x − 3)2 + ( x2 + 1)2 [Q ( a + b )2 + ( a − b )2 = 2( a 2 + b 2 )]
x ( x − 1) x ( x − 1) + =
x − 3x + x − 3
3 2
x + 3x − 6x + 10
4 2 x2 + 1 x −3 ( x2 + 1)( x − 3) 4( x 4 + 2x 2 + 1) 4x 4 + 8x 2 + 4
(a) (b) = = 4
1 4. (c) We know that, x + 9 − 6x + x 4 + 1 + 2x 2
2
x 4 − 2x 2 + 1 x − 2x 2 + 1
x4 + 3x2 − 6x + 10 x3 − 3x2 + x − 3 (a) (b) 1 =
x2 + y2 + z2 ( x − 1)( x − 2)( x2 − 9x + 14) x − 3x + x − 3
3 2
x4 − 3x2 + 6x + 10 (x − 1)3 x4 + 3x2 − 6x + 10
9. (d) Given,
(c) (d) (c) −1 (d) 0 ( x − 7)( x2 − 3x + 2) = 1 a2
x3 − 3x2 + x − 3 (x + 3)2 x 3 − 3x 2 + x − 3 +
1+ x 1 2a + 1 ( x − 1)( x − 2)( x − 7x − 2x + 14)
2
( 1 − a )( 1 − b ) ( 1 − a )( b − a )
14. If a = , then what is + equal to? =
7. The value of 2− x a + 1 a2 − 1 ( x − 7)( x2 − 2x − x + 2) 7. (b) We know that, b2
a b c −
+ + (1 + x)(2 + x) (1 − x)(2 − x) ( x − 1)( x − 2)( x − 7)( x − 2) a
+
b ( b − 1)( a − b )
(a) (b) = =x−2
( a − b) ( a − c) ( b − c) ( b − a ) ( c − a ) ( c − b) 2x − 1 x −2 ( x − 7)( x − 2)( x − 1) ( a − b )( a − c ) ( b − c )( b − a ) ( b − a) + a2( 1 − b) − b 2( 1 − a)
(1 + x)(2 − x ) (1 − x)(2 − x) c =
(a) 2a (b) 0 (c) 2b (d) b − c (c) (d) + ( 1 − a )( 1 − b )( b − a )
2x − 1 2x + 1 ( c − a )( c − b )
17
MATHEMATICS Rational Expressions 143 144 CDS Pathfinder

b − a + a 2 − a 2 b − b 2 + ab 2 1
+
2
+
1009
=1
= 15. (b) Given,
( 1 − a )( 1 − b )( b − a ) x + 1 y + 2 z + 1009
( b − a ) + ( a 2 − b 2 ) + ab( b − a ) 1 2 1009
= ⇒ − 1+ − 1+ − 1 = 1− 3
( 1 − a )( 1 − b )( b − a ) x+ 1 y+2 z + 1009
( b − a ) − ( a + b )( b − a ) + ab( b − a ) 1 − a − b + ab x y z
= = =1 ⇒ − − − = −2
( 1 − a )( 1 − b )( b − a ) 1 − b − a + ab x + 1 y + 2 z + 1009
x y z
∴ + + =2
 x y   y z  z x x + 1 y + 2 z + 1009
 −   −   − 
 y x z y x z
10. (b) Given, x 2 − 3x + 2 x 2 − 5x + 4
 1 1 1 1 1 ÷

LINEAR EQUATIONS
1 16. (b) Given,
 2 −  −  − x2 − 5x + 6 x2 − 7x + 12
x y 2   y 2 z 2   z 2 x2  x2 − 3x + 2 ( x2 − 7x + 12)
= 2 ×
 x2 − y 2   y 2 − z 2  z 2 − x 2  x − 5x + 6 ( x 2 − 5x + 4)
   
=  2
xy   zy   xz  ( x − 1)( x − 2) ( x − 4)( x − 3)
= × =1
 y − x2  z 2 − y 2   x2 − z 2  ( x − 3)( x − 2) ( x − 4)( x − 1)
 2 2  2 2  2 2 
 x y  z y  xz  17. (a) Given, x + y + z = 0
( x2 − y 2 )( y 2 − z 2 )(z 2 − x2 ) ⇒ x + y = −z , y + z = − x and z + x = − y
x2 y 2 z2 x 4 y 4z 4 ∴
xyz
=
xyz
=
xyz
= −1
=− 2 = − 2 2 2 = − x 2 y 2z 2 Generally (3-5) questions have been definitely asked from this chapter in CDS examination. Beside this,
( x + y )( y + z )(z + x) ( −z )( − x)( − y ) − xyz
( x − y 2 )( y 2 − z 2 )(z 2 − x2 ) x y z
the linear equations are also applicable in solving various word problems related to number system,
x 4 y 4z 4 ( x 2 + y 2 )( x − y ) − ( x − y )3
18. (b) Given, mensuration, time and work, etc.
 x 2 − 3x + 2   x 2 − 9   x 3 + 2x 2 + 4x  x 2 y − xy 2
11. (a) Given,   ÷ 2  × 2 
 x − 8   x + 7x + 12   x + 3x − 4 
3
x 3 + xy 2 − x 2 y − y 3 − ( x 3 − y 3 − 3x 2 y + 3xy 2 )
=
 x2 − 3x + 2 x2 + 7x + 12   x3 + 2x2 + 4x  x 2 y − xy 2
= ×  × 
 x −8
3
x 2 − 9   x 2 + 3x − 4  x 3 + xy 2 − x 2 y − y 3 − x 3 + y 3 + 3x 2 y − 3xy 2
= A linear equation is an equation for a straight line.
( x − 1)( x − 2) ( x + 4)( x + 3) x( x2 + 2x + 4) x 2 y − xy 2
= × × So, the equation which has degree 1, i.e., which has linear power of the variables, is called a linear
( x − 2)( x2 + 4 + 2x) ( x − 3)( x + 3) ( x − 1)( x + 4) 2x 2 y − 2xy 2 2( x 2 y − xy 2 )
= = =2 equation.
=
x
[Q a 3 − b 3 = ( a − b )( a 2 + b 2 + ab )] x 2 y − xy 2 x 2 y − xy 2
x −3 1 1 2b 4b 3 8b 7
12. (b) Given, pq + qr + r p = 0
19. (d) − −
a − b a + b a2 + b2 a4 + b4 a8 − b8
− − LINEAR EQUATIONS IN ONE VARIABLE
p2 q2 r2 ( a + b) − ( a − b) 2b 4b 3 8b 7 A linear equation in one variable is an equation which can be written in the form of ax + b = 0,
∴ + + = − 2 − −
p 2 − qr q 2 − r p r 2 − pq ( a − b )( a + b ) a + b2 a4 + b4 a8 − b8 or ax = c, where a, b, c are real numbers with a ≠ 0.
2 2 2 2b 2b 4b 3 8b 7 2 y
p q r = 2 − − −
= + + a − b2 a2 + b2 a4 + b4 a8 − b8 e.g. 5x + 8 = 9 − x , y + 7 = and t + 3t = 9 − t are linear equations in one variable.
p 2 + rp + pq q 2 + pq + qr r 2 + qr + rp 3 2
2b( a + b ) − 2b( a − b )
2 2 2 2
4b 3
8b 7
p2 q2 r2 p+ q+ r = − 4 −
= + + = =1 ( a 2 − b 2 )( a 2 + b 2 ) a + b4 a8 − b8
p ( p + r + q) q ( p + q + r) r ( p + q + r) p + q + r
4b 3 4b 3 8b 7
Solving a linear Equation in One Variable
= 4 − −
13. (d) Given, x + y + z = 0 ⇒ x + y = − z. a − b4 a4 + b4 a8 − b8 1. Simplify both sides of the equation.
On squaring both sides, we get x2 + y 2 + 2xy = z 2 4b ( a + b ) − 4b ( a − b 4 )
3 4 4 3 4
8b 7 2. Use the addition and subtraction properites to get all variables of terms on the LHS and all constant
= − 8 =0
x 2 + y 2 − z 2 = − 2 xy ( a 4 − b 4 )( a 4 + b 4 ) a − b8 terms on the RHS.
Similarly, y 2 + z 2 − x 2 = −2 yz and z 2 + x 2 − y 2 = −2zx 20. (c) Given, a 2 − by − cz = 0 …(i) 3. Simplify and divide both sides of the equation by the coefficient of the variable.

1
+
1
+
1 ax − b 2 + cz = 0 …(ii)
x + y − z 2 y 2 + z 2 − x2 z 2 + x2 − y 2
2 2 and ax + by − c 2 = 0 …(iii) EXAMPLE 1. Solve 2(x − 3) − (5 − 3x) = 3 (x + 1) − 4 (2 + x).
1 1 1 1z + x + y  On adding Eqs. (i), (ii) and (iii), we get a. 1 b. −1 c. 0 d. 3
= + + =  =0
−2xy −2 yz −2zx 2  xyz  b 2 − a 2 + c2 x b2 − a 2 + c2
x= ⇒ = Sol. a. 2 ( x − 3) − ( 5 − 3x) = 3 ( x + 1) − 4 ( 2 + x) ⇒ 2x − 6 − 5 + 3x = 3x + 3 − 8 − 4x
2a a + x a2 + b 2 + c2
1+ x
3   y a −b + c
2 2 2
z a 2 + b 2 − c2 ⇒ 5x − 11 = − x − 5 ⇒ 6x = 6 ⇒ x = 1, Hence, the value of x is 1.
1+ x 1 2a + 1 3a  2 − x Similarly, = and =
14. (c) Given, a = So, + = = b + y a 2 + b 2 + c2 c+z a 2 + b 2 + c2
2−x a + 1 a2 − 1 a2 − 1  1 + x 2

 2 − x
 −1
( b 2 − a 2 + c2 ) + ( a 2 − b 2 + c2 )
LINEAR EQUATION IN TWO VARIABLES
3( 1 + x)( 2 − x) 3( 1 + x)( 2 − x) ( 1 + x)( 2 − x)
x y z + ( a 2 + b 2 − c2 ) An equation which can be put in the form ax + by + c = 0, where a, b and c are real numbers and a, b
= = = ∴ + + = =1
1 + x 2 + 2x − ( 4 + x 2 − 4x) 6x − 3 ( 2x − 1) a+ x y + b c+z a 2 + b 2 + c2 both are not zero, is called a linear equation in two variables.
e.g. 2x + 3y = 5, 2x + 3y = 0 and 2a + 3b = 0 are linear equations in two variables.
MATHEMATICS Linear Equations 145 146 CDS Pathfinder

Sol. a. Given equations are, kx + 2y − 5 = 0 and Step II Substitute this value of x (or y) in other equation,
Pair of Linear Equations 3. Cross-multiplication Method
3x + y − 1 = 0 and reduce it to a linear equation in one variable
Two linear equations in the two same variables are called On comparing the given equations, with standard form Let us consider a general system of two simultaneous
pair of linear equations in two variables. The general form i.e. in terms of y (or x) which can be solved easily.
of equations linear equations
of pair of linear equations in two variables x and y is a1x + b1y + c1 = 0 and a2x + b2y + c2 = 0 Step III Substitute the value of y (or x) obtained in Step II a1 x + b1 y + c1 = 0 …(i)
a1 x + b1 y + c1 = 0 and a 2 x + b2 y + c 2 = 0 in the equation which is used to obtain the value
We have, a1 = k, b1 = 2, c1 = − 5 a 2 x + b2 y + c 2 = 0 …(ii)
where, a1 , a 2 , b1 , b2 and c1 , c 2 are all real numbers and and a2 = 3, b2 = 1, c2 = − 1
of the other variable in Step I.
a1 b1
a12 + b12 ≠ 0, a 22 + b22 ≠ 0. where, ≠ then the solution of the system is given by,
(i) Here, the equations have a unique solution, if EXAMPLE 4. The solutions of the system of a2 b2
a1 b1
Consistency Pair of ≠ equations x + y = 14 and x − y = 4, is x=
b1c2 − b2 c1 c a − c2 a1
and y = 1 2
a2 b2
a. x = 5 and y = 9 b. x = 9 and y = 5 a1b2 − a2 b1 a1b2 − a2 b1
k 2
linear Equations i.e. if ≠
3 1
⇒ k ≠6
c. x = 5 and y = 3 d. x = 3 and y = 5 ∴
x
=
y
=
1
A pair of linear equations which has no solution is called b1c2 − b2 c1 c1a2 − c2 a1 a1b2 − a2 b1
(ii) The equations have no solution, if Sol. b. Given equations are, x + y = 14 …(i)
an inconsistent pair of linear equations. A pair of linear a1 b1 c1 This method is called cross multiplication method.
= ≠ and x−y =4 …(ii)
equations which has atleast one solution is called a a2 b2 c2 from eq. (ii), y = x−4 …(iii) Remembering Technique
consistent pair of linear equations. Let us consider two k 2
linear equations as a1 x + b1 y + c1 = 0 and a 2 x + b2 y + c 2 = 0. ∴ = ⇒ k =6 On substituting the value of y from Eq. (iii) in Eq. (i), we get (Cross-multiplication)
3 1 x+ x − 4 = 14 ⇒ 2x = 18 ⇒ x = 9 The following diagram helps in remembering the above
Pair of linear equations in two variables will represent On substituting x = 9 in Eq. (iii), we get
two straight lines, both to be considered together. The EXAMPLE 3. The value of k for which the system solution.
y =9 − 4 = 5 ⇒ y = 5 x y 1
table given below repersents the three possibilities. of equations kx − y = 2, 6 x − 2y = 3 has infinitely many Hence, x = 9 and y = 5
solutions is b1 c1 a1 b1
Ratios Consistency Graphical Algebraic
Representation interpretation a. 3 b. 4 2. Elimination Method b2 c2 a2 b2
a1 b Consistent Exactly one c. 12 d. No such value of k exists.
≠ 1 In this method, one variable is eliminated from both The arrows between the two numbers indicate that they
a2 b 2 solution (unique).
Sol. d. Given equations are, equations to have an equation in one variable, so it are to be multiplied. The downward arrows indicate first
kx − y − 2 = 0 and 6x − 2y − 3 = 0 is called elimination method. product while upward arrows indicate the second product.
Intersecting lines. On comparing the given equations, with standard form The second product is to be subtracted from the first.
Steps used in this method are given below.
a1 b c Dependent of equations
= 1 = 1 Infinitely many Step I Make the coefficient of one variable ( x or y )
a2 b 2 c 2
(consistent) solutions. a1 x + b1 y + c1 = 0 and a2 x + b2 y + c 2 = 0 EXAMPLE 6. The system of equations
We have, a1 = k , b1 = − 1, c1 = − 2 numerically equal by multiplying both equations ax − by = a 2 − b 2 and x + y = a + b has the solution.
by some suitable non zero constant. a. x = a and y = b b. x = − a and y = − b
and a2 = 6, b2 = − 2, c2 = − 3
Coincident lines. Step II Now, add or subtract both equations, so that one c. x = b and y = a d. x = − b and y = − a
Here, the equations have infinite number of solutions, if
a1 b c Inconsistent No solution. variable is eliminated and the remaining equation
= 1 ≠ 1 a1 b1 c1 Sol. a. Given, pairs of linear equations are
a2 b 2 c 2 = = is in one variable only.
a2 b2 c2 a x − by − ( a2 − b2) = 0 ...(i)
a1 k b1 − 1 1 c −2 2 Step III Solve the equation in one variable to get the value and x + y − ( a + b) = 0 ...(ii)
But, = , = = and 1 = = of this variable (x or y).
Parallel lines. a2 6 b2 − 2 2 c2 −3 3 On comparing Eqs. (i) and (ii) with a1x + b1y + c1 = 0
b1 c1 Step IV Substitute this value (x or y) in either of the given and a2x + b2y + c2 = 0, we get
So, ≠ .
b2 c2 equations to get the value of other variable. a1 = a, b1 = − b, c1 = − ( a2 − b2)
HOMOGENEOUS PAIR OF Thus, the system has no such value of k, for which the a2 = 1, b2 = 1, c2 = − ( a + b)
EXAMPLE 5. The solution of the given system of
LINEAR EQUATIONS given system has infinitely many solutions.
equations 2x + 5y = 11 and 3x + 4y = 13 is
By cross multiplication, we have
x y 1
The pair of linear equations a1 x + b1 y = 0, a 2 x + b2 y = 0 has Algebraic Methods of Solutions a. (4, 2) b. (3, 1) c. (5, 2) d. (1, 1) –b –(a2–b2) a –b
a b
1. A unique solution, if 1 ≠ 1 and the solution is Different algebraic methods for solving pair of linear Sol. b. Given equations are, 2x + 5y =11 ...(i)
1 –(a+b) 1 1
x = 0, y = 0. a 2 b2 equations are discussed below in detail. x y 1
and 3x + 4y = 13 ...(ii) ⇒ = =
a b b( a + b) + ( a2 − b2) − ( a2 − b2 + a( a + b)) a + b
2. An infinite number of solutions, if 1 = 1 . On multiplying Eq. (i) by 3 and Eq. (ii) by 2, we get
a 2 b2 1. Substitution Method 6x + 15y = 33 …(iii) x y 1 ab + a2 b2 + ab
⇒ = = ⇒x= and y =
This pair of linear equations is always consistent. In this method, we substitute the value of one variable 6x + 8y = 26 …(iv) ab + a2 b2 + ab a + b a+ b a+ b
in terms of other variable to solve the pair of linear a( a + b) b ( b + a)
EXAMPLE 2. The values of k for which the system On subtracting Eq. (iv) from Eq. (iii), we get ⇒ x= = a and y = =b
equations, so this method is called substitution method. a+ b a+ b
kx + 2y = 5 and 3x + y = 1 has (6 x + 15y) − (6x + 8y) = 33 − 26 ⇒ 7y = 7 ⇒ y = 1
Steps used in this method are given below. So, x = a and y = b is the required solution.
(i) unique solution (ii) no solution are Put y = 1 in Eq. (i), 2x + 5 × 1 = 11
a. k ≠ 6 and k = 6 b. k ≠ 3 and k = 3 Step I Find the value of one variable x (or y) in terms of ⇒ 2x = 11 − 5 ⇒ 2x = 6 ⇒ x = 3
c. k ≠ 2 and k = 2 d. None of these other variable i.e. y (or x) from an equation. Hence, solution of system of equations is (3, 1).
MATHEMATICS Linear Equations 147 148 CDS Pathfinder

⇒ 4A + 6 = 18 ⇒ 4A = 12 ⇒ A = 3 EXAMPLE 10. Solve the system x + y + z = 5,


EQUATIONS REDUCIBLE TO A 1 1 1 1 2x − y + z = 9 and x − 2y + 3z = 16.
Problem Based on Numbers
Now = B = 2 ⇒ x = and = A = 3⇒ y = Let the digit in unit’s place be x and that in ten’s place
PAIR OF LINEAR EQUATIONS x 2 y 3 a. ( 2, − 1, 4 ) b. ( 3, 5, 6 ) c. (1, − 2, 5 ) d. ( 6 , 1, − 3)
be y. Then, the two-digits number is given by 10 y + x.
1 1
We have several situations when there are two So, x = and y = is the required solution. Sol. a. Given equations are, x + y + z = 5 …(i) On interchanging the positions of the digits, the digit in
2 3
equations that are not linear, but can be reduced to a pair 2x − y + z = 9 …(ii) unit place becomes y and that in ten’s place becomes x,
of linear equations. By making suitable substitutions. 3 2 9 4 x − 2y + 3z = 16 …(iii) and the number becomes 10 x + y. Formulate the
EXAMPLE 9. If + = 2 and − = 1,
2 3 5 4 x+y x−y x+y x−y First we eliminate y by adding Eqs. (i) and (ii), we get equations and then solve them.
EXAMPLE 7. If + = 13 and − = − 2 , then x x+ y + z = 5
x y x y then what is value of ? 2x − y + z = 9 EXAMPLE 12. A number consists of two digits,
x y whose sum is 8. If 18 is added to the number, the
what is the value of ? 3 2 1 3x + 2z = 14 …(iv)
y a. b. 5 c. d. digits are reversed. The number is equal to
2 3 5 Next we eliminate y by multiplying Eq. (i) by 2 and then
2 3 1 1 adding it to Eq. (iii), we get a. 26 b. 35 c. 53 d. 62
a. b. c. d.
3 2 3 2 Sol. b. Given equations are,
2x + 2y + 2z = 10 Sol. b. Let the unit's place digit and ten's place digit be x
1 1 3 2
Sol. b. Let = p and = q + =2 …(i) x − 2y + 3z = 16 and y, respectively.
x+ y x−y
x y
3x + 5z = 26 …(v) ∴Original number = 10y + x
So, we have, 2p + 3q = 13 9 4
…(i) and − =1 …(ii) Now subtract Eq. (iv) from Eq. (v), we get Number obtained by reversing the order of digit = 10x+ y
and 5p − 4q = − 2 …(ii) x+ y x−y
3z = 12 ⇒ z = 4 Condition I Sum of digits = 8
1 1
On multiplying Eq. (i) by 4 and Eq. (ii) by 3 and then Let = a and =b ⇒ x+ y =8
x+ y x−y Substitute z = 4 in Eq. (iv), 3x + 2 × 4 = 14 ⇒ 3x = 6 ⇒ x = 2. …(i)
adding, we get Condition II 10x + y = 10y + x + 18
∴ 3a + 2b = 2 Finally put x = 2 and z = 4 in Eq. (i), we get
8 p + 15p = 52 − 6 ⇒ 23p = 46 ⇒ p = 2 …(iii)
⇒ 10x − 10y + y − x = 18 ⇒ 9x − 9y = 18
9a − 4b = 1 2 + y + 4 = 5 ⇒ y = −1
On putting p = 2 in Eq. (i), we get …(iv) ⇒ x−y = 2 …(ii)
9 On multiplying Eq. (iii) by 2 and adding with Eq. (iv), we ∴ (2, −1, 4) is the required solution.
2 × 2 + 3q = 13 ⇒ 3q = 13 − 4 = 9 ⇒ q = = 3 On adding Eqs. (i) and (ii), we get
3 get
x + y + x − y = 8 + 2 ⇒ 2x = 10
Now, x=
1
p
1
⇒ x = and y =
2
1
q
⇒ y= .
1
3
⇒ 6 a + 9 a = 5 ⇒ 15a = 5
1
APPLICATION OF LINEAR ∴ x=5
∴ a=
x 3 3 EQUATIONS On putting x = 5 in Eq. (i), we get
Hence, the value of is . 1 1 5+ y = 8 ⇒ y = 3
y 2 ∴ = ⇒ x+ y =3 …(v) The application of linear equations are discussed below.
x+ y 3 Hence, Required number = 10y + x = 10 × 3 + 5 = 35
EXAMPLE 8. Find the solution of the system of On putting the value of a in Eq. (iii), we get Problem Based on Ages
equations 4x + 3y = 18xy and 2x − 5y = − 4xy. 1
3 × + 2b = 2 ⇒ b =
1 If the problem involves finding out the ages of two Problem Based on Fractions
1 1
a. x = 2, y = 3 b. x = 3, y = 2 c. x = , y =
1
d. x = , y =
1 3 2 persons, then take the present age of one person as x Let the numerator of the fraction be x and denominator
1 1
3 2 2 3 ⇒ = ⇒ x−y =2 …(vi) and that of the other as y. be y, then the fraction is x / y.
Sol. d. Given equations are, 4x + 3y = 18xy ...(i) x−y 2
5 Then, ‘a’ years ago, age of Ist person was ( x − a) years Formulate the linear equations on the basis of
2x − 5y = −4xy On adding Eqs. (v) and (vi), we get, 2x = 5 ⇒ x =
and ...(ii)
2 and that of 2nd person was ( y − a) years. conditions given and solve them.
On dividing both the equations by xy, we get 5 1 After b years, age of Ist person will be ( x + b) years and
From Eq. (v), x + y = 3 ⇒ y = 3 − = EXAMPLE 13. If we add 1 to the numerator and
4x 3y 18xy 2x 5y − 4xy 2 2 that of 2nd person will be ( y + b) years.
+ = and − = 5 subtract 1 from the denominator, a fraction becomes
xy xy xy x y xy xy Formulate the equations and then solve them. 1
x 2
4 3 2 5 ∴ = =5 1. It also becomes , if we only add 1 to the
or + = 18 and − =−4 y 1 2
y x y x
2
EXAMPLE 11. Age of X is six times that of Y. After 4 yr, denominator. The fraction is
1 1 X is 4 times elder to Y. What is the present age of Y?
On putting = A and = B, we get a. 4/5 b. 3/5 c. 2/5 d. 1/5
y x
4A + 3B = 18 …(iii)
LINEAR EQUATION IN a. 4 yr b. 5 yr c. 6 yr d. 7 yr
Sol. b. Let the numerator and the denominator of the
Sol. c. Let the present age of X and Y be x and y yr,
2A − 5B = − 4 …(iv) THREE VARIABLES respectively.
fraction be x and y, respectively.
x
On multiplying Eq. (iv) by 2 and subtracting from Eq. (iii), An equation in the form of ax + by + cz = r , where a, b, c ∴ Fraction =
Condition I Age of X = Age of Y × 6, x = 6y ...(i) y
we get and r are real numbers and a, b and c are not all zeros is
4A + 3B = 18 After 4 yr, Age of X = ( x + 4) yr , Age of Y = ( y + 4) yr Condition I When 1 is added to the numerator and 1 is
called a linear equation in three variables x+1
4A − 10B = − 8 Condition II ( x + 4) = ( y + 4) × 4, x + 4 = 4y + 16 ...(ii) subtract from the denominator, then new fraction =
− + + e.g. 3x + 4y − 7 z = 2 , −2x + y − z = 6 and x + y + z = 2 are all y −1
linear equation in three variables. Put the value of x = 6y in Eq. (ii), we get
13B = 26 According to the question,
⇒ 6y + 4 = 4y + 16 ⇒ 2y = 12
⇒ B=2 Use substitution and elimination method to solve the x+1
On putting, the value of B in Eq. (iii), we get ∴ y =6 = 1 ⇒ x + 1= y − 1 ⇒ x − y = − 2 …(i)
system of three equations in three variables. y −1
So, the present age of Y is 6 yr.
MATHEMATICS Linear Equations 149 150 CDS Pathfinder

Condition II When 1 is added to denominator, then EXAMPLE 15. A railway ticket for a child costs half EXAMPLE 17. The area of a rectangle is decreased ⇒ xy − 2x + 2y − 4 = xy − 28
x
new fraction = the full fare but the reservation charge is the same on by 28 m 2 , if the length is increased by 2 m and the ⇒ 2x − 2y = 24 ⇒ x − y = 12 …(i)
y+1 half tickets as much as on full ticket. One reserved first breadth is decreased by 2 m. The area of a rectangle
x 1 Condition II If, length = ( x − 1) m and breadth = ( y + 2) m
According to the question, = ⇒ 2x = y + 1 class ticket for a journey between two stations is ` 362, is increased by 33 m 2 , if the length is decreased by
y+1 2 ∴ Now the area of rectangle = ( x − 1)( y + 2) m 2
⇒ 2x − y = 1 …(ii) one full and one half reserved first class tickets cost 1m and the breadth is increased by 2 m, then the area
On subtracting Eq. (ii) from Eq. (i), we get ` 554. What is the reservation charge? of rectangle is According to the question,
−x = − 3 ⇒ x = 3 a. ` 18 b. ` 22 c. ` 38 d. ` 46 a. 253 m2 b. 235 m2 c. 532 m2 d. 352 m2 ( x − 1)( y + 2) = xy + 33
Put the value of x in Eq. (i), we get Sol. b. Let full fare and reservation charge be ` x and Sol. a. Let the length and breadth of a rectangle be x and y, ⇒ xy + 2x − y − 2 = xy + 33
3 − y = − 2⇒ y = 5 ` y, respectively. Then, full ticket = x + y respectively. ⇒ 2x − y = 35 …(ii)
x 3
Hence, the fractions is = x ∴ Original area of rectangle = xy m 2 On subtracting Eq. (i) from (ii), we get, x = 23
y 5 half ticket = + y
2 Condition I If length = ( x + 2) m and breadth = ( y − 2) m On putting x = 23 in Eq. (i), we get,
According to question, x + y = 362 ...(i) 23 − y = 12
Problem Based on Distance, ∴ Area of rectangle = ( x + 2)( y − 2) m 2
( x+ y) +  + y = 554 = x + 2y = 554 ...(ii)
x 3
and According to the question, ∴ y = 23 − 12 = 11
Speed and Time 2  2
( x + 2)( y − 2) = xy − 28 So, original area of rectangle = xy = 23 × 11= 253 m 2
Speed is the distance covered by an object per unit time. On multiplying Eq. (i) by 2, we get
Distance 2x + 2y = 724 ...(iii)
i.e. Speed =
Time On subtracting Eq. (ii) from Eq. (iii), we get
If the speed of boat in still water be x km/h and speed
of stream be y km/h. Then, the speed of boat
downstream = ( x + y) km/h
x
2
= 170 ⇒ x = 340

On putting value of x in Eq. (i), we get


PRACTICE EXERCISE
and speed of boat upstream = ( x − y) km/h 340 + y = 362 ⇒ y = 22
Hence, the reservation charge is ` 22. 1. Find x, if 25x − 19 − [3 − { 4x − 5}] = 3x − ( 6x − 5). 7. The system of equations 6x + 5 y = 11 and
EXAMPLE 14. A motorboat takes 2h to travel a 1 15
(a) x = 1 (b) x = − 1 (c) x = (d) x = 2 9x + y = 21 has
distance of 9 km down the current and it takes 6h to Problems Based on Mensuration 2 2
travel the same distance against the current. What is Make use of given formulae to formulate the equation 2. If a number is subtracted from three-fourth of (a) a unique solution (b) many solution
the speed of the boat in still water ? itself, the value so obtained is − 130. Then, what (c) no solution (d) None of these
1. Area of rectangle = length × breadth
a. 3 km/h b. 2 km/h c. 1.5 km/h d. 1 km/h is the number?
Perimeter of rectangle = 2 (length+Breadth) 8. The sum of two numbers is 2490 and if 6.5% of
(a) 540 (b) 560 (c) 420 (d) 520
Sol. a. Let speed of the motorboat in still water be x km/h one number is equal to 8.5% of the other, then
and speed of the stream be y km/h. 2. The sum of angles of a triangle is 180° 3. Sum of two numbers is 21 and their difference is numbers are
Then, speed of the motorboat downstream = ( x + y) km/h 3. The sum of opposite angles of cyclic quadrilateral is 11, then the greatest number is (a) 1414, 1076 (b) 1411, 1079
and speed of the boat upstream = ( x − y) km/h 180° (a) 5 (b) 16 (c) 9 (d) 10 (c) 1412, 1078 (d) None of these
Condition I When motorboat goes 9 km downstream, then 4. In parallelogram, opposite angles are equal. 4. Which of the following equations have x = 2 and 9. Given two linear equations a1x + b1 y = c1 and
9  Distance  y = 1 as a solution? a1 b1
2= Q Time =  EXAMPLE 16. The length of a rectangle is 8 cm a2x + b2 y = c2 , if ≠ , then the graph is
x+ y Speed  I. 2x + 5 y = 9 II. 5x + 3 y = 14 a2 b2
 more than its breadth. If the perimeter of the III. 2x + 3 y = 7 IV. 2x − 3 y = 1
⇒ 2x + 2y = 9 …(i) rectangle of 68 cm, then its length and breadth are (a) parallel (b) intersection at one point
Select the correct answer using the codes given (c) coincident (d) None of these
Condition II When motorboat goes 9 km upstream, then a. 21 cm, 13 cm b. 14 cm, 23 cm
below
9 c. 19 cm, 20 cm d. 9 cm, 15 cm 10. For what value of k, the following equations will
6= ⇒ 6x − 6y = 9 …(ii) (a) I and IV (b) II and III (c) Only I (d) I, III and IV
x−y be inconsistent? 4x + 6 y = 11 and 2x + ky = 7
Sol. a. Let the length and breadth of a rectangle be x and y, x
On multiplying Eq. (i) by 3, we get respectively.
5. The solution of the pair of equation + y = 0.8 (a) k = − 3 (b) k =
12
(c) k = 12 (d) k = 3
2 5
6x + 6y = 27 …(iii) Condition I x= y + 8 ⇒ x− y =8 …(i) 7
and = 10 is 11. For what value of k, the system of equations has
Now, adding Eqs. (ii) and (iii), we get y
Condition II Perimeter of rectangle = 68 cm x+ infinitely many solutions 2x − ky = 4 and
12x = 36 ⇒ x = 3 km/h 2
⇒ 2[ x + y ] = 68, x + y = 34 …(ii) 2 3 2 3x + 2 y = 6?
(a) x = , y = (b) x = , y = 5 4 −4 2 3
Problem Based on Fixed On adding Eq. (i) and (ii), we get 5
2
5
5
3
3 2
(a)
3
(b)
3
(c)
3
(d)
2
and running Charges 2x = 42 ⇒ x = 21 (c) x = , y = (d) x = , y =
On putting x = 21in Eq. (ii), we get
5 3 5 5 12. For what value of α, the system of equation
Let the fixed charge of any commodity be ` x and rate 6. A system of two simultaneous linear equations in
21 + y = 34 ⇒ y = 34 − 21 = 13 αx + 3 y = α − 3, 12x + αy = α will have a unique
of running charges be ` y, then total cost = x + total two variables has a unique solution if their graphs
Hence, length of rectangle = 21cm solution?
running charges. (a) are coincident (b) are parallel
(a) α = ± 6 (b) α = 6 (c) α ≠ ± 6 (d) α = − 6
Formulate the equations and then solve them. and breadth of rectangle = 13 cm (c) intersect in one point (d) None of these
MATHEMATICS Linear Equations 151 152 CDS Pathfinder

13. Consider the following statements: 44 30 55 40 31. A train started from a station with a certain 41. What is the value of k for which the system of
23. If + = 10, + = 13, then find
I. The solution of equation 2x + 3 y = 425 and x+ y x − y x+ y x − y number of passengers. At the first halt, 1/3rd of equations x + 2 y − 3 = 0 and 5x + ky + 7 = 0 has no
3x + 2 y = 350 is a positive integral pair. the value of x and y. its passengers got down and 120 passengers got solution?
(a) x = 2, y = 8 (b) x = 8, y = 3 in. At the second halt, half of the passengers 3 14 1
II. for k = −2, the equations x − ky = 2 and 3x + 6 y = −5 (a) − (b) − (c) (d) 10
have infinitely many solutions. (c) x = 8, y = 2 (d) x = 3, y = 8 got down and 100 persons got in. Then, the 14 3 10
train left for its destination with 240 passengers.
III. (2, 5) and (−1, 3) are solutions of 2x + 5 y = 13 24. Consider the following sets of equations How many passengers were there in the train 42. What is the solution of the equation x − y = 0.9
Select the correct answer using the codes given I. 2x − y = 0 and 6x − 3 y = 0 when it started ? and 11( x + y )−1 = 2 ?
below: II. 3x − 4 y = 0 and 12x − 20 y = 0 Then, (a) 540 (b) 480 (c) 360 (d) 240 (a) x = 3.2 and y = 2.3 (b) x = 1 and y = 0.1
(a) Only I (b) Both I and II (a) Both sets I and II possess unique solution (c) x = 2 and y = 11
. (d) x = 1.2 and y = 0. 3
(c) Both II and III (d) All of these 32. The system of equations x + 2 y = 3 and 3x + 6 y = 9
(b) Set I possesses unique solution and set II has
infinitely many solutions has 43. Pooja started her job with certain monthly
14. A horse and two cows together cost ` 680. If a (a) unique solutions (b) no solution salary and gets a fixed increment every year.
horse costs ` 80 more than a cow, then the cost (c) Set I possesses infinitely many solutions and set II
possess unique solution (c) infinitely many solutions (d) finite number of solutions If her salary was ` 4200 after 3 yr and ` 6800
of horse is after 8 yr of service, then what are her
(d) None of the sets I and II possesses a unique solution
(a) ` 170 (b) ` 280 (c) ` 200 (d) ` 220 33. The sum of digits of a two-digit number is 8 and initial salary and the annual increment,
4
15. Sunita has 10 paise and 50 paise coins in her 25. A fraction becomes , if 1 is added to both the difference between the number and that respectively ?
5 formed by reversing the digits is 18. What is the
purse. If the total number of coins is 17 and (a) ` 2640, ` 320 (b) ` 2460, ` 320
numerator and denominator. If however 5 is difference between the digits of the number?
their total value is ` 4.50, then number of 10 (c) ` 2460, ` 520 (d) ` 2640, ` 520
subtracted from both numerator and denominator (a) 1 (b) 2 (c) 3 (d) 4
paise coins is 1
the fraction becomes . Then, the fractions is 44. A person bought 5 tickets from the station P to a
(a) 9 (b) 7 (c) 10 (d) 5 2 34. If ( x , y ) = ( 4, 1) is the solution of the pair of linear station Q and 10 tickets from the station P to a
16. The solution of the equation (a)
7
(b)
9
(c)
3
(d)
4 equations mx + y = 2x + ny = 5, then what is station R. He paid ` 350. If the sum of fare of a
3x − y + 1 2x + y + 2 3x + 2 y + 1 9 7 5 3 m + n equal to? ticket from P to Q and a ticket from P to R is `
= = is given by
3 5 6 26. For what value of k, the following system of (a) −2 (b) −1 (c) 1 (d) 1 42, then what is the fare from P to Q ?
which one of the following? equations 3x + 4 y = 6 and 6x + 8 y = k represents 35. If 2a + 3b = 17 and 2a + 2 − 3b + 1 = 5, then (a) ` 12 (b) ` 14
(a) x = 2, y = 1 (b) x = 1, y = 1 coincident lines? (c) ` 16 (d) ` 18
(c) x = − 1, y = − 1 (d) x = 1, y = 2 (a) 12 (b) 11 (c) 13 (d) 10 (a) a = 2, b = 3 (b) a = − 2, b = 3
(c) a = 2, b = − 3 (d) a = 3, b = 2 45. The Community Relief fund receives a large
17. The line 3x − 5 y = −10 cuts Y-axis at 27. The area of a rectangle remains the same if the donation of $ 2800. The foundation agrees to
length is increased by 7 m and the breadth is 36. The sum of two numbers is 80. If the larger spend the money on $ 20 school bags, $ 25
(a) (0, 2) (b) (0, 1) (c) (0, 3) (0, 4)
decreased by 3 m. The area remains unaffected if number exceeds four times the smaller by 5, sweaters, $ 5 books. They want to buy 200 items
18. The equations px + q = 0 and rx + s = 0 are the length is decreased by 7 m and the what is the smaller number ? and send them to schools in earthquake-hit
consistent, if breadth is increased by 5 m, then area of (a) 5 (b) 15 (c) 20 (d) 25
areas. They must order as many books as school
(a) ps = qr (b) ps + qr = 0 (c) pq − rs = 0 (d) pq + rs = 0 rectangle is 2 3 9 4 9 21 bages and sweaters combined.
(a) 280 m 2
(b) 320 m 2
(c) 420 m 2
(d) 400 m 2 37. If + = and + = , where x, y ≠ 0
19. A streamer goes downstream and covers the x y xy x y xy How many of each item should they order?
distance between two ports in 4 h while it covers 28. The value of x in the solution of the equation and y ≠ 0, then what is the value of x + y ? (a) (40, 60, 100) (b) (20, 30, 80)
x + y x − y
the same distance upstream in 5 h. If the speed 2 =2 = 8 is (a) 2 (b) 3 (c) 4 (d) 8 (c) (50, 100, 60) (d) 40, 80, 25)
of the stream is 2 km/h, then the speed of the 3 1 1
(a) 0 (b) (c) (d) 38. Under what condition do the equation kx − y = 2
streamer in still water is 2 4 8 Directions (Q. Nos. 46-47) Some money is divided
(a) 20 km/h (b) 19 km/h (c) 18 km/h (d) 19.5 km/h and 6x − 2 y = 3 have a unique solution? among Rajesh, Sonal and Chetan in such a way that
29. A and B each have a certain number of mangoes. (a) k = 3 (b) k ≠ 3 (c) k = 0 (d) k ≠ 0 2 times share of Rajesh, 3 times share of Sonal and
20. The solution of the system of linear equations A says to B : “If you give 30 of your mangoes, I
0.4x + 0.3 y = 1.7 and 0.7x − 0.2 y = 0.8 is 5 times share of Chetan are all equal.
will have twice as many as left with you” B 39. Let there be three simultaneous linear equations
(a) x = 3, y = 2 (b) x = 2, y = −3 replies “If you give me 10, I will have thrice as in two unknowns, which are non-parallel and Now, answer the following questions based on above
(c) x = 2, y = 3 (d) None of these many as left with you”. How many mangoes did information.
non-collinear. What can be the number of
A has? solutions (if they do exist)?
21. If 2x − 3 y = 0 and 7x + 2 y = 0, then find the (a) 41 (b) 62 (c) 34 (d) 32
46. If the sum of 6 times the share with Rajesh and
(a) One or infinite (b) Only one
value of x + y. 6 times the share with Sonal is ` 150, then find
30. There are two examination rooms A and B. If 10 (c) Exactly two (d) Exactly three
(a) 1 (b) 2 (c) 3 (d) 0 the share of Chetan?
candidates are sent from room A to room B, 40. A number consists of two digits, whose sum is (a) ` 5 (b) ` 6 (c) ` 10 (d) ` 12
22. A man starts his job with a certain monthly the number of students in each room is the
salary and earns a fixed increment every year. If 10. If 18 is subtracted from the number, digits of
same. If 20 candidates are sent from B to A, the the number are reversed. What is the product? 47. If the sum of shares of Rajesh, Chetan and Sonal
his salary was ` 1500 after 4 yr of services and number of students in A is double the number is ` 155, then find the share of Rajesh.
` 1800 after 10 yr of service. What was his starting (a) 15 (b) 18
of students in B. Then, number of students in (a) ` 75 (b) ` 35 (c) ` 50 (d) ` 25
salary? (c) 24 (d) 32
room B is
(a) ` 1300 (b) ` 1200 (c) ` 50 (d) ` 1100 (a) 40 (b) 100 (c) 80 (d) 60
MATHEMATICS Linear Equations 153 154 CDS Pathfinder

PREVIOUS YEARS’ QUESTIONS 57. A bus starts with some passengers. At the first 67. A number consists of two digits, whose sum is 7. 70. There are three brothers. The sums of ages of
stop, one-fifth of the passengers gets down If the digits are reversed, the number is increased two of them at a time are 4 yr, 6 yr and 8 yr.
48. The graphs of ax + by = c, dx + ey = f will be and 40 passengers get in. At the second stop, by 27. The product of digits of the number is The age difference between the eldest and the
I. Parallel, if the system has no solution. half of the passengers gets down and 30 get in. e 2015 II youngest is e 2016 I
II. Coincident, if the system has finite number of The number of passengers now is 70. The (a) 6 (b) 8 (c) 10 (d) 12 (a) 3 yr (b) 4 yr (c) 5 yr (d) 6 yr
solutions. number of passengers with which the bus started p q q p x
was e 2013 II 68. If + = m and + = n, then what is equal 71. The annual incomes of two persons are in the
III. Intersecting, if the system has only one solutions
x y x y y ratio 9 : 7 and their expenses are in the ratio
(a) 40 (b) 50 (c) 60 (d) 70
Which of the statement(s) given above is/are to? e 2016 I 4 : 3. If each of them saves ` 2000 per year, then
correct? e 2012 I 58. If x + y − 7 = 0 and 3x + y − 13 = 0, then what is (a)
np + mq
(b)
np + mq what is the difference in their annual incomes?
(a) Both I and II (b) Both II and III 4x 2 + y 2 + 4xy equal to? mp + nq mp − nq e 2016 I
e 2013 II
(c) Both I and III (d) All of these np − mq np − mq
(a) 75 (b) 85 (c) 91 (d) 100 (c) (d) (a) ` 4000 (b) ` 4500 (c) ` 5000 (d) ` 5500
49. The sum of two numbers is 20 and their product is x y 2 3 mp − nq mp + nq
59. If + = 4 and + = 1, then what is x + y 72. Let a two digits number be k times the sum of its
75. What is the sum of their reciprocals? e 2012 I 2 3 x y digits. If the number formed by interchanging
1 1 4 7 69. The value of k, for which the system of equations
(a) (b) (c) (d) equal to? e 2013 II 3x − ky − 20 = 0 and 6x − 10 y + 40 = 0 has no the digits is m times the sum of the digits, then
15 5 15 15
(a) 11 (b) 10 (c) 9 (d) 8 solution, is e 2016 I the value of m is e 2016 I
50. If 3x + y = 81 and 81x − y = 3, then what is the value (a) 10 (b) 6 (c) 5 (d) 3 (a) 9 − k (b) 10 − k (c) 11 − k (d) k − 1
60. If x + y = 5, y + z = 10 and z + x = 15, then which
of x? e 2012 I
17 17 17 15 one of the following is correct? e 2014 I
(a) (b) (c) (d)
16 8 4 4 (a) z > x > y (b) z > y > x (c) x > y > z (d) x > z > y
a b x a b 61. The present age of Ravi’s father is 4 times Ravi’s ANSWERS
51. If − = and + = x − y, then what is the present age. 5 yr back, Ravi’s father was seven
b a y b a 1 a 2 d 3 b 4 d 5 a 6 c 7 c 8 b 9 b 10 d
value of x ? e 2013 I
times as old as Ravi was at that time. What is
the present age of Ravi’s father? e 2014 I
11 b 12 c 13 a 14 b 15 c 16 b 17 a 18 a 19 c 20 c
a+ b a+ b a−b
(a) (b) (c) (d) None of these 21 d 22 a 23 b 24 c 25 a 26 a 27 c 28 b 29 c 30 c
a b a (a) 84 yr (b) 70 yr (c) 40 yr (d) 35 yr
62. The sum of two positive number x and y is 2.5 31 d 32 c 33 b 34 a 35 d 36 b 37 c 38 b 39 b 40 c
52. The system of equations 3x + y − 4 = 0 and
times their difference. If the product of numbers 41 d 42 a 43 d 44 b 45 a 46 b 47 a 48 c 49 c 50 b
6x + 2 y − 8 = 0 has e 2013 I
(a) a unique solution x = 1, y = 1 is 84, then what is the sum of those two 51 d 52 d 53 c 54 b 55 a 56 a 57 b 58 d 59 b 60 a
(b) a unique solution x = 0 , y = 4 numbers? e 2014 I 61 c 62 d 63 a 64 c 65 a 66 b 67 c 68 c 69 c 70 b
(c) no solution (a) 26 (b) 24 (c) 22 (d) 20
71 a 72 c
(d) infinite solutions 63. Two chairs and one table cost ` 700 and one
53. The sum of two numbers is 7 and the sum of chair and Two tables cost ` 800. If cost m tables
their squares is 25. The product of the two and m chairs is ` 30000, then what is m equal
numbers is e 2013 I to? e 2014 I
(a) 6 (b) 10 (c) 12 (d) 15 (a) 60 (b) 55 (c) 50 (d) 45 HINTS AND SOLUTIONS
Directions (Q. Nos. 54-55) A number consists of two 64. A certain number of two digits is three times the
sum of its digits. If 45 is added to the number,
digits whose sum is 10. If the digits of the number 1. (a) We have, 25 x − 19 − [3 − { 4x − 5}] 3. (b) Let numbers be x and y, respectively. ⇒ 13 = 14, it is false.
then the digits will be reversed. What is the sum
are reversed, then the number is decreased by 36. = 3x − ( 6x − 5) Then by given condition, III. 2x + 3 y = 7 ⇒ 2 ( 2) + 3 ( 1) = 7
of the squares of the two digits of the number?
54. Which of the following is/are correct? ⇒ 25x − 19 − [3 − 4x + 5] x + y = 21 …(i) 7 = 7 , it is true.
(a) 41 (b) 45 e 2014 II
(c) 53 (d) 64 = 3x − 6x + 5 x − y = 11 …(ii)
I. The number is divisible by a composite number. IV. 2x − 3 y = 1 ⇒ 2 ( 2) − 3 ( 1) = 1
II. The number is a multiple of a prime number. ⇒ 25x − 19 + 4x − 8 = − 3x + 5 On adding Eqs. (i) and (ii), we get
65. A student was asked to multiply a number by 25. 1 = 1 , it is true.
⇒ 29x + 3x = 5 + 27 2x = 32
Select the correct answer using the codes given He instead multiplied the number by 52 and got 32 So, x = 2 and y = 1
32 x= = 16
below e 2013 I the answer 324 more than the correct answer. ⇒ 32x = 32 ⇒ x = =1⇒ x=1 2 is a solution of I, III and IV.
The number to be multiplied was e 2015 I 32
(a) Only I (b) Only II On putting y = 16 in Eq. (i), we get
Hence, the value of x is 1. 5. (a) Given equations are,
(c) Both I and II (d) Neither I nor II (a) 12 (b) 15 (c) 25 (d) 32 x + 16 = 21
x + 2 y = 16
. ...(i)
4 2. (d) Let the number be x. ∴ x=5
55. What is the products of the two digits? e 2013 I 66. A tin of oil was full. When 6 bottles of oil were y 7
5 According to the question, Hence, the greatest number is 16. and x+ = ...(ii)
(a) 21 (b) 24 (c) 36 (d) 42 2 10
taken out from this tin and 4 bottles of oil were 3x 3x − 4x
− x = − 130 ⇒ = − 130 4. (d) Put x = 2 and y = 1in each equation
56. Ten chairs and six tables together cost ` 6200, poured into it, it was 3 / 4 full. Oil of how many 4 4 On multiplying Eq. (i) by 10 and
I. 2x + 5 y = 9 ⇒ 2 ( 2) + 5 ( 1) = 9
three chairs and two tables together cost ` 1900. bottles can the tin contain? (All bottles are of −x Eq. (ii) by 10, we get
⇒ = − 130 ⇒ x = 520 9 = 9 , it is true.
The cost of 4 chairs and 5 tables is e 2013 I equal volume) e 2015 II 4 or 10x + 20 y = 16 …(iii)
II. 5x + 3 y = 14
(a) ` 3000 (b) ` 3300 (c) ` 3500 (d) ` 3800 (a) 35 (b) 40 (c) 45 (d) 50 Hence, the number is 520. ⇒ 5( 2) + 3( 1) = 14 and 10x + 5 y = 7 …(iv)
MATHEMATICS Linear Equations 155 156 CDS Pathfinder

On subtracting, Eq. (iv) from Eq. (iii), a 2 b1 −k c 4 On subtracting Eq. (i) from Eq. (iii), we 8x + 6 y = 34 ...(iii) 24. (c) Set I 2x − y = 0 and 6x − 3 y = 0 27. (c) Let the length of rectangle be x m
Now, 1 = , = and 1 =
we get a 3 b 2 c 6 get and 21x − 6 y = 24 ...(iv) Here, a = 2, b = − 1 and breadth be y m.
( 10x + 20 y ) − ( 10x + 5 y ) = 16 − 7
2 2 2 ( x + 5 y ) − ( x + y ) = 45 − 17 1 1
∴ Original area of rectangle = xy
On adding Eqs. (iii) and (iv), we get and a = 6 , b = −3
Since, the system has infinitely many 28 2 2
⇒ 15 y = 9 ⇒ y =
9
⇒y=
3 ⇒ 4 y = 28 ⇒ y = =7 (8x + 6 y ) + ( 21x − 6 y ) = 34 + 24 a b Case I Length = ( x + 7) m and breadth
solutions 4 ⇒ 1 = 2 = 1 ⇒ 1 = 1
15 5 ⇒ 29x = 58 a 6 3 b 3 = ( y − 3) m
1 = 1 = 1, 2 = − k = 2 ∴ Number of 10 paise coins
a b c
On putting the value of y in Eq (i), we ∴ x=2 2 2
get a b c 3 2 3 = x = 17 − y = 17 − 7 = 10 a b ∴ New area of rectangle
2 2 2 On putting x = 2 in Eq. (i), we get ∴ 1 = 1
8
x + 2y = , x + =
6 8 −4 a b = ( x + 7) ( y − 3)
5 5 5 ⇒ − 3k = 4 ⇒ k = 16. (b) Given, 8 + 3 y = 17 ⇒ y = 3 2 2
3 Given, xy = xy − 3x + 7 y − 21
8 6 2 3x − y + 1 2x + y + 2 3x + 2 y + 1 Hence, x = 2 and y = 3 is the required So, system of equations have infinitely
⇒ x= − = 12. (c) Given equations are, = = ⇒ 3x − 7 y = − 21 …(i)
5 5 5 3 5 6 solution. many solutions.
αx + 3 y = α − 3 and 12x + αy = α ( I) ( II) ( III)
21. (d) Given, 2x − 3 y = 0 Case II Length = ( x − 7)
6. (c) A pair of linear equations in two
Here, a = α, b = 3, c = α − 3 Taking Ist and IInd terms, Set II 3x − 4 y = 0 and 12x − 20 y = 0
variables has a unique solution if their 1 1 1
7x + 2y = 0 and Breadth = ( y + 5)
a = 12, b = α, c = α 5(3x − y + 1) = 3( 2x + y + 2)
and Here, a = 3, b = − 4
graphs intersect in one point. 2 2 1 1 1
∴ New area of rectangle
Here, a = 2, b = − 3
a 6 2 b 5 2 Since, system has unique solution, ⇒ 9x − 8 y = 1 …(i) 1 1 and a = 12, b = − 20
7. (c) Here, 1 = = , 1 = = 2 2 = ( x − 7) ( y + 5)
a b
1 ≠ 1 ⇒ α 3 Taking IInd and IIIrd terms a = 7 and b = 2 a b −4
a 9 3 b
2
15 / 2 3
2 So, ≠ 2 2
∴ 1 =
3
=
1
⇒ 1 = =
1 Given, xy = xy + 5x − 7 y − 35
a b 12 α 6 ( 2x + y + 2) = 5(3x + 2 y + 1) As the equations are homogeneous a 12 4 b −20 5
c 11 a b c 2 2 ⇒ 5x − 7 y = 35 …(ii)
and 1 = ⇒ 1 = 1 ≠ 1 ⇒ α 2 ≠ 36 ⇒ α≠± 6 ⇒ 3x + 4 y = 7 …(ii)
equations and also 2 2
c 21 a b c a b 1 1 On subtracting Eq. (ii) from Eq. (i), we
2 2 2 2
13. (a) I. 2x + 3 y = 425, 3x + 2 y = 350 On solving Eqs. (i) and (ii), we get y = 1 1 ≠ 1 . So, equation has one solution, Here ≠
4 5 get
So, the system has no solution. and x = 1 a b
Solving both the equations, we get 2 2 So, system of equatons has unique 3x − 7 y = − 21
8. (b) Let the numbers be x and 2490 − x. x = 40 and y = 115. ∴ x = y = 0 ⇒x+ y = 0 solution. 5x − 7 y = 35
65
. 13x 17. (a) Given, 3x − 5 y = −10
6.5% of x = ×x= II. When k = −2 Hence, the value of x + y is zero. x – + −
100 200 At Y-axis, x = 0 25. (a) Let the fraction be .
y − 2x = − 56
1 = 1, 1 = 2 = 1, 1 = −2
.
85 a b c 10 22. (a) Let the starting salary be ` x and
8.5% of ( 2490 − x) = ( 2490 − x) ⇒ 0 − 5 y = −10 ⇒ y = =2 x+ 1 4
100 a
2
3 b
2
6 3 c
2
5 5 annual increment be ` y.
Condition I = ∴ x = 28 m
17 a b c So, the point on Y-axis is (0, 2). By given condition, x + 4 y = 1500 ...(i) y+ 1 5 On putting x = 28 in Eq. (ii), we get
= ( 2490 − x) Q 1 = 1 ≠ 1
200 a b c 18. (a) Given, px + q = 0 and rx + s = 0 and x + 10 y = 1800 ...(ii) 5 × 28 − 7 y = 35 ⇒ − 7 y = 35 − 140
⇒ 5( x + 1) = 4 ( y + 1)
13x 17( 2490 − x) 2 2 2
−q −s
By given condition, = Hence, the given equations have no ⇒ x= and x = On subtructing Eq. (i) from Eq. (ii), we
⇒ 5x − 4 y = − 1 …(i) ∴ y = 15
200 200 p r get
solution. x −5 1 ∴ Area of rectangle
⇒ 13x = 17( 2490 − x) −q −s 6 y = 300 ⇒ y = 50 Condition II =
III. (2, 5) is not a solution of So, = ⇒ ps = qr y −5 2 = xy = 15 × 28 = 420 m 2
⇒ 13x + 17x = 42330 2x + 5 y = 13 as p r On putting y = 50 in Eq. (i), we get
42330 ⇒ 2( x − 5) = ( y − 5) 28. (b) Given, 2x + y
= 8 , 2x − y
= 8
⇒ x= = 1411 2( 2) + 5( 5) = 4 + 25 = 29 ≠ 13. 19. (c) Let the speed of the streamer in still x + 200 = 1500
30 ⇒ 2x − y = 5 …(ii) or 2x + y
= 2 2 , 2x − y
=2 2
Hence, only I is correct. water = x km/h ∴ x = ` 1300
∴ Second number = ( 2490 − x) On multiplying Eq. (ii) by 4 and or 2 x+ y
=2 3/2
,2 x− y
= 23 / 2
then speed of streamer downstream 1 1
= 2490 − 1411 = 1079 14. (b) Let cost of one horse and one cow be 23. (b) Put = A and =B subtracting from Eq. (i), we get
`x and `y, respectively. = ( x + 2) km/h x+ y x− y
On comparing both sides, we get
9. (b) As this is the case of unique solution speed of streamer upstream 5x − 4 y = − 1 x+ y =
3
...(i)
a b Condition I x + 2 y = 680 ...(i) ⇒ 44 A + 30 B = 10
1 ≠ 1 , so the graph of the equations
= ( x − 2) km/h
...(i) 8x − 4 y = 20 2
a b Condition II x = y + 80 and 55A + 40B = 13 ...(ii) − + − 3
2 2
Distance travelled by streamer and x − y = ...(ii)
will intersect at one point. ⇒ x − y = 80 ...(ii) On multiplying Eq. (i) by 4 and Eq. (ii) −3 x = − 21 2
downstream in 4h
On subtracting Eq. (ii) from Eq. (i), we by 3 and subtracting, we get ∴ x=7 On adding Eqs. (i) and (ii), we get 2x = 3
10. (d) Given equations are, = 4 ( x + 2) km 3
get 176 A + 120B = 40 Put the value of x in Eq. (i),
4x + 6 y = 11 and 2x + ky = 7 [Q distance = time × speed] ⇒ x=
( x + 2 y) − ( x − y ) = 680 − 80 ⇒ 35 − 4 y = − 1 ⇒ y = 9 2
Here, a = 4, b = 6, c = 11 165 A + 120B = 39 3
1 1 1 ⇒ 3 y = 600 and distance travelled by streamer − − − 7 Hence, the value of x is .
Hence, the fraction is .
and a = 2, b = k , c = 7 ∴ y = 200 upstream in 5 h = 5( x − 2) 11A =1 9 2
2 2 2
a 4 b 6 c 11 On putting y = 200 in Eq. (i), we get So, 4 ( x + 2) = 5 ( x − 2) ⇒ A = 1 / 11 ⇒ x + y = 11 ...(iii) 26. (a) Given equations are, 29. (c) Let A has x mangoes and B has y
Here, 1= , 1 = and 1 = mangoes.
a 2 b k c 7 x + 2 × 200 = 680 ⇒ 4x + 8 = 5x − 10 On putting the value of A in Eq. (i), we 3x + 4 y = 6 ...(i)
2 2 2
⇒ x + 400 = 680 ⇒ x = 680 − 400
get and 6x + 8 y = k ...(ii) Case I x + 30 = 2 ( y − 30)
Since, system is inconsistent, so Hence, x = 18 km/h is the speed of
streamer in still water. 4 + 30 B = 10 ⇒ 30B = 6 Here, a = 3, b = 4, c = b ⇒ x + 30 = 2 y − 60
a b c
1 = 1 ≠ 1 ⇒ 4 = 6 ∴ x = 280 1 1 1
1 a = 6, b = 8, c = k
a b c 2 k Hence, the cost of one horse is ` 280. 20. (c) Given, 0.4x + 03
. y = 17
. and ∴ B = , x− y =5 ...(iv) 2 2 2 x − 2 y = − 90 …(i)
2 2 2
0.7x − 0.2 y = 0.8 5 Since the system of equations represents
⇒ 4k = 12 ⇒ k = 3 15. (c) Let number of 10 paise coins be x and Case II ( y + 10) = 3 ( x − 10)
On adding Eqs. (iii) and (iv), we get concident line
Hence, the value of k is 3. number of 50 paise coins be y. On multiplying above equation by 10, y = 3x − 30 − 10
we get 2x = 16 a b c 3 4 6
According to the questions, ∴ 1 = 1 = 1, i.e. = = 3x − y = 40 …(ii)
11. (b) Given equations are, ∴ x=8 a b c 6 8 k
x + y = 17 4x + 3 y = 17 ...(i) 2 2 2
2x − ky = 4 and 3x + 2 y = 6 Then, …(i) On putting x = 8 in Eq. (iii), we get 48 On solving Eqs. (i) and (ii), we get
and 7x − 2 y = 8 ...(ii) ⇒ 4k = 6 × 8 ⇒ k = = 12
Here, a = 2, b = − k, c = 4 and 10x + 50 y = 450 …(ii) 8 + y = 11 ⇒ y = 3 4 x = 34, y = 62
1 1 1
and a = 3, b = 2, c = 6 On multiplying Eq. (i) by 2 and Eq. (ii)
From Eq. (ii), x + 5 y = 45 …(iii) Hence, x = 8 and y = 3. Hence, the value of k is 12.
2 2 2 by 3, we get So, A has 34 mangoes.
MATHEMATICS Linear Equations 157 158 CDS Pathfinder

30. (c) Let number of students in room A be 35. (d) Given, 2a + 3b = 17 41. (d) Since, given system of equations 46. (b) Let the share of Rajesh be R, Sonal 52. (d) Given equations are, ⇒ 9x + 6 y = 5700 …(ii)
x and in room B be y. and 2a + 2
− 3b +1
=5 x + 2 y − 3 = 0 and 5x + ky + 7 = 0 has be S and Chetan be C, respectively. 3x + y = 4 …(i) On subtracting Eq. (ii) from Eq. (i), we
So, by given condition ⇒ 2 × 2 −3 ×3 =5
a 2 b 1 no solution. According to question, 2R = 3S = 5C 6x + 2 y = 8 …(ii) get
x − 10 = y + 10 ⇒ x − y = 20 …(i) a b c also, 6 R + 6S = 150 Here, a = 3 , b = 1 and c = 4 ( 10x + 6 y ) − ( 9x + 6 y ) = 6200 − 5700
⇒ 4 ⋅ 2a − 3 ⋅ 3b = 5 Then, 1 = 1 ≠ 1 1 1 1
3 a = 6 , b = 2 and c = 8
and x + 20 = 2 ( y − 20) Let 2a = x and 3b = y
a
2
b
2
c
2 ⇒ × 6 S + 6 S = 150 and
2 2 2 ⇒ x = 500
2 a b c
x − 2 y = − 60 …(ii) 1 2 −3 1 = 1 = 1 = 1 ∴ x = `500
Then, x + y = 17 …(i) ∴ = ≠ ⇒ k = 10 ⇒ 9S + 6S = 150 ⇒ S = 10 Q
On subtracting Eq. (ii) from Eq. (i), we 5 k 7 a b c 2 From Eq. (i), 5000 + 6 y = 6200
4x − 3 y = 5 3 3 2 2 2
…(ii) ∴ C = S = × 10 = ` 6.
get 5 5 So, the system of equations has ⇒ 6 y = 1200 ⇒ y = ` 200
On multiplying Eq. (i) by 3 and adding 42. (a) Given, x − y = 0.9 …(i)
( x − y ) − ( x − 2 y ) = 20 + 60 ⇒ y = 80 infinitely many solutions.
to Eq. (ii), we get and 11( x + y )−1 = 2 …(ii) 47. (a) R + C + S = 155 ∴ Cost of 4 chairs and 5 tables
Hence, number of students in room 2 2 53. (c) Let the two numbers be x and y,
3x + 3 y = 51 ⇒ 2x + 2 y = 11 R + R + R = 155 = 4x + 5 y = 4 × 500 + 5 × 200
B is 80. 5 3 respectively.
4x − 3 y = 5 Given, sum of two numbers = 7 = 2000 + 1000 = `3000
31. (d) Let the number of passengers in the On multiplying Eq. (i) by 2 and adding 15R + 6R + 10R
7x = 56 = 155 ⇒ R = ` 75 ⇒ x+ y =7 …(i) 57. (b) Let bus starts with x number of
starting be x. Eqs. (i) and (ii), we get 15
∴ x =8 and sum of their squares = 25 passengers.
Number of passengers after first halt 4x = 128
. ⇒ x = 3.2 48. (c) The graph of ax + by = c,
On putting the value of x in Eq. (i), we ⇒ x + y = 25
2 2
dx + ey = f will be coincident, if the …(ii) After 1st stoppage, number of passengers
=  x −  + 120 = x + 120
x 2 get From Eq. (i) y = 3.2 − 0.9 = 2.3
system has infinite number of solutions. Now, we have x 5x − x + 200
 3  3 8 + y = 17 ⇒ y = 9 = x − + 40 =
43. (d) Let Pooja’s initial salary be ` x and So, statement II is false. ( x + y ) 2 = ( x 2 + y 2 ) + 2 xy 5 5
and number of passengers after second Now, 2a = x ⇒ 2a = 8 = ( 2)3 4x + 200
halt ∴ a = 3 and 3b = y = 9 ⇒ 3b = 32
fixed increment every year be ` y. Thus, statements I and III are correct. ⇒ ( x + y ) 2 − ( x 2 + y 2 ) = 2 xy =
5
=  x + 120 + 100
1 2 By given condition, x + 3 y = 4200 …(i) 49. (c) Let the two numbers be x and y, 1
∴ b=2 ⇒ xy = [( x + y )2 − ( x2 + y 2 )]
2  3 
Hence, a = 3 and b = 2
respectively. According to the question, After 2nd stoppage, number of passengers
and x + 8 y = 6800…(ii) 2 4x + 200 4x + 200
x + y = 20 …(i) 1 − + 30
But number of passengers after second = [( 7)2 − ( 25)] 5 5×2
36. (b) Let smaller number be x. On subtracting Eq. (ii) from Eq. (i), we
halt = 240 and xy = 75 …(ii) 2
and larger number = 80 − x get According to question,
1 2 1 1 y+x 20 4
∴ x + 120 + 100 = 240 By given condition, 80 − x = 4x + 5
( x + 3 y ) − ( x + 8 y ) = 4200 − 6800 ⇒ + = = = [from Eqs. (i) and (ii)] ⇒
4x + 200 4x + 200
− + 30 = 70
2  3 
⇒ −5 y = −2600 ⇒ y = `520
x y xy 75 15
1 1 5 10
2 2 ⇒ 5x = 75 ⇒ x = 15 = ( 49 − 25) = × 24 = 12 4x + 200  1
On putting y = 520 in Eq. (i), we get [from Eqs. (i) and (ii)]
⇒ x + 120 = 280 ⇒ x = 160 2 2  1 −  + 30 = 70
3 3 Hence, the smaller number is 15. Hence, the product of the two numbers 5  2
x = ` 2640 50. (b) Given equations are,
4x + 200 1
⇒ x = 240 37. (c) Given equations are, 3x+ y = 81 ⇒ 3x + y = 34 is 12. ⇒ × = 40
2 3 9 44. (b) Let the fare of ticket from station P 5 2
32. (c) Given system of equations are + = ⇒ 2 y + 3x = 9 …(i) ⇒ x+ y =4 …(i) Solutions (Q. Nos. 54-55) Let the digit in
to station Q is ` x and that from station ⇒ 4x + 200 = 400 ⇒ 4x = 200
x + 2 y = 3 and 3x + 6 y = 9
x y xy
P to station R is ` y. and 81x − y = 3 ⇒ (34 )x − y = 31 unit’s place by y and that in ten’s place
200
4 9 21 1 be x . ∴ x= = 50
a b c
1 = 1 = 1 = 1.
and + = ⇒ 4 y + 9x = 21 …(ii) By given condition, x + y = 42 ⇒ x− y = …(ii) Then, the two-digit number is given by 4
Here, x y xy 4
a b c 3 and 5x + 10 y = 350 10x + y. Hence, the number of passengers is 50.
2 2 2 On solving Eqs. (i) and (ii), we get On adding Eqs. (i) and (ii), we get
Hence, given system of the equation has On solving Eqs. (i) and (ii), we get Number obtain by reversing the order of 58. (d) We have,x + y − 7 = 0
x = 1 and y = 3 ⇒ 2x =
17
⇒ x=
17
digit = 10 y + x
infinitely many solutions. x = 14 and y = 28 ⇒ x+ y =7 …(i)
∴ x + y = 1+ 3 = 4 4 8
33. (b) Let x be the ten’s digit and y be the Hence, fare from station P to station Q Now, by given condition, and 3x + y − 13 = 0
38. (b) Since, the equation kx − y = 2 and 51. (d) Given equations are, x + y = 10 …(i) ⇒ 3x + y = 13 …(ii)
unit's digit of two-digit number. is ` 14. a b x
6x − 2 y = 3 have a unique solution − = …(i)
By given condition, x + y = 8 45. (a) Let x represent school bags, y and ( x + 10 y ) + 36 = ( y + 10x) On subtracting Eq. (i) from Eq. (ii), we
k 1 b a y
∴ ≠ ⇒ k ≠3 ⇒ − 9 y + 9x = 36 get
and ( 10x + y ) − ( 10 y + x) = 18 …(i) 6 2 represent sweaters, z represent books.
and
a b
+ =x− y …(ii) 3x + y = 13
System of equations: b a ⇒ x− y =4 ...(ii)
⇒ 9x − 9 y = 18 ⇒ x − y = 2 …(ii) 39. (b) Three lines a x + b y + c = 0 x+ y =7
1 1 1 x + y + z = 200 …(i) From Eqs. (i) and (ii), On adding Eqs. (i) and (ii), we get − − −
On adding Eqs. (i) and (ii), we get a x+b y+c =0  a − bx − x
2 2 2 z=x+ y …(ii) 2x = 14, x = 7 2x = 6
2x = 10 ⇒ x = 5 a x+b y+c =0  
and a b x   ∴ x=3
3 3 3 20x + 25 y + 5z = 2800 …(iii) + =x− = b a On putting the value of x in Eq. (i), we
On putting x = 5 in Eq. (i), we get which are non-parallel and non-collinear b a  − 
a b a b get
putting z = x + y in Eq. (i)   − On putting the value of x in Eq. (i), we
⇒ y = 8 − 5 = 3 ⇒ x = 5 and y = 3 they have only one solution if they meet  b a b a 7 + y = 10 ⇒ y = 3 get
in a common point in this case these ( x + y ) + x + y = 200 , 2x + 2 y = 200 3+ y =7 ⇒ y =4
∴ Required difference of digits,
⇒  +   −  = x − − 1
a b a b a b 54. (b) The required number is 73.
lines are called concurrent lines. x + y = 100 , x = 100 − y
x− y =5−3= 2  b a  b a b a  So, the number is a multiple of a prime Now, 4x2 + y 2 + 4xy = ( 2x + y )2
40. (c) Let the unit’s digit be x and ten’s putting x = 100 − y in Eq. (iii)  a2 b2   a 2 − b 2 − ab  number. = ( 2 × 3 + 4)2
34. (a) Given, ( x, y ) = ( 4, 1) and ⇒  2 − 2 = x 
digit’s be y. Then the two-digit number 20 ( 100 − y ) + 25 y + 5 b a   ab  55. (a) Required product of two digits = ( 6 + 4)2
mx + y = 2x + ny = 5 = 10 y + x [( 100 − y ) + y ] = 2800 ab  a4 − b4  = 3 × 7 = 21 = 102 = 100
On putting x = 4 and y = 1, we get By given condition, x + y = 10 …(i) ⇒ x= 2 × 2 2 
2000 − 20 y + 25 y + 500 = 2800 ( a − b − ab )  a b 
2 x y 3x + 2 y
∴ m( 4) + 1 = 2 × 4 + n = 5 and 10 y + x − 18 = 10x + y
56. (a) Let the cost of one chair and one 59. (b) Given, + =4⇒ =4
5 y = 300 ⇒ y = 60 ( a4 − b4 ) 1 table be ` x and ` y, respectively. 2 3 6
I II III ⇒ 9x − 9 y = − 18 ⇒ x − y = − 2 ⇒ x= 2 ⋅ ⇒ 3x + 2 y = 24 …(i)
∴ 4m + 1 = 5 and 8 + n = 5 x = 100 − 60 ⇒ x = 40 a − b 2 − ab ab By given condition,
On solving Eqs. (i) and (ii), we get 2 3 2 y + 3x
⇒ n = − 3 and m = 1 z = 40 + 60 ⇒ z = 100 ( a − b )( a + b )( a 2 + b 2 ) 10x + 6 y = 6200 …(i) and + = 1⇒ =1
x = 4 and y = 6 = x y xy
∴ m + n = 1−3 = −2 ∴ Required product = xy = 4 × 6 = 24 40 school bags, 60 sweaters, 100 books ab( a 2 − b 2 − ab ) and 3x + 2 y = 1900
⇒ 2 y + 3x = xy …(ii)
18
MATHEMATICS Linear Equations 159 160 CDS Pathfinder

p q q p
On comparing Eqs. (i) and (ii), we get 63. (a) Let the cost of one chair be ` x and 68. (c) Given, + = m and + = h
xy = 24 that of one table be ` y. x y x y
1 1
There are 8 possibilities for x and y, Then, 2x + y = 700 …(i) Let = u and = v
respectively. x y
and x + 2 y = 800 …(ii)
1 × 24 = 24 6 × 4 = 24 Then, pu + qv = m …(i)
On multiplying Eq. (ii) by 2 and
2 × 12 = 24, 8 × 3 = 24 and qu + pv = n …(ii)
subtracting from Eq. (i), we get
3 × 8 = 24, 12 × 2 = 24 2x + y = 700
On solving Eqs. (i) and (ii), we get
4 × 6 = 24, 24 × 1 = 24 mp − nq mq − np
2x + 4 y = 1600 u= 2 and v = 2
On putting all the values of x and y in − − − p − q2 q − p2
given equations we get that only x = 4
and y = 6 satisfy the equations.
∴ x + y = 4 + 6 = 10
− 3 y = − 900 ⇒ y = 300
On putting y = 300 in Eq. (ii), we get
x + 2 × 300 = 800

x 1 / u v −(mq − np )
=
y 1/ v u
= =

=
mp − nq
np − mq
QUADRATIC EQUATIONS
60. (a) Given equations are, x = 800 − 600 = 200 mp − nq
x + y =5
y + z = 10
z + x = 15
from Eq. (i), we have y = 5 − x
…(i)
…(ii)
…(iii)
Since, m chairs and m tables are to be
purchased for ` 30000.
∴Cost of m table and m chairs = 30000
69. (c) Given equations are,
3x − ky − 20 = 0 …(i)
6x − 10 y + 40 = 0 …(ii)
AND INEQUALITIES
put the value of x in Eq. (ii) , ∴ 200m + 300m = 30000
since, the given system has no solution
z + 5 − x = 10 30000
⇒ m= ⇒ m = 60 a b c
∴ 1 = 1 ≠ 1 ⇒ 3 = k ⇒k =5
⇒ z−x =5 …(iv) 500
a b c 6 10
Add Eq. (iv) and Eq. (iii) 64. (c) Let the two-digit number be 10x + y, 2 2 2
Regularly (9-10) questions have been asked from this chapter. Questions from this section
(z + x ) + (z − x ) = 15 + 5 Condition I 70. (b) Let the ages of three brothers be a, b
⇒ 2z = 20 ⇒ z = 10 According to the question, and c. generally focus on finding roots of quadratic equation and their factorisation.
from Eq. (iii), we have 10 + x = 15 10x + y = 3( x + y )
⇒ x =5 Then, a + b = 4, b + c = 6
⇒ 10x + y − 3x − 3 y = 0
and hence y = 5 − x = 5 − 5 = 0 and c+ a =8
⇒ 7x − 2 y = 0 …(i)
∴ correct sequence is z > x > y Condition II On solving these three equations, we get
⇒ 10x + y + 45 − 10 y − x = 0 a = 3, b = 1 and c = 5 QUADRATIC EQUATION
61. (c) Let present age of Ravi be x yr.
⇒ 9x − 9 y + 45 = 0 ∴ Age difference between eldest and
∴ Present age of Ravi’s father = 4x yr A quadratic equation is an equation whose degree is 2, meaning that the highest exponent of variable is 2.
⇒ x − y = − 5 …(ii) youngest = 5 − 1 = 4 yr
Now, 5 yr ago, age of Ravi’s father
On solving Eqs. (i) and (ii), we get 71. (a) Let annual incomes of two persons The general form of a quadratic equation is ax 2 + bx + c = 0, where a, b and c are real numbers and a ≠ 0.
= ( 4x − 5) yr
x = 2 and y = 7 be 9x and 7x and expenses be 4 y and 3 y,
e.g. 2x 2 + 3x + 5 = 0, x 2 + x + 1 = 0 and 5x 2 + 6x + 8 = 0 are all quadratic equations.
and age of Ravi = ( x − 5) yr respectively.
∴ Sum of the squares of digits
According to the question, Then, according to the question,
4x − 5 = 7( x − 5)
= ( 2)2 + ( 7)2 = 4 + 49 = 53 Roots of a Quadratic Equation
9x − 4 y = 2000 …(i)
⇒ 4x − 5 = 7x − 35 ⇒ 3x = 30 65. (a) Let x be the required number. A real number α is said to be a root of the quadratic equation ax 2 + bx + c = 0 ( a ≠ 0 ) , if it satisfies the
and 7x − 3 y = 2000 …(ii)
∴ x = 10 ∴ 52x − 25x = 324 equation i.e. aα 2 + bα + c = 0. We can also say that x = α is a solution of the quadratic equation.
324 From Eqs. (i) and (ii), we get
∴ Present age of Ravi = x = 10 yr ⇒ 27x = 324 ⇒ x = = 12 e.g. 2 is the root of the quadratic equation, x 2 − 6x + 8 = 0 since ( 2) 2 − 6( 2) + 8 = 0.
27 9x − 4 y = 7x − 3 y ⇒ y = 2x
Present age of Ravi’s father On putting the value of y in Eq. (i), we
Hence, the required number is 12. Note A quadratic equation has exactly two roots.
= 4x = 4 × 10 = 40 yr get If α be the root of the quadratic equation ax 2 + bx + c = 0, then ( x − α ) is the factor of ax 2 + bx + c.
66. (b) Let a tin of oil contain x bottles of 9x − 8x = 2000 ⇒ x = ` 2000
62. (d) Let the two positive numbers be x oil.
and y, respectively. According to the ∴ Difference between their annual Solutions of Quadratic Equations
According to the question, incomes = 9x − 7x = 2 x = ` 4000
question,
4 6
− + =
4 3

4 3 6 4
− = − Solutions of quadratic equations can be determined by using any of the following two methods
( x + y ) = 25
. (x − y) 72. (c) Let the unit’s place number be y and
5 x x 4 5 4 x x
⇒ x + y = 25
. x − 25
. y 1 2
ten’s place number be x. 1. By Factorisation
x 7 7 ⇒ = ⇒ x = 40 Then, number = 10x + y
⇒ 35
. y = 15
. x ⇒ = or x = y 20 x In this method, the middle term of the quadratic equation is splitted and the given quadratic equation
y 3 3 Now, after interchanging the digits,
∴ A tin contains 40 bottles of oil. is converted into a product of two linear factors. Then, the roots of the equation are obtained by
Now, product of numbers xy = 84 New number = 10 y + x and sum of equating each factor equal to zero.
67. (c) Let the number be 10x + y .
7 84 × 3 digits = x + y
⇒ y × y = 84 ⇒ y 2 = We have, x + y = 7 …(i) Let the quadratic equation be ax 2 + bx + c = 0 and its linear factors are ( px + q) and (rx + s ) , then
3 7 According to the question,
and 10 y + x = 10x + y + 27 ax 2 + bx + c = ( px + q)(rx + s )
⇒ y 2 = 12 × 3 10x + y = k( x + y ) …(i)
⇒ 9 y − 9x = 27 ⇒ y − x = 3 …(ii) −q −s
7 and 10 y + x = m( x + y ) …(ii) Now, ax 2 + bx + c = 0 ⇒ ( px + q)(rx + s ) = 0 ⇒ px + q = 0 or rx + s = 0 ⇒ x = or x=
∴ y = 6 ⇒ x = × 6 = 14 Solving Eqs. (i) and (ii), we get y = 5
3 and x = 2 On adding Eqs. (i) and (ii), we get p r
∴ Sum of numbers = x + y 11 ( x + y ) = ( k + m ) ( x + y ) −q −s
∴ Number is 25. Thus, and are the roots of the equation, ax 2 + bx + c = 0.
= 14 + 6 = 20 Product of number = 2 × 5 = 10 ⇒ k + m = 11 ⇒ m = 11 − k p r
MATHEMATICS Quadratic Equations and Inequalities 161 162 CDS Pathfinder

EXAMPLE 1. Solve the equation x 2 − 7 x + 12 = 0, and Nature of Roots of Quadratic Equation EXAMPLE 5. What is the least integral value of k for ( a2 − b 2 ) 2 a2 − b 2
⇒ α −β = = ...(ii)
find the value of x. 2 which the equation x 2 − 2(k −1) x + (2k + 1) = 0 has both ( a2 b 2 ) 2 a2 b 2
Let D = b − 4ac be the discriminant of the quadratic
a. −3, − 4 b. −3, 4 c. 3, − 4 d. 3, 4 the roots positive? On solving Eqs. (i) and (ii), we get
equation, ax 2 + bx + c = 0, where a ≠ 0. Then, the 1
b. − 1 1
Sol. d. Given, x2 − 7x + 12 = 0 following cases arise
a. 1 c. 4 d. 0 α = 2 and β = 2
2 b a
x2 − 3x − 4x + 12 = 0 [splitting the middle term]
(i) If D > 0, then the two roots are real and distinct. Sol. a. Both the roots of the equation ax2 + bx + c = 0 are
⇒ x ( x − 3) − 4 ( x − 3) = 0 x2 x 1
(ii) If D = 0, then the two roots are real and equal and are b c EXAMPLE 7. If one root of the equation + + =0
⇒ ( x − 3) ( x − 4) = 0 −b positive, if − > 0 and > 0 a b c
⇒ ( x − 3) = 0 or ( x − 4) = 0 ⇒ x = 3 or x = 4 given by α = β = . a a
2a is reciprocal to the other, then which one of the
Given equation is x2 − 2 ( k − 1) x + ( 2k + 1) = 0.
So, x = 3, 4 are roots of the given equation. (iii) If D < 0, then there are no real roots, i.e. given following is correct?
b 2( k − 1) a. a = b b. b = c c. ac = 1 d. a = c
equation has imaginary roots. Now, − = ⇒ 2( k − 1) > 0, if k > 1
EXAMPLE 2. Solve the equation a 1
3a 2 x 2 + 8abx + 4b 2 = 0, a ≠ 0, and find the value of x. c 2k + 1 x2 x 1 ...(i)
and = ⇒ ( 2k + 1) > 0, if k > −
1 Sol. d. Given, + + =0
2b 2b − 2b 2b − 2b − 2b 2b −2b a 1 2 a b c
a. , b. , c. , d. , IMPORTANT POINTS
3a a 3a a 3a a 3a a ∴ k >1 Now, let one root be α, then the other root is 1/ α.
1. If a , b , c ∈Q and D is a perfect square, then equation
We know that,
Sol. c. The equation is 3a2x2 + 8abx + 4b2 = 0 has rational roots. Hence, the least value k as per given in options is 1. Constant term 1 /c
Product of roots = =
2. If one roots of quadration equation is p + q , then Coefficient of x2 1 / a
⇒ 3a x + 2abx + 6abx + 4b = 0
2 2 2
other root will be p − q . Relation between Roots and ⇒
1 a
α. = ⇒ c=a
⇒ ax ( 3ax + 2b) + 2b ( 3ax + 2b) = 0
Coefficients α c
∴ ( 3ax + 2b) ( ax + 2b) = 0 EXAMPLE 4. If the equation x 2 + 2(1 + k )x + k 2 = 0 which is the required relation.
⇒ 3ax + 2b = 0 or ax + 2b = 0
has equal roots, then what is the value of k?
1. Quadratic Equation
− 2b − 2b Let α, β be the roots of the equation ax 2 + bx + c = 0 Symmetric Functions of the Roots
∴ x= or a.
1
b. −
1
3a a 2 2 ( a ≠ 0 ) , then Let α and β be the roots of a quadratic equation. An
2. By Using the Quadratic Formula c. 1 d. − 1 b − Coeff icient of x expression in α and β which remains same when α and β
Sum of the roots = α + β = − = are interchanged is known as a symmetric function in α
Let the quadratic equation be ax 2 + bx + c = 0, where Sol. b. Given equation, x2 + 2(1 + k ) x + k 2 = 0. a Coeff icient of x 2
and β. To evaluate the value of symmetric function of the
a ≠ 0. Then, solution of this equation can be find using If it has equal roots, then D = 0 c Constant term roots, express the given function in terms of (α + β ) and
the formula. Product of the roots = αβ = = αβ.
⇒ {2 (1 + k )}2 − 4k 2 = 0 a Coeff icient of x 2
− b ± b 2 − 4ac You can use the following results.
x= ⇒ 4(1 + k 2 + 2k ) − 4k 2 = 0
2a 2. Cubic Equation • (α 2 + β 2 ) = [(α + β ) 2 − 2αβ ]
• If α and β be considered as roots of the quadratic
⇒ 4 + 4k 2 + 8k − 4k 2 = 0
4 If α, β, γ are the roots of the cubic equation • (α 3 + β 3 ) = [(α + β) 3 − 3αβ (α + β )]
equation, then ⇒ 4 + 8k = 0 ⇒ k = −
8 ax 3 + bx 2 + cx + d = 0, a ≠ 0, then
− b + b 2 − 4ac − b − b 2 − 4ac 1
• (α − β ) 2 = [(α + β ) 2 − 4αβ ]
α= and β = ∴ k=− Sum of roots = α + β + γ = − b / a
2a 2a 2 • (α 3 − β 3 ) = [(α − β ) 3 + 3αβ (α − β )]
• The quantity b 2 − 4ac is called the discriminant of the 1 Product of roots two at a time = αβ + βγ + γα = c / a
Hence, the value of k is − . d
quadratic equation ax 2 + bx + c = 0 and is denoted by D. 2 Product of three roots = αβγ = − EXAMPLE 8. If α and β are the roots of the equation
a x 2 − 6 x + 6 = 0, what is α 3 + β 3 + α 2 + β 2 + α + β equal
So, D = b 2 − 4ac.
Roots Under Particular Conditions to?
EXAMPLE 6. What are the roots of the quadratic
EXAMPLE 3. Solve the equation x 2 − 9x + 18 = 0 and Consider the quadratic equation ax 2 + bx + c = 0. equation a 2b 2 x 2 − (a 2 + b 2 ) x + 1 = 0? a. 150 b. 138 c. 128 d. 124
find the value of x. (i) Both roots are positive, if a and b are opposite in − Coefficient of x
1 1 1 1 1 1 1 1 Sol. b. Here, α + β = =6
a. 3, 6 b. −3, − 6 c. −3, 6 d. 3,− 6 −b c a. , b. − ,− c. ,− d. − , Coefficient of x2
sign and a and c are same in sign, i.e. > 0 and > 0. a2 b2 a2 b2 a2 b2 a2 b2
a a Constant term
Sol. a. We have, x2 − 9x + 18 = 0 Sol. a. Let roots of the equation a b x − ( a + b ) x + 1 = 0
2 2 2 2 2 and αβ= =6
(ii) Both roots are negative, if a, b and c are of same sign. Coefficient of x2
Here, a = 1, b = − 9 and c = 18 be α and β.
b c ∴ (α + β) 2 = 62 ⇒ α 2 + β 2 + 2αβ = 36
−b± b2 − 4ac − ( − 9) ± ( − 9) 2 − 4 (1) (18) i.e. > 0 and > 0. a2 + b2
∴ x= = a a ∴ Sum of roots = α + β = ...(i) ⇒ α 2 + β 2 = 36 − 2 (6 ) = 24
2 2
2a 2 (1) a b
(iii) Roots are of opposite signs, if a and c are of opposite
9 ± 81 − 72
9± 9 9± 3 −a 1 ∴ (α + β ) + (α + β ) + (α + β)
3 3 2 2

= = = sign, i.e. > 0. and product of roots = αβ = 2 2


2 2 2 a b = (α + β) (α 2 + β 2 − αβ) + (α 2 + β 2) + (α + β)
c
9+ 3 9−3 2 2
∴ x= or x = ⇒ x = 6 or 3 (iv) Roots are equal but opposite in signs, if b = 0.  a2 + b  4 = 6 ( 24 − 6) + ( 24) + (6)
2 2 Now, α − β = (α + β) 2 − 4 αβ =  2 2  − 2 2
 a b  = 6 (18) + 30 = 108 + 30 = 138
Hence, the values of x are 6 and 3. (v) Roots are reciprocal to each other, if a = c. a b
MATHEMATICS Quadratic Equations and Inequalities 163 164 CDS Pathfinder

Formation of a Quadratic Equation EXAMPLE 11. Solve 2x −


3
= 5. Then, the value of x is EXAMPLE 14. Solve 11y − 6 + y − 1 − 4y + 5 = 0. Now,
x Then, the value of y is x2 + 5x = − 8 x2 + 5x = − 2
If α, β are the roots of a quadratic equation, then the
a. 1/3, −3 b. 1/2, −3 c. −1/2, 3 d. 1/2, 4 a. {5, 6/5} b. {1, 2} ⇒ x2 + 5x + 8 = 0 ⇒ x2 + 5x + 2 = 0
equation is ( x − α )( x − β ) = 0 ⇒ x 2 − (α + β ) x + αβ = 0 c. {3, 5/2} d. None of these
3
Sol. c. Here, 2x − =5 ...(i) − 5 ± 25 − 32 − 5 ± 25 − 8
i.e. x 2 − (sum of the roots) x + product of the roots = 0 x Sol. a. Given, 11y − 6 + y − 1 − 4y + 5 = 0 ⇒ x= ⇒ x=
On multiplying Eq. (i) by x ⇒ 2x2 − 3 = 5x 2 2
EXAMPLE 9. The quadratic equation whose roots are 11y − 6 + y − 1 = 4y + 5 − 5± −7 − 5 ± 17
⇒ 2x2 − 5x − 3 = 0 ⇒ 2x2 − 6x + x − 3 = 0 ∴ x= ∴ x=
3 and −1, is On squaring both sides, we get 2 2
⇒ ( 2x + 1) ( x − 3) = 0 ⇒ x = − 1/ 2 or x = 3
a. x 2 − 4 x + 3 = 0 b. x 2 − 2x − 3 = 0 −1 [ 11y − 6 + y − 1]2 = [ 4y + 5]2
Hence, the number of real roots of the given equation is 2.
c. x 2 + 2x − 3 = 0 d. x 2 + 4 x + 3 = 0 Hence, the solution is x = , 3.
2  1  1
⇒(11y − 6) + ( y − 1) + 2 (11y − 6) ( y − 1) = ( 4y + 5) Rule 6 (i) a  x 2 +  + b  x +  + c = 0
Sol. b. Given that, the roots of the quadratic equation are  x2   x
3 and – 1.
Rule 3 x + b + x = c or a − x 2 = bx + c ⇒ 12y − 7 + 2 11y − 17y + 6 = 4y + 5
2
 1  1
Let α = 3 and β = −1 (ii) a  x 2 +  + b  x −  + c = 0
• Square both sides to get a quadratic equation without ⇒ 2 11y 2 − 17y + 6 = 12 − 8y  x2   x
∴ Sum of roots = α + β = 3 − 1 = 2
the radical.
and product of roots = αβ = ( 3)( −1) = −3 ⇒ 11y 2 − 17y + 6 = 6 − 4y To solve an equation of the form (i) .
• Solve the equation by factorisation or by quadratic  1
So, required quadratic equation is formula. Again, squaring both sides, we get put  x +  = y and to solve an equation of
x 2− (α + β) x + αβ = 0  x
11y 2 − 17y + 6 = (6 − 4y) 2
⇒ x 2− ( 2 ) x + ( −3 ) = 0
EXAMPLE 12. Solve 2x + 9 + x = 13, then the value  1
the form (ii), put  x −  = y
of x is ⇒ 11y 2 − 17y + 6 = 36 + 16y 2 − 48y  x
⇒ x 2− 2x − 3 = 0 d. −4 , − 10
a. 4, 10 b. 8, 20 c. 3, 15 ⇒ 5y 2 − 31y + 30 = 0
 1 
EXAMPLE 16. Solve 4  x 2 + 2  − 4  x +  − 7 = 0;
1
Equations Reducible to Quadratic Equations Here, a = 5, b = − 31and c = 30 
Sol. b. Given, 2x + 9 + x = 13 ⇒ 2x + 9 = 13 − x  x  x
The equations which at the outset are not quadratic On squaring both sides, we get ( 2x + 9) 2 = (13 − x ) 2 31 ± ( − 31) − 4 ( 5) ( 30) 
2
− b ± b − 4ac  2
x ≠ 0 and find the real roots of x.
⇒ y= Q y = 
equations, but can be reduced to quadratic equations by 2× 5  2a  a. {−2, −1/2} b. {0, 2}
suitable substitutions or simplifications are called ⇒ 2x + 9 = 169 + x2 − 26x
31 ± 361 31 ± 19 c. {2, 1/2} d. None of these
equations reducible to quadratic equations. ⇒ x 2 − 28x + 160 = 0 ⇒ x 2 − 20x − 8x + 160 = 0 ⇒ y= ⇒y =
Sol. c. 4 x 2 +
1
 − 4  x +  − 7 = 0
10 10 1
⇒ ( x − 8) ( x − 20) = 0 ⇒ x = 8 or x = 20 …(i)
Rule 1 ax 2n + bx n + c = 0, where n ≥ 2 Hence, the solution is x = 8, 20. 31 + 19 31 − 19  x 2  x
⇒ y= and y = 1
Use substitution x n = y 10 10 Put x+ =y
Rule 4 ax + b ± cx + d = e x
y = 5,  is the required solution.
6
⇒ x 2n = y 2 and equation reduces to ay 2 + by + c = 0. ∴ On squaring both sides, we get
or ax + b ± cx + d ± ex + f = 0  5 2
 x +  = y 2 ⇒ x2 + 2 + 2 = y 2 ⇒ x 2 + 2 = y 2 − 2
1 1 1
Rule 5 ( x + a) ( x + b) ( x + c ) ( x + d ) + k = 0, where 
EXAMPLE 10. Solve the quadratic equation To solve such type of equations, follow the steps given x x x
x 4 − 26 x 2 + 25 = 0, and find the value of x. below. a + b = c + d and k may or may not be zero. ⇒ 4 ( y 2 − 2) − 4y − 7 = 0 [from Eq. (i)]
a. ± 1, ± 25 b. ± 1, ± 5 • Square both sides once, so that only one term To solve such type of equations, put x 2 + ( a + b)x = T . ⇒ 4y 2 − 8 − 4y − 7 = 0 ⇒ 4y 2 − 4y − 15 = 0
c. 1, 5 d. 1, 25 containing radical is obtained.
• Keep the term containing radical on one side and all EXAMPLE 15. Solve the equation ⇒ 4y 2 − 10y + 6y − 15 = 0 ⇒ 2y( 2y − 5) + 3( 2y − 5) = 0
Sol. b. Put x2 = z ⇒ x4 = z 2 other terms on the other side. (x + 1) (x + 2) (x + 3) (x + 4) − 8 = 0, and find the number 5 3
⇒ ( 2y − 5) ( 2y + 3) = 0 ⇒ y = or −
x4 − 26x2 + 25 = 0 can be written as • Square again to get, the quadratic equation and solve it. of real roots of this equation. 2 2
5 3
a. 1 b. 2 c. 4 d. No real roots Now, For y = For y = −
z 2 − 26z + 25 = 0 ⇒ z 2 − 25z − z + 25 = 0 EXAMPLE 13. Solve the following equation 2 2
⇒ z ( z − 25) − 1( z − 25) = 0 4 − x + x + 9 = 5. Then, the value of x is Sol. b. ( x + 1) ( x + 2) ( x + 3) ( x + 4 ) − 8 = 0 ⇒ x+ =
1 5
⇒ x+ =−
1 3
x 2 x 2
⇒ ( z − 1) ( z − 25) = 0 ⇒ z = 1or z = 25 a. 1, 5 b. −1, 3 c. 0 , − 5 d. 1, 4 Here, 1 + 4 = 2 + 3,
x2 + 1 5 x +1
2
3
when z = 1⇒ x 2 = 1 ⇒ x = ± 1 so, [( x + 1) ( x + 4 )][( x + 2) ( x + 3)] − 8 = 0 ⇒ = ⇒ =−
Sol. c. Given, 4 − x + x + 9 = 5 x 2 x 2
when z = 25 ⇒ x2 = 25 ⇒ x = ± 5 [ x2 + 5x + 4][ x2 + 5x + 6] − 8 = 0
On squaring both sides, we get ( 4 − x + x + 9) 2 = 25 ⇒ 2x 2 + 2 = 5x ⇒ 2x + 2 = − 3x
2

Hence, the solutions are ± 1 and ± 5. Put x2 + 5x = t, so the equation becomes


⇒ ( 4 − x) + ( x + 9) + 2 ( 4 − x x + 9) = 25 ⇒ 2x 2 − 5x + 2 = 0 ⇒ 2x2 + 3x + 2 = 0
Q ( t + 4) ( t + 6) − 8 = 0
Rule 2 Px + =R ⇒ 2 ( 4 − x) ( x + 9) = 12 ⇒ ( 4 − x) ( x + 9) = 6 ⇒ ( x − 2) ( 2x − 1) = 0 Here, D = 9 − 16 = −7 < 0
x ⇒ t 2 + 10 t + 24 − 8 = 0 ⇒ t 2 + 10 t + 16 = 0 1
Again, squaring both sides, we get ∴ x = 2 or x = So, there are no real roots.
Reduce the given equation to a quadratic equation by ⇒ ( 4 − x ) ( x + 9 ) = 36 ⇒ − x 2 + 36 − 5x = 36 ⇒ t 2 + 8 t + 2t + 16 = 0 2
multiplying both sides by ‘x’. So, Px 2 + Q = Rx ⇒ t (t + 8) + 2 (t + 8) = 0 ⇒ (t + 8) (t + 2) = 0 [QD = b2 − 4ac ]
⇒ x2 + 5x = 0 ⇒ x ( x + 5) = 0 ⇒ x = 0 or x = − 5 1
or Px 2 − Rx + Q = 0 ⇒ t = − 8 or − 2 Hence, the values of x are 2 and .
∴ x = 0, − 5 is the required solution. 2
MATHEMATICS Quadratic Equations and Inequalities 165 166 CDS Pathfinder

(iii) Sometimes it may happen that, out of the roots of Clearly, here (0, 0) does not Y
Rule 7 ax 4 + bx 3 + cx 2 + bx + a = 0 Linear inequality in one variable
the quadratic equation only one satisfies the given satisfies the inequality, so we (0, 2)
A linear inequality which has only one variable is called shade the plane which does not
• To solve such type of equations, we divide the equation condition in the problem. So, the root which does 0)
linear inequality in one variable. contain (0, 0). (3,
by x 2 , to obtain a( x 2 + 1 / x 2 ) + b ( x + 1 / x) + c = 0, and, not satisfy the condition of the problem will be
 − b Also, since x ≥ 0 and y ≥ 0, so the X´ X
1
then put x + = y
rejected. e.g. ax + b < 0, where a ≠   , 4x + 7 ≥ 0 solution set must be in Ist
 a
x quadrant only.
EXAMPLE 18. A positive number, when increased by
EXAMPLE 17. If x 4 − 3x 3 − 2x 2 + 3x + 1 = 0, x ≠ 0, 10 equals 200 times its reciprocal. What is number?
Linear inequality in two variables Hence, the shaded region Y´
represents the solution set of the given system of
then the roots of the equation is a. 100 b. 10 c. 20 d. 200 A linear inequality which has two variables, is called linear inequalities.
 −3 ± 13   3 ± 13  inequality in two variables.
a. ± 2,  b. ± 1,  Sol. b. Let the positive number be x. Note (i) If the inequality is strict ( > or <), graph a dashed line. If the
 2   2  e.g. 3x + 11y ≤ 0, 4t + 3y > 0
Then, according to the question, inequality is not strict ( ≥ or ≤), graph a solid line.
 3 ± 11  200
c. ± 3,
2 
 d. None of these x + 10 = ⇒ x 2+ 10x = 200 General Rules to Solve (ii) In order to solve a system of two or more linear
inequalities with the same variables, graph each inequality
 x
⇒ x2 + 10x − 200 = 0 ⇒ x2 + 20x − 10x − 200 = 0 Linear Inequalities on the same set of axes. The intersection of the region of
Sol. b. x4 − 3x3 − 2x2 + 3x + 1 = 0 Equal numbers may be added or subtracted from both
all the inequalities is the required solution set.
⇒ x ( x + 20) − 10( x + 20) = 0 ⇒ ( x − 10) ( x + 20) = 0 •
On dividing throughout by x 2 and rearranging the terms, ∴ x = 10, − 20 sides of an inequality without changing the sign of in EXAMPLE 21. Find the solution set of −2x + 3y > 6
we get But x ≠ −20, since x is a positive number. equality. and 4x − 6 y > 12 .
 x2 + 1  − 3  x − 1 − 2 = 0 e.g. If a > b, then for any number c,
…(i) So, the required number is 10. a. x < −1, y > −3 b. x < 1, y > 3
 x2   x a + c > b + c or a − c > b − c.
1 c. x ≥ 3, y ≥ 2 d. No solution
Put x− =y EXAMPLE 19. Out of group of swans, 7/2 times the • If both sides of an inequality are multiplied or divided by
x square root of the number are swimming in the pool. the same positive number, then the sign of inequality Sol. d. Consider, −2x + 3y = 6
On squaring both sides, we get remains the same. e.g. Y
While the two remaining are playing outside the pool. 0 −3
1 1 a b x
x2 + 2 − 2 = y 2 ⇒ x2 + 2 = y 2 + 2 What is the total number of swans? If a > b and c > 0, then > and ac > bc. (0, 2)
x x y 2 0
a. 4 b. 8 c. 12 d. 16 c c
⇒ ( y 2 + 2) − 3y − 2 = 0 [from Eq. (i)] Again, 4x − 6y = 12
• If both sides of an inequality are multiplied or divided X´ X
⇒ y 2 − 3y = 0 ⇒ y ( y − 3) = 0 Sol. d. Let total number of swans be x.
7 by the same negative number, then the sign of (– 3, 0) O
(3, 0)
Number of swans swimming in the pool = x x 0 3
∴ y = 0 or 3 Remaining swans = 2 2 inequality is reversed. y −2 0 (0, – 2)
For y = 0 For y = 3
a b
7 7 e.g. If a > b and c < 0, then < and ac < bc.
1 1 By given condition, x+ 2= x ⇒ x = x−2 c c Now, graph each equation. Since, Y´
⇒ x− =0 ⇒ x− =3 2 2 there is no intersection of these two
x x 49 shaded regions, therefore there is no
x −1
2
x −1
2 On squaring both sides, we get x = x2 + 4 − 4 x Graphical Solution of Linear
⇒ =0 ⇒ =3 4 solution.
x x ⇒ 4x2 − 65x + 16 = 0, 4x2 − 64x − x + 16 = 0 Inequalities in Two Variables
⇒ x2 = 1 ⇒ x2 − 3x − 1 = 0 ⇒ 4x ( x − 16) − 1( x − 16) = 0 ⇒ ( x − 16)( 4x − 1) = 0 Let the inequality be ax + by + c ≥ 0. Quadratic Inequalities
3± 9 + 4 Q x ≠ 1  An equation of the form
∴ x=±1 ⇒ x= ∴ x = 16 Step I Consider it as ax + by + c = 0 by changing the
2  4  ax 2 + bx + c ≥ 0 or ax 2 + bx + c ≤ 0
inequality sign to equality sign, Draw the graph
3 ± 13 Thus, total number of swans = 16 or ax 2 + bx + c > 0 or ax 2 + bx + c < 0
∴ x= of this equation.
2 where, a ≠ 0 is called a quadratic inequality in one
Step II Choose any point not on the line, [usually (0, 0 )].

So, x = ± 1,
3 ± 13 
 is the required solution. INEQUALITIES Find whether this point satisfy the inequality or
variable x. It is same as quadratic equations except for
 2  the inequality sign.
Two real numbers or two algebraic expressions related not. If it does, then shade the half-plane
Note An equation of degree n has n roots.
by the symbols >, <, ≤ or ≥ form inequality. Here, the containing that point. Otherwise shade the other
symbols < (less than), > (greater than), ≤ (less than or Solution of Quadratic Inequalities
half-plane.
Word Problems Involving equal) and ≥ (greater than or equal) are known as Solving the inequality means finding the values of x
inequality signs. EXAMPLE 20. The solution set of inequality that make the inequality true. Solution sets of
Quadratic Equation e.g. 5 < 7, x ≤ 2, x + y ≥ 11 are all inequalities. 2x + 3y ≥ 6, x ≥ 0 and y ≥ 0 is quadratic inequalities are expressed in the form of
In this section, we will discuss some problems based on a. x = 1, y = 1 b. x = − 1, y = − 1 intervals. Here are the steps:
practical applications of quadratic equation. Linear Inequalities c. x = 3, y = 1 d. None of these • Replace the inequality symbol with an equal sign.
In such type of problems, A linear inequality is an inequality which involves a • Solve the quadratic equation by factorisation or by
Sol. d. Draw the graph of 2x + 3y = 6.
(i) We formulate a quadratic equation in variable x with linear function i.e. each variable occurs in first degree quadratic formula and find the real roots of the
The values of ( x, y) satisfying 2x + 3y = 6 are equation.
the help of the given conditions. only and there is no term involving the product of the
(ii) We then solve the quadratic equation to find the variables. x 0 3 • Plot the two real roots on the numbers line. They
answer to the given problem. e.g. ax + b ≤ 0, ax + by + c > 0, ax ≤ 4. y 2 0 divide the line into three sections or three intervals.
MATHEMATICS Quadratic Equations and Inequalities 167 168 CDS Pathfinder

• Pick a number from each interval and test it in the Take x = − 4, Take x = −2 , Take x = 0
9. If α , β are the roots of a quadratic equation such 19. Sum of roots is − 1 and sum of their reciprocals is
original inequality. ⇒ ( −4) 2 + 4( −4) + 3 > 0,
that α + β = 24 and α − β = 8, then the equation is 1/6, then equation is
• If the inequality holds true for the choosen point. then ( −2) 2 + 4( −2) + 3 < 0, 02 + 4(0) + 3 > 0
(a) x2 − 24x − 128 = 0 (b) x2 + 24x + 128 = 0 (a) x2 − 6x + 1 = 0 (b) x2 − x + 6 = 0
that interval is the solution of the given quadratic + – +
(c) x + 24x − 128 = 0
2
(d) None of these (c) 6x2 + x + 1 = 0 (d) x2 + x − 6 = 0
inequality.
–3 –1
10. Which one of the following is the equation whose 20. If sum of the roots of the equation
Note (i) If the quadratic equation does not have real roots, then the So, the intervals which satisfy the given equation are
roots are respectively three times the roots of the ax 2 + bx + c = 0 is equal to the sum of their squares,
quadratic expression is always positive (or always negative) (− ∞ , − 3] and [−1, ∞)
depending on the sign of a meaning that the solution set will equation ax 2 + bx + c = 0? then which one of the following is correct?
i.e. x ≤ −3 and x ≥ −1 (a) ax2 + bx + c = 0 (b) ax2 + 3bx + 9c = 0
either be empty or the entire real number line. (a) a2 + b 2 = c 2 (b) a2 + b 2 = a + b
e.g. 15x 2 − 18x + 7 > 0 has no real roots as EXAMPLE 23. The real values of x which satisfy (c) ax2 − 3bx + 9c = 0 (d) ax2 + bx + 3c = 0 (c) 2 ac = ab + b 2 (d) 2c + b = 0
D = b 2 − 4ac = 64 − 420 < 0 . Since a > 0, so the
expression is always positive and hence the solution set x 2 − 4x + 3 ≥ 0 and x 2 − 3x − 4 ≤ 0 is 11. How many real values of x satisfy the equation 21. If the roots of x 2 − lx + m = 0 differ by 1, then
will be the entire real number line. a. ( −1, 1) ∪ ( 3, 4) b. [ −1, 1] ∩ [ 3, 4] x 2/ 3 + x1/ 3 − 2 = 0?
(a) l 2 = 4m − 1 (b) l 2 = 4m + 2
(ii) When the inequality has an additional ‘=’ sign ( ≥ or ≤), use c. [ −1, 1] ∪ [ 3, 4] d. ( −1, 1) ∪ [ 3, 4] (a) only one value (b) two values
closed intervals like [a, b] to indicate that the two end points (c) three values (d) No value (c) l = 4m2 + 1 (d) l 2 = 4m + 1
are also included in the solution set. If the inequality is strict Sol. c. Given, x − 4x + 3 ≥ 0
2
(>or<), use open intervals like (a, b) as end points are not 12. If α , β are the roots of the quadratic equation 22. If α, β are the roots of the equation
⇒ x2 − 4x + 3 = 0 ⇒ ( x − 1)( x − 3) = 0 ⇒ x = 1, 3 1
included.
+ – + 2x 2 − 4x + 1 = 0. Then, the value of x 2 − (1 + a 2 ) x + (1 + a 2+ a 4 ) = 0, then α 2 + β 2 is
1 1 2
EXAMPLE 22. The solution of the inequality + is equal to equal to
x 2 + 4x + 3 ≥ 0 is
1 3 α + 2β β + 2α
⇒ x ∈ ( −∞ , 1] or x ∈[ 3, ∞). ⇒ x ∈ ( − ∞ 1] ∪ [ 3, ∞). 12 17 11 13 (a) a4 + a2 (b) a2
a. x < − 3 and x > − 1 b. x ≤ − 3 and x ≥ − 1 (a) (b) (c) (d)
Again, x2 − 3x − 4 ≤ 0 ⇒ x2 − 3x − 4 = 0 17 12 17 17 (c) (a2 + a4 )2 (d) None of these
c. x < − 3 and x ≥ − 1 d. x ≤ − 3 or x > − 1 ⇒ ( x + 1)( x − 4) = 0 ⇒ x = −1, 4
+ – +
13. Solve the equation 2x + 2 + 2− x = 5 and find the 23. If the roots of
1
+
1
=
1
are equal in
Sol. b. We have, x2 + 4x + 3 ≥ 0 roots of the equation. x+ p x+q r
Change the inequality to equality sign and solve –1 4 (a) {0, 2} (b) {−2, 0} (c) {0, 1} (d) {−1, 0} magnitude and opposite in sign, then product of
x2 + 4x + 3 = 0 ⇒ x2 + 3x + x + 3 = 0 ⇒ x ∈[ −1, 4] 14. If α and β are roots of the equation x 2 + p = 0 roots is
∴ Solution set is 1 2 p2 + q 2
⇒ x ( x + 3) + ( x + 3) = 0 ⇒ ( x + 1)( x + 3) = 0 ⇒ x = −1, − 3. where p is a prime, then which equation has the (a) − (p + q2) (b)
x = ( − ∞ , 1] ∪ [ 3, ∞) ∩ [ −1, 4] = [ −1, 1] ∪ [ 3, 4] 2 2
Place the value of x on the number line to create roots 1/α and 1/β? p+q 1
intervals. 1 1 1 1 (c) (d) ( p + q )2
(a) − = 0 (b) px2 + 1 = 0 (c) px2 − 1 = 0 (d) 2 + = 0 2 2
x2 p x p
24. If α , β are the roots of 3x 2 + 2x + 1 = 0, then the
15. The roots of the equation x 2 + px + q = 0 are 1 1−α 1−β

PRACTICE EXERCISE and 2. The roots of the equation qx 2 − px + 1 = 0


must be
−1 1
equation whose roots are

(a) x + 2 x + 3 = 0
2
1+α
and
1+β
is

(b) x2 − 2 x + 3 = 0
(a) and 1 (b) and 1 (c) x2 + 2 x − 3 = 0 (d) x2 − 2 x − 3 = 0
2 2
1. The quadratic equation has maximum 4+ 7 −1
5. The quadratic equation whose roots are (c) and − 1 (d) None of these 25. If one root of px 2 + qx + r = 0 is double of the
(a) one root (b) two roots 2 2
4− 7 other root, then which one of the following is
(c) four roots (d) three roots and is
2
16. Which one of the following is the quadratic correct?
2. The values of x in the equation (a) 4x + 16x + 9 = 0
2
(b) 4x − 16x − 9 = 0
2 equation whose roots are reciprocal to the roots (a) 2 q 2 = 9 pr (b) 2 q 2 = 9 p (c) 4q 2 = 9 r (d) 9q 2 = 2 pr
a 2b2x 2 − ( a 2 + b2 ) x + 1 = 0, a ≠ 0, b ≠ 0 is of the quadratic equation 2x 2 − 3x − 4 = 0 ?
(c) 4x2 − 16x + 9 = 0 (d) 4x2 + 16x − 9 = 0 26. If the roots of the equation x 2 + x + 1 = 0 are in
(a) 3x2 − 2 x − 4 = 0 (b) 4x2 + 3x − 2 = 0
(a) 1/a2 (b) 1/b 2 6. If α and β are the roots of the equation the ratio m : n, then
(c) 3x2 − 4x − 2 = 0 (d) 4x2 − 2 x − 3 = 0
(c) 1/a2 , 1/b 2 (d) None of these x 2 − 8x + p = 0 and α 2 + β 2 = 40, then p is equal to m n
(a) + + 1= 0 (b) m+ n + 1= 0
(a) 12 (b) 10 (c) 9 (d) 11 17. The value of x satisfying the equation n m
3. The value of ‘a’ for which the equation
7. If α and β are the roots of the equation x + 4 = x − 2 is (c)
m n
+ + 1= 0 (d) m + n + 1 = 0
ax 2 − 2 5 x + 4 = 0 has equal roots is n m
x 2 − 5x + 6 = 0, then the value of α 2 − β 2 (a) 0, 5 (b) 0, 4
(a) 5/4 (b) 4/5
(a) 5 (b) − 5 (c) ± 5 (d) ± 4 (c) 5 (d) None of these 27. What is one of the value of x in the equation
(c) − 5/ 4 (d) − 5/ 3
8. If α , β are the roots of the equation 18. If the roots of the quadratic equation x 1 − x 13
4. If one root of 3x 2 = 8x + ( 2k + 1) is seven times the + = ?
ax 2 + bx + c = 0, then an equation whose roots are px 2 + qx + r = 0 are reciprocal to each other, then 1− x x 6
other, then the value of k is 1 / α and 1 / β is (a) q = r (b) p = r 5 7 9 11
(a) 5/3 (b) − 5/ 3 (a) (b) (c) (d)
(a) bx2 + ax + c = 0 (b) ax2 − bx + c = 0 (c) q divides r (d) p divides q 13 13 13 3
(c) 2/3 (d) − 3/2
(c) cx + ax + b = 0
2
(d) cx + bx + a = 0
2
MATHEMATICS Quadratic Equations and Inequalities 169 170 CDS Pathfinder

28. What are the roots of the equation 40. The solution of the equation 50. If α and β are the roots of the equation 61. An equation equivalent to the quadratic
( a + b + x )− 1 = a − 1 + b− 1 + x − 1 ? x 2 − x − 1 = 0, then what is the value of (α 4 + β 4 )? equation x 2 − 6x + 5 = 0 is
x 2 − 16 − ( x − 4) = x 2 − 5x + 4 is
(a) 7 (b) 0 (a) x2 − 5x + 6 = 0 (b) 5x2 − 6x + 1 = 0
(a) a, b (b) − a, b (c) a, − b (d) − a, − b
(a) 4, 5, −  (b) {4, 5} (d) 5, − 
13 13
(c) {4} (c) 2 (d) None of these (c) | x − 3| = 2 (d) 6x2 − 5x + 1 = 0
29. The number of roots of the quadratic equation  3  3
10
8 sec2 φ − 6 sec φ + 1 = 0 is 41. If sinθ and cosθ are the roots of the equation 51. When the roots of the quadratic equation 62. If the sum of a number and its reciprocal is ,
ax 2+ bx + c = 0 are negative and reciprocals of each then the numbers are 3
(a) n (b) 2 (c) 0 (d) No solution ax − bx + c = 0, then which one of the following
2
1 1 1 1
2
− 7x + 7 other, then which one of the following is correct? (a) 3, (b) 3, − (c) −3, (d) −3, −
30. The number of real roots of 32x = 9 is is correct?
(a) b = 0 (b) c = 0 (c) a = c (d) a = − c 3 3 3 3
(a) 3 (b) 1 (c) 2 (d) 4 (a) a + b + 2 ac = 0
2 2
(b) a − b + 2 ac = 0
2 2

(c) a2 + c 2 + 2 ab = 0 (d) a2 − b 2 − 2 ac = 0 52. If sum as well as product of roots of a quadratic 63. Divide 16 into two parts such that the twice of
31. If x = 2 + 2, then equation is 9, then what is the equation?
the square of the greater part exceeds, the
(a) x + 4x + 2 = 0
2
(b) x − 2 x − 2 = 0
2 42. The equation (1 + n 2 ) x 2 + 2ncx + ( c2 − a 2 ) = 0 will (a) x2 + 9x − 18 = 0 (b) x2 − 18x + 9 = 0
square of the smaller part by 164. Then, the
have equal roots, if greater part is
(c) x2 − 4x + 2 = 0 (d) x2 − 4x − 2 = 0 (c) x2 + 9x + 9 = 0 (d) x2 − 9x + 9 = 0 (a) 58 (b) 10 (c) 6 (d) 15
(a) c 2 = 1 + a2 (b) c 2 = 1 − a2
x − bx 2
(c) c 2 = 1 + n2 + a2 (d) c 2 = (1 + n2 ) a2 64. The number of straight lines that can connect
32. For what value of k will the equation 53. What are the roots of the equation
ax − c log10( x 2 − 6x + 45) = 2 ? x ( x − 1)
k−1 ‘x’ points is given by the equation y = .
= have roots reciprocal to each other? 43. What is the condition that the equation 2
k+1 ax + bx + c = 0, where a ≠ 0 has both the roots
2 (a) 9, − 5 (b) − 9, 5 (c) 11, − 5 (d) − 11, 5
How many points does a figure have if only 15
1+ c c+1 b+c 1 1
(a) (b) (c) a − c (d) positive? 54. The sum of the roots of the equation + lines can be drawn connecting them?
1− c c −1 c −1 x+a x+ b (a) 15 (b) 10 (c) 6 (d) 5
(a) a, b and c are of same sign
1
33. If ‘α’ and ‘β’ be the roots of ax 2 − bx + b = 0, the (b) a and b are of same sign = is zero. What is the product of the roots of the 65. The two successive natural numbers whose
(c) b and c have the same sign opposite to that of a c squares have sum 221 are
α β
value of + is (d) a and c have the same sign opposite to that of b equation? (a) 10 and 11 (b) 11 and 12
β α (a + b) (a + b)
(a) − (b) (c) − 10 and − 11 (d) None of these
44. The equation whose roots are twice the roots of
(a) a / b (b) b/ a (c) a/ b (d) − a / b 2 2
the equation x 2 − 2x + 4 = 0 is ( a2 + b 2 ) ( a2 + b 2 ) 66. The two consecutive positive odd integers, the
34. If the equations 2x 2 − 7x + 3 = 0 and (c) − (d) sum of whose squares is 130.
(a) x2 − 2 x + 4 = 0 (b) x2 − 2 x + 16 = 0 2 2
4x 2 + ax − 3 = 0 have a common root, then what (c) x2 − 4x + 8 = 0 (d) x2 − 4x + 16 = 0 (a) − 7 and − 9 (b) 7 and 9
is the value of a? 55. For what value of k, will the roots of the equation (c) 7 and 5 (d) 3 and − 5
(a) − 11 or 4 (b) − 11 or − 4 (c) 11 or − 4 (d) 11 or 4 45. If α and β are the roots of the equation kx 2 − 5x + 6 = 0 be in the ratio of 2 : 3?
67. The solution set of inequation 2x + 1 ≥ 7 is
35. If the roots of x 2 + bx + c = 0 be α and β and those x 2 + px + q = 0, then − α −1 , − β −1 are the roots of (a) 0 (b) 1 (c) − 1 (d) 2
(a) x ≥ 8 (b) x ≥ 3
of x 2 + px + q = 0 be k α and k β, then which one of the following equations? 56. What is the ratio of sum of squares of roots to the (c) x ≥ 6 (d) None of these
(a) qx − px + 1 = 0
2 2
+ px + 1 = 0 product of the roots of the equation
(a) cb 2 = qp2 (b) qc 2 = b 2 p2 (b) q x−1
7x 2 + 12x + 18 = 0 ? 68. The values of x satisfying inequation ≥ 4 is
(c) qb 2 = cp2 (d) None of these (c) x2 + px − q = 0 (d) x2 − px + q = 0 3
(a) 6 : 1 (b) 1 : 6 (c) − 6 : 1 (d) − 6 : 7
(a) x ≤ 13 (b) x ≥ 12 (c) x ≥ 13 (d) x = 13
36. If − 4 is a root of the equation x 2 + px − 4 = 0 and 46. If one root of the equation ax 2 + x − 3 = 0 is − 1, x ( x − 1) − ( m + 1) x
the equation x 2 + px + q = 0 has equal roots, then then what is the other root? 57. If the roots of the equation = 69. The values of x satisfying 3x + 2 ≤ 5x − ( 4 − x ) is
( x − 1) ( m − 1) m
1 1 3 (a) x ≤ 2 (b) x ≥ 2
the values of p and q are, respectively (a) (b) (c) (d) 1 are equal, then what is the value of m ?
4 2 4 (c) x = 2 (d) None of these
9 9 9 (d) − 1/2
(a) − 3 and (b) 3 and (c) and 3 (d) 4 and 3 (a) 1 (b) 1 / 2 (c) 0
4 4 4 47. If the equation ( a 2 + b2 ) x 2 − 2 ( ac + bd ) x 70. The solution set of x for the inequations
58. If 3x + 27( 3− x ) = 12, then what is the value of x? 2x + 3 ≥ 8 and 3x + 1 ≤ 12 is
37. If α , β are the roots of 2x 2 − 6x + 3 = 0, then the + ( c2 + d 2 ) = 0 has equal roots, then which one of 5 11 5 11
(a) 1 (b) 2 (c) 1, 2 (d) 0, 1 (a) < x≤ (b) < x<
α β   1 1 the following is correct? 2 3 2 3
value of  +  + 3  +  + 2 αβ is
 β α α β (a) ab = cd (b) ad = bc 59. What is the magnitude of difference of the roots of 5 11 5 11
(c ) ≤ x ≤ (d) ≥ x ≥
(c) a 2 + c 2 = b 2 + d 2
(d) ac = bd x 2 − ax + b = 0 ? 2 3 2 3
(a) 12 (b) 23 (c) 13 (d) − 13
a2 − 4b b2 − 4 a 1 3  1
38. If the roots of the equation x + 2ax + b = 0, are 2 48. What is the solution of the equation (a) (b) 71. The values of x satisfying  x + 4 ≥ ( x − 6)
2 5  3
real and distinct and they differ by atmost 2m, (c) 2 a2 − 4b (d) b 2 − 4 ab are
x x+3 3
then b lies in the interval − =− ? (a) x ≥ 120 (b) x ≤ 120 (c) x ≤ 12 (d) x ≥ 12
x+3 x 2 60. What can be said about the roots of the equation
(a) (a2 − m2 , a2 ) (b) [a2 − m2 , a2 ) 72. 4x 2 − 1 ≤ 0, then the solution set is
(a) 1 (b) 2 (c) 4 (d) None of these x 2 − x − 2 = 0?
(c) (a , a + m )
2 2 2
(d) None of these −1 1 −1 1
(a) both of them are integers (a) ≤ x≤ (b) < x<
49. What are the roots of the equation 2 2 2 2
39. The solution of the equation x+2
(b) both of them are natural numbers
−1 1
( x + 2) ( x − 5) ( x − 6) ( x + 1) = 144 is 4 − 3⋅ 2
x
+ 32 = 0? (c) the latter of the two is negative (c) ≥ x≥ (d) None of these
2 2
(a) {7, − 3} (b) {7, − 3, 2} (c) {7, − 3, 2, 1} (d) {7, 2} (a) 1, 2 (b) 3, 4 (c) 2, 3 (d) 1, 3 (d) None of the above
MATHEMATICS Quadratic Equations and Inequalities 171 172 CDS Pathfinder

73. The shaded region, including the boundary in the Select the answer using the codes given below. PREVIOUS YEARS’ QUESTIONS 97. If x 2 = 6 + 6 + 6 + 6 + L ∞ , then what is one
given graph, is exactly represented by (a) Only I (b) Only II
(c) Both I and II (d) Neither I nor II 87. If the roots of the equation x − 2ax + a + a − 3 = 0
2 2 of the values of x equal to? e 2013 II
Y (a) 6 (b) 5 (c) 4 (d) 3
A (0, 6) 81. Consider the following statements are real and less than 3, then which one of the
I. Every quadratic equation has atleast one real following is correct? e 2012 I 98. Consider the following statements in respect of
root. (a) a < 2 (b) 2 < a < 3 (c) 3 < a < 4 (d) a > 4 the quadratic equation ax 2 + bx + b = 0, where
II. A quadratic equation with integral coefficients 88. If one of the roots of quadratic equation a ≠ 0.
has integral roots. 7x 2 − 50x + k = 0 is 7, then what is the value of k? I. The product of the roots is equal to the sum of
O III. If the coefficient of x2 and the constant term of e 2012 I
the roots.
X´ X
B a quadratic equation have opposite signs, then (a) 7 (b) 1 (c) 50/7 (d) 7/50 II. The roots of the equation are always unequal
(4, 0) and real.
the quadratic equation has real roots.
Y´ 89. Two students A and B solve an equation of the Which of the above statements is/are correct?
(a) 3x + 2 y ≤ 12, x < 0, y ≥ 0 (b) 3x + 2 y ≤ 12, x ≥ 0, y ≥ 0 Which of the above statements is/are correct? form x 2 + px + q = 0. A starts with a wrong value e 2014 I
(c) 3x + 2 y < 12, x ≥ 0, y ≥ 0 (d) 3x + 2 y > 12, x ≥ 0, y ≥ 0 (a) Only I and II (b) Only II and III of p and obtains the roots as 2 and 6. B starts (a) Only I (b) Only II
(c) Only III (d) All of these with a wrong value of q and gets the roots as
74. When is the expression x + 3x − 10 positive only?
2 (c) Both I and II (d) Neither I nor II
82. Which of the following are quadratic equations? 2 and – 9. What are the correct roots of the
(a) x ≤ −5 (b) x ≥ 2 (c) − 5 < x < 2 (d) x < −5 or x > 2 equation? e 2012 II
99. If the roots of the equation Ax 2 + Bx + C = 0 are
1 3
75. The solution set for the quadratic inequation I. x 2 + =2 II. x + = x2 (a) 3 and − 4 (b) − 3 and − 4 (c) − 3 and 4 (d) 3 and 4 −1 and 1, then which one of the following is
x2 x correct? e 2014 I
x 2 − 5x + 6 ≥ 0 is 90. If one of the roots of the equation x 2 − bx + c = 0 is
III. 2x2 − x + 2 = x2 + 4x − 4 IV. x3 + 6x2 + 2x − 1 = 0 (a) A and C are both zero
(a) [ − ∞, 2 ] ∪ [ 3, ∞ ] (b) [ − ∞, 2 ] ∪ [ 3, ∞ ] the square of the other, then which of the (b) A and B are both positive
(c) [ − ∞, 2 ] ∪ [ 3, ∞ ] (d) [2, 3] Select the correct answer using the codes given following option is correct? e 2013 I (c) A and C are both negative
x−2 x−3 below. (a) b 3 = 3 bc + c 2 + c (b) c 3 = 3 bc + b 2 + b (d) A and C are of opposite sign
76. All real values of x for which < are
3x + 1 3x − 2 (a) I, III are quadratic (b) II, III and IV are quadratic (c) 3 bc = c 3 + b 2 + b (d) 3 bc = c 3 + b 3 + b 2 100. If the roots of the equation ( a 2 − bc) x 2
(c) Only III is quadratic (d) None of these
−1 2
(a)  ,  (b) 
− 1 2
,
−1 2
(c)  ,  (d) R + 2 ( b2 − ac) x + ( c2 − ab) = 0 are equal, where
 3 3   3 3   3 3  83. Consider the following statements 91. The difference of the roots of the equation
2x − 11x + 5 = 0 is
2 b ≠ 0, then which one of the following is correct?
e 2013 I
I. x = 1 is a root of 3x2 − 2x − 1 = 0. e 2014 I
77. If x + 2 y ≤ 3, x > 0 and y > 0, then one of the (a) 4.5 (b) 4 (c) 3.5 (d) 3
II. x = − 2 2 is a root of x2 + 2x − 4 = 0. (a) a + b + c = abc (b) a2 + b 2 + c 2 = 0
solution is 92. The sum of the squares of two numbers is 97 and (c) a3 + b 3 + c 3 = 0 (d) a3 + b 3 + c 3 = 3abc
(a) x = − 1, y = 2 (b) x = 2, y = 1 III. x2 + x + 1 = 0; x = 1, x = − 1. the squares of their difference is 25. The product
(c) x = 1, y = 1 (d) x = 0, y = 0 1 2 of the two numbers is 101. If m and n are the roots of the equation
IV. 9x2 − 3x − 2 = 0; x = − , x = . e 2013 I
78. The shaded region in the given figure is the 3 3 (a) 45 (b) 36 (c) 54 (d) 63 ax 2 + bx + c = 0, then the equation whose roots
solution set of the inequalities Which of the equation(s) given above is/are 1 1 are ( m 2 + 1) / m and ( n 2 + 1) / n is
Y correct? 93. If x + = 2, then what is value of x − ? e 2014 I
x x e 2013 I (a) acx2 + (ab + bc )x + b 2 + (a − c )2 =0
(a) I and II (b) I, II and IV
(a) 0 (b) 1 (c) 2 (d) −2 (b) acx2 + (ab − bc ) x + b 2 + (a − c )2 =0
x

(c) III and IV (d) Only IV


+

(c) acx2 + (ab − bc ) x + b 2 − (a − c )2 =0


y=

94. There are some benches in a classroom having the


Directions (Q. Nos. 84-86) A ball is thrown (d) acx2 + (ab + bc ) x + b 2 − (a − c )2 =0
2

number of rows 4 more than the number of


x+
3y upwards from a rooftop, 80 m above the ground. columns. If each bench is seated with 5 students, 102. In solving a problem, one student makes a
= It will reach a maximum vertical height and there are two seats vacant in a class of 158
3 mistake in the coefficient of the first degree
O
X then fall back to the ground. The height of the ball students. The number of rows is e 2013 I term and obtains −9 and −1 for the roots. Another
from the ground at time t is h which is given by, (a) 4 (b) 8 (c) 6 (d) 10 student makes a mistake in the constant term
(a) x+ y ≤ 2, x + 3 y ≥ 3, x ≥ 0, y ≥ 0 h = −16t 2 + 64 t + 80 Now, answer the following 95. Which one of the following is factor of of the equation and obtains 8 and 2 for the
(b) x+ y ≥ 2, x + 3 y ≥ 3, x ≥ 0, y ≥ 0
questions based on the above information. 1  1 roots. The correct equation was e 2014 I
(c) x+ y ≥ 2, x + 3 y ≤ 3, x ≥ 0, y ≥ 0 x2 + + 8  x +  + 14 ? (a) x2 + 10x + 9 = 0 (b) x2 − 10x + 16 = 0
(d) x+ y ≤ 2, x + 3 y ≤ 3, x ≥ 0, y ≥ 0 84. What is the height reached by the ball after x2  x e 2013 II
1 1 (c) x2 − 10x + 9 = 0 (d) None of these
1 sec? (a) x + +1 (b) x + + 3
79. If a + b = 2m 2, b + c = 6m, a + c = 2, where m is a x x
(a) 150 m (b) 128 m 103. If m and n ( m > n ) are the roots of the equation
real number and a ≤ b ≤ c, then which one of the 1 1
(c) 64 m (d) None of these (c) x + + 6 (d) x + +7 7( x + 2a )2 + 3a 2 = 5a( 7x + 23a ), where a > 0, then
following is correct? x x
(a) 0 ≤ m ≤ 1/2 (b) − 1 ≤ m ≤ 0 85. What is the maximum height reached by the what is 3m − n equal to? e 2014 II
(c) 1/ 3 ≤ m ≤ 1 (d) 1 < m ≤ 2 ball? 96. If α and β are the roots of the equation (a) 12a (b) 14a (c) 15a (d) 18a
(a) 110 m (b) 132 m α2 + β2 104. If one of the roots of the equation px 2 + qx + r = 0
80. If α and β are the roots of the equation (c) 144 m (d) cannot be determined x 2 − x − 1 = 0, then what is equal
(α 2 − β 2 ) (α − β ) is three times the other, then which one of the
( x 2 − 3x + 2 = 0), then which of the following
86. How long will it take before hitting the ground? to? e 2013 II following relations is correct? e 2014 II
equation has the roots (α + 1) and (β + 1)? 2 3 4
(a) 5 sec (b) 6 sec (a) (b) (c) (d) None of these (a) 3q 2 = 16 pr (b) q 2 = 24 pr
I. x2 + 5x + 6 = 0 II. x2 − 5x − 6 = 0 (c) 3 sec (d) cannot say 5 5 5 (c) p = q + r (d) p + q + r = 1
MATHEMATICS Quadratic Equations and Inequalities 173 174 CDS Pathfinder

105. If m and n are the roots of the equation


x 2 + ax + b = 0 and m 2, n 2 are the roots of the
114. 5x – 2y =10 ANSWERS
4
equation x 2 − cx + d = 0, then which of the 3
1 b 2 c 3 a 4 b 5 c 6 a 7 c 8 d 9 d 10 b
following is/are correct? 11 b 12 a 13 b 14 b 15 c 16 b 17 c 18 b 19 d 20 c
2 2x + 6 y =21
I. 2b − a = c
2
II. b = d
2 21 d 22 b 23 a 24 b 25 a 26 a 27 c 28 d 29 d 30 c
1 31 c 32 b 33 b 34 a 35 c 36 b 37 c 38 b 39 b 40 b
Select the correct answer using the codes given
below. e 2014 II 41 b 42 d 43 d 44 d 45 a 46 c 47 b 48 a 49 c 50 a
(a) Only I (b) Only II 0 1 2 3 4
51 c 52 d 53 c 54 c 55 b 56 d 57 d 58 c 59 a 60 a
(c) Both I and II (d) Neither I nor II The linear inequations, for which the shaded
61 c 62 a 63 b 64 c 65 a 66 b 67 b 68 c 69 b 70 c
106. The sign of the quadratic polynomial ax 2 + bx + c area in the figure given above is the solution set,
71 b 72 a 73 b 74 d 75 a 76 b 77 c 78 d 79 c 80 d
is always positive, if e 2015 I are e 2016 I
(a) 2 x + 6 y ≤ 21, 5x − 2 y ≤ 10 81 c 82 c 83 b 84 b 85 c 86 a 87 a 88 a 89 b 90 a
(a) a is positive and b 2 − 4ac ≤ 0
(b) a is positive and b 2 − 4ac ≥ 0 (b) 2 x + 6 y ≤ 21, 5x − 2 y ≥ 10 91 a 92 b 93 a 94 b 95 c 96 b 97 d 98 d 99 d 100 d
(c) a can be any real number and b 2 − 4ac ≤ 0 (c) 2 x + 6 y ≥ 21, 5x − 2 y ≤ 10 101 a 102 c 103 c 104 a 105 b 106 a 107 c 108 c 109 c 110 c
(d) a can be any real number and b 2 − 4ac ≥ 0 (d) 2 x + 6 y ≥ 21, 5x − 2 y ≥ 10 111 a 112 d 113 c 114 a 115 b 116 c 117 a 118 a 119 a 120 d
100
107. The number of values of x satisfying x + > 50,
115. Let p and q be non-zero integers. Consider the
x
where x is a natural number less than or equal polynomial A ( x ) = x 2 + px + q. It is given that
to 100, is e 2015 II ( x − m ) and ( x − km ) are simple factors of A ( x ),
(a) 51 (b) 53 (c) 55 (d) 57 where m is a non-zero integer and k is a positive
108. If the roots of the quadratic equation integer, k ≥ 2. Which one of the following is
correct?
HINTS AND SOLUTIONS
x 2 − 4x − log10 N = 0 are all real, then the e 2016 I
(a) (k + 1)2 p2 = kq (b) (k + 1)2 q = kp2
minimum value of N is e 2015 II
1. (b) An equation of n degree has n roots. 5. (c) Here, sum of roots, 8. (d) Here, α + β = − b / a and αβ = c / a
1 1 1 (c) k 2q = (k + 1)p2 (d) k 2 p2 = ( k + 1)2 q
(a) (b) (c) (d) 10000 So, quadratic equation has two roots. 4+ 7 4− 7 Now, roots of required equation are
100 1000 10000 S= + =4
116. Which of the points P ( 5, − 1), Q ( 3, − 2) and 2. (c) a 2 b 2 x2 − a 2 x − b 2 x + 1 = 0 2 2 1 1
, .
109. What is 4 + 4 − 4 + ... equal to? Product of roots, α β
e 2015 II R (1, 1) lie in the solution of the system of ⇒ a 2 x ( b 2 x − 1) − 1 ( b 2 x − 1) = 0
13 − 1 13 + 1 1 1 α + β −b / a −b
inequations x + y ≤ 4 and x − y ≥ 2? e 2016 I ⇒ ( b 2 x − 1) ( a 2 x − 1) = 0 4+ 7   4 − 7  16 − 7 9 ⇒S = + = = =
(a) 3 (b) (c) (d) 0 P=    = = α β αβ c/ a c
2 2 (a) Q and R (b) P and R  2   2  4 4
So, either a 2 x − 1 = 0 ⇒ a 2 x = 1
1 1 1 a
110. If k = x − y + 2z, where −2 ≤ x ≤ 1 and −1 ≤ y ≤ 2 (c) P and Q (d) P, Q and R
or b 2 x − 1 = 0 or b 2 x = 1 The required equation is P= ⋅ = =
α β c/ a c
and 3 ≤ z ≤ 6, then which one of the following is a + 2b + a − 2b ⇒ x = 1/ a 2 or x = 1/ b 2 x2 − Sx + P = 0
correct? e 2015 II 117. If x = , then bx − ax + b is 2
9 ∴ Required quadratic equation is
a + 2b − a − 2b 3. (a) Given, ax2 − 2 5x + 4 = 0 has x 2 − 4x + = 0
(a) 0 ≤ k ≤ 9 (b) 5 ≤ k ≤ 11 (c) 2 ≤ k ≤ 14 (d) 2 ≤ k ≤ 11 4 x2 − S x + P = 0
equal to (given that, b ≠ 0) equal roots.
e 2016 I ⇒ 4x2 − 16 x + 9 = 0 −b
111. If the sum of the roots of ax 2 + bx + c = 0 is equal ∴ Discriminant ⇒ x2 −   x + = 0
a
(a) 0 (b) 1 (c) ab (d) 2ab 6. (a) Given, x − 8x + p = 0
2  c 
to the sum of the squares of their reciprocals, = ( − 2 5 ) 2 − 4 ( a) 4 = 0 c
[Q D = B 2 − 4 AC ] Sum of roots α + β = 8 and product of ⇒ cx2 + bx + a = 0
then which one of the following relations is 118. If 3x 2 − 7x − 30 − 2x 2 − 7x − 5 = x − 5 has α roots αβ = p
correct? e 2016 I ⇒ 20 − 16 a = 0 ⇒ a = 5 / 4 9. (d) Given, α + β = 24 and α − β = 8
and β as its roots, then the value of αβ is e 2016 I α + β = (α + β ) − 2 αβ
2 2 2
…(i)
(a) ab 2 + bc 2 = 2 a2c (b) ac 2 + bc 2 = 2 b 2 a (a) − 15 (b) − 5 (c) 0 (d) 5 4. (b) Given, 3x2 = 8x + ( 2k + 1) On solving, we get α = 16 and β = 8
(c) ab 2 + bc 2 = a2c (d) a2 + b 2 + c 2 = 1 ⇒ 40 = (8)2 − 2 ( p ) [Q α 2 + β 2 = 40] ∴ Sum of roots = α + β = 24
or 3x2 − 8x − ( 2k + 1) = 0
119. If the roots of the equation lx 2 + mx + m = 0 are Let first and second root be α and 7α ,
⇒ 40 − 64 = − 2 p and product of roots = 16 × 8 = 128
112. If the equations x 2 − px + q = 0 and x 2 + qx − p = 0
p q m respectively. ⇒ − 24 = − 2 p ⇒ p = 12 So, required equation is
have a common root, then which one of the in the ratio p : q, then + + is equal to 8
q p l ∴ Sum of roots = α + 7α = 7. (c) Given, x2 − 5x + 6 = 0 x2 − 24x + 128 = 0.
following is correct? e 2016 I e 2016 I 8 1 3
(a) p − q = 0 (b) p + q − 2 = 0 ⇒ 8α = ⇒ α = Sum of roots = −
Coefficient of x
(a) 0 (b) 1 (c) 2 (d) 3 3 3 10. (b) Let α and β be the roots of the
(c) p + q − 1 = 0 (d) p − q − 1 = 0 Coefficient of x 2
1 1 1 7 equation ax2 + bx + c = 0.
120. The solution of the inequation 1 + − ≥ 0 is So, roots are and . ⇒ α+β =5 −b c
113. Under what condition on p and q, one of the x x2 3 3 Constant term Then, α + β = and αβ =
roots of the equation x 2 + px + q = 0 is the square 1 7 7 Product of roots = a a
(given that, x ≠ 0) e 2016 I Also, product of roots = × = Coefficient of x 2
3 3 9 −3 b 9c
of the other? e 2016 I (a) x > 0 (b) x < 0 ⇒ αβ = 6 Now, 3α + 3β = and 3α ⋅3 β =
7 − ( 2k + 1) a a
(a) 1 + q + q 2 = 3 pq (b) 1 + p + p2 = 3 pq −1 − 5 −1 + 5 −1 − 5 −1 + 5 ⇒ = Now, (α − β )2 = (α + β )2 − 4αβ
(c) ≤ x≤ (d) x ≤ or x ≥ 9 3 ∴ Required equation = x2 −
(c) p3 + q + q 2 = 3 pq (d) q 3 + p + p2 = 3 pq 5 2 2 2 = (5 ) 2 − 4 × 6 = 1
7 (Sum of roots) x
⇒ = − ( 2k + 1) ⇒ α − β = ± 1 and α + β = 5
3 + Product of roots
⇒ α 2 − β 2 = (α + β ) (α − β ) −3 b 
⇒ x2 − 
9c
⇒ 7 = − 6 k − 3 ⇒ 10 = − 6 k x+ =0
= 5 ( ± 1)  a  a
−5
⇒ k= ⇒ α2 − β2 = ± 5 ⇒ ax2 + 3bx + 9c = 0
3
MATHEMATICS Quadratic Equations and Inequalities 175 176 CDS Pathfinder

11. (b) Given equation is 15. (c) The equation is x2 + px + q = 0. By given condition, 1 − ( −2 / 3 ) + 1 / 3 2 29. (d) 8 sec 2 φ − 6 sec φ + 1 = 0 Now, qb 2 = k 2 αβ (α + β )2
= = =3
x2 / 3 + x1 / 3 − 2 = 0 Sum of roots = − p = 1 + 2 α + β = α2 + β2 1 + ( −2 / 3 ) + 1 / 3 2 / 3 ⇒ 8 sec 2 φ − 4 sec φ − 2 sec φ + 1 = 0 = k 2 (α + β )2 αβ
⇒ p = −3 ∴ Required equation is
⇒ ( x1 / 3 )2 + x1 / 3 − 2 = 0 ⇒ α+β = (α + β )2 − 2αβ ⇒ ( 4 sec φ − 1) ( 2 sec φ − 1) = 0 = [ k (α + β )]2 αβ
x1 / 3 = x Product of roots = q = 1 × 2 = 2 2 x − Sx + P = 0 or x − 2x + 3 = 0
2 2
Let
⇒ −
b
=  −  − 2  
b c ⇒ sec φ = 1/ 4 or sec φ = 1/ 2 ∴ qb 2 = p 2 c
⇒ x2 + x − 2 = 0, ∴ Equation qx2 − px + 1 = 0 becomes a  a  a 25. (a) Given, px2 + qx + r = 0 But sec φ ≥ 1 or sec φ ≤ − 1 36. (b) Since, − 4 is a root of
It is a quadratic equation in x. 2x 2 − ( − 3) x + 1 = 0 −b b 2 2c Let the roots be α and β.
⇒ = − ⇒ − ba = b 2 − 2 ca Hence, the equation has no solution. x 2 + px − 4 = 0
∴ Discriminant of x2 + x − 2 = 0 is ⇒ 2x 2 + 3x + 1 = 0 a a2 a By given condition, β = 2α 2
r 30. (c) Here, 32 x − 7x + 7
= 9 = 32 ( − 4)2 + p ( − 4) − 4 = 0
B − 4 AC = 1 − 4 ( 1) ( − 2) = 9 > 0
2 2
⇒ ( 2x + 1) ( x + 1) = 0 ⇒ 2 ac = b + ab
2
Product of roots (αβ ) = = 2 α 2
p On comparing, we get ⇒ 16 − 4 p − 4 = 0 ⇒ p = 3
Hence, two real values of x satisfy the

1
x = − or x = − 1 21. (d) Here, roots are α and α + 1 . As roots of x2 + px + q = 0 are equal.
given equation. ⇒ α2 =
r
…(i) 2x 2 − 7x + 7 = 2
2 ∴ α + (α + 1) = l [sum of roots] 2p ∴ D = B 2 − 4 AC = 0
4 1 2x − 7x + 5 = 0
2
12. (a) Here, α + β = = 2, αβ = 16. (b) Given, 2x2 − 3x − 4 = 0 l −1 q
2 2 ⇒ 2α = l − 1 ⇒ α = Sum of roots (α + β ) = − = 3 α …(ii) Here, D = b 2 − 4 a c = 49 − 4 ( 2) (5) = 9 ⇒ p 2 − 4 q = 0 or 32 − 4 q = 0 ⇒ q =
9
For getting a reciprocal roots, we replace 2 p 4
1 1 Also, α (α + 1) = m or α 2 + α = m q2 Since, D > 0, so it has two real roots.
Now, + 1
x by , we get On squaring Eq. (ii), we get 9α 2 = 2 9
α + 2β β + 2α 2 So, p = 3, q =
x
⇒  l − 1 +  l − 1 = m p 31. (c) Here, x= 2 + 2 ⇒x− 2= 2 4
2    
β + 2α + α + 2β 2   − 3   − 4 = 0
1 1  2   2   r  q2 On squaring both sides, we get
=  x  x ⇒ 9  = [from Eq. (i)] 37. (c) Here, α + β = 3 and αβ = 3 / 2
(α + 2 β ) (β + 2 α ) ⇒ ( l − 1) 2 + 2 ( l − 1) = 4m  2 p  p2 ( x − 2)2 = 2 ⇒ x 2 − 4 x + 4 = 2
2 3   α β   1 1
⇒ − −4= 0 ⇒ l 2 − 1 = 4m ⇒ l 2 = 4 m + 1 ⇒  +  + 3  +  + 2 αβ
3α + 3β x2 x ⇒ x2 − 4 x + 2 = 0 β α α β
= ⇒ 9rp = 2q 2
αβ + 2α 2 + 2β 2 + 4αβ 22. (b) Here, α + β = ( 1 + a 2 ) x2 − bx k − 1
⇒ − 4x 2 − 3x + 2 = 0 α m = α 2 + β 2 3(α + β )
1 26. (a) Let roots be α , β, then = 32. (b) = + + 2 αβ
⇒ 4x + 3x − 2 = 0
2 and αβ = ( a 4 + a 2 + 1) β ax − c k+1 αβ αβ
3(α + β ) 3( 2) 12 n
= = = 2
2 (α + β )2 + αβ 2 ( 2)2 + 1 17 17. (c) As, x + 4 = x − 2 [given] ⇒ ( x2 − bx) ( k + 1 ) = ( k − 1 ) ( ax − c ) (α + β )2 − 2αβ 3(α + β )
Q α 2 + β 2 = (α + β )2 − 2 αβ α + β = − 1, αβ = 1 = + + 2αβ
2 On squaring both sides, we get ⇒ x2 k + x 2 − bxk − bx = kax αβ αβ
= ( 1 + a 2 ) 2 − ( a 4 + a 2 + 1) α β
13. (b) Given equation is 2x + 2 + 2− x = 5
m
+
n
+ 1= + + 1 − kc − ax + c 32 − 2 ⋅ 3 / 2 3⋅3
( x + 4) = ( x − 2)2 + 2  
3
= 1 + a 4 + 2a 2 − a 4 − a 2 − 1 n m β α = +
1 ⇒ ( k + 1 ) x2 − x ( bk + b + ka − a ) 3/2 3/2  2
⇒ 2x 22 + 2−x = 5 ⇒ 4 ⋅ 2x + x = 5 ⇒ x + 4 = x 2 + 4 − 4x ∴ α2 + β2 = a2
2 α+β −1 + kc − c = 0 6 9
⇒ x 2 − 5x = 0 ⇒ x = 0 , x = 5 = + 1= + 1= − 1+ 1= 0 = + + 3 = 4 + 6 + 3 = 13
1 1 1 1 αβ 1
Put 2 = y, 4 y + = 5
x
23. (a) Here, + = Since, roots are reciprocal to each other. 3/2 3/2
y But for x = 0, 0+ 4 = 0 − 2 x+ p x+ q r x 1− x 1 So, product = 1 38. (b) Let α, β be the roots of
4≠−2 27. (c) Let = y ⇒ =
⇒ 4y2 + 1 = 5y ⇒ 4y2 − 5y + 1 = 0 ⇒ r ( x + p + x + q) = ( x + p) ( x + q) 1− x x kc − c x 2 + 2ax + b = 0
So, x = 5 is the only solution.
y i.e. = 1 ⇒ kc − c = k + 1 …(i)
⇒ ( y − 1) ( 4 y − 1) = 0 ⇒ x2 + ( p + q − 2r ) x + pq 1 13 ( k + 1) ⇒ α + β = −2a and αβ = b.
∴ y+ = ⇒ ( y + 1) 6 = 13 y
2
18. (b) Let roots of equation be α and
1 c+ 1
⇒ y − 1 = 0 or 4y − 1 = 0 . − ( p + q) r = 0 y 6 ⇒ k ( c − 1) = c + 1 ⇒ k = By hypothesis, |α − β| ≤ 2m
α ⇒ 6 y 2 − 13 y + 6 = 0 c−1
⇒ y = 1 or 4y = 1 Let roots be α and ( − α ), then ⇒ (α − β)2 ≤ 4m 2
∴ Product of roots ⇒ 6 y − 9y − 4y + 6 = 0
2
33. (b) Here, α + β = b / a and αβ = b / a
⇒ y = 1,
1
1 Constant term r α + ( − α) = 0 ⇒ (α + β)2 − 4αβ ≤ 4m 2
=α × = = p+ q ⇒ 3 y ( 2 y − 3 ) − 2 ( 2 y − 3) = 0 α β α+β b/a b
4
α Coefficient of x2 p ⇒ − ( p + q − 2r ) = 0 ⇒ r = So, + = = = ⇒ 4a 2 − 4b ≤ 4m 2
By condition, 2 ⇒ (3 y − 2 ) ( 2 y − 3 ) = 0 β α αβ b/a a
⇒ 1=
r
⇒ r= p So, product of roots = pq − ( p + q ) r ⇒ a 2− b ≤ m 2 …(ii)
2x = y 2x = y 2 3
p ( p + q) ∴ y = and and discriminant of Eq. (i) is greater
⇒ 2x = 1 ⇒ 2x =
1 = pq − ( p + q ) ⋅ 3 2 34. (a) Given, 2x 2 − 7 x + 3 = 0 than 0.
19. (d) Let roots be α and β, then 2
4
When, y =
2

x
=
4 ∴ 2x 2 − 6 x − x + 3 = 0 ⇒ 4a 2 − 4b > 0 ⇒ b < a 2 …(iii)
1 α+β=−1 ( p + q ) 2 −1 2 1− x 9
⇒ 2x = 20 ⇒ 2 = 2 = 2 −2
x = pq − = ( p + q2 ) 3 ⇒ 2x ( x − 3) − 1 ( x − 3) = 0 From Eqs. (ii) and (iii),
1 1 1 β+α 1 2 2
2 + = ⇒ = ⇒ 9x = 4 − 4x ⇒ x =
4
⇒ ( 2x − 1 ) ( x − 3) = 0 b ∈[ a 2 − m 2 , a 2 )
∴ x=0 ∴ x = −2 α β 6 αβ 6 2 1 13
24. (b) Here, α + β = − and αβ = When, x=
1 39. (b) Refer to example 15. [Rule 5]
−1 1 3 3 3 x 9
Hence, the roots of the equation are 0 = ⇒ αβ = − 6 When, y = ⇒ = 2 40. (b) Refer to example 14. [Rule 4]
and − 2. αβ 6 1− α 1− β 2 1− x 4 2
⇒ S= +
4   + a   − 3 = 0
1 1 41. (b) Since, sin θ and cos θ are the roots of
14. (b) Since, α and β are roots of the ∴ Required equation is, 1+ α 1+ β ⇒ 4x = 9 − 9 x ⇒ x =
9
 2  2 the equation ax2 − bx + c = 0.
x2 − (α + β ) x + αβ = 0 ( 1 − α )( 1 + β ) + ( 1 + α )( 1 − β ) 13
equation x2 + p = 0. = a
⇒1 + − 3 = 0 ⇒
a
=2⇒ a=4 ∴ sin θ + cos θ =
b
...(i)
( 1 + α )( 1 + β ) 1 1 1 1
= + +
∴ α + β = 0 and αβ = p ⇒ x2 − ( −1)x + ( −6) = 0 28. (d) Given, 2 2 a
2−2×
1 a+ b+ x a b x c
1 1 α+β ∴ x2 + x − 6 = 0 2 − 2αβ When, x = 3, 4 (3)2 + a (3) − 3 = 0 and sin θ cos θ =
So, + = =0 = = 3 1 1 1 1
α β αβ ∴ − = + ⇒ 36 + 3 a − 3 = 0 ⇒ a = − 11
a
20. (c) Let α and β be the roots of the 1 + (α + β ) + αβ 1 + ( −2 / 3) + 1 / 3 a+ b+ x x a b On squaring both sides of Eq. (i), we get
1 1 1 1 equation 4 3 − ( a + b) ( a + b) ∴ a = − 11 or 4
and ⋅ = = = × =2 ⇒ = b2
α β αβ p ax2 + bx + c = 0 ⇒ sin θ + cos θ + 2 sin θ cos θ = 2
2 2
b 3 2 ( a + b + x )x ab 35. (c) Here, for equation x2 + bx + c = 0 a
21
∴ Required equation x − 0 ⋅ x + = 0 ∴ Sum of roots (α + β ) = − 1 − α 1 − β 1 − (α + β ) + αβ ⇒ x + ( a + b ) x + ab = 0
2
α + β = − b and αβ = c b2 b2 − a2
⇒ 1 + 2  = 2 ⇒ 2   =
p a P= × = c c
c ⇒ ( x + a) ( x + b) = 0
and product of roots (αβ ) = 1 + α 1 + β 1 + (α + β ) + αβ and for x2 + px + q = 0  a a  a a2
⇒ px + 1 = 0
2
a ⇒ x = − a, − b
kα + kβ = − p and k 2 αβ = q ⇒ 2ac = b 2 − a 2 ⇒ a 2 − b 2 + 2ac = 0
MATHEMATICS Quadratic Equations and Inequalities 177 178 CDS Pathfinder

42. (d) The equation will have equal roots, x x+3 3 1 1 1 Also, α 2 = −m (m + 1) x( x − 1) 1 3  1
48. (a) Given, − =− 54. (c) Given, + = 2 64. (c) Here, y = 15, then 15 = 71. (b)  x + 4 ≥ ( x − 6)
B 2 − 4 AC = 0 x+3 x 2 x+ a x+ b c  1  = −m (m + 1) 2 2 5  3
if ⇒  
x+3 1 ( x + b) + ( x + a) 1  2 ⇒ x2 − x − 30 = 0 ⇒
3 1
x + 2≥ x − 2
∴ ( 2nc )2 − 4 ( 1 + n2 ) ( c 2 − a 2 ) = 0 Let, y =
x
, then = ⇒ =
x+3 x y ( x + a )( x + b ) c ⇒ 4m + 4m + 1 = 0
2
⇒ x2 − 6x + 5x − 30 = 0
10 3
⇒ 4 n2 c 2 − 4 ( c 2 + n2 c 2 − a 2 ⇒ ( 2m + 1) 2 = 0 ⇒ 9x + 60 ≥ 10 x − 60
1 3 ⇒ 2cx + ( a + b ) c = x + ( a + b )x + ab
2
1 ⇒ x ( x − 6) + 5 ( x − 6) = 0
− n2 a 2 ) = 0 y− =− ⇒ 2y − 2 = − 3y
2
∴ m=− ⇒ − x ≥ − 120
y 2 ⇒ x2 + ( a + b − 2c ) x + ab − ac − bc = 0 2 ⇒ ( x + 5) ( x − 6) = 0
⇒ − 4 c 2 + 4 a 2 + 4 n2 a 2 = 0 [multiplying both sides by − 1]
⇒ 2y2 + 3y − 2 = 0 Let the roots of above equation be α 58. (c) Given, 3x + 27 (3− x ) = 12 Either x = 6 or x = − 5
⇒ c 2 = a 2 ( 1 + n2 ) and β. ⇒ x ≤ 120
⇒ 2y2 + 4y − y − 2 = 0 Let 3x = y But x ≠ − 5, so x = 6
43. (d) a and c have the same sign opposite Given, α+ β=0 27 Thus, all real numbers x which are less
⇒ 2 y ( y + 2) − 1 ( y + 2) = 0 ∴ y+ = 12 Thus, figure has 6 points. than or equal to 120 satisfies the
to that of b.
⇒ ( 2 y − 1) ( y + 2) = 0 ⇒ − ( a + b − 2c ) = 0 ⇒ a + b = 2c …(i) y
inequality.
⇒ y 2 − 12 y + 27 = 0 65. (a) Let the natural numbers be x and
44. (d) Let the roots of equation ( 2 y − 1) = 0 Now, αβ = ab − ac − bc x + 1 , respectively.
If, ⇒ y 2 − 9 y − 3 y + 27 = 0 72. (a) We have, 4x2 − 1 ≤ 0
x2 − 2x + 4 = 0 be α and β. 1 x x 1 = ab − ( a + b ) c Then, x2 + ( x + 1)2 = 221
⇒ y= ⇒ y= ⇒ = ⇒ ( y − 3) ( y − 9) = 0 ⇒ y = 3, 9 ⇒ ( 2x)2 − 1 ≤ 0
Then, α + β = 2 and αβ = 4 x+3 x+3 2 ( a + b)
2 = ab − ( a + b ) ⇒ 2x2 + 2x + 1 = 221 ⇒ ( 2x − 1) ( 2x + 1) ≤ 0
2 ⇒ 3x = 3 or 3x = 9 = 32
On taking, α → 2α and β → 2β On squaring both sides, we get ∴ x = 1 or x = 2 ⇒ 2x2 + 2x − 220 = 0 So, either ( 2x − 1) ≥ 0
[from Eq. (i)]
2α + 2β = 4 and 2α ⋅ 2β = 4 × 4 = 16 x 1 ⇒ x2 + x − 110 = 0
= 2ab − ( a 2 + b 2 + 2ab ) ( a2 + b2 ) 59. (a) Let the roots of the given equation be and ( 2x + 1) ≤ 0 …(i)
So, the new equation becomes x+3 4 = =−
2 2 α and β. ⇒ ( x + 11) ( x − 10) = 0 or ( 2x − 1) ≤ 0
x2 − 4x + 16 = 0 ⇒ 4x = x + 3 ⇒ x = 1 ∴ α + β = a and αβ = b ( 2x + 1) ≥ 0
55. (b) Let the roots of the equation ⇒ x = − 11 and x = 10 and …(ii)
45. (a) Since, α and β are the roots of the or y + 2=0 ⇒ y=−2 Now, |α − β | = (α + β )2 − 4αβ x ≠ −11 [Qnumbers are natural] 2x ≥ 1 and 2x ≤ − 1
kx2 − 5x + 6 = 0 be α and β. From Eq. (i),
equation x2 + px + q = 0 Since, y cannot be negative. So, for x = 10, next consecutive natural 1 −1
5
∴ α + β = and αβ =
6 ⇒ x ≥ and x ≤
∴ α + β = − p and αβ = q Hence, x = 1 is the required solution. = a 2 − 4b number 2 2
k k = x + 1 = 10 + 1 = 11
α 2 which is not possible.
−1 −1 1 1 49. (c) Given, 4x − 3 ⋅ 2x + 2
+ 32 = 0 = ⇒ α= β
2
60. (a) x2 − x − 2 = 0
Now, −α −β = −  +  Given, 66. (b) Let the consecutive positive odd From Eq. (ii), ( 2x − 1) ≤ 0
α β ⇒ 22 x − 3 ⋅ 2x ⋅ 22 + 32 = 0 β 3 3
( x − 2) ( x + 1) = 0 ⇒ x = 2,−1 integers be 2x + 1 and 2x + 3,
 α + β  2 5 2 2 6 and ( 2x + 1) ≥ 0
=−
p ⇒ 22x − 12 ⋅ 2x + 32 = 0 ∴ β + β = and β = so ( 2x + 1)2 + ( 2x + 3)2 = 130
 = So, both the roots are integers.
2x ≤ 1 and 2x ≥ − 1
 αβ  q ⇒ 22x − 8 ⋅ 2x − 4 ⋅ 2x + 32 = 0
3 k 3 k ⇒ ( 4x2 + 4x + 1)
61. (c) x2 − 6x + 5 = 0 ⇒ ( x − 5) ( x − 1) = 0 1 −1

5 5
β = and β 2 =
9 x ≤ and x ≥
 1   1 1 1 ⇒ ( 2 x − 8) ( 2 x − 4) = 0 ⇒ x = 5 or 1 + ( 4x2 + 12x + 9) = 130
and −  −  = = 3 k k 2 2
 α   β  αβ q ⇒ x2 + 2x − 15 = 0 −1 1
Either, 2x = 8 ⇒ x = 3 3 9 9 9 Also, |x − 3 | = 2 ⇔ ( x − 3 ) = 2 ∴ ≤ x ≤ is the required solution set.
or 2x = 4 ⇒ x = 2 β = and β 2 = ⇒ = 2 2
Hence, required equation is k k k2 k or − ( x − 3) = 2 ⇒ x = 5 or x = 1 ⇒ ( x + 5) ( x − 3) = 0
x 2 − ( − α −1 − β −1 ) x + ( − α −1 ) 50. (a) Since, α and β are the roots of the ⇒ x = 3, x = − 5 , but x ≠ − 5 73. (b) Here, 3x + 2 y = 12 passes through
⇒ k = 1 and k ≠ 0 ∴ x2 − 6 x + 5 = 0
−1 equation x2 − x − 1 = 0. the points ( 0, 6) and ( 4, 0) . So, its
(− β ) = 0 [Q leading coefficient is never zero.] and |x − 3 |= 2 are equivalent. [Q integers are positive.] graph is line AB. Also, 3x + 2 y ≤ 12 as
p 1 ∴ α + β = 1 and αβ = − 1
⇒ x2 − x + = 0 56. (d) Let α and β be the roots of the 1 ∴ Two consecutive integers are the shaded region is below AB.
q q Now, α 4 + β 4 = (α 2 + β 2 ) 2 − 2 (αβ )2 62. (a) Let the number are x and .
x = 0 and y = 0 are the Y-axis and
equation 7x2 + 12x + 18 = 0. x 2x + 1 = 7 and 2x + 3 = 9
⇒ q x 2 − px + 1 = 0 = [(α + β ) 2 − 2αβ ] 2 − 2 (αβ ) 2 X-axis, respectively. x ≥ 0 and y ≥ 0
−12 18 1 10 x2 + 1 10 i.e. 7 and 9.
= ( 1 + 2) 2 − 2 = 9 − 2 = 7 ∴ α+β= and αβ = Then, x + = , = implies the region on the right hand
46. (c) Since, one root of the equation 7 7 x 3 x 3 67. (b) Here, 2x + 1 ≥ 7 ⇒ 2x ≥ 7 − 1 side of Y-axis and region above X-axis,
ax2 + x − 3 = 0 is − 1 . 51. (c) Let the roots of equation
⇒ α 2 + β 2 + 2αβ =
144 ⇒ 3x2 − 10x + 3 = 0 respectively.
1 ⇒ 2x ≥ 6 ⇒ x ≥ 3
∴ a ( − 1) 2 + ( − 1) − 3 = 0 ⇒ a = 4 ax2 + bx + c = 0 be − α and − . 49 ⇒ 3x 2 − 9x − x + 3 = 0 The three portion x ≥ 0, y ≥ 0 and
α x −1
∴ Product of roots =
c 144 36 108 ⇒ 3x ( x − 3) − 1 ( x − 3) = 0 68. (c) Here, ≥4 3x + 2 y ≤ 12 intersects to give the
∴ 4x 2 + x − 3 = 0 ⇒ α2 + β2 = − =− 3
a 49 7 49 shaded portion OAB.
Let other root of this equation be α. ⇒ (3x − 1) ( x − 3) = 0
( − α )  −  = ⇒ x − 1 ≥ 12 ⇒ x ≥ 13
1 c c
⇒ 1= ⇒c = a −
108
Now, product of roots =
c  α a ∴
1
x= ,x=3 74. (d) Given, x 2 + 3x − 10 > 0
a α2 + β2 49 6 69. (b) Here, 3x + 2 ≤ 5x − ( 4 − x)
a ∴ = = − = −6 : 7 3 ⇒ x 2 + 5x − 2x − 10 > 0
3 3 52. (d) Let the roots of the quadratic αβ 18 7 ⇒ 3x + 2 ≤ 5x − 4 + x
∴ − 1⋅ α = − ⇒ α= equation be α and β. 7 63. (b) Let the smaller part = x and greater ⇒ ( x + 5) ( x − 2) > 0
4 4 part = 16 − x ⇒ 3x + 2 ≤ 6x − 4 Either, ( x + 5) > 0 and ( x − 2) > 0
Then, α + β = 9 and αβ = 9 x ( x − 1) − (m + 1) x ⇒ 3x − 6x ≤ − 4 − 2 ⇒ − 3x ≤ − 6
47. (b) Since, the roots of equation Hence, equation is 57. (d) Given, = By given condition, or ( x + 5) < 0 and ( x − 2) < 0
( x − 1)(m − 1) m ⇒ − x≤ − 2 ⇒ x≥ 2
( a 2 + b 2 ) x2 − 2( ac + bd ) x x2 − (α + β ) x + (αβ ) = 0 2 ( 16 − x)2 − x2 = 164 ⇒ x > 2 or x < −5
⇒ m ( x2 − x − m − 1)
+ ( c 2 + d 2 ) = 0 are equal. ⇒ x 2 − 9x + 9 = 0 ⇒ 2 ( 256 + x2 − 32 x) − x2 = 164 70. (c) Here, 2x + 3 ≥ 8 ⇒ 2x ≥ 8 − 3 75. (a) x2 − 5x + 6 ≥ 0
= x (mx − x − m + 1) 5
∴ B 2 = 4 AC 53. (c) Given, log ( x2 − 6x + 45) = 2 ⇒ x2 − 64 x + 348 = 0 ⇒ 2x ≥ 5 ⇒ x ≥ ⇒ ( x − 2)( x − 3) ≥ 0
10 ⇒ mx2 − mx − m (m + 1) 2
⇒ ( x2 − 6x + 45) = 102 = 100 ⇒ ( x − 58) ( x − 6) = 0
⇒ 4 ( ac + bd )2 = 4 ( a 2 + b 2 ) ( c 2 + d 2 ) = mx2 − x2 − mx + x Again, 3x + 1 ≤ 12 + – +
⇒ x2 − 6x − 55 = 0 ⇒ x = 58, x = 6
⇒ a 2 c 2 + b 2 d 2 + 2abcd = a 2 c 2 ⇒ x2 − x − m (m + 1) = 0 ⇒ 3x ≤ 11 ⇒ x ≤
11 2 3
⇒ x2 − 11x + 5x − 55 = 0 Here x ≠ 58 ⇒ x ≤ 2 or x ≥ 3
+ a 2 d 2 + b 2 c2 + b 2 d 2 Let one root be α, then other root is 3
⇒ x ( x − 11) + 5( x − 11) = 0 ∴ x=6 By combining values, we get ⇒ x ∈ [ −∞ ,2] or [3, ∞ ]
⇒ ( ad − bc )2 = 0 also α.
⇒ ( x + 5) ( x − 11) = 0 1 and, hence larger part 5 11
∴ α + α = 1, ⇒ α = ≤ x≤ ⇒ x ∈ [ −∞ ,2] ∪ [3, ∞ ]
∴ ad = bc ∴ x = 11, − 5 2 = 16 − x = 16 − 6 = 10 2 3
MATHEMATICS Quadratic Equations and Inequalities 179 180 CDS Pathfinder

x−2 x −3 ⇒ m 2 − 3m + 1 ≤ m 2 + 3m − 1 III. x2 + x + 1 = 0 ; x = 1, x = − 1 89. (b) Let α and β be the roots of the ⇒ ( x2 + y 2 ) − 2x y = 25 −b c


76. (b) < ∴ α+β = = 1 and αβ = = −1
3x + 1 3x − 2 and m 2 + 3m − 1 ≤ − m 2 + 3m + 1 Put x = 1, we get 12 + 1 + 1 = 3 ≠ 0 quadratic equation ⇒ 97 − 2x y = 25 [from Eq. (i)] a a
x−2 x −3 ∴ LHS ≠ RHS x2 + px + q = 0. α2 + β2 (α + β )2 − 2αβ
⇔ − <0 ⇒ 6m ≥ 2 and 2m 2 ≤ 2 ⇒ 2x y = 72 =
3x + 1 3x − 2 ∴ x = 1 is not a solution. Given that, A starts with a wrong value (α 2 − β 2 )(α − β ) (α − β )2 (α + β )
1 ∴ x y = 36
( x − 2)(3x − 2) − ( x − 3)(3x + 1) ⇒ m≥ and −1 ≤ m ≤ 1 of p and obtains the roots as 2 and 6.
⇔ <0 3 Put x = −1, we get 1 (α + β )2 − 2αβ
(3x + 1)(3x − 2) 1 But this time q is correct. 93. (a) Given that, x + = 2 …(i) =
∴ ≤m≤ 1 ( −1)2 + ( −1) + 1 = 1 − 1 + 1 x [(α + β ) 2 − 4αβ ](α + β )
7 3 i.e. Product of roots
⇔ <0 = 1≠ 0 = q = α ⋅ β = 6 × 2 = 12 ...(i) On squaring both sides, we get
(3x + 1)(3x − 2) 80. (d) Given equation, x 2 − 3x + 2 = 0 ( 1) 2 − 2 × −1 3
∴ LHS ≠ RHS and B starts with a wrong value of q and
2
 x + 1  = 4 ⇒ x2 + 1 + 2 = 4 = =
⇔ (3 x + 1 ) (3 x − 2 ) < 0 ⇒ x 2 − 2x − x + 2 = 0 ∴ x = − 1 is not a solution.   [( 1) 2 − 4 × −1] × 1 5
gets the roots as 2 and − 9. But this time  x x2
⇔ 3  x +  3  x −  < 0 ⇒ ( x − 2)( x − 1) = 0 ⇒ x = 2, 1
1 2 IV. 9x 2 − 3x − 2 = 0 p is correct. 1 97. (d) Given,
 3  3 Let α = 1 and β = 2 ⇒ x2 + 2 = 2 …(ii)
Put x = −1 / 3 i.e. Sum of roots
 1  2
x x2 = 6 + 6+ 6+ 6+ L ∞
⇔ x +  x −  < 0 ∴ α + 1 = 2 and (β + 1) = 3. ⇒ 9 ( −1 / 3 ) 2 − 3 ( −1 / 3 ) − 2 = 0 = p = α + β = − 9 + 2 = − 7 ...(ii) 2
Now,  x −  =  x2 + 2  − 2
 3  3 1 1 So, x 2 = 6 + x2 ⇒ x2 = 6 + x
Now, sum of roots = 2 + 3 = 5 ∴ LHS = RHS We know that,  x  x 
– and product of roots = 2 × 3 = 6 Now, put x = 2 / 3 (α − β )2 = (α + β )2 − 4αβ ⇒ x2 − x− 6 = 0
+ + = 2 − 2 = 0 [from Eq. (ii)]
∴ Required equation ⇒ 9 ( 2 / 3)2 − 3 ( 2 / 3) − 2 = 0 = ( −7)2 − 4 × 12 = 49 − 48 = 1
–∞ –1/3 2/3 +∞ 1 ⇒ x 2 + 2x − 3x − 6 = 0
= x 2 − (sum of roots) ∴ LHS = RHS [from Eqs. (i) and (ii)] ∴ x− = 0
−1 2
⇔ < x< + product of roots = 0 ⇒ α −β = 1 ...(iii)
x ⇒ x( x + 2) − 3( x + 2) = 0
3 3 84. (b) Given, h = −16 t 2 + 64 t + 80
⇒ x 2 − 5x + 6 = 0 From Eqs. (ii) and (iii), α = − 3 and 94. (b) Let the number of columns be x. ⇒ ( x − 3)( x + 2) = 0
− 1 2 put t = 1
So, x ∈  ,  Hence, the equation is neither I nor II. ⇒ h = −16 ( 1) 2 + 64 ( 1) + 80 = 128 m β=−4 Then, number of rows = x + 4 ∴ x = 3, −2
 3 3  which are correct roots. According to the question,
81. (c) I. Every quadratic equation has two 85. (c) By rearranging, we get 98. (d) Given, a x 2 + b x + b = 0
77. (c) Here, as x > 0 and y > 0 so both are roots, which may or may not be real. h = −16 ( t 2 − 4t − 5) 90. (a) Let one root be α, then other roots is x( x + 4) × 5 − 2 = 158 b b
⇒ x2 + x + = 0
positive and satisfies x + 2 y ≤ 3. II. x2 − 4x + 2 = 0 has integral ⇒ h = −16 [( t − 2)2 − 9] α2 ⇒ 5x( x + 4) = 160 ⇒ x( x + 4) = 32 a a
∴ When, x = 3 and y = 1, we get −b
coefficients but does not have ⇒ h = −16 ( t − 2)2 + 144 Given equation is x − bx + c = 0
2 ⇒ x2 + 4x − 32 = 0 ∴ Sum of roots, α + β =
x + 2 y = 5. integral roots. when the height is maximum, t = 2 ( −b) a
∴ Sum of roots = α + α 2 = − ⇒ x2 + 8x − 4x − 32 = 0
Clearly, this does not satisfy x + 2 y ≤ 3 III. Since, discriminant = b − 4ac
2 b
∴ maximum height = 144 m 1 and products of roots, αβ =
⇒ x ( x + 8) − 4 ( x + 8) = 0 a
When x = 1 and y = 1, If a and c have opposite sign, then 86. (a) When the ball hits the ground h = 0 ⇒ α(α + 1) = b ...(i)
b 2 − 4ac ≥ 0 c ⇒ ( x + 8)( x − 4) = 0 Hence, product of roots is not equal to
then, x + 2 y ≤ 3. ⇒ −16 t 2 + 64t + 80 = 0 and product of roots = α ⋅ α =
2

∴ The quadratic equation has real ⇒ t 2 − 4t − 5 = 0 1 So, x = 4 as x = −8 is not possible. the sum of roots,
So, x = 1, y = 1 is one of the solutions. ⇒ α 3 = c ⇒ α = c1 / 3 ...(ii) So, statement I is not correct.
roots ⇒ t2 −5 t + t −5 = 0 ∴ Number of rows
78. (d) As, ( 0, 0 ) satisfies x + 3 y ≤ 3. Now, for roots to be real and unequal,
82. (c) I. x2 +
1
=2 ⇒ ( t − 5)( t + 1) = 0 ⇒ t = 5 , − 1 From Eqs. (i) and (ii), = x+ 4 = 4+ 4 =8
And ( 0, 0 ) satisfies x + y ≤ 2. D>0
x2 Since, the time cannot be negative. 1/3
(c1/3
+ 1) = b ...(iii)
95. (c) Given, x2 + 2 + 8 x +  + 14
c 1 1 ⇒ b 2 − 4 ac > 0
Clearly, shaded region is the position or x4 − 2x2 + 1 = 0 So, t = 5 sec 
common to the line On cubing both sides, we get x x ⇒ b2 − 4 a ( b) > 0 [Q c = b]
is not a quadratic equation. 87. (a) Let f ( x ) = x 2 − 2ax + a 2 + a − 3 c ( c1 / 3 + 1)3 = b 3 1
Let x + = y
x + 3 y = 3 and below it and that on the 3
...(i) ⇒ b 2 − 4ab > 0 ⇒ b 2 > 4ab
line x + y = 2 and below it. II. x + = x2 f (3) > 0 ⇒ 9 − 6a + a 2 + a − 3 > 0 …(i) ⇒ c { c + 1 + 3 c1 / 3 ( c1 / 3 + 1)} = b 3
x
x On squaring both sides, we get ∴ b > 4a
So, shaded region is the solution set Also, since roots are real
x2 + 3 ⇒ c ( c + 1 + 3b ) = b 3 [from Eq. (iii)] 2 So, if b > 4a, then roots are unequal and
⇒ = x2 ∴ B 2 − 4 AC ≥ 0 ∴  1 1
x +  = x + 2 + 2 = y
2 2
x + y ≤ 2, x + 3 y ≤ 3, x ≥ 0, y ≥ 0. ⇒ b 3 = 3 bc + c 2 + c real, so statement II is not always true, it
x  x x
79. (c) Given, a + b = 2m 2
…(i) or x + 3 = x3
2 ⇒ 4 a 2 − 4 ( a 2 + a − 3) ≥ 0 …(ii) depends on values of a and b.
91. (a) Let α and β be the roots of the given 1
b + c = 6m …(ii) ⇒ x3 − x2 − 3 = 0 From Eqs. (i) and (ii), we have ⇒ x + 2 =y −2
2 2
...(ii) 99. (d) Given, Ax2 + Bx + C = 0 ...(i)
quadratic equation 2x2 − 11x + 5 = 0. x
a+ c=2 is not a quadratic equation. a 2 − 5 a + 6 > 0 and a − 3 ≤ 0 Since, the given roots are − 1 and 1 .
and …(iii) ( −11) 11
III. 2x2 − x + 2 = x2 + 4x − 4 ⇒ a 2 − 3a − 2a + 6 > 0 ∴ α+β=− = ...(i) Now, on putting values from Eqs. (i) and
∴ Sum of roots = − 1 + 1 = 0
On adding Eqs. (i), (ii) and (iii), we get 2 2 (ii) in the given equation, we get
or 2x2 − x + 2 − x2 − 4x + 4 = 0 ⇒ ( a − 3)( a − 2) > 0 and product of roots = 1 × ( − 1) = − 1
2 ( a + b + c ) = 2m 2 + 6m + 2 and α ⋅β =
5
...(ii) y 2 − 2 + 8 y + 14 = y 2 + 8 y + 12 ⇒ quadratic equation is x2 −
or x2 − 5x + 6 = 0 ⇒ a ≤ 3 and a < 2 or a > 3
⇒ a + b + c = m 2 + 3m + 1 …(iv) 2 (sum of roots) x + product of roots = 0
is a quadratic equation. = y 2 + 2 y + 6 y + 12
On subtracting Eq. (ii) from Eq. (iv), we Combining the above inequalities, we Now, (α − β )2 = (α + β )2 − 4αβ ⇒ x2 − 0 ⋅ x − 1 = 0 ⇒ x2 − 1 = 0
IV. x3 + 6x2 + 2x − 1 = 0 = y ( y + 2) + 6 ( y + 2)
have a < 2 2
=   − 4  
get is not a quadratic equation. 11 5 On comparing with above equation
a = m − 3m + 1
2 88. (a) Given, 7x − 50x + k = 0
2
 2  2 = ( y + 6) ( y + 2)
Hence, only III statement is correct. from Eq. (i), we get
=  x + + 6  x + + 2
2 1 1
On subtracting Eq. (iii) from Eq. (iv), Here, a = 7, b = − 50 and c = k 121 40 81  9  A = 1 and C = −1
83. (b) I. 3x 2 − 2x − 1 = 0 = − = =     
we get Since, α and β are the roots of the given 4 4 4  2 x x
Put x = 1 ⇒ 3 ( 1)2 − 2 ( 1) − 1 = 0 So, A and C are of opposite sign.
So, one factor is  x + + 6 .
1
b = m 2 + 3m − 1 ∴ LHS = RHS
equation 9
∴Difference of roots = (α − β ) = = 4.5
−b 50  x  100. (d) Given, ( a 2 − bc )x2 + 2( b 2 − ac )x
On subtracting Eq. (i) from Eq. (iv), we ∴α + β = ⇒α+β= 2
II. x 2 + 2x − 4 = 0 a 7 + ( c 2 − ab ) = 0
get 1 92. (b) Let the two numbers be x and y. 96. (b) Given, x2 − x − 1 = 0
Put x = −2 2 ⇒ β= [Q α = 7, given] Since, the given roots are equal.
c = − m 2 + 3m + 1 7 Now, by given condition, Here, a = 1, b = −1 and c = −1 ∴ D=0
⇒ ( − 2 2 ) 2 + 2( − 2 2 ) − 4 = 0
As, a ≤ b and b ≤ c c 1 k x2 + y 2 = 97 Q α and β are the roots of the equation i.e. [ 2( b 2 − ac )] 2 − 4( a 2 − bc )
∴ LHS = RHS and αβ = ⇒ 7× = ⇒ k=7 …(i)
a 7 7 ( x − y )2 = 25 ( c 2 − ab ) = 0
and …(ii) x2 − x − 1 = 0
MATHEMATICS Quadratic Equations and Inequalities 181 182 CDS Pathfinder

⇒ 4( b 4 + a 2 c 2 − 2ab 2 c ) When mistake is done in constant term, II. From Eq. (ii), m 2 n2 = b 2 111. (a) Let α and β be the roots of equation 115. (b) We have, A( x ) = x 2 + px + q ⇒ 3x 2 − 7x − 30 − x 2 + 10x
−4( a 2 c 2 − bc 3 − a 3 b + ab 2 c ) = 0 then the roots of equation are 8 and 2. ⇒ b2 = d ax 2 + bx + c = 0. Q ( x − m ) and ( x − km ) are factors of + 7x − 2x 2 − 20
∴ Equation is ( x − 2)( x − 8) −b A( x ), then m and km are roots of
⇒ 4 b 4 + 4a 2 c 2 − 8ab 2 c − 4a 2 c 2 Hence, statement II is correct. Then, α+β= = ( 2x − 10) 2x 2 − 7x − 5
= x2 − 10x + 16 ...(ii) a A (x ) = 0
+ 4bc 3 + 4a 3 b − 4ab 2 c = 0 106. (a) If a is positive and b 2 − 4ac ≤ 0, c
and αβ = ∴ m + km = − p ⇒ m ( k + 1) = − p ⇒ ( 10x − 50)
⇒ 4b 4 − 12ab 2 c + 4bc 3 + 4a 3 b = 0 From Eqs. (i) and (ii), we get then the sign of quadratic polynomial a −p
x2 − 10x + 9 = 0 is the correct equation. ax2 + bx + c is always positive. ⇒ m= …(i) = ( 2x − 10) 2x 2 − 7x − 5
⇒ b + c + a − 3abc = 0
3 3 3
 1
2 2
( k + 1)
α + β =   +  
100 1
107. (c) We have, x + > 50 Now,
∴ a + b + c = 3abc
3 3 3 103. (c) Given equation, α β and m ⋅ km = q ⇒ m k = q 2
⇒ 10( x − 5) = 2( x − 5) 2x 2 − 7x − 5
x
101. (a) Given m and n are the roots of the 7( x + 2a )2 + 3a 2 = 5a ( 7x + 23a ) p2
and 1 ≤ x ≤ 100 ⇒ x 2 − 50x + 100 > 0 β2 + α2 ⇒ ⋅k = q [from Eq. (i)]
given equation ax2 + bx + c = 0 ⇒ 7( x2 + 4a 2 + 4ax ) + 3a 2 ⇒ α+β= ( k + 1) 2 ⇒ 2x 2 − 7x − 5 = 5 …(i)
50 ± 2500 − 400 (αβ ) 2
∴ Sum of roots = m + n = − b /a …(i) = 35ax + 115a 2 ⇒ x = ⇒ ( k + 1)2 q = kp 2 Again, on squaring both sides of Eq. (i),
c 2
and product of roots = mn = …(ii) ⇒ 7x2 − 7ax − 84a 2 = 0 (α + β ) − 2αβ2 we get
50 ± 10 21 116. (c) Given points are P (5, − 1), Q(3, − 2)
a = =
⇒ x2 − ax − 12a 2 = 0 (αβ ) 2 and R ( 1, 1). 2x 2 − 7x − 5 = 25
m 2 + 1 n2 + 1 2
Now, + ⇒ ( x + 3a )( x − 4a ) = 0 If points P,Q and R lie in the solution of ⇒ 2x 2 − 7x − 30 = 0
x < 25 − 5 21 or x > 25 + 5 21 − b b / a − 2c / a
2 2 …(ii)
m n ⇒ = inequations x + y ≤ 4 and x − y ≥ 2,
m 2 n + n + mn2 + m ⇒ x = − 3a and x = 4a ⇒ x < 2. 087 or x > 47. 91 a c2 / a 2 If α and β are roots of Eq. (ii), then
= then points P, Q and R satisfy these
mn Since, m and n are the roots of the given ⇒ x = 1 and 2 or x = 48, 49,… ,100 inequations. −30
mn (m + n) + (m + n) equation. ⇒ ab 2 − 2a 2 c = − bc 2 αβ = = − 15
= ∴ Number of total values of For point P (5, − 1), 5 − 1 ≤ 4 [true] 2
mn Let m = 4a and n = − 3a (Q m > n) ⇒ ab 2 + bc 2 = 2a 2 c
x = 2 + 53 = 55 5 + 1 ≥ 2 [true]
(m + n)(mn + 1) ∴ 3 m − n = 3 ( 4a ) − ( − 3a ) 119. (a) Since, the roots of equation
= 112. (d) Here, a = 1, b = − p , c = q For point Q (3, − 2), 3 − 2 ≤ 4 [true]
mn 108. (c) We have, x 2 − 4x − log N = 0 lx 2 + mx + m = 0 are in the ratiop : q.
= 12 a + 3 a = 15 a 10
and a ′ = 1, b ′ = q , c ′ = − p
We know that, roots are real, if D ≥ 0 3 + 2 ≥ 2 [true]
−  + 1
b c Then, we can take roots as pk and qk.
  − b( a + c ) 104. (a) Given equation is px + qx + r = 0.
2
= a ac = ⇒ 16 − 4( − log10 N ) ≥ 0
Applying common root condition, and for point R ( 1, 1), 1 + 1 ≤ 4 [true] ∴ Sum of roots = pk + qk = −
m
Let one root of the equation be α.
ac ( a ′ c − ac ′ )2 = ( bc ′ − b ′ c )( ab ′ − a ′ b ) 1 − 1 ≥ 2 [false] l
a Then, other root = 3α ⇒ 16 + 4 log N ≥ 0 m
10
⇒ ( q + p )2 = ( p 2 − q 2 ) ( q + p ) So, the point R ( 1, 1) does not satisfy the ⇒ ( p + q) k = − …(i)
[using Eqs. (i) and (ii)] q ⇒ log N ≥ − 4 l
∴ Sum of roots = α + 3α = − 10 inequations.
m 2 + 1 n2 + 1 p 1 ⇒ ( p + q) = ( p − q) ( p + q) and product of roots
and × −q −q ⇒ N ≥ 10 −4
⇒ N ≥ Hence, points P and Q lie in the solution m
m n
⇒ 4α = ⇒ α= ...(i) 10000 ⇒ p−q=1 = ( pq ) k 2 = …(ii)
of inequations. l
(m 2 + 1)( n2 + 1) p 4p 1 ⇒ p − q − 1= 0
= r Minimum value of N = a + 2b + a − 2b
mn and product of roots = (α ) ⋅ (3α ) = 10000 117. (a) Given, x = On dividing Eq. (i) by Eq. (ii), we get
m 2 n2 + n2 + m 2 + 1 p 113. (c) Let α and α 2 be the roots of equation a + 2b − a − 2b ( p + q )k
= − q 2 109. (c) Let x = 4+ 4 − 4 + ... By rationalising, we get =−1
mn r   r x 2 + px + q = 0. ( pq ) k 2
(mn)2 + (m + n)2 − 2mn + 1 ⇒ 3α 2 = ⇒ 3  =
= p  4p  p Then, α ⋅ α2 = q ( a + 2b )2 + ( a − 2b )2
∴ x = 4+ 4−x  p + q m/l
mn
[from Eq. (i)] ∴ α = ( q) 1/ 3 …(i) + 2 ( a + 2b )( a − 2b ) ⇒   =−k=
2 2
 c  +  − b  − 2  c  + 1 ⇒ x2 = 4 + 4−x  pq  ( p + q)
3q 2 r x =
 a  a   a ⇒ = ⇒ 3q 2 p = 16 p 2 r and α+α =−p 2
( a + 2b ) − ( a − 2b )
2 2
[using Eq. (i)]
= 16 p 2 p ⇒ ( x 2 − 4)2 = 4 − x
c ∴ ( q )1 / 3 + ( q )2 / 3 = − p
⇒ ( x 2 − 4)2 + x = 4
…(ii)
a + 2b + a − 2b + 2 a 2 − 4b 2 ( p + q )2 m p+ q m
a ⇒ 3 q 2 = 16 pr …(i) ⇒x = ⇒ = ⇒ =±
13 + 1 On cubing both sides, we get a + 2b − a + 2b pq l pq l
c 2 + b 2 − 2ac + a 2 For option (c), x =
= 105. (b) I. Given, m and n are the roots of the p+ q m
ac equation 2 q + q 2 + 3 q ( q1 / 3 + q 2 / 3 ) = − p 3 2a + 2 a 2 − 4b 2 ⇒ =−
b 2 + ( a − c )2 From Eq. (i), ⇒x = pq l
= x2 + ax + b = 0. 2 ⇒ q + q 2 + 3 q ( − p) = − p3 4b
ac
∴ Sum of roots, m + n = − a ...(i)   13 + 1  2
13 + 1 a+ a 2 − 4b 2 ⇒
p
+
q
+
m
=0
We know that, quadratic equation can LHS =    − 4 + [from Eq. (ii)] ⇒x = q p l
be written as and product of roots, mn = b ...(ii)   2   2 ⇒ q + q 2 + p 3 = 3 pq
2b
x2 − (Sum of roots) x + Product of roots Also, given m 2 and n2 are the roots 2 ⇒ bx − ax + b = 0
2
120. (d) We have, 1 +
1

1
≥0
 13 + 1 + 2 13 − 16  13 + 1 ⇒ p 3 + q 2 + q = 3 pq
=0 of the equation =  + x x2
− b( a + c )  4 2 118. (a) Given, 3x 2 − 7x − 30
⇒ x2 −  x − cx + d = 0.
2   114. (a) Consider the line, x + x −1
2
 x ⇒ ≥0
 ac  = 4 = RHS 2x + 6 y = 21 = ( x − 5) + 2x 2 − 7x − 5 x2
∴ m 2 + n2 = c ...(iii) …(i)
 b 2 + ( a − c )2  110. (c) We have, −2 ≤ x ≤ 1 …(i)
+   =0 and m n =d
2 2
...(iv) The point (0, 0) does not satisfy Eq. (i) On squaring both sides, we get As, x 2 ≥ 0, then x 2 + x − 1 ≥ 0
 ac  −1 ≤ y ≤ 2
On squaring both sides of Eq. (i), we but the point (0,0) satisfy the equation 3x 2 − 7x − 30 = ( x − 5)2 If x 2 + x − 1 = 0,
⇒ acx + b( a + c )x + b + ( a − c ) = 0
2 2 2
−2 ≤ − y ≤ 1 …(ii) 2x + 6 y ≤ 21 . + ( 2x 2 − 7x − 5) −1 ± 5
get Q
⇒ acx2 + ( ab + bc )x + b 2 + ( a − c )2 = 0 then x =
m 2 + n2 + 2mn = a 2 and 3≤z≤ 6 Now, consider the line, + 2( x − 5) 2x 2 − 7x − 5 2
102. (c) When mistake is done in first degree [from Eqs. (i) and (ii)] 6 ≤ 2z ≤ 12 …(iii) 5x − 2 y = 10 … (ii) ⇒ 3x 2 − 7x − 30 = x 2 + 25 − 10x   −1 + 5     −1 − 5  
term, then the roots of the equation are ∴ x −    x −   ≥ 0
⇒ c + 2b = a 2 ⇒ c = a 2 − 2b On adding Eqs. (i), (ii) and (iii), we get The point (0, 0) does not satisfy   2    2 
− 9 and − 1. + 2x 2 − 7x − 5
⇒ 2b − a 2 = − c −2 − 2 + 6 ≤ x − y + 2z ≤ 1 + 1 + 12 −1 − 5 −1 + 5
∴ Equation is ( x + 1) ( x + 9) Eq. (ii) but the point (0,0) satisfy the + ( 2x − 10) 2x 2 − 7x − 5 ⇒ x≤ or x ≥
= x2 + 10x + 9 ...(i) Hence, statement I is not correct. 2 ≤ k ≤ 14 equation 5x − 2 y ≤ 10. 2 2
19
MATHEMATICS Set Theory 183 184 CDS Pathfinder

(iii) The set A = {0, 1, 4, 9, 16, . . . } can be written as 7. Subsets Let A and B be two sets. If every element
A = { x : x = n 2 , n ∈ Z}, where Z is the set of of A is an element of B, then A is called a subset of B.
integers. If A is subset of B, then we can write A ⊆ B which is
read as ‘A is a subset of B’ or ‘A is contained in B’.
Types of Sets (i) Every set is a subset of itself i.e. A ⊆ A, B ⊆ B.
1. Empty set A set which does not contain any (ii) Empty set is a subset of every set i.e. φ ⊆ A.
element is called an empty set or null set or void set.
e.g. Let A = { 2, 4, 6} and B = {6, 4, 2, 8} .
It is denoted by φ or { }.
Then, A ⊆ B but B ⊆/ A i.e. A is a subset of B but B is
e.g. A = set of all odd numbers divisible by 2

SET THEORY and B = { x : x ∈ N and 5 < x < 6}


The sets which have atleast one element are called
non void or non-empty set.
not a subset of A.
Note The total number of subsets of a finite set containing n
elements is 2 n.

2. Singleton set A set consisting of a single element is 8. Super set If A is a subset of B, then we say that B is
called a singleton set. e.g.
superset of A and we write B ⊇ A .
(i) The set {5} is a singleton set. e.g. If A = {1, 2, 3, 4} and B = {1, 2, 3, 4, 5, 6} ,
(ii) { x | x ∈ W and x + 6 = 6} = {0}
Usually (1-2) questions have been asked from this chapter. Generally questions are asked then, B ⊇ A .
which is a singleton set.
from these topics more results on operations on set. 9. Comparability of sets Two sets A and B are said to
3. Finite set A set which consists of a definite number
be comparable, if either A ⊂ B or B ⊂ A or A = B,
of elements, is called a finite set. Empty set is also a
finite set. e.g. otherwise A and B are said to be incomparable.
(i) The set {1, 2, 3, 4} is a finite set, because it contains e.g. Let A = {1, 2, 3}, B = {1, 2, 4, 6} and C = {1, 2, 4}
a definite number of elements i.e. only 4 elements. Since, A ⊆/ B and B ⊆/ A.
SET (ii) B = Set of vowels in English Alphabets So, A and B are incomparable but C ⊂ B and so
A well defined collection of objects, is called a set. The objects in a set are called its members or
= {a, e, i, o, u}; B and C are comparable sets.
elements. Sets are usually denoted by the capital letters A, B, C, X, Y and Z etc. And the elements of a set
(iii) A = Set of even prime natural numbers. 10. Proper subset If A ⊆ B and A ≠ B, then A is called
are denoted by small letters a, b, c etc. If x is an element of set A, we can write x ∈ A , which means that ‘x
⇒ A = { 2} a proper subset of B and we write A ⊂ B.
belongs to A’ or that x is an element of A.
If x does not belong to A, we can write, x ∉ A . e.g. Note The number of distinct elements contained in a set A is e.g. If A = {1, 2, 3, 4, . . . }
• The collection of vowels in English alphabet is a set A containing five elements namely a, e, i, o and u, called the cardinal number of A and is denoted as n( A ). and B = {. . . , −3, −2, −1, 0, 1, 2, 3, . . . . },
If A = { a, e, i , o, u}, then n ( A ) = 5.
where a ∈ A but b ∉ A. then A ⊂ B.
• The collection of first four prime numbers is a set A containing the elements 2, 3, 5 and 7, where 3 ∈ A 4. Infinite set A set which consists of infinite number
Note The total number of proper subset of a finite set containing n
but 1 ∉A. of elements is called an infinite set. It is represented elements is (2 n − 1).
by writing a few elements of the set followed by
Representation of Sets e.g. 11. Universal set If there are some sets under
Sets are generally represented by following two ways: (i) Set of square of natural numbers is an infinite set, consideration, then there happens to be a set which is
1. Roster or Tabular form or listing method In this method, all the elements of a set are listed, within because natural numbers are infinite and it can be a superset of each one of the given sets. Such a set is
curly braces { } being separated by commas. e.g. represented as {1, 4, 9, 16, 25, ...}. known as the universal set and it is denoted by U.
(i) If A is a set of first eight prime numbers, then A = { 2, 3, 5, 7, 11, 13, 17, 19} (ii) Set of all points in a plane. e.g.
(ii) If B is a set of squares of first five natural numbers, then B = {1, 4, 9, 16, 25} 5. Equal sets Two sets A and B are said to be equal, if • If A = {1, 2, 3}, B = { 2, 3, 4, 5, 7 } and
(iii) If A is a set of vowels of English alphabets, then A = { a, e, i, o, u} they have exactly the same elements and we write C = { 2, 4, 6, 8}, then U = {1, 2, 3, 4, 5, 6, 7, 8} is a
Note The order in which the elements are written in a set makes no difference and also the repeated elements are taken A = B. Otherwise, two sets are said to be unequal universal set for A, B and C.
only once each. and we write A ≠ B.
• For the set of all integers, the universal set can be
2. Set Builder form or Rule method In this method, instead of listing all the elements of a set, we write e.g. Let A = {1, 2, 3, 4} and B = { 4, 3, 1, 2}, then A = B
the set of rational numbers or the set of real
the set by some special property or properties satisfied by all its elements and write it as. because each element of A is in B and vice-versa.
numbers.
(i) The set B of all even natural numbers can be written as 6. Equivalent sets Two sets A and B are equivalent, if 12. Power set The collection of all subsets of a set A, is
B = { x : x is a natural number and x = 2n for n ∈ N} or B = { x : x ∈ N, x = 2n, n ∈ N} and neads it as their cardinal numbers are same i.e. n ( A ) = n ( B) and called power set of A and it is denoted by P ( A ) . In
‘B’ is the set consisting of all elements x such that x has the property of even natural number. The we write A ↔ B or A ~ B
symbol ‘:’ or ‘1’ stands or such that P ( A ), every element is a set.
e.g. Let A = {1, 2, 3} and B = { 4, 5, 6},
(ii) The set A = { 3, 5, 7, 9, 11}, then it is represented as e.g. Let A = {1, 2, 3}
then n ( A ) = n ( B)
A = { x : x = 2n + 1 where n ∈ N, n < 6} A ↔ B or A ~ B Then, P ( A ) = { φ , {1}, { 2}, { 3}, {1, 2}, { 2, 3}, { 3,1}, {1, 2, 3}}
MATHEMATICS Set Theory 185 186 CDS Pathfinder

Thus, A ∪ B = { x : x ∈ A or x ∈ B} Note The sets A − B, A ∩ B and B − A are mutually disjoint sets i.e.
Properties of Power Sets U Symmetric Difference of Two Sets the intersection of any of these two sets is an empty set.
In the given Venn diagram A ∪ B is
1. Each element of a power set is a set. Let A and B be two sets. The U
denoted by the shaded region.
2. If set A has n elements, then P ( A ) has 2n elements. A B symmetric difference of sets A and Laws of Algebra of Sets
e.g. If A = {1, 2, 3, 4} B is the set ( A − B) ∪ ( B − A ) and is
3. If A is an empty set φ or { }, then P ( A ) has just one and B = {1 , 2, 3, 5 , 7}, then A ∪ B = {1, 2, 3, 4, 5, 7} denoted by A ∆ B i.e. it is a set
1. Idempotent laws For any set A, we have
element i.e. P( A ) = { φ}. A–B B–A (i) A ∪ A = A (ii) A ∩ A = A
consisting of all those members of A
4. If A ⊆ B, then P ( A ) ⊆ P ( B) . Note • x ∈ A ∪ B ⇔ x ∈ A or x ∈ B 2. Identity Laws For any set A,
which are not in B or all those
• x ∉ A ∪ B ⇔ x ∉ A and x ∉ B • A ⊆ A ∪ B, B ⊆ A ∪ B. (i) A ∪ φ = A (ii) A ∩ U = A
which are in B but not in A.
EXAMPLE 1. The set S = {x ∈ N: x + 3 = 3} is a i.e. φ and U are identity elements for union and
2. Intersection of sets Let A and B be any two sets. Thus, A ∆ B = { x : x ∈ A but x ∉ B} ∪ { x : x ∈ B but x ∉ A }
a. null set b. singleton set intersection, respectively.
The intersection of A and B is the set of all those A ∆ B = ( A − B) ∪ ( B − A )
c. infinite set d. None of these 3. Commutative laws For any two sets A and B, we
elements which belongs to both A and B. It is The shaded part in given Venn diagram represents
Sol. a. Given, S = {x ∈ N : x + 3 = 3} denoted by A ∩ B and read as A intersection B. A ∆ B. have
∴ S = {} Thus, A ∩ B = { x : x ∈ A and x ∈ B} U e.g. If A = {1, 2, 3, 4, 5, 6, 7, 8} and B = {1, 3, 5, 6, 7, 8, 9}, (i) A ∪ B = B ∪ A (ii) A ∩ B = B ∩ A
So, S is a null set. A ∩ B is represented by the shaded ∴ A − B = { 2, 4} and B − A = {9}, then A ∆ B = { 2, 4, 9}. 4. Associative laws If A , B and C are any three sets,
region in the given Venn diagram A B
EXAMPLE 2. If A = {x : x is an odd integer} and then
B = {x : x 2 − 8x + 15 = 0 }. Then, which one of the e.g. If A = {1, 2, 3, 4} and Complement of a Set (i) ( A ∪ B) ∪ C = A ∪ ( B ∪ C )
B = {1, 3, 7, 9}, then A ∩ B = {1, 3}.
following is correct. e 2013 I Let U be the universal set and A be A’ U (ii) A ∩ ( B ∩ C ) = ( A ∩ B) ∩ C
a. A = B b. A ⊆ B c. B ⊆ A d. A ⊆ B C Note • x ∈ A ∩ B ⇔ x ∈ A and x ∈ B any set such that A ⊂ U . Then, the 5. Distributive laws If A , B and C are any three sets,
• x ∉ A ∩ B ⇔ x ∉ A or x ∉ B • A ∩ B ⊆ A and A ∩ B ⊆ B complement of A with respect to U is A
Sol. c. Given that, A = {x : x is an odd integer} then
the set of all those elements of U
and B = {x : x2 − 8x + 15 = 0} = {x : x2 − 5x − 3x + 15 = 0} 3. Disjoint sets Two sets A and B (i) A ∪ ( B ∩ C ) = ( A ∪ B) ∩ ( A ∪ C )
U which are not in A. It is denoted by
= {x : x ( x − 5) − 3( x − 5) = 0} are said to be disjoint, if they have A ′ or A C or U − A (ii) A ∩ ( B ∪ C ) = ( A ∩ B) ∪ ( A ∩ C )
no common element i.e. A B
= {x : ( x − 5)( x − 3) = 0} = {3, 5} Thus, A ′ = { x : x ∈U and x ∉ A } 6. De-Morgan’s laws If A and B are any two sets,
Since, B has two odd elements.
A ∩B=φ then
e.g. If A = {1, 2, 4}, B = { 3, 5, 6}, The shaded portion in the above Venn diagram shows
∴ B⊆A (i) ( A ∪ B) ′ = A ′ ∩ B ′ (ii) ( A ∩ B) ′ = A ′ ∪ B ′
then A ∩ B = φ the complement of set A. If A is a subset of the
EXAMPLE 3. If A = {a, b }, then power set of A is universal set U, then its complement A ′ is also a subset
So, A and B are disjoint sets.
of U. e.g. More Results on
a. {φ, a , b} b. {φ, a , b , A}
4. Difference of sets Let A and B be two sets. The • Let U = {1, 2, 3, 4, 5, 6, 7 , 8, …}and A = {1, 3, 5, 7 , . . . } . Operations on Sets
c. {φ, a , b , ab} d. {φ, { a }, {b }, A} difference of two sets is the set of all those elements
Then, the complement of A is 1. If A , B and C are any three sets, then
of A which do not belong to B. It is denoted by
Sol. d. A = {a, b} A ′ = U − A = { 2, 4, 6, 8, . . . } (i) A − ( B ∩ C ) = ( A − B) ∪ ( A − C )
A − B and read as A minus B.
The subsets of A are φ, {a}, {b} {a, b}. • If U = { a, b, c, d , e, f } and A = { a, e}, (ii) A − ( B ∪ C ) = ( A − B) ∩ ( A − C )
∴ A − B = { x : x ∈ A and x ∉B}
∴ P ( A) = {φ , {a}, {b}, {a, b} = {φ , {a}, {b}, A} then A ′ = { b, c, d , f }. (iii) A ∩ ( B − C ) = ( A ∩ B) − ( A ∩ C )
U
EXAMPLE 4. If A = {0, 1 2, 3, 4}, B = {1, 2, 3}, (iv) A ∩ ( B ∆ C ) = ( A ∩ B) ∆ ( A ∩ C )
VENN DIAGRAM A–B C = {3, 5, 7, 9}, then ( A − B) ∩ (B − C) is (v) ( A − B) ∪ ( B − A ) = ( A ∪ B) − ( A ∩ B)
‘Venn diagram’ is the diagrammetic A B a. {1, 2} b. { 0 , 4 } c. { 0 , 1, 2, 4 } d. φ 2. If A , B and C are finite sets and U be the finite
representation of various types of sets U
Sol. d. Given, A = { 0, 1, 2, 3, 4}, B = { 1, 2, 3} and C = {3, 5, 7, 9} universal set, then
and operations on sets. The shaded part in the given Venn diagram
A represents A − B. (i) n ( A ∪ B) = n ( A ) + n ( B) − n ( A ∩ B)
∴ A − B = { 0, 4} and B − C = { 1, 2}
The universal set is usually
Similarly, the difference B − A is (ii) n ( A ∪ B) = n ( A ) + n ( B) ⇔ A , B are disjoint sets.
represented by a rectangular region U ∴ ( A − B) ∩ ( B − C ) = φ
the set of all those elements of B (iii) n ( A − B) = n ( A ) − n ( A ∩ B)
and other subsets of the universal set i.e. n ( A − B) + n ( A ∩ B) = n ( A )
are represented by circles inscribed in it. Each inscribed
that do not belong to A. B–A Properties of Operations on Sets
∴ B − A = { x : x ∈ B and x ∉ A } (iv) Number of elements which belong to exactly one
circle represents a set (subset of universal set). A B 1. (A ′ )′ = A 2. A ∪ A ′ = U
of A or B
The shaded part in the given Venn diagram 3. A ∩A′ = φ 4. φ ′ = U = n ( A ∆ B) = n ( A ) + n ( B) − 2n ( A ∩ B)
Operations on Sets represents B − A . 5. U′=φ 6. A − B = A ∩ B ′ (v) If A, B and C are finite sets, then
1. Union of Sets Let A and B be any two sets. The e.g.
7. B−A = B∩ A′ 8. A − B ⊆ A n ( A ∪ B ∪ C ) = n ( A ) + n ( B) + n (C ) − n ( A ∩ B)
union of A and B is the set of all the elements of A and • If A = { 2 , 4, 6, 8, 10} and B = { 2, 4, 6, 12, 14}, then
9. B−A ⊆ B 10. A − B ≠ B − A − n (B ∩ C) − n (A ∩ C) + n (A ∩ B ∩ C)
all the elements of B, the common elements being A − B = {8, 10} and B − A = {12, 14}.
taken only once. It is denoted by A ∪ B and read as ‘A 11. A − φ = A and A − A = φ 12. A ⊆ B ⇒ A − B = φ (vi) n ( A ′ ∪ B ′ ) = n (( A ∩ B) ′ ) = n (U ) − n ( A ∩ B)
• If A = {1, 2, 3} and B = {1, 2, 3, 4}, then A − B = φ.
union B.’ 13. A −B = B−A ⇔A = B (vii) n ( A ′ ∩ B ′ ) = n (( A ∪ B) ′ ) = n (U ) − n ( A ∪ B)
MATHEMATICS Set Theory 187 188 CDS Pathfinder

EXAMPLE 5. If a set A contains 60 elements and ∴ n (E ∪ H ) = n (E) + n (H ) − n (E ∩ H ) A U U


14. The shaded region in the adjoining diagram is A
another set B contains 70 elements and there are 50 40 = n ( E ) + 22 − 12 ⇒ n ( E ) = 30 B B
elements in common, then how many elements does Number of people who speak English only U
A ∪ B contain? = n ( E ) − n ( E ∩ H ) = 30 − 12 = 18 A
a. 130 b. 100 c. 80 d. 70
Sol. c. Here, n ( A) = 60, n ( B) = 70 and n ( A ∩ B) = 50
Ordered Pairs (c) (d)
An ordered pair consists of two objects or elements in a (a) A ∪ A ′ (b) U (c) A ′ (d) A ∩ A ′ 20. Which one of the following is a correct statement?
Now, n( A ∪ B) = n( A) + n( B) − n( A ∩ B)
given fixed order.
= 60 + 70 − 50 = 130 − 50 = 80 15. The shaded region in the adjoining diagram (a) φ ∈ φ (b) φ ∉P (φ) (c) φ = P (φ) (d) φ ∈ P (φ)
If A and B are two sets and a ∈ A , b ∈ B, then the
21. Let A =  3, π , 2 , , − 5, 3 + 7 . The subset of A
represents 2
EXAMPLE 6. A group of 40 people who speaks either ordered pair of elements a and b is denoted by (a, b). U
 7 
english or Hindi, out of these 12 speak both English The natural numbers and their squares can be containing all the elements from it which are
and Hindi and 22 speak Hindi. How many people represented by ordered pair in the following way. irrational numbers is
speak only english not Hindi? A B
(1, 1), (2, 4), (3, 9), (4, 16), .... (a) 3, , − 5
2
(b) 3, π, , − 5, 3 +
2
7 
a. 30 b. 10 c. 18 d. 28
Two ordered pairs (a, b) and (c, d) will be equal, if and (a) ( A ∩ B)′ (b) A ∪ B (c) A − B (d) A ∩ B  7   7 
Sol. c. Let E be the set of students who speak English and H be the set only if a = c and b = d . (c) { π, 2 , 3 + 7} (d) {3, − 5}
of students who speak Hindi. 16. The shaded portion in the following Venn
Given, n ( E ∪ H ) = 40, n ( H ) = 22 and n ( E ∩ H ) = 12 diagram represents 22. If U = { x : x ∈ N }, A = { x : x is an odd number },
Note { a, b} = { b, a} but ( a, b ) ≠ ( b, a ) .
then A′ is equal to
B (a) {x : x is an even number} (b) {x : x is an odd number}
(c) {x : x is a natural number} (d) {x : x is an integer}

A 23. If two sets are disjoint, then their intersection is


PRACTICE EXERCISE (a) A ∪ (B ∩ C )
C

(b) A ∪ (B ∪ C )
(a) null set
(c) a infinite set
(b) singleton
(d) None of these

(c) ( A ∩ B) ∩ C (d) ( A ∩ B) ∪ C 24. If A and B are two non-empty sets, then


A − ( A − B) equals
1. If A = { 5, 6, 7} and B = { 7, 8, 9}, then A ∪ B is 8. The number of non-empty proper subsets of set 17. The shaded region in the adjoining diagram is (a) B (b) A − B (c) A ∩ B (d) A ′ ∩ B′
equal to A = { 2, 5, 7, 10} is
(a) 16 (b) 15 (c) 14 (d) 8 25. Let two sets A and B have 2n and 4n elements
(a) {5, 6, 7, 8, 9} (b) {7, 8, 9} (c) {5, 6, 7} (d) φ B respectively, where n is a natural number. What
2. Given that A = { 2, 6, 8, 9}, B = { 7, 8, 9, 12}, then 9. Which of the following statements is false for the can be the minimum number of elements in A ∪ B ?
B − A is equal to sets A, B and C, where (a) 2n (b) 3n (c) 4n (d) 6n
A
(a) {7, 8, 9, 12} (b) {7, 12} A = { x|x is letter of the word ‘BOWL’} C 26. If P = { x : x 2 − 3x + 2 = 0}, Q = { x : x 2 + 4x − 12 = 0},
(c) {2, 6, 7, 8, 9, 12} (d) {2, 6, 8, 9, 12} B = { x|x is a letter of the word ‘ELBOW’} then P − Q is
3. If U is the universal set of all natural numbers C = { x|x is a letter of the word ‘BELLOW’} (a) A ∩ (B − C ) (b) A − (B ∪ C ) (a) {1, 2} (b) {2} (c) {1} (d) {4, 3}
and A = { 1, 2, 3, 4, 5}, then compute A ∩ U . (a) A ⊂ B (b) B ⊃ C (c) A ∩ (B ∪ C ) (d) A ∪ (B ∩ C ) 27. If A = {( 22n − 3n − 1)| n ∈ N } and
(a) {1, 2, 3, 4} (b) φ (c) {1, 2, 3, 4, 5} (d) U (c) B ≠ C (d) B is a proper subset of C 18. In the Venn diagram below, shaded portion B = { 9 ( n − 1)| n ∈ N }, then which one of the
4. The set { 2, 4, 16, 256,... } can be represented as 10. If A and B are two sets, then A ∩ ( A ∪ B) equals represents following is correct?
A B C U (a) A ⊂ B (b) A ⊂ A (c) A = B
which one of the following? (a) A (b) B
n (d) Neither A is a subset of B nor B is a subset of A
(a) {x ∈ N| x = 2 2 , n ∈ N} (c) φ (d) None of these
n
(b) {x ∈ N| x = 2 2 , n = 0, 1, 2,...} 28. If A, B and C are any three sets, then
11. The smallest set B such that (a) A − (B ∪ C ) = ( A − B) ∪ ( A − C )
(c) {x ∈ N| x = 2 4 n , n = 0, 1, 2,...}
B ∪ { 1, 2} = { 1, 2, 3, 5, 9} is (b) A − (B ∪ C ) = ( A − B) − ( A − C )
(d) {x ∈ N| x = 2 2 n , n = 0, 1, 2,...}
(a) {3, 5, 9} (b) {3, 5, 8} (a) A ∪ B ∪ C (b) A ∩ B ∩ C (c) A − (B ∪ C ) = ( A − B) ∩ ( A − C )
5. If P and Q are any two sets and P ⊂ Q , then (c) {1, 2, 3} (d) None of these (c) ( A ∩ B) ∪ (B ∩ C ) (d) A ∪ B ∩ C (d) A − (B ∪ C ) = ( A ∪ B) − ( A ∪ C )
(a) P ∩ Q = φ (b) P ′ ∩ Q = P (c) P ∩ Q = P (d) P ∩ Q = Q 12. If U = { 0, 1, 2, 3, 4, 5, 6, 7, 8, 9}, P = { 0, 1, 2, 3}, 19. The Venn diagram for A ∪ B when B ⊂ A is 29. If P and Q are two sets such that
6. Which one of the following is a true statement? Q = { 2, 3, 4, 5}, and R = { 4, 5, 6}, then Q ′ ∩ ( P ∪ R ) U A B U n ( P ) = m , n (Q ) = n and n ( P ∩ Q ) = p, then
(a) ( A − B) ∩ (B − A) = φ (b) ( A − B) ∩ (B − A) = A is equal to B
A n ( P ∪ Q ) is equal to
(c) ( A − B) ∩ (B − A) = U (d) ( A − B) ∩ (B − A) = B (a) {0, 1, 6} (b) {0, 1, 2, 3, 4, 5} (a) m + n (b) m + n + p (c) m + n − p (d) m − n − p
7. If P and Q are any two sets, then P ∪ Q = P ∩ Q, if (c) {6, 7, 8, 9} (d) {2, 3} 30. A and B are two sets such that n ( A) = 17,
(a) P is the empty set (b) Q is the empty set 13. If P is a non-empty set, then ( P ′ )′ is equal to n( B) = 23, n ( A ∪ B) = 38. Then, n ( A ∩ B) is
(c) Both P and Q are empty sets (a) (b) (a) 40 (b) 78 (c) 2 (d) None of these
(d) P and Q are non-empty sets (a) φ (b) U (c) U − P (d) P
MATHEMATICS Set Theory 189 190 CDS Pathfinder

31. Let A = { x : x 2 − 6x + 8 = 0} and 42. In an examination, 52% candidates failed in 49. State which of the sets given below are infinite set? 56. Consider the following in respect of the sets A
B = { x : 2x 2 + 3x − 2 = 0}. Then, which one of the English and 42% failed in Mathematics. If 17%
I. Set of all concentric circles. and B. e 2013 I
candidates failed in both English and
following is correct? II. { x : x is a multiple of 2, x is an integer.} I. ( A ∩ B) ⊆ A II. ( A ∩ B) ⊆ B III. A ⊆ ( A ∪ B)
(a) A ⊆ B (b) B ⊆ A
Mathematics, what percentage of candidates
passed in both the subjects? III. The set of lines which are parallel to X-axis. Which of the statement(s) given above is/are
(c) Neither A ⊆ B nor B ⊆ A (d) A = B correct?
(a) 18% (b) 21% (c) 23% (d) 25% IV. The set of positive integers greater than 100.
32. If A = { 1, 2, 3, 4}, B = { 2, 3, 4, 5}, C = { 1, 3, 4, 5, 6, 7}, 43. State which of the following statements about Select the correct answer using the codes given
(a) I and II (b) II and III (c) I and III (d) All of these
then A ∩ ( B ∪ C ) is equal to sets is/are true? below 57. In a school there are 30 teachers who teach
(a) {1, 2, 3, 4, 5, 6, 7} (b) {1, 2, 3, 4} I. Every subset of a finite set is finite. (a) I and II (b) II and III Mathematics or Physics. Of these teachers, 20
(c) {1, 2, 3, 4, 5} (d) φ II. φ is a subset of {0}. (c) Only I (d) All of these teach Mathematics and 15 teach Physics, 5 teach
33. Let the set A and B be given by A = { 1, 2, 3, 4} Select the correct answer using the codes given 50. Which of the following sets are equivalent?
both Mathematics and Physics. The number of
and B = { 2, 4, 6, 8, 10} and the universal set below teachers teaching only Mathematics is e 2013 I
I. A = {1, 2, 3, 4, 5}, B = { 7, 8, 9, 10, 11} (a) 5 (b) 10 (c) 15 (d) 20
U = { 1, 2, 3, 4, 5, 6, 7, 8, 9, 10}, then ( A ∪ B)′ is (a) Only I (b) Only II
II. A = { x, y, z } B = { p, q}
(a) {2, 4} (b) U (c) Both I and II (d) Neither I nor II 58. In a class of 110 students, x students take both
III. P = {2, 4, 6, 8}, R = { a , b, c, d } Mathematics and Statistics, x + 20 students take
(c) {1, 3, 5, 6, 7, 8, 9, 10} (d) {5, 7, 9} 44. Which of the following is/are examples of empty IV. A = {36, 39, 42, 45} , B = {42, 39, 45, 36} Mathematics and x + 30 students take Statistics.
34. What is {[( A ∪ B)′ ∩ A]} − ( A − B) equal to? set?
I. A = { x : x + 3 = 3, x ∈ I } Select the correct answer using the codes given There are no students who take neither Mathematics
(a) φ (b) A (c) B (d) B′ nor Statistics. What is x equal to? e 2013 II
II. B = { x : x is a positive even integer and prime} below
35. Which one of the following is a correct statement? (a) I, II and IV (b) II, III and IV (a) 15 (b) 20 (c) 25 (d) 30
III. C = { x : x2 = 16, x is odd integer}
I. { a } ∈ {{ a }, { b}, { c} II. { a } ⊆ {{ a }, b, c} (c) I, III and IV (d) None of these 59. If A is a non-empty subset of a set E, then what
Select the correct answer using the codes given
III. { a , b} ⊆ {{ a }, b, c} IV. a ⊆ {{ a }, b, c} below is E ∪ ( A ∩ φ ) − ( A − φ ) equal to? e 2014 I
(a) Only I (b) Only II (c) III and IV (d) Only IV (a) Only I (b) I and III (c) Only III (d) I, II and III Directions (Q. Nos. 51-52) Answer the questions (a) A (b) Complement of A
36. Which one of the following is an infinite set? based on the following information. (c) φ (d) E
45. Consider the following statements :
(a) {x : x is a whole number less than or equal to 1000} I. A′ ∪ B = ( A ∩ B)′ II. (φ′ )′ = ∪ In a survey of 250 students, it was found that 150 play 60. If A and B are any two non-empty subsets of a
(b) {x : x is a natural number less than 1000} cricket, 100 play basketball and 120 play football, set E, then what is A ∪ ( A ∩ B) equal to? e 2014 I
III. A ∩ (B ∪ C ) = ( A ∩ B) ∪ ( A ∩ C ) further, 30 of them play both basketball and football, 50
(c) {x : x is a positive integer less than or equal to 1000}
Which of the statement(s) given above is/are (a) A ∩ B (b) A ∪ B (c) A (d) B
(d) {x : x is an integer and less than 1000} play both cricket and basketball and 60 play both cricket
correct? and football. 61. Out of 105 students taking an examination
37. Which one of the following is not correct in (a) Only II (b) I and III (c) Only III (d) II and III
respect of the sets A and B ? English and Mathematics, 80 students pass in
(a) If A ⊆ B, then B ∪ A = B 46. Let P = Set of all integral multiples of 3 51. The maximum number of students who play all English, 75 students pass in Mathematics, 10
the three sports students fail in both the subjects. How many
(b) If A ⊆ B, then A ∩ ( A − B) = φ Q = Set of all integral multiples of 4
(a) 20 (b) 10 (c) 15 (d) None of these students pass in only one subject? e 2014 I
(c) If A ⊆ B, then B ∩ A = A R = Set of all integral multiples of 6
52. If 5 students play none of the three sports then (a) 26 (b) 30 (c) 35 (d) 45
(d) If A ∩ B = φ, then either A = φ or B = φ Consider the following relations
↔ ↔ ↔
38. If R ∈ PQ , S ∉ PQ and S does not lie on PQ numbers of students who play at least two sports 62. Let A denotes the set of quadrilaterals having
I. P ∪ Q = R II. P ⊂ R III. R ⊂ (P ∪ Q )
extended, then (a) 100 (b) 110 (c) 120 (d) 130 two diagonals equal and bisecting each other.
↔ ↔ ↔ ↔ Which of the relation(s) given above is/are correct? Let B denotes the set of quadrilaterals having
(a) PQ ∩ RS = φ (b) PQ ∩ RS = {R} (a) Only I (b) Only II (c) Only III (d) II and III
↔ ↔ ↔ ↔ diagonals bisecting each other at 90°. Then,
(c) PQ ∩ RS = {S} (d) PQ ∩ RS = {P}
47. Consider the following statements:
PREVIOUS YEARS’ QUESTIONS A ∩ B denotes e 2015 II
39. In a group of 1000 people, there are 750 people I. Set of all points of a given line is a infinite set. 53. If A = {x : x is an even natural number}, B = {x : x (a) the set of parallelograms (b) the set of rhombuses
who can speak Hindi and 400 who can speak is a natural number and multiple of 5} and C = { (c) the set of squares (d) the set of rectangles
II. The set of all birds in a zoo is a infinite set.
English. How many can speak Hindi only? III. Good books in a school library is a set. x : x is a natural number and multiple of 10}, 63. Let S be a set of first fourteen natural numbers.
(a) 600 (b) 150 (c) 300 (d) 500
Which of the statement(s) given above is/are then what is the value of A ∩ ( B ∪ C )? e 2012 I The possible number of pairs ( a , b), where a,
40. In a committee 50 people speak French, 20 speak correct? (a) {10, 20, 30,…} (b) {5, 10, 15, 20,…} b ∈ S and a ≠ b such that ab leaves remainder 1
Spanish and 10 speak both Spanish and French. (a) Only I (b) Only II (c) Only III (d) All of these (c) {2, 4, 6,…} (d) {20, 40, 60,…} when divided by 15, is e 2016 I
How many speak atleast one of these two languages? 48. Given that the set A = { 0, 1, 2, 3}, which of the (a) 3 (b) 5 (c) 6 (d) None of these
(a) 60 (b) 50 (c) 30 (d) 70
54. Which one of the following is a null set? e 2012 II
following statements about A are true? (a) A = {x is a real number : x > 1 and x < 1} 64. In a gathering of 100 people, 70 of them can speak
41. Every student in a class of 42 students, studies I. A is a finite set. (b) B = {x : x + 3 = 3} Hindi, 60 can speak English and 30 can speak
atleast one of the subjects, Mathematics, English II. A is a subset of the set of integers. (c) C = {φ} French. Further, 30 of them can speak both Hindi
and Commerce, 14 students study Mathematics, III. {1, 2} is a proper subset of A. (d) D = {x is a real number : x ≥ 1 and x ≤ 1} and English, 20 can speak both Hindi and French.
20 Commerce and 24 English. 3 students study IV. A is the null set. If x is the number of people who can speak both
Mathematics and Commerce, 2 English and 55. Let x ∈ {2, 3, 4} and y ∈ {4, 6, 9, 10}. If A be the
Select the correct answer using the codes given English and French, then which one of the following
Commerce and there is no student who studies set of all order pairs (x, y) such that x is a factor
below is correct? (Assume that everyone can speak
all the three subjects. The number of students of y. Then, how many elements does the set A
(a) I, II and III (b) I and IV atleast one of the three languages) e 2016 I
who study Mathematics but not Commerce is contain? e 2012 II
(c) I and III (d) All of these (a) 12 (b) 10 (c) 7 (d) 6 (a) 10 < x ≤ 30 (b) 0 ≤ x < 8 (c) x = 9 (d) x = 8
(a) 4 (b) 3 (c) 12 (d) 11
MATHEMATICS Set Theory 191 192 CDS Pathfinder

↔ ↔
28. (c) ∴ PQ ∩ RS = { R} n ( E ∪ M ) = n( E ) + n( M ) − n ( E ∩ M )
ANSWERS A B

A B

where R is point of intersection of the = 52 + 42 − 17 = 77%
straight lines PQ and RS. ∴ Total candidates passed = ( 100 − 77)
1 a 2 b 3 c 4 b 5 c 6 a 7 c 8 c 9 a 10 a = 23%
(A–B) 39. (a) Here, H = People, who can speak Hindi
11 a 12 a 13 d 14 c 15 c 16 a 17 b 18 c 19 a 20 d
A–(B ∪ C) C ∩(A–C) C E = People, who can speak English 43. (c) Clearly, both I and II are true.
21 c 22 a 23 a 24 c 25 c 26 c 27 a 28 c 29 c 30 c
Given, n ( H ∪ E ) = 1000, n ( H ) = 750, 44. (c) I. A = { 0} II. B = { 2 }
31 c 32 b 33 d 34 a 35 a 36 d 37 d 38 b 39 a 40 a From the above two figures, shaded III. C = { } ; + 4 is not an odd integer
n ( E ) = 400
portion of
41 d 42 c 43 c 44 c 45 c 46 c 47 a 48 a 49 d 50 c n (H ∪ E ) = n (H ) + n (E ) Here, only III is empty set.
A − ( B ∪ C ) = ( A − B) ∩ ( A − C )
51 a 52 b 53 a 54 a 55 d 56 d 57 c 58 b 59 b 60 c − n (H ∩ E ) 45. (c) By distributive law in sets
29. (c) n ( P ∪ Q ) = n ( P ) + n (Q )
61 c 62 c 63 d 64 a − n ( P ∩ Q) 1000 = 750 + 400 − n ( H ∩ E ) A ∩ ( B ∪ C ) = ( A ∩ B) ∪ ( A ∩ C )
∴ n ( P ∪ Q) = m + n − p ⇒ n ( H ∩ E ) = 1150 − 1000 = 150 So, only III is correct.
30. (c) n ( A ∪ B ) = n ( A ) + n ( B ) Number of people who can speak Hindi 46. (c) Here, P = {… − 6, − 3, 0, 3, 6,… },
− n ( A ∩ B) only Q = {… − 8, − 4, 0, 4, 8,… } and
∴ n ( A ∩ B) = n ( A) + n ( B) = n (H ) − n (H ∩ E ) R = {… − 12, − 6, 0, 6, 12,… }
HINTS AND SOLUTIONS − n ( A ∪ B ) = 17 + 23 − 38 = 2
31. (c) Given, A = { x : x2 − 6x + 8 = 0}
= 750 − 150 = 600 I. P ∪ Q = {… − 8, − 6, − 4, − 3, 0,
3 , 4 , 6 , 8 ,… } ≠ R
40. (a) Let S be the set of people who speak
1. (a) A ∪ B = {5, 6, 7} ∪ { 7, 8, 9} 9. (a) Given, A = { B , O , W , L } 21. (c) As, π, 2 , 3 + 7 are irrational ⇒ A = { x :( x – 4)( x − 2)} spanish and F be the set of people who II. P ⊄ R as 3 ∈ P but 3 ∉R.
= {5, 6, 7, 8, 9} B = {B, O, W , L, E } numbers. ∴ A = { 4, 2} speak French III. R ⊂ ( P ∪ Q ) is true.
So, required set is { π , 2 , 3 + 7 }. And B = { x : 2x 2 + 3x − 2 = 0} ∴ n ( S ) = 20, n ( F ) Hence, only statement III is correct.
2. (b) B − A = { 7, 8, 9, 12} − { 2, 6, 8, 9} C = {B, O, W , L, E }
22. (a) Here, U = { 1, 2, 3, 4, 5, ...} ⇒ B = { x:( 2x − 1)( x + 2)} = 50, n ( S ∩ F ) = 10
= { 7, 12} ∴ A ⊂ B and B = C 47. (a) I. There are infinite points lie on a line
B =  , − 2
and A = { 1, 3, 5, 7, 9, 11,… } 1 ⇒ n (S ∪ F ) = n (S) + n (F ) segment so, it is an infinite set.
10. (a) Clearly, A ∩ ( A ∪ B ) = A ∴
3. (c) Given, A = { 1, 2, 3, 4, 5} and A ⊆ U . So, A ′ = U − A = { 2, 4, 6, 8, 10,… } 2  − n (S ∩ F ) II. Number of birds in zoo are
⇒A ∩ U = { 1, 2, 3, 4, 5} ∩ {U } 11. (a) B ∪ { 1, 2} = { 1, 2, 3, 5, 9} ∴ A′ = { x : x is an even number} Hence, neither A ⊆ B nor B ⊆ A. = 20 + 50 − 10 = 60 countable so, it is a finite set.
= { 1, 2, 3, 4, 5} ⇒ B = {3, 5, 9} is the required smallest 32. (b) ( B ∪ C ) = { 1, 2, 3, 4, 5, 6, 7} III. It is not a well-defined set.
23. (a) Disjoint sets have no common 41. (d) Here, M = Students who study
set element. Hence, only I is correct.
4. (b) Let A = { 2, 4, 16, 256,...} ∴ A ∩ ( B ∪ C ) = { 1, 2, 3, 4} Mathematics
0 12. (a) Here, ( P ∪ R) = { 0, 1, 2, 3, 4, 5, 6} e.g. A = { 1, 2, 3} , B = { 4, 5, 6} ∩ { 1, 2, 3, 4, 5, 6, 7} E = Students who study English 48. (a) I. A is a finite set.
for n = 0, 22 = 21 = 2
1 Q ′ = U − Q = { 0, 1, 6, 7, 8, 9} ⇒ A∩B=φ = { 1, 2, 3, 4} C = Students who study Commerce II. As all elements of A are integers, so
for n = 1, 22 = 22 = 4
2 So, Q ′ ∩ ( P ∪ R) = { 0, 1, 6} 24. (c) A − ( A − B ) = A − ( A ∩ B ′) 33. (d) ( A ∪ B ) = { 1, 2, 3, 4, 6, 8, 10} ∴ n ( M ∪ E ∪ C ) = 42 A is subset of integers.
for n = 2, 22 = 24 = 16 = A ∩ ( A ∩ B ′) ′ = A ∩ ( A ′ ∪ B ) III. { 1, 2} is a proper subset of A.
13. (d) Let U = { 1, 2, 3, 4, 5, 6} and ( A ∪ B )′ = U − ( A ∪ B ) Also, n( M ) = 14, n( E ) = 24, n(C ) = 20,
Thus,
P = { 1, 3, 4} = ( A ∩ A ′) ∪ ( A ∩ B ) IV. A ≠ φ.
n
A = { x ∈ N | x = 22 , n = 0, 1, 2,...} = { 1, 2, 3, 4, 5, 6, 7, 8, 9, 10} n ( M ∩ C ) = 3, n ( E ∩ C ) = 2
P ′ = U − P = { 2, 5, 6} = φ ∪ ( A ∩ B) = A ∩ B So, I, II and III are true.
Then, − { 1, 2, 3, 4, 6, 8, 10} and n ( M ∩ E ∩ C ) = 0
5. (c) Q P ⊂ Q and ( P ′)′ = U − P ′ = { 1, 3, 4} = P 25. (c) Q n ( A ∩ B ) = 2n, = {5, 7, 9} ) 49. (d) All sets are infinite set.
14 C(20)
∴ n ( A ∪ B) = n ( A) + n ( B) M(
Q U 14. (c) The shaded region is U − A = A ′. 34. (a) {( A ∪ B )′ ∩ A } − ( A − B )
3 50. (c) Equivalent sets have same cardinal
− n ( A ∩ B) 0 numbers. Here, cardinal numbers of I,
P 15. (c) The shaded region represents the = {(U − ( A ∪ B )) ∩ A } − ( A − B ) 2
elements in A and not in B, so it = 2n + 4n − 2n = 4n III, IV sets are same.
= {(U ∩ A ) − {( A ∪ B ) ∩ A }}
represents A − B. Hence, minimum number of elements E(24)
in A ∪ B is 4n. − ( A − B)
16. (a) We have,
From the Venn diagram, it is clear that
Solutions (Q. No. 51-52) Let C,
(B ∩ C) A ∪ (B ∩ C) 26. (c) Here, P = { x: x2 − 3x + 2 = 0} = { A − A} − ( A − B) = φ − ( A − B) = φ B and F denotes the number of
So, P ∩ Q = P ⇒ P = { x : ( x − 1) ( x − 2) = 0 } 35. (a) {a} is an element of {{ a },{ b },{ c }},
number of students who study
Mathematics but not Commerce students who play cricket,
6. (a) Let x ∈ A − B ⇒ x ∈ A and x ∉B B B
⇒ P = { x : x = 1, x = 2} subsets of {{ a }, b , c } are φ, {{ a }, { b }, { c }, basketball and football
{{ a }, b }, { b , c }, {{ a }, c }, {{ a }, b , c } = 14 − 3 = 11
⇒ x ∉B − A A A ∴ P = { 1, 2} and respectively.
Also, x ∈ B − A ⇒ x ∈ B Hence, only statement I is correct. 42. (c) Let number of failed candidates in
Q = { x : x2 + 4x − 12 = 0} Mathematics and English be M and E, Given, n (C ) = 150, n (B) = 100,
and x ∉ A ⇒ x ∉ A − B C C ⇒ Q = { x : ( x + 6) ( x − 2) = 0} 36. (d) {x : x is an integer and less than respectively. n (F) = 120, n (B ∩ F) = 30,
So, ( A − B ) ∩ ( B − A ) = φ 1000} = [..., 998,999] n (C ∩ B) = 50 and n (C ∩ F) = 60
⇒ Q = { − 6, 2} Given, candidates failed in English,
17. (b) The shaded region represents the i.e. x ∈ ( − ∞ , 1000) is an infinite set. n( E ) = 52%, Let x be the number of students
7. (c) Only if P and Q both are empty set or elements in A and not in B or C . So, P − Q = { 1, 2} − { − 6, 2} = { 1}
P and Q have same elements, then 37. (d) If A ∩ B = φ, then it is not necessary Candidates failed in Mathematics, who play all the three sports
P ∪ Q = P ∩ Q. ∴ A − (B ∪ C ) 27. (a) Given, A = {( 22 n − 3n − 1)| n ∈ N } that either. n( M ) = 42% C B (100)
18. (c) Clearly, ( A ∩ B ) ∪ ( B ∩ C ) = { 0, 9, 54, 243, … } A = φ or B = φ. 0)
8. (c) Number of elements in A = 4 Candidates failed in both English and 5
↔ ↔ (1
19. (a) Clearly, (a) is correct. and B = { 9 ( n − 1) | n ∈ N } 38. (b) Since R ∈ PQ i.e R belongs to PQ. Mathematics, x
∴ Total number of non-empty
20. (d) Since, P( φ) = { φ} = { 0, 9, 18, 27,… } ↔ ↔
n ( E ∩ M ) = 17%
Proper subsets of A = 2n − 2 Again S , PQ and S does not lie on PQ
Hence, φ ∈ P( φ) From above, it is clear that A ⊂ B. Q Total candidates, failed F (120)
= 24 − 2 = 14 extended
20
MATHEMATICS Set Theory 193 194 CDS Pathfinder

51. (a) n (C ∪ B ∪ F ) = n (C ) + n ( B ) ∴A ∪ ( A ∩ B ) = A ∪ (Shaded portion)


U
+ n ( F ) − n (C ∩ F ) = A
− n (C ∩ B ) − n ( B ∩ F ) + n (C ∩ B ∩ F ) A B If A and B are two non-empty disjoint
⇒ n (C ∪ B ∪ F ) = 150 + 100
sets.
+ 120 − 60 − 50 − 30 + x
then, A ∩B = φ
⇒ n (C ∪ B ∪ F ) = 230 + x
Since, maximum value of n (C ∪ B ∪ F ) (A ∩ B) ∴ A ∪ ( A ∩ B) = A ∪ φ = A
is 250 I. ( A ∩ B ) ⊆ A [true] 61. (c) Let E and M represent the students
Hence, maximum value of II. ( A ∩ B ) ⊆ B [true] pass in English and Mathematics.
x = 250 − 230 = 20
52. (b) Here, n(C ∪ B ∪ F ) = 250 − 5 = 245
∴ 245 = 230 + x ⇒ x = 15
III. A ⊆ ( A ∪ B ) [true]
So, all three statements are correct.
Shaded region = ( A ∪ B )
57. (c) Let M and P be the teachers who
Given, n (U ) = 105, n( E ) = 80,
n( M ) = 75, and n( E ∩ M ) = 10
Now, n( E ∪ M ) = n(U ) − n( E ∩ M )
MEASUREMENTS OF
(1
50
)C
35
15
45 15
B (100) teach Mathematics and Physics,
respectively.
Given,
= 105 − 10 = 95
Also, n( E ∪ M ) = n( E ) + n( M )
− n( E ∩ M )
ANGLES AND
n ( M ∪ P ) = 30, n ( M ) = 20, n ( P ) = 15 ⇒ 95 = 80 + 75 − n( E ∩ M )
F (120)

Number of students who play atleast


two sports = 15 + 35 + 45 + 15 = 110
and n ( M ∩ P ) = 5
∴ Number of teachers teaching only
Mathematics is
⇒ n( E ∩ M ) = 60
The number of students who pass in
only one subject
TRIGONOMETRIC RATIOS
n ( only M ) = n ( M ) − n ( M ∩ P ) = n( E ) + n( M ) − 2 n ( E ∩ M )
53. (a) Given, A = { 2, 4, 6, 8, 10, 12, ...},
= 20 − 5 = 15 = 80 + 75 − 2 × 60
B = {5, 10, 15, 20, 25, 30, ...}
and C = { 10, 20, 30, 40, 50, 60, ...} 58. (b) Venn diagram of given conditions is = 155 − 120 = 35 Generally (13-14) questions have been asked from this chapter. Generally questions asked from this
as shown in figure given below chapter are based on the trigonometric identities and formulae. So, detailed study of this chapter
Now, 62. (c) A = diagonal equal and bisecting each
Students take Students take other.
( B ∪ C ) = {5, 10, 15, 20, 25, 30, ...} only Mathematics only Statistics will help you to score in exam as good number of questions have been asked from this chapter.
∴ A ∩ ( B ∪ C ) = { 2, 4, 6, 8, 10, 12, ...} A is square or rectangle. and B diagonal
bisecting each other at 90°.
∩ {5, 10, 15, 20, 25, 30, ...} So, A ∩ B = the set of squares.
x + 20 x x + 30
= { 10, 20, 30, ...} 63. (d) The possible set of pairs (a, b) such
54. (a) From option (a), A = {x is a real that ab leaves remainder 1 when divided Trigonometry is the branch of Mathematics which deals with the measurements of sides and angles of
number : x > 1 and x < 1 }. Students take by 15 are (2, 8), (8, 2) (7, 13)
both Mathematics and Statistics triangles and the problems based on these angles.
Since, there is no such element which is and (13, 7).
greater than 1 and less than 1. Total students = Students those take ∴ Number of possible set of pairs = 4
So, A is a null set. only Mathematics + Student those take
64. (a) Let A , B and C be the number of
ANGLE
From option (b), B = {x : x + 3 = 3} = {0} only Statistics + Student those take An angle is considered as the figure obtained by rotating a given ray about its
people who can speak Hindi, English
= Singleton set both Mathematics and Statistics endpoint. The original ray is called the initial side and the ray into which the
and French.
From option (c),C = {φ} = Singleton set Q 110 = x + 20 + x + x + 30
Then, n ( A ) = 70, n ( B ) = 60, initial side rotates is called terminal side. In figure, OA is initial side and OB is the
From option (d), D = {x is a real number ⇒ 110 = 3x + 50 ⇒ 3x = 60 terminal side of ∠AOB. The point O is called its vertex.
: x ≥ 1 and x ≤ 1} n(C ) = 30, n ( A ∩ B ) = 30,
∴ x = 20
= {1} = Singleton set n ( A ∩ C ) = 20, and n ( B ∩ C ) = x

55. (d) Given that, x ∈ {2, 3, 4}


59. (b) E ∪ {( A ∩ φ) − ( A − φ)}
= E ∪ φ − A = E − A = A′
n ( A ∪ B ∪ C ) = n ( A ) + n ( B ) + n (C ) QUADRANTS
and y ∈ {4, 6, 9, 10} −n ( A ∩ B ) − n ( B ∩ C ) Let XOX ′ and YOY ′ be two lines at right angles in the plane of paper. These
[Q ( A ∩ φ) = φ and ( A − φ) = A] −n ( A ∩ C ) + n ( A ∩ B ∩ C ) two perpendicular lines divide the plane of the paper into four equal parts, these
and also A =( x, y ), such that x is a factor
of y. 60. (c) Since, A and B are non-empty subsets ⇒ 100 = 70 + 60 + 30 − 30 − 20 four parts are known as four quadrants and lines XOX ′ and YOY ′ are known as
of E. X-axis and Y-axis, respectively.
∴ A = {(2, 4), (2, 6), (2, 10), (3, 6), (3, 9), −x + n ( A ∩ B ∩ C )
(4, 4)} A B E ⇒ x = 10 + n ( A ∩ B ∩ C ) The parts XOY , YOX ′, X ′ OY ′ and Y ′ OX are known as 1st, 2nd, 3rd and 4th
Hence, A contain 6 elements. Also, n ( A ∩ B ∩ C ) can’t be more than quadrant, respectively.
56. (d) From figure, n ( A ∩C )
⇒ 10 ≤ x ≤ 30 Relation between Angles, Radius and Arc Length
If l ’ is the length of a circle of radius r. And this arc subtends an angle θ radians
l
at the centre of the circle, then l = r θ ⇒ θ =
r
195 196 CDS Pathfinder

Relation between Degree and Radian Sol. a. I. We know that, Relation Between T-ratios TRIGONOMETRIC IDENTITIES
arc
Radius = (given, arc length is constant) 1 1 1
A circle subtends an angle, at the centre whose radian angle (i) sin θ = (ii) cos θ = (iii) tan θ = A statement of equality involving trigonometric ratios of
measure is 2π and its degree measure is 360 °. It follows 1 cosec θ sec θ cot θ
∴ Radius ∝ an angle is called a trigonometric identity. It is valid for
that angle sin θ cos θ all values of the angles. The three important identities
2π radian = 360 ° (iv) tan θ = (v) cot θ =
So, angular measure in radian decreases, if the cos θ sin θ are
π radian = 180 ° radius of the arc increases.
180 π 8 1. sin 2 θ + cos 2 θ = 1 2. 1 + tan 2 θ = sec 2 θ
∴ 1 radian = degree = 57 ° 16′ 22′ ′ (approx) II. 1800°× = 10π EXAMPLE 5. If sin θ = , then the other
π 180° 17 3. 1 + cot 2 θ = cosec 2 θ
π Hence, only statement I is correct. trigonometric ratios cos θ, tan θ, cosec θ, sec θ, cot θ are
1 degree = radian = 0 . 01746 rad (approx)
180 15 8 17 17 15 15 8 17 −17 8 Note sin2 A + cos 2 B cannot be equal to 1 because the angles are
EXAMPLE 4. The angle between the hour hand and a. , , , , b. − , , , , different.
Note 1 radian is written as 1c . the minute hand of a clock at half-past three is 17 15 8 15 8 17 15 8 15 15
17 15 8 15 8 tan A − sin A
a. 54° b. 63° c. 75° d. 85° c. , , , , d. None of these
EXAMPLE 1. The radian measure corresponding to 15 8 17 17 15 EXAMPLE 7. What is the value of ?
Sol. c. Angle traced by hour hand in 1 hr = 30° sin 3 A e 2013 I
− 22° 30′ is 8 perpendicular
7 ° Sol. a. As, sin θ = = sec A sec A
Angle traced by hour hand in 3 hr =  30 ×  = 105°
1 17 hypotenuse a. b.
πc πc πc πc  1 + cos2 A
2
1− cos A
a. b. − c. d. − 2
8 8 4 4 In right angled ∆PQR, sec A
Angle traced by minute hand in 1 min = 6° c. d. None of these
∴ PQ2 + QR 2 = PR 2 ⇒ PQ2 = PR 2 − QR 2 1+ cos A
Sol. b. As, 180° = π Angle traced by minute hand in 30 min = 6 × 30° = 180°
sin A
45 ° π c πc Angle between two hands = 180°−105° = 75° ⇒ PQ = PR 2 − QR 2 = 172 – 82 − sin A
∴−22°30 ′ = −  22 +  = −   = − 
1 45
×  =− tan A − sin A cos A (1− cos A) (1+ cos A)
 Sol. c. = = ×
2  2  2 180 8 = 289 − 64 = 225 = 15 sin3 A sin3 A cos A sin2 A (1+ cos A)
TRIGONOMETRIC RATIOS PQ 15 RQ 8 =
1− cos A2
=
2
sin A
IMPORTANT POINTS ∴ cos θ = = , tan θ = = cos A sin2 A (1+ cos A) cos A sin2 A (1+ cos A)
The ratio of the sides of a right PR 17 PQ 15
The angle between two consecutive digits in a clock is angled triangle with respect to its 1 1 sec A
PR 17 PR 17 = ⋅ =
l

π cosec θ = = , sec θ = = cos A 1+ cos A 1+ cos A


30 ° =  radian  . angle are called trigonometric ratios. RQ 8 PQ 15
6 
The side opposite to the right angle is PQ 15 Values of Trigonometric Ratios for Some Specific Angles
l The hour hand rotates through an angle of 30° in one called the hypotenuse. Relative to the and cot θ = =
1 °
RQ 8
hour i.e.   in one minute.
Angles 0° 30° 45° 60° 90°
angle θ, the side opposite to angle θ is
 2 1 1 3
caused the prependicular side and the remaining side is 1 sin θ 0 1
The minute hand rotates through an angle of 6° in one called base. EXAMPLE 6. If cot A = , then the value of 2 2 2
( 2 − 1)
l

minute. 1 1
(i) For angle θ, perpendicular = BC, base = AB sin A cos A is cos θ 1
3
0
2 2 2
EXAMPLE 2. If an arc 24π cm long of a circle and hypotenuse = AC 1 1 1 2
a. b. c. d. 1
subtends an angle of 72° at its centre, then the (ii) For angle α, perpendicular = AB, base = BC 4 2 2 4 tan θ 0 1 3 ∞
3
radius of the circle is
and hypotenuse = AC Sol. d. Q cot A =
1
=
base 1
a. 30 cm b. 40 cm c. 50 cm d. 60 cm cot θ ∞ 3 1 0
Trigonometric ratios of θ in right angled ∆ABC are 2 − 1 perpendicular 3
Sol. d. Given, length of arc ( l ) = ( 24π) cm defined below 2 −1  1  2
⇒ tan A = Q tan A =  sec θ 1 2 2 ∞
π  c  2π  c Perpendicular BC P  cot A 3
Angle (θ) = 72° = 72 ×
1
 =  sin θ = = =
 180  5 AC H 2
Hypotenuse In right angled ∆ABC, cosec θ ∞ 2 2 1
3
Let r be the radius of circle Base AB B ∴ AC 2 = AB 2 + BC 2
l l 24π cos θ = = =
Since, θ = ⇒ r= = = 60 cm Hypotenuse AC H
r θ 2π ⇒ AC = AB + BC 2 2
EXAMPLE 8. What is the value of sin 60 °cot 30 ° 3

5
tan θ =
Perpendicular
BC P
== = 12 + ( 2 − 1) 2 = 1 + 2 + 1 − 2 2 = 4 − 2 2 −2sec 2 45°+3cos 60 ° tan 2 45°− tan 2 60 ° ?
EXAMPLE 3. Consider the following statements: Base AB B 35 –35 −11 11
BC 2 −1 AB 1 a. b. c. d.
I. The angular measure in radian of a circular arc of Hypotenuse AC H ∴ sin A = = and cos A = = 8 8 8 8
sec θ = = = AC 4 −2 2 AC 4 −2 2
fixed length subtending at its centre decreases, if Base AB B Sol. b. sin360° cot 30°−2sec 2 45°+3cos 60° tan2 45°− tan2 60°
the radius of the arc increases. Base AB B 2 −1 1 2 −1
cot θ = = = ∴ sin A cos A = ⋅ =  3
3
1
II. 1800° is equal to 5π radian. Perpendicular BC P 4 −2 2 4−2 2 4−2 2 =   × 3 − 2 × ( 2) 2 + 3 × × (1) 2 − ( 3) 2
Which of the statements given above is/are correct? e 2012 I  2 2
Hypotenuse AC H 2 −1 1 2 2 9 3 9 − 32 + 12 − 24 21− 56 −35
a. Only I b. Only II cosec θ = = = = = × = = −4+ −3= = =
c. Both I and II d. Neither I nor II Perpendicular BC P 2 2 ( 2 − 1) 2 2 2 4 8 2 8 8 8
197 198 CDS Pathfinder

π EXAMPLE 11. What is the value of ( A − B) ( A + B)


Sign of trigonometric ratio in different Sol. a. Given, 0 < θ < , then 1− 2sin θ cos θ • sin A − sin B = 2 sin ⋅ cos
4 sec(90 °−θ)sinθ sec 45°? 2 2
quadrants ( A + B) ( A − B)
= sin θ + cos θ − 2sin θ cos θ
2 2
a. 1 b.
3
c. 2 d. 3 cos A + cos B = 2 cos ⋅ cos
Y •
II Quadrant [Q sin2 θ + cos2 θ = 1] 2 2 2
(90°<θ<180°) sinθ I Quadrant (0<θ<90°) Sol. c. Given, sec (90°− θ) sin θ sec 45° = cosec θ × sin θ × 2 ( A + B) ( A − B)
= (cos θ − sin θ) 2 cos A − cos B = − 2 sin ⋅ sin
and cosec θ are positive. All trigonometric •
All other trigonometric ratios are positive. Q 0 < θ < π ,cos θ > sin θ , so we take (cosθ − sinθ) 2  =
1
× sin θ × 2 = 2
2 2
 4  sin θ ( A + B) (B − A )
ratios are negative. = 2 sin ⋅ sin
X´ X = cos θ − sin θ 2 2
IV Quadrant sin 135° − cos 120 °
III Quadrant
(270°<θ < 360°) cos θ Domain and Range of Trigonometrical Functions EXAMPLE 12. The value of is sin ( A + B)
(180°<θ<270°) tan θ sin 135° + cos 120 ° • tan A + tan B =
and sec θ are positive. cos A cos B
and cot θ are positive. Trigonometric Domain Range
All other trigonometric
All other trigonometric Ratio a. 2 + 3 2 b. 2 − 3 2 c. 3 − 2 2 d. 3 + 2 2 sin ( A − B)
ratios are negative.
ratios are negative.
sin θ R [− 1, 1 ]
• tan A − tan B =

Sol. d. sin 135° = sin (180° − 45° ) = sin 45° =
1 cos A cos B
cos θ R [− 1, 1 ]
π
2 sin ( A + B)
π tan θ R ~ (2 n + 1) , n ∈ I  ( − ∞, ∞ ) = R 1 • cot A + cot B =
EXAMPLE 9. If 0 < θ < , then what is 1 − 2sinθ cos θ  2  cos 120° = cos (180° − 60° ) = − cos 60° = − sin A sin B
4 2
cot θ R ~ { n π, n ∈ I} ( − ∞, ∞ ) = R sin ( B − A )
equal to? e 2014 II
π
1  1
− − 
• cot A − cot B =
sec θ R ~ (2 n + 1) , n ∈ I  ( − ∞, − 1 ] ∪ [1, ∞ ) sin 135° − cos 120° 2  2 = 2+1 2+1 sin A sin B
a. cosθ − sinθ b. sinθ − cosθ  2  ∴ = ×
c. ± (cosθ − sinθ) d. cosθ sinθ sin 135° + cos 120° 1
+  − 
1 2 −1 2 +1
cosec θ R ~ { n π, n ∈ I} ( − ∞, − 1 ] ∪ [1, ∞ )
2  2
π
EXAMPLE 13. If A − B = , then the value of
( 2 + 1) 2 3
The behaviour of the trigonometric function in different quadrants are defined in following table. = = 2 + 1+ 2 2 = 3 + 2 2 cos A cos B + sin A sin B is
2−1
Trigonometric Ist Quadrant IInd Quadrant III Quadrant IVth Quadrant a. 1/2 b. 1
Ratio (θ increase from 0 to π /2) (θ increase from π /2 to π) (θ increase from π to 3 π /2) 3π c. 3/2 d. None of these
(θ increase from
2
to 2 π) Sum, Difference and Product Formulae
Sol. a. cos A cos B + sin A sin B = cos ( A − B)
sin θ Increases from 0 to 1 Decreases from 1 to 0 Decreases from 0 to –1 Increases from –1 to 0 • sin ( A + B) = sin A cos B + cos A sin B
π π
= cos   = Q A − B = 
1
cos θ Decreases from 1 to 0 Decreases from 0 to –1 Increases from –1 to 0 Increases from 0 to 1 • sin ( A − B) = sin A cos B − cos A sin B
 3 2  3
tan θ Increases from 0 to ∞ Increases from – ∞ to 0 Increases from 0 to ∞ Increases from – ∞ to 0 • cos ( A + B) = cos A cos B − sin A sin B
cot θ Decreases from ∞ to 0 Decreases from 0 to – ∞ Decreases from ∞ to 0 Decreases from 0 to – ∞
• cos ( A − B) = cos A cos B + sin A sin B 3 cos 23° –sin23°
sec θ Increases from 1 to ∞ Increases from – ∞ to –1 Decreases from –1 to – ∞ Decreases from ∞ to 1 EXAMPLE 14. The value of is
tan A + tan B 2
cosec θ Decreases from ∞ to 1 Increases from 1 to ∞ Increases from – ∞ to –1 Decreases from –1 to – ∞ • tan ( A + B) =
1 − tan A tan B a. cos 7° b. sin 53° c. cos 53° d. sin 7°

EXAMPLE 10. Which one of the following statements TRIGONOMETRIC RATIOS tan A − tan B 3 cos 23° − sin 23°
• tan ( A − B) = Sol. c. =
3 1
cos 23°– sin 23°
is true in respect of the expression sin 31°+ sin 32° ? 1 + tan A tan B
a. Its value is 0 b. Its value is 1
OF ALLIED ANGLES cot A cot B − 1
2 2 2
= cos 30° cos 23°– sin 30° sin 23°
c. Its value is less than 1 d. Its value is greater than 1 Two angles are said to be allied when their sum or • cot ( A + B) =
cot A + cot B = cos ( 30° + 23° ) = cos 53°
difference is either zero or a multiple of 90 °.
Sol. d. We know that, sin 30° =
1
cot A cot B + 1 [Q cos ( A + B) = cos A cos B – sin A sin B ]
2 The angles like − θ, 90 ° ± θ, 180 ° ± θ, 360 ° ± θ etc., are • cot ( A − B) =
angles allied to the angle θ, if θ is measured in degrees. cot B − cot A tan 47 ° + tan 43°
Value of sin increases 0° to 90°
EXAMPLE 15. The value of is
∴ sin 31° > sin 30° and sin 32° > sin 30° Angle sin θ cos θ tan θ cot θ sec θ cosec θ • sin ( A + B) sin ( A − B) = sin 2 A − sin 2 B 1 − tan 47 ° tan 43°
1 1
⇒ sin 31° > and sin 30° > −θ − sinθ cosθ − tanθ − cot θ sec θ − cosec θ = cos 2 B − cos 2 A a. 1 b. ∞
2 2
Adding both 90°− θ cosθ sinθ cot θ tanθ cosec θ sec θ • cos ( A + B) cos ( A − B) c. 0 d. −1
∴ sin 31°+ sin 31° ⇒ +
1 1 90°+ θ cosθ − sinθ − cot θ − tanθ − cosec θ sec θ = cos 2 A − sin 2 B = cos 2 B − sin 2 A tan A + tan B tan 47° + tan 43°
2 2
180°− θ − cosθ − tanθ − cot θ − sec θ cosec θ
Sol. b. Q = tan ( A + B) ⇒
⇒ sin 31°+ sin 32° > 1
sinθ • 2 sin A cos B = sin ( A + B) + sin ( A − B) 1 − tan A tan B 1 − tan 47° tan 43°
180°+ θ − sinθ − cosθ tanθ cot θ − sec θ − cosec θ 2 cos A sin B = sin ( A + B) − sin ( A − B)
• = tan ( 47° + 43° ) = tan 90° = ∞
IMPORTANT POINTS 270°− θ − cosθ − sinθ cot θ tanθ − cosec θ − sec θ
2 cos A cos B = cos ( A + B) + cos ( A − B)
EXAMPLE 16. The value of sin 20 °sin 40 °sin 60 °sin 80 °

| sin θ | ≤ 1 | cos θ | ≤ 1 sec θ ≥ 1 270°+ θ − cosθ sinθ − cot θ − tanθ cosec θ − sec θ
l l l
• 2 sin A sin B = cos ( A − B) − cos ( A + B) is
sec θ ≤ −1 cosec θ ≥ 1 cosec θ ≤ −1 360°− θ − sinθ cosθ − tanθ − cot θ sec θ − cosec θ
l l l
( A + B) ( A − B) 1 3 1 3
tan θ and cot θ can take any value. 360°+ θ sinθ cosθ tanθ cot θ sec θ cosec θ • sin A + sin B = 2 sin ⋅ cos a. b. c. d.
l
2 2 2 4 8 16
199 200 CDS Pathfinder

Sol. d. sin 20° sin 40° sin 80° sin 60° EXAMPLE 18. The value of 3 sin 15° − 4 sin 3 15° is EXAMPLE 21. The value of φ for maximum value of 3. Cosine rule In a ∆ABC, if a, b, c be the sides
= sin 60°[sin 80° sin 40° ]sin 20° 1 1 sin 3φ + cos 3φ is opposite to angles A, B and C respectively, then
a. 1 b. c. d. 0
=
3
[sin 80° sin 40° ]sin 20° 2 2 a. 2 b. 90° c. 1 d. 15° b2 + c 2 − a 2
2
cos A = , cos B = c 2 + a 2 − b 2 and
Sol. d. sin 3φ + cos 3φ = 2  
1 1 2bc
Sol. c. As, 3 sin θ − 4 sin θ = sin 3 θ
3
cos 3φ + sin 3φ
3 1  2 2 
= × [ 2sin 80° sin 40° ]sin 20° 1 a 2 + b2 − c 2
2 2 So, 3 sin 15° − 4 sin3 15° = sin 3(15° ) = sin 45° = = 2 (sin 45° cos 3φ + cos 45° sin 3φ) cos C =
2 2ab
3 = 2 sin ( 45° + 3φ)
= [cos(80°−40° ) − cos(80° + 40° )]sin 20°
4 Trigonometric Ratios of Submultiple Angles ∴ The maximum value occurs when EXAMPLE 22. A triangle ABC is inscribed in a circle.
3 θ θ sin ( 45° + 3φ) = 1 = sin 90° If sum of the squares of sides of the triangle is equal
= [cos 40°− cos 120° ]sin 20° • sin θ = 2 sin cos
4 2 2 ∴ 45° + 3φ = 90° ⇒ φ = 15° to twice the square of the diameter, then
θ θ θ θ sin 2 A + sin 2 B + sin 2 C is equal to
cos 40°−  −   sin 20°
3 1
= cos θ = cos 2 − sin 2 = 2 cos 2 − 1 = 1 − 2 sin 2
4   2  •
Properties of Triangle a. 2 b. 3 c. 4 d. None of these
2 2 2 2
Q cos 120° = − 1 1 − cos θ θ 1 + cos θ θ 1. Circumcircle of a triangle The A Sol. a. Let the radius of inscribed circle be R.
 = tan = cot
2
• •
sin θ 2 sin θ 2 circle passing through the vertices c ∴ R=
a
=
b
=
c
1 − cos θ θ 1 + cos θ of a ∆ABC is called circumcircle.
2 θ
3 1 = tan 2 2sin A 2sin B 2sin C
= cos 40°+  sin 20° •
= cot b
4  2 1 + cos θ 2 •
1 − cos θ 2
Its radius R is called the B
R a b c
⇒ sin A = , sin B = and sin C =
circumradius and its centre is a 2R 2R 2R
3 θ θ
= [sin( 40°+20° ) − sin( 40°−20° ) + sin 20° ] known as circumcentre. C 2
+ 2
+ 2
8 2 tan 1 − tan 2
sin2 A + sin2 B + sin2 C =
a b c
sin θ = 2 cos θ = 2 a b c 4 R2
3 •
θ

θ Here, R = = =
= [sin 60°− sin 20°+ sin 20° ] 1 + tan 2 1 + tan 2 2sin A 2sin B 2sin C
8 a2 b2 c2
2 2 ⇒ sin2 A = , sin2 B = and sin2 C =
3 3 3 3
2. Sine rule In a ∆ABC, if a, b, c be A 4R 2 4R 2 4R 2
= × sin 60° = × = Maximum and Minimum Values the three sides opposite to the
8 8 2 16 According to the question,
of Trigonometrical Functions angles A, B, C respectively, then
cos 4x + cos 3x + cos 2x b c a2 + b2 + c2 = 2 × 2R 2 = 8R 2
EXAMPLE 17. is equal to a b c
sin 4x + sin 3x + sin 2x We know that, − 1 ≤ sin θ ≤ 1, − 1 ≤ cos θ ≤ 1 = = 8R 2
sin A sin B sin C ∴ sin2 A + sin2 B + sin2 C = =2
Maximum and minimum value of a trigonometrical 4R 2
a. cot 3x b. tan 3x c. cot x d. cot 2x C a B

cos 4x + cos 3x + cos 2x (cos 4x + cos 2x) + cos 3x


function of the form a sin θ ± b cos θ are a 2 + b2
Sol. a. =
sin 4x + sin 3x + sin 2x (sin 4x + sin 2x) + sin 3x and − a 2 + b 2 , respectively.

=
 4x + 2x
2cos
 2 
 4x + 2x
2sin
 4x − 2x
 cos
 2 
 + cos 3x

 4x − 2x
 cos  + sin 3x
EXAMPLE 19. The maximum value and minimum
value of 8 sin θ cos θ + 4 cos 2θ is
PRACTICE EXERCISE
 2   2  a. 4 and −4 b. 16 and −16
2cos 3x ⋅ cos x + cos 3x cos 3x ( 2cos x + 1) c. 4 2 and −4 2 d. None of these 1. A unit radian is approximately equal to 5. The angle between the minute hand and the
= = = cot 3x (a) 57° 17′ 43′′ (b) 57° 16′ 22′′ hour hand of a clock when the time is 7:35 pm is
2sin 3x ⋅ cos x + sin 3x sin 3x ( 2cos x + 1) Sol. c. Here, 8 sin θ cos θ + 4 cos 2θ = 4 sin 2θ + 4 cos 2θ
(c) 57° 17′ 47′′ (d) 57° 17′ 49′′ (a) 16° 45′ (b) 17 ° 30′ (c) 18°15′ (d) 19° 30′
Trigonometric Ratios of Multiple Angles So, maximum value = 4 + 4 = 4 22 2
2. Find the radian measure corresponding to the 6. When do the hands of a clock coincide between
2 tan θ degree − 47° 30′. 5 pm and 6 pm?
• sin 2θ = 2 sin θ cos θ = and minimum value = − 42 + 42 = − 4 2 − 19 π 17 π
1 + tan 2 θ (a) rad (b) rad (a) 5:30 pm (b) 5:27:16 pm
72 72 (c) 5:32:16 pm (d) 5:28:56 pm
• cos 2θ = cos 2 θ − sin 2 θ = 2 cos 2 θ − 1 EXAMPLE 20. Solve cos θ + sin θ = 2, then the value 13 π −15 π
(c) rad (d) rad 1
of θ is 7. If sin A = , then cos A ⋅ cosec A + tan A ⋅ sec A is
1 − tan 2 θ 72 72
3
= 1 − 2 sin 2 θ = a. π/3 b. π/4 c. π/6 d. π/2
1 + tan 2 θ 3. The length of a pendulum is 60 cm. The angle equal to
cos θ sin θ through which it swings when its tip describes 16 2 + 3 4 2 + 3
1 + cos 2θ 1 − cos 2θ Sol. b. cos θ + sin θ = 2 , + =1 (a) (b)
• = cos θ and
2
= sin 2 θ 2 2 an arc of length 16.5 cm is 8 8
2 2 (Q dividing throughout by 2) (a) 15° 30′ (b) 15° 45′ (c) 16°15′ (d) 16° 45′ 3+2 3 −1
2 tan θ (c) (d)
• tan 2θ = • sin 3 θ = 3 sin θ − 4 sin 3 θ ⇒ cos θ cos 45° + sin θ sin 45° = 1 4. A railway train is moving on a circular curve of 4 8
1 − tan 2 θ cos (θ − 45° ) = 1 = cos 0° radius 1500 m at a speed of 90 km/h. Through 8. The value of cos 15° − sin 15° is equal to
cos 3θ = 4 cos 3 θ − 3 cos θ • tan 3θ = 3 tan θ − tan θ
3
• ∴ θ − 45° = 0 ⇒ θ = 45° = π /4 what angle has it turned in 11 seconds? 1 1 1 3
1 − 3 tan 2 θ (a) 12° (b) 16° 30′ (c) 10° 30′ (d) 11° 40′ (a) (b) (c) (d)
3 2 2 2
MATHEMATICS Measurements of Angles and Trigonometric Ratios 201 202 CDS Pathfinder

1 1
1 − sin θ 22. If cosθ = 0.96, then + is equal to 34. What is cot 15° cot 20° cot 70° cot 75° equal to? 45. If tan A =
5 1
and tan B = , then A + B is equal to
9. If 3 tanθ = 4, then is equal to sin θ tan θ
1 + sin θ (a) − 1 (b) 0 (c) 1 (d) 2 6 11
(a) 0.98 (b) 3 (c) 4 (d) 7 π π 3π
1 2 1 sin θ cos θ (a) (b) (c) (d) π
(a) (b) (c) (d) None of these
23. If 0 < x < 45° and 45° < y < 90°, then which one of 35. If + = 2 with 0 < θ < 90°, then what is θ 2 4 2
2 3 3 cos θ sin θ
the following is correct? 1
10. If tan A = 1 and tan B = 3, then equal to? 46. The value of (cos A − sin A) is
(a) sin x = sin y (b) sin x < sin y (a) 30° (b) 45° (c) 60° (d) 75° 2
cos A ⋅ cos B − sin A ⋅ sin B is equal to (c) sin x > sin y (d) sin x ≤ sin y π π
(a) cos  + A (b) cos  + A
1+ 3 1− 3 2 2 36. In a right ∆ ABC, right angled at B, the ratio of 3  2 
(a) (b) (c) (d) 1 24. For what value of θ is (sin θ + cosec θ) = 2.5, where 2 tan A π π
2 2 2 2 3
0 < θ ≤ 90° ? AB to AC is 1 : 2, then is equal to (c) cos  + A (d) sin  + A
1 − tan2 A 4  4 
11. The value of cosec2 θ − 2 + sin2 θ is always (a) 30° (b) 45° (c) 60° (d) 90°
cos x sin x (a) 2 (b) 1 (c) 3 (d) undefined π π π π
(a) less than zero (b) non-negative 25. If = n and = m, then ( m 2 − n 2 ) sin2 y 47. sin cos − cos sin is equal to
(c) zero (d) 1 cos y sin y 37. In figure, AD = DB, ∠B = 90°, then cosθ is equal to 4 12 4 12
1 3 1
is equal to A (a) (b) 3 (c) (d)
12. What is the value of 2 2 2
(a) 1 − n2 (b) 1 + n2 (c) m2 (d) n2
sin 15° + sin 20° + sin 25°+L + sin 75°?
2 2 2 2
x sin φ y sin θ x
26. If p = tan2 x + cot2 x, then which one of the 48. If tan θ = and tan φ = , then
(a) 0 (b)
13
(c) 6 (d)
11 b 1 − x cos φ 1 − y cos θ y
following is correct? Da
2 2 is equal to
(a) p ≤ 2 (b) p ≥ 2 (c) p < 2 (d) p > 2
13. What is the value of sin φ sin φ sin θ sin θ
(a) (b) (c) (d)
5 sin 75° sin 77° + 2 cos 13° cos 15° 7 sin 81° 27. The difference of the two angles in degree θ B sin θ cos θ sin φ 1 − cos φ
− ? C
measure is 1 and their sum in circular measure
cos 15° sin 77° cos 9° is also 1. What are the angles in circular measure? 49. If sin (θ + φ) = 2 sin (θ − φ), then
2 b 2 − a2 a
(a) − 1 (b) 0 (c) 1 (d) 2 (a) (b) (a) cot φ = 3 tan θ (b) tan θ = 3 tan φ
π  1 π 
(a)  − (b)  −
1 1 90   1 90  4b 2 − 3 a2 4b 2 − 3 a2
,  +  ,  +  (c) sin θ = 3 sin φ (d) sin φ = sin 2 θ
14. What is log (tan 1° ) + log (tan 2° ) + log (tan 3° )  2 360   2 360  2 π  2 π
+ L + log (tan 89° ) equal to? 4b − 3 a
2 2
π  1 π  (c) (d) None of these 50. If sin (θ + α ) = cos (θ + α ), then tan θ is equal to
(c)  −
1
,  +  (d) None of these 2
(a) 0 (b) 1 (c) 2 (d) − 1  2 180   2 180  1 − tan α tan α + 1 1 − cot α sin2(θ + α )
(a) (b) (c) (d)
1 38. If 0° ≤ x ≤ 90° and sin x + 3 cos x = 1, then what 1 + tan α tan α − 1 1 + cot α cos (θ + α )
15. If tan θ + = 2, then the value of 28. What is the value of [(1 − sin2 θ) sec2 θ + tan2 θ]
tan θ is the value of x? θ−φ θ+φ
(cos2 θ + 1) when 0 < θ < 90° ? 51. cos2 − sin2 is equal to
1 (a) 30° (b) 45° (c) 60° (d) 90° 2 2
tan θ +
2
is equal to (a) 2 (b) > 2 (c) ≥ 2 (d) < 2
tan2 θ 39. If tan A = 2 − 1, then the value of cosec A ⋅ sec A (a) cos φ sin θ (b) cos 2θ sin φ
π (c) cos θ cos φ (d) sin θ sin φ
(a) 6 (b) 4 (c) 2 (d) 3 29. If 7 cos2 θ + 3 sin2 θ = 4 and 0 < θ < , then what is is equal to
13 2 sin θ − 3 cos θ the value of tan θ? 2
16. If sec θ = , then is equal to (a)
1
(b)
2
(c) 2 2 (d)
3 52. If x = r sin θ cos φ, y = r sin θ sin φ, z = r cos θ, then
5 4 sin θ − 9 cos θ 7 2 2 2
(a) 7 (b) (c) 3 (d) 3 x 2 + y 2 + z 2 is equal to
3
(a) 1 / 3 (b) − 3 (c) 3 (d) 3 40. tan θ = 3 and θ lies in third quadrant, then the (a) r 2 cos 2 φ (b) r 2 sin2 θ + r 2 cos 2 φ
17. sin ( 45° + A) − cos ( 45° − A) is equal to 30. If cos 1° = p and cos 89° = q, then which one of value of sinθ is 1
2
(c) r (d)
the following is correct? 1 1 −3 3 r2
(a) 1 / 2 (b) 1 (c) 0 (d) 2 (a) (b) − (c) (d)
(a) p is close to 0 and q is close to 1 10 10 10 10
1 + sin θ 7 3π
18. What is equal to? (b) p< q 53. If cotθ = and π < θ < , then the value of
1 − sin θ (c) p=q
41. The value of (sin 20° cos 70° + cos 20° sin 70° ) is 24 2
(c) − 1
1 cos θ − sin θ is
(a) sec θ − tan θ (b) sec θ + tan θ (d) p is close to 1 and q is close to 0 (a) 1 (b) 0 (d)
2 19 18
(c) cosec θ + cot θ (d) cosec θ − cot θ π (a) (b)
31. If 0 ≤ θ < and p = sec2 θ, then which one of the 42. If φ and θ are supplementary angles, then 25 25
19. sin ( n + 1) A sin ( n − 1) A + cos ( n + 1) A cos ( n − 1) A 2 17 18
following is correct? (a) sin θ = sin φ (b) cos θ = cos φ (c) (d)
is equal to 25 25
(a) p < 1 (b) p = 1 (c) p > 1 (d) p ≥ 1 (c) tan θ = tan φ (d) sec φ = cosec θ
(a) sin 2 A (b) cos 2 A (c) tan 2 A (d) cot 2 A π 1
cos 11° + sin 11° 32. What is the value of cos 1° cos 2° cos 3°… cos 90° ? 43. If x + y = 90°, then what is the value of 54. If α + β = and sinα = , then β is
20. The value of is 2 2
cos 11° − sin 11° 1 cos x cosec y − cos x sin y ? 1°
(a) (b) 0 (c) 1 (d) 2 (a) 30° (b) 60° (c) 45° (d) 22
(a) tan 56° (b) tan 34° (c) cot 56° (d) tan 11° 2 2
1 − cos B 2 tan A (a) cos x (b) sin x (c) cos x (d) sin x
a 2 − b2 33. If tan A = , then what is equal 55. If sin x − cos x = 0, then what is the value of
21. If sinθ = , then tanθ is equal to sin B 1 − tan2 A 44. The value of 2 + 2 + 2 cos 4 θ is
a 2 + b2 sin4 x + cos4 x ?
to? θ 3 1 1
a + b
2 2
2 ab a −b2 2
ab tan B (a) 2 cos θ 2
(b) 2 cos 2 θ (c) 2 cos θ (d) 2 cos (a) 1 (b) (c) (d)
(a) (b) (c) (d) (a) (b) 2 tanB (c) tan B (d) 4 tan B 2 4 2 4
2 ab a2 − b 2 2 ab a2 + b 2 2
MATHEMATICS Measurements of Angles and Trigonometric Ratios 203 204 CDS Pathfinder

67. cos 3 θ + sin 3 θ is maximum, when θ is sin 38° − cos 68° 91. What is the value of x in the equation
56. If 3 cos x = 5 sin x , then the value of 80. The value of is equal to
cos 68° + sin 38° cosec 2 30° sec2 45°
5 sin x − 2 sec3 x + 2 cos x (a) 15° (b) 30° (c) 45° (d) 60°
x = tan2 60° − tan2 30° ?
is 3 tan 8° 3 cot 8°
5 sin x + 2 sec3 x − 2 cos x 68. If sin2 x + sin2 y + sin2 z = (sin x + sin y + sin z )2,
(a) (b) 8 cos2 45° sin2 60°
(c) 3 sin 38° (d) 3 sin 8° 1 3
361 271 541 127 then which of the following expressions must (a) x = 1 (b) x = 2 (c) x = (d) x =
(a) (b) (c) (d) 81. 2 cos x − cos 3x − cos 5x is equal to 2 2
2397 979 979 979 necessarily vanish?
(a) tan x + tan y + tan z (b) cos x + cos y + cos z 3
(a) 8 cos x sin x 2 2
(b) 12 cos x sin x 3
92. Under which one of the following conditions is
1 + cos x 1 1 1 1 1 1
57. is equal to (c) + + (d) + + (c) 16 cos 3 x sin2 x (d) 32 cos 3 x sin x the trigonometrical identity
1 − cos x sin x sin y sin z cos x cos y cos z sin x/(1 + cos x ) = (1 − cos x )/ sin x true?
(a) sec x + tan x (b) cosec x + cot x 82. cos 4x is equal to (a) x is not a multiple of 360°
(c) sec x − tan x (d) cosec x − cot x
69. If sin x + sin x = 1, 2
then the value of
(a) 1 + 2 sin2 2 x (b) 2 cos 2 2 x (b) x is not an odd multiple of 180°
cos2 x + cos4 x is equal to (c) x is not a multiple of 180°
58. cos4 x − sin4 x is equal to (a) −1 (b) 2 (c) − 2 (d) 1 (c) 1 − 8 sin2 x cos 2 x (d) 1 + 8 sin2 x cos 2 x
(d) None of the above
(a) 2 cos 2 x − 1 (b) 2 sin2 x − 1
70. cos 35° + cos 85° + cos 155° is 83. 2 sin A cos3 A − 2 sin3 A cos A is equal to 93. If cos x = k cos ( x − 2 y ), then tan ( x − y ) tan y is
(c) sin2 θ − cos 2 θ (d) None of these 1 1 1 1 1 1
(a) 0 (b) (c) (d) cos 275° (a) sin 4 A (b) cos 4 A (c) tan 4 A (d) cot 4 A equal to
sin θ 3 2 2 2 2 2 1+ k 1− k 2k k −1
59. is equal to (a) (b) (c) (d)
1 + cos θ 71. sin 12° sin 24° sin 48° sin 84° is 84. If a cos θ − b sin θ = c, then what is the value of 1− k 1+ k k+1 2k + 1
1 − cos θ sin θ cos θ − 1 1 + sin θ sin ( x + y ) − 2 sin x + sin ( x − y )
(a) (b) (c) (d) (a)
1
(b)
3
(c)
3
(d) 0
a sin θ + b cos θ ? 94. The value of is
sin θ cos θ sin θ cos θ 16 64 15 (a) ± a2 + b 2 + c 2 (b) ± a2 − b 2 + c 2 cos ( x + y ) − 2 cos x + cos ( x − y )
 5 cos θ − 4 3 + 5 sin θ  p (a) cot x (b) tan x (c) sin x (d) cosec x
60. If 0 ≤ θ ≤ 90° , then  −  is 72. If tan θ = , then the value of p cos 2 θ + q sin 2 θ is (c) ± a2 + b 2 − c 2 (d) ± a2 − b 2 − c 2
 3 − 5 sin θ 4 + 5 cos θ  q 95. If sin x + sin y = a and cos x + cos y = b, then
equal to (a) p (b) q 85. If cos x + cos2 x = 1, then the value of x+ y
q (3q 2 − p2 ) p (3q 2 − p2 ) tan is
(a) 0 (b) 1 (c)
1
(d)
1 (c) (d) sin8 x + 2 sin6 x + sin4 x is 2
4 2 p +q
2 2
p +q
2 2
(a) 0 (b) − 1 (c) 2 (d) 1 4 b a 4
(a) (b) (c) (d)
4 5 2π 4π a2 + b 2 a b a2 − b 2
61. If cos (α + β ) = and sin (α − β ) = , α , β lies 73. If x = y cos = z cos , then xy + yz + zx is 86. If m = cosec x − sin x and n = sec x − cos x, then
5 13 3 3 tan x is equal to 96. The largest hand of a clock is 42 cm long, then
π equal to 2/ 3 1/ 3 the distance covered by the extremity in 20 min
between 0 and , then the value of tan 2α is
(a)   (b)  
n n
4 (a) 1 (b) − 1 (c) 0 (d) 2 is
56 56 43 34  m  m
(a) (b) (c) (d) 74. If sin θ + cos θ = m and sec θ + cosec θ = n, then 2 (a) 88 cm (b) 80 cm (c) 82 cm (d) 84 cm
(c)   (d)  
33 23 33 33 n n
n ( m 2 − 1) is equal to  m  m 97. If 0° < x < 90° and 2 sin x + 15 cos2 x = 7, then
tan ( 45° + x )
62. is equal to 1 m 1 what is the value of tan x ?
tan ( 45° − x ) (a) (b) m (c) 2m (d)
87. If 2 cos θ = x + , then 2 cos 3 θ is equal to
2m 2 2 3 3 4
2 2 x (a) (b) (c) (d)
 1 + tan x   1 − tan x  3 2 4 3
(a)   (b)   75. If cosec θ − sin θ = a3 , sec θ − cos θ = b3 , then (a) x3 +
1
(b) x2 +
1 x2
 1 − tan x   1 + tan x  (c) (d) None of these
98. If ABC is a triangle and A + B + C = 180°, then
a 2b2 ( a 2 + b2 ) is equal to x3 x2 1+ x2
2 sin2 x
(a) − 1 (b) 1 (c) 2 (d) − 2
tan A + tan B + tan C is
1 A+ B B+ C
(c) 2
x
(d) None of these 88. If cosθ ≥ in the first quadrant, then which one (a) tan A tan B tan C (b) tan + tan
1 + cos 2 76. The maximum value and minimum value of 2 2 2
2 (1 + cos 2x ) are of the following is correct? tan A + tan B
(c) (d) cot A cot B cot C
−1 1 π π π π tan C
63. What is the simplest value of (a) − 1 and 1 (b) 1 and 2 (c) and (d) 0 and 2 (a) θ ≤ (b) θ ≥ (c) θ ≤ (d) θ ≥
3 3 6 6
(1 − sin A cos A) (sin2 A − cos2 A) 2 2
99. If x = a sec θ cos φ, y = b sec θ sin φ and z = c tanθ,
? 3 12
cos A (sec A − cosec A) (sin3 A + cos3 A) 77. If tan θ + tan φ = a and cot θ + cot φ = b, then 89. If sin A = and cos B = , then the value of x2 y2 z2
cot (θ + φ ) is equal to 5 13 then the value of + − is
(a) sin A (b) cos A (c) sec A (d) cosec A tan A − tan B a2 b2 c2
1 1 1 1 is equal to
64. If A + B = 45°, then (1 + tan A) (1 + tan B) is equal (a) + (b) − 1 + tan A tan B (a) 9 (b) 0 (c) 1 (d) 4
a b a b
to (c) a − b (d) a + b 23 16 1 13 100. Which one of the following statements is correct?
(a) (b) (c) (d)
(a) 1 (b) − 1 (c) 2 (d) − 2 16 63 63 63
78. tan 75° − tan 30° − tan 75° tan 30° is equal to (a) The squares of the tangents of the angles 30°, 45°,
 cos 2B − cos 2 A 90. If B + C = 60°, then which of the following is 60° are in G.P.
65.   is equal to (a) − 1 (b) 1 (c) 0 (d) 2 (b) The squares of the sines of the angles 30°, 45°, 60°
 sin 2B + sin 2 A correct statement?
79. The value of sin2(15° + A) − sin2(15° − A) is equal are in G.P.
(a) tan ( A − B) (b) tan ( A + B) (c) cot ( A − B) (d) cot ( A + B) (a) sin (120° − B) = sin (120° − C )
(c) The squares of the secants of the angles 30°, 45°,
to (b) sin (120° + B) = sin (180° + C ) 60° are in A.P.
66. sec2 x + tan2 x = 7, the value of x is (a)
1 1
cos 2 A (b) sin 2 A (c)
1
tan 2 A (d) cot 2 A (c) cos (120° − B) = sin (120° + C ) (d) The squares of the tangents of the angles 30°, 45°,
(a) 15° (b) 30° (c) 45° (d) 60° 2 2 2 (d) tan (B) = tan (120° + C ) 60° are in A.P.
MATHEMATICS Measurements of Angles and Trigonometric Ratios 205 206 CDS Pathfinder

π π Directions. Q. Nos. (115-117) Let sin( A + B) = 1 and Which of the above statement(s) is/are correct?
101. If A, B, C and D are the successive angles of a 110. If A = and B = , then which of the following π
sin( A − B) = , where A , B ∈ 0, .
1
cyclic quadrilateral, then what is 6 3 (a) Only I (b) Only II e 2012 I
2  2 
cos A + cos B + cos C + cos D are equal to? is/are correct? (c) Both I and II (d) Neither I nor II
(a) 4 (b) 2 (c) 1 (d) 0 I. sin A + sin B = cos A + cos B 115. What is the value of A? 126. Consider the following statements
II. tan A + tan B = cot A + cot B π π π π I. sin 2 1°+ cos 2 1° = 1
102. Which one of the following is correct? (a) (b) (c) (d)
(a) There is only one θ with 0° < θ < 90° such that Select the correct answer using the codes given 6 3 4 8 II. sec2 33°− cot2 57° = cosec2 37°− tan 2 53°
sin θ = a, where a is a real number below 116. What is the value of tan ( A + 2B) tan (2 A + B)? Which of the above statement(s) given above
(b) There is more than one θ with 0° < θ < 90° such that (a) Only I
(a) −1 is/are correct? e 2012 I
sin θ = a, where a is a real number (b) Only II (b) 0 (c) 1 (d) 2
(c) There is no θ with 0° < θ < 90° such that sin θ = a, (a) Only I (b) Only II
where a is a real number
(c) Both I and II 117. What is the value of sin2 A − sin2B ? (c) Both I and II (d) Neither I nor II
(d) Neither I nor II (a) 0 (b) 1 / 2 (c) 1 (d) 2
(d) There are exactly two θ ’s with 0° < θ 90° such that
sin θ = a, where a is a real number 127. Consider the following statements:
111. For any quadrilateral ABCD which of the
Directions. Q. Nos. (118-120) ABC is an obtuse I. There is only one value of x in the first quadrant
103. If sin ( B + C − A) = cos (C + A − B) = tan ( A + B − C ) following statements are true?
that satisfies sin x + cos x = 2.
triangle and tanA, tanB are the roots of the equation
= 1, then the angles A, B, C which are positive I. sin ( A + B) + sin (C + D ) = 0
3x 2 − 2 3x + 1 = 0. II. There is only one value of x in the first quadrant
acute angles are, respectively II. cos ( A + B) + cos (C + D ) = 0 that satisfies sin x − cos x = 0.
1° 1°
(a) 45° , 80° , 105° (b) 22 , 67 , 45° Select the correct answer using the codes given 118. The measure of angle C is Which of the above statement(s) given above
2 2 below 2π 5π
(c) 20°, 70°, 90° (d) 30°, 60°, 90° (a) π / 3 (b) π / 2 (c) (d) is/are correct? e 2012 I
(a) Only I (b) Only II 3 6 (a) Only I (b) Only II
104. If A, B, C and D be the angles of a cyclic (c) Both I and II (d) None of these (c) Both I and II (d) Neither I nor II
119. Find the value of sin C + cos C.
quadrilateral taken in order, then cos (180° + A) 112. Consider the following statements
+ cos (180° + B) + cos (180° + C ) + cos (180° + D ) is (a)
2
(b)
3+1
(c)
3 −1
(d) 3 +
1 128. If sin θ cos θ = 3 / 4, then sin4 θ + cos4 θ is equal
I. 1° in radian measure is less than 0.03 radians. 3+1 2 2 2
equal to to e 2012 II
II. 1 radian in degree measure is greater than 45°.
(a) 2 (cos A + cos B) (b) 2 (cos A + cos D) (a) 7/8 (b) 5/8 (c) 3/8 (d) 1/8
1 + tan2 C
(c) 0 (d) 4 cos A Which of the above statement is/are correct? 120. If 2sin B = , then the value of B is
(a) Only II (b) Only I cosec2 C 129. If the angle θ is in the first quadrant and
105. If ABC is a right angled triangle at C and having tan θ = 3, then what is the value of (sin θ + cos θ )?
(c) Neither I nor II (d) Both I of II π π 2π π
u units, v units and w units as the lengths of its (a) (b) (c) (d) e 2012 II
sides opposite to be vertices A, B and C 2 3 3 6 1 2 3 4
113. Consider the following statements (a) (b) (c) (d)
respectively, then what is tan A + tan B equal to? 1 10 10 10 10
I. sin θ ⋅ sin(60°+ θ ) ⋅ sin(60°− θ ) = sin 3 θ
(a)
u 2
(b) 1 (c) u + v (d)
w 2
4 PREVIOUS YEARS’ QUESTIONS 130. If 0° < θ < 90°, then all the trigonometric ratios
vw uv 1
II. cos θ ⋅ sin(30°+ θ ) ⋅ sin(30°− θ ) = cos 3 θ 121. The expression sin2 x + cos2 x − 1 = 0 is satisfied can be obtained when e 2012 II
4 by how many values of x? e 2012 I (a) only sin θ is given
106. The Earth takes 24 h to rotate about its own axis. 1
Through what angle will it turn in 4 h and 12 min? III. sin θ ⋅ cos(30°+ θ ) ⋅ cos(30°− θ ) = sin 3 θ (a) Only one value of x (b) Two values of x (b) only cos θ is given
4 (c) Infinite values of x (d) No value of x (c) only tan θ is given
(a) 63° (b) 64° (c) 65° (d) 70°
Which of the above statement is/are correct? (d) any one of the six ratios is given
107. Assume the Earth to be a sphere of radius R. 122. If 3 sin x + 5 cos x = 5, then what is the value of
(a) Only I ( 3 cos x − 5 sin x ) ?
What is the radius of the circle of latitude 40°S? e 2012 I
131. What is the value of sin A cos A tan A + cos A
(a) R cos 40° (b) R sin 80° (b) Only II (a) 0 (b) 2 (c) 3 (d) 5 sin A cot A ? e 2012 II
(c) R sin 40° (d) R tan 40° (c) Only III
123. If p = a sin x + b cos x and q = a cos x − b sin x, then (a) sin2 A + cos 2 A (b) sin 2 A + tan 2 A
(d) All of the above
108. If sin 3 θ = cos (θ − 2° ), where 3θ and (θ − 2° ) are what is the value of p2 + q 2? e 2012 I (c) sin2 A + cot 2 A (d) cosec 2 A + cot 2 A
acute angles, then what is the value of θ? 114. If ABC is a right angled triangle, then which of (a) a + b (b) ab (c) a2 + b 2 (d) a2 − b 2 sin θ 1 + cos θ
(a) 22° (b) 23° the following statements are correct? 132. What is the value of + ?
124. If α and β are complementary angles, then what 1 + cos θ sin θ e 2012 II
(c) 24° (d) 25° I. sin ( A + B) = sin C
1 (a) 2 cosec θ (b) 2 sec θ
109. Consider the following statements:  A + B C −
II. sin   = cos  sin α cos α  2 (c) sec θ (d) cosec θ
cos 2 θ − sin 2 θ  2  2 is cosec α ⋅ cosec β  +  equal to?
I. = cos 2 θ (1 + tan θ ) (1 − tan θ )  sin β cos β 
cos 2 θ + sin 2 θ (A + B − C ) e 2012 I 133. What is the value of sec2 D − tan2 D ?
III. tan = cot C e 2013 I
2 (a) 0 (b) 1
1 + sin θ (a) 1/2 (b) 2/3
II. = (tan θ + sec θ )2 (A − B − C ) (c) 2 (d) None of these
1 − sin θ IV. tan = − cot A (c) 1 (d) None of these
2 125. Consider the following statements
Which of the statement(s) given above is/are cot 30° + 1 134. If cos A + cos2 A = 1, then what is the value of
Select the correct answer using the codes given I. = 2(cos 30° + 1)
correct?
below cot 30° − 1 2 (sin2 A + sin4 A)? e 2013 I
(a) Only I (b) Only II
(a) I and II (b) I, II and III
(c) Both I and II (d) Neither I nor II II. 2 sin 45° cos 45°− tan 45° cot 45° = 0 (a) 4 (b) 2 (c) 1 (d) 1/2
(c) I, II and IV (d) All of these
MATHEMATICS Measurements of Angles and Trigonometric Ratios 207 208 CDS Pathfinder

154. If α and β are complementary angles, then what sec x


135. (1 − tan A)2 + (1 + tan A)2 + (1 − cot A)2 143. What is cosec ( 75° + θ) − sec (15° − θ) equal to? 166. What is equal to?
is cos α cosec β − cos α sin β equal to? cot x + tan x e 2014 II
+ (1 + cot A)2 is equal to e 2013 I
e 2013 II e 2014 I
(a) sinx (b) cos x (c) tanx (d) cot x
(a) 0 (b) 1 (c) 2 sin θ (d) 2cosθ (a) sec β (b) cos α (c) sin α (d) − tan β
(a) sin2 A cos 2 A (b) sec 2 A cosec 2 A
144. If 5 sin θ + 12 cos θ = 13, then what is 167. What is (1 + cot x − cosec x )(1 + tan x + sec x ) equal
(c) 2 sec 2 A cosec 2 A (d) None of these 155. If sec θ + tan θ = 2, then what is the value of
5 cos θ − 12 sin θ equal to? to? e 2014 II
e 2013 II sec θ? e 2014 I (a) 1 (b) 2 (c) sinx (d) cos x
1 + 2 sin θ cos θ (a) − 2 (b) − 1 (c) 0 (d) 1 3 5 5
136. If a 2 = , then what is the value of (a) (b) 2 (c) (d)
1 − 2 sin θ cos θ 145. If sin θ − cos θ = 0, then what is sin4 θ + cos4 θ 2 2 4 168. What is (cosec x − sin x )(sec x − cos x )(tan x + cot x )
a+1 equal to? e 2014 II
? equal to? e 2013 II 156. What is cos ec ( 75° + θ) − sec (15° − θ)
a−1 e 2013 I (a) sin x + cos x (b) sin x − cos x
(a) 1 (b)
3
(c)
1
(d)
1 − tan( 55° + θ ) + cot ( 35° − θ) equal to? e 2014 I
(c) 2 (d) 1
(a) sec θ (b) 1 (c) 0 (d) tan θ 4 2 4 3
(a) − 1 (b) 0 (c) 1 (d) sin x − cos x + 1
x2 − y2 cos2( 45° + θ ) + cos2( 45° − θ ) 2 169. What is equal to?
137. If sin θ = , then which one of the following 146. What is equal to? sin x + cos x − 1 e 2014 II
x +y 2 2 tan( 60° + θ ) tan( 30° − θ ) e 2013 II 157. What is sin 25° sin 35° sec 65° sec 55° equal to? sin x − 1 sin x + 1 sin x − 1 sin x + 1
(a) −1 (b) 0 (c) 1 (d) 2 (a) (b) (c) (d)
is correct? e 2013 I e 2014 I cos x cos x cos x + 1 cos x + 1
1
(a) −1 (b) 0 (c) (d) 1
(a) cos θ =
2x y
(b) cos θ =
2x y 147. If tan θ + sec θ = m, then what is sec θ equal to? 2 170. What is (sin2 x − cos2 x ) (1 − sin2 x cos2 x ) equal
x 2 − y2 x 2 + y2 e 2013 II 2 cos θ − sin θ to? e 2014 II
(c) cos θ =
x− y
(d) cos θ =
x y (x − y) m2 − 1 m2 + 1 m+ 1 m2 + 1 158. If 2 cot θ = 3, then what is equal
(a) (b) (c) (d) 2 cos θ + sin θ (a) sin4 x − cos 4 x (b) sin6 x − cos 6 x
x 2 + y2 x 2 + y2 2m 2m m m
to? e 2014 I (c) cos x − sin x
8 8
(d) sin x − cos x
8 8

138. Consider the following statements for 0 ≤ θ ≤ 90°: 3 2 1 1 3


148. If α , β and γ are acute angles such that sinα = , (a) (b) (c) (d)
171. What is (sin x cos y + cos x sin y )
2 3 3 2 4
I. The value of sin θ + cos θ is always greater
3 1 (sin x cos y − cos x sin y ) equal to? e 2014 II
than 1. cosβ = and tan γ = 1, then what is α + β + γ 159. If sin θ cos θ = , then what is sin6 θ + cos6 θ equal (a) cos 2 x − cos 2 y (b) cos 2 x − sin2 y
II. The value of tan θ + cot θ is always greater 2 2
than 1. equal to? e 2013 II to? e 2014 I (c) sin x − cos y
2 2
(d) sin x − sin y
2 2

(a) 105° (b) 120° 1


Which of the statement(s) given above is/are
(c) 135° (d) 150°
(a) 1 (b) 2 (c) 3 (d)
4
172. If A + B + C = 180°, then cot A cot B + cot B cot C
correct? + cot C cot A is equal to e 2014 II
(a) Only I (b) Only II e 2013 I (1 + sec θ − tan θ ) cos θ 160. If cos x + sec x = 2 , then what cosn x + secn x equal
149. What is equal to? (a) − 1 (b) 2 (c) π (d) 1
(c) Both I and II (d) Neither I nor II (1 + sec θ + tan θ ) (1 − sin θ ) e 2013 II to, where n is a positive integer? e 2014 I
173. If sin x + cosec x = 2, then what is sin x + cosec9x 9
(a) 1 (b) 2 (c) tan θ (d) cot θ (a) 2 (b) 2 n − 2 (c) 2 n − 1 (d) 2 n
Directions (Q. Nos. 139-141) Read the following equal to? e 2014 II
information carefully to answer the questions that 150. If ∆ABC is right angled at C, then what is 161. If sin θ + 2 cos θ = 1, where 0 < θ < π / 2, then what (a) 2 (b) 18 (c) 512 (d) 1024
follow. cos( A + B) + sin( A + B) equal to? e 2013 II is 2 sin θ − cos θ equal to? e 2014 I
The angles A, B, C and D of a quadrilateral ABCD are in 1 (a) − 1 (b) 1 / 2 (c) 2 (d) 1
174. If sin x + cos x = p and sin3 x + cos3 x = q, then
(a) 0 (b) (c) 1 (d) 2
2 what is p3 − 3 p equal to e 2014 II
the ratio 1 : 2 : 4 : 5. π
162. If tan 8 θ = cot 2 θ, where 0 < 8 θ < , then what is (a) 0 (b) −2q (c) 2q (d) 4q
151. Consider the following statements: 2
139. What is the value of cos ( A + B) ? e 2013 I
I. tan θ increases faster than sin θ as θ increases. the value of tan 5θ? e 2014 I 175. Consider the following statements e 2014 II
(a) 0 (b) 1/2
II. The value of sin θ + cos θ is always greater (a)
1
(b) 1 (c) 3 (d) 0 I. sin 1° > sin 1 II. cos 1° < cos 1
(c) 1 (d) None of these
than 1. 3
140. What is the value of cosec (C − D + B)? Which of the statement(s) given above is/are
e 2013 I Which of the statement(s) given above is/are 163. If sin ( A + B ) = 1, where 0 < B < 45°, then what is correct?
(a) 1 (b) 2 (c) 3 (d) 4 correct? e 2013 II cos( A − B) equal to? e 2014 I (a) Only I (b) Only II
141. Consider the following statements: (a) Only I (b) Only II (a) sin 2B (b) sin B (c) cos 2B (d) cos B (c) Both I and II (d) Neither I nor II
(c) Both I and II (d) Neither I nor II
I. ABCD is a cyclic quadrilateral. 164. At what point of time after 3 O’clock, hour hand 176. The value of cosec 67°+ sec2 57°– cot2 33°– tan2 23°
2

II. sin ( B − A ) = cos ( D − C ) 152. The value of cos 25° − sin 25° is e 2014 I and the minute hand of a clock occur at right is e 2015 I
(a) positive but less than 1 angles for the first time? e 2014 I (a) 2 2 (b) 2 (c) 2 (d) 0
Which of the statement(s) given above is/are 1
(b) positive but greater than 1 (a) 9 O’clock (b) 4 h 37 min
correct? e 2013 I
(c) negative 6 177. If tan( A + B) = 3 and tan A = 1, then tan( A – B)
(a) Only I (b) Only II 8 8 is equal to e 2015 I
(d) 0 (c) 3 h 30 min (d) 3 h 32 min
(c) Both I and II (d) Neither I nor II 11 11 1
(a) 0 (b) 1 (c) (d) 2
153. In a right angled ∆ABC, right angle at B, if 3
142. If sin θ + cos θ = 3, then what is tan θ + cot θ 4 165. If tan θ + cot θ = 2, then what is sin θ + cos θ equal
cos A = , then what is sin C is equal to? 178. If tan A + cot A = 4, then tan4 A + cot4 A is equal
equal to? e 2013 II 5 e 2014 I to? e 2014 II
3 4 3 2 1 1 to e 2015 I
(a) 1 (b) 2 (c) 2 (d) 3 (a) (b) (c) (d) (a) (b) (c) 2 (d) 1
5 5 4 5 2 3 (a) 110 (b) 191 (c) 80 (d) 194
MATHEMATICS Measurements of Angles and Trigonometric Ratios 209 210 CDS Pathfinder

198. If p = cot θ + tan θ and q = sec θ − cos θ, then 1 + cos θ


179. ABC is a triangle right angled at B and 188. If tan( x + 40)° tan( x + 20)° tan( 3x )° II. = cosec θ + cot θ
AB : BC = 3 : 4. What is sin A + sin B + sin C equal ( p2q )2/ 3 − ( q 2 p)2/ 3 is equal to 1 − cos θ
tan( 70 − x )° tan( 50 − x )° = 1, then the value of x is e 2016 I
to? e 2015 I equal to e 2015 II (a) 0 (b) 1 (c) 2 (d) 3 Which of the above is/are identity/ identities?
11 12 e 2016 I
(a) 2 (b) (c) (d) 3 (a) 30 (b) 20 (c) 15 (d) 10 x y x y
5 5 199. If − tanθ = 1 and tan θ + = 1, then the (a) Only I (b) Only II
189. The value of a b a b (c) Both I and II (d) Neither I nor II
180. If sin x + cos x = C, then sin x + cos x is equal to
6 6
 π  π  π x 2
y 2
32 cot2   − 8 sec2   + 8 cos3   is equal to value of 2 + 2 is
e 2015 I  4  3  6 cos2 θ − 3cos θ + 2 π
e 2015 II a b e 2016 I 203. If = 1, where 0 < θ < , then
1 + 6 C – 3C
2 4
1 + 6 C – 3C
2 4
(a) 2sec 2 θ (b) sec 2 θ (c) cos 2 θ (d) 2 cos 2 θ sin2 θ 2
(a) (b) (a) 3 (b) 2 3 (c) 3 (d) 3 3
16 4 which of the following statement(s) is/are correct?
1 + 6 C 2 + 3C 4 1 + 6 C 2 + 3C 4
200. Which of the following is correct in respect of the
190. If x = a cosθ and y = b cotθ, then equation 3 − tan2 θ = α(1 − 3tan2 θ )?
(c) (d) I. There are two values of θ satisfying the above
16 4 ( ax −1 − by −1 ) ( ax −1 + by −1 ) is equal to e 2015 II (given that α is a real number.) e 2016 I equation.
3 – tan2 A (a) 0 (b) 1 (c) tan2 θ (d) sin2 θ II. θ = 60° is satisfied by the above equation.
= k, where k is a real number, then (a) α∈  , 3 (b) α∈  −∞,  ∪ [3, ∞ )
181. If 1 1
1 – 3 tan2 A 191. If θ is an acute angle and  3   3  Select the correct answer using the codes given
cosec A (3 sinA – 4 sin3 A) is equal to sin θ cos θ = 2 cos3 θ − 1.5 cos θ, then what is sinθ (c) α∈  −∞,  ∪ [3, ∞ )
e 2015 I 1 below e 2016 I
(d) None of these
2k 2k 1 equal to? e 2015 II  3 (a) Only I (b) Only II
(a) (b) , where ≤ k ≤ 3
k–1 k–1 3 5 −1 1− 5 5+ 1 5+1 201. A person goes to a market between 4 pm and (c) Both I and II (d) Neither I nor II
(a) (b) (c) (d) −
2k 1 2k 4 4 4 4 5 pm. When he comes back, he finds that the 204. Consider the following statements
(c) , where k < or k > 3 (d)
k–1 3 k+1 192. A clock is started at noon by 10 minutes past 5, hour hand and the minute hand of the clock I. There exists a positive real number m such that
through what angle, the hour hand moves? have interchanged their positions. For how much cos x = 2m + 1.
1 – sin x 1 – sin x cos x
182. If p = , q= , r= , then time (approximately) was he out of his house?
II. mn ≥ m + n, for all m, n belonging to set of
1 + sin x cos x 1 + sin x e 2015 II
e 2016 I
(a) 160° (b) 145° (c) 150° (d) 155° natural numbers.
which of the following is/are correct? e 2015 I (a) 55.38 min (b) 55.48 min (c) 55.57 min (d) 55.67 min
I. p = q = r II. p2 = qr 193. Consider the following statements e 2015 II Which of the above statement(s) is/are correct?
202. Consider the following (a) Only I (b) Only II e 2016 I
I. sin 66° is less than cos 66°.
Select the correct answer using the codes given 1 − cos θ (c) Both I and II (d) Neither I nor II
below II. sin 26° is less than cos 26°. I. = cosec θ − cot θ
1 + cos θ
(a) Only I (b) Only II Which of the above statement(s) is/are correct?
(c) Both I and II (d) Neither I nor II (a) Only I (b) Only II

183. Consider the following identity


(c) Both I and II (d) Neither I nor II ANSWERS
cos A sin A 194. Consider the following statements.
I. + = sin A + cos A 2
1 b 2 a 3 b 4 c 5 b 6 b 7 a 8 b 9 c 10 b
1 – tan A 1 – cot A 1 + tan 2 θ  1 − tan θ 
I. =  is true for all 11 b 12 b 13 b 14 a 15 c 16 c 17 c 18 b 19 b 20 a
II. (1 – sin A – cos A )2 = 2(1 – sin A )(1 + cos A ) 1 + cot2 θ  1 − cot θ 
21 c 22 d 23 b 24 a 25 a 26 b 27 a 28 b 29 d 30 d
Which of the above identity/identities is/are π π
0 < θ < ,θ ≠ 31 d 32 b 33 c 34 c 35 b 36 d 37 a 38 d 39 c 40 c
correct? 2 4
1 41 a 42 a 43 b 44 c 45 b 46 c 47 d 48 c 49 b 50 a
(a) Only I (b) Only II e 2015 I II. cot θ = is true for θ = 45° only.
(c) Both I and II (d) Neither I nor II tan θ 51 c 52 c 53 c 54 b 55 c 56 b 57 b 58 a 59 a 60 a
Which of the above statements is/are correct? 61 a 62 a 63 a 64 c 65 a 66 d 67 a 68 c 69 d 70 a
184. If a and b are positive, then the relation
2a + 3b e 2015 II
sinθ = is (a) Only I (b) Only II 71 a 72 d 73 c 74 c 75 b 76 d 77 b 78 b 79 b 80 a
3b e 2015 II
(c) Both I and II (d) Neither I nor II 81 c 82 c 83 a 84 c 85 d 86 b 87 a 88 a 89 b 90 a
(a) not possible (b) possible, if a = b
(c) possible, if a > b (d) possible, if a < b π 91 a 92 c 93 b 94 b 95 c 96 a 97 d 98 a 99 c 100 a
195. In a ∆ABC, if A − B = , then C + 2B is equal to
2 101 d 102 a 103 b 104 c 105 d 106 a 107 a 108 b 109 c 110 c
185. The minimum value of cos x + cos y − cos z is
2 2 2 e 2016 I
2π 3π π 111 a 112 d 113 d 114 d 115 b 116 c 117 b 118 c 119 c 120 b
e 2015 II (a) (b) (c) π (d)
3 4 2 121 c 122 c 123 c 124 b 125 c 126 c 127 b 128 b 129 d 130 d
(a) − 1 (b) 0
2 2
(c) 2 (d) 2  sin 35°   cos 55°  131 a 132 a 133 c 134 b 135 c 136 d 137 b 138 b 139 a 140 b
196.   −  + 2 sin 30° is equal to
186. If tan θ + sec θ = 2, then tanθ is equal to e 2015 II  cos 55°   sin 35°  e 2016 I
141 d 142 a 143 a 144 c 145 c 146 c 147 b 148 c 149 a 150 c
3 5 3 5 (a) −1 (b) 0 (c) 1 (d) 2 151 a 152 a 153 b 154 c 155 d 156 b 157 d 158 c 159 d 160 a
(a) (b) (c) (d)
4 4 2 2
4 π 161 c 162 b 163 a 164 d 165 c 166 a 167 b 168 d 169 b 170 b
cos θ  π 197. If tan θ + cot θ = , where 0< θ< , then
187. is equal to  where, θ ≠  3 2 171 d 172 d 173 a 174 b 175 d 176 b 177 c 178 d 179 c 180 b
1 − sin θ  2 e 2015 II sin θ + cos θ is equal to e 2016 I 181 c 182 c 183 a 184 a 185 a 186 a 187 b 188 c 189 d 190 b
tan θ − 1 1 + sin θ tan θ + 1 1 + cos θ 3 −1 3+1 191 a 192 d 193 b 194 a 195 d 196 c 197 c 198 b 199 d 200 c
(a) (b) (c) (d) (a) 1 (b) (c) (d) 2
tan θ + 1 cos θ tan θ − 1 sin θ 2 2 201 a 202 c 203 b 204 d
MATHEMATICS Measurements of Angles and Trigonometric Ratios 211 212 CDS Pathfinder

14. (a) log (tan 1° ) + log (tan 2° ) +…+ log (tan 89° ) 23. (b) As we know, sin x is increasing from 0 to 90°.
HINTS AND SOLUTIONS = log (tan 1° tan 2°… tan 45° … tan 88° tan 89° )
= log [(tan 1° cot 1° ) (tan 2° cot 2° ) … tan 45° ]
∴ sin y > sin x
24. (a) Given, (sin θ + cosec θ ) = 2.5
[Q tan 89° = tan( 90° − 1° ) = cot 1°]  
⇒  sin θ + 1  = 5 ⇒ 2 sin 2 θ − 5 sin θ + 2 = 0
1. (b) We know that, π radian = 180° 6. (b) Let the hands of a clock coincides 9. (c) Given, 3 tan θ = 4 ⇒ tanθ =
4 = log( 1°⋅1° ... 1° ) = 0 
 sin θ  2
180° 180° after ‘t’ min. 3 1
⇒ 1 radian = = ×7 15. (c) Q Given, tan θ + =2
π 22 Angle traced by hour hand in 1 − sin θ 1 − sin θ 1 − sin θ tan θ ⇒ 2 sin 2 θ − 4 sin θ − sin θ + 2 = 0
= × 2
630° 3° 3 × 60
On squaring both side, we get  tan θ +
1 1 + sin θ 1 + sin θ 1 − sin θ 1  ⇒ 2 sin θ (sin θ − 2) − 1 (sin θ − 2) = 0
 =2
2
= = 57 = 57° + min t min = t
11 11 11 2 1 – sin θ  tan θ  1
4 = ⇒ ( 2 sin θ − 1 ) (sin θ − 2) = 0 ⇒ sin θ = (Q sin θ ≠ 2)
= 57° + 16′ + 1 1
min Angle traced by minute hand in 1 – sin 2 θ ⇒ tan θ +
2
+ 2 = 4 ⇒ tan θ +2
=2 2
11 t min = 6t tan 2 θ tan 2 θ ∴ θ = 30°
1 − sin θ 1 sin θ
4 = = −
= 57° + 16′ + × 60 s Angle between hour hand and minute cos θ cos θ cos θ 13 25. (a) Given,
cos x
= n and
sin x
=m ...(i)
11 16. (c) Given, sec θ =
hand at 5 pm = 5 × 30 = 150° 5 cos y sin y
= 57° + 16′ + 21.8′′ = sec θ − tan θ
1 150 × 2 300 169 144 12  sin 2 x cos 2 x 
= 57° 16′ 218
. ′′ = 57° 16′ 22′′ ∴ 6t − t = 150° ⇒ t = = = 1 + tan θ − tan θ
2 ∴ tan θ = sec 2 θ − 1 = −1 = = Now, ( m − n ) sin y =  2 −
2 2 2 2
 sin y
2 11 11 25 25 5  sin y cos 2 y 
30  °
2. (a) − 47° 30′ = −  47 +
2
1 +   2 sin θ
 3 4 4
 = 27 + = 27 min 16 sec = − 2 sin θ − 3 cos θ −3 ( 1 − cos 2 x) cos 2 y − cos 2 x ( 1 − cos 2 y )
60  3 =
1 °
11 3 ∴ = cos θ cos 2 y
= −  47 +  Q 1′ =  1  °  Therefore, required time = 5:27:16 pm 4 sin θ − 9 cos θ sin θ
    16 4 5 4 1 4 −9
 2  60   = 1+ − = − = cos θ cos 2 y − cos 2 x
1 9 3 3 3 3 = = 1 − n2 [from Eq. (i)]
94 + 1 ° −95° 7. (a) Given, sin A = , (dividing numerator and denominator by cos θ) cos 2 y
= −   = 3
 2  2 10. (b) Given, tan A = 1 = tan 45° 2×
12
−3 1
C 2 tan θ − 3 9/5 9 26. (b) Given, p = tan 2 x + cot 2 x = tan 2 x +
−95 π −19 π ⇒ A = 45° and tan B = 3 = tan 60° = = 5 = = =3 tan 2 x
= × = rad 4 tan θ − 9 4 × 12
∴ B = 60° − 9 3/5 3 1/2
, ≥ 2  tan 2 x ⋅
2 180 72 1 1 
5 Q AM ≥ GM ⇒ tan 2 x + 
3. (b) Here, r = 60 cm, l = 16.5 cm 3 Now, cos A cos B − sin A sin B tan 2 x  tan 2 x 
c 1 17. (c) sin ( 45° + A ) − cos ( 45° − A )
= cos 45° cos 60°− sin 45° sin 60° 1
∴ θ = = 
l 16. 5  = sin( 45°+ A ) − cos[( 90°− ( 45°+ A )] = sin( 45°+ A ) − sin( 45°+ A ) ⇒ tan 2 x + ≥ 2 ⇒ p≥ 2
 tan 2 x
r  60  1 1 1 3 1− 3 =0
° ° = × − × =
= 
165.
×
180   165
. 180
× 7 B 2 2 2 2 2 2 1 + sin θ ( 1 + sin θ)( 1 + sin θ) 27. (a) Let angles in circular measure are A and B, then degree
 =  × A 18. (b) =
measures will be A 
 60 π   60 22  2√2 1 − sin θ ( 1 − sin θ)( 1 + sin θ) 180   180  .
11. (b) cosec 2 θ − 2 + sin 2θ  and B  
°  π   π 
=   = 15° 45′
63 In right angled ∆ABC , ( 1 + sin θ)2 ( 1 + sin θ)2 1 + sin θ 1 sin θ
 4 = (sin θ − cosec θ)2 = = = = + By given condition, A+ B=1 …(i)
AB 2 = AC 2 − BC 2 = 32 − 12 = 8 1 − sin 2 θ cos 2 θ cos θ cos θ cos θ
A 
Hence, it is always non-negative. 180   180  = 1
4. (c) Speed of train = 90 km/h ⇒ AB = 2 2 = sec θ + tan θ  − B  …(ii)
 π   π 
12. (b) sin 2 15° + sin 2 20° + sin 2 25°
=  90 ×  m/sec = 25 m/sec
5 AB 2 2 19. (b) sin ( n + 1) A sin ( n − 1) A + cos ( n + 1) A cos ( n − 1) A
∴ cos A = = 180
 18  + ... + sin 2 75° = cos [( n + 1) A − ( n − 1) A ] On multiplying Eq. (i) by and adding to Eq. (ii), we get
AC 3 π
Distance moved in 11 sec 3 = (sin 15°+ sin 75° )
2 2 [Q cos A cos B + sin A sin B = cos( A – B )] π
2 A 
AC 180  180 1 11  180
+ 1 = +
and sec A = = 1
= cos [ nA + A − nA + A ] = cos 2 A  = + 1 ⇒ A = ×
= ( 25 × 11) m = 275m AB 2 2
+ (sin 2 20°+ sin 2 70° ) + ...+ sin 2 45°
 π  π 2 180  π  2 360
cos 11°+ sin 11°
∴ l = 275 m, r = 1500 m tan A =
BC
=
1 20. (a)  1 π  1 π
= [sin 2 15°+ sin 2 ( 90°−15° )] cos 11°− sin 11° ∴ B = 1− A = 1−  +  = −
l  275 
c AB 2 2  2 360  2 360
Q θ= ⇒ θ=   + [sin 2 20°+ sin 2 ( 90°−20° )]+ ...+ sin 2 45°
Divide numerator and denominator by cos 11°
r  1500  and cosec A =
AC
=3 1 + tan 11° tan 45°+ tan 11° 28. (b) [( 1 − sin 2 θ) sec 2 θ + tan 2 θ] (cos 2 θ + 1)
° ° BC = (sin 2 15°+ cos 2 15° ) = = = tan ( 45° + 11° ) = tan 56°
1 − tan 11° 1 − tan 45° tan 11° = [cos 2 θ ⋅ sec 2 θ + tan 2 θ] (cos 2 θ + 1) (Q sin 2 θ + cos 2 θ = 1)
= 
275 180   275 180
×  = × × 7 ∴ cos A ⋅ cosec A + tan A ⋅ sec A + (sin 20°+ cos 20° )+ ...+ 1 / 2
2 2
 1500 π   1500 22  a2 − b2 = ( 1 + tan 2 θ) (cos 2 θ + 1) = sec 2 θ (cos 2 θ + 1)
 21
° 2 2 3 1 3 = 6 + 1 / 2 = 13 / 2 sin θ sin θ a2 + b2 (Q sec 2 θ − tan 2 θ = 1)
=   = 10°30′ = ⋅ + ⋅ 21. (c) tan θ = = =
 2 3 1 2 2 2 2 ( Q sin 2 θ + cos 2θ = 1) cos θ 1 − sin θ
2
( a 2 − b 2 )2 = sec 2 θ ⋅ cos 2 θ + sec 2θ = 1 + sec 2 θ > 2
1− 2
3 16 2 + 3 ( a + b 2 )2 (Q sec 2 θ > 1 for 0 < θ < 90° )
5. (b) Angle traced by hour hand in =2 2+ = 5 sin 75° sin 77°+ 2 cos 13° cos 15°
1 hr = 30° 8 8 13. (b) a2 − b2 29. (d) Given, 7 cos 2 θ + 3 sin 2 θ = 4 ⇒ 7 cos 2 θ + 3 ( 1 − cos 2 θ) = 4
cos 15° sin 77° a2 − b2 a2 − b2
Angle traced by hour hand in 8. (b) cos 15° − sin 15° = cos 15° 7 sin 81° = a2 + b2 = = ⇒ 3 − 4 cos 2 θ = 4 ⇒ cos 2 θ = 1/ 4
− ( a + b 2 )2 − ( a 2 − b 2 )2
2
4a 2 b 2 2ab 1
°
− sin ( 90° − 75° ) cos 9° Now, tan 2 θ = sec 2θ − 1 = − 1 ⇒ tan 2 θ = 4 − 1 = 3
hrs =  30 ×  = 227°30′
91 91
5 cos 15° sin 77°+ 2 sin 77° cos 15° a2 + b2 cos 2θ
12  12  = cos 15° − cos 75° = ∴ tanθ = 3 (Q 0 < θ < π / 2)
Angle traced by minute hand in 15°+75° 75°−15° cos 15° sin 77° 22. (d) Given, cosθ = 0.96, sin θ = 1 − cos 2 θ = 1 − (0.96)2
= 2 sin ⋅ sin 7 cos 9° 30. (d) We know that, the value of cos θ is decreasing in the interval
1min = 6° 2 2 − sin θ 0.28 7
= 1 − 0.9216 = 0.28, tan θ = = = 0 ≤ θ ≤ 90°
Angle traced by minute hand in Q cos C − cos D = 2 sin C + D sin D − C  cos 9°
  7 cos 15°⋅ sin 77° 7 cos 9° cos θ 0.96 24 ∴ cos 1° > cos 89° ⇒ p > q
35 min = ( 6 × 35)° = 210°  = −
2 2  1 1 100 24 196
∴ Required angle = 227°30′ − 210° 1 1 1 cos 15°⋅ sin 77° cos 9° + = + = =7 Also, cos 1° is close to 1 and cos 89° is close to 0.
= 2 sin 45°⋅ sin 30° = 2 ⋅ = =7−7=0 sin θ tan θ 28 7 28 Hence, option (d) is correct.
= 17°30′ 2 2 2
MATHEMATICS Measurements of Angles and Trigonometric Ratios 213 214 CDS Pathfinder

31. (d) We know in the interval θ ∈( 0, π / 2),sec 2 θ is increasing from 1 38. (d) Given, sin x + 3 cos x = 1 x sin φ sin x
48. (c) Given, tan θ = ⇒ x sin φ = tan θ − x cos φ tan θ 56. (b) Given, 3 cos x = 5 sin x ⇒ =
3 3
⇒ tan x = ,
to ∞. 1 3 1 1 − x cos φ cos x 5 5
∴ p≥ 1 On dividing both sides by 2, we get sin x + cos x =
2 2 2 tan θ sin θ sin θ
⇒ x= = = 5 sin x − 2 sec 3 x + 2 cos x 5 tan x − 2 sec 4 x + 2
32. (b) Q cos 90° = 0 ⇒ sin x sin 30° + cos x cos 30° = 1 / 2 =
sin φ + cos φ tan θ cos θ sin φ + cos φ sin θ sin(θ + φ) 5 sin x + 2 sec 3 x − 2 cos x 5 tan x + 2 sec 4 x − 2
∴ cos 1° cos 2° cos 3°… cos 90° = 0 (Q cos 90° = 0) ⇒ cos ( x − 30° ) = cos 60° ⇒ x − 30° = 60°
sin φ x sin θ (dividing numerator and denominator by cos x)
1 − cos B ∴ x = 90° Similarly, y = , Hence, =
33. (c) Given, tan A = sin ( θ + φ) y sin φ 5 tan x − 2( 1 + tan 2 x)2 + 2
sin B 1 + tan 2 A =
( 1 − cos B ) 39. (c) cosec A ⋅ sec A = ⋅ 1 + tan 2 A 5 tan x + 2( 1 + tan 2 x)2 − 2
2⋅ tan A 49. (b) Given, sin ( θ + φ) = 2 sin ( θ − φ) 2
− 2  1 +

2 tan A
= sin B 3 9 1156
1 + tan A 1 + ( 2 − 1)2
2
⇒ sin θ cos φ + cos θ sin φ = 2 (sin θ cos φ − cos θ sin φ) 5 ×  + 2 3+ 2−2×
1 − tan 2 A 1 − cos B 
2
= = (Q tan A =  25 
1 − 
2 – 1) 5 625
 tan A 2−1 3 sin φ sin θ = =
 sin B  ⇒ 3 cos θ sin φ = sin θ cos φ ⇒ = 2 1156
5 × + 2  1 + 
3 9
1 + 2 + 1 − 2 2 4 − 2 2 2 2( 2 − 1) cos φ cos θ  − 2 3 − 2 + 2 × 625
2( 1 − cos B ) sin B 2 ( 1 − cos B ) sin B = = = =2 2 5  25 
= = 2−1 ( 2 − 1) ( 2 − 1)
sin B − 1 − cos 2 B + 2 cos B −2 cos 2 B + 2 cos B
2 ⇒ 3 tan φ = tan θ or tan θ = 3 tan φ 3125 − 2312 813 271
3 sin(θ + α ) = = =
2 sin B( 1 − cos B ) sin B 40. (c) Given, tan θ = ⇒ sinθ = 3 cos θ ⇒ sinθ = 3 1 − sin 2θ 50. (a) Given, sin ( θ + α ) = cos ( θ + α ) ⇒ =1 625 + 2312 2937 979
= = = tan B 1 cos(θ + α )
2 cos B ( 1 − cos B ) cos B
Squaring both sides, we get sin 2 θ = 9( 1 − sin 2θ) tan θ + tan α 1 + cos x ( 1 + cos x) 1 + cos x
⇒ tan ( θ + α ) = 1 ⇒ =1 57. (b) = ×
34. (c) cot 15° cot 20° cot 70° cot 75° 9 3 1 − tan θ tan α 1 – cos x ( 1 − cos x) 1 + cos x
⇒ 10 sin 2 θ = 9 ⇒ sinθ = =−
= tan ( 90° − 15° ) tan ( 90° − 20° ) cot 70° cot 75° 10 10 ⇒ tan θ + tan α = 1 − tan θ tan α 1 + cos x 1 cos x
1 1 (Q θ lies in IIIrd quadrant, so sin θ will be negative) = = + = cosec x + cot x
= tan 75° tan 70° ⋅ =1 1 − tan α
tan 70° tan 75° ⇒ tan θ ( 1 + tan α ) = 1 − tan α ⇒ tan θ = sin x sin x sin x
41. (a) sin 20° cos 70° + cos 20° sin 70° 1 + tan α
58. (a) cos x − sin 4 x = (cos 2 x − sin 2 x) (sin 2 x + cos 2 x)
4

sin θ cos θ = sin( 20°+70° ) = sin 90° = 1 θ− φ θ+ φ


35. (b) Given, + =2 51. (c) cos 2 − sin 2
cos θ sin θ [Q sin A cos B + cos A sin B = sin( A + B )] 2 2 = [(cos 2 x) − ( 1 − cos 2 x)] = 2 cos 2 x − 1 (Q sin 2 x + cos 2 x = 1)
θ − φ θ + φ  θ − φ − θ + φ
∴ sin 2 θ + cos 2 θ = 2 sin θ cos θ 42. (a) φ + θ = 180° ⇒ θ = 180° − φ ⇒ sin θ = sin ( 180° − φ) = cos  +  ⋅ cos   sin θ sin θ 1 − cos θ sin θ( 1 − cos θ) 1 − cos θ
 2 2   2 2  59. (a) = × = =
⇒ sin 2θ = 1 = sin 90° ⇒ 2θ = 90° ⇒ θ = 45° sin θ = sin φ 1 + cos θ 1 + cos θ 1 − cos θ sin 2 θ sin θ
36. (d) Given, in right angled ∆ABC , ∠B = 90° and AB : AC = 1 : 2 43. (b) cos x cosec y − cos x sin y [Q cos A − sin B = cos ( A + B ) cos ( A − B )]
2 2
5 cos θ − 4 3 + 5 sin θ
= cos θ ⋅ cos ( − φ) = cos θ ⋅ cos φ 60. (a) −
Let AB = x, AC = 2x = cos x ⋅ cosec ( 90°− x) − cos x ⋅ sin( 90°− x) 3 − 5 sin θ 4 + 5 cos θ
C ( Q x + y = 90°, given) 52. (c) x + y + z = r 2 sin 2 θ cos 2 φ + r 2 sin 2 θ sin 2 φ + r 2 cos 2 θ
2 2 2
(5 cos θ − 4)( 4 + 5 cos θ) − (3 + 5 sin θ)(3 − 5 sin θ)
=
= r 2 sin 2 θ (cos 2 φ + sin 2 φ) + r 2 cos 2 θ (3 − 5 sin θ)( 4 + 5 cos θ)
= cos x ⋅ sec x − cos 2 x
√2 x = r 2 sin 2 θ + r 2 cos 2 θ = r 2 (sin 2 θ + cos 2 θ) = r 2 ( 25 cos 2 θ − 16) − ( 9 − 25 sin 2 θ)
= 1 − cos x =
2
sin x = sin x
2 =
7 49 625 (3 − 5 sin θ)( 4 + 5 cos θ)
53. (c) Given, cot θ = , cosec 2 θ = 1 + cot 2 θ = 1 + = 25(sin 2 θ + cos 2 θ) − 25 25 − 25
44. (c) 2+ 2 + 2 cos 4 θ = 2+ 2( 1 + cos 4 θ) 24 576 576 = = =0
A B 25 (3 − 5 sin θ)( 4 + 5 cos θ) (3 − 5 sin θ)( 4 + 5 cos θ)
∴ cosecθ = ±
x
⇒ AB 2 + BC 2 = AC 2 (using pythagoras theorem) = 2+ 2( 2 cos 2 2θ) = 2+ 4 cos 2 2θ = 2 + 2 cos 2θ 24 16 9 3
61. (a) sin (α + β ) = 1 − cos 2 ( α + β ) = 1− = =
⇒ x2 + BC 2 = ( 2 x)2 = 2( 1 + cos 2θ) = 2( 2 cos 2 θ) = 4 cos 2 θ = 2 cos θ ⇒ sin θ = ±
24
, cos 2 θ = 1 − sin 2 θ = 1 −
576
=
49 25 25 5
⇒ BC 2 = 2x2 − x2 ⇒ BC = x 5 1 25 625 625
cos ( α − β ) =
25
1 − sin 2 ( α − β ) == 1−
144 12
=
45. (b) Given, tan A = and tan B = 7 3π
BC x 6 11 ∴ cosθ = ± , As π < θ < 169 169 13
∴ tan A == =1
AB x 5 1 55 + 6 25 2 sin(α + β )  3 5  3
+ ∴ tan ( α + β ) = = ×  =
tan A + tan B 61 ∴ sin θ and cosθ both are negative. cos(α + β )  5 4  4

2 tan A
=
2( 1) 2
= =∞ (undefined) Q tan ( A + B ) = = 6 11 = 66 = =1
1 − tan A tan B 1 − 5 ⋅ 1 66 − 5 61 − 24 −7
1 − tan 2 A 1 − 12 0 ∴ sin θ = , cos θ = sin(α − β ) 5 13 5
6 11 66 tan (α − β ) = = × =
a
37. (a) Given, AD = DB = and ∠B = 90° π π
25 25 cos(α − β ) 13 12 12
2 ⇒ tan ( A + B ) = 1 ⇒ tan( A + B ) = tan ⇒ A + B = − 7 24 17
4 4 ∴ cos θ − sin θ = + = ∴ tan ( 2α ) = tan[(α + β ) + (α − β )]
In right angled ∆ ABC , BC 2 + AB 2 = AC 2 1 cos A sin A
25 25 25 3
+
5
46. (c) (cos A − sin A ) = − 1 tan(α + β ) + tan(α − β ) 56
⇒ BC + a = b
2 2 2
⇒ BC = b −a2 2
2 2 2 54. (b) Given, sin α = = sin 30° ⇒ α = 30° = = 4 12 =
2 1 − tan(α + β ) ⋅ tan(α − β ) 1 − 3 × 5 33
π π π
In right angled ∆ BCD, CD = BC + BD = cos cos A − sin sin A = cos  + A 
2 2 2
π 4 12
 4  Also, β = − α = 90° − 30° = 60°
a2 
2 4 4
CD 2 = ( b 2 − a 2 )2 +   =  b 2 − a 2 + tan 45°+ tan x 1 + tan x
a 2
 62. (a) tan ( 45° + x) = =
 2  4 [Q cos A cos B – sin A sin B = cos( A + B )] 1 − tan 45° tan x 1 − tan x
55. (c) Given, sin x − cos x = 0 ⇒ sin x = cos x ⇒ tan x = 1
π π π π π π π π tan 45°− tan x 1 − tan x
4b 2 − 3a 2 4b 2 − 3a 2 ⋅ cos − cos ⋅ sin = sin  − ⇒ tan x = tan ⇒ x = tan ( 45° − x) = =
⇒ CD 2 = ⇒ CD = 47. (d) sin  1 + tan 45° tan x 1 + tan x
4 2 4 12 4 12  4 12  4 4
4 4 1 + tan x
b2 − a2 2 b2 − a2 [Q sin A cos B – cos A sin B = sin ( A − B )] π π  1   1 
∴ cosθ =
BC
= = ∴ sin 4 x + cos 4 x = sin 4 + cos 4 =  +   tan( 45°+ x) 1 − tan x  1 + tan x 
2
π 4  2  2 ∴ = =
= sin   =
1 4
CD 4b 2 − 3a 2 4b 2 − 3a 2 tan( 45°− x) 1 − tan x  1 − tan x 
 6 2 1 1 1
2 = + = 1 + tan x
4 4 2
MATHEMATICS Measurements of Angles and Trigonometric Ratios 215 216 CDS Pathfinder

( 1 − sin A cos A )(sin 2 A − cos 2 A ) 71. (a) Refer to example 16. 1 1 − tan θ tan φ 1 − cos 2 x
63. (a) 77. (b) cot ( θ + φ) = =
cos A (sec A − cosec A )(sin 3 A + cos 3 A ) tan ( θ + φ) tan θ + tan φ n sec x − cos x
p
72. (d) Given, tan θ = , 86. (b) = = cos x2
m cosec x − sin x 1 − sin x
( 1 − sin A cos A )(sin 2 A − cos 2 A ) q 1 tan θ tan φ
= = − sin x
 1 − tan 2 θ   2 tan θ  tan θ + tan φ tan θ + tan φ
 cos A  1 − 1  (sin A + cos A ) p cos 2θ + q sin 2θ = p   + q  sin 2 x sin x
3
= 
   1 + tan 2 θ  sin x 
  cos A sin A    1 + tan 2 θ  = ⋅ 
  =
1

1 cos x cos 2 x  cos x 
(sin A + cos A − sin A cos A )
2 2
  p2   2p  tan θ + tan φ  
  1−     2 2   2   1 + 1 
1/3
(tan x)3 =   ⇒ tan x =  
n n
= p
q2  + q q  = p  q − p  +  2 pq   tan θ tan θ 

m m
[( 1 − sin A cos A )(sin A ⋅ cos A )      q2 + p2   q2 + p2 
p2 p2  
× (sin A + cos A ) (sin A − cos A )] 1+  1+ 2      1
 q2   q  1 1 1 1 87. (a) 2 cos θ = x + …(i)
= = sin A = − = − x
[cos A (sin A − cos A )(sin A + cos A ) p( q 2 − p 2 + 2q 2 ) p(3q 2 − p 2 ) tan θ + tan φ (cot θ + cot φ) a b 3
On cubing both sides, we get 8 cos θ =  x + 
1
= = 3
( 1 − sin A cos A )] p2 + q2 p2 + q2 [Q tan θ + tan φ = a, or cot θ + cot φ = b]  x
1 1 1
64. (c) ( 1 + tan A ) ( 1 + tan B ) 2π 4π − y −z 78. (b) Q tan 75° = tan ( 45° + 30° ) 8 cos θ = x + 3 + 3x ⋅  x + 
3 3
73. (c) Given, x = y cos = z cos ⇒ x= = =k (let) x x
tan 45°+ tan 30° 1 + tan 30°
x
= ( 1 + tan A ) [ 1 + tan ( 45° − A )] 3 3 2 2 ⇒ tan 75° = ⇒ tan 75° = 1
tan 45°− tan A  1 – tan 45° tan 30° 1 − tan 30° 8 cos 3 θ = x3 + 3 + 3 ( 2 cos θ) [from Eq. (i)]
x = k , y = − 2k , z = − 2k
= ( 1 + tan A )  1 + (Q A + B = 45°) x
 1 + tan 45° tan A  ⇒ xy + yz + zx = k ( − 2k ) + ( − 2k ) ( − 2k ) + ( − 2k ) k ⇒ tan 75°− tan 75°⋅ tan 30° = 1 + tan 30°
⇒ 8 cos 3 θ − 6 cos θ = x3 + 3
1
1 − tan A  ⇒ tan 75°− tan 30°− tan 75°⋅ tan 30° = 1
= ( 1 + tan A )  1 + = − 2k 2 + 4k 2 − 2k 2 = 0 x
 1 1
1 + tan A  79. (b) sin 2 ( 15° + A ) − sin 2 ( 15° − A ) = sin ( 15° + A + 15° − A ) ⇒ 2 [ 4 cos 3 θ − 3 cos θ] = x3 + 3 ⇒ 2 [cos 3θ] = x3 + 3
74. (c) Given, sin θ + cos θ = m ...(i) x x
+ A + 1 − tan A 
= ( 1 + tan A ) 
1 tan ⋅ sin ( 15° + A − 15° + A )
1 1 88. (a) We know that, the value of cosθ decreases in the interval
 1 + tan A  and sec θ + cosec θ = n ⇒ + =n
cos θ sin θ [Q sin 2 A − sin 2 B = sin ( A + B ) ⋅ sin ( A − B )] 0≤ θ≤ π / 2
= ( 1 + tan A ) 
2 =2 sin θ + cos θ 1 1 π π
⇒ =n = sin 30° sin ( 2 A ) = Q cos θ ≥ ⇒ cos θ ≥ cos ⇒ θ≤
 1 + tan A  sin 2 A
2 3 3
sin θ cos θ 2
( A + B) ( B − A) m m sin 38° − cos 68° cos( 90° − 38° ) − cos 68° 3 12
65. (a) Q cos A − cos B = 2 sin ⋅ sin ⇒ = n ⇒ sin θ cos θ = [from Eq. (i)] 80. (a) = 89. (b) Given, sin A = and cos B =
2 2 sin θ cos θ n cos 68° + sin 38° cos 68° + sin( 90° − 52° ) 5 13
( A + B) ( A − B) sin A sin A 3/5 3/5 3
and sin A + sin B = 2 sin ⋅ cos 52° + 68°   68° − 52°  ∴ tan A = = = = =
2 2 On squaring Eq. (i), we get sin θ + cos θ + 2 sin θ cos θ = m 2
2 2 2 sin   sin   cos A 1 − sin 2 A 1 − (3 / 5 ) 2 4 /5 4
cos 52° − cos 68°  2   2 
cos 2B − cos 2 A 2 sin( A + B ) sin( A − B ) m 2m = = 144
∴ = = tan ( A − B ) 1 + 2 = m2 ⇒ = m2 − 1 [Q sin 2 θ + cos 2 θ = 1] cos 68° + cos 52° 2 cos  52° + 68°  cos  68° − 52°  1−
sin 2B + sin 2 A 2 sin( A + B ) cos( A − B )     sin B 1 − cos 2 B
169 = 5 / 13 = 5
n n  2   2  and tan B = = =
⇒ 2m = ( m − 1) n
2 cos B cos B12 12 / 13 12
66. (d) sec x + tan x = 7
2 2
(Q sec x = 1 + tan x)
2 2
sin 60°⋅ sin 8°
= = tan 60°⋅ tan 8° = 3 tan 8° 13
1 + tan 2 x + tan 2 x = 7 ⇒ 1 + 2 tan 2 x = 7 75. (b) Given, cosecθ − sin θ = a 3 ⇒
1
– sin θ = a 3 cos 60°⋅ cos 8° 3 5 9 −5

⇒ 2 tan x = 6 ⇒ tan x = 3 = tan 60°
2 sin θ 81. (c) 2 cos x − (cos 3x + cos 5x) tan A − tan B
= 4 12 = 12
∴ x = 60° 1 − sin 2 θ cos 2 θ 1 + tan A tan B 1 + 3 ⋅ 5 48 + 15
⇒ = a3 ⇒ = a 3 ⇒ cos 2 θ = a 3 sin θ …(i) = 2 cos x − 2 cos 4x ⋅ cos x = 2 cos x( 1 − cos 4x)
4 × 12
67. (a) cos 3θ + sin 3θ = 2  sin 3θ sin θ sin θ
1 1 4 12
cos 3θ + = 2 cos x ⋅ 2 sin 2 2x = 4 cos x( 2 sin x cos x)2 = 16 cos 3 x sin 2 x 1
 2 2 
1 1 − cos θ
2
82. (c) cos 4x = 1 − 2 sin 2 2x = 1 − 2 ( 2 sin x cos x)2 = 3 = ×
1 48 16
=
= 2 (sin 45° cos 3θ + cos 45° sin 3θ) = 2 [sin ( 45° + 3θ)] and sec θ − cos θ = b 3 ⇒ – cos θ = b 3 ⇒ = b3
cos θ cos θ 63 3 63 63
cos3θ + sin 3θ is maximum when sin( 45°+3θ) is maximum = 1 − 2( 4 sin 2 x cos 2 x) = 1 − 8 sin 2 x cos 2 x
sin 2 θ 48
The maximum value of sin ( 45° + 3θ) = 1 = sin 90° ⇒ = b 3 ⇒ sin 2 θ = b 3 cos θ …(ii) 83. (a) 2 sin A cos 3 A − 2 sin 3 A cos A
⇒ 45° + 3θ = 90° ⇒ θ = 15° cos θ 90. (a) Given, B + C = 60° ⇒ B = 60° − C
= 2 sin A cos A (cos 2 A − sin 2 A )
sin 2 θ sin ( 120° − B ) = sin [ 120° − ( 60° − C )] = sin ( 60° + C )
68. (c) sin 2 x + sin 2 y + sin 2 z = (sin x + sin y + sin z )2 On putting cos θ = in Eq. (i), we get 2 sin 2 A cos 2 A 1
b3 = sin 2 A (cos 2 A ) = = sin 4 A = sin [ 180° − ( 60° + C )] = sin ( 120° − C )
⇒ sin x sin y + sin y sin z + sin z sin x = 0 2 2
sin θ
4
x ⋅ cosec 2 30° sec 2 45°
[Q ( a + b + c )2 = a 2 + b 2 + c 2 + 2 ( ab + bc + ca )] = a sin θ ⇒ sin 3 θ = a 3 b 6
3
84. (c) Given, a cos θ − b sin θ = c 91. (a) Given, = tan 2 60° − tan 2 30°
b6 8 cos 2 45° sin 2 60°
On dividing both sides by sin x sin y sin z On squaring both sides, we get
∴ sin θ = ab 2 …(iii) x × ( 2)2 × ( 2 )2
2
= ( 3 )2 −  
1 1 1 1
⇒ + + =0 a 2 cos 2 θ + b 2 sin 2 θ − 2ab cos θ sin θ = c 2 ⇒
Similarly, cos θ = a 2 b …(iv) 2
 3
2  3
8 × 
sin x sin y sin z
⇒ a ( 1 − sin θ) + b ( 1 − cos θ) − 2ab sin θ cos θ = c
2 2 2 2 2 1 
 × 
On squaring and adding Eqs. (iii) and (iv), we get  2  2 
69. (d) Given, sin x + sin 2 x = 1 …(i) ⇒ a 2 + b 2 − c 2 = a 2 sin 2 θ + b 2 cos 2 θ + 2ab sin θ cos θ
1 = a2b4 + a4 b2 ⇒ 1 = a2b2 ( a2 + b2 ) x×4×2 1 8x 8
⇒ ( a sin θ + b cos θ)2 = a 2 + b 2 − c 2 ⇒ =3− ⇒ = ⇒x= 1
⇒ sin x = 1 − sin 2 x = cos 2 x (Q sin 2 x + cos 2 x = 1) 1 3 3 3 3
8× ×
∴cos 2 x + cos 4 x = (sin x) + sin 2 x = 1 [from Eq. (i)] 76. (d) As, we know that cos θ ≥ − 1 and cos θ ≤ 1 ⇒ a sin θ + b cos θ = ± a 2 + b 2 − c2 2 4
∴ cos 2x ≥ − 1 ⇒ 1 + cos 2x ≥ 0 sin x 1 − cos x
70. (a) (cos 35° + cos 85° ) + cos 155° 85. (d) Given, cos x + cos 2 x = 1 ⇒ cos x = 1 − cos 2 x = sin 2 x …(i) 92. (c) =
cos 2x ≤ 1 ⇒ 1 + cos 2x ≤ 2 1 + cos x sin x
= 2 cos 60° cos 25° + cos ( 180° − 25° ) sin 8 x + 2 sin 6 x + sin 4 x = (cos x)4 + 2 cos 3 x + cos 2 x
1 So, minimum value of 1 + cos 2x is 0 and maximum value of The above identity is possible for all values of x except multiples of
=2× cos 25° − cos 25° = cos 25° − cos 25° = 0 1 + cos 2x is 2. = (cos 2 x + cos x)2 [from Eq. (i)] 180°. Since, for x = 180°, sin x = 0 and 1 + cos x = 0.
2
= ( 1)2 = 1
MATHEMATICS Measurements of Angles and Trigonometric Ratios 217 218 CDS Pathfinder

1 cos( x − 2 y ) π cos π A + B  180°−C 


93. (b) Given, cos x = k cos ( x − 2 y ) ⇒ = 99. (c) Given, x = a sec θ cos φ, y = b sec θ sin φ and z = c tanθ, 106. (a)Q In 24 h, Earth rotate about its own RHS cos A + cos B = cos + II. sin   = sin  
k cos x x 2 y 2 z2 a 2 sec 2 θ cos 2 φ b 2 sec 2 θ sin 2 φ c 2 tan 2 θ axis = 360° 6 3  2   2 
+ − = + − 3 1 3+1
a 2 b 2 c2 a2 b2 c2 = + = = sin  90° −  = cos
Apply componendo and dividendo theorem In 1 h Earth rotate about its own axis C C
x – 2y + x x – x + 2y = sec 2 θ [cos 2 φ + sin 2 φ] − tan 2 θ = sec 2 θ − tan 2 θ = 1 2 2 2  2 2
2 sin sin 360° ⇒ sin A + sin B = cos A + cos B
1 − k cos( x − 2 y ) − cos x = = 15° A + B −C 
= = 2 2 100. (a) The squares of the tangents of the angles 30°, 45° and 60° are in 24 π π III. tan  
1 + k cos( x − 2 y ) + cos x x – 2y + x x – 2y – x G.P. II. LHS tan A + tan B = tan + tan  2 
2 cos cos In 4 h Earth rotate about its own axis 6 3
2
( 180°−C ) − C
⇒ tan 2 30°, tan 2 45°, tan 2 60° are in G.P. ⇒   , ( 1)2 , ( 3 )2
2 2 1 = tan
2 sin( x − y ) sin y = 15° × 4 = 60° 1 4
 3 = + 3= 2
= = tan( x − y ) tan y Since, in 60 min Earth rotate about its 3 3
2 cos( x − y ) cos y ( 180°−2C )
1
are in G.P. ⇒ , 1, 3 are in G.P. own axis = 15° π π = tan
RHS cot A + cot B = cot + cot 2
sin( x + y ) − 2 sin x + sin( x − y ) 3
In 12 min Earth rotate about its own 6 3
94. (b)
cos( x + y ) − 2 cos x + cos( x − y ) 1 1 4 = tan ( 90° − C ) = cot C
which is true as 12 = × 3 ⇒ 1 = 1 15°×12 = 3+ =
3 axis = = 3° (A − B −C ) A − (B + C )
sin( x + y ) + sin( x – y ) – 2 sin x 60° 3 3 IV. tan = tan
= 101. (d) We know that, in a cyclic quadrilateral sum of opposite angle is ⇒ tan A + tan B = cot A + cot B 2 2
cos( x + y ) + cos( x – y ) – 2 cos x ∴ In 4 h 12 min Earth rotate about its A − ( 180°− A )
180°. own axis Hence, both statements are true. = tan
x+ y + x− y x+ y −x+ y 2
2 sin ⋅ cos − 2 sin x ∴ A + C = 180° …(i)
2 2 = 60° + 3° = 63° 111. (a) Q A + B + C + D = 360°
= = tan [ − ( 90° − A )]
(x + y + x − y) (x + y − x + y) and B + D = 180° …(ii)
2 cos ⋅ cos − 2 cos x 107. (a) In right angled ∆OAB, ∴ A + B = 360° − (C + D )
∴ cos A + cos B + cos C + cos D = − tan ( 90° − A ) = − cot A
2 2 ∴ sin ( A + B ) = sin [360° − (C + D )]
2 sin x cos y − 2 sin x 2 sin x (cos y − 1) sin x = cos A + cos B + cos ( 180° − A ) + cos ( 180° − B ) Hence, all options are correct.
= = = = tan x r = − sin (C + D )
2 cos x cos y − 2 cos x 2 cos x (cos y − 1) cos x = cos A + cos B − cos A − cos B = 0 A B 115. (b) Given, sin( A + B ) = 1
40° ∴ sin ( A + B ) + sin (C + D ) = 0
x+ y x− y [∴ cos ( 180°− θ) = − cos θ] R and sin( A − B ) = 1 / 2
95. (c) Given, sin x + sin y = a ⇒ 2 sin cos =a …(i) Also, cos( A + B ) = cos[360° − (C + D )] π
2 2 102. (a) We know that, for 0° < θ < 90°, there exist only one θ such that
O ∴ A+B = …(i)
cos ( A + B ) = cos (C + D ) 2
x+ y (x − y) sin θ = a. π
and cos x + cos y = b ⇒ 2 cos ⋅ cos = b …(ii) Hence, only statement I is correct. ∴ A−B = …(ii)
2 2 103. (b)Q sin ( B + C − A ) = 1 = sin 90° ⇒ B + C − A = 90° …(i) 6
π 3.14
On dividing Eq. (i) from Eq. (ii), we get Q cos (C + A − B ) = 1 = cos 0° 112. (d) I. 1° = radian = = 0.017 On adding Eqs. (i) and (ii), we get
x+ y x− y AB r 180 180 2π π π
2 sin cos ∴ C + A − B = 0° …(ii) cos 40° = ⇒ cos 40° = 2A = ⇒ A = and B =
2 2 a x+ y a OB R = 0.02 radians (approx) 3 3 6
∴ = ⇒ tan = and tan ( A + B − C ) = 1 = tan 45°
x+ y (x − y) b 2 b ⇒ r = R cos 40° which is less than 0.03 radians.
2 cos cos ∴ A + B − C = 45° …(iii) 116. (c) Now, tan ( A + 2B ) ⋅ tan( 2 A + B )
2 2 180 π π 2π π
II. 1 radian = = tan  +  ⋅ tan 
So, the radius of the circle of latitude degree
On adding Eqs. (i), (ii) and (iii), we get π + 
96. (a) Here, r = length of largest hand of clock 40°S is R cos 40°. 3 3  3 6
A + B + C = 135° …(iv) 180
108. (b) Given, sin 3θ = cos ( θ − 2° ) = = 57.32 degree 2π 5π
l = distance covered by largest hand of clock
On subtracting Eqs. (i), (ii), (iii) from Eq. (iv), we get 3.14 = tan   ⋅ tan  
Angle traced in 60 min = 360° ⇒ sin 3θ = sin [ 90° − ( θ − 2° )]  3   6 
2 A = 45°, 2B = 135°, 2C = 90° which is greater than 45°.
π π π π
Angle traced in 20 min =
360°
× 20 = 120° ⇒ θ =
2π 1° 1° ⇒ 3θ = 90° − θ + 2° Hence, both statements are true. = tan  +  ⋅ tan  + 
A = 22 , B = 67 , C = 45°  2 6  2 3
60 3 92°
2 2 ⇒ 4θ = 92° ⇒ θ = = 23° π π −1
l
As, θ = ⇒ l = r θ, ⇒ l = 42 ×
2 π 42 × 2 × 22
= = 88 cm 4
113. (d) I. sin θ ⋅ sin( 60°+ θ) ⋅ sin( 60°− θ) =  − cot   − cot  = ( − 3 )  = 1
r 3 3×7 104. (c) cos ( 180° + A ) + cos ( 180° + B ) = sinθ[sin 2 60°− sin 2θ]  6   3   3
+ cos ( 180° + C ) + cos ( 180° + D ) 109. (c) I. RHS = cos 2 θ( 1 + tan θ)( 1 − tan θ)
= sin θ  − sin 2 θ
3 117. (b) Now, sin 2 A − sin 2 B
97. (d) Given, 2 sin x + 15 cos 2 x = 7 ⇒ 2 sin x + 15 − 15 sin 2 x = 7 = cos θ ( 1 − tan θ)
2 2
= − cos A − cos B − cos C − cos D  4  = sin 2 ( π / 3) − sin 2 ( π / 6)
⇒ 15 sin 2 x − 2 sin x − 8 = 0  cos 2 θ − sin 2 θ  1 1 2
= − cos A − cos B − cos ( 180° − A ) − cos ( 180° − B ) = cos 2 θ   = [3 sin θ − 4sin 3θ] = sin 3θ  3 2
=   −   = − = =
1 3 1 2 1
⇒ 15 sin 2 x − 12 sin x + 10 sin x − 8 = 0  cos 2 θ  4 4
 2
(Q A + C = B + D = 180°)  2  4 4 4 2
⇒ 3 sin x ( 5 sin x − 4) + 2 ( 5 sin x − 4) = 0 cos 2 θ − sin 2 θ II. cosθ ⋅ sin (30°+θ) ⋅ sin(30°− θ)
= − cos A − cos B + cos A + cos B = 0 = = LHS
⇒ ( 3 sin x + 2) ( 5 sin x − 4) = 0 cos 2 θ + sin 2 θ = cosθ[sin 2 30°− sin 2θ] 118. (c) Since, tanA, tanB are the roots of
105. (d) In ∆ABC , equation 3x 2 − 2 3x + 1 = 0
⇒ sin x =
4 Q sin x ≠ − 2 , 0° < x < 90°  1 + sin θ ( 1 + sin θ)2 = cos θ  − ( 1 − cos 2 θ)
1
  BC u v II. LHS = =  4  2 3
5  3  tan A = = and tan B = 1 − sin θ 1 − sin 2 θ ∴ tan A + tan B =
2 2 AC v u 2 1 1 3
1 + sin θ 
1 −   3 =  = [ 4 cos 3 θ − 3cosθ] = cos 3θ
4 3
∴ cos x = 1 − sin 2 x = =   = A  4 4 and tan A ⋅ tan B = 1 / 3
5 5 5  cos θ 
tan A + tan B
sin x 4 / 5 4 = (sec θ + tan θ )2 III. sin θ ⋅ cos(30°+ θ) ⋅ cos(30°− θ) ∴ tan( A + B ) =
∴ tan x = = = 1 − tan A ⋅ tan B
cos x 3 / 5 3 v w Hence, both statements are correct. = sin θ [cos 2 30°− sin 2 θ]
2 3
π π
= sin θ  − sin 2 θ = sin 3θ
98. (a) In ∆ABC , A + B + C = 180° 3 1 π
110. (c) Given, A = and B = = 3 = 3 = tan
6 3  4  4
∴ A + B = 180° − C ⇒ tan ( A + B ) = tan ( 180° − C ) C u B π π 1− 1/ 3 3
tan A + tan B I. LHS sin A + sin B = sin + sin Hence, all three statements are correct.
⇒ = − tan C 6 3 ∴ A + B = π /3
Also, u 2 + v 2 = w 2 (by pythagoras theorem) …(i)
1 − tan A tan B 1 3 1+ 3 114. (d) In right angled ∆ABC , Now, A + B + C = π
⇒ tan A + tan B = − tan C + tan A tan B tan C u v u 2 + v2 w2 = + = π 2π
∴ tan A + tan B = + = = [from Eq. (i)] 2 2 2 A + B + C = 180° ⇒ C = π − ( A + B) = π − =
⇒ tan A + tan B + tan C = tan A tan B tan C v u uv uv I. sin( A + B ) = sin ( 180° − C ) = sin C 3 3
MATHEMATICS Measurements of Angles and Trigonometric Ratios 219 220 CDS Pathfinder

−1 / 2
119. (c) Now, sin C + cos C 1  sin 90°  128. (b) Given that, sin θ ⋅ cos θ =
3
…(i) 135. (c) ( 1 − tan A )2 + ( 1 + tan A )2 + ( 1 − cot A )2 + ( 1 + cot A )2 140. (b) Now, cosec (C − D + B ) = cosec( 120° − 150° + 60° )
=  
= sin( π − ( A + B )) + cos( π − ( A + B )) (sin α sin β )1 / 2  cos( 90°−α ) sin β  4 = 2 [ 1 + tan 2 A ] + 2[ 1 + cot 2 A ] = cosec (180° − 150° ) = cosec 30° = 2
[as A + B + C = π, C = π − ( A + B )] ∴ sin 4 θ + cos 4 θ
[from Eq. (i)] [Q ( a − b ) + ( a + b ) = 2( a + b )]
2 2 2 2
141. (d) If ABCD is a cyclic quadrilateral, then sum of opposite angles
= sin( A + B ) − cos( A + B ) 1 = (sin θ + cos θ) − 2 sin θ .cos 2 θ
2 2 2 2
= 2 sec 2 A + 2 cosec 2 A = 2 
= sin( π / 3) − cos( π / 3) = × (sin α sin β )1 / 2 = 1 1 1  should be 180° but here
+ 
(sin α sin β )1 / 2 = ( 1)2 − 2 (sin θ ⋅ cos θ )2  cos 2 A sin 2 A  ∠A + ∠C = 30° + 120° = 150° ≠ 180°
3 1 3−1
= − = cot 30°+1 2 and ∠B + ∠D = 60° + 150° = 210° ≠ 180°
2 2 2 125. (c) I. = 2 (cos 30° + 1)  3 3 3 5  sin 2 A + cos 2 A  2 ⋅ ( 1)
cot 30°−1 = 1− 2   = 1− 2. = 1− = =2  = = 2 sec 2 A ⋅ cosec 2 A So, statement I is incorrect.
1 + tan 2 C  4  16 8 8  sin 2 A ⋅ cos 2 A  sin 2 A ⋅ cos 2 A
120. (b) 2 sin B = 3+1  3  Now, sin ( B − A ) = cos (D − C )
cosec 2C ⇒ =2 + 1 1 + 2 sin θ ⋅ cos θ
3−1  2  129. (d) Given that, θ lies is in first quadrant 136. (d) Given, a =
2
⇒ sin ( 60° − 30° ) = cos ( 150° − 120° )
1 + tan 2 ( 2 π / 3) and tanθ=3 1 − 2 sin θ ⋅ cos θ
= 3+1 3+1  3 + 2 1 3
cosec 2 ( 2 π / 3) ⇒ × =2  ∴ tan 2 θ = 9 (sin 2 θ + cos 2 θ) + 2 sin θ ⋅ cos θ ⇒ sin 30° = cos 30° ⇒ ≠
3−1 3+1 ⇒ a2 = 2 2
 2  (sin 2 θ + cos 2 θ) − 2 sin θ ⋅ cos θ
1 + tan 2 ( π − π / 3) On adding both sides by 1, we get So, statement II is also incorrect.
= 3 + 1+ 2 3
cosec 2 ( π − π / 3) ⇒ = 3+ 2 ⇒ 1 + tan 2 θ = 10 (sin θ + cos θ)2 a sin θ + cos θ
3−1 ⇒ a2 = ⇒ = 142. (a) Given, sin θ + cos θ = 3
1 + tan 2 π / 3 ⇒ sec 2 θ= 10 ⇒ secθ= 10 (sin θ − cos θ)2 1 sin θ − cos θ On squaring both sides, we get (sin θ + cos θ)2 = ( 3 )2
2 sin B = 4+ 2 3
cosec 2 π / 3 ⇒ = 3+ 2 a + 1 (sin θ + cos θ) + (sin θ − cos θ)
2 (since, θ lies in first quadrant) ⇒ = ⇒ sin 2 θ + cos 2 θ + 2 sin θ cos θ = 3 ⇒ 1 + 2 sin θ cos θ = 3
1+ 3 4 a − 1 (sin θ + cos θ) − (sin θ − cos θ) 3−1
= = ×3 = 3 2 (2 + 3) 1 ⇒ sin θ cos θ = =1 ...(i)
4/3 4 ⇒ = 3+ 2 ⇒ cosθ = ...(i) 2
2 10 (applying componendo dividendo formula)
3 π π sin θ cos θ sin 2 θ + cos 2 θ 1
sin B = = sin , B = ⇒ 3+ 2= 3+ 2 sin θ = 1 − cos 2 θ = 1 −
2 1
=
9 a + 1 2 sin θ Now, tan θ + cot θ = + = =
2 3 3 Q ⇒ = = tan θ cos θ sin θ sin θ cos θ sin θ cos θ
10 10 a − 1 2 cos θ
So, it is true. 3
121. (c) Given that, sin 2 x + cos 2 x − 1 = 0 ⇒ sinθ = ...(ii) 1
II. 2 sin 45° cos 45° − tan 45° cot 45° = 0 x2 − y 2  x2 − y 2 
2
From Eq. (i), tan θ + cot θ = =1
⇒ sin 2 x + cos 2 x = 1 10 137. (b) Given, sin θ = ⇒ cos 2 θ = 1 − sin 2 θ = 1 −  2  1
3 1 4 x2 + y 2  x + y2 
⇒ 2 × 
which is an identity of trigonometric 1 1  Now, sin θ + cos θ = + =
×  – 1× 1= 0 143. (a) cos ec ( 75° + θ ) − sec ( 15° − θ ) = cos ec ( 75° + θ )
ratio and always true for every real value  2 2 10 10 10
of x. ( x2 + y 2 )2 − ( x2 − y 2 )2 4x 2 y 2  2xy 
2
− sec [ 90° − ( 75° + θ )]
or
1
2 × – 1× 1= 0 130. (d) If 0°< θ < 90°, then all the = = = 
So, the equation has infinite solutions. ( x2 + y 2 )2 ( x2 + y 2 )2  x2 + y 2  = cos ec ( 75° + θ ) − cos ec ( 75° + θ ) = 0
2 trigonometric ratios can be obtained
122. (c) Given that, 3 sin x + 5 cos x = 5
⇒ 1 − 1 = 0, It is true. when any one of the six ratios is given.
∴ cos θ = 2x y / x 2 + y 2 144. (c) Given, 5 sin θ + 12 cos θ = 13
On squaring both sides, we get
Hence, both statements I and II are 131. (a) sin A .cos A . tan A + cos A . sin A . cot A On squaring both sides, we get
9 sin 2 x + 25 cos 2 x + 30 sin x cos x = 25 true. 138. (b) Let f (θ) = sin θ + cos θ
sin A cos A 25 sin 2 θ + 144 cos 2 θ + 120 sin θ cos θ = 169
= sin A . cos A . + cos A . sin A . Maximum and minimum value of a cos θ + b sin θ is
⇒ 9( 1− cos 2 x) + 25( 1− sin 2 x) 126. (c) I. We know that, sin 2 θ + cos 2 θ = 1 cos A sin A ⇒ 25( 1 − cos 2 θ) + 144( 1 − sin 2 θ) + 120 sin θ cos θ = 169
+ 30 sin x cos x = 25 ∴ sin 2 1° + cos 2 1° = 1 = sin 2 A + cos 2 A − a + b ≤ a cosθ + b sin θ ≤
2 2
a +b
2 2

⇒ 9 + 25 − { 9 cos 2 x + 25 sin 2 x ⇒ 25 − 25 cos 2 θ + 144 − 144 sin 2 θ +120 sin θ cos θ = 169
II. We have sin θ 1 + cos θ
− 30 sin x cos x} = 25 132. (a) + ∴ − 1 + 1 ≤ cos θ + sin θ ≤ 1+ 1 ⇒ 25 cos 2 θ + 144 sin 2 θ − 120 sin θ cos θ = 169 − 169
sec 2 33°− cot 2 57° 1 + cos θ sin θ
⇒ 9 = (3 cos x − 5 sin x)2 ⇒ − 2 ≤ cos θ + sin θ ≤
= cosec 2 37°− tan 2 53° 2 ⇒ ( 5 cos θ − 12 sin θ)2 = 0 ⇒ 5 cos θ − 12 sin θ = 0
⇒ 3 cos x − 5 sin x = 3 sin θ + ( 1 + cos θ)2
2
= for 0 ≤ θ ≤ 90°, 1 ≤ cosθ + sin θ ≤
LHS = sec 2 33°− cot 2 57° sin θ ( 1 + cos θ) 2 145. (c) Given, sin θ − cos θ = 0
123. (c) Given, p = a sin x + b cos x and
q = a cos x − b sin x = sec 2 ( 90°−57° ) − cot 2 57° Hence, statement I is false. ∴ sin θ = cos θ
= cosec 2 57°− cot 2 57° = 1 sin 2 θ + 1 + cos 2 θ + 2 cos θ Since, sin θ and cosθ are equal for θ = 45°.
⇒ p = a sin x + b cos x
2 2 2 2 2 = And let g (θ) = tan θ + cot θ = tan θ +
1
(Q AM ≥ GM)
RHS = cosec 2 37°− tan 2 53° sin θ ( 1 + cos θ) tan θ ∴ sin 4 θ + cos 4 θ = (sin 45° )4 + (cos 45° )4
+ 2ab sin x cos x
and q 2 = a 2 cos 2 x + b 2 sin 2 x = cosec 2 37°− tan 2 ( 90°−53° ) sin θ + cos θ + 1 + 2 cos θ
2 2
1 4 4
= 1 1 1+ 1 2 1
= 
tan θ + 1 1   1 
− 2 ab sin x cos x = cosec 2 37°− cot 2 37° = 1 sin θ ( 1 + cos θ) tan θ ≥  tan θ ⋅ 1  2  +   = + = = =
⇒  2  2 4 4 4 4 2
∴ p 2 + q 2 = a 2 (sin 2 x + cos 2 x) + b 2 LHS = RHS 1 + 1 + 2 cos θ 2  tan θ 
(cos 2 x + sin 2 x) = cos 2 ( 45°+ θ) + cos 2 ( 45°− θ)
Hence, both statements are correct. sin θ ( 1 + cos θ) ⇒ (tan θ + cot θ) ≥ 2 146. (c)
= a2 + b2 tan( 60°+ θ) tan(30°− θ)
127. (b) I. Given that, sin x + cos x = 2 2 ( 1 + cos θ) So, (tan θ + cot θ) is always greater than 1.
124. (b) Given, α + β = 90° ...(i) = = 2 cosecθ cos 2 ( 45°+ θ) + cos 2 [ 90°− ( 45°+ θ)]
⇒ (sin x + cos x)2 = 4 sin θ ( 1 + cos θ) Hence, statement II is true. =
 sin α cos α 
− 1/2 tan( 60°+ θ) ⋅ tan[ 90°− ( 60°+ θ]
∴ ( cosec α ⋅ cosec β )  +  ⇒ (sin 2 x + cos 2 x) + 2 sin x cos x = 4
 sin β cos β  133. (c) sec 2 D – tan 2 D = 1 Solutions (Q. Nos. 139-141) Let the angle of A, B, C and D of a cos 2 ( 45°+ θ) + sin 2 ( 45°+ θ) 1
⇒ 1 + sin 2x = 4 ⇒ sin 2x = 3 quadrilateral be x, 2x, 4x and 5x. = = =1
=
1 Since, it is a trigonometric identity. tan( 60°+ θ) ⋅ cot( 60°+ θ) 1
As maximum value of sinθ is 1. Q x + 2x + 4x + 5x = 360°
(sin α sin β )1/2
134. (b) Given, cos A + cos 2 A = 1
− 1/2 Therefore, no value of x satisfy above (Q sum of all interior in a quadrilateral is 360°) 147. (b) Given, tanθ + sec θ = m ⇒ tanθ = m − sec θ
 sin α cos β + cos α sin β  ⇒ cos A = 1 − cos 2 A = sin 2 A ...(i)
  equation. Thus, statement I is false. ⇒ 12x = 360° , x = 30° On squaring both sides, we get
 sin β cos β 
II. sin x − cos x = 0 (Q sin θ + cos θ = 1)
2 2
⇒ tan 2 θ = m 2 + sec 2 θ − 2m sec θ
− 1/2 ∴ ∠A = x = 30° , ∠B = 2x = 60°
 sin(α + β )  ⇒ tan x = 1 = tan π / 4 Now, 2(sin 2 A + sin 4 A )
=
1
∠C = 4x = 120° , ∠D = 5x = 150° ⇒ tan 2 θ = m 2 + 1 + tan 2 θ − 2m sec θ
  It x lies in first quadrant, then
(sin α sin β )1 / 2  sin β cos β  tan x = tan π / 4 ⇒ x = π / 4 = 2 (sin 2 A + cos 2 A ) [from Eq. (i)] ⇒ 0 = m 2 + 1 − 2m sec θ ⇒ sec θ =
m2 + 1
139. (a) Now, cos ( A + B ) = cos (30° + 60° ) = cos 90° = 0
Thus, statement II is true. = 2 ⋅ ( 1) = 2 2m
MATHEMATICS Measurements of Angles and Trigonometric Ratios 221 222 CDS Pathfinder

148. (c) Q sinα =


3
⇒ sin α = sin 60° 156. (b) cosec( 75° + θ ) − sec ( 15° − θ ) − tan( 55° + θ ) + cot( 35° − θ ) 165. (c) Given, tan θ + cot θ = 2 = (sin 2 x − cos 2 x ) [(sin 4 x + cos 4 x + 2 sin 2 x cos 2 x )
2 = cosec ( 75° + θ ) − sec [ 90° − ( 75° + θ )] sin θ cos θ sin 2 θ + cos 2 θ − sin 2 x cos 2 x]
 3 ⇒ + =2 ⇒ =2
⇒ α = 60° Q sin 60° =  − tan(55° + θ ) + cot[ 90° − ( 55° + θ )] cos θ sin θ sin θ cos θ = (sin 2 x − cos 2 x ) (sin 4 x + cos 4 x + sin 2 x cos 2 x )
 2  1 1
= cosec ( 75° + θ ) − cosec ( 75° + θ ) ⇒ =2 ⇒ sin θ cos θ = ...(i) = (sin 2 x )3 − (cos 2 x )3 [Q ( a − b )( a 2 + b 2 + ab ) = a 3 − b 3 ]
3  3 sin θ cos θ 2
Now, cosβ = ⇒ β = 30° Q cos30° =  − tan( 55° + θ ) + tan( 55° + θ ) = 0 = sin 6 x − cos 6 x
2  2  Now, (sin θ + cos θ) = sin θ + cos θ + 2 sin θ cos θ
2 2 2
157. (d) sin 25° sin 35° sec 65° sec 55° 1 171. (d) (sin x ⋅ cos y + cos x ⋅ sin y ) (sin x ⋅ cos y − cos x ⋅ sin y )
and tan γ = 1 ⇒ γ = 45° (Q tan 45° = 1) 1 1 = 1+ 2 × = 1+ 1 [from Eq. (i)]
= sin 25°⋅ sin 35°⋅ ⋅ 2
∴ α + β + γ = 60° + 30° + 45° = 135° cos 65° cos 55° = sin ( x + y ) ⋅ sin ( x − y ) = sin 2 x − sin 2 y
⇒ (sin θ + cos θ ) = 2 ⇒ sin θ + cos θ = 2
2
 1 + 1 − sin θ  cos θ = sin 25°⋅ sin 35° ⋅
1

1 [Q sin 2 A − sin 2 B = sin( A + B ) sin( A − B )]
  cos( 90°−25° ) cos( 90°−35° )
( 1 + sec θ − tan θ) cos θ  cos θ cos θ  166. (a)
sec x
=
1 / cos x
149. (a) = 1 1 172. (d) In ∆ABC , A + B + C = 180° ⇒ A + B = 180° –C
( 1 + sec θ + tan θ) ( 1 − sin θ)  1 + 1 + sin θ  ( 1 − sin θ) = sin 25° ⋅ sin 35° ⋅ ⋅ =1 cot x + tan x cos x + sin x
  sin 25° sin 35° tan( A + B ) = tan( 180° –C ) = − tan C
 cos θ cos θ  sin x cos x
 cos θ + 1 − sin θ  cos θ 3 1 / cos x 1 sin x cos x tan A + tan B
  158. (c) Given, 2 cot θ = 3 ⇒ cot θ = = = × = sin x ⇒ = − tan C
 cos θ  2 cos 2 x + sin 2 x cos x 1 1 − tan A tan B
= 3
(cos θ + 1 + sin θ)( 1 − sin θ) 2× −1 sin x cos x tan A + tan B + tan C = tan A tan B tan C
2 cos θ − sin θ 2 cot θ − 1 2 3−1 2 1
cos θ ∴ = = = = = 167. (b) ( 1 + cot x − cosec x )( 1 + tan x + sec x ) On dividing both sides by tan A tan B tan C
cos θ + 1 − sin θ 2 cos θ + sin θ 2 cot θ + 1 2 × + 1 3 + 1 4 2
3 1 1 1
+ + =1
= = ( 1 + cot x − cosec x ) 1 + + sec x
2 1
cos θ + 1 + sin θ − sin θ cos θ − sin θ − sin 2 θ  cot x  tan B tan C tan A tan C tan A tan B
cos θ 159. (d) sin 6 θ + cos 6 θ = (sin 2 θ )3 + (cos 2 θ )3 ⇒ cot B cot C + cot A cot C + cot A cot B = 1
( 1 + cot x − cosec x)( 1 + cot x + cosec x)
cos θ + 1 − sin θ = (sin 2 θ + cos 2 θ )(sin 4 θ + cos 4 θ − sin 2 θ cos 2 θ ) =
= (Q 1 − sin 2 θ = cos 2 θ) cot x 1
cos θ + 1 − sin 2 θ − sin θ cos θ [Q a 3 + b 3 = ( a + b )( a 2 + b 2 − ab )] 173. (a) sin x + cosecx = 2 ⇒ sin x + = 2 ⇒ sin 2 x + 1 = 2 sin x
( 1 + cot x)2 − ( cosec x)2 sin x
cos θ = (sin 2 θ + cos 2 θ )2 − 2 sin 2 θ cos 2 θ − sin 2 θ cos 2 θ =
cos θ + 1 − sin θ cos θ + 1 − sin θ cot x ⇒ sin x − 2 sin x + 1 = 0 ⇒ (sin x − 1)2 = 0 ⇒ sin x = 1
2
= = =1 1
= ( 1 − 3 sin 2 θ cos 2 θ ) = 1 − 3 × = 1 − =
3 1
12 + cot 2 x + 2 cot x − cosec 2 x 1
cos θ + cos 2 θ − sin θ cos θ cos θ (cos θ + 1 − sin θ) 4 4 4 = cosec x = =1
cos θ cot x sin x
cos θ
160. (a) Given, cos x + sec x = 2 ...(i) 1 + 2 cot x − ( cosec 2 x − cot 2 x) 1 + 2 cot x − 1 ∴ sin x + cosec 9 x = ( 1)9 + ( 1)9 = 1 + 1 = 2
9
150. (c) In right angled ∆ABC , if ∠C is 90°, then 1 = = =2
⇒ cos x + = 2 ⇒ cos 2 x + 1 = 2 cos x cot x cot x 174. (b) Given, sin x + cos x = p ...(i)
∠ A + ∠B = 180° − 90° = 90° cos x
168. (d) ( cosec x − sin x )(sec x − cos x )(tan x + cot x ) and sin 3 x + cos 3 x = q ...(ii)
Now, cos( A + B ) + sin( A + B ) = cos 90° + sin 90° = 0 + 1 = 1 ⇒ cos 2 x − 2 cos x + 1 = 0 ⇒ (cos x − 1)2 = 0 ⇒ cos x = 1
=  − sin x  − cos x 
1 1 sin x cos x  On cubing Eq. (i) on both sides, we get
151. (a) Only statement I is correct as tanθ increases faster than sin θ as ∴ sec x =
1
=1 + 
 sin x   cos x   cos x sin x  sin 3 x + cos 3 x + 3 sin x cos x (sin x + cos x ) = p 3
θ increases while statement II is wrong as the value of sin θ + cos θ cos x
is not always greater than 1. It may also be equal to 1. ∴ cos n x + sec n x = ( 1)n + ( 1)n = 1 + 1 = 2 ( 1 − sin x)( 1 − cos x)(sin x + cos x) cos x sin x × 1
2 2 2 2 2 2
⇒ q + 3 sin x cos x( p ) = p 3 [from Eqs. (i) and (ii)] …(iii)
= = =1
152. (a) Since, value of cosθ decreases, from 0° to 90° and at 45° it is 161. (c) Given, sin θ + 2 cos θ = 1 sin x cos x ⋅ sin x cos x sin 2 x cos 2 x On squaring Eq. (i) on both sides, we get
equal to the value of sin θ. On squaring both sides, we get (sin θ + 2 cos θ ) = 1 2
sin x + cos x + 2 sin x cos x = p
2 2 2
sin x − cos x + 1 (sin x − cos x) + 1 (sin x + cos x) + 1
Similarly, value of sin θ increases from 0 to 90° and at 45° it is 169. (b) = ×
⇒ sin 2 θ + 4 cos 2 θ + 4 sin θ cos θ = 1 sin x + cos x − 1 (sin x + cos x) − 1 (sin x + cos x) + 1 p2 − 1
equal to the value of cosθ. ⇒ sin x cos x = (Q sin 2 x + cos 2 x = 1)
⇒ ( 1 − cos θ) + 4( 1 − sin θ) + 4 sin θ cos θ = 1
2 2
(sin x − cos x + 1)(sin x + cos x + 1) 2
For 0° < θ < 45°, cos θ > sin θ = 3( p 2 − 1) p
⇒ − (cos θ + 4 sin θ) + 4 sin θ cos θ = 1 − 5
2 2
(sin x + cos x)2 − 1 On putting this value in Eq. (iii), we get q + = p3
So, value of cos 25° − sin 25° is always positive but less than 1. 2
⇒ cos 2 θ + 4 sin 2 θ − 4 sin θ cos θ = 4 [Q ( a − b )( a + b ) = a 2 − b 2 ]
153. (b) In right angled ∆ABC, ∠B = 90° ⇒ 2q + 3 p 3 − 3 p = 2 p 3 ⇒ p 3 − 3 p = − 2q
⇒ ( 2 sin θ − cos θ)2 = 4 ⇒ 2 sin θ − cos θ = 2 sin 2 x + sin x cos x + sin x − cos x sin x − cos 2 x
∠C = 180°− ( ∠B + ∠A ) = 180°−90°−∠A = 90°−∠A 175. (d) Since, 1 radian = 57° 16′22′′
− cos x + sin x + cos x + 1
∴ sin C = sin( 90°− A ) = cos A = 4 / 5 162. (b) Given, tan 8θ = cot 2θ = So, sin 1°< sin 1 and cos 1°> cos 1
⇒ tan 8θ = tan( 90° − 2θ) ⇒ 8θ = 90° − 2θ ⇒ θ = 9° sin 2 x + cos 2 x + 2 sin x cos x − 1
154. (c) Since, α and β are complementary angle. Hence, neither statement I nor II is correct.
∴ tan 5θ ⇒ tan 45° = 1 sin 2 x + 2 sin x − cos 2 x + 1
∴ α = 90° − β = 176. (b) cosec 2 67°+ sec 2 57° –cot 2 33° –tan 2 23°
cos α 1 + 2 sin x cos x − 1
Now, cos α cosec β − cos α sin β = − cos α sin β 163. (a) Q sin ( A + B ) = 1 = sin 90° ⇒ ( A + B ) = 90° = cosec 2 (90° –23° ) + sec 2 (90° –33° ) – cot 2 33° –tan 2 23°
sin β sin 2 x + 2 sin x − ( 1 − sin 2 x) + 1
⇒ A = 90° − B = [Q cos 2 x = 1 − sin 2 x] = sec 2 23°+ cosec 2 33° – cot 2 33° – tan 2 23°
cos α 2 sin x cos x
= − cos α cos( 90°−β ) Now, cos ( A − B ) = cos A cos B + sin A sin B
sin 2 x + 2 sin x − 1 + sin 2 x + 1 = 1 + tan 2 23°+1 + cot 2 33° – cot 2 33° – tan 2 23° = 2
cos( 90° − β ) = cos ( 90° − B ) cos B + sin ( 90° − B ) sin B =
cos α = sin B cos B + cos B sin B = 2 sin B cos B = sin 2 B 2 sin x cos x (Q 1 + tan 2 q = sec 2 q and 1 + cot 2 q = cosec 2 q )
= − cos α ⋅ cos α = 1 − cos 2 α = sin α = sin α
2
cos α 2 sin 2 x + 2 sin x 2 sin x (sin x + 1) sin x + 1 177. (c) Given, tan ( A + B ) = 3 = tan 60°
12 = = =
164. (d) Clock will make right angle at ( 5n + 15) × min past n. ∴ A + B = 60°
155. (d) Given, sec θ + tan θ = 2 ...(i) 11 2 sin x cos x 2 sin x cos x cos x ...(i)
By trigonometric identity, sec 2 θ − tan 2 θ = 1 Here, n = 3 and tan A = 1 ⇒ tan A = tan 45°
⇒ (sec θ + tan θ)(sec θ − tan θ) = 1⇒ sec θ − tan θ = 1 / 2 ...(ii) 12 12 170. (b) (sin x − cos x ) ( 1 − sin x cos x )
2 2 2 2 ∴ A = 45°
∴ ( 5 × 3 + 15) × min past 3 = 30 × min past 3
On adding Eqs. (i) and (ii), we get 11 11 From Eq. (i), A + B = 60° ⇒ 45°+ B = 60° ⇒ B = 15°
= (sin 2 x − cos 2 x ) [(sin 2 x + cos 2 x )2 − sin 2 x cos 2 x]
1 5 = 32
8 8
min past 3 i.e. 3 h 32 min Now, tan( A – B ) = tan( 45° – 15° ) = tan 30° = 1 / 3
⇒ 2 sec θ = + 2 ⇒ sec θ = (Q sin 2 x + cos 2 x = 1)
2 4 11 11 Hence, the value of tan( A – B ) is 1 / 3.
MATHEMATICS Measurements of Angles and Trigonometric Ratios 223 224 CDS Pathfinder

2 2
178. (d) Given, tan A + cot A = 4 ⇒ 3 – tan 2 A = k – 3k tan 2 A II. [( 1 – sin A ) – cos A ]2 190. (b) We have, x = a cosθ and y = b cot θ sin 35°   cos 55°  + 2 sin 30° 200. (c) Q 3 − tan 2 θ = α ( 1 − 3 tan 2 θ)
196. (c)   − 
On squaring both sides, we get ⇒ 3k tan 2 A – tan 2 A = k – 3 = ( 1 – sin A )2 + cos 2 A  a b a b  a2 b2  cos 55°   sin 35°  ⇒ (3α − 1) tan 2 θ = α − 3
∴  −  +  = 2 − 2
(tan A + cot A )2 = ( 4)2 –2 cos A( 1 – sin A )
x yx y x sin( 90°−55° ) 
2 α −3
⇒ tan 2 A(3k – 1) = k – 3 y
= ⇒ tan 2 θ =
= 1 + sin 2 A – 2 sin A + cos 2 A  cos 55°  3α − 1
⇒ tan 2 A + cot 2 A + 2 tan A cot A = 16 k –3 = sec 2 θ − tan 2 θ = 1
⇒ tan 2 A = –2 cos A( 1 – sin A ) α −3
⇒ tan 2 A + cot 2 A + 2 = 16 3k – 1 cos( 90°−35° ) 
2 As, tan 2 θ ≥ 0, then ≥ 0 ⇒α≥3
−
= 1 + sin 2 A + cos 2 A – 2 sin A 3 1
⇒ tan 2 A + cot 2 A = 14 k −3 1 191. (a) We have, sin θ cos θ=2 cos 3 θ − cos θ + 2× 3α − 1
where > 0 ⇒ k < or k > 3 –2 cos A( 1 – sin A ) 2  sin 35°  2
3k − 1 3 or, α <
1  1
⇒ α∈  −∞ ,  ∪ [3, ∞ )
Again, squaring both sides, we get = 2 – 2 sin A – 2 cos A( 1 – sin A ) ⇒ 2 sin θ = 4 cos 2 θ − 3 cos 55° 
2
 sin 35°  + 1
2
 3
(tan 2 A + cot 2 A )2 = ( 14)2 Now, cosec A (3 sin A – 4 sin 3 A ) = 2 ( 1 – sin A ) – 2 cos A( 1 – sin A ) =   − 
3
⇒ 2 sin θ = 4 − 4 sin 2 θ − 3  cos 55°   sin 35° 
⇒ tan 4 A + cot 4 A + 2 tan 2 A = cosec A × sin A (3 – 4 sin A )
2 201. (a) To exchange the position, both hands
= 2 ( 1 – sin A )( 1 – cos A ) ≠ RHS ⇒ 4 sin 2 θ + 2 sin θ − 1 = 0,
1 = 12 − 12 + 1 = 1 has to cover 360° together.
cot 2 A = 196 = × sin A (3 – 4 sin 2 A ) Hence, only statement I is correct. −2 ± 4 + 16 −2 ± 2 5
⇒ sinθ = = 4 Angle traced by hour hand in 1 min
⇒ tan 4 A + cot 4 A + 2 = 196 sin A 2a + 3b 2a 197. (c) We have, tan θ + cot θ =
Q cosecA = 1  184. (a) We have, sinθ = = 1+ 8 8 3 = ( 1 / 2)°
⇒ tan 4 A + cot 4 A = 194   3b 3b
−1 ± 5 Angle traced by minute hand in 1 min = 6°
 sinA  ⇒ sinθ = sin θ cos θ 4
Since, sinθ is always smaller or equal to ⇒ + =
179. (c) In right angled ∆ABC , AB : BC = 3: 4 4 cos θ sin θ Let the required time be t min. Then,
 tan 2 A  1 but 1 +
2a
> 1.
3
AB 3 = 3 − 4 sin A = 3 − 4 
2
 1 360 × 2
= C Since, θ is acute angle sin 2 θ + cos 2 θ 6t + t = 360° ⇒ t =
 1 + tan A 
or 2 3b 4
BC 4 ⇒ = 2 13
k −3 Hence, it is not possible. ∴ sinθ > 0 cos θ ⋅ sin θ 3
Now, in ∆ABC 3− 720
3 − tan A
2
3k − 1 5−1 = min = 55.38 min
4 = = 185. (a) Since, 0 ≤ cos 2 x ≤ 1 ⇒ sinθ = ⇒
1
=
4
AC 2 = AB 2 + BC 2 1 + tan 2 A 1 + k − 3 4 sin 2θ 3
13
3k − 1 Q − 1 ≤ cos 2 x + cos 2 y − cos 2 z ≤ 2 1 − cos θ
=3 + 4
2 2
= 1 − cos θ 1 − cos θ
A B 9k − 3 − k + 3 ∴ Minimum value of the given 192. (d) Hours moved by hour hand = 5 hrs 2 202. (c) I. ×
= 9 + 16 = 25
3
= expression is − 1 . 10 min 1 + cos θ 1 + cos θ 1 − cos θ
3
3k − 1 + k − 3 ⇒ sin 2θ =
⇒ AC = 5 ⇒ sin A =
BC
=
4 10 31 ( 1 − cos θ)2 ( 1 − cos θ)2
8k 2k 186. (a) We have, tan θ + sec θ = 2 =5+ = hrs 2 = =
AC 5 = = , 60 6 π π 1 − cos 2 θ sin 2 θ
4k − 4 k − 1 ⇒ sec θ = 2 − tan θ ⇒ 2θ = ⇒ θ =
sin B = 90° = sin 90° = 1, sinC =
AB 3
= Angle traced by hour hand in 1 hr = 30° 3 6 1 − cos θ
1
where k < or k > 3 On squaring both sides, we get = = cosec θ − cot θ
AC 5 ∴ Angle traced by hour hand is π π sin θ
3 ∴ sin θ + cos θ = sin + cos
4
Now, sin A + sin B + sin C = + 1 +
3 sec 2 θ = 4 + tan 2 θ − 4 tan θ 6 6 Hence, statement I is true.
 31 hrs = 30 × 31 = 155°
5 5 182. (c) Given, p =
1 – sin x
⇒ 1 + tan 2 θ = 4 + tan 2 θ − 4 tan θ  
 6 6 1 3 3+1 1 + cos θ 1 + cos θ 1 + cos θ
4 + 5 + 3 12 1 + sin x 3 = + = II. = ×
= = ⇒ 4 tan θ = 3 ⇒ tan θ = 2 2 2 1 − cos θ 1 − cos θ 1 + cos θ
5 5 ( 1 – sin x)( 1 – sin x) 4 193. (b) I. If 45°< θ < 90°, then sin θ > cos θ
p=
180. (b) Given, sin x + cos x = C ( 1 + sin x)( 1 – sin x) cos θ 1 + sin θ ∴ sin 66°> cos 66° 198. (b) We have, ( 1 + cos θ)2 1 + cos θ
187. (b) We have, × = =
On squaring both sides, we get 1 – sin x 1 – sin x 1 − sin θ 1 + sin θ II. When 0°< θ < 45°, cos θ > sin θ p = cot θ + tan θ = cosecθ ⋅ sec θ sin 2 θ sin θ
= = cos θ ( 1 + sin θ) cos θ ( 1 + sin θ) = cosec θ + cot θ
(sin x + cos x)2 = C 2 1 – sin 2 x cos x = = ∴ cos 26°> sin 26° q = sec θ − cos θ = sin 2 θ ⋅ sec θ
1 − sin 2 θ cos 2 θ Hence, statement II is true.
⇒ sin 2 x + cos 2 x + 2 sin x cos x = C 2 cos x ( 1 – sin x) cos( 1 – sin x) or sin 26°< cos 26°. ∴ ( p 2 q )2 / 3 − ( q 2 p )2 / 3
r= × = 1 + sin θ
⇒ 1 + 2 sin x cos x = C 2 1 + sin x ( 1 – sin x) 1 – sin 2 x = Hence only II is correct. = ( cosec 2θ ⋅ sec 2 θ ⋅ sin 2 θ ⋅ sec θ)2 / 3 cos 2 θ − 3 cos θ + 2
cos θ 203. (b) Q =1
C2 −1 cos x ( 1 – sin x) 1 – sin x 1 + tan 2 θ sec 2 θ sin 2 θ
⇒ sin x cos x = …(i) r= = 188. (c) We have, 194. (a) I. LHS = = − (sin 4 θ ⋅ sec 2 θ ⋅ cosecθ ⋅ sec θ)2 / 3
2 cos 2 x cos x 1 + cot 2 θ cosec 2θ ⇒ cos 2 θ − 3 cos θ + 1 + 1 − sin 2 θ = 0
tan( x + 40)° tan( x + 20)° tan(3x )° = (sec 3 θ)2 / 3 − (sin 3 θ ⋅ sec 3 θ)2 / 3
Now, sin 6 x + cos 6 x = (sin 2 x)3 ⇒ p=q=r = tan 2 θ ⇒ cos 2 θ − 3 cos θ + 1 + cos 2 θ = 0
1 − sin x cos x tan( 70 − x )° tan(50 − x )° = 1 = sec 2 θ − sin 2 θ ⋅ sec 2 θ
+(cos 2 x)3 Also, q × r = × 1 − tan θ 
2
 tan θ[cot θ − 1] 
2 ⇒ 2 cos 2 θ − 3 cos θ + 1 = 0
1 + sin x [ Q cot( 90° − θ) = tan θ] RHS =   = = sec 2 θ( 1 − sin 2 θ) = 1
= (sin 2 x + cos 2 x)[sin 4 x + cos 4 x cos x  1 − cot θ   1 − cotθ  ⇒ ( 2 cos θ − 1) (cos θ − 1) = 0
1 − sin x ⇒ tan( x + 40)° tan( x + 20)° tan(3x )° 199. (d) Given,
x y
− tan θ = 1 ...(i) 1
– sin 2 x cos 2 x] = = p 2
= ( − tan θ)2 = tan 2 θ ⇒ cos θ = or, cos θ = 1
1 + sin x cot( 90° − 70° + x )° a b 2
[Q a 3 + b 3 = ( a + b )( a 2 + b 2 – ab )] cot( 90° − 50° + x )° = 1 π
Hence, both statements are correct. ∴ LHS = RHS and
x
tan θ + = 1
y
...(ii) ⇒ θ = [Q 0 < θ < π / 2]
= 1 [(sin 2 x + cos 2 x)2 – 3 sin 2 x cos 2 x]
cos A sin A ⇒ tan( x + 40)° tan( x + 20)° tan(3x )° II. cot θ =
1
⇒ tan θ ⋅ cot θ = 1
a b 3
= ( 1 – 3 sin 2 x cos 2 x) 183. (a) I. LHS = + tan θ
1 – tan A 1 – cot A cot( 20 + x )° cot( 40 + x )° = 1 On solving Eqs. (i) and (ii), we get Hence, only II is correct.
∴ sin 6 x + cos 6 x = 1 – 3 sin 2 x cos 2 x
2
=
cos A
+
sin A ⇒ tan(3x )° = 1 ⇒ tan(3x )° = tan 45° which true for all value of θ. x 1 + tan θ y 1 − tan θ 204. (d) I. As, cos x lies between −1and 1, then
 C 2 – 1 = and =
= 1 – 3  [from Eq. (i)] 1–
sin A
1–
cos A ⇒ 3x = 45 ⇒ x = 15 So, statement II is false. a sec 2 θ b sec 2 θ cos x = 2m + 1 does not exist for positive
 2  π π
− 8 sec 2  
cos A sin A Hence, only statement I is correct. value of m.
189. (d) We have, 32 cot 2
 C 4 + 1 – 2C 2  cos 2 A sin 2 A 4 3 x 2 y 2 ( 1 + tan θ)2 + ( 1 − tan θ)2 II. The given relation m n ≥ m + n is true
= 1 – 3  = + 195. (d) We have, A + B + C = π ∴ + =
  3  π a2 b2 sec 4 θ for m , n ∈ N and m > 1 and n > 1.
4 cos A – sin A sin A – cos A + 8 cos   π
4 – 3C 4 – 3 + 6C 2 1 + 6C 2 – 3C 4  6 ⇒ + B + B + C = π [A = B + π/ 2] 2 sec 2 θ
= = cos 2 A – sin 2 A 2 = Thus, statement II is not true.
=  3
3
sec 4 θ
4 4 (cos A – sin A ) = 32 ⋅ ( 1) − 8 ⋅ ( 2)2 + 8 ⋅   π π Hence, neither I nor II is correct
⇒ 2B + C = π − =
3 – tan 2 A (cos A – sin A )(cos A + sin A )  2  2 2 = 2 cos 2 θ
181. (c) Given, =k =
1 – 3 tan 2 A (cos A – sin A ) = 32 − 8 ⋅ ( 4) + 8 ⋅
3 3
⇒ 3 – tan 2 A = k( 1 – 3 tan 2 A ) = (cos A + sin A ) = RHS
8
= 32 − 32 + 3 3 = 3 3
21
MATHEMATICS > Height and Distance 225 226 CDS Pathfinder

EXAMPLE 2. The angle of elevation of a tower at a


point is 45°. After going 40 m towards the foot of the ANGLE OF DEPRESSION
tower, the angle of elevation of the tower becomes Angle of depression is defined as an angle formed by
60°. Then, the height of the tower is the line of sight with the horizontal line when an object
a. 20( 3 + 1) m b.
20 3
m
is viewed in the downward direction.
3+1 Horizontal line
A
O
c.
40 3
m d.
40 3
m θ Angle of
depression
3+1 3 −1

Lin
eo
P

HEIGHT AND DISTANCE


Sol. d. Let PQ ( = h) be the height of the

f sig
tower. Let S and R be the points

ht
P
where the angles subtended are
h Let P be the position of the object below the horizontal
45° and 60°, respectively.
line OA and O be the eye of the observer, then ∠AOP
In right angled ∆PQR ,
60°
is called angle of depression.
PQ 45°
tan 60° = Q
RQ S R
40 m x
IMPORTANT POINTS
h h
Regularly (1-2) questions have been asked from this chapter. Generally this is the simplest application ⇒ 3= ⇒ x= …(i)
x 3 l The angle of depression of a point P as seen from a
of trigonometry related to our real world and a little practice can get you command over this section. PQ PQ h point O is numerically equal to the angle of elevation of O
In right angled ∆PQS, tan 45° = = = as seen from P .
SQ SR + RQ 40 + x
l Angle of elevation and angle of depression are always
h acute angle.
⇒ h = 40 + x ⇒ h = 40 + [from Eq. (i)]
3 Unless mentioned, the height of the observer is not
It is an important application of trigonometry which helps l

IMPORTANT POINTS  1  3 − 1 considered.


us to find the height of any object and distance of that ⇒ h 1 −  = 40 ⇒ h   = 40
 3  3 
object from any point which are not directly measurable. If l If the observer moves towards the perpendicular line EXAMPLE 4. The angle of depression of two ships
the angle of elevation/depression from a point is known. (tower/building), then angle of elevation increases 40 3 from the top of a light house are 45° and 30° towards
and if the observer moves away from the perpendicular ∴ h= m
3 −1 East. If the ships are 200 m apart, the height of the
Line of sight line (tower/building), then the angle of elevation
light house is
decreases.
A line of sight is the line drawn from the eye of an observer EXAMPLE 3. The angles of elevation of the top of a 200 200 100 100
a. m b. m c. m d. m
to the point, where the object is viewed by the observer.
l If the angle of elevation of Sun above a tower decreases, tower from two points at distances ‘a’ and ‘b’ (a > b) 3 −1 3+1 3 −1 3+1
then the length of shadow of a tower increases.
D from the base and in the same straight line with it are
l If height of tower is doubled and the distance between complementary. Then, the height of the tower is Sol. a. Let AB( = h) be the X A
ig ht the observer and foot of the tower is also doubled, then 30°
fs 1 a height of the light house 45°
e
o Tower the angle of elevation remains same. a. b. ab c. d. ab and D, C be the ships.
Lin ab b
h
A Given, DC = 200 m
Horizontal line EXAMPLE 1. The angle of elevation of the top of a Sol. b. Let AD = h be the height of the
A E ∠ADC = ∠XAD = 30°
Person tower of height 100 3 m from a point at a distance tower and the angle of elevation 30° 45°
B C of the top of tower at B and C be and ∠ACB = ∠XAC = 45° D C y B
of 100 m from the foot of the tower on a horizontal h 200 m
(90° − θ) and θ, respectively. Now, in ∆ACB,
Horizontal Line plane is θ AB h
In right angled ∆ADC, 90
°– θ tan 45° = ⇒1= ⇒ h=y …(i)
The line of sight which is parallel to ground level is a. 30° b. 60° c. 75° d. 105° AD h  B CB y
Q tan θ = 
P C b D
tan θ = = a
known as horizontal line. Sol. b. Let φ be the angle of elevation. CD b  B  and in ∆ADB, tan 30º =
AB

1
=
h
h = b tan θ …(i) BD 3 200 + y
Given, AC = Height of tower
ANGLE OF ELEVATION = 100 3 m C In right angled ∆ADB, tan (90° − θ) =
AD
⇒ cot θ =
h ⇒ 200 + y = 3 h ⇒ 200 + h = 3 h [from Eq. (i)]
Angle of elevation is defined as an angle formed by the BD a ⇒ 200 = 3 h − h ⇒ 200 = h ( 3 − 1)
and AB = 100 m
line of sight with the horizontal [Q tan (90° − θ) = cot θ]  200 
(Object) P In right angled ∆ABC, ∴ h=  m
line when an object is viewed in 100√3 m h a  3 − 1
AC 100 3 ⇒ = cot θ ⇒ = tan θ …(ii)
the upward direction. ht tan φ = = = 3 a h
ig
of
s AB 100 φ EXAMPLE 5. From the top of a building 60 m high
Let P be the position of the object e B A From Eqs. (i) and (ii), we get
Lin Angle of ⇒ tan φ = tan 60° 100 m the angles of depression of the top and bottom of a
h = b   ⇒ h2 = ab
above the horizontal line OA and a
θ elevation
 h
tower are observed to be 30° and 60°. Then, the
O be the eye of the observer, then ⇒ φ = 60°
Eye
O Horizontal line A height of the tower is
∠AOP is called angle of elevation. Hence, angle of elevation is 60°. ∴ h = ab a. 10 m b. 20 m c. 30 m d. 40 m
MATHEMATICS > Height and Distance 227 228 CDS Pathfinder

Sol. d. Let x be the height of the tower. AB 60


= tan 60° ⇒ = 3 11. The length of the shadow of a person s cm tall 18. The angle of elevation of the top of an
Here, AB = 60 m and BD = y m BD y incomplete vertical pillar at a horizontal
when the angle of elevation of the Sun is α is
In right angled ∆ABD, 60 distance of 100 m from its base is 45°. If the
⇒ y= …(i) p cm. It is q cm when the angle of elevation of the
A 3 Sun is β. Which one of the following is correct angle of elevation of the top of complete pillar at
30° In right angled ∆AEC, when β = 3 α? the same point is to be 60°, then the height of
60°
AE AE 1  tan α − tan 3α   tan 3α − tan α  the incomplete pillar is to be increased by
= tan 30° ⇒ = [QCE = DB] (a) p − q = s   (b) p − q = s  
 tan 3α tan α   3 tan 3α tan α  (a) 50 2 m (b) 100 m
C 30° EC BD 3
(c) 100 ( 3 − 1) m (d) 100 ( 3 + 1) m
y E AE 1 AE 1  tan 3α − tan α   tan 2 α 
⇒ = ⇒ = [from Eq. (i)] (c) p − q = s   (d) p − q = s  
19. A man on the top of a vertical observation tower
60 m

y 3 60 3  tan 3α tan α   tan 3α tan α 


3 observes a car moving at a uniform speed
12. Two posts are ‘K ’ m apart and the height of one coming directly towards it. If it takes 12 min for
x 1 60
⇒ AE = × = 20 m [Q 3 × 3 = 3] is double that of the other. If from the middle the angle of depression to change from 30° to 45°,
3 3 point of the line joining their feet, an observer how soon after this will the car reach the
60° ∴ x = AB − AE = 60 − 20 = 40 m finds the angular elevations of their tops to be observation tower?
Hence, the height of tower is 40 m complementary, then the height (in metre) of the
D y B (a) 14 min 35 s (b) 15 min 49 s
shorter post is
K K (c) 16 min 23 s (d) 18 min 5 s
(a) 3K (b) (c) 2 K (d)
2 2 2 20. You are stationed at a radar base and you
13. A flagstaff 5 m high stands on a building of 25 m observe an unidentified plane at an altitude

PRACTICE EXERCISE height. At an observer at a height of 30 m, the


flagstaff and the building subtend equal angles.
The distance of the observer from the top of the
h = 1000 m flying towards your radar base at an
angle of elevation = 30°. After exactly one
minute, your radar sweep reveals that the plane
flagstaff is is now at an angle of elevation = 60° maintaining
1. The length of shadow of a tree is 16 m when the 7. An aeroplane flying horizontally 1 km above the 3 5 5 3 3 the same altitude. What is the speed (in m/s) of
angle of elevation of the Sun is 60°. What is the ground is observed at an elevation of 60°. After (a) m (b) 3 m (c) m (d) 5 m the plane?
2 2 2 2
height of the tree? 10 s, its elevation is observed to be 30°, the (a) 15.58 m/s (b) 19.25 m/s
uniform speed of the aeroplane in km/h is 14. A man is watching from the top of a tower a boat
(a) 8 m (b) 16 m (c) 16 3 m (d) 3m (c) 18 m/s (d) 11.25 m/s
240 speeding away from the tower. The boat makes
2. From a light house the angles of depression of (a) (b) 240 3 an angle of depression of 45° with the man’s eye 21. The angle of elevation of the top of the tower
two ships on opposite sides of the light house are
2
when at a distance of 60 m from the bottom of from a point on the ground is sin −1 (3/5). If the
observed to 30° and 45°. If the height of light (c) 240 (d) 260 3 point of observation is 20 m away from the foot
tower. After 5 s, the angle of depression becomes
house is h, then what is the distance between 8. A person of height 2m wants to get a fruit which 30°. What is the approximate speed of the boat of the tower, what is the height of the tower?
the ships? is on a pole of height (10 / 3) m. If he stands at a assuming that it is running in still water? (a) 9 m (b) 18 m (c) 15 m (d) 12 m

(d)  1 +
1  distance of ( 4 / 3 ) m from the foot of the pole, (a) 31.5 km/h (b) 36.5 km/h (c) 38.5 km/h (d) 40.5 km/h 22. A balloon of radius r makes an angle α at the
(a) ( 3 + 1) h (b) ( 3 − 1) h (c) 3h h
 3 eye of an observer and the angle of elevation of
then the angle at which he should throw the 15. Suppose the angle of elevation of the top of a tree
3. The angle of elevation of the top of a tower from stone, so that it hits the fruit is at a point E due East of the tree is 60° and that its centre is β. The height of its centre from the
the bottom of a building is twice that from its (a) 60° (b) 45° (c) 90° (d) 30° at a point F due West of the tree is 30°. If the ground level is given by
top. What is the height of the building, if the distance between the points E and F is 160 ft, (a) r sin β cosec α /2 (b) r cosec α /2 sin α
height of the tower is 75 m and the angle of 9. A vertical tower stands on a horizontal plane (c) r cosecα sin β (d) None of these
and is surmounted by a vertical flagstaff of then what is the height of the tree?
elevation of the top of the tower from the bottom 40
height h. At a point on the plane, the angle of (a) 40 3 ft (b) 60 ft (c) ft (d) 23 ft
of the building is 60°? 3 Directions (Q. Nos. 23-25) The height of a tower is h
elevation of the bottom of the flagstaff is α and and the angle of elevation of the top of the tower is
(a) 25 m (b) 37.5 m (c) 50 m (d) 60 m 16. A telegraph post gets broken at a point against a
that of the top of the flagstaff is β. Then, the
4. A radio transmitter antenna of height 100 m storm and its top touches the ground at a α . When observer on moving a distance h / 2
height of the tower is
distance 20 m from the base of the post making towards the tower, the angle of elevation becomes β.
stands at the top of a tall building. At a point on h tanβ h tan α
(a) (b) an angle 30° with the ground. What is the height
the ground, the angle of elevation of bottom of tan α − tan β tan α + tan β 23. What is the value of cot α − cot β?
the antenna is 45° and that of top of antenna is of the post?
h tan β h tan α (a) 1 / 2 (b) 2 / 3 (c) 1 (d) 2
60°. What is the height of the building? (c) (d) 40
tan α + tan β tan β − tan α (a) m (b) 20 3 m (c) 40 3 m (d) 30 m
24. If α = 30°, then find the value of cot β.
(a) 100 m (b) 50 m (c) 50 ( 3 + 1) m (d) 25 3 m 3
10. From a window (h m high above the ground) of a 2
5. The shadow of a tower is 15 m when the Sun’s 17. If the angle of elevation of a cloud from a point (a) 3 − 1 / 2 (b)
house in a street, the angle of elevation and 2 3 −2
altitude is 30°. What is the length of the shadow h m above a lake is β and the angle of depression
depression of the top and the foot of another 1 1
when the Sun’s altitude is 60°? of its reflection in the lake is α, then the height of (c) (d)
house on the opposite side of the street are θ and 1+ 3 1− 3
the cloud is
(a) 3 m (b) 4 m (c) 5 m (d) 6 m φ, respectively. Then, the height of the opposite h cosec (α − β ) 25. If tan β = 4, then distance between observer and
(a) (b) h cosec (α − β ) sin (α − β )
6. The angle of elevation of a moon when the length house is cosec (α − β ) tower when angle of elevation is β
of the shadow of a pole is equal to its height is (a) h tan θ cot φ (b) h [tan θ cot φ + 1] h cosec (α + β ) 3h h
(c) h sin (α + β ) cosec (α − β ) (d) (a) h (b) h / 4 (c) (d)
(a) 60° (b) 45° (c) 90° (d) 30° (c) h [cot θ tan φ + 1] (d) h cot θ tan φ sin (α − β ) 4 3
MATHEMATICS > Height and Distance 229 230 CDS Pathfinder

PREVIOUS YEARS’ QUESTIONS 34. What is the angle of elevation of the Sun, when 42. The angles of elevation of the top of a tower from 44. An aeroplane flying at a height of 3000 m passes
x two points P and Q at distance m 2 and n 2 vertically above another aeroplane at an instant,
26. What is the angle of elevation of the Sun when the the shadow of a pole of height x m is m?
3 respectively, from the base and in the same when the angles of elevation of the two planes
shadow of a pole is 3 times the length of the e 2013 II straight line with it are complementary. The from some point on the ground are 60° and 45°,
pole? e 2012 I (a) 30° (b) 45° (c) 60° (d) 75° height of the tower is e 2015 I respectively. Then, the vertical distance between
(a) 30° (b) 45°
35. A spherical balloon of radius r subtends angle (a) (mn)1/ 2 (b) mn1/ 2 the two planes is e 2015 II
(c) 60° (d) None of these
60° at the eye of an observer. If the angle of (a) 1000 ( 3 − 1) m (b) 1000 3 m
(c) m1/ 2 n (d) mn
27. From the top of a cliff 200 m high, the angles of elevation of its centre is 60° and h is the height (c) 1000 (3 − 3 ) m (d) 3000 3 m
depression of the top and bottom of a tower are of the centre of the balloon, then which one of 43. A pole is standing erect on the ground which is
observed to be 30° and 45°, respectively. What is 45. Two observers are stationed due North of a
the following is correct? e 2013 II horizontal, The tip of the pole is tied tight with a
the height of the tower? e 2012 I
tower (of height x m) at a distance y m from each
(a) h = r (b) h = 2 r rope of length 12 m to a point on the ground. If
other. The angles of elevation of the tower
(a) 400 m (b) 400 3 m the rope is making 30° with the horizontal, then
(c) h = 3 r (d) h = 2 r observed by them are 30° and 45°, respectively.
(c) 400 / 3 m (d) None of these the height of the pole is e 2015 II
36. The angle of elevation of the top of a tower 30 m Then, x / y is equal to e 2016 I
28. The angles of elevation of the top of a tower from (a) 2 3 m (b) 3 2 m 2 −1 3 −1 3+1
high from the foot of another tower in the (a) (b) (c) (d) 1
two points which are at distances of 10 m and (c) 3 m (d) 3 m 2 2 2
same plane is 60° and the angle of elevation
5 m from the base of the tower and in the same of the top of the second tower from the foot of
straight line with it are complementary. The the first tower is 30°. The distance between
height of the tower is
(a) 5 m (b) 15 m (c)
e 2012 II
50 m (d) 75 m
the two towers is m times the height of the ANSWERS
shorter tower. What is m equal to? e 2014 I
1 1 1 c 2 a 3 c 4 c 5 c 6 b 7 b 8 d 9 d 10 b
29. A ladder 20 m long is placed against a wall, so (a) 2 (b) 3 (c) (d)
that the foot of the ladder is 10 m from the wall. 2 3 11 c 12 b 13 d 14 a 15 a 16 b 17 c 18 c 19 c 20 b
The angle of inclination of the ladder to the 37. The shadow of a tower standing on a level 21 c 22 a 23 a 24 a 25 b 26 a 27 d 28 c 29 c 30 b
horizontal will be e 2012 II plane is found to be 50 m longer when the 31 a 32 c 33 a 34 c 35 c 36 b 37 b 38 c 39 d 40 c
(a) 30° (b) 45° Sun’s elevation is 30°, then when it is 60°. 41 c 42 d 43 d 44 c 45 c
(c) 60° (d) 75° What is the height of the tower? e 2014 I
25
(a) 25 m (b) 25 3 m (c) m (d) 30 m
Directions (Q. Nos. 30-33) Read the following 3
information carefully to answer the questions that 38. From a certain point on a straight road, a person
follow.
As seen from the top and bottom of a building of height
observe a tower in the West direction at a
distance of 200 m. He walks some distance along
HINTS AND SOLUTIONS
h m, the angles of elevation of the top of a tower of the road and finds that the same tower is 300 m 1. (c) Let the height of the tree be h m. ∴ y=h …(i) and in right angled ∆BCE ,
(3 + 3)h South of him. What is the shortest distance of ∴ BC = h m and AB = 16 m and in right angled ∆PTA , CE 75
height m are α and β, respectively. tan 60° = ⇒ 3=
the tower from the road? e 2014 II h x
2 e 2013 I C tan30° = ⇒ x = 3h …(ii) BC
300 500 600 900 x ⇒ x 3 = 75
30. If β = 30°, then what is the value of tan α? (a) m (b) m (c) m (d) m ∴ Required distance, AB = x + y ⇒( 75 3 − h 3 ) 3 = 75 [from Eq. (i)]
13 13 13 13
(a) 1/2 (b) 1/3 hm x + y = 3h + h = h ( 3 + 1) m
39. If from the top of a post a string twice the length ⇒ 75 × 3 − 3h = 75
(c) 1/4 (d) None of these 75 × 2
of the post is stretched tight to a point on the 3. (c) Let height of the building be h m ⇒ 3h = 75 × 3 − 75 ⇒ h =
31. If α = 30°, then what is the value of tan β? ground, then what angle will the string make A
60°
B
and distance between building and 3
16 m tower be x m. ∴ h = 50 m
(a) 1 (b) 1/2 with the post? e 2014 II
π π 5π π In right angled ∆ABC , ∴ AB = h m and BC = x m Hence, the height of the building is
(c) 1/3 (d) None of these (a) (b) (c) (d) BC h h E 50 m.
6 4 12 3 tan 60° = = ⇒ 3=
32. If α = 30° and h = 30 m, then what is the

(75 – h) m
AB 16 16 4. (c) Let BC be a building of height h m
distance between the base of the building and 40. The angle of elevation of a cloud from a point ⇒ h = 16 3 m and CD be a height of antenna. and let
the base of the tower? 200 m above a lake is 30° and the angle of distance between A and B = x m
depression of its reflection in the lake is 60°. The Hence, the height of the tree is 16 3 m.
30°
(a) (15 + 15 3 ) m (b) (30 + 15 3 ) m height of the cloud is e 2015 I 2. (a) In right angled
A x D 75 m D
(c) (45 + 15 3 ) m (d) None of these (a) 200 m (b) 300 m (c) 400 m (d) 600 m ∆PBT, tan 45° =
h
⇒ 1=
h
y y h 100 m
33. If β = 30° and if θ is the angle of depression of the 41. From the top of a tower, the angles of depression P
foot of the tower as seen from the top of the of two objects P and Q (situated on the ground 30° 45° 60° C
building, then what is the value of tan θ? on the same side of the tower) separated at a B x C
(3 − 3) (3 + 3 ) distance of 100( 3 − 3 ) m are 45° and 60°, h
In right angled ∆ADE,
hm
(a) (b) ED 1 75 − h 60°
3 3 3 3 respectively. The height of the tower is e 2015 I tan 30° = ⇒ =
(2 − 3) (a) 200 m (b) 250 m 30° 45° AD 3 x 45°
(c) (d) None of these A xm B
A x T y B ⇒ x = 75 3 − h 3 …(i)
3 3 (c) 300 m (d) None of these
MATHEMATICS > Height and Distance 231 232 CDS Pathfinder

In right angled ∆ABC , 7. (b) Let A be the observer. When the 9. (d) Let BC be the tower and CD be the 11. (c) In right angled ∆BAC , On multiplying Eq. (i) by Eq. (ii), we get In right angled ∆ACB, tan 45° =
AB
BC h aeroplane was at point B the angle of flagstaff. BC s s 4h 2h AC
tan 45° = = ⇒x = h ...(i) tan α = = ⇒p = …(i) ⇒ × =1
AB x elevation was 60° and after 10 s when tanα h
∴ ∠BAC = α and ∠BAD = β AC p K K ⇒ 1= ⇒ h = 60 m ...(i)
and in right angled ∆ABD, it was at point C the angle of elevation K2 60
In right angled ∆ABC , B ⇒ 8h 2 = K 2 ⇒ h 2 =
was 30°. 8 Now, in right angled ∆ADB,
DB DC + CB 100 + h
tan 60° = = = Here, in the figure, D ∴ h=
K
m AB AB 60
BA AB x BE = CD = 1 km = 1000 m tan 30° = = =
2 2 AD AC + CD 60 + x
100 + h
⇒ 3= [from Eq. (i)] B C h Hence, the height of shorter post is
h [from Eq. (i)]
s K
m. 1 60
⇒ 3h = 100 + h 1 km 1 km C 2 2 ⇒ = ⇒ 60 + x = 60 3
3 60 + x
100
⇒ ( 3 − 1) h = 100 ⇒ h = 60° β
3−1 30°
13. (d) Let O be an observer at a height ⇒ x = 60( 3 − 1) = 60( 173
. − 1)
D A α 30 m. Let x be the distance of observer
( 3 + 1)
A E B = 60 × 0.73 = 438
. m
100 α β from the top of the flagstaff.
⇒ h= × In right angled ∆BAE, A D q C Now, given time = 5 s
( 3 − 1) ( 3 + 1) tan α =
BC
…(i) i.e. OC = x
BE 1000 p Distance
tan 60° = = AB x C We know that, Speed =
O
= 50 ( 3 + 1) m AE AE α Time
and in right angled ∆ABD, In right angled ∆BDC , α 5m
⇒ AE = 1000 cot 60° 438
. 18 788.4
Hence, the height of the building is BD BC + h BC s s B ∴ Speed of boat = × =
1000 = = tan β …(ii) tan β = = ⇒q= …(ii) 5 5 25
50 ( 3 + 1) m. = m …(i) AB AB DC q tan 3α
3 30 m 25 m = 31.5 km/h
5. (c) Let the height of the tower be h m On dividing Eq. (ii) by Eq. (i), we get
In right angled ∆ACD, [Q β = 3α, given] 15. (a) Let AC = h ft = Height of a tree
and length of the shadow ( BC ) be x m.
DC 1000 BC + h tan β
In right angled ∆ACD, tan 30° = = = On subtracting Eq. (ii) from Eq. (i), we A and x ft = Distance between A and F
AD AD BC tan α get
tan 30° =
CD Here, AB be the tower of the flag staff then AE = ( 160 − x) ft
⇒ ( BC + h ) tan α = BC tan β s s
AC ⇒ AD = 1000 cot 30° = 1000 3 p−q= − = 25 m and BC be the flag staff = 5 m In right angled ∆AFC ,
1 h 15 ∴ ED = AD − AE ⇒ BC (tan β − tan α ) = h tan α tan α tan 3α
⇒ = ⇒h= m …(i) 1000 2000  tan 3α − tan α  In right angled ∆OBC , tan 30° =
AC h
= ⇒
1
=
h
= 1000 3 − = h tan α
= s 
3 15 3 m BC 5 AF x 3 x
D 3 3 ∴ BC =  tan α tan 3α  tan α = =
tan β − tan α   OC x ⇒ x = 3h …(i)
So, distance travelled by plane in 10 s,
then ⇒ x = 5 cot α ...(i) C N
2000 10. (b) Let O be the window and AB be 12. (b) Let PQ and RS be the two posts such
h BC = DE = m In right angled ∆OCA,
3 the house on the opposite side of the that PQ = 2RS [given] h W E
street. AC 30
∴ Speed of aeroplane =
Distance Q tan 2 α = = [Q AC = AB + BC ]
Time OC x F 30° 60° E
30° 60° A x
A 2000 S 30 A (160 – x) S
B x C ∴ tan 2 α = [from Eq. (i)] 160 ft
3 2000 × 60 × 60 2h 5 cot α
15 m = m/s = km/h h´ and in right angled ∆AEC ,
and in right angled ∆BCD, 10 10 3 × 1000 O θ h ⇒ tan 2α = 6 tan α
φ P θ 90°– θ tan 60° =
AC
=
h
CD h h = 240 3 km/h 2 tan α
tan 60° = ⇒ 3= ⇒ =x R M P ⇒ = 6 tan α AE 160 − x
BC x 3 Hence, the uniform speed of the h h K 1 − tan 2 α
h
15 aeroplane is 240 3 km/h.   ⇒ 3= [Qtan 60° = 3]
∴ x= =5m O´ B Given, M is the mid-point of RP. 2 tan α 160 − x
Q tan 2α = 
3 K  1 − tan 2 α  ⇒ 3( 160 − x) = h
[from Eq. (i)]
8. (d) Let TP be the man and SQ be the ∴ RM = PM = [Q RP = K given]  
pole. S Let AP = h ′ 2 ⇒ 3( 160 − 3h ) = h [from Eq. (i)]
2
Hence, the length of shadow is 5 m. BP = OO ′ = h ⇒ 3 − 3 tan α = 1 ⇒ tan α =
2 2
and Let ∠RMS = θ 3 ⇒ 160 3 − 3h = h ⇒ 4h = 160 3
When sun’s altitude is 60°. ∴ ∠QMP = 90° − θ
T θ In right angled ∆AOP, 2 ∴ h = 40 3 ft
R 10/3 m ⇒ tan α = or cot α = 3 / 2
6. (b) Let the height of pole be h m, then AP h′ Let RS = h, then PQ = 2h 3 Hence, the height of the tree is 40 3 ft.
tan θ = = …(i)
PQ = RQ = h 2m
OP OP 3
In right angled ∆PRQ , Now, in right angled ∆PMQ, or x=5 m [from Eq. (i)] 16. (b) Let the height of the post be h m, and
Q and in right angled ∆BOP,
P 4/√3 m PQ
2 AB = x m.
R
moon
PB h tan ( 90° − θ) = 14. (a) Let height of towerAB be h m and Given, BC = 20 m
Let ∠STR = θ tan φ = = …(ii) MP
OP OP distance between C and D be x m. D
Now, SR = SQ − RQ ⇒ cot θ =
2h
=
4h
h ′ tan θ

(h – x) m
h ∴ = [by Eq. (i) ÷ Eq. (ii)] k/2 k B
pole 10 4
= − 2 = m [QRQ = TP] h tan φ 4h 30°
45°
θ
3 3 or cot θ = …(i)
P Q In right angled ∆STR , ⇒ h ′ = h tan θ cot φ K
hm
h A
SR SR ∴ Height of the house In right angled ∆SRM, hm
tan θ =
RQ h
= =1 tan θ = = [Q TR = PQ] (h – x) m
TR PQ SR h
PQ h AB = AP + PB = h + h ′ tanθ = =
RM K / 2 xm
⇒ tan θ = 1 = tan 45° ∴ θ = 45° 4 3 1
= × = = tan 30° = h tan θ cot φ + h 2h 30°
Hence, the angle of elevation of moon 3 4 3 ⇒ = tan θ …(ii) 45° 30°
= h [tan θ cot φ + 1] K x C 20 m B
is 45°. ∴ θ = 30° A 60 m C D
MATHEMATICS > Height and Distance 233 234 CDS Pathfinder

In right angled ∆ABC , 18. (c) Let the height of the incomplete 20. (b) Let the radar base is at point A. The x + h/2
22. (a) Let O be the position of the man’s eye In ∆ABD, cot α = 28. (c) Refer to example 3.
AB 1 x pillar be x m and the increasing height plane is at point D in the first sweep and
tan 30° = ⇒ = and C be the centre of the balloon. h
BC 3 20 be PC = h m. at point E in the second sweep. The 29. (c) Let θ be the inclination of the ladder
Given, AB = 100 m B 3 x + h/2
20 distance it covers in the one minute ∴ = to the horizontal.
⇒ x= m …(i) P interval is DE. 4 h
3 h
E D C ⇒ 3h = 4x + 2h ⇒ x =
BC 20 h
and cos 30° = = 4 B
AC h−x α/2 h
Hence, required distance is
C α/2 A 4

m
3 20 40
⇒ = ⇒ h−x= 1000 m O D

20
2 h−x 3 β 26. (a) Let θ be the angle of elevation of Sun
°
60 α and height of the pole be h m.
From Eq. (i), putting the value of x , x 30° Now, ∠BOC = ∠COA = θ
A B C 2 A A 10 m O
40 20 60 3 Given, CA = BC = r
h= + = × = 20 3 m 60° From the figure,
3 3 3 3 45° CD Now, in right angled ∆ AOB ,
A B In right angled ∆COD, sin β = h
100 m In right angled ∆ADC , we get OC AO 10 1
Hence, the height of the post is 20 3 m.
θ cos θ = = =
DC 1000 1000 ∴ CD = OC sin β …(i)
In right angled ∆ABC , tan30° = = ⇒ AC = C AB 20 2
17. (c) Let P be the cloud and P ′ its image in AC AC tan 30° In right angled ∆ COA, √3 h
B
the lake. Let T be the point ‘h’ m above tan 45° =
BC
=
x
⇒ x = 100 m...(i) ⇒ cos θ = cos 60°
Similarly, in ∆EAB, we get α CA
the surface of the lake and let x be the AB 100 sin = In right angled ∆ABC , ∴ θ = 60°
EB 1000 1000 2 OC
height of the cloud. and in right angled ∆APB, tan 60° = = ⇒ AB = AB h Hence, the angle of inclination of the
AB AB tan 60° r α tan θ = ⇒ tan θ =
P PB PC + BC x+ h ∴ OC = α = r cosec 2 ladder is 60°.
tan 60° = = = Total distance covered by plane in 1 min, sin
BC 3h
β AB AB 100 1
T Mx then 2 ⇒ tan θ = = tan 30° Solutions (Q. Nos. 30-33)
α ⇒ x + h = 100 3 α 3
h DE = AC − AB From Eq. (i), CD = r cosec sin β 30. (b) Given, β = 30°
⇒ h = 100 3 − x = 100 3 − 100 1000 1000 2 ∴ θ = 30°
S N DE = − AE
In right angled ∆ADE, tan β =
[from Eq. (i)] tan 30° tan 60° Hence, the height of centre of the
α Hence, the angle of elevation of Sun is 30°. DE
∴ h = 100 ( 3 − 1) m 1000 balloon is r sin β cosec .
x = 1000 3 − = 1154.70 m 2 27. (d) Let AE be the height of the cliff and tan30° =
AE

1
=
AE
Hence, the height of the incomplete 3 h m (BD) be the height of the tower. DE 3 DE
pillar is 100( 3 − 1) m. 23. (a) Let AB be the tower of height h m,
The speed of the plane is given by A 3 + 3 
19. (c) Let AB be the tower and C , D be the BC be x m and DC be h / 2 m. ⇒ DE = 3 AE = 3   h
P´ s = distance covered/time taken 30°

200 − h
two positions of the car. A  2 
∴ ST = h = DE / 60 = 19.25 m/s. 45°
Then, ∠ACB = 45° , ∠ADB = 30°. 3
Also, NP = NP ′ = x [say] 21. (c) R 30° ⇒ BC = DE = ( 1 + 3 ) h ... (i)
Let AB = h , CD = x and AC = y . B C 2
Then, PM = x − h x

200 m
B
P ′M = x + h h A
tower

In right angled ∆PTM, x h


h
PM
TM
= tan β h ()
3
q = sin–1 5
D
α
C
β
x B 45°
B α
θ
C
3+√3 h m
∴ x − h = TM tan β …(i) P 20 m Q h/2
D x E 2
In right angled ∆P ′TM, Let P be the point of observation and In right angled ∆ABD,
30° 45° In right angled ∆ABC , hm
P ′M D x C y A QR be the tower. h x+ h/ 2 200 − h
= tan α tanα = ⇒ cot α = . tan30° =
AC
=
Given that, θ = sin −1   , PQ = 20 m. β
TM 3 x+ h/ 2 h θ
In right angled ∆ABC , we get 5 BC x D E
x + h = TM tan α …(ii) AB h …(i) 1 200 − h
= tan 45° = 1 ⇒ =1 Let the height of the tower, QR = h and Now, in right angled ∆ABC , ⇒ = [Q BC = DE ]
From Eqs. (i) and (ii), we get AC y 3 x
x − h tan β PR = x h x Now, in right angled ∆ABC ,
= ⇒ y=h …(i) tanβ = ⇒ cot β = …(ii) ⇒ x = ( 200 − h ) 3 …(i)
x + h tan α From the right angled ∆PQR, x h AC
In right angled ∆ABD, we get and in right angled ∆ADE, ⇒ tanα =
 3 
⇒ sin sin −1   = h / x
QR Subtracting eq. (ii) from eq. (i), we get BC
Using componendo and dividendo rule, AB
= tan30° =
1

h
=
1 sin θ = AE 200
we get AD 3 x+ y 3 PR   5   cot α − cot β =
x + h/2 x
− tan 45° = = ⇒ BC tan α = ( AE − CE )
DE x
x − h + x + h tan β + tan α 3 h 5h h h
= ( AE − BD ) [Q BD = CE ]
= ⇒ x + y = 3h ⇒ = ⇒x= …(i) 200
x − h − x − h tan β − tan α
…(ii)
5 x 3 x + h/ 2−x h/ 2 1 ⇒ 1= ⇒ x = 200 m
= = = x 3 + 3 
∴ x = 3h − h = h ( 3 − 1) h h 2 ⇒ BC tanα =   h−h
2x tan β + tan α From pythagoras theorem, we get,
 2 
⇒ = [by Eqs. (i) and (ii)] From Eq. (i), 200 = ( 200 − h ) 3
−2h tan β − tan α PQ 2 + QR2 = PR2 24. (a) We have, cot α − cot β = 1/2
Now, h ( 3 − 1) is covered in 12 min. ⇒ 200 = 200 3 − h 3 3 + 3 − 2
x tan β + tan α ⇒ 202 + h 2 = x2 ⇒ cot 30°− cot β = 1/ 2 =h 
⇒ = So, h will be covered in 2 ⇒ h 3 = 200( 3 − 1)  2 
h tan α − tan β ⇒ cot β = cot 30°−1 / 2
202 + h 2 =   [using Eq. (i)]
5h
  ⇒  3 − 1  1+ 3 
h = 200   m
3 = 3 − 1/ 2 3
x sin β cos α + cos β sin α 
12 12 ∴ ⇒ (1 + 3 ) h tan α =   h
⇒ = × h = min.  3  2  2 
h sin α cos β − cos α sin β  h ( 3 − 1)  ( 3 − 1) 25h 2 16 h 2 25. (b) tanβ = 4 ⇒ cot β = 1/ 4  
  ⇒ 202 + h 2 = ⇒ = 202
sin(α + β ) 9 9 we have, cot α − cot β = 1 / 2 Hence, the height of tower is [from Eq. (i)]
= 
= 1200 
sin(α − β )  min ≈ 16 min 23 s 3 × 20 3 200( 3 − 1) 1
 73  ∴ h= = 15 m ⇒ cot α = cot β + 1/2 = 1/ 4 + 1/2 = m. ∴ tan α =
∴ x = h sin (α + β ) cosec (α − β ) 4 4 3 3
MATHEMATICS > Height and Distance 235 236 CDS Pathfinder

31. (a) Given, α = 30 ° ( 3 − 1) 3 37. (b) Let h m be the height of the tower We know that, cos 2 θ + sin 2 θ = 1 In right angled ∆QMB, In ∆ABC , sin30° =
AB
=
h
⇒ tan θ = ⋅
In right angled ∆ABC , 3 3 and BC be x m. 2 2
 BD  +  BD  = 1 MQ H + 200 AC 12
⇒     tan 60° = =
AC 1 AC (3 − 3 ) In right angled ∆BCD,  300   200  ⇒
h
=
1
tan α = tan 30° = , = ∴ tan θ = BM BM
BC 3 BC DC h H + 200 12 2
3 3 tan 60° = = [from Eqs. (i) and (ii)] ⇒ 3= [from Eq. (i)]
⇒ BC = 3 AC = 3 ( AE − CE ) BC x 3( H − 200) 12 2 3
40000 + 90000 
34. (c) Let θ be the angle of elevation,
⇒ 3=
h
⇒ h=x 3 ...(i) ⇒ BD 2  =1 ∴ h= = = 3m
= 3 ( AE − BD ) [Q BD = CE ] A x  90000 × 40000  ⇒ H + 200 = 3( H − 200)
2 2
3 + 3  D 2  130000  44. (c) Let A and B be the position of two
= 3 − 1 h ⇒ BD =1 ⇒ H + 200 = 3H − 600
 2   3600000000  ⇒ 2H = 800 planes and D be a point.
3 360000 ∴ H = 400 m A
= (1 + 3) h ...(ii) x
h ⇒ BD 2 =
2 13 Hence, the height of the cloud is 400 m.
Now, in right angled ∆ADE, 360000 600
∴ BD = =

3000 m
θ m 41. (c) Let BC = h m be height of tower. Let B
AE 30° 60° 13 13
tan β = B C C P and Q be the points, where the
DE x/ 3 A B
x 39. (d) Let AB be the height of the post, AC angle subtended are 45° and 60°, h
AE In right angled ∆ABC , 50 m
⇒ tan β = [Q DE = BC ] be the string and the angle made by respectively. 60°
AC x 3x Now, in right angled ∆ACD, 45°
BC tanθ = = x = = 3 string with the post be θ. 45° C D
x
C
3 + 3  3( 1 + 3 ) BC x DC
  h h 3 tan30 ° = A
 2  2 AC 60° In ∆BCD, tan 45° =
h
⇒ = Here, tanθ = 3 = tan 60°
3 3 (1 + 3) DC h x
(1 + 3) h ∴ θ = 60° [Q tan 60° = 3 ] = =
h AB + BC 50 + x θ ⇒ h=x
2 2 35. (c) Given, radius of circle (OC ) = r 2h h
3000
∴ tan β = 1 1 x 3 h In ∆ACD, tan 60° = 3 =
⇒ = [from Eq. (i)] x
32. (c) Given, α = 30° and h = 30 m O 3 50 + x
60° 3000 3
In right angled ∆ABC , 45° Q x= × = 1000 3 m
r ⇒ 50 + x = 3x ⇒ x = 25 m 3 3
1 AC P Q x B
tan α = tan 30° = = C ∴ h = 25 3 m B C
100 (3–√3) m
3 BC h ∴ AB = 3000 − h
BC Hence, the height of tower is 25 3 m. AB h 1 In right angled ∆BCQ,
⇒ = ( AE − CE ) = ( AE − BD ) 60
° Now, cosθ = = ⇒ = cos 60° = 3000 − 1000 3
3 38. (c) Let person be at point C and observes AC 2h 2
60° π tan 60° =
BC
⇒ 3=
h = 1000(3 − 3 ) m
[Q BD = CE ] A B a tower in West direction at B. ∴ θ=
3 + 3  In right angled ∆ABO, BQ x
∴ BC = 200 m 3
45. (c) In right angled ∆ADC , tan 45° =
AC
⇒ BC = 3  − 1 h h
 2  OB OB N ⇒ x= DC
sin 60° = ⇒ AO = ...(i) 40. (c) Let P be the cloud at a height Hm ...(i)
(1+ 3) AO sin 60° above the level of the water in the lake 3 AC
⇒ BC = 3 ⋅ 30 ⇒ DC = =x …(i)
2 Now, in right angled ∆AOC , A and Q be its image in the water. In right angled ∆PBC , tan 45°
= ( 3 + 3) ⋅ 15 60° OC OC P tan 45° =
BC
=
BC A
= ⇒ AO =

(H – 200)
sin ...(ii)
∴ DE = BC = ( 45 + 15 3 ) m 2 AO sin30° 300 m D PB PQ + QB
[Q DE = BC ] From Eqs. (i) and (ii), θ h
⇒ 1= x
C B 30° 100(3 − 3 ) + x
33. (a) Given, β = 30° OB
=
OC

h
=
r W E M H
B 200 m
In right angled ∆ ADE , tan β =
AE sin 60 ° sin 30 ° 3 1 60° ⇒ 100 (3 − 3 ) + x = h
DE S 200 m 200 m 30° 45°
2 2 h
3 + 3  He walks some distance and reach at A. ⇒ 100(3 − 3 ) + = h [from Eq. (i)] B y D C
 h ∴ h = 3r O 3
Now, he observe tower in South A
 2  In right angled ∆BCA, we get
⇒ tan 30° = 36. (b) Let h m be the height of shorter direction at B. ⇒ h−
h
= 100(3 − 3 )
DE tower and the distance between the two ∴ AB = 300 m AC x
3 tan30° = ⇒
 1+ 3  towers is mh m. BC DC + BD
3 h Let BD be the shortest distance of tower h( 3 − 1)
Given, ∠ABD = 30° and ∠BAC = 60° ⇒ = 100 3( 3 − 1)
1  2  from the road, which is a perpendicular H

1
=
x
⇒ y + x = 3x
⇒ = C 3
3 DE distance. 3 y+x
⇒ h = 100 3 × 3
3
⇒ DE = ( 1 + 3 ) h If ∠ABD = θ , then ⇒ y = x ( 3 − 1)
…(i) D ∴ h = 300 m
2 ∠CBD = 90° − θ
Q x 1
In right angled ∆BDE, 30 m [Q angle between S and W = 90° ] 42. (d) Refer to example 3. ⇒ =
h ∴ OQ = OP = H y ( 3 − 1)
BD h In right angled ∆ADB, 43. (d) AB is a pole and AC is rope.
tan θ = =
DE DE Given, ∠PBM = 30° and ∠MBQ = 60° x 1 × ( 3 + 1)
h cosθ =
BD
⇒ cosθ =
BD
...(i)
A ⇒ =
⇒ tan θ = [from Eq. (i)] A
60° 30°
B AB 300 In right angled ∆PBM, y ( 3 − 1)( 3 + 1)
3 mh m PM H − 200
(1 + 3) h tan30° = = ( 3 + 1)
In right angled ∆CDB, x

pe
⇒ =

m
2 In right angled ∆ABD,

√1
BM BM

Ro
y ( 3 )2 − ( 1)2

2
2 ( 3 − 1) cos ( 90° − θ ) =
BD h
⇒ tan θ = tan 30° =
h

1
=
1 1 H − 200
3 ( 3 + 1) ( 3 − 1) BC ⇒ = Pole
x ( 3 + 1)
mh 3 m
BD 3 BM ∴ =
2( 3 − 1) ⇒ m= 3 ⇒ sinθ = ...(ii) 30° y 2
⇒ tan θ = 200 ⇒ BM = 3( H − 200) ...(i) C B
3⋅ 2 Hence, the value of m is 3.
22
237 238 CDS Pathfinder

Concurrent Lines 5. Reflex angle An angle whose measure is more than


180°, but less than 360°, is called a reflex angle.
Three or more lines intersecting A S
at the same point are said to be O O A
concurrent. This common point P Q
is the point of concurrence i.e. O. B
R B
In the given figure, 180 ° < ∠ AOB < 360 °
Non Intersecting lines/Parallel Lines
6. Complete angle An angle whose
If two lines lie in the same plane and do not intersect A
measure is 360°, is called a complete O
when produced on either side, then such

LINES AND ANGLES lines are said to be parallel to each other.


If l and m are two parallel lines, we write l || m and read
it as l is parallel to m .
l
m
angle. In the given figure, ∠AOA is a complete angle.

Pairs of Angle
1. Complementary Angles
ANGLES Two angles are said to be
complementary, if the sum of
An angle is formed by two rays with a common initial
their measure is 90°. Thus, ∠θ1
point. Let ‘O’ is the initial point, then O is called the
Usually (2-3) questions have been asked from this chapter. And this is the easiest topic of geometry and ∠θ 2 are complementary, if
vertex.
and so candidate can easily score marks in examinations. ∠ θ1 + ∠ θ 2 = 90 ° θ1
θ2
A • Complementary angles are O
complement of each other.

R
P • Complement of x is (90 ° − x ).
m
In this chapter, we will study about the properties of ar
Ray angle
EXAMPLE 1. The measure of an angle which is 28°
the angles which formed, when two lines intersect each A ray extends indefinitely in one direction. This is
vertex
other and when a line intersect two or more parallel →
O arm B more than its complement is
lines at distinct points. exhibited by an arrow i.e. PQ . Where, P is a point in the interior of ∠AOB and R is a a. 23° b. 59° c. 77° d. None of these
• P is called the initial point of the ray. The ray has no point in the exterior of ∠AOB. Sol. b. Let the measure of the required angle be x°. Then,
Basic Terms and Definitions definite length. measure of its complement = 90° − x
Types of Angle ∴ x − (90° − x) = 28° ⇔ 2x = 118°
Point Collinear Points 1. Acute angle An angle whose measure is more than
The figure of which length, breadth and height cannot ⇔ x = 59°
Three or more points are said to be collinear if a single 0°, but less than 90°, is called an acute angle.
be measured is called a point. It is infinitesind. Hence, the measure of the required angle is 59°.
straight line passes through them. Here, A, B and C are
collinear. A
Line Segment EXAMPLE 2. The measure of the complement of an
The straight path between two points P and Q is called angle of 48° 36 ′ 24′ ′ is
a line segment. O B a. 41°23′ 36′′ b. 42°23′ 36′ c. 41°24′ 36′′ d. 42°24′ 36′′
Non-collinear Points
This can be represented as PQ. In the given figure, 0 ° < ∠AOB < 90 ° Sol. a. As, 90° = 89° 59′ 60′′
Three or more points not lying on a single straight line A
• P and Q are called the end points of the line segment.
2. Right angle An angle whose measure is ∴ Complement of an angle of ( 48° 36′ 24′′)
are called non-collinear points. Here, A, B and C are
• The line segment has a definite length.
non-collinear points. 90°, is called a right angle. = Angle of [90° − 48° 36′ 24′′ ]
• Distance between P and Q is called the length of the In the given figure, ∠AOB = 90 °. = Angle of [89° 59′ 60′′ − 48° 36′ 24′′]
O B
line segment PQ. = Angle of ( 41° 23′ 36′′)
3. Obtuse angle An angle whose measure is more than
Line 90° but less than 180°, is called an obtuse angle. 2. Supplementary Angles
A line segment extended endlessly on both sides, is A
Two angles are said to be supplementary, if the sum of
↔ their measures is 180°. Thus, ∠ θ1 and ∠θ 2 are
called a line. It is denoted by PQ Intersecting Lines B supplementary if θ1 + θ 2 = 180 °
↔ O
or QP. Two lines having a common In the given figure, 90 ° < ∠ AOB < 180 °.
point are called intersecting θ2
• A line is a set of infinite number of points. 4. Straight angle An angle whose θ1
lines. This common point • • •
• A line have no end points and no definite length. measure is 180°, is called a straight A O B
is the point of intersection • Supplementary angles are supplement of each other.
angle.
i.e. O. Supplement of x is (180 ° − x).
In the figure, ∠ AOB = 180 ° •
MATHEMATICS Lines and Angles 239 240 CDS Pathfinder

EXAMPLE 3. The measure of an angle, which is 32° If two lines PQ and RS intersects at a point O, then the 1. Corresponding angles Corresponding angle pairs EXAMPLE 6. In the given figure, PQ || RS . The value of
less than its supplement is pair of ∠POR and ∠QOS or pair of ∠POS and ∠ROQ are x is
a. 31° b. 64° c. 74° d. 148° is said to be a pair of vertically opposite angles. ∠1 and ∠5, ∠ 2 and ∠6, ∠ 4 and ∠8, ∠ 3 and ∠7 and T
all the corresponding pair are equal x
Sol. c. Let the measure of the required angle be x. Then, P
S 100
measure of its supplement = (180° − x) i.e., ∠1 = ∠5, ∠2 = ∠6, ∠4 = ∠8 and ∠3 = ∠7. R S
50
∴ (180° − x) − x = 32° ⇔ 2x = 148° ⇔ x = 74° O 2. Vertically opposite angles Vertically opposite P Q

Q angles pairs are a. 50° b. 80° c. 75° d. 65°


EXAMPLE 4. Two supplementary angles are in the R
ratio 3 : 2. Then, the measurement of the smaller angle is • Vertically opposite angles are always equal ∠1 and ∠ 3, ∠ 4 and ∠ 2, ∠8 and ∠6, ∠5 and ∠7 and Sol. a. Draw AB || PQ
a. 36° b. 72° c. 108° d. 112° i.e. ∠POS = ∠ROQ and ∠POR = ∠SOQ . all vertically opposite angles are equal ∴ ∠ATP = ∠TPQ = 50° [alternate angles]
i.e., ∠1 = ∠3, ∠4 = ∠2, ∠8 = ∠6 and ∠5 = ∠7. and ∠BTR + ∠TRS = 180°
Sol. b. Let the supplementary angles be 3x and 2x, • Sum of the angles around a point is 360 °.
[Angles on the same side of transversal]
respectively. Then, according to the definition of
EXAMPLE 5. In the given figure if DOC is a straight 3. Alternate angles Alternate angles pairs are T
supplementary angle.
ray OB is bisector of ∠AOC, where ∠AOC = 110 ° and ∠ 3 and ∠5, ∠ 4 and ∠6 and they are equal A B
3x + 2x = 180° ∠COE = 120 °, then the sum of ∠x, ∠y and ∠z is i.e., ∠3 = ∠5 and ∠4 = ∠6. x
100°
⇒ 5x = 180° ⇒ x = 36° S
A • The sum of interior angles on the same side of R
∴ Angles will be 3x = 3 × 36 = 108° B 50°
transversal is equal to 180 °. P Q
and 2x = 2 × 36 = 72°. 110° i.e., ∠3 + ∠6 = 180 ° and ∠4 + ∠5 = 180 °
y z ⇒ ∠BTR = 80°
Thus, the smaller angle is 72°. O • The sum of exterior angles on the same side of and ∠ATP + x + ∠BTR = 180° [ Q ATB is a straight line]
D x C transversal is equal to 180°.
Bisector of an Angle 120°
i.e., ∠2 + ∠7 = 180 ° and ∠1 + ∠8 = 180 ° ∴ x = 180° − ( 50° + 80° ) = 50°

A ray which divides the angle into


P E
two equal parts, is called the bisector
of an angle. S a. 160° b. 115° c. 170° d. 180°
If a ray OS is the bisector of ∠POQ,
then ∠POS = ∠QOS.
1
O Q
Sol. d. Since, DOC is a straight line
∠AOC + ∠DOA = 180°
110° + ∠DOA = 180°
[Linear pair]
[Q ∠AOC = 110° ]
PRACTICE EXERCISE
• ∠POS = ∠QOS = ∠POQ
2 ∠DOA = 70° 1. Three lines intersect each other in pairs. What is 7. If ∠1 = ( 5x − 20)° and ∠7 = ( 2x + 10)°, then ∠7 is
∠DOA = ∠y = 70° the total number of angles so formed? l
Adjacent Angles Also, ray OB is angle bisector of ∠AOC
(a) 3 (b) 6 (c) 9 (d) 12
1 2
Two angles are said to be adjacent, if ∠AOC 110° 2. An angle is 14° more than its complement. Then, m 3 4
∠z = = = 55° its measure is
(i) they have a common vertex. 2 2
R (a) 166° (b) 86° (c) 76° (d) 52°
(ii) they have a common arm Q Line BE and DC intersect each other at O. 5 6
Q 3. The measure of an angle is twice the measure of
(iii) their non-common arms are on ∠x = ∠z = 55° [Q vertically opposite angle] n 8 7
either side of the common arm. O P its supplementary angle. So, its measure is
∴ ∠x + ∠y + ∠z = 55°+70°+55° = 180° (a) 120° (b) 60° (c) 100° (d) 90°
Here, ∠POQ and ∠ROQ are adjacent angles. (a) 38° (b) 10°
4. What is the least number of straight lines for a (c) 30° (d) 70°
Linear Pairs of Angles Some Other Angles bounded plane figure?
8. The measure of complementary angle of 12° 25′
Let l and m are parallel lines and n is the transversal (a) 1 (b) 2 (c) 3 (d) 4
If the non-common arms of two adjacent angles form a 40′′ is
which cuts these parallel lines. The different angles 3 (a) 77° 34′ 20′′ (b) 77° 36′ 20′′ (c) 77° 24′ 20′′ (d) 77° 34′
line, then these angles are called linear pair of angles. 5. The supplement of of a right angle is
formed are as follows. 5
∠ROP and ∠ROQ form a linear pair of angles. 9. ∠POR and ∠QOR form a linear pair and if
n (a) 122° (b) 126° (c) 130° (d) 132°
R a − b = 80°, then the values of a and b are,
6. In the given figure, if PQ||SR, then the relation respectively
2 1 l between ∠a and ∠b is R
P O Q 3 a
4
P Q
∴ ∠POQ = ∠ROP + ∠ROQ = 180 ° A a°

6 5 m
B P O Q
Vertically Opposite Angles 7 8
S
b
R
(a) 95°, 85° (b) 108°, 72°
Two angles are called a pair of vertically opposite
angles, if their arms form two pairs of opposite rays. (a) ∠a ≠ ∠b (b) ∠a < ∠b (c) ∠a = ∠b (d) ∠a > ∠b (c) 130°, 50° (d) 105°, 75°
MATHEMATICS Lines and Angles 241 242 CDS Pathfinder

10. In the given figure, the value of y is 17. The value of x in the given figure is Which of the statement(s) given above is/are 27. In the given figure, which of the
C F A correct? following statements must be true? b c
P 45° (a) I and II (b) II and III I. a + b = d + c II. a + c + e = 180° a d
5y O 2y (c) All of these (d) None of these III. b + f = c + e IV. a + b + c = d + e + f f e
A B
5y x B 22. In the given figure, AB||CD, then which one of Select the correct answer using the codes
E D the following is true? given below
30°
A E B (a) Only I (b) I, II and III
(a) 25° (b) 35° (c) 15° (d) 40° Q C
p° (c) II, III and IV (d) All of these
11. AB and CD are two parallel lines. PQ cuts AB and (a) 75° (b) 185°
CD at E and F, respectively. EL is the bisector of (c) 285° (d) 245° q° F 28. In the given figure, AB is parallel to CD. If
∠FEB. If ∠LEB = 35°; then ∠CFQ will be 1 ∠BAF = 98° and ∠AFC = 144°, then ∠ECD is
18. In the given figure, AB||CD, ∠DPL = ∠NPO, r° equal to?
(a) 110° (b) 85° (c) 70° (d) 95° and OP⊥ XY the value of x° is 2
C G D A
X B
12. AB and CD are two A B
parallel lines. The 65°
(a) p + q − r = 180° (b) p + q + r = 180° 98°
E F
points B and C are M 60° (c) p − q + r = 180° (d) p + q − 2 r = 180° F 144°
35° A B
joined such that 23. LM is a straight line and O is a point on LM.
∠ABC = 65°. A line D O L Line ON is drawn not coinciding with OL or OM.
C C D
CE is drawn making N x If ∠MON is one-third of ∠LON , then what is
angle of 35° with the line CB, EF is drawn parallel C D E
P ∠MON equal to?
to AB, as show in figure, then ∠CEF is equal to Y (a) 45° (b) 60° (a) 62° (b) 64°
(a) 160° (b) 155° (c) 150° (d) 145° (a) 30° (b) 40° (c) 15° (d) 25° (c) 75° (d) 80° (c) 82° (d) 84°
13. In the given figure, if EC|| AB, ∠ECD = 70°, 19. In the given figure, AB||CD and they cut PQ 24. Consider the following statements : 29. Consider the following statements :
∠BDO = 20°, then ∠OBD is equal to and QR at E, F and G , H respectively. Then find If two parallel lines are intersected by a
the value of x + y Two lines intersected by a transversal are
transversal, then
E A A C parallel, if
I. each pair of corresponding angles are equal.
x Gx Q I. The pairs of corresponding angles are equal.
y II. each pair of alternate angle are unequal.
70° E II. The interior angles on the same side of the
O 140° transversal are supplementary. Which of the statement(s) given above is/are
C 20° D H correct?
P 70° Which of the statement(s) given above is/are (a) Only I (b) Only II
B
correct? (c) Both I and II (d) Neither I nor II
(a) 70° (b) 60° (c) 50° (d) 20°
(a) Only I (b) Only II
14. Two parallel lines AB and CD are intersected by F (c) Both I and II (d) Neither I nor II 30. In the given figure, PQ is parallel to RS. Then
R
a transversal line EF at M and N , respectively. ∠NMS is equal to
The lines MP and NP are the bisectors of the
B D 25. In the given figure AC|| BD and AE|| BF . What L
P Q
interior angles BMN and DNM on the same side (a) 120° (b) 130° is the value of ∠x ? 55° 47°
(c) 150° (d) 132°
of the transversal. Then, ∠MPN is equal to C D
67° N
(a) 90° (b) 45° (c) 135° (d) 60° 20. Two parallel lines are cut by a transversal, then
which of the following are true? 130°
15. In the given figure if l|| m, then the value of x is A B
A I. Pair of alternate interior angles are congruent. x R M S
l 60°
100° II. Pair of corresponding angles are congruent. (a) 20° (b) 23° (c) 27° (d) 47°
III. Pair of interior angles on the same side of the G
O x+5 transversal are supplementary. E F 31. The line segments AB and CD intersect at O. OF
30° m is the internal bisector of obtuse ∠BOC and OE
Select the correct answer using the codes given (a) 130° (b) 110°
B
(c) 70° (d) 50° is the internal bisector of acute ∠AOC. If
(a) 105° (b) 100° (c) 110° (d) 115° below ∠BOC = 130°, then what is the measure of
16. In the given figure, if ∠COE = 90°, then the (a) I, II and III are true (b) I and III are true 26. The necessary conditions for the line l and m to ∠FOE ?
value of x is (c) I and II are true (d) II and III are true be parallel when these lines are intersected by a (a) 90° (b) 110° (c) 115° (d) 120°
A transversal line n is that
D
21. Consider the following statements related to 32. In the given figure below, AB is parallel to LM.
z three lines L1 , L2 and L3 in the same plane. I. Interior angles on the same side are equal.
O y
What is the angle a equal to?
2x II. Corresponding angles are equal.
I. If L 2 and L3 are both parallel to L1 , then they A B
x b
90° are parallel to each other. III. Vertically opposite angles are equal.
a
C B II. If L 2 and L3 are both perpendicular to L1 then IV. Alternate interior angles are equal.
L c M
E they are parallel to each other. Select the correct answer using the codes given
III. If there is acute angle between L1 and L3 , then below (a) π + b + c (b) 2 π − b + c
(a) 120° (b) 60° (c) 45° (d) 30° L 2 is parallel to L3 . (a) II and III (b) I, II and III (c) II and IV (d) I, II and IV (c) 2 π − b − c (d) 2 π + b − c
MATHEMATICS Lines and Angles 243 244 CDS Pathfinder

33. In the given figure, PQ is parallel to RS. What is


the angle between the lines PQ and LM?
Which of the statement(s) given above is/are
correct? e 2012 I
ANSWERS
P (a) Only III (b) Only I (c) I and III (d) II and III 1 d 2 d 3 a 4 c 5 b 6 c 7 c 8 a 9 c 10 c
Q
55°
40. In the given figure, LOM is a straight line. 11 a 12 c 13 c 14 a 15 a 16 d 17 c 18 c 19 c 20 a
What is the value of x° ? e 2012 II 21 a 22 a 23 a 24 c 25 b 26 c 27 c 28 a 29 a 30 a
L
M Q 31 a 32 c 33 d 34 b 35 c 36 b 37 d 38 a 39 b 40 b
P
155°
30° 41 b 42 c 43 d 44 a 45 d
R 25° S
50°
(a) 175° (b) 177° (c) 179° (d) 180° (x + 20)° (x – 10)°
L M
O
34. The length of a line segment AB is 2 units. It is
divided into two parts at the point C such that
AC 2 = AB × CB. What is the length of CB ?
(a) 45° (b) 60° (c) 70°
41. If the arms of one angle are respectively parallel
(d) 80°
HINTS AND SOLUTIONS
(a) 3 + 5 units (b) 3 − 5 unit to the arms of another angle, then the two 1. (d) We know that, when two lines 7. (c) Here, ∠ 1 = ∠5
(c) 2 − 5 unit (d) 3 units angles are e 2013 I intersect each other, it makes 4 angles. [corresponding angles] P
(a) Neither equal nor supplementary Since, the total number of pairs = 3 E
∠7 = ∠5 (vertical opposite) A B
(b) not equal but supplementary 35°
Directions (Q.Nos 35-36) Read the information and ⇒ ∠1 = ∠7 (angle)
answer the questions.' (c) equal but not supplementary L
(d) Either equal or supplementary So, 5x − 20° = 2x + 10° F
In the given figure, the lines CB and AC of a 3x = 30° ⇒ x = 10° C D
triangle ABC are extended to D and F, respectively 42. The complement angle of 80° is e 2015 I Hence, Q
and CF || GE. 18 5π
(a) radian (b) radian ∠ 7 = 2x + 10° = 2 ( 10° ) + 10° = 30° ⇒ ∠AEB = ∠AEF + ∠BEF = 180°
D 95° E F π 9 [straight line]
π 9 ∴ Total number of angles = 3 × 4 = 12 8. (a) Complementary angle of 12° 25′40′′ ⇒ ∠AEF = 180° − 70° = 110°
(c) radian (d) radian = 90° −12° 25′ 40′′
G 18 5π 2. (d) Let angle be x and its complement be ⇒ ∠CFQ = ∠AEF = 110°
90° − x. = 89° 59′ 60′′ − 12° 25′ 40′′ [corresponding angles]
C y° 43. Let OA,OB,OC and OD be rays in the According to the question, = (89 − 12)° + (59′ − 25′) + ( 60 − 40)′′ 12. (c) Since, AB || CD
anti-clockwise direction? Such that = 77° + 34′ + 20′′ = 77° 34′ 20′′ B
x° x = ( 90° − x) + 14 A
∠AOB = ∠COD = 100°, ∠BOC = 82° ⇒ 2x = 104° 9. (c) Since, ∠POR and ∠QOR is a linear 65° E F
35° z° and ∠AOD = 78°. ⇒ x=
104°
= 52°
pair.
35°
A B ∴ ∠POR + ∠ROQ = 180°
Consider the following statements 2
35. The sum of the value of x, y and z. R D
I. AOC and BOD are lines. 3. (a) Let angle be x and its supplementary C
(a) 250° (b) 180° (c) 230° (d) 110° II. ∠BOC and ∠AOD are supplementary. e 2015 I = 180° − x a
∴ ∠ABC = ∠BCD = 65°
Then, x = 2 ( 180° − x) b [alternate angles]
36. The value of the angle which equals one-fifth of Which of the statement(s) given above is/are ⇒ 3x = 360° P O Q ∠ECD = 65° − ∠BCE
the supplement of z ? correct? ⇒ x = 120° = 65° − 35° = 30°
⇒ a + b = 180° ... (i) ∠CEF + ∠ECD = 180°
(a) 50° (b) 26° (c) 10° (d) 130° (a) Only I (b) Only II
4. (c) The least number of straight lines for and a – b = 80° ... (ii) [Angles on the same side of
(c) Both I and II (d) Neither I nor II
a bounded plane figure is 3. On solving Eqs. (i) and (ii), we get transversal]
PREVIOUS YEARS’ QUESTIONS 44. If a transversal intersects four parallel straight 3 2a = 260° ⇒ a = 130° ⇒ ∠CEF = 180° − 30° = 150°
5. (b) Given, angle = of right angle
lines, then the number of distinct values of the 5 and b = 180° – 130° = 50° 13. (c) Since, EC || AB
37. AB is a straight line. C is a point whose distance angles formed will be e 2016 I 3
=
× 90° = 3 × 18° = 54° So, a = 130° , b = 50°
from AB is 3 cm. What is the number of points (a) 2 (b) 4 (c) 8 (d) 16 5 E A
which are at a distance of 1 cm from AB and 10. (c) Since, AOB is a straight line.
Supplement of 54° = ( 180°−54° )
5 cm from C ? e 2012 I
45. p ∴ ∠AOB = 180°
z 40 x 3
= An angle of measure 126° 70°
(a) 1 (b) 2 (c) 3 (d) 4 80° ∠AOC + ∠COF + ∠FOB = 180° O
° C D
6. (c) We have, ∠QAB = ∠a [straight line]
20°
38. The ratio of two complementary angle is 1 : 5. [vertically opposite angles] B
What is the difference between the two angles? 5 y + 5 y + 2 y = 180°
e 2012 I 100° x y a [Q ∠COF = ∠DOE , vertically ∠AOD = ∠ECO
q P Q
(a) 60° (b) 90° A opposite angles] [Q corresponding angles]
In the figure given above, p and q are parallel
(c) 120° (d) Cannot be determined
lines. What are the values of the angles x, y B ⇒ 12 y = 180° ⇒ ∠AOD = 70°
S R 180° So, ∠BOD = 180°−70°
39. Consider the following statements : and z ? e 2016 I b ⇒ y= = 15°
If two straight lines intersect, then (a) x = 80°, y = 40°, z = 100° 12°
= 110° [linear point]
and ∠QAB = ∠b Hence, the value of y is 15°.
I. vertically opposite angles are equal. (b) x = 80° , y = 50°, z = 105° Now, in ∆BOD
II. vertically opposite angles are supplementary. (c) x = 70°, y = 40° , z = 110°
[corresponding angles] 11. (a) Given, ∠LEB = 35°
⇒ ∠a = ∠b ∠OBD + ∠BOD + ∠ODB = 180°
III. adjacent angles are complementary. (d) x = 60°, y = 20°, z = 120° ∠FEB = 2 × ∠LEB = 2 × 35° = 70° ⇒ ∠OBD = 180° − ( 110 + 20)° = 50°
MATHEMATICS Lines and Angles 245 246 CDS Pathfinder

1 and Q XY ||QC , ∠QCB = ∠CBY = 30° 24. (c) Both the statements I and II are 30. (a) Since, PQ || RS ∴ ∠EOP = ∠OPB = b [alternate angle] 37. (d) ∴ Required number of points
14. (a) Given, ∠PMN = ∠BMN
2 [alternate angles] correct. ∴ ∠PLM = ∠LMS = 55° and ∠EON = ∠ONM = c = 4 (P , P , P , P )
1 1 2 3 4
and ∠PNM = ∠DNM ⇒ ∠ABC = ∠ABY + ∠CBY 25. (b) Since, AC || BD [alternate angle] [alternate angle] C
2
⇒ ∠ABC = 45° + 30° = 75° C D
Draw a line EF which is parallel to PQ. ⇒ ∠PON = b + c
As, ∠BMN + ∠DNM = 180°
Q x = 360° − ∠ABC = 360° − 75° Then, ∠QLN = ∠LNE = 47° Q ∠PON + a = 2 π
[angles on the same side of transversal] 130°
∴ x = 285°
E A B Q ∠ENL + ∠MNE = 67° ∴ a = 2 π − ∠PON = 2 π − b − c 5 cm 5 cm
M 18. (c) Q AB || CD, ∠ONP = ∠XMB = 60° x
A B 60° L 33. (d) 3 cm

5c
[corresponding angles] P Q P

m
G 55° 47° Q

5c

m
P ∠OPN = 90° − ∠ONP E F 55°
21 cm 21 cm
= 90° − 60° ⇒ ∠OPN = 30° 67° N P1 P2
∴ ∠DBA = 180° − 130° = 50° E F 1 cm
1 1 L
C
N
D But, ∠DPL = ∠NPO = 30° [interior angle] M A B
2 2 30° 155° 1 cm
F ⇒ ∠DPL = 15° ⇒ x = 15° [Q BAC = 130° ] R 25° S P4
R M S P3
∴ In ∆MPN , ∠PMN + ∠PNM = 90° DBG is straight line. 3 cm 3 cm
19. (c) Since, AB || CD and PQ is transversal PQ || RS …(i)
⇒ ∠MPN = 180° − ( ∠PMN + ∠PNM ) ∴ ∠DBA + ∠ABF + ∠FBG = 180° ⇒ 47° + ∠MNE = 67°
∠PEF = ∠EGH [corresponding angles] [linear pairs] Since, ∠PQR = ∠QRS = 30°+25° = 55° α 1
[angle sum property] ⇒ ∠MNE = 67° − 47° 38. (a) Given that, =
⇒ ∠EGH = 70° [Q ∠PEF = 70°] ⇒ 50° + ∠ ABF + 60° = 180° [alternate angle] β 5
∴ ∠MPN = 180° − 90° = 90° ⇒ ∠MNE = 20°
Now, ∠EGH + ∠HGQ = 180° ⇒ ∠ABF = 70° PQ || RS …(i) Let α = k and β = 5k
15. (a) Draw a line n passing through O and [linear pair] Similarly, EF || RS, then
Since, AE || BF and ∠SRL + ∠RLM = 180° Sum of two complementary angles
parallel to l and m ⇒ ∠HGQ = 180°−70° = 110° ∠ENM = ∠NMS = 20°
∴ x = 180° − ∠ABF ⇒ RS || LM …(ii) = 90°
Since, l || n, ∠1 + 100° = 180° Also, ∠DHQ + ∠GHQ = 180° [alternate angle]
= 180° − 70° = 110° From Eqs. (i) and (ii), we get ⇒ α + β = 90°,
[Sum of the interior angles on the [linear pair] 31. (a) Given, ∠BOC = 130°
26. (c) When two lines are parallel PQ || LM α = 90° − β
same side of the transversal] ∠GHQ = 180°−140° = 40° A D
intersected by a transversal then So, the angle between the lines PQ and
∠1 = 80° ∴ x + y = 110° + 40° = 150° corresponding as well as alternate ⇒ k = 90° − 5 k
LM is 180°.
Since, n || m, ∠2 = 30° [alternate angles] 20. (a) All the three statements are true. interior angles are equal. O ⇒ k = 15°
E 34. (b) Given, AC 2 = AB × CB
A 21. (a) Only statements I and II are true. ∴ α = 15° and β = 75°
l Hence, the statement II and IV are
correct. ⇒ x 2 = 2 × ( 2 − x)
100° 22. (a) Draw FH || AB || CD ∴ Difference between angles
C B A B
O
1 (x + 5)
n (sum of interior angles) 27. (c) We have, a = d , b = e and c = f F x C (2 – x) = 75° − 15° = 60°
2
30° m ∴ ∠1 + p = 180° ...(i) [vertically opposite angles] 2
Q AOB is a straight line. 39. (b) Here, AB and CD are two lines.
B A E B and a + b + c = d + e + f = 180° ⇒ x 2 = 4 − 2x
Q ∠BOC + ∠AOC = 180° [linear pair] A D
Now, ∠AOB = ∠1 + ∠2 p II. a + b + c = 180° ⇒ a + c + e = 180° ⇒ x 2 + 2x − 4 = 0
⇒ 130° + ∠AOC = 180°
= (80° + 30° ) = 110° III. b + f = c + e −2 ± 4 + 16
1
⇒ ∠FOC + ∠FOC = 130° ⇒ x=
But, ∠AOB = ( x + 5)° = 110° F
IV. a + b + c = d + e + f 2×1
H 2 ⇒ ∠AOC = 50°
⇒ x = − 1± B
x = ( 110 − 5)° = 105° 180–r Hence, statements II, III and IV are true. 5 C
r Now, ∠BOC = 130°
16. (d) Here, ∠BOD = ∠AOC 28. (a) Draw a line MN parallel to AB and Now, BC = 2 − ( − 1 ± 5) = 3 − 5 If two straight lines intersect, then
C G D ⇒∠BOF + ∠FOC = 130°
∴ 2x = y [vertically opposite angles] CD. (neglect 3 + 5 Q3 + 5 > 2) vertically opposite angles are equal.
∠2 + 180° − r = 180° ...(ii) A B [Q OF is bisector of ∠BOC ]
Now, COD is a straight line. 35. (c) We have, ∠DGE = ∠GCF = 95° 40. (b) From the given figure,
(sum of interior angle) ⇒ ∠FOC = 65°
∠COD = 180° 98°
[corresponding angles] ∠ LOQ + ∠QOP + ∠POM = 180°
On adding Eqs. (i) and (ii), we get Now, ∠AOC = 50°
⇒ ∠COE + ∠EOB + ∠BOD = 180° N [straight line]
∠1 + ∠ 2 + p + 180° − r = 360° M ⇒ ∠AOE + ∠EOC = 50° Also, ∠GCF + ∠FCB = 180°
⇒ 90°+ x + 2x = 180° F 144°
∴ ( x° + 20° ) + 50° + ( x° − 10° ) = 180°
⇒p + q − r = 180° [Q∠1 + ∠2 = q °] ⇒ ∠EOC + ∠EOC = 50° [linear pair]
⇒ 3x = 90° ⇒ x = 30° ⇒ 95°+ y = 180° ⇒ 2x° + 60° = 180°
Hence, the value of x is 30°. 1
23. (a) Given that, ∠ MON = ∠ LON C ⇒ ∠EOC = 25°
3 D ⇒ y = 180°−95° = 85° ⇒ 2x° = 120°
17. (c) From figure, PA || QC x [Q OE is bisector of ∠AOC ]
Let ∠ LON = x, then, ∠ MON = E Again, ∠ACB + y = 180° ∴ x ° = 60°
Draw a line XY parallel to PA and QC. 3 ∴ ∠EOF = ∠EOC + ∠FOC
⇒ ∠AFN = 180° − 98° = 82° ⇒ x = 180°−85° = 95° 41. (b) Case I When both pairs of arms are
P A = 25° + 65° = 90°
N (sum of interior angles) Now, In ∆ABC ∠A + ∠B + ∠C = 180° parallel same sense.
32. (c) Draw a line parallel to AB i.e.
45° x ∠CFN = 144° − 82° = 62° ⇒ 35°+ z + 95° = 180° A
x/3 EF || AB.
and ∠ECD = ∠CFN = 62° P ⇒ z + 130° = 180° D
x B L O M A B
X Y [corresponding angles] b ⇒ z = 180°−130° = 50°
We know that,
30° ∠ LON + ∠ MON = 180° [linear pair] 29. (a) If two parallel lines are intersected by ∴ x + y + z = 95° + 85° + 50° = 230° C
B G
a transversal, then each pair of E F
Q C x 180°×3 O a
36. (b) Supplement of z = 130° F
⇒ x + = 180° ⇒ x = = 135° corresponding angles and of alternate E
Q PA||XY , 3 4 1 130°
x 135° angles are equal. ∴ th of 130° = = 26°
⇒ ∠ABY = ∠PAB = 45° c Here, ∠ ABC = ∠DEF
Thus, ∠ MON = = = 45° Therefore, statement I is correct.
L N M 5 5
[alternate angles] 3 3
23
MATHEMATICS Lines and Angles 247 248 CDS Pathfinder

π π
Case II When both pairs of arms are = 10 × = radian. 44. (a) If, line P intersect four parallel lines
parallel in opposite sense. 180° 18 l, m, n and o, then 16 angles will be
43. (d) formed.
A B P
F E
100° 1 2
82° l
O 78° A
B C m
C
n
D D
o
Given, ∠AOB = ∠COD = 100°,
Here, ∠ ABC = ∠DEF
Case III When one pair of arms is
parallel in same direction and other pairs
are parallel in opposite direction.
and
∠BOC = 82°
∠AOD = 78°
If, AOC is a straight line, then
∠AOB + ∠BOC = 180°
As these lines are parallel, hence distinct
angle will be ∠1 and ∠2.
45. (d) In the given figure, lines p and q are
parallel.
TRIANGLES
A A
⇒ 100°+82° = 180° p
D 80° z x/3
⇒ 182° ≠ 180°

40
°
B If, BOD is a straight line, then
C ∠BOA + ∠AOD = 180° Generally (8-10) questions have been asked from this chapter. Generally questions are asked from
F E 100° x y
⇒ 100° + 78° = 180° q the topic of pythagoras theorem, similarity of triangles and mid-point theorem.
C B
Here, ∠ ABC + ∠DEF = 180° ⇒ 178° ≠ 180° x
∴ x = 40° + [alternate angles]
So, the two angles are not equal but If, ∠BOC and ∠AOD 3
supplementary. 2x
are supplementary angles, then ⇒ = 40° ⇒ x = 60°
42. (c) Let the angle be θ. 3
∴ θ = 80°
∠BOC + ∠AOD = 180° y = x /3
60°
[alternate angle] TRIANGLE
Complement angle ⇒ 82°+78° = 180° y=
x
= 20°, z + 40° + = 180°
3 3
A plane (closed) figure bounded by three line segments is called a triangle. It is A
= 90° – θ = 90° – 80° = 10° ⇒ 160° ≠ 180° denoted by ∆.
⇒ z + 20° = 180° − 40°
[Q sum of two complement angles is Hence, statements I and II are incorrect.
⇒ z = 120° ∆ABC has
90°]
• three vertices, namely A , B and C. • three sides, namely AB, BC and CA .
• three angles, namely ∠A , ∠B and ∠C.
B C
• Sum of three angles of a triangle is 180 °. i.e., ∠A + ∠B + ∠C = 180 ° D
1 1
• Area of a triangle = × base × height = × BC × AD
2 2

Classification of Triangle
A
1. On the Basis of Angles
Hy

Perpendicular
(i) Right Angled Triangle A triangle in which one of the angle measures 90 ° po
t en
is called a right angled triangle. The side opposite to the right angle is called us
e
its hypotenuse and the remaining two sides are called as perpendicular and
base depending upon conditions. Here, ∆ABC is a right angled triangle in θ
which ∠B = 90 ° and AC is hypotenuse. B Base C
C
(ii) Acute Angled Triangle A triangle in which every angle measure more than
0° but less than 90° is called an acute angled triangle.
Here, ∆ABC is an acute angled triangle. A B
(iii) Obtuse Angled Triangle A triangle in which one of the angle measures A
more than 90° but less than 180° is called an obtuse angled triangle.
Here, ∆ ABC is an obtuse angled triangle and ∠ABC is the obtuse angle.
B C
MATHEMATICS Triangles 249 250 CDS Pathfinder

2. On the Basis of Sides • Centroid divides the median in the ratio 2 : 1. Sol. c. Q ∠CAX = 137° • In ∆ABC, if ∠B > 90 ° and AD is perpendicular dropped
(i) Equilateral Triangle A triangle having A • A median bisects the area of the triangle i.e. X on BC, then AC 2 = AB 2 + BC 2 + 2BC ⋅ BD
all sides equal is called an equilateral 1 A
Ar( ∆ABD) = Ar( ∆ADC ) = Ar( ∆ABC ), etc. A
triangle. Here ∆ABC, is an equilateral 2
triangle in which AB = BC = AC. Incentre and Angle Bisector The H
K
B C A
All angles are equal and are of measure point of intersection of all the three
60°. angle bisectors of a triangle is called its
r
incentre. It is denoted by I.
(ii) Isosceles Triangle A triangle in which two sides x r D B C
• The circle with centre I and touches all B C
are equal, is called an isosceles triangle. Here, ∆ABC B C
the sides is called incircle and radius of
D

1
∠ABC =
1
(137° ) = 68 °
• In ∆ABC, if ∠B < 90 ° and AD ⊥ BC, then
is an isosceles triangle as AB = AC.
this circle is called inradius denoted by ‘r’. 2 2 AC 2 = AB 2 + BC 2 − 2BC ⋅ BD
• Angle opposite to equal sides are equal. 1
1 Side ∴ Again, BC = CH and ∠ABC = 68 ° A
i.e. ∠B = ∠C • Inradius = × Height = , 2
3 2 3 1
A Therefore, ∠CHB = 68 ° , Therefore, ∠HCB = 43°
ID → Inradius, AD → Height 2
1 1
Perpendicular Bisector and Circumcentre The point Hence, ∠HCK = 68 ° − 43° = 25 °
2 2
B C of intersection of the perpendicular bisectors of the sides B D C
of a triangle is called its A Some Important Results of a Triangle In any triangle, the sum of the
(iii) Scalene Triangle A triangle in which all the sides • A
circumcentre. It is denoted by C.
are of different lengths is called a scalene triangle. • If two angles of a triangle are equal, then the sides square of any two sides is equal
∆ ABC is a scalene triangle as AB ≠ BC ≠ AC. The circle with centre C and opposite to them are also equal. to twice the square of half of the
A passing through vertices C third side together with twice
• In a triangle, the side opposite to a greater angle is the
A , B and C is called circumcircle. R the square of the median which B D E C
B longest side.
Radius of circumcircle is called O D
bisect the third side.
circumradius denoted by R . • The sum of all the three interior angles of a triangle is
• Here, AD is median, so
2 Side 180°.
B C Circumradius (CO) = × Height = 1 
2
3 3 • If a side of a triangle is produced, then the exterior
AB 2 + AC 2 = 2AD 2 + 2  BC
Note The sum of the lengths of three sides of a triangle is called angle so formed is equal to the sum of the two interior 2 
its perimeter. EXAMPLE 1. In the diagram given below what is the
So, perimeter of ∆ABC = AB + BC + AC
opposite angles. • In a ∆ ABC, three times the sum of the squares of the
sum of all the angles ∠A, ∠B, ∠C, ∠D, ∠E and ∠F? • An exterior angle of a triangle is greater than either of sides of a triangle is equal to four times the sum of the
A
Some Term Related to a Triangle the interior opposite angles. squares of the medians of the triangle.
• A triangle must have atleast two acute angles. A
Altitudes and Orthocentre The F B
altitude of a triangle is a line segment A • If a perpendicular is drawn from the vertex of a
perpendicular drawn from vertex to F E
E C right angled triangle to the hypotenuse, then the
the side opposite to it. The side on F E triangles on both sides of the perpendicular are
which the perpendicular is being drawn H B C
D similar to the original triangle and also to each other. D
is called its base. a. 120° b. 180° • In a right angle ∆ABC, ∠B = 90 ° and AC is hypotenuse. So, in triangle, if AD, BE, FC are the medians, then
Here, In ∆ABC, AD, BE and CF are B D C
c. 290° d. 360° The perpendicular BD is dropped on hypotenuse AC • 3( AB 2 + BC 2 + AC 2 ) = 4 ( AD 2 + BE 2 + CF 2 )
altitudes drawn on BC, AC and AB, respectively.
Sol. d. Since, sum of the angles of a triangle is 180°. from right angle vertex B, then • In an equilateral ∆ ABC, the side BC is trisected at D.
• Altitudes of a triangle are concurrent. AB × BC
In ∆ AEC , ∠A + ∠C + ∠E = 180° …(i) (i) BD = A Then, 9 AD 2 = 7 AC 2
• The point of intersection of all the three altitudes of a AC
and In ∆ BDF , ∠B + ∠D + ∠F = 180° …(ii) • When the bisector of one internal base angle and the
triangle is called its orthocentre.
On adding the Eqs. (i) and (ii), we get AB 2 bisector of external angle meet at a point then the
Medians and Centroid A line (ii) AD =
A ∠A + ∠B + ∠C + ∠D + ∠E + ∠F = 180° + 180° = 360° AC D formed angle is equal to one half of the vertical angle.
segment joining the mid-point of a side
to the opposite vertex is called median. EXAMPLE 2. ABC is an isosceles triangle in which BC 2 A
F (iii) CD = E
E AB = AC, CH = CB and HK is parallel to BC. If the AC
Here, In ∆ABC, AD, BE and CF are exterior ∠CAX = 137 °, then what is the measure of
medians. G 1 1 1 B C
∠HCK? (iv) = +
• The medians of a triangle are B D C 1° BD 2 AB 2 BC 2
concurrent. a. 68 b. 43°
2 • The area of the equilateral triangle described on the side B C D
• The point of intersection of all the three medians of a 1
1° of a square is half the area of the equilateral triangle Here, ∠BEC = ∠BAC
triangle is called its centroid. It is denoted by G. c. 25 d. 137° •
2 described on its diagonal. 2
MATHEMATICS Triangles 251 252 CDS Pathfinder

• The side BC of ∆ ABC is produced to D. The bisector • Sum of any two sides of a triangle is greater than its EXAMPLE 3. In the given figure, I is the incentre of In congruent triangles, corresponding parts are equal
of ∠A meets BC in L . Then third side. ∆ABC. What is the measure of angle A . and we write in short ‘CPCT’, i.e. corresponding part of
∠ABC + ∠ACD = 2 ∠ALC. D A congruent triangles.
A
A
Criteria for Congruence of Triangles
I
108°
Theorem 1 Two triangles are congruent if two sides
B C
B C
and the included angle of one triangle are equal to the
• Here, in ∆ ABC, AB + AC > BC corresponding sides and the included angle of other
a. 54° b. 18° c. 36° d. None of these triangle. (SAS criteria)
B L C D • similarly, AB + BC > AC, BC + AC > AB
In a ∆ ABC the bisector of ∠B and ∠C intersect each Sol. c. Here, I is the incentre of the ∆ ABC. Theorem 2 Two triangles are congruent if two angles
• • The difference between any two sides of a triangle is
other at a point O. ∴ BI and CI are the bisectors of ∠B and ∠C, then
and the included side of one triangle are equal to the
less than its third side.
A 1 corresponding two angles and the included side of the
A we know that, ∠BIC = 90° + ∠A other triangles. (ASA criteria)
2
O
D 1 Theorem 3 Two triangles are congruent if three sides
⇒ 108° = 90° + ∠A
2 of one triangle are equal to the corresponding three
B C 1 sides of the other triangle (SSS).
B C ⇒ ∠A = 108° − 90° = 18°
2 Theorem 4 Two triangles are congruent if the
1 Here, in ∆ ABC, AC − AB < BC ∴ ∠A = 36°
• then, ∠BOC = 90 ° + ∠A •
hypotenuse and other side of one triangle are equal to
2 Similarly, BC − AC < AB, BC − AB < AC

EXAMPLE 4. In a triangle ABC, ∠BAC = 90 ° and AD the hypotenuse and the corresponding side of the other
• In a ∆ABC, the side AB and AC are produced to P and triangle (RHS).
• If the bisector of the vertical angle of a triangle bisects is perpendicular to BC. If AD = 6 cm and BD = 4 cm,
Q , respectively. The bisectors of ∠PBC and ∠QCB
the base, then that triangle is an isosceles. then the length of BC is Theorem 5 Two triangles are congruent if two angles
intersect at a point O. and a side other than the included side of one triangle
• If the altitude from one vertex of a triangle bisects the a. 8 cm b. 10 cm c. 9 cm d. 13 cm
A are equal to the corresponding two angles and a side
opposite side, then the triangle is an isosceles. Sol. d. In ∆ABC, ∠BAC = 90° other than the included side of other triangle.
• The perpendiculars drawn from the vertices of equal ∴ AB = AD 2 + BD 2 [By pythagoras theorem] (AAS Criteria)
B C
angles of an isosceles triangle to the opposite sides are = 36 + 16 = 52
equal. We know that, in a right angled C EXAMPLE 5. In a ∆ABC, the altitudes BD and CE are
P O Q triangle the perpendicular is drawn on equal and ∠A = 36 °. What is the value of the ∠B?
• Medians of equilateral triangle are equal.
hypotenuse from right angle vertex, a. 72° b. 84° c. 18° d. 36°
1
Then, ∠BOC = 90 ° − ∠A • If D is the mid-point of the hypotenuse AC of a right then
2 1 D Sol. a. For the ∆BDC and ∆BEC,
angled ∆ ABC. Then, BD = AC AB 2
BD =
• In ∆ ABC, ∠B > ∠C. If AN is the bisector of ∠BAC 2 BC 6 4 A
and AM ⊥ BC, then AB 2 = BD × BC
A
A ⇒ ( 52) 2 = BC × 4 A B E D
D 52
⇒ BC = = 13 cm
4 B C

BD = EC, BC = BC and ∠BEC = ∠BDC = 90°


B C Congruent Figures Thus, ∆ BEC ~
= ∆ BDC [by RHS rule]
B M N C The sum of any two sides of a triangle is greater than The geometrical figures having the same shape and size 180° − 36°

are known as congruent figures. ∴ ∠B = ∠C = = 72° each
1 twice the median drawn to the third side. 2
∠MAN = ( ∠B − ∠C ) e.g. Two circles of the same radii and two squares of the
2 Perimeter of a triangle is greater than the sum of its

same sides are congruent. EXAMPLE 6. In the given figure, D is the midpoint of
• The bisectors of the base angles of a triangle can never three medians. BC, DE ⊥ AB and DF ⊥ AC such that DE = DF , then
enclose a right angle.
A
Congruent Triangles which of the following is correct?
A triangle is an isosceles if and only if any two altitudes A
• Two triangles are said to be congruent, if both are
are equal. D exactly of same size i.e. all angles and sides are equal to
F E
• If the three sides of a triangle be corresponding angles and sides of other.
A
produced in order, then the sum of all • Every triangle is congruent to itself ∆ ABC ~ = ∆ ABC E F
B C
the exterior angles so formed is 360°. D • If ∆ ABC ~ = ∆ DEF , then ∆DEF ~ = ∆ ABC
B F B D C
• So, ∠DAB + ∠EBC + ∠ACF = 360 ° C
• So, AB + BC + AC > AD + BE + CF . • If ∆ ABC ~ = ∆ DEF and a. AB = AC b. AC = BC
E ∆ DEF ~= ∆ PQR , then ∆ ABC ~ = ∆ PQR c. AB = BC d. None of these
MATHEMATICS Triangles 253 254 CDS Pathfinder

Sol. a. In right angled ∆BED and right angled ∆CFD Theorem 3 The line joining the Criteria for Similarity of Two Triangles Sol. d. In ∆ABC and ∆PQC.
A
DE = DF (given) mid-points of any two sides of a
hypotenuse BD = hypotenuse CD triangle is parallel to the third side 1. AAA similarity If the corresponding angles of two B Q
[QD is the mid-point of BC ] and is half of the third side. P Q triangles are equal then their corresponding sides are
x a
∴∆BED ≅ ∆CFD [by RHS congruency] proportional and hence the two triangles are similar.
Here, P and Q are mid-point of AB
⇒ ∠B = ∠C ⇒ AC = AB [sides opposite to equal angles] 1 Corollary AA similarity If two angles of one
c
and AC. So, PQ = BC. B C A P b
C
2 triangle are equal to the corresponding two angles of
Similar Figures AB || PQ
Theorem 4 If two triangles are equiangular, then another triangle, then the two triangles are similar.
The geometrical figures having the same shape, but different the ratio of their corresponding sides is the same as ∴ ∆ABC ~ ∆PQC
sizes are known as similar figures. 2. SSS similarity If the corresponding sides of two
the ratio of the corresponding altitudes. triangles are proportional, then their corresponding ∴
PC PQ
= ⇒
b
=
a
• The congruent figures are always similar, but two similar A P AC AB c+ b x
figures need not be congruent. angles are equal, and hence the two triangles are
similar. a ( c + b) ac
e.g. Any two circles are similar. Any two rectangles are ∴ x= = +a
b b
similar. 3. SAS similarity If one angle of a triangle is equal to
the corresponding angle of the other triangle and the Areas of Similar Triangles
Similar Triangles B D C Q S R sides including these angles are proportional, then
Theorem 1 The ratio of areas of two similar triangles
Two triangles are said to be similar to each other, if Here, ∆ ABC ~ ∆ PQR and AD and PS are altitude the two triangles are similar.
is equal to the square of the ratio of their corresponding
(i) their corresponding sides are proportional. on BC and QR, respectively, then 4. RHS similarity If hypotenuse and one side of a right sides.
BC AD
(ii) their corresponding angles are equal. = triangle are proportional to the hypotenuse and A P
QR PS
A P corresponding side of other right triangle then the
Theorem 5 If two triangles are equiangular, then two triangles are similar.
the ratio of the corresponding sides is the same as
B C Q R the ratio of the corresponding angle bisector EXAMPLE 7. If AB = 4.7, BC = 8.9, CA = 11.5, then EA is B C Q R

Here, ∆ABC and ∆PQR are similar triangles. segments. A Here, ∆ ABC ~ ∆ PQR
∴ ∠A = ∠P, ∠B = ∠Q, ∠C = ∠R Area ( ∆ ABC ) AB 2 BC 2 AC 2
A R 4.7 = = =
AB BC AC Area ( ∆ PQR) PQ2 QR2 PR2
and = = B E 11.5
PQ QR PR Theorem 2 The areas of two similar triangles is equal
Then, ∆ABC ~ ∆PQR to the ratio of the squares of corresponding altitudes.
8.9
Here, ∆ ABC ~ ∆ PQR
where, symbol ~ is read as, ‘is similar to’. B D C S P T C
A P
Here, ∆ ABC and ∆ RST are equiangular/similar and a. 6.07 b. 3.97
Some Results on Similar Triangles AD, RP are the angle bisectors of ∠A and ∠R, then c. 2.37 d. None of these
Theorem 1 If a line is drawn parallel to one side of a BC AD Sol. b. Since, BE is the bisector of ∠ABC.
=
triangle to intersect the other two sides in distinct point, ST RP AB AE 4 .7 AE
then it divides these sides in the same ratio. It is also called ∴ = ⇒ = [Q EC = AC − AE ]
Theorem 6 If two triangles are equiangular, then BC EC 8.9 115. − AE B D C Q S R
basic proportionality theorem.
the ratio of the corresponding sides is the same as ⇒ AE = 397
. Area ( ∆ ABC ) AD 2
AD AE A Then , =
Here, DE | | BC, then = the ratio of the corresponding medians.
Area ( ∆ PQR) PS 2
DB EC A P EXAMPLE 8. In the given triangle, AB is parallel to
AD AE
or = D E PQ. AP = c, PC = b, PQ = a, AB = x. What is the value of Theorem 3 The areas of two similar triangles is equal
AB AC x? to the ratio of the squares of the corresponding
AB AC medians.
or = B C B Q
DB EC A P
A
Theorem 2 The internal bisector of an B D C Q S R
angle of a triangle divides the opposite sides Here, ∆ ABC and ∆ PQR are equiangular and AD, PS
internally in the ratio of the sides containing are the medians, then C
A P b
the angle. BC AD s

= ab bc B D C Q S R
Here, AD is internal bisector of ∠A, then B D C QR PS a. a + b. a +
c a
AB BD Here, ∆ ABC ~ ∆ PQR ,
= c. b +
ca
d. a +
ac
AC DC Note: If two triangles are similar, then ratio of their Area (∆ ABC ) AD 2
corresponding sides is same as ratio of their perimeters. b b then =
Area (∆ PQR) PS 2
MATHEMATICS Triangles 255 256 CDS Pathfinder

Theorem 4 The areas of two similar triangles is equal EXAMPLE 10. In a given figure, QR is parallel to AB Converse of Pythagoras theorem B
to the ratio of squares of the corresponding angle and DR is parallel to QB. What is the number of distinct
bisector segments. pairs of similar triangles? In a triangle, if the square of one side is equal to the
A P P sum of square of the other two sides then angle D L
opposite to the first side is a right angle i.e. In ∆ABC, if 11 m
D
AC 2 = AB 2 + BC 2 , then ∠B = 90 °. 6m
Q R
EXAMPLE 12. Two poles of heights 6m and 11m
stand vertically on a plane ground at a distance of C 12 m A
B X C Q Y R
A Bs
12m from each other. The distance between their Draw DL ⊥ AB
Here, ∆ ABC ~ ∆ PQR a. 1 b. 2 c. 3 d. 4 tops is Then, BL = AB − AL = AB − CD = (11− 6) m = 5m
Area (∆ ABC ) AX 2 Sol. c. Since, QR is parallel to AB.
a. 9 m b. 10 m and DL = AC = 12m
So, here = c. 13 m d. 15 m
Area (∆ PQR) PY 2 ∴ ∆PQR ~ ∆PAB In right angled ∆BDL,
Sol. c. Let AB and CD be two poles such that AB = 11m, BD 2 = DL2 + BL2 = 52 + 122 = 169
Theorem 5 If the areas of two similar triangles are Also, DR is parallel to QB., ∆PQB ~ ∆PDR
equal, then the triangles are congruent. CD = 6 m and AC = 12 m. ⇒ BD = 169 = 13m
Again, DR || QB and QR || AB,
or ∴ ∆DQR ~ ∆QAB
Equal and similar triangles are congruent.
Theorem 6 The line segments is joining the mid-points
of the sides of a triangle form four triangles, each of
which is similar to the original triangle.
EXAMPLE 11. In a triangle, a line XY is drawn parallel
to BC meeting AB in X and AC in Y. The area of the
∆ ABC is 2 times the area of the ∆ AXY. In what ratio X
PRACTICE EXERCISE
A divides AB?
1. If the bisector of an angle of a triangle bisects 7. Let ABC be an isosceles triangle in which AB = AC
a. 1: 2 b. 2 : 1 c. ( 2 − 1) : 1 d. 1: ( 2 − 1) the opposite side, then the triangle is and BD ⊥ AC. Then, BD 2 − CD 2 is equal to
F E Area ( ∆ ABC) AB 2 (a) equilateral (b) isosceles A
Sol. d. Q =
Area (∆ AXY) AX 2 (c) scalene (d) right angled triangle
B D C A 2. The line segments joining the mid-points of the D
Here, D, E and F are mid-point of BC, AC and AB. Then, sides of a triangle form four triangles each of
∆ AFE, ∆ FBD, ∆ EDC and ∆ DEF is similar to ∆ ABC. X Y which is
(a) similar to the original triangle
Here, also B C
2 (b) congruent to the original triangle
1  (a) 2DC ⋅ AD (b) 2 AD ⋅ BC (c) 3DC ⋅ AD (d)
1
AD ⋅ DC
 AB B C (c) an equilateral triangle (d) an isosceles triangle
Area (∆ DEF ) DE 2 2  1 2
= = = 2 Area (∆ AXY) 3. In a ∆ ABC , BD and CE are perpendicular on AC 8. D and E are the points on the sides AB and
Area (∆ ABC ) AB 2 AB 2 4 AB 2 2 AB 2 AB
⇒ = ⇒ = , = 2 and AB, respectively. If BD = CE , then the AC respectively of a ∆ ABC and AD = 8 cm,
Area (∆ AXY) AX 2 1 AX 2 AX
So, area ( ∆ DEF ) : area ( ∆ ABC ) = 1 : 4 ∆ ABC is DB = 12 cm, AE = 6 cm and EC = 9 cm, then BC is
⇒ AB = 2 AX ⇒ AX + BX = 2 AX (a) equilateral (b) isosceles (c) right angled (d) scalene equal to
EXAMPLE 9. In the figure given below, BC is parallel ⇒ BX = A X ( 2 − 1) 2 5 3 2
to DE and DE : BC = 3 : 5. What is the ratio of area of AX 1 4. If the length of hypotenuse of a right angled (a)
5
DE (b)
2
DE (c)
2
DE (d)
3
DE
the ∆ABC to that of ∆ADE? ∴ = triangle is 5 cm and its area is 6 cm 2, then the
BX 2 −1
A length of the remaining sides are 9. A vertical stick 15m long casts a shadow 12m
So, X divides AB in 1 : ( 2 − 1). long on the ground. At the same time, a tower
(a) 1 cm and 3 cm (b) 3 cm and 2 cm
(c) 3 cm and 4 cm (d) 4 cm and 2 cm casts a shadow 50m long on the ground. The
D E
PYTHAGORAS THEOREM 5. ∆ ABC is such that AB = 3 cm, BC = 2 cm and
height of the tower is
(a) 60 m (b) 62 m (c) 62.5 m (d) 63 m
B C In a right angled triangle, the square of the hypotenuse is AC = 2.5 cm. ∆ DEF is similar to ∆ ABC. If
equal to the sum of the square of the other two sides. EF = 4 cm, then the perimeter of ∆ DEF is 10. The areas of two similar triangles are 81 cm 2
a. 3 : 1 b. 5 : 3 c. 9 : 2 d. 25 : 9 and 49 cm 2, respectively. The ratio of their
i.e. In ∆ABC , if ∠B = 90 ° , then AB 2 + BC 2 = AC 2 (a) 5 cm (b) 7.5 cm (c) 15 cm (d) 18 cm
Sol. d. Q Given, DE : BC = 3 : 5 corresponding heights is
A 6. A soldier goes to a warfield and runs in the (a) 9 : 7 (b) 7 : 9 (c) 6 : 5 (d) 81 : 49
Since, DE || BC ⇒ ∠ADE = ∠ABC following manner. From the starting point, he
and ∠AED = ∠ACB goes West 25 m, then due North 60 m, then due 11. If D and E are points on the sides AB and AC,
∴ ∆ABC ~ ∆ADE [by AA similarity] East 80 m, and finally due South 12 cm. The respectively of a ∆ ABC such that DE|| BC. If
Area of ∆ABC  BC  2 25 distance between the starting point and the AD = x, DB = x − 2, AE = x + 2 and EC = x − 1.
∴ =  = or, 25 : 9 finishing point is The value of x is
Area of ∆ADE  DE  9 B C
(a) 177 m (b) 103 m (c) 83 m (d) 73 m (a) 2.5 (b) 2 (c) 3 (d) 4
MATHEMATICS Triangles 257 258 CDS Pathfinder

12. In the adjoining figure, ABCD is 18. In ∆ ABC , ∠C = 90° and CD ⊥ AB, also ∠A = 65°, (a) 60°, 75°, 75°, 60° (b) 50°, 75°, 75°, 65°
A B 24. In ∆ABC , AD ⊥ BC, then
a trapezium in which AB||CD (c) 60°, 70°, 60°, 70° (d) 60°, 60°, 70°, 70°
O then ∠CBA is equal to (a) AB2 − BD2 = AC 2 − CD2 (b) AB2 + BD2 = AC 2 − CD2
and its diagonals intersect at O. C 31. In the given figure, AB||CD , ∠PTB = 55° and
If AO = ( 3x − 1), OC = ( 5x − 3), (c) AB2 + BD2 = AC 2 + CD2 (d) AB2 + AC 2 = BD2 + DC 2
∠DVS = 45°, then what is the sum of the
BO = ( 2x + 1) and OD = (6x − 5), D
then x is equal to
C 25. ∆ ABC is a right angled at C and P is the length measures of ∠CUQ and ∠RTP?
65° of the perpendicular from C to AB. If BC = a, R P
(a) 1 (b) 2 (c) 3 (d) 4 A D B AC = b, AB = c, then 55°
A B
a p 1 1 1 T
13. In the adjoining figure, AE is the bisector of (a) 25° (b) 35° (c) 65° (d) 40° (a) = (b) pc = ab (c) + = (d) None of these
exterior ∠CAD meeting BC produced in E. If b c a b ab
AB = 10 cm, AC = 6 cm and BC = 12 cm, then CE 19. The angles of a triangle are in the ratio 2 : 3 : 4.
The angles of triangle are, respectively
26. ABC is a triangle and the perpendicular drawn C
U V 45°
D
is equal to from A meets BC in D. If AD 2 = BD ⋅ DC, then Q S
D (a) 30°, 60°, 90° (b) 40°, 60°, 80°
which one of the following is correct? (a) 180° (b) 135° (c) 110° (d) 100°
A (c) 60°, 40°, 80° (d) 20°, 60°, 80°
(a) ABC must be an obtuse angled triangle
20. In figure, D and E are points on sides AB, AC of (b) ABC must be an acute angled triangle 32. What is the value of x in the figure given below?
∆ ABC such that DE|| BC. If ∠B = 30° and (c) Either ∠B ≥ 45° or ∠C ≥ 45° (d) BC 2 = AB2 + AC 2 A B
a c
B C E
∠A = 40°, then x , y and z are, respectively
A 27. If ∆ is the area of a right angled triangle and b is b
(a) 6 cm (b) 12 cm (c) 18 cm (d) 20 cm one of the sides containing the right angle, then x
40°
14. If D , E and F are respectively the mid-points of what is the length of the altitude on the hypotenuse? C
x° z° 2 ∆b 2∆ b2
sides BC , AC and AB of a ∆ ABC. If EF = 3 cm, D
E (a) (b) (a) b − a − c (b) b − a + c (c) b + a − c (d) π − (a + b + c )
FD = 4 cm and AB = 10 cm, then DE , BC and CA, b + 4∆
4 2
b + 4∆
4 2
33. In the given figure, what is the value of x?
respectively will be equal to y° 2 ∆b 2 2 ∆2 b 2
10 30° (c) (d) B C
D E
(a) 6, 8 and 20 cm (b) , 9 and 12 cm B C b 4 + 4∆2 b 4 + ∆2
3 2x 120°
(c) 4, 6 and 8 cm (d) 5, 6 and 8 cm (a) 30°, 110°, 110° (b) 30°, 105°, 105° 4 B D
(c) 30°, 85°, 85° (d) 30°, 95°, 95°
28. In figure, AB||CD. If x = y x°
y° z°
3 x
15. In ∆ PQR , ∠Q = 3a, ∠P = a , ∠R = b and 3
21. In figure, AB, EF and CD are parallel lines. Given and y = z, then the value of
3b − 5a = 30, then the triangle is 8
C A
that GE = 5 cm, GC = 10 cm and DC = 18 cm, then A (a) 30° (b) 40° (c) 45° (d) 60°
(a) scalene (b) isosceles x is
EF is equal to
(c) equilateral (d) right angled D (a) 48° (b) 96° (c) 36° (d) None of these 34. PQR is a triangle right angled at Q. If X and Y
16. In ∆ ABC show in the figure are the mid-points of the sides PQ and QR
B A 29. In the given figure, AB||CD , then the values of
respectively, then which one of the following is
∠A = 90°. Let D be a point on x, y and z are, respectively not correct?
BC such that BD : DC = 1 : 3. If D E
M G B D 2
DM and DL, respectively are (a) RX 2 + PY 2 = 5 XY 2 (b) RX 2 + PY = XY 2 + PR 2
perpendicular on AB and AC, z° (c) 4 (RX 2 + PY 2 ) = 5PR 2 (d) RX 2 + PY 2 = 3 (PQ 2 + QR 2 )
B x°
then DM and LC are in the A C F C P
ratio of
L
(a) 11 cm (b) 5 cm (c) 6 cm (d) 9 cm 35. In the given figure, AB is parallel to CD. If
y° ∠DCE = x and ∠ABE = y, then what is ∠CEB
(a) 1 : 3 (b) 1 : 2 (c) 1 : 1 (d) 4 : 1 22. In the given figure, PQ > PR. QS and RS are the 75°
35
° equal to?
bisectors of ∠Q and ∠R respectively, then which C D
17. In a right angled ∆ ABC, right angled at B, if P of the following is correct?
x
A C
and Q are points on the sides AB and AC P B
(a) 75°, 35°, 80° (b) 70°, 35°, 60° A
respectively, then
(c) 35°, 70°, 75° (d) 70°, 35°, 80° y E
A
30. In the given figure, AB||CD , and EF ||GH . The (a) y − x (b) (x + y) / 2 (c) x + y − ( π / 2 ) (d) x + y − π
S
P
values of x, y , z and t are respectively 36. In the figure given below, what is the sum of the
G angles formed around A, B, C except the angles of
E
Q R the ∆ ABC?
P Q
B Q C (a) SQ < SR (b) SQ = SR A
z° B A
(c) SQ > SR (d) None of these 110° y°
(a) AQ 2 + CP 2 = 2 ( AC 2 + PQ 2 )
(b) 2 ( AQ 2 + CP 2 ) = AC 2 + PQ 2 23. In a ∆ABC, AB = AC and 60° < A < 90°. If AB = c,
AC = b and BC = a, then which of the following is x°
t° S
(c) AQ 2 + CP 2 = AC 2 + PQ 2 C R
D
correct? B C
1 (a) b < a < 2 b (b) c / 3 < a < 3c
(d) AQ + CP = ( AC + PQ ) 60° H
2 (c) b < a < b 3 (d) c < a < c 2 F (a) 360° (b) 720° (c) 900° (d) 1000°
MATHEMATICS Triangles 259 260 CDS Pathfinder

37. In the given figure, ABC is A 43. In the figure, ∠B = 38°, A PREVIOUS YEARS’ QUESTIONS 54. I. Let the side DE of a ∆DEF be divided at S, so
an equilateral triangle of side AC = BC and AD = CD. What 1 1 1 DS
=
1
⋅ If a line through S parallel to EF
length 30 cm. XY is parallel is the value ∠D? 48. In a ∆ ABC, ∠ A + ∠ C + ∠B = 80°, then that
DE
2 3 2 2
to BC, XP is parallel to AC (a) 26° (b) 28° 38° what is the value of ∠C ? e 2012 II meets DF at T, then the area of ∆DEF is twice
and YQ is parallel to AB. If X Y B C D
(c) 38° (d) 52° (a) 35° (b) 40° (c) 60° (d) 70° the area of the ∆DST.
( XY + XP + YQ ) is 40 cm,
B C II. The areas of the similar triangles are proportional
then what is PQ equal to? P Q 44. Consider the following statements in respect of 49. In a ∆ ABC, side AB is extended beyond B, side
any triangle. to the squares of the corresponding sides.
(a) 5 cm (b) 12 cm BC beyond C and side CA beyond A. What is the
(c) 15 cm (d) None of these I. The three medians of a triangle divide it into six sum of the three exterior angles? e 2012 II Which one of the following is correct in respect of
triangles of equal area. (a) 270° (b) 305° (c) 360° (d) 540° the above statements? e 2013 II
38. O is any point on the X
II. The perimeter of a triangle is greater than the (a) Both statements I and II are true and statement II is
bisector of the acute sum of the lengths of its three medians. 50. In the given figure, ABC is a triangle, BC is the correct explanation of statement I
angle ∠ XYZ. The line P parallel to AE. If BC = AC, then what is the (b) Both statements I and II are true but statement II is
OP is parallel to ZY . O Which of the statement(s) given above is/are value of ∠ CAE ? e 2012 II not the correct explanation of statement I
Then, ∆ YPO is Y Z correct? (c) Statement I is true but statement II is false
C
(a) Only I (b) Only II (c) Both I and II (d) Neither I nor II B (d) Statement II is true but statement I is false
(a) scalene 65°
(b) isosceles but not right angled 45. Consider the following in respect of C D 95° 55. Let ABC be a triangle with AB = 3 cm and
(c) equilateral the given figure : AC = 5 cm. If AD is a median drawn from the
(d) right angled and isosceles I. ∆ DAC ~ ∆ EBC E D A
E vertex A to the side BC, then which one of the
II. CA /CB = CD /CE following is correct? e 2013 II
39. The median BD of the ∆ ABC meets AC at D. If (a) 20° (b) 30° (c) 40° (d) 50°
A B (a) AD is always greater than 4 cm but less than 5 cm
1 III. AD /BE = CD /CE
BD = AC, then which one of the following is 51. In the given figure, AB is parallel to CD. (b) AD is always greater than 5 cm
2 Which of the statement(s) given above is/are
correct? correct? ∠ABC = 65° , ∠CDE = 15° and AB = AE. What is (c) AD is always less than 4 cm
(a) II and III (b) I and II (c) I and III (d) All of these the value of ∠AEF ? e 2012 II (d) None of the above
(a) ∠ACB = 1 right angle (b) ∠BAC = 1 right angle
(c) ∠ABC = 1 right angle (d) None of these 46. Consider the following statements : A F 56. In the figure given above, ∠PQR = 90° and QL is
I. Let PQR be a triangle in which PQ = 3 cm, a median, PQ = 5 cm and QR = 12 cm. Then, QL
40. ABC is a triangle, X is a point outside the
QR = 4 cm and RP = 5 cm. If D is a point in the is equal to e 2013 II
∆ ABC such that CD = CX , where D is the point P
of intersection of BC and AX and ∠BAX = ∠XAC. plane of the ∆ PQR such that D is either outside
Which one of the following is correct? it or inside it, then B C
E
L
(a) ∆ABD and ∆ACX are similar (b) ∠ABD < ∠ACD DP + DQ + DR > 6 cm
(c) AC = CX (d) ∠ADB > ∠DXC II. PQR is a right angled triangle.
π
41. In the figure given, ∠ABD = ∠PQD = ∠CDQ = . Which one of the following is correct in respect of D Q R
2 the above two statements?
(a) 30° (b) 35° (c) 40° (d) 45° (a) 5 cm (b) 5.5 cm (c) 6 cm (d) 6.5 cm
If AB = x, PQ = z and CD = y, then which one of (a) Both statements I and II are individually true and
the following is correct?
A
statement II is the correct explanation of statement I 52. The angles x° , a ° , c° and ( π − b)° are indicated in 57. In a ∆ABC, ∠BCA = 90° and CD is perendicular
1 1 1 C (b) Both statements I and II are individually true but the figure given below : e 2012 II to AB. If AD = 4 cm and BD = 9 cm, then the
(a) + = statement II is not the correct explanation of statement I
x y z x P E value of DC will be e 2013 II
y (c) Statement I is true and statement II is false x°
1 1 1 z D (a) 18 cm (b) 20 cm (c) 65 cm (d) 6 cm
(b) + = (d) Statement I is false and statement II is true
x z y B Q D 58. ABC is a triangle, where BC = 2 AB , ∠C = 30°
1 1 1 47. I. Let LMN be a triangle. Let P , Q be the (π−b)°
(c) + = c° a° and ∠A = 90°. The magnitude of the side AC is
z y x A T
mid-points of the sides LM , LN , respectively. If B C e 2013 II
1
(d) +
1 2
= PQ 2 = MP 2 + NQ 2, then LMN is a right angled Which one of the following is correct ? e 2012 II 2 BC 3 BC BC 3 BC
x y z triangle at L. (a) (b) (c) (d)
(a) x° = a° + c ° − b ° (b) x° = b ° − a° − c ° 3 4 3 2
42. ∆ PQR is right angled at Q, PR = 5 cm and II. If in a ∆ ABC , AB > BC + CA , then ∠ ACB is
2 2 2
(c) x° = a° + b ° + c ° (d) x° = a° − b ° − c °
obtuse. 59. Let ABC be an equilateral triangle. If the side
QR = 4 cm. If the lengths of sides of another 53. Consider the following statements: BC is produced to the point D so that BC = 2 CD,
∆ ABC are 3 cm, 4 cm and 5 cm, then which one Which of the following is correct in the light of
then AD 2 is equal to e 2013 II
the above statements? I. If G is the centroid of ∆ABC, then GA = GB = GC.
of the following is correct? (a) 3 CD2 (b) 4 CD2 (c) 5 CD2 (d) 7 CD2
(a) Area of ∆ PQR is double that of ∆ ABC (a) Both statements I and II are individually true and II. If H is the orthocentre of ∆ABC, then
statement II is the correct explanation of statement I HA = HB = HC. 60. ABC is a right angled triangle such that
(b) Area of ∆ ABC is double that of ∆ PQR
∠Q (b) Both statements I and II are individually true but Which of the statement(s) given above is are AB = a − b, BC = a and CA = a + b . D is a point
(c) ∠B = statement II is not the correct explanation of statement I
2 correct? e 2013 II on BC such that BD = AB. The ratio of BD : DC
(c) Statement I is true and statement II is false
(d) Both triangles are congruent (d) Statement I is false and statement II is true
(a) Only I (b) Only II for any value of a and b is given by e 2013 II
(c) Both I and II (d) Neither I nor II (a) 3 : 2 (b) 4 : 3 (c) 5 : 4 (d) 3 : 1
MATHEMATICS Triangles 261 262 CDS Pathfinder

I. AD > A′ D′, BE > B′ E′ and CF > C′ F ′ are always


61. The side BC of a ∆ABC is produced to D, 71. If triangles ABC and DEF are similar such that Select the correct answer using the codes given
true.
bisectors of the ∠ABC and ∠ACD meet at P. If 2AB = DE and BC = 8 cm, then what is EF equal below. e 2016 I
∠BPC = x ° and ∠BAC = y °, then which one of the to? e 2014 I (a) I and II (b) II and III AB2 + BC 2 + CA 2 A′ B′ 2 + B′ C′ 2 + C′ A′ 2
II. =
following option is correct? e 2013 II (a) 16 cm (b) 12 cm (c) 10 cm (d) 8 cm (c) I and III (d) All of these AD 2 + BE 2 + CF 2 A′ D′ 2 + B′ E′ 2 + C′ F ′ 2
(a) x° = y° (b) x° + y° = 90° 72. The sides of a right angled triangle are equal to 80. ABC is a triangle right angled B
Which one of the following is correct in respect of
(c) x° + y° = 180° (d) 2x° = y° three consecutive numbers expressed in at C as shown below. Which the above statements?
62. The side AC of a ∆ABC is produced to D such centimeters. What can be the area of such a one of the following is correct? (a) Both statement I and statement II are true and
that BC = CD. If ∠ACB is 70°, then what is triangle? e 2014 I e 2016 I Q statement II is the correct explanation of statement I
∠ADB equal to? e 2013 II (a) 6 cm 2 (b) 8 cm 2 (c) 10 cm 2 (d) 12 cm 2 (a) AQ 2 + AB2 = BP 2 + PQ 2 (b) Both statement I and statement II are true but statement
(a) 35° (b) 45° (c) 70° (d) 110°
73. The three sides of a triangle are 15, 25, x units. (b) AQ 2 + PQ 2 = AB2 + BP 2 II is not the correct explanation of statement I
(c) Statement I is true but statement II is false
63. The heights of two trees are x and y, where x > y. Which one of the following is correct? e 2014 I (c) AQ 2 + BP 2 = AB2 + PQ 2 A C
P (d) Statement I is false but statement II is true
The tops of the trees are at a distance z a part. If (a) 10 < x < 40 (b) 10 ≤ x ≤ 40 (d) AQ 2 + AP 2 = BK 2 + KQ 2
s is the shortest distance between the trees, then (c) 10 ≤ x < 40 (d) 10 < x ≤ 40 84. ABC and DEF are similar triangles. If the ratio
what is s2 equal to? e 2013 II 81. In the given figure, AD = CD = BC. What is the of side AB to side DE is ( 2 + 1) : 3, then the
74. In a ∆ABC, if ∠B = 2 , ∠C = 2∠A. Then, what is
(a) x + y − z − 2 x y
2 2 2
(b) x + y − z
2 2 2
value of ∠CDB? e 2016 I ratio of area of ∆ABC to that of ∆DEF is e 2016 I
the ratio of AC to AB ? e 2014 II
(c) x + y + z − 2 x y
2 2 2
(d) z − x − y + 2 x y
2 2 2
C (a) (3 − 2 2 ) : 3 (b) (9 − 6 2 ) : 2
(a) 2 : 1 (b) 3 : 1 (c) 1 : 1 (d) 1 : 2 96°
(c) 1 : (9 − 6 2 ) (d) (3 + 2 2 ) : 2
64. In a ∆ABC, ∠B = 90° and ∠C = 2∠ A, then what 75. Three straight lines are drawn through the three
is AB 2 equal to? e 2013 II vertices of a ∆ABC, the line through each vertex 85. Let ∆ABC and ∆DEF be such that ∠ABC = ∠DEF,
(a) 2 BC 2 (b) 3BC 2 (c) 4BC 2 (d) 5BC 2 being parallel to the opposite side. The ∆DEF is ∠ACB = ∠DFE and ∠BAC = ∠EDF. Let L be the
bounded by these parallel lines. e 2014 II mid-point of BC and M be the mid-point of EF.
65. PQR is an equilateral triangle. O is the point of Consider the following statements in respect of
A
D
B
intersection of altitudes PL, QM and RN . If Consider the following statements e 2016 I
the ∆DEF. (a) 32° (b) 64°
OP = 8 cm, then what is the perimeter of the (c) 78° I. ∆ABL and ∆DEM are similar.
I. Each side of ∆DEF is double the side of ∆ABC to
∆PQR ? e 2013 II (d) Cannot be determined due to insufficient data II. ∆ALC is congruent to ∆DMF even, if AC ≠ DF .
which it is parallel.
(a) 8 3 cm (b) 12 3 cm (c) 16 3 cm (d) 24 3 cm
II. Area of ∆DEF is four times the area of ∆ABC. 82. ABC is an equilateral triangle and X, Y and Z Which one of the following is correct in respect of
66. ∆DEF is formed by joining the mid-points of Which of the statement(s) given above is/are correct? are the points on BC, CA and AB respectively, the above statements?
the sides of ∆ABC. Similarly, a ∆PQR is formed (a) Only I (b) Only II such that BX = CY = AZ . Which of the following (a) Both statement I and statement II are true and
by joining the mid-points of the sides of the (c) Both I and II (d) Neither I nor II is/are correct? e 2016 I statement II is the correct explanation of statement I
∆DEF. If the sides of the ∆PQR are of lengths (b) Both statement I and statement II are true but
76. In a ∆ ABC, AD is the median through A and E I. XYZ is an equilateral triangle.
1, 2 and 3 units, what is the perimeter of the statement II is not the correct explanation of statement I
II. ∆XYZ is similar to ∆ABC.
∆ABC ? e 2013 II is the mid-point of AD and BE produced meets (c) Statement I is true but statement II is false
(a) 18 units (b) 24 units AC at F. Then, AF is equal to e 2014 II Select the correct answer using the codes given (d) Statement I is false but statement II is true
(c) 48 units (d) Cannot be determined (a) AC/5 (b) AC/4 (c) AC/3 (d) AC/2 below.
(a) Only I (b) Only II (c) Both I and II (d) Neither I nor II
86. ABC is a triangle in which D is the mid-point of
67. E is the mid-point of the median AD of a ∆ABC. 77. The point O is equidistant from the three sides BC and E is the mid-point of AD.
If BE produced meets the side AC at F, then CF of a ∆ABC. 83. Let ABC and A′ B′ C ′ be two triangles in which I. The area of ∆ABC is equal to four times the area
is equal to e 2013 II Consider the following statements: e 2015 II AB > A′ B′, BC > B′C ′ and CA > C ′ A′. Let D , E and of ∆BED.
(a) AC /3 (b) 2 AC / 3 (c) AC /2 (d) None of these I. ∠OAC + ∠OCB + ∠OBA = 90° F be the mid-points of the sides BC, CA and AB, II. The area of ∆ADC is twice the area of ∆BED.
II. ∠BOC = 2∠BAC respectively. Let D′, E′ and F ′ be the mid-points
68. If AD is the internal angular of ∆ABC with Select the correct answer using the codes given
III. The perpendiculars drawn from any point on OA of the sides B′ C ′, C ′ A′ and A′ B′, respectively.
AB = 3 cm and AC = 1 cm, then what is BD : BC to AB and AC are always equal
below. e 2016 I
equal to? e 2014 I
Consider the following statements e 2016 I (a) Only I (b) Only II (c) Both I and II (d) Neither I nor II
Which of the above statements are correct?
(a) 1 : 3 (b) 1 : 4 (c) 2 : 3 (d) 3 : 4
(a) I and II (b) II and III (c) I and III (d) All of these
69. In a ∆ABC, AD is perpendicular to BC and BE is 78. An equilateral ∆BOC is drawn inside a square
ANSWERS
perpendicular to AC. Which of the following is ABCD. If ∠AOD = 2θ, what is tan θ equal to? 1 b 2 a 3 b 4 c 5 c 6 d 7 a 8 b 9 c 10 a
correct? e 2014 I e 2015 II 11 d 12 b 13 c 14 d 15 d 16 a 17 c 18 a 19 b 20 a
(a) CE × CB = CA × CD (b) CE × CA = CD × CB (a) 2 − 3 (b) 1 + 2 (c) 4 − 3 (d) 2 + 3
21 d 22 c 23 d 24 a 25 b 26 d 27 a 28 a 29 c 30 d
(c) AD × BD = AE × BE (d) AB × AC = AD × BE
79. In a ∆PQR, point X is on PQ and point Y is on 31 b 32 a 33 d 34 d 35 d 36 c 37 d 38 b 39 c 40 a
70. The sides of a triangle are in geometric PR such that XP = 1 . 5 units, XQ = 6 units, 41 a 42 d 43 b 44 c 45 d 46 b 47 b 48 c 49 c 50 d
progression with common ratio r < 1. If the PY = 2 units and YR = 8 units. Which of the
51 b 52 c 53 d 54 a 55 c 56 d 57 d 58 d 59 d 60 d
triangle is a right angled triangle, the square of following are correct?
common ratio is given by 61 d 62 a 63 d 64 b 65 d 66 b 67 b 68 d 69 b 70 b
e 2014 I I. QR = 5XY II. QR is parallel to XY .
5+1 5 −1 3+1 3 −1 III. ∆PYX is similar to ∆PRQ.
71 a 72 a 73 a 74 a 75 c 76 c 77 c 78 d 79 d 80 c
(a) (b) (c) (d)
2 2 2 2 81 b 82 c 83 b 84 c 85 c 86 c
MATHEMATICS Triangles 263 264 CDS Pathfinder

11. (d) As DE || BC , by basic proportionality A So, angles are 2x = 40°,


HINTS AND SOLUTIONS theorem,
A F E
3x = 60°, 4x = 80°.
20. (a) In ∆ABC , ∠A + ∠B + y = 180°
2
x x+2 y = 180° − ( 40 + 30)° = 110°
1. (b) Let ABC is any triangle and AD is  12  7. (a) As ADB is a right angled triangle. B D C ∠ADE = ∠ABC [corresponding angle]
52 =   + p 2 D E
the bisector of angle A.
 p So, AB 2 = AD 2 + BD 2 ∴ DE =
1 1
AB = × 10 = 5 cm x = 30°
Also, BD = DC (given) ⇒ AC 2 = AD 2 + BD 2 [Q AB = AC ] x–2 x–1 2 2
144 Similarly, y = z = 110°
A ⇒ 25 = 2 + p 2 ⇒ ( AD + DC )2 = AD 2 + BD 2 EF =
1
BC ⇒ BC = 2EF
p ⇒ AD 2 + DC 2 + 2 AD ⋅ DC 2 21. (d) In ∆GEF and ∆GCD, we have
B C
25 p 2 = 144 + p 4 = AD 2 + BD 2 AD AE x x+ 2 = 2 × 3 = 6 cm ∠EFG = ∠GDC [alternate angles]
⇒ p 4 − 25 p 2 + 144 = 0 ⇒ BD 2 − CD 2 = 2DC ⋅ AD = ⇒ = 1
DB EC x−2 x−1 DF = AC ⇒ AC = 2 × DF ∠EGF = ∠CGD
⇒ p 4 − 16 p 2 − 9 p 2 + 144 = 0 8. (b) As in ∆ ADE and ∆ ABC , 2
[vertically opposite angles]
B D C AD
=
8 2
= and
AE
=
6
=
2 ⇒ x ( x − 1) = ( x + 2) ( x − 2) = 2 × 4 = 8 cm
⇒ p 2 ( p 2 − 16) − 9 ( p 2 − 16) = 0 ∆GEF ~ ∆GCD [by AA similarity]
Q AD is the internal bisector of ∠A. AB 20 5 AC 15 5 ⇒ x 2 − x = x 2 − 2x + 2x − 4 15. (d) In ∆PQR, ∠ P + ∠ Q + ∠ R = 180°
⇒ ( p 2 − 9) ( p 2 − 16) = 0 A ∴
GE EF
=

AB
=
BD
⇒ x=4 ⇒ a + 3a + b = 180°
⇒ p = 3 or p = 4 CG DC
AC DC
Hence, the value of x is 4. ⇒ 4a + b = 180° …(i) 5 EF
AB BD Hence, other sides are 3 cm and 4 cm. Given, − 5a + 3b = 30° ⇒ =
⇒ = = 1 [QBD = DC given] D E …(ii)
10 18
AC DC 5. (c) As ∆ ABC ~ ∆ DEF 12. (b) As AB || CD and the diagonals of a Solving Eqs.(i) and (ii), we get a = 30°
trapezium divide each other 5 × 18
⇒ AB = AC AB AC BC and b = 60° ⇒ EF = = 9 cm
⇒ = = B C proportionally. 10
Hence, triangle is an isosceles triangle. DE DF EF ∴ ∠ P = 30° , ∠ Q = 90° and ∠ R = 60°
AD AE AO BO 22. (c) Given, In ∆PQR, PQ > PR
A D ∴ = So, = So, ∆ PQR is right angled triangle.
2. (a) The line segments joining the AB AC OC OD
16. (a) Consider ∆BMD and ∆DLC , ⇒ ∠PRQ > ∠PQR
mid-points of the sides of a triangle 2.5 and ∠A = ∠A [common] 3x − 1 2x + 1 [angle opp to larger side is greater]
form four triangles each of which is 3 ⇒ = As ∠ BMD = ∠ DLC = 90° [each]
∴ ∆ ADE ~ ∆ ABC 5x − 3 6x − 5 1 1
similar to the original triangle. Also, ∠ BDM = ∠ DCL corresponding ⇒ ∠PRQ > ∠PQR
[by SAS similarity] 2 2
A B 2 C E 4 F angle

DE
=
AD

DE 2
= (3x − 1) ( 6x − 5) = (5x − 3) ( 2x + 1) ⇒ ∠SRQ > ∠SQR ⇒ SQ > SR
BC 2 1 ∴∆BMD ~ ∆DLC [by AA similarity]
Q = = BC AB BC 5 ⇒ 18x2 − 15x − 6x + 5 BD DM BM [side opp to greater angle is larger]
F E EF 4 2 5 ∴ = =
⇒ BC = DE . = 10x2 + 5x − 6x − 3 DC LC DL 23. (d) Given, AB = AC
∴ DE = 2 AB = 2 × 3 = 6 cm 2 BD DM 1
⇒ 8x2 − 20x + 8 = 0 ⇒ = = A
B D C and DF = 2 × AC = 2 × 2.5 = 5 cm 9. (c) Let AB be a vertical stick and AC be DC LC 3
⇒ 4x − 10x + 4 = 0
2
Here, ∆ BDF ~ ∆ ABC ∴ Perimeter of ∆ DEF = ( 6 + 5 + 4) its shadow. Also, let PQ be a tower ∴ DM : LC = 1 : 3
having shadow PR . ⇒ 2( 2x − 5x + 2) = 0
2
Also, ∆ DEC , ∆ DEF = 15 cm 17. (c) In ∆ABC , by pythagoras theorem, c b
Sun Sun ⇒ 2( 2x2 − 4x − x + 2) = 0
∆ AFE ~ ∆ ABC Shortcut Method B Q A
Perimeter of ∆ABC ⇒ 2[ 2x( x − 2) − 1( x − 2) = 0
3. (b) As BD = EC ,
⇒ 2( 2x − 1)( x − 2) = 0
15 m

Perimeter of ∆DEF P B a C
∠AEC = ∠BDA = 90°each x
1
= Ratio of corresponding sides But as x = will make OC negative. ⇒ ∠ABC = ∠ACB
[Q BD⊥AC and CE⊥AB] 2
(3 + 2 + 2.5) 1 If ∠A = 60°, then
A ∴ = A C P R B Q C
Perimeter of ∆DEF 2 12 m 50 m ∴ x= 2 AC 2 = AB 2 + BC 2 …(i) ∆ABC is an equilateral triangle a = b = c
E ∴Perimeter of ∆DEF = 2 ( 75
. ) = 15 cm. ∠A = ∠P 13. (c) Exterior angle bisector theorem The And in ∆PBQ, If ∠A = 90°, then
D
∠B = ∠Q exterior bisector of an angle of a triangle PQ 2 = PB 2 + BQ 2 …(ii) ∆ABC is right angled triangle
6. (d) Let P be the starting point of his run, [Q Sun with tower and stick forms divides the opposite side externally in
then PT is the distance between the Adding Eqs. (i) and (ii), a 2 = b 2 + c 2 = c 2 + c 2 = 2c 2 [Q b = c ]
same angle] the ratio of the sides containing the
B C starting and the finishing point. AC 2 + PQ 2 = ( AB 2 + BC 2 ) ⇒a =
As, ∆ ABC ~ ∆ PQR angle. 2c
Also, ∠A = ∠A [common] + PB 2 + BQ 2
80 m [ ∴ By AA similarity] BE AB ∴ If 60°< A < 90°, then c < a < c 2
~ ∆ AEC R S Q = as AE is an exterior angle = ( AB 2 + BQ 2 ) + ( PB 2 + BC 2 )
∴ ∆BDA = AB AC 15 12
12 m N ∴ = ⇒ = CE AC AC + PQ = AQ + CP
2 2 2 2 24. (a) Here, in ∆ABC
[by AAS congruency] U x 50 bisector.
T PQ PR AB 2 = AD 2 + BD 2 …(i)
⇒ AB = AC [by cpct] 60 m 15 × 50 18. (a) In ∆BCA , ∠CAB = 65° and
⇒ x= = 62.5 m Let CE = x, BE = BC + EC = 12 + x In right angled ∆ADC , we have
12 m W E ∠ACB = 90° [Q ∠C = 90° given]
So, triangle is an isosceles triangle. 12 12 + x 10 AC 2 = AD 2 + CD 2 …(ii)
⇒ = ∠CBA = 180° − ( ∠BCA + ∠CAB )
4. (c) Let the other side be b and p. Q 25 m P Hence, the height of the tower is 62.5 m. x 6 Subtracting Eq. (ii) from Eq. (i),
S [by angle sum property of a triangle]
1 10. (a) Let the ratio of their corresponding ⇒ ( 12 + x) 6 = 10x = 180° − ( 65° + 90° ) AB 2 − AC 2 = BD 2 − CD 2
∴ b × p = 6 ⇒ b × p = 12 A
2 ∴ PU = RQ − ST = 60 − 12 = 48 m height be h : h . ⇒ 72 + 6x = 10x ⇒ 4x = 72 = 180° − 155° = 25°
1 2
12 UT = RS − QP = 80 − 25 = 55 m But the ratio of the areas of two similar
⇒ b= and ⇒ x = 18 cm 19. (b) Let the angles of a triangle be
p triangles is equal to the ratio of the
∴ In ∆ PUT, PT 2
= ( PU )2 + ( TU )2 14. (d) As the line joining the mid-points of 2x, 3x, 4x, then
squares of their corresponding heights.
Also, by pythagoras theorem any two sides or a triangle is parallel to 2x + 3x + 4x = 180°
∴PT = ( 48)2 + (55)2 = 2304 + 3025 h 2 81
h2 = b 2 + p2 ∴ 1 = ⇒ h : h =9: 7 the third side and is half of the third [by angle sum property of a triangle] B D C
= 5329 = 73 m h 2 49 1 2
side. 9x = 180° ⇒ x = 20°
2 AB 2 − BD 2 = AC 2 − CD 2
MATHEMATICS Triangles 265 266 CDS Pathfinder

25. (b) Since, c is the base and p is the 7 3  7 33. (d) Q ∠ABC = 180°−∠DBA = 180°−2x 35. (d) We produced line AB to meet line Also, XY + XP + YQ = 40 [given] Then, =
x BD
 z  + z = 180° ⇒ z + z = 180° …(i)
altitude of ∆ABC . 3 8  8 B C CE at M. ⇒ AX + XB + YQ = 40 z QD
C D E
Q y = 3 z  2x 120°
C x D [Q XY = AX , XP = XB] Since CD ||PQ
 8  ⇒ AB + YQ = 40 So, ∆ BCD ~ ∆ BPQ,
b z BQ
p
a
15z B ⇒ YQ = 40 − 30 = 10 cm ∴ =
= 180° ⇒ z = 96° x A M y BD
8 y E ∴ YQ = XP = 10 cm z BD − QD
3 A ∴ BP = CQ = 10 cm ⇒ =
A D c
B So, y = × 96° = 36° Since, AM || CD. y BD
8 and ∠ACB = 180° − ∠ACE ∴ PQ = 30 − BP − CQ z QD
1 4 ∠DCM = ∠BMC = x ⇒ = 1−
Here, area of ∆ABC = pc …(i) and x = × 36° = 48° = 180° − 120° = 60° = 30 − 10 − 10 = 10 cm y BD
2 3 [alternate angles] z z
In ∆ABC ,∠ABC + ∠ACB + ∠BAC = 180° 38. (b) As, OP || YZ ⇒ = 1−
1 CME is a straight line [from Eq. (i)]
Also, area of ∆ABC = ab …(ii) 29. (c) We have, x = 35° [alternate angles] ⇒ 180°−2x + 60°+ x = 180° ⇒ x = 60° ⇒∠POY = ∠OYZ [alternate angles] y x
2
z° = 75° ∴ ∠BME = π − x ⇒ ∠PYO = ∠POY z z 1 1 1
[alternate angles] ⇒ + =1 ⇒ + =
From Eqs. (i) and (ii), we get 34. (d) In ∆PQY , by pythagoras theorem, Also, ABM is a straight line. [since OY is angle bisector of ∠Y ] x y x y z
1 1 In ∆ABP, x + y + 75° = 180°
pc = ab ⇒ pc = ab PY 2 = PQ 2 + QY 2 ∴ ∠EBM = π − y ∴ PY = PO
2 2 y = 180°−( 75°+35° ) ⇒ y = 70° 42. (d) In right angle ∆ PQR , by pythagoras
P Now, in ∆BEM As ∠XYZ is an acute angle.
1 theorem,
26. (d) Hence, AD 2 = BD ⋅ DC Hence the value of ∴ ∠ XYZ < 45°
∠EBM + ∠BME + ∠BEM = π R
C x = 35° , y = 70° ,z = 75° X 2
⇒ π − y + π − x + ∠E = π ∴ ∠POY = ∠PYO < 45°
D 30. (d) x° = 60° [vertically opposite angles] ⇒ ∠E = x + y − π ∴ ∠YPO > 90° 4 cm 5 cm
Q R
y = 60° [corresponding angles] Y
36. (c) ∴ ∠A = 360° − Ext ∠A Hence, ∆PYO is an isosceles triangle but
90° 2
∠PRS = 110°
⇒ PY 2 = PQ 2 + 
A B [alternate angles] QR  not a right angled triangle. Q P
 A
AD DC ∠QRS + x° = 110° [Alternate angles]  2  39. (c) Here, we see that C
⇒ = QP 2 = (5)2 − ( 4)2 = 9 ⇒ QP = 3 cm
BD AD ∠QRS = 110° − 60° = 50° QY = YR = QR  CD = BD = DA In second ∆ABC whose sides are 3 cm,
∴ ∆ADB ~ ∆CDA ⇒ ∠BAD = ∠ACD ∴ t = 180° − ( y + ∠QRS )  2  …(i) This is possible D 4 cm and 5 cm. So, the sides of both
Now, ∠CAB = ∠CAD + ∠BAD = 180° − ( 60°+50° ) and in ∆ XQR, RX = QX + QR 2 2 2 only when ABC is triangle are same, hence they are
= ∠CAD + ∠ACD = 90° right angled congruent.
t = 70° 2
⇒ RX 2 = 
PQ  A
 + QR B
2 B C triangle.
So, ∆ABC must be right angled triangle. Also, t =z [alternate angles]  2  43. (b) Given, AC = BC
Hence, BC 2 = AC 2 + AB 2 ∴ z = 70° Similarly, ∠B = 360° − Ext ∠B and 40. (a) In ∆DCX ,CD = CX [given] ∴ ∠ABC = ∠BAC
Q P X = XQ = PQ  …(ii) ∠C = 360° − Ext ∠C ∠3 = ∠ 4
27. (a) In ∆ABC , 31. (b) Since, ∠PTB = 55°  [angles of an isosceles triangle]
[given] 2  [opposite angle of same sides]
C Then, ∠TUV = 55° We know that, the sum of all the angles ⇒ ∠BAC = ∠ABC = 38°
On adding Eqs. (i) and (ii), we get of a triangle is 180°. But ∠ 3 = ∠ 5, So, ∠ 4 = ∠ 5
[corresponding angle] In ∆ABC ,
D In ∆ ABD and ∆ ACX ,
Also, ∠TUV = ∠CUQ = 55° 5PQ 2
5QR 2
∴ ∠A + ∠B + ∠C = 180° ∠ACB = 180° − ( ∠BAC + ∠ABC )
PY 2 + RX 2 = + ∠1 = ∠2 [given]
[vertically opposite angle] 4 4 ⇒ 360° − Ext ∠A + 360° − Ext ∠B [by angle sum property of a triangle]
A
A B Also given, ∠DVS = 45° + 360° − Ext ∠C = 180° = 180° − (38° + 38° )
b ⇒ PY 2
+ RX = 5 / 4( PQ + QR )
2 2 2
Then, ∠UVT = 45° ⇒ Ext ∠A + Ext ∠B + Ext ∠C 12
1 ⇒ 4 ( PY + RX ) = 5( PR )
2 2 2 = 180° − 76° = 104°
Area of ∆ABC = × Base × Altitude [vertically opposite angle] = 1080° − 180° = 900°
2 In ∆ACD, ∠ACD = 180° − 104° = 76°
In ∆UTV , In ∆XQR, by pythagoras theorem, 37. (d) Since, XP || AC and YQ || AB
1 2∆ [by linear pair]
∆ = b × AC , AC = ∠T = 180° − (55° + 45° ) = 80° XR2 = XQ 2 + QR2 …(iii)
2 b A and ∠CAD = ∠ACD = 76°
[angle sum property of a triangle] 5
In ∆ABC , Using pythagoras theorem, and In ∆PQY , by pythagoras theorem, B C [Q CD = AD]
⇒ ∠T = ∠PTR = 80° D 3
AC 2 + AB 2 = BC 2 PY 2
= PQ + QY
2 2
…(iv) ∴ ∠ADC = 180° − ( ∠CAD + ∠ACD )
[vertically opposite angle]
4∆2 ∴ ∠CUQ + ∠RTP = 55° + 80° = 135° On adding Eqs. (iii) and (iv), = 180° − ( 76° + 76° ) = 28°
⇒ BC = + b2 4
b2 32. (a) XR2 + PY 2 = XQ 2 + QR2 44. (c)I. It is true that the three medians of a
X Y X
Again in ∆ ABC , area of ∆ABC A a 3c
B triangle divide it into six triangles of
1
4 + PQ + QY 2 2 ∠4 = ∠5
∆ = × BC × AD 5 B C equal area.
C P Q ∴ ∆ABD ~ ∆ACX
2 b
1 ⇒ XR2 + PY 2 = XQ 2 + QY 2 II. It is also true that, the perimeter of a
2∆ 2 ∆b ∴ ∠XBP = ∠YQC and ∠XPB = ∠YCQ [by AA similarity]
⇒ AD = = + QR2 + PQ 2 triangle is greater than the sum of
4∆2 + b 4 4∆2 + b 4 6
2
Also, XY ||BC 41. (a) Since, AB ⊥ BD and PQ ⊥ BD
the lengths of its three medians.
x ⇒ XR2 + PY 2 = XY 2 + PR2
b2 ∴ XYQB and XYCP are parallelogram ⇒ AB ||PQ Hence, I and II are correct.
D and again,
28. (a) As, AB||CD and BC cuts them. ⇒ XB = YQ and XP = YC So, ∆ABD ~ ∆PQD
45. (d) In ∆CAD and ∆CBE,
∴ ∠BCD = ∠ABC = x [alternate ∠S] ∠1 = ∠3 + ∠4, ∠2 = ∠5 + ∠6 XR + PY
2 2
= XY + PR 2 2
So, ∆XBP and ∆YCQ are congruent A
C
∴ In ∆BCD, x + y + z = 180° ⇒ ∠1 + ∠2 = ∠3 + ∠4 + ∠5 + ∠6 = XY 2 + ( 2XY )2 triangles.
C
4
y + y + z = 180°
[since, exterior angle is equal to sum XR + PY
2 2
= XY + 4XY 2 2
Now, XY || BC x P E D
3 of two opposite interior angles] AX XY y
⇒ XR2 + PY 2 = 5XY 2 ∴ = ⇒ AX = XY z
7y ⇒ b = c+ a+ x AB BC
⇒ + z = 180° Thus, option (d) is incorrect. B D A B
3 ∴ x= b−c−a [Q AB = BC = 30 cm] Q
MATHEMATICS Triangles 267 268 CDS Pathfinder

∠C = ∠C [common] ⇒ 3( ∠A + ∠B ) + 3∠C = 540° …(ii) Q∠DEC + ∠FEC = 180° [linear pair] ∆DST  1  1
2 9+ 4 1 1
⇒ = = ⇒ = A
Given that, AB = AE  = 13 × 4 × 9 CD 2 36
∠CEB = ∠ADC [each 90°] On subtracting Eq. (i) from Eq. (ii), we ∆DEF  2  2
∴ ∆CAD ~ ∆CBE get ∴ ∠ABE = ∠AEB = 65° ∴ CD = 6 cm
⇒ ∆ DEF = 2 ∆ DST
[by AA similarity] ∠C = 60° Q ∠AEB + ∠AEF + ∠FEC = 180° Shortcut Method a+b
Hence, both statements I and II are true a–b
Sides will be in same proportion. 49. (c) [straight line] and statement II is the correct CD 2 = AD ⋅ DB
CA CD
= and
AD CD
= ⇒ 65° + x° + 80° = 180° explanation of statement I. CD = AD ⋅ DB = 4×9
C
CB CE BE CE ∴ x° = 180° − 145° = 35°
3
55. (c) We know that, A = 36 = 6 cm (a – b) b
Hence, all three statements are correct. B D C
52. (c)Q∠PCT + ∠PCB = π [linear pair] The sum of any 58. (d) Given that, ∠A = 90° and ∠C = 30° a
46. (b) Given, PQ = 3 cm, QR = 4 cm and S two sides of a
E B
RP = 5 cm 1 2 D x° triangle is greater 3 cm 5 cm ⇒ a 2 + b 2 + 2ab = a 2 + b 2 − 2ab + a 2
A
Here, RP 2 = PQ 2 + QR2 B than twice the ⇒ 4ab = a 2 ⇒ 4b = a
P median drawn to
BD a − b
B C
So, PQR is a right angled triangle. (π−b)° D
By Exterior angle theorem, c° a° the third side. Now, =
A T DC b
Let D is an interior point in ∆PQR ext. ∠A = ∠2 + ∠3
B C i.e. ( AB + AC ) > 2 AD
4b − b 3b 3
P
ext. ∠B = ∠1 + ∠3 ∠PCB = π − ( π − b ° ) = b ° …(i) ⇒ (3 + 5) > 2 AD ⇒ AD < 4
90º = = =
30º
ext. ∠C = ∠1 + ∠2 In ∆BPC , ∠PCB + ∠BPC + ∠PBC = π Hence, AD is always less than 4 cm. b b 1
A C or 3 : 1
3
5 Sum of the three exterior angles [by angle sum property of a triangle] 56. (d) Given that, PQ = 5 cm,QR = 12 cm ∴ ABC is a right angled triangle.
D 61. (d) Given that, ∠BPC = x°
= ( ∠1 + ∠2) + ( ∠2 + ∠3) + ( ∠3 + ∠1) ⇒ ∠PBC = π − ∠PCB − ∠BPC and QL is a median.
Also, given that BC = 2 AB
= 2( ∠1 + ∠2 + ∠3) P and ∠BAC = y °
= π − b °− a ° …(ii) BC
= 2 × 180° = 360° ⇒ AB = …(i)
Q 4 R Q ∠ABE + ∠EBC = π [linear pair] 2 A P
[Q ∠1 + ∠2 + ∠3 = 180°]
In ∆DPQ, DP + DQ > 3 …(i) ⇒ ∠ABE = π − ∠PBC By pythagoras theorem, y° x°
similarly, DQ + DR > 4 50. (d) Given that, BC || AE [Q ∠PBC = ∠EBC ] L
…(ii) 5 BC 2
= AC 2
+ AB 2

DP + DR > 5 ∠CBA + ∠EAB = 180° = π − ( π − b °− a ° ) = a ° + b °…(iii)


and …(iii) ⇒ ( 2 AB )2 = AC 2 + AB 2
adding (i), (ii) and (iii), we get C Now, in ∆ABE
B
65° 90º ⇒ AC = 4 AB − AB 2 = 3 AB 2
2 2 B C D
2 (DP + DQ + DR) > 12 Sum of two interior angles = Exterior Q R Since, BP and CP are the angle bisectors
12 3
⇒ DP + DQ + DR > 6 95° angle ⇒ AC = 3 ⋅ AB = ⋅ ( 2 AB ) of ∠ABC and ∠ACD, respectively.
PR 2
∠EAB + ∠ABE = ∠BES ∴ PL = LR = ...(i)
Hence, both statements I and II are x°
c ° + b ° + a ° = x° 2 3 ∴ ∠ABC = 2∠PBC …(i)
E ∴ AC = ⋅ BC [from Eq.(i)]
individually true and statement II is not A
∠ACD = 2∠PCD
∴ x° = a °+ b °+ c ° In ∆PQR, ( PR)2 = ( PQ )2 + (QR)2 2 and …(ii)
correct explanation of statement I. ⇒ ∠EAB = 180° − 65° = 115°
BC = AC 53. (d) GA = GB = GC is true only and only [by pythagoras theorem] 59. (d) Draw AM ⊥ BC since sum of two interior angles is equal
47. (b) Given, Q
to exterior angle
N Hence, ∆ABC is an isosceles triangle. for equilateral triangle and here it is not ( PR)2 = (5)2 + ( 12)2 Let AB = BC = AC = x
⇒ ∠CBA = ∠CAB = 65°
given that ABC is an equilateral triangle.
( PR)2 = 25 + 144 ⇒ 169 = ( 13)2 x ∴ ∠PCD = x °+∠PBC …(iii)
So, statement I is not correct. Similarly, Then, BM = MC =
Q Now, ∠EAB = ∠EAC + ∠CAB ⇒ PR2 = ( 13)2 ⇒ PR = 13 2 and ∠ACD = y °+∠ABC
statement II will also hold only for
⇒ 115° = x + 65° ⇒ x = 50° equilateral triangle. Now, by mid-point theorem, if L is the A ⇒ 2∠PCD = y °+2∠PBC
[using (i) and (ii)]
51. (b) Given that, ∠ABC = 65° A mid-point of the hypotenuse PR of a
L M right angled ∆PQR, then ⇒ 2 [ x °+∠PBC ] = y °+ 2∠PBC
P and ∠CDE = 15°
1 1 ⇒ 2x ° = y °
I. PQ 2 = MP 2 + NQ 2 Since, AB || CD QL = PR = ( 13) = 6.5 cm
⇒ PQ 2 = LP 2 + LQ 2 G 2 2 B M C D 62. (a)Q∠ACB + ∠BCD = 180° [linear pair]
∠DCB = ∠ABC = 65° x
[Q LP = MP and NQ = LQ] 57. (d) In ∆ABC and ∆ACD, Also, CD = [Q BC = 2CD] ∠BCD = 180° − 70° = 110°
⇒ ∠QLP = 90° [alternate angles] AC 4 2
Q = B
It means, ∆NLM is a right angled Also, ∠DCE = ∠DCB [same angle] AB AC Now, in ∆AMC , by pythagoras theorem,
triangle. B C
Now, ∠FEC = ∠CDE + ∠DCE ∴ AC 2 = 4 × 13 = 52 ... (i)
II. It also true that if in a ∆ABC . DS 1 AM 2 = AC 2 − MC 2
[exterior angle] 54. (a) Given, = [Q AB = 4 + 9 = 13]
AB 2 > BC 2 + CA 2 , then ∠ACB is DE 2 x 2 3x 2
15° + 65° = 80° BC 9 = x2 − =
obtuse. D In ∆ABC and ∆BCD, = 70°
Hence, both statements are AB BC 4 4 A D
A F C
individually true but statement II A In ∆AMD, by pythagoras theorem,
is not the correct explanation of T S T In ∆BCD, BC = CD
4 AD = AM + MD
2 2 2
statement I. ⇒ ∠CBD = ∠CDB ...(i)
D 3x 2 7x 2
x° = + x2 = [angles opposite to equal side]
48. (c) Given that, 65° 65° 80° E F
9
B
4 4 Also, ∠BCD + ∠CBD + ∠CDB = 180°
1 1 1
∠A + ∠C + ∠B = 80° E C ∴ DE = 2 × DS x 2
= 7   = 7CD 2 Q CD = 
65° 90° x
2 3 2 [by angle sum property of a triangle]
Since, ST is parallel to EF. C B  4  2 
⇒ 3∠A + 2∠C + 3∠B = 480° 2∠CDB = 180 ° − ∠BCD
∴ ∠S = ∠E and ∠T = ∠F ⇒ BC 2 = 9 × 13 = 117 ...(ii) 60. (d) In right angle ∆ABC ,
⇒ 3 ( ∠A + ∠B ) + 2∠C = 480° …(i) 15° So, ∆DTS and ∆DEF are similar. 1 1 1 1 1 = 180 ° − 110 ° = 70 °
Now, = + = + By pythagoras theorem,
Also, in ∆ABC , Ratio of areas = Ratio of squares of CD 2
AC 2
BC 2
52 117 ( a + b )2 = ( a − b )2 + a 2 70°
∴ ∠CDB = ∠ADB = = 35°
∠A + ∠B + ∠C = 180° D corresponding sides. 2
MATHEMATICS Triangles 269 270 CDS Pathfinder

63. (d) Now, applying pythagoras theorem in 3a 3a CD AC 72. (a) Since, the triangle is right angled. Obviously, A , B and C are the 78. (d)
PO + OL = , 8+ 4= ⇒ + 1= + 1 A x B
∆ABE, 2 2 BD AB So, all the three mid-points of DE , EF and DF,
A
AE 2 = AB 2 + BE 2 12 × 2 24 CD + BD AC + AB consecutive sides respectively.
a= = cm ⇒ = must satisfy
⇒ z 2 = ( x − y )2 + s 2 3 3 BD AB 5 E
O
pythagoras 4 θ
A ∴ Perimeter of ∆PQR = 3a BC 3 + 1 BD 3 xQ Px
⇒ = ⇒ = theorem.
BD 3 BC 4 A B
x–y z 3 × 24 Hence, 3, 4 and 5 B C
= = 24 3 cm 3
E 3 ∴ BD : BC = 3 : 4 are the sides of
x B s triangle which satisfy pythagoras D C F
66. (b) Perimeter of ∆PQR = 1+ 2 + 3 =6 units 69. (b) In ∆ADC and ∆BEC D x C
y theorem.
A C By mid-point theorem, Let side of square ABCD be x and
1
∴ Area of triangle = × 4 × 3 = 6 cm 2 1
BC = DE or DE = 2BC drawn equilateral ∆BOC inside ABCD.
C s
2 2
D D P E Such that, BO = OC = BC = x
73. (a) In a triangle, Similarly, DF = 2 AB BC x
⇒ z 2 = x2 + y 2 − 2 x y + s 2 Q R E D Sum of two sides is always greater than and EF = 2 AC and BP = =
2 2
∴ s 2 = z 2 − x2 − y 2 + 2 x y the third side Hence, statement I is correct.
B F C 2
x2
x 2 −  
x < 40 x
64. (b) Given, ∠C = 2∠ A and ∠B = 90° Q P , Q , R are the mid-points of DE,DF
i.e. ...(i) II. Also, area of ∴ PO = = x2 −
A B  2 4
and FE of ∆DEF . Difference of two sides is always less 1
∴ ∠A + ∠C = 90° ∆ABC = area of ∆DEF
∴ 2 PQ = FE ∠BEC = ∠ADC = 90° than third side 4 3x 2 3
A = = x
∠ACD = ∠BCE [common] i.e. 10 < x ...(ii) or area of ∆DEF = 4 area of ∆ABC 4 2
[by mid-point theorem]
From Eqs. (i) and (ii), 10 < x < 40 Hence, statement II is also correct. 3
Similarly, DF = 2 PR and DE = 2QR ∴ ∆ADC ~ ∆BEC [by AA similarity] and OQ = AB − PO = x − x
AC CD 74. (a) Given, in ∆ABC , ∠B = 2∠C = 2∠A 76. (c) See solved answer of 67. 2
∴Perimeter of ∆DEF = 2 × 6 = 12 units =
90° We know that, sum of all angles of a 77. (c) Given, O is equidistant from AB, BC 2− 3
Similarly, perimeter of ∆ ABC BC CE = x 
B C
⇒ AC × CE = BC × CD triangle = 180° and AC  2 
= 2 × perimeter of ∆DEF ∴ O is the incentre of ∆ABC
⇒ ∠A + 2∠ A = 90° [Q ∠C = 2∠A] ⇒ ∠A + ∠B + ∠C = 180°
= 2 × 12 = 24 units ⇒ CE × CA = CD × CB A Since, ∠AOD = 2θ
⇒ 3∠ A = 90° ⇒ ∠ A = 30° ⇒ ∠A + 2∠A + ∠A = 180°
a ⇒ ∠AOQ = θ
So, ∠C = 90° − 30° = 60° 67. (b) Draw a line segment DG parallel to 70. (b) Let the sides of triangle be , a , ar
r [Q 2∠C = 2∠A = ∠B] In ∆AOQ,
BF. Then, in ∆ADG,
Now, if the angles of a triangle are of and since r < 1. ⇒ 4∠A = 180° O AD x
measure 30°, 60° and 90°. A

a
> a > ar 180° AQ
Then, side opposite to 30° i.e. F ⇒ ∠A = = 45° tanθ = = 2 = 2
1 E G
r 4 OQ OQ 2− 3
BC = × hypotenuse B C x 
Now, triangle is right angled. A  2 
2
1 B C Join OA , OB and OC .
⇒ BC = AC ⇒ AC = 2 BC Using pythagoras theorem, x 2
D = ×
2 2 In ∆ABC , OA , OB and OC are angle 2 x( 2 − 3 )
BF || DG  a  = ( a )2 + ( ar )2
In right angled ∆ABC ,   bisectors.
∴ EF || DG  r 1 2+ 3 2+ 3
by pythagoras theorem 1 = × =
and AE = ED a2 Q ∠OAB = ∠OAC = ∠BAC 2− 3 2+ 3 4 −3
AC 2 = AB 2 + BC 2 ⇒ = a2 + a2r2 90° 2
[since, E is mid-point of AD] r2 B C
1 [Q ( a + b )( a − b ) = a 2 − b 2 ]
⇒ 4BC 2 = AB 2 + BC 2 [Q AC = 2BC ] a2 ∴ ∠OBA = ∠OBC = ∠ABC
∴ AF = FG ...(i) ⇒ = a2( 1 + r2 ) ⇒ r2 + r4 = 1 ∴ ∠B = 90° and ∠C = 45° 2 = 2+ 3
⇒ AB = 4BC − BC
2 2 2
= 3BC 2
r2
Similarly, in ∆BCF Thus, ∆ABC is a right angled triangle, 1
and ∠OCB = ∠OCA = ∠ACB 79. (d) In ∆PQR,
65. (d) Since, PQR is an equilateral triangle. DG || BF and BD = DC Put r 2 = x, right angle at B and AB = BC . 2
Then, PL is also the median of ∆PQR.
∴ FG = GC ...(ii) ⇒ x2 + x − 1 = 0 In ∆ABC , by pythagoras theorem, Now, in ∆ABC , PX 1.5 1
= = and
PY 2 1
= =
Similarly, RN and QM are also the
median of ∆PQR and O is the centroid. From Eqs. (i) and (ii), we get −1 ± 1 − 4( −1) −1 ± 5 AB 2 + BC 2 = AC 2 ∠ABC + ∠BAC + ∠ACB = 180°
XQ 6 4 YR 8 4
x= =
2 2 2 P
PO 2 PO 8 CF = AC ⇒ AB 2 + AB 2 = AC 2
So, = ⇒ OL = = = 4 cm [Q angle sum property]
1.5 2
3 Since, sides of triangle cannot be negative
OL 1 2 2 [Q AB = BC ] ∴ ∠OAC + ∠OCB + ∠OBA = 90°
3a 68. (d) In ∆ABC , AD is the internal angle 5−1 ⇒ 2 AB 2 = AC 2 X Y
Now, altitude of ∆PQR = ∴ r = 2
[option]
2 bisector of ∠A 2 ⇒ 2 AB = AC II. This statement is false as 6 8
[where, a = length of side of an Using property of internal angle bisector.
71. (a) ∆ABC ~ ∆DEF Q R
[taking square root on both sides] 1
equilateral ∆PQR] C
A
∠BOC = 90°+ ∠BAC PX PY
P D AC 2 2 So, =
D ⇒ = XQ YR
8 cm 1 AB 1 III. The statement perpendiculars
PX PQ 1.5 7.5
∴ AC : AB = 2 : 1 drawn from any point on OA to AB I. = ⇒ =
N M A 3 B E F XY QR XY QR
75. (c) I. On drawing the three straight lines and AC are always equal so it is true,
B C
BD
=
AB

CD
=
AC through the three vertices of ∆ABC , because O is equidistant from AB ⇒ QR = 5X Y
O CD AC BD AB AB BC 1 8 PX PY 1
∴ = ⇒ = we get the following figure and AC . II. Also, = =
Q R On adding both sides, we get DE EF 2 EF Here, AB || DF , BC || DE and PQ PR 5
L Hence, the statements I and III are
EF = 16 cm AC || EF correct. ⇒ QR is parallel of XY .
24
MATHEMATICS Triangles 271 272 CDS Pathfinder

III. ∆PYX is similar to ∆PRQ. ∴ XY = YZ = XZ So, ∆ABC and ∆DEF are similar.
Hence, all statements are correct. So, ∆XYZ is an equilateral triangle and ∴
AB BC
= =
AC
80. (c) See solved answer 17. ∆XYZ is similar to ∆ABC . DE EF DF
81. (b) Let ∠CDB = y and ∠CAD = x So, both statements are correct. Now, L is the mid-point of BC , then
As, AD = CD 1
83. (b) I. This statement is always true. BL = BC
C 2
96° II. In any triangle three times the sum
x of squares of the sides of a triangle is Also, M is the mid-point of EF , then
equal to four times the sum of 1 AB 2BL BL
EM = EF ⇒ = =
squares of medians 2 DE 2EM EM
A
x
D
y

⇒ ∠ACD = ∠CAD = x and CD = CB


⇒ ∠CBD = ∠CDB = y
y
B i. e. 3 ( AB 2 + BC 2 + CA 2 )
= 4 ( AD 2 + BE 2 + CF 2 )
Similarly 3 ( A' B' 2 + B' C' 2 + C' A' 2 )
and ∠ABL = ∠DEM
∴ ∆ABL is similar to ∆DEM.
[By SAS Similarity]
QUADRILATERAL
From exterior angle property,


∠CDB = ∠CAD + ∠ACD
y = x + x ⇒ y = 2x
Now, ∠ ACD + ∠DCB + 96° = 180°

= 4 ( A' D' 2 + B' E' 2 + C' F ' 2 )
AB 2 + BC 2 + CA 2
AD 2 + BE 2 + CF 2
A' B' 2 + B' C' 2 + C' A' 2 4
Hence, statement I is true but statement
II is false.
86. (c) In ∆ABC , AD is the median and
median AD bisects the area of ∆ABC .
AND POLYGON
= = 1
Area of ∆ABD = × Area of ∆ABC
⇒ x + 180° − 2 y + 96° = 180° A' D' 2 + B' E' 2 + C' F ' 2 3 2
y 3
⇒ − 2 y + 96° = 0 ⇒ y = 96° Hence, both statement are correct but
2 2 A
statement II is not correct explanation
∴ y = 64°
of statement I. Regularly (2-3) questions have been asked from this chapter. Generally, questions from this
82. (c) In an equilateral ∆ABC,
AB = BC = CA and ∠B = ∠C = ∠A Area of ∆ABC AB 2 E chapter are tricky and mostly statement based. So, a clear concept of all the properties
84. (c) =
Area of ∆DEF DE 2 related to quadrilateral is necessary to do well.
A
( 2 + 1)2 3 + 2 2
= = C
( 3) 2 3 B D
Z
(3 + 2 2 )(3 − 2 2 )
= Since, E is the mid-point of AD. Then,
3(3 − 2 2 ) BE is the median of ∆ABD. So, BE
Y
[Q a − b = ( a − b )( a + b )]
2 2 bisects of area of ∆ABD.
1
QUADRILATERAL
B X C =
1 I. Area of ∆BED = × Area of ∆ABD A figure enclosed by four sides is called a quadrilateral. A quadrilateral has four angles and sum of
9−6 2 2
11 these angles is equal to 360°.
Given that, BX = CY = AZ Hence, the required ratio is = × Area of ∆ABC 
2  2  Various types of quadrilateral are discussed below.
Now, in ∆XYC , ∆ZYA and ∆XZB, 1 : ( 9 − 6 2 ). 1
BX = CY = AZ = × Area of ∆ABC
…(i) 85. (c) Given that, ∠ABC = ∠DEF , 4 Parallelogram
⇒ ( BC − XC ) = ( AC − AY ) D b C
∠ACB = ∠DFE Hence, statement I is correct. A quadrilateral, in which opposite sides are parallel is called a
= AB − BZ parallelogram.
and ∠BAC = ∠EDF II. Area of ∆ADC = Area of ∆ABD
⇒ XC = AY = BZ …(ii)
A D = 2 × Area of ∆BED (i) Area = Base × Height = b × h (ii) Perimeter = 2 ( a + b) a h a
and ∠B = ∠C = ∠A …(iii)
So, statement II is also correct.
From Eqs. (i), (ii) and (iii), we get
Hence, the statements I and II are
Properties of Parallelogram
∆XYC , ∆ZYA and ∆XZB are congruent A b B
triangles. B L C E M F correct. (i) Diagonals of a parallelogram bisect each other.
(ii) Each diagonal of a parallelogram divides it into two congruent triangles.
(iii) Sum of any two adjacent angles is 180°.
(iv) A parallelogram inscribed in a circle is a rectangle.
(v) A parallelogram circumscribed about a circle is a rhombus.
(vi) Two parallelograms have equal areas if they are on the same base and between the same parallel lines.
(vii) Lines joining the midpoints of the sides of parallelogram is a parallelogram.
(viii) The opposite angles of parallelogram are equal. (∠ A = ∠ C and ∠ B = ∠ D) (from above figure)
(ix) The sum of the squares of the four sides is equal to the sum of squares of diagonal.
AC 2 + BD 2 = AB 2 + BC 2 + CD 2 + AD 2
MATHEMATICS > Quadrilateral and Polygon 273 274 CDS Pathfinder

EXAMPLE 1. In a parallelogram ABCD, the bisectors of Sol. c. Since, diagonals of rectangle bisect each other Properties of Square EXAMPLE 6. In the adjoining figure, ABCD is a
∠A and ∠B meet at O. Then, the value of ∠AOB is ∴ OD = OA ⇒ ∠ODA = ∠OAD trapezium in which AB || DC. If ∠A = 55° and ∠B = 70 °,
(i) Diagonal of a square are equal and bisect each other then the value of ∠C and ∠D is
a. 55° b. 75° c. 90° d. 120° Also, ∠DOA = ∠COB = 42°
at right angles (90°).
Sol. c. As, ABCD is a parallelogram. D C In ∆DOA, ∠DOA + ∠ODA + ∠OAD = 180° D C
⇒ 42° + x + x = 180° ⇒ 2x = 138° ⇒ x = 69° (ii) All square are rhombus but converse is not true.
∴ ∠A +∠B = 180° [cointerior angle] O (iii) Diagonal of square is the diameter of the
a
1 1
2
∠A + ∠B = 90°
2
Rhombus d2
circumscribing circle that circumscribes the square.
55° 70°
A
B
It is a parallelogram with all 4 (iv) Side of a circumscribed square is equal to the
⇒ ∠OAB + ∠OBA = 90° a a diameter of the inscribed circle. A B
sides equal. The opposite angles in
In ∆AOB, ∠AOB = 180°− ( ∠OAB + ∠OBA) = 180° − 90° = 90° d1 (v) The figure formed by joining the mid-points of the a. 75° and 85° b. 90 ° and 120 °
a rhombus are equal but they are
sides of square is a square. c. 110 ° and 125° d. 115° and 120 °
D C not right angle. a
EXAMPLE 2. In the adjoining figure, F 1 S R
ABCD is a parallelogram and E , F are O (i) Area = × d 1 × d 2 EXAMPLE 5. PQRS is a square, P is Sol. c. As, AB || CD
E 2 joined to a point L on QR and S is
the centroids of ∆ABD and ∆BCD, 2
∠A + ∠D = 180° ⇒ ∠D = 180° − 55° = 125°
A B (ii) Perimeter = 4a (iii) Side (a) =
1
d 12 + d 22
joined to a point M on PQ. If PL = SM, L
So,
respectively, then the length of EF is then Also, ∠B + ∠C = 180° ⇒ ∠C = 180° − ∠B
2 3
1 1 (iv) 4a 2 = d 12 + d 22 1 ⇒ ∠C = 180° − 70° = 110°
a. AE b. OB c. AE d. FC a. ∠POM > 90 ° b. ∠POM = 90 ° P
4
3 3 Q
where, a = side, d 1 and d 2 are diagonals. c. ∠POM<90 ° d. None of these M So, ∠C = 110° and ∠D = 125°
Sol. a. As E is the centroid of ∆ABD and AO is one of its medians.
Sol. b. In ∆SPM and ∆PQL, ∠SPM = ∠PQL = 90°

1
OA : EO = 3 : 1 ⇒ EO = OA Properties of Rhombus SP = PQ and SM = PL ⇒ ∆SPM ≅ ∆PQL [By AAS] POLYGON
3
1 (i) The diagonale of a rhombus bisect each other at ⇒ ∠1 = ∠ 2 and ∠ 3 = ∠ 4
Similarly, FO = OC A polygon is a closed, plane figure bounded by ‘n’
right angles (90°). But they are not necessarily In ∆SPM, ∠ 2 + 90°+∠ 4 = 180° ⇒ ∠ 2 + ∠ 4 = 90°
3 straight lines ( n ≥ 3). Each of the line segment forming
1 1 1 equal. In ∆ OPM , ∠1+ ∠ 4 + ∠POM = 180°
∴ EO + OF = OA + OC = AC = AE the polygon is called its sides.
3 3 3 (ii) Diagonals bisect the vertex angles. ⇒ ∠ 2 + ∠ 4 + ∠POM = 180°
(iii) The figure formed by joining the mid-points of the ⇒ 90°+∠POM = 180° [Q ∠ 2 + ∠ 4 = 90°] A polygon may be a triangle, quadrilateral, pentagon
∴ EF = AE
sides of rhombus is a rectangle. ⇒ ∠POM = 90° etc. Polygons are classified according to the number of
(iv) All rhombus are parallelogram but reverse is not true. sides as given below:
Rectangle Trapezium
L (v) A rhombus may or may not be a square but all Number of sides Name
It is a parallelogram with It is a quadrilateral with any one pair of opposite sides
squares are rhombus.
opposite sides equal and each D C
parallel. 3 Triangle
B d B a
angle is equal to 90°. EXAMPLE 4. In a rhombus ABCD, D C 4 Quadrilateral
O
(i) Area = Length × Breadth diagonals intersect each other at O. If 4 5 Pentagon
= L× B L AO = 3 cm and OB = 4 cm, then find 3 c h d
the perimeter of ABCD. 6 Hexagon
A B
(ii) Perimeter = 2( L + B) (iii) Diagonal (d ) = L + B 2 2
A B 7 Heptagon
a. 14 cm b. 30 cm c. 20 cm d. 28 cm b
1 8 Octagon
Properties of Rectangle Sol. c. The diagonals of a rhombus are perpendicular (i) Area = (Sum of the parallel sides × Height)
bisectors. Which means they form right angles at the 2 9 Nonagon
(i) The diagonals of a rectangle are of equal lengths and 1
point of their intersection. = ( a + b) h where, a and b are parallel sides and h is 10 Decagon
they bisect each other. ∴In ∆AOB, by pythagoras theorem,
2
the height or perpendicular distance between a and b.
(ii) All rectangles are parallelograms but reverse is not true. AO2 + OB 2 = AB 2 ⇒ AB = 32 + 42 = 9 + 16 = 25 Regular Polygon
(iii) Diagonal of a rectangle inscribed in a circle is equal to ∴ AB = 5 cm
(ii) Perimeter = AB + BC + CD + AD
In regular polygons, all sides and all interior angles are
the diameter of the circle. Hence, perimeter of rhombus equal. If each side of a regular polygon of n sides = a,
(iv) The figure formed by joining the mid-points of the ABCD = AB + BC + CD + DA = 5 + 5 + 5 + 5 = 20 cm Properties then
sides of a rectangle is a rhombus. (i) If the non-parallel sides are equal then the diagonals
(v) A rectangle and a parallelogram have equal area if they
Square a
will also be equal to each other. (i) Area of regular pentagon = 5a 2
3 A a B

It is a parallelogram with all four sides (ii) Diagonals intersect each other proportionally in the 4
are on the same base and between same parallel lines. F C
equal and each angle is equal to 90°. ratio of lengths of parallel sides. 3
d (ii) Area of regular hexagon = 6a 2
EXAMPLE 3. In adjoining figure, D C 1 a a (iii) If a trapezium is inscribed in a circle, then it is an 4
(i) Area = (side) 2 = a 2 or d 2
if ABCD is a rectangle and x 2 isosceles trapezium with equal oblique sides. (iii) Area of regular octagon
E D
Regular Hexagon
diagonals AC and BD intersect 42° (ii) Perimeter = 4 × side = 4a (iv) The line joining the mid-points of non-parallel sides
each other at O. Find the value
O a = 2 ( 2 + 1) a 2
(iii) Diagonal (d ) = a 2 is half the sum of parallel side and is called median.
of x A B 1 360 °
where, a = side, d = diagonal i.e. Median = × sum of parallel sides. (iv) Each exterior angle =
a. 75° b. 70° c. 69° d. 63° 2 n
MATHEMATICS > Quadrilateral and Polygon 275 276 CDS Pathfinder

(v) Each interior angle = 180 ° – Exterior angle Sol. c. Sum of interior angles of a hexagon = (n − 2) × 180°
6. If ABCD is a rhombus, then 13. In the given figure, ABCD is a parallelogram
( n − 2) × 180 = 4 × 180° = 720° (a) AC 2 + BD2 = 4 AB2 (b) AC 2 + BD2 = AB2 and E is the mid-point of AD. A line through D,
=
n ∴ x° + ( x − 5) ° + ( x − 5) ° + ( 2x − 5) ° (c) AC + BD = 2 AB
2 2 2
(d) 2 ( AC + BD ) = 3 AB
2 2 2 drawn parallel to EB, meets AB produced at F
 n ( n − 1)  + ( 2x − 5) ° + ( 2x + 20) ° = 720° and BC at L. Then,
(vi) Number of diagonals =  − n 720° 7. A point O in the interior of a rectangle ABCD is
∴ 9x = 720° = = 80° D C
 2  9 joined with each of the vertices A, B, C and D.
(vii) Sum of all interor angles = ( n − 2) × 180 ° . Hence, the value of x is 80°. Then,
E L
(a) OB + OD = OC + OA (b) OB2 + OA 2 = OC 2 + OD2
EXAMPLE 7. A polygon has 35 diagonals. Then, the EXAMPLE 9. The difference between the interior (c) OB ⋅ OD = OC ⋅ OA (d) OB2 + OD2 = OC 2 + OA 2
number of sides of that polygon is and exterior angles of a regular polygon is 60°. Then,
a. 7 b. 10 c. 11 d. 12 how many sides are there in that polygon? 8. In a trapezium ABCD, if AB||CD , then A B F

a. 5 b. 6 c. 7 d. 8 AC 2 + BD 2 is equal to (a) AF = 2 DC (b) 2 AF = 3 DC


Sol. b. Let number of sides be n, then
n (n − 1) n2 − n − 2n (a) BC 2 + AD2 + 2 AB ⋅ CD (b) AB2 + CD2 + 2 AD ⋅ BC (c) 2 DF = 3 DL (d) AF + DF = 3 DC + DL
− n = 35 ⇒ = 35 Sol. b. Here, (interior angle) − (exterior angle) = 60°
2 2 (c) AB2 + CD2 + 2 AB ⋅ CD (d) BC 2 + AD2 + 2 BC ⋅ AD 14. Let ABCD be a trapezium in which AB|| DC and
(n − 2) × 180 360
⇒ n2 − 3n − 70 = 0 ⇒ n2 − 10n + 7n − 70 = 0 ⇒ − = 60 let E be the mid-point of AD. Let F be a point on
n n 9. ABCD is a trapezium in which AB||CD and
⇒ n(n − 10) + 7 (n − 10) = 0 ⇒ (n − 10) (n + 7) = 0 BC such that EF || AB. Then, consider the

1
[(n − 2) × 180 − 360] = 60 AB = 2 CD. Its diagonals intersects each other at
⇒ n = 10 and n ≠ − 7 following statements.
n O, then the ratio of the areas of the ∆AOB and
Hence, the number of sides of the polygon is 10. 1 1 D C
⇒ [180 n − 360 − 360] = 60 ⇒ [180n − 720] = 60 ∆COD is
n n (a) 1 : 2 (b) 2 : 1 (c) 4 : 1 (d) 1 : 4
EXAMPLE 8. The angles of a hexagon are x°,
⇒ 180 n − 720 = 60n ⇒ 180n − 60n = 720 E F
(x − 5)°, (x − 5)°, (2x − 5)°, (2x − 5)° and (2x + 20)°. 10. In the figure, ABCD is a parallelogram with
Then, the value of x is ⇒ 120n = 720 ⇒ n =6 AD = a units, DC = 2a units and DE : EC = 1 : 2.
A B
a. 60° b. 75° c. 80° d. 90°
Therefore, the polygon contains 6 sides. CEFG is a rectangle with FE = 3 AE. What is the
1
ratio of the area of a parallelogram and the (a) F is the mid-point of BC (b) EF = ( AB + DC )
rectangle? 2
F G
(c) Both (a) and (b) (d) None of these

A H B 15. In the figure given below, ABCD and PQRC are

PRACTICE EXERCISE D
E
C
a2
rectangles, where Q is the mid-point of AC, then
D
P C

(a) 1 : 1 (b) 1 : 2 (c) :1 (d) 2 : 1 R


5 Q
1. The sides BA and DC of quadrilateral ABCD are 3. If ABCD is a quadrilateral whose diagonals AC
produced as shown in figure. Then, which of the and BD intersects of O, then 11. In the adjoining figure, ABCD is a quadrilateral A B
following statement is correct? D in which AB is the longest side and CD is the 1 3
F shortest side, then (a) DP = PC (b) PR = DB
D C 3 2
y° a° C C 1
D (c) DP = PQ (d) DP = AC
O 2
b° x°
E A B B 16. How many diagonals are there in a octagon?
A
1 a° + b ° A B (a) 8 (b) 16 (c) 18 (d) 20
(a) 2x° + y° = a° + b ° (b) x° + y° = (a) ( AB + BC + CD + DA) < ( AC + BD)
2 2 (a) ∠C > ∠A and ∠D > ∠B (b) ∠C > ∠A and ∠B > ∠D 17. The angles of a pentagon are in the ratio
(c) x° + y° = a° + b ° (d) x° + a° = y° + b ° (b) ( AB + BC + CD + DA) > 2 ( AC + BD) (c) ∠C < ∠A and ∠D < ∠B (d) ∠C < ∠A and ∠D = ∠B 1 : 2 : 3 : 5 : 9, the largest angle is
(c) ( AB + BC + CD + DA) > ( AC + BD) (a) 81° (b) 135° (c) 243° (d) 249°
2. In the quadrilateral ABCD, the line segments (d) ( AB + BC + CD + DA) = 2 ( AC + BD) 12. In the given figure, ABCD is a square. A line
bisecting ∠C and ∠D meet at E. Then, the segment DX cuts the side BC at X and the 18. The ratio of an interior angle to the exterior
correct statement is 4. If area of a parallelogram with sides p and q is R diagonal AC at O such that ∠COD = 105° and angle of a regular polygon is 5 : 1. The number
D C and that of a rectangle with sides p and q is S , ∠OXC = x. The value of x is of sides of polygon is
2 1
then D C (a) 10 (b) 11 (c) 12 (d) 14
(a) R > S (b) R < S
E 105° 19. The measure of each angle of a regular hexagon
(c) R = S (d) None of these
x° is
5. Two parallelogram stand on equal bases and O X (a) 90° (b) 120° (c) 105° (d) 135°
between the same parallel. The ratio of their
A B
areas is
20. Each interior angle of a regular polygon is 150°.
A B The number of sides of the polygon is
(a) ∠A + ∠B = ∠CED (b) ∠A + ∠B = 2 ∠CED
(a) 1 : 2 b) 2 : 1 (c) 1 : 3 (d) 1 : 1 (a) 4 (b) 8 (c) 12 (d) 16
(c) ∠A + ∠B = 3 ∠CED (d) None of these (a) 40° (b) 60° (c) 80° (d) 85°
MATHEMATICS > Quadrilateral and Polygon 277 278 CDS Pathfinder

21. Let LMNP be a parallelogram and NR be 27. In the given figure ABCD is a A B 34. A parallelogram and a rectangle stand on the 41. ABCD is a parallelogram whose diagonals AC
60° 60°
perpendicular to LP. If the area of the quadrilateral with AB parallel to same base and on the same side of the base and BD intersect at O. A line segment PQ
parallelogram is six times the area of ∆RNP and DC and AD parallel to BC, ∠ADC with the same height. If l1 , l2 be the perimeters of through O meets AB at P and DC at Q.
RP = 6 cm what is LR equal to? is a right angle. If the perimeter the parallelogram and the rectangle respectively, D Q C
D C
(a) 15 cm (b) 12 cm (c) 9 cm (d) 8 cm of the ∆ABE is 6 units, what is the E then which one of the following is correct?
area of the quadrilateral? (a) l1 < l2 (b) l1 = l2
22. ABCD is a trapezium in which AB||CD. M and (c) l1 > l2 but l1 ≠ 2 l2 (d) l1 = 2 l2 O
(a) 2 3 sq units (b) 4 sq units
N are the mid-points of AD and BC, respectively.
(c) 3 sq units (d) 4 3 sq units 35. Two similar parallelograms have corresponding
If AB = 12 cm and MN = 14 cm find CD. A B
B 28. A square and a rhombus have the same base and sides in the ratio 1 : k. What is the ratio of their P
A 1
the rhombus is inclined at 30°. What is the ratio areas? (a) ar (quadrilateral APQD) = ar (|| gm ABCD)
(a) 1 : 3k 2 (b) 1 : 4k 2 (c) 1 : k 2 (d) 1 : 2 k 2 2
O of area of the square to the area of the rhombus? 1
M N (b) ar (quadrilateral APQD) = ar (|| gm ABCD)
(a) 2:1 (b) 2 : 1 (c) 1 : 1 (d) 2 : 3 36. Let WXYZ be a square. Let P , Q and R be the 4
1
D
29. P is the mid-point of side AB A P B mid-points of WX , XY and ZW , respectively and (c) ar (quadrilateral APQD) = ar (|| gm ABCD)
C 3
of parallelogram ABCD. A line K , L be the mid-points of PQ and PR,
(a) 2 cm (b) 5 cm (c) 12 cm (d) 16 cm (d) None of the above
through B parallel to PD D
respectively. What is the value of
23. ABCD is a parallelogram. CL is perpendicular to meets DC at Q and AD Q C area of ∆ PKL 42. In the figure, ABCD is a ||gm, L is the
AD and DM is perpendicular to BA produced. If produced at R. Then BR is ? mid-point of DC and M is the mid-point of LC. If
area of square WXYZ
CD = 16 units, DM = 12 units and CL = 15 units equal to R
ar (∆AMC) = 4 cm 2, then ar (|| gm ABCD) is
1 1 1
then AD = ? (a) BQ (b) (c) 2BQ (d) None of these (a) (b)
D L M C
2 32 16
D C
1 1
30. ABCD is a trapezium in which AB||DC and (c) (d)
8 64
AD = BC. If P , Q , R , S be respectively the
L
mid-point of BA, BD, CD and CA, then PQRS is a 37. ABC is a triangle in which AB = AC. Let BC be
D R C produced to D. From a point E on the line AC let A B
M A B EF be a straight line such that EF is parallel to
(a) 32 cm 2 (b) 16 cm 2
(a) 12.8 units (b) 13.6 units (c) 11.1 units (d) 12.4 units AB. Consider the quadrilateral ECDF thus
(c) 64 cm 2 (d) Data insufficient
S Q formed. If ∠ ABC = 65° and ∠ EFD = 80°, then
24. ABCD is a rectangle. PC = 9 cm, BP = 15 cm, what is the value of ∠ FDC ? 43. Consider the following statements.
AB = 14 cm. Then, the angles of ∆APB are such (a) 43° (b) 41° I. All squares are not parallelograms.
that A P B (c) 37° (d) 35° II. All parallelograms are trapezium.
A D (a) rhombus (b) rectangle III. All squares are rhombuses and also rectangles.
38. In the given figure, ABCD is a square in which
α (c) parallelogram (d) square IV. All rhombuses are parallelograms.
AO = AX . What is ∠ XOB?
31. ABCD is a parallelogram, E is the mid-point of Which of the above statements are correct?
D C
14 β P AB and CE bisects ∠BCD. Then ∠DEC is O
(a) I and II (b) III and IV (c) I and III (d) II and IV
15 D C 44. Consider the following statements.
9
γ I. If the side of a rhombus is 10 cm. Its diagonals
A X B
should have values 16 cm and 12 cm.
B C
(a) 22.5° (b) 25° II. The diagonals of a rhombus cut at right angles.
(a) α > β > γ (b) α > γ > β (c) β > γ > α (d) α < β < γ (c) 30° (d) 45°
A E B III. The diagonals do not bisect each other.
25. ABCD is a rhombus with ∠ABC = 56° and 39. Two sides of a parallelogram are 10 cm and Which of the above statements are true?
∠BAC = 62°, then ∠ACD is equal to (a) 60° (b) 90° (c) 100° (d) 120°
15 cm. If the altitude corresponding to the side of (a) I and II (b) II and III (c) I and III (d) I, II and III
A D 32. ABCD is a square, P Q , R and S are points on length 15 cm is 5 cm, then what is the altitude
to the side of length 10 cm ? 45. Consider the following statements in respect of a
the sides AB, BC , CD and DA, respectively such
62º quadrilateral
that AP = BQ = CR = DS . What is the value of (a) 5 cm (b) 7.5 cm
∠ SPQ? (c) 10 cm (d) 15 cm I. The line segments joining the mid-points of the
(a) 30° (b) 45° (c) 60° (d) 90° two pairs of opposite sides bisect each other at
56º
40. In the given figure, M is the mid-point of the the point of intersection.
B 33. The middle points of the parallel sides AB and side CD of the parallelogram ABCD. What is
C II. The area of the quadrilateral formed by joining
CD of a parallelogram ABCD are P and Q, ON : OB ? B
(a) 90° (b) 60° (c) 56° (d) 62° A the mid-points of the four adjacent sides is half
respectively. If AQ and CP divide the diagonal O of the total area of the quadrilateral.
26. One angle of a pentagon is 140°. If the remaining BD into three parts BX , XY and YD, then which D C Which of the statement(s) given above is/are
one of the following is correct? M
angles are in the ratio 1 : 2 : 3 : 4, then the size correct?
of the greatest angle is (a) BX ≠ XY ≠ YD (b) BX = YD ≠ XY N (a) Only I (b) Only II
(a) 150° (b) 180° (c) 160° (d) 170° (c) BX = XY = YD (d) XY = 2 BX (a) 3 : 2 (b) 2 : 1 (c) 3 : 1 (d) 5 : 2 (c) Both I and II (d) Neither I nor II
MATHEMATICS > Quadrilateral and Polygon 279 280 CDS Pathfinder

PREVIOUS YEARS’ QUESTIONS 54. ABCD is a trapezium with parallel sides 62. If a star figure is formed by elongating the sides of 66. If each interior angle of a regular polygon is 140°
AB = 2 cm and DC = 3 cm. E and F are the a regular pentagon, then the measure of each angle then number of vertices of the polygon is
Directions (Q.Nos. 46-47) Let ABCD be a quadrilateral, mid-points of the non-parallel sides. The ratio at the angular points of the star figure is e 2015 I equal to e 2016 I
the diagonals AC and BD meet at O and the of area of ABFE to area of EFCD is e 2013 I (a) 36° (b) 35° (c) 32° (d) 30° (a) 10 (b) 9 (c) 8 (d) 7
perpendicular drawn from A to CD, meet CD at E. (a) 9 : 10 (b) 8 : 9 (c) 9 : 11 (d) 11 : 9
Further, AO : OC = BO : OD, AB = 30 cm, CD = 40 cm 63. If X is any point within a square ABCD and on 67. Consider the following statements.
and the area of the quadrilateral ABCD is 1050 cm 2.
55. If the diagonals of a rhombus are 4.8 cm and AX a square AXYZ is described, which of the I. There exists a regular polygon whose exterior
1.4 cm, then what is the perimeter of the following is/are correct? angle is 70°.
46. What is the value of BE? e 2012 I rhombus? e 2013 II I. BX = DZ or BZ = DX II. Let n ≥ 5, then the exterior angle of any regular
(a) 30 cm (b) 30 2 cm (a) 5 cm (b) 10 cm (c) 12 cm (d) 20 cm II. ∠ABX = ∠ADZ or ∠ADX = ∠ABZ polygon of n sides is acute.
(c) 30 3 cm (d) None of these
56. ABCD is a quadrilateral such that BC = BA and Select the correct answer using the code given Which of the above statements is/are correct?
47. What is the area of the ∆ADC ? e 2012 I CD > AD. Which one of the following is correct? below. e 2015 II e 2016 I
(a) 300 cm 2 (b) 450 cm 2 e 2013 II (a) Only I (b) Only II (a) Only I (b) Only II
(c) 600 cm 2
(d) None of these (a) ∠BAD = ∠BCD (b) ∠BAD < ∠BCD (c) Both I and II (d) Neither I nor II (b) Both I and II (d) Neither I nor II
(c) ∠BAD > ∠BCD (d) 2∠BAD = ∠BCD
48. ABCD is a rhombus with diagonals AC and BD. 64. In the adjoining figure, A B 68. Consider the following statements.
57. Let ABCD be a parallelogram. Let P, Q, R and ABCD is a parallelogram. Q
Then, which one among the following is correct? I. If n ≥ 3 and m ≥ 3 are distinct positive integers,
e 2012 I
S be the mid-points of sides AB, BC, CD and P is a point on BC such P then the sum of the exterior angles of a regular
(a) AC and BD bisect each other but not necessarily DA, respectively. that PB : PC = 1 : 2. DP polygon of m sides is different from the sum of
Consider the following statements. and AB when both D C
perpendicular to each other the exterior angles of a regular polygon of n sides.
(b) AC and BD are perpendicular to each other but not I. Area of triangle APS < Area of triangle DSR, produced meet at Q. If area of ∆BPQ is 20 sq II. Let m, n be integers such that m > n ≥ 3. Then,
necessarily bisect each other if BD < AC. units, the area of ∆DCP is e 2015 I the sum of the interior angles of a regular
(c) AC and BD bisect each other and perpendicular to II. Area of triangle ABC = 4 (Area of triangle BPQ). (a) 20 sq units (b) 30 sq units polygon of m sides is greater than the sum of the
each other Select the correct answer using the codes given (c) 40 sq units (d) None of these interior angles of a regular polygon of n sides
(d) AC and BD neither bisect each other nor perpendicular below e 2014 I and their sum is (m + n )π / 2.
to each other (a) Only I (b) Only II
65. ABCD is a parallelogram with AB and AD as
adjacent sides. If ∠A = 60° and AB = 2 AD , then Which of the above statements is/are correct?
49. The sides of a parallelogram are 12 cm and 8 cm (c) Both I and II (d) Neither I nor II e 2016 I
the diagonal BD will be equal to e 2015 II
long and one of the diagonals is 10 cm long. If d is 58. In a trapezium, the two non-parallel sides are (a) Only I (b) Only II
(a) 2 AD (b) 3AD (c) 2 AD (d) 3AD
the length of other diagonal, then which one of the equal in length, each being of 5 cm. The (c) Both I and II (d) Neither I nor II
following is correct? e 2012 I parallel sides are at a distance of 3 cm apart. If
(a) d < 8 cm (b) 8 cm < d < 10 cm the smaller side of the parallel sides is of
(c) 10 cm < d < 12 cm (d) d >12 cm length 2 cm, then the sum of the diagonals of ANSWERS
50. Let X be any point within a square ABCD. On AX a the trapezium is e 2014 I
(a) 10 5 cm (b) 6 5 cm (c) 5 5 cm (d) 3 5 cm 1 c 2 b 3 c 4 b 5 d 6 a 7 d 8 a 9 c 10 b
square AXYZ is described such that D is within it.
Which one of the following is correct? e 2012 II 59. Two light rods AB = a + b, CD = a − b 11 a 12 b 13 a 14 b 15 a 16 d 17 c 18 c 19 b 20 c
(a) AX = DZ (b) ∠ADZ = ∠BAX symmetrically lying on a horizontal plane. 21 b 22 d 23 a 24 a 25 d 26 c 27 a 28 b 29 c 30 a
(c) AD = DZ (d) BX = DZ They are kept intact by two strings AC and 31 b 32 d 33 c 34 c 35 c 36 b 37 d 38 a 39 b 40 b
51. ABCD is a parallelogram. If the bisectors of the BD. The perpendicular distance between rods 41 a 42 a 43 b 44 a 45 c 46 b 47 c 48 c 49 d 50 d
∠ A and ∠C meet the diagonal BD at points P and is a. The length of AC is given by e 2014 I
51 b 52 a 53 c 54 c 55 b 56 c 57 b 58 b 59 d 60 b
Q respectively, then which one of the following is (a) a (b) b (c) a2 − b 2 (d) a2 + b 2
61 c 62 a 63 c 64 d 65 b 66 b 67 b 68 d
correct? e 2012 II
60. Let ABCD be a parallelogram. Let X and Y be
(a) PCQA is a straight line (b) ∆APQ is similar to ∆PCQ
the mid-points of the sides BC and AD,
(c) AP = CP (d) AP = AQ
52. The locus of a point in rhombus ABCD which is
respectively. Let M and N be the mid-points of
the sides AB and CD, respectively. HINTS AND SOLUTIONS
equidistant from A and C is e 2012 II Consider the following statements.
1 1 3. (c) In ∆ABC , ∆ACD , ∆BCD and ∆ABD
(a) a fixed point on diagonal BD I. The straight line MX cannot be parallel to YN . 1. (c) As ∠A + b ° = 180° [linear pair] 2. (b) Q ∠1 = ∠C and ∠2 = ∠D
⇒ ∠A = 180° − b 2 2 AB + BC > AC , CD + DA > AC
(b) diagonal BD (c) diagonal AC II. The straight lines AC, BD, XY and MN meet In ∆DEC , ∠1 + ∠2 + ∠CED = 180°
Also, ∠C + a ° = 180° [linear pair] BC + CD > BD, DA + AB > BD
(d) None of the above at a point. [Sum of any two sides is greater, than
[angle sum property of a triangle]
P Q Which of the statement(s) given above is/are ⇒ ∠C = 180° − a ° the third side.]
53. In the figure given below, PQRS correct? ∴ ∠CED = 180° − ( ∠1 + ∠2)
C e 2014 II But ∠A + ∠B + ∠C + ∠D = 360° adding above inequalities,
is a parallelogram. If AP, AQ, CR (a) Only I (b) Only II In quadrilateral ABCD, 2 ( AB + BC + CD + DA ) > 2
and CS are bisectors of D B [by angle sum property of a ( AC + BD )
(c) Both I and II (d) Neither I nor II quadrilateral] ∠A + ∠B + ∠C + ∠D = 360°
∠P , ∠Q , ∠R and ∠S respectively, ( AB + BC + CD + DA ) > ( AC + BD )
A 61. If each interior angle of a regular polygon is ∠A + ∠B + 2 ( ∠1 + ∠2) = 360°
then ABCD is a e 2013 I ⇒ ( 180°− b ° ) + x°+ ( 180°− a ° ) + y ° 4. (b) Let h be the height of the
S R 135°, then the number of diagonals of the ∠A + ∠B = 360° − 2 ( ∠1 + ∠2) parallelogram. Then, clearly h < q
(a) square (b) rhombus = 360°
polygon is equal to e 2015 I ∠A + ∠B = 2 [ 180° − ( ∠1 + ∠2)] ∴ R= p × h< p × q = S
(c) rectangle (d) None of these ⇒ x° + y ° = a ° + b °
(a) 54 (b) 48 (c) 20 (d) 18 ∠A + ∠B = 2 ∠CED R< S
MATHEMATICS Quadrilateral and Polygon 281 282 CDS Pathfinder

5. (d) Two parallelograms on the same base + 2 AB ( AB − BP − AQ ) In ∆OCX , 18. (c) By condition, 23. (a) ∆ADM ~ ∆DCL So, ∆ ABE is an equilateral triangle.
and between the same parallels are equal = ( BC 2 + AD 2 ) + 2 AB ⋅ PQ ∠OCX + ∠COX + ∠OXC = 180° Interior angle of a regular polygon Since, ∠DMA = ∠CLD = 90° and Now, perimeter of ∆ABE = 6
in area. So, the ratio of their areas is [Q P Q = AB − AQ − B P] [by angle sum of property of a triangle] Exterior angle a regular polygon ∠CDL = ∠DAM as they are alternate ⇒ AB + BE + EA = 6
1: 1. angles
= BC 2 + AD 2 + 2 AB ⋅ CD ⇒ 45° + 75° + ∠OXC = 180° 5 ⇒ AB = 2 units
6. (a) As ABCD is a rhombus. =
[Q PQ = DC ] ⇒ ∠OXC = 180° − 120° = 60° 1 D C Now, In ∆ADE and ∆BCE,
So, AO = OC = AC
1
⇒ x = 60° ( n − 2) ∠ADE = ∠BCE [each 90°]
9. (c) Let PQ be the perpendicular × 180°
2 5 ( n − 2) 5
distance between the two parallel sides 13. (a) EB || DL and ED || BL ⇒ n = ⇒ = ∠DAC = ∠CBE [each 30°]
D C AB and CD. [Q PO = 2OQ ] 360° L
⇒ EBLD is a parallelogram.
1 2 1 and AE = BE
O D Q C n ∴ ∆ ADE =~ ∆BCE [By AAS]
1 1
∴ BL = ED = AD = BC = LC ⇒ n − 2 = 10 ⇒ n = 12 M A B
2 2 ⇒ DE = CE [By CPCT]
O
Also, in ∆DCL and ∆FBL , 19. (b) Here, n = 6 ( AD / CD ) = (DM / CL ) In ∆ ADE by pythagoras theorem,
A B DM × CD 12 × 16
1 LC = BL , ∠DLC = ∠FLB Sum of interior angles of a hexagon ⇒ AD = = AE 2 = AD 2 + ED 2
BO = OD = BD, ∠AOB = 90° = ( n − 2) 180° = ( 6 − 2) × 180° CL 15
2 A
P B Q [vertically opposite angles] ⇒ 4 = AD 2 + 1
∴ AB 2 = OA 2 + OB 2 1 = 4 × 180° = 720° = 12 .8 units
AB × OP and ∠CDL = ∠BFL [alternate angles] (Q E is mid-point of CD)
2
BD 2 Area of ∆AOB Number of angles = 6 24. (a) Evidently DP = 14 − 9 = 5 cm
AB 2 =
AC
+ So, = 2 ∴ ∆DCL ≅ ∆FBL [By AAS rule] ⇒ AD = 3 units
Area of ∆COD 1 ∴ Each angle of a regular hexagon
4 4 CD × OQ ∴ DC = BF and DL = FL From ∆BPC , Hence, area of quadrilateral
720°
⇒ 4 AB 2 = AC 2 + BD 2 2
∴ BF = DC = AB = = 120° BC 2 = 152 − 92 = 122 ABCD = AB × AD
2CD × 2OQ 4 6
7. (d) Draw EF || AB. = = or 4 : 1 ⇒ 2 AB = 2DC ⇒ BC = 12 cm = 2 × 3 = 2 3 sq units
In right angled ∆EOA and ∆OCF , CD × OQ 1 20. (c) Each interior angle of a regular From ∆APD , AP 2 = AD 2 + DP 2
⇒ AF = 2DC [Q BF = AB] polygon
28. (b) ABCD is square and ABEF is a
OA 2 = OE 2 + AE 2 … (i) = 122 + 52 = 169 rhombus.
10. (b) Area of parallelogram ABCD 14. (b) Join BD, cutting EF at M. ( n − 2) × 180°
= ⇒ AP = 13 cm FM 1
and OC 2 = OF 2
+ CF 2 … (ii) = 2 (Area of ∆ADE ) D C n = sin30° =
( n − 2) × 180° In ∆ABP , AP < AB < BP. AF 2
D C + (Area of rectangle AECH ) ∴ = 150° (given) Therefore, γ < β < α (i.e.) α > β > γ AF
1
= 2 × DE × AE + AE × EC
E F n ∴ FM = , AF = AB
E F M 25. (d) Since, AB = BC
O 2 ( n − 2) 180 = n × 150 2
A B = DE × AE + AE × EC A B
⇒ 30n = 360 ⇒ n =
360 ∴ ∠BAC = ∠BCA = 62° D C

On adding Eqs. (i) and (ii), we get = AE (DE + EC ) = AE × [DC ] So, M is mid-point of BD. 30 Also as AB||CD and AC transversal
∴ OA 2 + OC 2 = OE 2 + AE 2 DE 1 DE 1 ⇒ n = 12 So, ∠BAC = 62° = ∠ACD F
Q = ⇒ + 1= + 1 [Q E is mid-point of AD and E
+ OF 2 + CF 2 …(iii) EC 2 EC 2 EM || AB] 21. (b) By given condition, (alternate interior angles)
DE + EC 3 DC 3
In right angled ∆DEO and ∆OBF , ⇒ = ⇒ = ∴ EM = AB
1
…(i)
N M ∴ ∠ACD = 62°
OD = OE + DE
2 2 2
… (iv) EC 2 EC 2 2
Area of parallelogram 26. (c) One angle of the pentagon is 140°. 30°
OB 2 = OF 2 + BF 2 …(v) Similarly, F is mid-point of BC in A
M B
FE 3EC FE × EC ∆BCD. Since, the remaining angles are in the
On adding Eqs. (iv) and (v), we get ABCD = × =
3 2 2 1 ratio 1 : 2 : 3 : 4. Area of square = a 2 (AB = AD = a)
⇒ Area of || gm ABCD ∴ FM = DC …(ii) a×a
[Q FE = 3 AE ] ∴ Let the remaining angles be x °, Area of rhombus =  FM = a 
⇒ OD 2 + OB 2 = OE 2 + OF 2 2 P
6 cm R L  
Area of rectangle EFGC 1 2  2
+ DE 2 + BF 2 …(vi) = ∴ EF = EM + MF = ( AB + DC ) Area of parallelogram LMNP ( 2x )° , (3x )° and ( 4x )°.
As FB = EA and DE = CF 2 2 But the sum of interior angles of a (Area of rhombus = base × height)
= 6 × Area of ∆NPR
∴ OD 2 + OB 2 = OE 2 ∴ Required ratio = 1 : 2 [from Eqs. (i) and (ii)] pentagon Area of square 2
1
∴ NR × PL = 6 × × NR × PR ∴ =
+ OF 2 + CF 2 + AE 2 …(vii) 11. (a) Join AC and BD. 15. (a) ∠CRQ = ∠CBA = 90° ⇒ QR || AB 2 ( 2 × 5 − 4) × 90 = 6 × 90° = 540° Area of rhombus 1
Here, from Eqs. (iii) and (vii), we get As AB > BC In ∆ABC , Q is the mid-point of AC and ⇒ PL = 3PR ∴ 140 + x + 2x + 3x + 4x = 540 29. (c) In ∆ARB, P is the mid-point of AB
OA + OC 2 2
= OD + OB2 2 ⇒ ∠ACB > ∠BAC …(i) QR || AB. ⇒ PR + RL = 3PR [Q PL = PR + RL] ⇒ 10x = 400 ⇒ x = 40 and PD||BR.
Also, AD > DC So, R is mid-point of BC . ⇒ RL = 2PR = 2 × 6 = 12 cm ∴ The angles of the pentagon are 140°, ⇒ D is the mid-point of AR .
8. (a) Draw DQ⊥AB and CP⊥AB.
⇒ ∠ACD > ∠CAD …(ii) Similarly, P is the mid-point of DC . 22. (d) ABCD is a trapezium in which 40°, 80°, 120° and 160°. Q ABCD is a parallelogram.
In ∆ABD , ∠A is acute.
D C On adding Eqs. (i) and (ii), we get ∴ DP = PC AB||DC and M, N are the mid-points of Hence, the size of the greatest angle ⇒ DC || AB ⇒ DQ|| AB
∴ ∠ACB + ∠ACD > ∠BAC + ∠CAD 16. (d) Number of diagonals of a polygon of AD and BC . = 160°
Thus, in ∆RAB, D is the mid-point of
⇒ ∠C > ∠A , similarly ∠D > ∠B n ( n − 1) Hence, MN || AB and MN ||DC . 27. (a) Given that, AB || DC and AD || BC AR and DQ|| AB.
8 sides = −n In ∆ACB, ON passes through the
12. (b) ∠COD = 105° 2 In ∆ ABE , ∴Q is the mid-point of RB ⇒ BR = 2BQ
A B 8 (8 − 1) mid-point N of BC and ON || AB
Q P ∴ ∠BOA = 105° = − 8 = 20 1 1 A B 30. (a) In ∆BDC , Q and R are the
2 ∴ ON = AB = × 12 cm = 6 cm 60° 60°
So, BD 2 = AD 2 + AB 2 − 2 AB ⋅ AQ [vertically opposite angles] 2 2 mid-points of BD and CD respectively.
…(i) 17. (c) Sum of all angles of a pentagon And, MO = MN − ON = ( 14 − 6) cm 1
∴ ∠AOD = ∠COX = λ ∴QR||BC and QR = BC
In ∆ABC , ∠B is acute. = (5 − 2) × 180° = 3 × 180° = 540° = 8 cm 2
⇒ 105° + 105°+ x + x = 360° 1
So, AC 2
= BC + AB − 2 AB ⋅ BP
2 2 Let the angle be x , 2x , 3x , 5x and 9x. Again, MO passes through the Similarly, PS||BC and PS = BC
⇒ 2x = 150° ∴ x + 2x + 3x + 5x + 9x = 540° mid-point M of AD and MO||DC 2
…(ii) D C
⇒ x = 75° 20x = 540° ⇒ x = 27° ∴
1
MO = DC = CD
1 E ∴ PS||QR and PS = QR
Adding Eqs. (i) and (ii), we get 1 1 2 2 ∠EAB = ∠ABE = 60°  each equal to 1 BC 
Also, ∠OCX = × ∠C = × 90° = 45° ∴ Largest angle = 9x = 9 × 27° = 243°
∴ AC 2 + BD 2 = ( BC 2 + AD 2 ) 2 2 Hence, CD = 2( MO ) = 2(8) = 16 cm ⇒ ∠AEB = 60°  2 
MATHEMATICS Quadrilateral and Polygon 283 284 CDS Pathfinder

Similarly, PQ||SR and PQ = SR 35. (c) Let the sides of two parallelograms In ∆ FGD, DN = BC = AD (by CPCT) 43. (b) I. All squares are parallelograms. OC AB 30 3
⇒ = = =
[each equal to 1 / 2 AD] are x, y and xk , yk, respectively. ∠1 + ∠F + ∠D = 180° AN 2 II. No parallelogram is a trapezium. OA CD 40 4
So, AN = 2BC ⇒ = …(i)
C E T S (by angle sum of property of a triangle) BC 1 ∴ ∠OAB = ∠OCD
∴PS = QR = SR = PQ [Q AD = BC ] ⇒ 65° + 80° + ∠D = 180°
III. All squares are rhombuses and also
In ∆OAN and ∆OBC , rectangles. and ∠OBA = ∠ODC
Hence, PQRS is a rhombus. y y yk ⇒ ∠D = 35° It means DC || AB.
∠5 = ∠6 (vertically opposite angle) IV. All rhombuses are parallelograms.
31. (b) AB||DC and EC is a transversal 38. (a) Let ∠XOB = θ So, it is a trapezium.
∠7 = ∠8 (alternate interior angle) So, statements (III) and (IV) are
⇒ ∠BEC = ∠ECD D C Area of trapezium
A B D P Q R correct.
⇒ ∠BEC = ∠ECB[Q ∠ECD = ∠ECB] x xk ∠9 = ∠10 (altenate angle)
1
⇒ EB = BC ⇒ AE = AD ∴ ∆OAN ~ ∆OBC (by AAA similarity) 44. (a) In a rhombus ABCD, if AC and BD ABCD = ( AB + CD ) × AE
Since, sides of two parallelogram are in 2
Now, AE = AD ⇒ ∠ADE = ∠AED O are two diagonals then
the ratio 1 : k. So, the sides will be in same ratio 1
⇒ ∠ADE = ∠EDC AB 2 + BC 2 + CD 2 + AD 2 ⇒ 1050 = (30 + 40) × AE
∴ ∆ ABC ~ ∆ PQT AN ON 2 ON
θ = ⇒ = or 2 : 1 2
[Q alternate Int. angles]

AC
=
BC BC OB 1 OB = AC 2 + BD 2
⇒ AE = 30 cm
∴DE bisects ∠ADC PT QT A B ⇒ ( 10)2 + ( 10)2 + ( 10)2 + ( 10)2
[from Eq. (i)]
Again, ∠ADC + ∠BCD = 180° BC y 1 X 46. (b) In right ∆EAB, EB = AE 2
+ AB 2
⇒ = = 41. (a) As ABCD is a parallelogram. = ( 16)2 + ( 12)2
[Co. Int. angles] QT yk k In ∆OXB, = 302 + 302 = 30 2 cm
∴ AB||CD and AB = CD ⇒ 400 = 400
1 1 BC = z and QT = zk ∠XOB + ∠OBX + ∠OXB = 180°
⇒ ∠ADC + ∠BCD = 90° Let D C 1
47. (c) Area of ∆ ADC = × CD × AE
2 2 ∴ Ratio of areas of two similar (by angle sum of property of a Draw QR ⊥ AB m
6 cm 2
⇒ ∠EDC + ∠DCE = 90° x×z 1 triangle) In ∆DOQ and ∆POB, 8c 1
parallelograms = = or 1 : k 2 ⇒ θ + 45° + ∠OXB = 180° = × 40 × 30 = 600 cm 2
But, ∠EDC + ∠DEC + ∠DCE = 180° xk × zk k 2 ∠ODQ = ∠OPB M 2
⇒ ∠OXB = 180° − 45° − θ = 135° − θ
[Q sum of the ∠ S of a ∆ is 180°] ∠OQD = ∠OBP 48. (c) ABCD is a rhombus.
1 ∠OXA + ∠OXB = 180° m 90°
∴ ∠DEC = 180°−90° 36. (b) Area of ( ∆PRQ ) = Area of (WXQR) Here,
and OD = OB 8c 6 cm
∴ AB = BC = CD = DA
∴ ∠DEC = 90° 2 (linear pair)
R ⇒ ∠OXA + 135° − θ = 180° ∴∆OQD ≅ ∆OPB [By AAS] A D
32. (d) In ∆APS and ∆PBQ, [given] Z Y
⇒ ∠OXA = 45° + θ
A 10 cm B
⇒ PB = DQ
PB = AS, AP = BQ
D C In ∆OXA, AO = AX (given) Q Hence, both I and II are true but III is
R D C
S
R Q ∴ ∠OXA = ∠AOX = 45° + θ false.
L K Since, ∠AOX + ∠XOB = 90° 45. (c) PQRS can be shown parallelogram, so B C
Q W X ⇒ 45° + θ + θ = 90° O the diagonal PR and SQ bisect each and diagonals bisect each other at right
P ⇒ 2θ = 45° ⇒ θ = 22.5° other. angles.
1 1 D R C
A B = Area of (WXYZ ) 39. (b) Area of parallelogram 49. (d) In parallelogram,
P 2  2 
B
= Base × height = 15 × 5 = 75 cm 2 A R P d 2 + d 2 = 2 (l 2 + b2 )
and ∠A = ∠B = 90° 1 S Q
= Area of (WXYZ ) …(i) 2
∴ ∆APS ≅ ∆BQP (by SAS rule) 4 D C Now, ar ( || gm ABCD) = QR × AB D C
∴ SP = PQ, ∠SPA = ∠BQP Area of ( ∆PRQ ) RP 2 = QR × ( AP + PB ) = QR × ( AP + DQ ) d
= A P B
cm

and ∠ASP = ∠BPQ Area of (∆PLK ) LP 2 5 cm N = 2 × ar (quadrilateral APDQ)


II. Area ( ∆RSQ ) b = 12 cm
10

∠SPQ = 180°−∠APS − ∠BPQ (by properties of similar triangle) 42. (a) Draw MN⊥AB and let MN = h 1 0c
m
Area of ( ∆PRQ ) ( 2LP )2 = Area ( SQCD ) …(i) =1
= 180°−( ∠APS + ∠ASP ) A B D L M C 2 d2
⇒ = M
1 A l = 8 cm B
= 180°−90° = 90° Area of (∆PLK ) LP 2 15 cm
and area ∆( PSQ ) = Area
Area of parallelogram = Base × Height 2 ∴ d 2 + ( 10)2 = 2 ( 64 + 144)
33. (c) As ABCD is a || gm [Q L and K are the mid-point of PR = 10 × DN h ( ABQS )…(ii) ⇒ d = 2 × 208 − 100
2
∴ AB||DC and AB = DC and PQ] On adding Eqs. (i) and (ii), we get
∴ 10 × DN = 75 ⇒ d 2 = 416 − 100 = 316
1
and AB = DC
1 D Q C ⇒ Area of ( ∆PRQ ) = 4Area of ( ∆PLK ) 1
1 ⇒ DN =
75
= 7. 5 cm Area ( RSQ ) + Area ( PSQ ) =
2 2 ⇒ Area of (WXYZ ) 10 A B 2 ⇒ d = 316 ⇒ d = 17.76 cm
⇒ AP = QC Y 4 N
[Area ( SQCD ) + Area ( ABQS )]
X 40. (b) In ∆DMN and ∆BMC , ∴ d > 12
∴ APCQ is a || gm = 4Area of ( ∆PLK ) [from Eq. (i)] Now, ar (trapezium ABCM) 1
Area of ( ∆PLK ) ⇒ Area ( PQRS ) = Area ( ABCD ) 50. (d) In ∆ABX and ∆ADZ,
⇒ AQ||PC 1 DM = MC (given) = ar ( ∆ AMC ) + ar ( ∆ABC ) 2
A P B ⇒ = A B AB = AD (side of a square ABCD)
In ∆BAY , XP|| AY Area of (WXYZ ) 16 1 1 Hence, both statements are true.
10 O ⇒ ( MC + AB ) × h = 4 + ar
and P is the mid-point of AB
7 2 2 Solutions (Q.Nos. 46-47) and AX = AZ (side of a square AXYZ )
37. (d) Here,∠B = ∠C = 65° [Q AB = AC ] 5 69
∴ BX = YX 24 ( parallelogram ABCD) Given, AO :OC = BO :OD Let ∠BAX = θ
A F D C
Similarly, in ∆DXC ,DY = YX 1 and AB = 30 cm and CD = 40 cm
⇒  + AB  × h = 4 + × AB × h ∴ ∠XAD = 90°− θ
3 M 1 AB 1
∴ BX = XY = DY 80° 8 2 4  2 A 30 cm B A B
34. (c) If a parallelogram and a rectangle
E Q MC = AB 
X
stand on the same base and on the same N  4  O Z
side of the base with the same height, ∠1 = ∠2 (vertically opposite angle) AB × h C
then perimeter of parallelogram is 65° 1 65° ⇒ = 4 ⇒ AB × h = 32 D
greater than perimeter of rectangle. ∠3 = ∠4 + ∠9 8 D C
B G C D E 40 cm Y
∴ l >l (alternate interior angle) ⇒ ar (||gm ABCD) = 32 cm 2
1 2
∠1 = ∠B = 65° (corresponding angles) Also, AXYZ is a square.
∆ DMN ≅ ∆BMC (as ASA rule) ∴ ∆AOB ~ ∆COD
MATHEMATICS Quadrilateral and Polygon 285 286 CDS Pathfinder

∴ ∠ZAX = 90° 54. (c) Join AC. In ∆ACD, EG || DC and E 57. (b) In ∆APS and ∆DSR Now, in ∆ACE, x2 + a 2 = AC 2 Since, GAF is a straight line. 65. (b)
D C
⇒ ∠ZAD + ∠XAD = 90° and G are mid-points of AD and AC, AP = DR [Q P and R are mid points] ⇒ AC 2 = b 2 + a 2
respectively. F
⇒ ∠ZAD = 90° − ( 90° − θ) = θ AS = DS [Q S is the mid point] ∴ AC = b + a
2 2
In ∆ADC , EG ||DC
i.e. ∠BAX = ∠ZAD 1 3 and SP = SR [Q P or RS us a] A
108°
∴ EG = DC = ∴ ∆APS =~ ∆DRS [By SSS] 60. (b) Given, ABCD is a parallelogram. X G
∴ ∆ABX ≅ ∆ADZ 2 2 108°
A
60° B
⇒ Area of ∆APS = Area of ∆DSR and Y are mid-points of BC and AD, B
∴ BX = DZ (by CPCT) A B
respectively. M and N are the C
h R Let AD = x , then AB = 2x
51. (b) Since, line segment AP and CQ D C mid-points of AB and CD, respectively.
bisects the ∠ A and ∠C , respectively.
E
G
F using cosine rule in ∆ABD, we have
h N
S Q
D C ∴ ∠GAF = 180° AD 2 + AB 2 − BD 2
Then, AP ||CQ D C cos A =
⇒ ∠GAB + ∠BAF = 180° 2 ⋅ AD ⋅ AB
Now, in ∆APQ and ∆CQP, A B
Y X
Similarly, in ∆ABC P ⇒ ∠GAB + 108° = 180° x 2 + ( 2x )2 − BD 2
Q AP ||QC 1 ⇒ cos 60° =
GF = AB = 1 Q AS = SD and AP = DR A B ⇒ ∠GAB = 180°−108° = 72° 2. x .( 2x )
D C 2 M
3 5 In ∆ABC and ∆PBQ Similarly, 1 5x 2 − BD 2
P EF = EG + GF = 1 + = ⇒ =
∠ABC = ∠PBQ [common] Now, join AC . 2 4x 2
2 2 ∠GBA = 180°−108° = 72°
∴ Area of trapezium =
1 ∠BAC = ∠BPQ [QPQ|| AC ] In ∆ABC , M and X are mid-points of ⇒ 2x 2 = 5x 2 − BD 2
In ∆ABG ,
Q 2 ∴ ∆ABC ~ ∆PBQ AB and BC .
∠GAB + ∠ABG + ∠BGA = 180° ⇒ BD 2 = 3x 2
(sum of parallel sides × height) ar ( ∆ABC ) AB 2
2 1
∴ MX || AC and MX = AC ...(i)
⇒
2PB 
Area of ABFE = ⇒ 72°+72°+∠BGA = 180° ⇒ BD = 3x = 3 AD.
 PB 
A B Now, required ratio = 2
ar ( ∆PBQ ) PB 2
Area of EFCD In ∆ADC , Y and N are mid-points of ⇒ ∠BGA = 180°−144°, ∠BGA = 36° 66. (b) Given, each interior angle = 140°
Q ∠APQ = ∠PQC (alternate angle) ∴ ar ( ∆ABC ) = 4 ar (∆BPQ)
1 5 AD and CD. 63. (c) Here, ∠DAB = ∠XAZ = 90°
PQ = PQ (common) 2 +  × h Then, each exterior angle
 2 9 Hence, only statement II is correct. 1
Also, PC || AQ = 2 = or 9 : 11 ∴ YN || AC and YN = AC ...(ii) ⇒ ∠DAB − ∠XAB = ∠XAZ − ∠XAB = ( 180°−140° ) = 40°
1 5 11 58. (b) 2
3 +  × h D C A D
∴ Number of sides
∴ ∠CPQ = ∠PQA (alternate angle) 2 2 From Eqs. (i) and (ii), we get
2 Z 360° 360°
∴ ∆APQ ~ ∆CQP (by ASA) 55. (b) Here, OD =
BD 48
=
.
= 2.4 MX || YN = = =9
2 2 X Each exterior angle 40°
Hence, ∆APQ is similar to ∆PCQ. 5 3 5 So, statement I is not correct. Y
AC 14. 3
52. (a) We know that, in a rhombus ABCD ⇒ OC = = = 0.7 II Obviously, straight lines Hence, the number of vertices of
2 2 AC , BD , XY and MN meet at a point. polygon is 9.
diagonals bisect each other at point O A D
So, statement II is correct. B C
which means that the distance of O from B 67. (b) Exterior angle of regular polygon
A 4 E F
four vertices of rhombus i.e., A, B, C and 2 4 61. (c) Given, each interior angle of a regular ⇒ ∠DAX = ∠BAZ 360°
D are equal. Even, if we take any fixed 90°
polygon = 135° Now, in ∆DAX and ∆BAZ , =
O In ∆BCF , Number of sides
point on diagonal BD of rhombus AD = AB (sides of squares)
By pythagoras theorem, Q Each exterior angle
ABCD and join with vertices A and C, B C I. If exterior angle is 70°
we get a point which is equidistant from (5)2 = (3)2 + ( BF )2 ⇒ BF = 4 cm = 180° − interior angle ⇒ ∠DAX = ∠BAZ (proved earlier)
Since, in rhombus diagonal bisect at 90°. ⇒ AX = AZ (sides of square) Then, number of sides
A and C. (by property of congruent ∴ AB = 2 + 4 + 4 = 10 cm = 180° − 135° = 45° 360° 36
triangle). Hence, the locus of a point in Then, in ∆ODC , OD + OC = CD2 2 2
⇒ ∆DAX ≅ ∆BAZ (SAS criterial) = = = A rational number
Now in ∆ACF , AC 2 = CF 2 + FA 2 Q Number of sides in the polygon ( n) 70° 7
rhombus ABCD which is equidistant ⇒ CD = OD 2 + OC 2 ⇒ BZ = DX (by CPCT)
⇒ AC 2 = 32 + 62 , AC = 45 cm 360° 360°
from A and C is a fixed point on = ( 2.4)2 + ( 0.7)2 = = =8 So, statement I is incorrect.
Similarly, BD = 45 cm Each exterior angle 45° and ∠ABZ = ∠ADX (by CPCT)
diagonal BD. 5 II. If number of sides ≥ 5, then
CD = 6.25 ⇒ CD = 2.5 = ∴ Sum of diagonal = 2 × 45 Hence, both statements are correct.
D C 2 Q Number of diagonals of a polygon 360°
= 2 × 3 5 = 6 5 cm PB 1 Exterior angle = = 72°
O2 D C ∴ Perimeter of rhombus n( n − 1) 8(8 − 1) 64. (d) Given as = …(i) 5
O1 5 59. (d) Since, they are symmetrically lying = −n= −8 PC 2
= 4a = 4 × = 10 cm 2 2 BQ||DC (Q AB||DC ) For number of sides ≥ 5, there is
2 on horizontal plane.
O3 8×7 always an acute exterior angle.
56. (c) Join AC. A = −8 In ∆BPQ and ∆DPC
O4 2 So, statement II is correct.
Now, in ∆ABC x ∠BPQ = ∠DPC
A B A B A B = 28 − 8 = 20
C (Q vertically opposite angles) 68. (d) I. In regular polygon,
QAB = BC (given) E
53. (c) In parallelogram PQRS Hence, the number of diagonals of a ⇒ ∠QBP = ∠PCD Sum of exterior angles = 360°
∴ ∠BAC = ∠BCA
a−b

a−b

∠P + ∠S = 180° a polygon is 20. (Q alternate interior angle) [which is always constant]


1 1 ...(i)D C
⇒ ∠P + ∠S = 90° (angles opposite to equal side) 62. (a) Q Each interior angle of a regular ⇒ ∆BPQ ~ ∆DPC [by AA similarity] So, statements I is incorrect.
F
2 2 D polygon of n side
In ∆ADC , x We know that area of two similar II. Sum of interior angles of m sides of
⇒ ∠DPS + ∠DSP = 90° ( n − 2) × 180° triangles are in the ratio of the square of regular polygon = (m − 2) × 180°
In ∆DPS, Q CD > AD B = the corresponding sides
∴ ∠DAC > ∠DCA n Sum of interior angles of m sides and
∠SDP = 180°− ( ∠DPS + ∠DSP ) = 90° ...(ii) 2
∴ AC = BD ∴ AE = BF = x (say) Here, n = 5 ar ( ∆BPQ )  PB  n sides of regular polygon
∴ ∠ADC = ∠SDP = 90° (since in a triangle, angle opposite = 
Now, AB = ( a − b ) + 2x ∴ Each interior angle of the pentagon ar ( ∆DPC )  PC  = (m − 2) × 180° + ( n − 2) × 180°
(vertically opposite ∠S) to greater side is bigger than the angle 2
i.e. a + b = a − b + 2x =   = (m + n − 4) × 180°
(5 − 2) × 180° 3 × 180° 20 1
Similarly, opposite to smaller side) ⇒ [from Eq. (i)]
⇒ 2b = 2x = = ar ( ∆DPC )  2 
∠DAB = ∠ABC = ∠BCD = 90° On adding Eqs. (i) and (ii), we get 5 5 So, statement II is incorrect.
Hence, ABCD is a rectangle. ∠BAD > ∠BCD ∴ x=b ∴ ar ( ∆DPC ) = 80 sq units Hence, neither I nor II is correct.
= 3 × 36° = 108°
25
MATHEMATICS Circle 287 288 CDS Pathfinder

Here, in the given figure GBH is a tangent touching the


Semi Circle circle at a single point B.
P
A diameter divides the circle into two
equal parts. Each of the two is called a Length of tangent The distance between external point
semi-circle. R O S
P from which tangent is drawn and the point of contact
of the tangent is called length of tangent.
In the above figure, RPSR and RQSR
are semi-circles. Q Properties of Tangent To a Circle
Segment • Only two tangents can be drawn from a point outside
Minor
the circle.
The area enclosed by an arc and its

CIRCLE Segment Only one tangent can be drawn through a point lying

corresponding chord is called a A B
on the circle.
segment of the circle. Major
Segment • No tangent can be drawn from a point lying inside the
• The segment containing the minor
circle.
arc is called minor segment.
• A tangent at any point of a circle is perpendicular to
• The segment containing the major
the radius through the point of contact.
arc is called major segment.

Usually (3-6) questions have been asked from this chapter. Generally questions are asked from the Sector
O
topics related to theorem of circle, tangent to a circle and locus. The area enclosed by any two radii O
and the arc determined by the end
points of the radii is called a sector of A B
Sector C
the circle. A B

A circle is a curved figure consisting of all points in a plane that Chord Circumference Relative Position of two Circles
are at a fixed distance from a fixed point. The fixed point is
called centre and the fixed distance is called radius of the circle. A chord is a line segment The length of the complete circle is called its circumference. r1 r2 r1 r2

Circumference = 2 × π × Radius of the circle


O O´ O O´ O O´
whose end points lie on the
• Circles are simple closed curves which
circle. A diameter is the or
divide the plane into two regions : an O (i) (ii) (iii)
interior and an exterior. longest chord in a circle. A B = π × Diameter of the circle
O
B A O
• In the given figure, O is the centre of Here, in the given figure PQ P Q O

circle, OA is the radius of the circle is a chord and AB is the


Concentric Circles O´ O´

and AB is the diameter of the circle. diameter of the circle which is Two or more circles having the C4 (iv) (v)
C3
the longest chord of that circle. same centre are called concentric C2 Two circles will
circles. C1
RADIUS Here, in the given figure C1 , C 2 , C 3 (i) be disjoint, when OO′ > r1 + r2 .
Radius is the fixed distance between the
B Secant and C 4 are known as concentric O (ii) be touching externally, when OO′ = r1 + r2 .
A line which intersect the circle (iii) be intersecting when OO′ < r1 + r2 .
centre of the circle and the points lying circles as they have a common
at two points is called secant of (iv) be touching internally, when OO′ = | r2 − r1 |
on the circle. centre ‘O’.
(v) one of the circle will lie inside the other, when
A the circle. O An infinite number of circles can be
Here, in the given figure OA , OB and OC O OO′ < | r2 − r1 | .
all are having fixed distance, so they are Here, in the figure AB is a drawn with same centre.
A C D B P
known as radius of the circle. It is C
secant cutting the circle at two EXAMPLE 1. Two circles with centres
denoted by r. distinct points as C and D. Tangent A and B touch each other internally,
A tangent to a circle is a straight line that touches the as shown in the figure given below.
Diameter Arc circle at a single point. Their radii are 5 and 3 units, A B
C respectively. Perpendicular bisector of
Diameter is any line segment that passes Any part of a circle between two P
A AB meets the bigger circle in P and Q.
through the centre of the circle and whose points is called an arc of the circle. B
O What is the length of PQ?
end points lie on the circle. The diameter Any two points, say A and B of a Q
A B H a. 2 6 b. 34 c. 4 6 d. 6 2
of a circle is twice the radius. Here, O circle divide it into two parts R
O
S

AB and CD are the diameters of the circle. called arcs. The smaller arc APB is B
Sol. c. Let PQ intersect AB at O. Join AP and AQ. Produce
AB = CD = 2(OA ) called minor arc and larger arc nt AB to intersect the circle at M.
ge
D an
GT
AQB is called major arc. Q
= 2(OB) = 2(OC ) = 2(OD) Here, AP = AM = 5 units, BM = 3 units
A circle can have an infinite number of diameters. It is usually denoted by APB and AQB.
MATHEMATICS Circle 289 290 CDS Pathfinder

∴ AB = AM − BM = 5 − 3 = 2 units P Theorem 7 Equal chords of congruent circles are Theorem 9 The angle subtended by an arc of a circle EXAMPLE 4. In the given figure, chords AB and CD
PQ bisects AB perpendicularly. equidistant from the corresponding centres. at the centre is double the angle subtended by it at any of a circle intersect externally at P. If AB = 6 cm, CD = 3
∴ AO = OB = 1unit point on the circumference of circle. Here, three case cm and PD = 5 cm, then the measurement of PB is
O arises.
Now, in right ∆AOP , A M O O´ A
B C C C
(PA) 2 = (OA) 2 + (OP) 2

cm
A B C D

6
P Q
⇒ ( 5) 2 = (1) 2 + (OP) 2 A B B

cm
⇒ 24 = (OP) 2 ⇒ OP = 2 6 units Q 3 cm

x
∴ PQ = 2OP = 2 × 2 6 = 4 6 units
Here, if AB = CD, then OP = O′ Q. O A O
B
O P 5 cm D C

Hence, the length of PQ is 4 6 units. Conversely if OP = O′ Q, then AB = CD.


A a. 2.5 cm b. 4 cm c. 10 cm d. None of these
Theorem 8 Equal chords of a circle are equidistant B
(i)
Important Theorems of Circles from the centre. (ii) (iii) Sol. b. Let the measurement of PB be x.
PA × PB = PC × PD ⇒ ( x + 6) × x = 8 × 5
Theorem 1 If two arcs of a circle are congruent, then Here, if AB and CD are equal chords of circle, then In all the three cases ∠AOB = 2∠ACB.
OP = OQ. ⇒ x2 + 6x − 40 = 0 ⇒ x2 + 10x − 4x − 40 = 0
the corresponding chords are equal.
EXAMPLE 3 In the figure given below, if ∠AOP = 75° ⇒ x ( x + 10) − 4( x + 10) = 0 ⇒ ( x + 10)( x − 4) = 0
Theorem 2 Equal arcs (or chords) subtend equal angle
P and ∠AOB = 120 °, then what is the value of ∠AQP ? ⇒ x = 4, as x ≠ − 10 [side cannot be negative]
at the centre. Here, if AB = PQ, then ∠AOB = ∠POQ. C D
R ∴ PB = 4 cm
B
O
A Q EXAMPLE 5. In the adjoining figure, ∆ABC is an
A
Q B
O 75° isosceles triangle with AB = AC and ∠ABC = 50 °. Then,
O
the measure of ∠BDC is
P 120°
Conversely, if OP = OQ, then also AB = CD A B A
Q
i.e. chords at equal distance from the centre are equal. D
Theorem 3 The perpendicular from the centre of a circle P
to a chord bisects the chord. EXAMPLE 2. AB and CD are two parallel chords on a. 45° b. 37.5° c. 30° d. 22.5°
Here, if OD ⊥ AB, then AD = DB. the opposite sides of the centre of the circle. If Sol. b. We know that the angle subtended by an arc of a
AB = 10 cm, CD = 24 cm and the radius of the circle is circle at the centre is double the angle subtended by it
13 cm, then what is the distance between the chords? at any point on the circumference of a circle. B C
O
a. 10 cm b. 17 cm 2∠AQP = ∠AOP
c. 24 cm d. None of these E
A B 1 75°
D ∠AQP = × ∠AOP = = 37.5° a. 80° b. 100° c. 40° d. 160°
Sol. b. From O draw OL ⊥ AB and OM ⊥ CD. Join 2 2
Theorem 4 The line joining the centre to the mid-point OA and OC. Sol. a. Given, AB = AC ⇒ ∠ACB = ∠ABC = 50°
of a chord is perpendicular to the chord. Theorem 10 The angle in a semi-circle is a right angle.
1 10 In ∆ABC, ∠BAC = 180° − ( 50° + 50° ) = 80°
Here, if AD = DB, then ∠ADO = ∠ODB = 90 °. AL = AB = cm = 5 cm, Theorem 11 Angles in the same segment of a circle are
2 2 [by angle sum property of a triangle]
Theorem 5 The perpendicular bisectors of two chords equal.
OA = 13 cm C M D
C ∴ ∠BDC = ∠BAC = 80°
of a circle intersect at its centre. In right angled ∆OLA, by pythagoras [angles lying in the same segment of the circle]
C theorem, D
O Theorem 13 Let PT be a line tangent to the circle at
A OL 2 = OA2 − AL 2 = (13) 2 − ( 5) 2 point T. From external point P, draw a secant which
O m = (169 − 25) = 144 A intersect the circle at two point A and B, then
A L B B
⇒ OL = 144 = 12 cm PA × PB = PT 2
l D 1 Here, ∠ACB = ∠ADB
B Now, CM = × CD = 12 cm B
Here, AB, CD are the chords and l, m are perpendicular 2 Theorem 12 If two chords AB and CD of a circle
bisector of AB and CD. So, l and m meet at centre ‘O’. and OC = 13 cm intersect inside or outside the circle when produced at a A
In ∆OMC, [by pythagoras theorem] point P O
Theorem 6 There is one and only one circle passing OM 2 = OC 2 − CM 2 then, AP × PB = DP × PC
through three non-collinear points.
= (13) 2 − (12) 2 P T
• An infinite number of circles can be drawn to pass A
= (169 − 144) = 25
through a single point.
⇒ OM = 25 = 5 cm O
B
EXAMPLE 6. In a ∆ABC, AB = AC. A circle through B
• An infinite number of circles can be drawn to pass O
A D P touches AC at D and intersects AB at P. If D is the
∴ ML = OM + OL mid-point of AC, then which one of the following is
through two given points.
= ( 5 + 12) cm D
correct?
• A unique circle can be drawn to pass through three P C
given non-collinear points. = 17 cm C B a. AB = 2 AP b. AB = 3AP c. AB = 4 AP d. 2 AB = 5AP
MATHEMATICS Circle 291 292 CDS Pathfinder

Sol. c. By using theorem, • Theorem 18 Tangents drawn at the end points of a EXAMPLE 9. In the given figure, if ∠PAQ = 59°, 2. The locus of points at a fixed
AB × AP = AD 2 A diameter of a circle are always parallel. ∠APD = 40 °, then what is ∠AQB ? distance d from a line mis a pair of d
P
AC  2 1 parallel lines at a distance of d m
=  Length of the direct common tangent is P d
 = ( AC ) 2 •
from line mand located on either
 2 4
D
T T ′ = (OO′ ) 2 − (r1 − r 2 ) 2 and length of transverse 40° side of m.
1
⇒ AB × AP = ( AB) 2
4 common tangent is MN = (OO′ ) 2 − (r1 + r 2 ) 2 B 3. The locus of points equidistant
B C from two points A and B is the
[Q AC = AB given]
T N T′
C perpendicular bisector of the line A B
⇒ AB = 4AP segment joining the two points.
r1 r2
Theorem 14 Alternate Segment 59°
O A D Q m1
Theorem In the given figure B, if BAC P D O´ 4. The locus of points equidistant
is a tangent to a circle at point A and if M from two parallel lines m 1 and
AD is any chord, then ∠DAC = ∠APD a. 19° b. 20° c. 22° d. 27° m2 , is a line parallel to both m 1 m2
and ∠PAB = ∠PDA. B A Sol. c. Given, ∠PAD = 59° and ∠APD = 40° and m 2 and midway between
C C them.
EXAMPLE 8. In the given figure, CP P
EXAMPLE 7. In the given figure, PAQ is the tangent. and CQ are tangent to a circle with 5. The locus of points in the interior
A
BC is the diameter of the circle. If ∠BAQ = 60 °, then 40°
of an angle equidistant from the B
D
centre O. ARB is another tangent
∠ABC is B
touching. The circle at R. If CP = 11 cm, A B
sides of an angle is the bisector of C
BC = 7 cm length of BR is P
R
Q
C the angle.
B
O a. 1 cm b. 2 cm 6. The locus of points equidistant l2
O 59°
c. 4 cm d. 3 cm A Q from two intersecting lines l1 and l1
C D l 2 is a pair of bisectors that bisect
60° Sol. c. Since, the length of two tangents In ∆APD, ∠PAD + ∠APD + ∠ADP = 180° the angles formed by l1 and l 2 .
P A Q drawn from external point are equal.
⇒ 59° + 40°+∠ADP = 180° ⇒ ∠ADC = 81°
a. 25° b. 30° c. 45° d. 50° ∴ CQ = CP = 11 cm and BQ = BR Also, ∠ABC + ∠ADC = 180°
Sol. b. Since BC is the diameter of the circle. [property of cyclic quadrilateral]
Locus in Some Standard Cases
Now, BQ = CQ − BC
∴ ∠BAC = 90° [Q angle in a semi-circle is 90°] ⇒ ∠ABC = 180°−81° = 99° 1. The loci of all points d units from a circle having a
= (11− 7) cm = 4 cm
∠ACB = 60° [Q alternate angle theorem] ∴ ∠CDQ = ∠ABC = 99° and ∠QCD = ∠BAD = 59° radius r units are two concentric circles with radii of
Now, ∠ACB + ∠BAC + ∠ABC = 180° ∴ BR = BQ = 4 cm In ∆CQD, ∠CQD + ∠CDQ + ∠QCD = 180° r − d and r + d units respectively.
⇒ ∠ABC + 90° + 60° = 180° ⇒ ∠CQD = 180°−59°−99° = 180°−158° = 22° r+d
⇒ ∠ABC = 180°− (90° + 60° ) = 30° Cyclic Quadrilateral ⇒ ∠AQB = ∠CQD = 22°
O
• Theorem 15 If two tangents are drawn to a circle from A quadrilateral whose all vertices lie on a circle is called
an external point, then they subtend equal angles at the a cyclic quadrilateral.
LOCUS r–d r

centre. These two tangents The path traced by a point moving under a given
A
are equally inclined to the condition is called locus.
line segment joining the
Properties of Cyclic Quadrilateral 2. The locus of all points equidistant from two
e.g. The locus of points (in a plane) equidistant from a concentric circles is a third circle whose radius is the
centre to that point. In the • The sum of either pair of opposite angles of a cyclic
C O fixed point (in a plane) is a circle with the fixed point as average of the radii of the given circles.
given figure, P quadrilateral is 180° i.e. the opposite angles of a cyclic
centre.
∠AOP = ∠BOP and quadrilateral are supplementary.
∠APO = ∠BPO B The plural of locus is loci. b
• If the sum of a pair of opposite angles of a quadrilateral a+b
Theorem 16 Line joining centre and external point is • The locus may be a line segment or lines, straight or 2
• is 180°, then quadrilateral is cyclic. Oa
perpendicular bisector of line joining point of contacts curved.
of tangents to the circle. In the above figure, OP ⊥ AB • The exterior angle, formed by producing a side of a • Every point which satisfies the given geometrical
and AC = BC cyclic quadrilateral is equal to the interior opposite angle. condition(s) lies on the locus and vice-versa.
• Theorem 17 The length of two tangents drawn from an A B
3. The locus of the mid-points of all parallel chords of a
external point to a circle are equal. Basic Locus Theorem circle is the diameter of the circle, which is
perpendicular to the given chords.
Here, PA = PB 1. The locus of points at a fixed distance d from point p
A
is a circle with the given point p as its centre and d as
its radius.
D C E
P O

Here, ∠BAD = ∠BCE p d


B
MATHEMATICS Circle 293 294 CDS Pathfinder

4. The locus of the mid-points of equal


chords is a circle with the same
centre as the given circle and radius
T
O
EXAMPLE 11. What is the number of points in a plane
two units from a given line and three units from a
given point of the line?
PRACTICE EXERCISE
equal to the distance from the centre a. 1 b. 2 c. 3 d. 4
of the given circle to the given
chord.
Sol. d. The answer will be 4 points. The dotted lines are first 1. In a circle with centre O and radius 5 cm, AB is 8. The perpendicular bisectors of the sides of a
locus condition and the circle is the second locus a chord of length 8 cm. If OM ⊥ AB, then what is triangle pass through the
5. The locus of the centre of condition. These two loci intersect at points marked the length of OM? (a) different point (b) more than 2 points
circle touching a given line at a O with A, B , C and D. (a) 4 cm (b) 5 cm (c) same point (d) None of these
given point is a line D C (c) 3 cm (d) None of these
perpendicular to the given line 9. If a point P moves such that the sum of the
through the given point.
A P B 3 units 2 units 2. An equilateral ∆ABC is inscribed in a circle with squares of its distance from two fixed points
given line centre O. Then, ∠BOC is equal to A and B is a constant, then the locus of P is
6. The locus of the centre of all circles given point 2 units A (a) a circle
passing through two given points is (b) a straight line
A B
the perpendicular bisector of the line (c) an arbitrary curve
segment joining the two points. O
(d) the perpendicular bisector of AB
EXAMPLE 12. Two points A and B are 6 units apart. O
7. If A , B are fixed points, then the How many points are there that are equidistant from B C
10. The locus of points equidistant from two fixed
A B
locus of a point P such that both A and B and also 5 units from A? points is a straight line which
∠APB = 90 ° is the circle with AB as (a) 120° (b) 75° (c) 180° (d) 60°
(a) is a right angles to the line joining the two fixed points
a. 0 b. 1 c. 2 d. 3
diameter. 3. A square ABCD is inscribed in a circle with (b) bisects the line joining the two fixed points
P (c) is the perpendicular bisector of the line joining the
P Sol. c. The answer will be 2 points. centre O. Then, the angle subtended by each side
The dotted line is first locus 5 units of the square at the centre O is two fixed points
condition and the circle is the (a) 120° (b) 180° (c) 45° (d) 90° (d) None of the above
A B
A B second locus condition. These 3 units 3 units 11. Diameter AB and CD of a circle intersect at O. If
two loci intersect at two points 4. The points of concurrency of the
marked with P and Q. perpendicular bisectors of the sides of a triangle m∠BOD = 50°, then m ∠AOD is
Q
is called its
A D
EXAMPLE 13. Given a circle with radius 1.5 cm. (a) incentre (b) circumcentre O
50°
Which of the following dotted lines may represent the (c) excentre (d) orthocentre
EXAMPLE 10. Given two parallel straight lines AB locus of a point P such that P is at a distance of 1 cm C B
and CD which are 2 cm apart which of the following 5. In the given figure, PQ is the diameter of a circle
from the nearest point on the circle? with centre at O. OS is perpendicular to PR. (a) 50° (b) 180° (c) 130° (d) 310°
dotted lines may represent the locus of a point P such 1.5 cm
cm Then, OS is equal to 12. CD is a direct common tangent to two circles
that it is equidistant from AB and CD ? 2.5 cm 2.5 P intersecting each other at A and B. Then,
C D
∠CAD + ∠CBD is equal to?
a. 2 cm 2 cm a. S
b. A
1 cm

1.5 cm P
O
A B
0.5 cm
R
A B
b. 1 cm
P Q
1 cm B
1 1 1 C D
C D (a) QR (b) QR (c) QR (d) QR
4 3 2 (a) 180° (b) 90° (c) 360° (d) 120°
3 cm
A B 6. In the given figure, OM and ON are the 13. In a circle of radius 17 cm, two parallel chords
1.5 cm
c. 1.5 cm perpendiculars drawn on the chords PQ and RS . are drawn on opposite side of a diameter. The
c. d. P
C D If OM = ON = 6 cm. Then, distance between the chords is 23 cm. If the
P R length of one chord is 16 cm, then the length of
0.5 cm
the other chord is
1 cm O (a) 34 cm (b) 15 cm (c) 23 cm (d) 30 cm
A B M N
1 cm Sol. b. 14. If AB is a chord of a circle, P and Q are the two
d.
cm
1 cm 2 .5 Q S points on the circle different from A and B, then
C D
1 cm 1.5 cm (a) PQ ≥ RS (b) PQ < RS (c) PQ ≤ RS (d) PQ = RS (a) the angles subtended at P and Q by AB are always equal
(b) the sum of the angles subtended by AB at P and Q
7. The locus of the mid-points of all radii of a circle
1 cm

Sol. b. A B P is always equal to two right angles


P 1 cm
is a
0.5 cm (c) the angles subtended by AB at P and Q are either
1 cm (a) circle (b) parallelogram
equal or supplementary
C D (c) rhombus (d) square
(d) None of the above
MATHEMATICS Circle 295 296 CDS Pathfinder

15. The locus of the centre of circles which pases 21. Two circles touch each other internally. Their 27. S1 and S 2 are two circles on a plane with radii 34. What is the locus of centres of circles which
through two given points is radii are 2 cm and 3 cm. The biggest chord of the 4 cm, and 2 cm, respectively and the distance touch a given line at a given point?
(a) perpendicular to the line joining the given points at outer circle which is outside the inner circle is of between their centres is 3 cm. Which one of the (a) A line perpendicular to the given line, passing
one of those points length following statements is true? through the given point
(b) perpendicular bisector of the line joining the given points (a) 2 2 cm (b) 3 2 cm (c) 2 3 cm (d) 4 2 cm (a) S 2 lies entirely within the circle S1 (b) A line parallel to the given line
(c) parallel to the line joining the given points (b) S1 and S 2 touch each other internally (c) A circle tangent to the given line at the given point
(d) None of the above 22. If two circles are such that the centre of one lies (d) A closed curve other than a circle
(c) S1 and S 2 touch each other externally
on the circumference of the other, then the ratio
16. In the adjoining figure, POQ is the diameter of of the common chord of the two circles to the (d) S1 and S 2 intersect in two distinct points 35. In the given figure, O is the centre of the circle,
the circle, R and S are any two points on the radius of any one of the circles is 28. ACB is a tangent to a circle at C , CD and CE are OA = 3 cm, AC = 3 cm and OM is perpendicular
circles. Then, (a) 2 : 1 (b) (c) (d) 4 : 1 to AC. What is ∠ABC equal to?
3 :1 5 :1 chords such that ∠ACE > ∠ACD. If
R Q B
23. In the given figure, ABCD is a cyclic ∠ACD = ∠BCE = 50°, then
quadrilateral. AE is drawn parallel to CD and BA (a) CD = CE
O
is produced. If ∠ABC = 92° and ∠FAE = 20°, then (b) ED is not parallel to AB O
P S ∠BCD is equal to (c) ED passes through the centre of the circle
(d) ∆CDE is a right angled triangle
(a) ∠PRQ > ∠PSQ (b) ∠PRQ < ∠PSQ B A
F
1 20° A C
(c) ∠PRQ = ∠PSQ (d) ∠PRQ = ∠PSQ 92° 29. A bicycle is running straight towards North. M
E
2 What is the locus of the centre of the front wheel
17. If O is the centre of the circle, then the value of x of the bicycle whose diameter is d ?
C (a) 60° (b) 45°
D (a) A line parallel to the path of the wheel of the bicycle
in the adjoining figure is (c) 30° (d) None of these
A
at a height d cm
(a) 88° (b) 98° (c) 108° (d) 72° (b) A line parallel to the path of the wheel of the bicycle 36. The distance between the centres of two circles
x at a height d/2 cm having radii 4.5 cm and 3.5 cm, respectively is
90° 24. The values of x + y in the figure is equal to (c) A circle of radius d/2 cm 10 cm. What is the length of the transverse

11

B (d) A circle of radius d cm


O A common tangent of these circles?
C 30. If ‘O’ is the centre of circle, then x is equal to (a) 8 cm (b) 7 cm
(2x + 4) (c) 6 cm (d) None of these
C
(a) 80° (b) 70° (c) 60° (d) 50°
O
37. ABC is an equilateral triangle inscribed in a
18. If S is a circle with centre C and P be a movable (x + 10) (5y + 5) x° 80° circle with AB = 5 cm. Let the bisector of the
B D
point outside S, then the locus of P such that the B angle, A meet BC in X and the circle in Y . What
A
tangents from P to S are of constant length is is the value of AX ⋅ AY ?
(a) the straight line CP (4y – 4) (a) 16 cm2 (b) 20 cm2 (c) 25 cm2 (d) 30 cm2
(a) 80° (b) 60° (c) 40° (d) 20°
(b) the circle through P with centre at C
31. If two equal circles touch each other externally, 38. Two unequal circles are touching each other
(c) a circle intersecting S C the common tangent divides the line of centres externally at P, APB and CPD are two secants
(d) a circle touching S (a) 90° (b) 85° (c) 75° (d) 65° in the ratio cutting the circles at A, B, C and D. Which one of
19. In the given figure A, B, C , D are the concyclic 25. Let A and B be two points. What is the locus of (a) 1 : 1 (b) 2 : 1 (c) 1 : 2 (d) 3 : 2 the following is correct?
points. The value of x is the point P such that ∠APB = 90° ? 32. With the vertices of a ∆ABC as centre three circles (a) ACBD is parallelogram (b) ACBD is a trapezium
E (a) The line AB itself (c) ACBD is a rhombus (d) None of these
x C are described, each touching the other two circle
(b) The point P itself externally. If the sides of the triangle are 9 cm, 39. ABCD is a quadrilateral, the sides of which
D
(c) The circumference of the circle with AB as diameter 7 cm and 6 cm. Then, the radius of the circle are touch a circle. Which one of the following is
(d) The line perpendicular to AB and bisecting AB (a) 4, 5, 2 (b) 4, 5, 6 correct?
130° 26. In the adjoining figure AD , AE and BC are (c) 3, 2, 3 (d) All equal to 3 cm (a) AB + AD = CB + CD (b) AB : CD = AD : BC
A F (c) AB + CD = AD + BC (d) AB : AD = CB : CD
B tangent to the circle at D , E , F respectively, then 33. In the given figure PT touches the circle with
(a) 50° (b) 60° (c) 70° (d) 90° centre O at R. Diameter SQ when produced meet 40. Let PAB be a secant to a circle intersecting at
E PT at P. If ∠SPR = x and ∠QSR = y, then x + 2y
C points A and B and PC is a tangent. Which one
20. If O is the centre of the circle, then x is is equal to of the following is correct?
D C A F S (a) The area of rectangle with PA, PB as sides is equal
x° O to the area of square with PC as sides
32° y
Q
(b) The area of rectangle with PA, PC as sides is equal
O B
28° D x to the area of square with PB as sides
50° R
(c) The area of rectangular with PC, PB as sides is equal
P T
A B (a) AD = AB + BC + AC (b) 2 AD = AB + BC + AC to the area of square with PA as side
1 (a) 180° (b) 90° (d) The perimeter of rectangle with PA, PB as sides is
(c) AD = ( AB + BC + AC ) (d) 3AD = AB + BC + AC (c) 135° (d) None of these
(a) 72° (b) 62° (c) 82° (d) 52° 4 equal to the perimeter of square with PC as side
MATHEMATICS Circle 297 298 CDS Pathfinder

41. In the given figure, if ∠BAD = 60°, ∠ADC = 105°, 47. In the given figure, a circle is inscribed in a 53. In the given figure, what is ∠BYX equal to? 59.
then what is ∠DPC equal to? quadrilateral ABCD. Given that, BC = 38 cm, A
B
Q
QB = 27 cm, DC = 25 cm and AD is perpendicular S
P 45°
to DC. What is the radius of the circle? R
B P X'
Q 50°
D C X Y
A X O
105°
O (a) 85° (b) 50° (c) 45° (d) 90° In the above figure which of the following holds
R
D 54. In the given figure, AD is a straight line, OP good?
C
60° perpendicular to AD and O is the centre of (a) ∠SOQ = ∠ROP (b) 2∠ROP = ∠SOR
A B (a) 11 cm (b) 14 cm (c) 15 cm (d) 16 cm (c) ∠POR = ∠ASO (d) ∠QOX' = ∠SOR + ∠ROP
both circles. If OA = 20 cm, OB = 15 cm and
(a) 40° (b) 45° (c) 50° (d) 60° 48. In the figure given below, what is ∠CBA ? OP = 12 cm, what is AB equal to? 60. Semi-circle C1 is drawn with a line segment PQ
42. In the given figure, PQ is a diameter of the circle A as its diameter with centre at R. Semicircles C2
and C3 are drawn with PR and QR as diameters
whose centre is at O. If ∠ROS = 44° and OR is a 75° C O respectively, both C2 and C3 lying inside C1. A
bisector of ∠PRQ, then what is the value of ∠RTS ? 3 0°
P full circle C4 is drawn in such a way that it is
R Q R
P T A D tangent to all the three semicircles C1, C2 and C3 ,
B B P C
C4 lies inside C1 and outside both C2 and C3 . The
44° S
O (a) 30° (b) 45° (c) 50° (d) 60° radius of C4 is
(a) 7 cm (b) 8 cm (c) 10 cm (d) 12 cm
49. PQ is a common chord of two circles. APB is a 1 1 1 1
Q x y z (a) PQ (b) PQ (c) PQ (d) PQ
secant line joining points A and B on the two 55. In the given figure, if = = , where ∠DCQ = x, 3 6 2 4
(a) 46° (b) 64° 3 4 5
(c) 69° (d) None of these circles. Two tangents AC and BC are drawn. If ∠BPC = y and ∠DQC = z, then what are the 61. Two circles are placed in an
∠ACB = 45°, then what is ∠AQB equal to? values of x, y and z, respectively ? equilateral triangle as shown in
43. In the given figure, O is the centre of the circle. the figure. What is the ratio of the
(a) 75° (b) 90° (c) 120° (d) 135°
AC and BD intersect at P. If ∠AOB = 100° and A
area of the smaller circle to that of
∠DAP = 30°, what is ∠APB ? 50. ABCD is concyclic quadrilateral. The tangents at the equilateral triangle?
E A and C intersect each other at P. If D (a) π : 36 3 (b) π : 18 3 (c) π : 27 3 (d) π : 42 3
∠ABC = 100°, then what is ∠APC equal to? B
D C
(a) 10° (b) 20° (c) 30° (d) 40° y C x 62. Consider the following statements :
P
O z I. The opposite angles of a cyclic quadrilateral are
51. In the given figure, quad. ABCD is circumscribed, P
B Q supplementary.
A
touching the circle at P , Q , R and S such that II. Angle subtended by an arc at the centre is
(a) 33°, 44° and 55° (b) 36°, 48° and 60°
∠DAB = 90°. If CS = 27 cm and CB = 38 cm and double the angle subtended by it at any point on
(c) 39°, 52° and 65° (d) 42°, 56° and 70°
(a) 77° (b) 80° (c) 85° (d) 90° radius of the circle is 10 cm, then AB = ? the remaining part of the circle.
44. In the given figure, AB is a C 27 cm 56. In the given figure, if ∠AOP = 75° and
S C Which one of the following is/are correct in
diameter of a circle and CD is D ∠AOB = 120°, then what is ∠AQP ? respect of the above statements?
O
perpendicular to AB, if A B R R (a) Statement I ⇒ Statement II
E
AB = 10 cm and AE = 2 cm, then P
10 cm (b) Statement II ⇒ Statement I
O 38 cm Q 75°
what is the length of ED? D (c) Statement I ⇔ Statement II
O
(a) 5 cm (b) 4 cm (c) 10 cm (d) 20 cm A (d) Neither Statement I ⇒ Statement II nor Statement II
Q B
C 120° ⇒ Statement I
45. In the given figure, PT is a B
A B
tangent to a circle of radius 6 (a) 28 cm (b) 21 cm (c) 19 cm (d) 17 cm 63. Consider the following statements :
P O P
cm. If P is at a distance of 10 cm 52. In the figure given below, YAX is a tangent to I. Let P be a point on a straight line L. Let Q , R
from the centre O and PB = 5 cm, T the circle with centre O. If ∠BAX = 70° and (a) 45° (b) 37.5° (c) 30° (d) 22.5° and S be the points on the same plane
then what is the length of the ∠BAQ = 40°, then what is ∠ABQ equal to? 57. A circle of radius 5 cm has chord RS at a containing the line L such that PQ , PR and PS
chord BC ? are perpendicular to L. Then, there exists no
E
distance of 3 units from it. Chord PQ intersects
triangle with vertices Q , R, S.
(a) 7.8 cm (b) 8 cm (c) 8.4 cm (d) 9 cm with chord RS at T such that TS = 1 / 3 of RT.
B Find minimum value of PQ. II. Let C be a circle passing through three distinct
46. In the given figure, ∠AOB = 46°, O points D , E and F such that the tangent at D to
(a) 6 3 (b) 4 3 (c) 8 3 (d) 2 3
AC and OB intersect each other O the circle C is parallel to EF. Then, DEF is an
C Q
at right angles. What is the Y X 58. Two mutually perpendicular chords AB and CD isosceles triangle.
measure of ∠OBC ? (O is the A intersect at P. AP = 4 cm , PB = 6 cm, CP = 3 cm . Which of the statement(s) given above is/are
centre of the circle.) A B Find radius of the circle.
(a) 20° (b) 30° correct?
(a) 44° (b) 46° (c) 67° (d) 78.5° (c) 35° (d) 40° 5 5 3 5 6 5 2 5 (a) Only I (b) Only II
(a) cm (b) cm (c) cm (d) cm
2 4 2 3 (c) Both I and II (d) Neither I nor II
MATHEMATICS Circle 299 300 CDS Pathfinder

Directions (Q. Nos. 68-69) BC is a chord with centre O.


64. The incircle of a ∆ABC touches the sides AB, BC 74. Consider the following statements : 81. If the angle between the radii of a circle is 130°,
A is a point on an arc BC as shown in the figure below.
and AC at the points P , Q , R, respectively, I. The perpendicular bisector of a chord of a circle then the angle between the tangents at the ends
A A
then which of the following statement is/are does not pass through the centre of the circle. of the radii is e 2015 I
x
correct? x B y t y C II. The angle in a semi-circle is a right angle. (a) 90° (b) 70° (c) 50° (d) 40°
I. AP + BQ + CR = PB + QC + RA O
z
O Which of the statement(s) given above is/are 82. Out of two concentric circles, the diameter of the
1 z
II. AP + BQ + CR = (perimeter of ∆ABC) y y correct? e 2013 II outer circle is 26 cm and the chord MN of length
2 B C
(a) Only I (b) Only II 24 cm is tangent to the inner circle. The radius
III. AP + BQ + CR = 3( AB + BC + CA ) of the inner circle is e 2015 I
68. What is the value of ∠BAC + ∠OBC if A is the (c) Both I and II (d) Neither I nor II
Select the correct answer using the codes given (a) 5 cm (b) 6 cm (c) 8 cm (d) 10 cm
point on the major arc? 75. The diameter of a circle with centre at C is 50
below
(a) 110° (b) 70° (c) 90° (d) 60° cm. CP is a radial segment of the circle. AB is a 83. AD is the diameter of a circle and AB is a chord.
(a) I, II and III are correct (b) Only I is correct
(c) II and III are correct (d) I and II are correct 69. What is the value of ∠BAC − ∠OBC if A is the chord perpendicular to CP and passes through If AD = 34 cm, AB = 30 cm, then the distance of
point on the minor arc? P. CP produced intersects the circle at D. If AB from the centre of the circle is e 2015 I
65. Points X and Y are two different points on a (a) 90° (b) 180° (c) 130° (d) 50° DP = 18cm, then what is the length of AB ? (a) 17 cm (b) 15 cm (c) 13 cm (d) 8 cm
circle. Point M is located so that line segment A
e 2013 II 84. From an external C
XM and line segment YM have equal length.
Which of the following could be true? PREVIOUS YEARS’ QUESTIONS (a) 24 cm (b) 32 cm point P, tangents PA
E P
(c) 40 cm (d) 48 cm and PB are drawn to
I. M is the centre of the circle. 70. A circular ring with centre O is kept in the vertical the circle as shown
II. M is on arc XY III. M is outside of the circle. position by two weightless thin strings TP and 76. Consider the following statements in respect of in the above figure. D
TQ attached to the ring at P and Q. The line OT two chords XY and ZT of a circle intersecting B
(a) Only 1 (b) Only II (c) I and II (d) I, II and III CD is the tangent to
meets the ring at E where as a tangential string at P. the circle at E. If AP = 16 cm, then the perimeter
66. Which of the following statement(s) are correct? at E meets TP and TQ at A and B, respectively. If I. PX ⋅ PY = PZ ⋅ PT of the ∆ PCD is equal to e 2015 II
I. A circle is the locus of points equidistant from a the radius of the ring is 5 cm and OT = 13 cm, II. PXZ and PTY are similar triangles. (a) 24 cm (b) 28 cm (c) 30 cm (d) 32 cm
fixed point. then what is the length of AB ? e 2012 I
II. The locus of centre of circle rolling on the (a) 10/3 cm (b) 20/3 cm (c) 10 cm (d) 40/3 cm Which of the statement(s) given above is/are 85. Chord CD intersects the diameter AB of a circle
circumference of fixed circle is a circle correct? e 2013 II at right angle at a point P in the ratio 1:2. If
71. The locus of the mid-points of all equal chords in (a) Only I (b) Only II diameter of circle is d, then CD is equal to
concentric with fixed circle.
a circle is e 2012 I (c) Both I and II (d) Neither I nor II e 2015 II
III. The locus of the points which are equidistant (a) The circumference of the circle concentric with
from three distinct points on a line, is a line the given circle and having radius equal to the chords 77. In a ∆ABC, AB = BC = CA. The ratio of the radius (a)
2d
(b)
2d
(c)
2 2d
(d)
2 3d
parallel to the given line. (b) The circumference of the circle concentric with the of the circumcircle to that of the incircle is 3 3 3 3
IV. If the bisectors of ∠B and ∠C of a quadrilateral given circle and having radius equal to the distance (a) 2 : 1 (b) 3 : 1 e 2014 I
of the chords from the centre
86. The diameter of a wheel that makes 452
ABCD intersect in P, then P is equidistant from (c) 3 : 2 (d) None of these revolutions to move 2 km and 26 dm is equal to
(c) The circumference of the circle concentric with the
AB and CD.
given circle and having radius equal to half of the 78. AB and CD are two chords of a circle meeting e 2015 II
Select the correct answer using the codes given radius of the given circle externally at P. Then, which of the following 9
(a) 1 m
13
(b) 1 m (c) 2
5
m (d) 2
7
m
below (d) The circumference of the circle concentric with the is/are correct? 22 22 11 11
given circle and having radius equal to half of the
(a) All are correct (b) Only IV
distance of the chords from the centre I. PA × PD = PC × PB 87. A boy is cycling such that the wheels of the cycle
(c) Both I and II (d) I, II and IV II. ∆ PAC and ∆ PDB are similar. are making 140 revolutions per minute. If the
72. Consider the following statements radius of the wheel is 30 cm, then the speed of
67. A square and a circle intersect at more than one Select the correct answer using the codes given
point. Consider the following statements: I. The locus of points which are equidistant from the cycle is e 2015 II
two parallel lines is a line parallel to both of
below e 2014 I
I If atleast two of the intersection points are on (a) Only I (b) Only II (a) 15.5 km/h (b) 15.84 km/h (c) 16 km/h (d) 16.36 km/h
them and drawn mid way between them.
the vertices of the square then the square is (c) Both I and II (d) Neither I nor II 88. The two adjacent sides of a cyclic quadrilateral
II. The perpendicular distances of any point on this
cyclic. are 2 cm and 5 cm and the angle between them
locus line from two original parallel lines are 79. For a triangle, the radius of the circumcircle is
II If there are exactly four intersection points then equal. Further, no point outside this locus line double the radius of the inscribed circle, then is 60°. If the third side is 3 cm, then the fourth
the square has less area than the circle. has this property. which one of the following is correct? e 2014 II side is of length e 2015 II
III If there are exactly four intersection point then Which of the statement(s) given above is/are (a) The triangle is a right angled (a) 2 cm (b) 3 cm (c) 4 cm (d) 5 cm
the diameter of circle is equal to the diagonal of correct? e 2012 II (b) The triangle is an isosceles
square.
89. AB is a line segment of length 2a, with M as
(a) Only I (b) Only II (c) The triangle is an equilateral mid-point. Semi-circles are drawn on one side
IV If atleast two intersection points are on the (c) Both I and II (d) Neither I nor II (d) None of the above with AM, MB and AB as
vertices of square. Then it may be possible to 73. Consider a circle with centre at O and radius r. diameter as shown in the above
have the sides of square equal to the diameter of 80. If the chord of an arc of a circle is of length x, r O
Points A and B lie on its circumference and a figure. A circle with centre O
circle. the height of the arc is y and the radius of the
point M lies outside of it such that M, A and O lie and radius r is drawn such that
Which of the statement (s) given above is/are circle is z. Then, which one of the following is
on the same straight line. Then, the ratio of MA
correct? e 2014 II
this circle touches all the three A M B
correct ? to MB is e 2013 I semi-circles. The value of r, is e 2015 II
(a) Only I (b) Only IV (a) equal to 1 (b) equal to r (a) y (2 z − y ) = x2 (b) y (2 z − y ) = 4x2
2a a a a
(c) greater than 1 (d) less than 1 (c) 2 y (2 z − y ) = x2 (d) 4y (2 z − y ) = x2 (a) (b) (c) (d)
(c) Both I and II (d) II, III and IV 3 2 3 4
MATHEMATICS Circle 301 302 CDS Pathfinder

90. A tangent is drawn from an external point O to a 94. Consider a circle at O and radius 7 cm. Let QR
circle of radius 3 units at P such that OP = 4
units. If C is the centre of the circle, then the
be a chord of length 2 cm and let P be the
mid-point of QR. Let CD be another chord of this
HINTS AND SOLUTIONS
sine of ∠COP is e 2016 I circle passing through P such that ∠CPQ is acute.
1 8
(a) 4/5 (b) 3/4 (c) 3/5 (d) 1/2 If M is the mid-point of CD and MP = 24 cm, 1. (c) Given,OA = 5 cm,AM = × AB = 9. (a) Let P be the moving point. 14. (c) There are two possibilities
2 2 Then, PA 2 + PB 2 = constant
91. Let ABC be a triangle in which AB = AC. Let L then which of the following statements are ⇒ AM = 4 cm Case I When P and Q are on the same
correct? side of AB. In this case ∠APB = ∠AQB
be the locus of points X inside or on the triangle P B (angle in the same segment).
such that BX = CX . I. ∠QPD = 135°
Q
II. If CP = m cm and PD = n cm, then m and n are O
Which of the following statements are correct? m P
I. L is a straight line passing through A and the roots of the quadratic equation x2 − 10x + 1 = 0. 5c B
incentre of ∆ ABC is on L. III. The ratio of the area of ∆OPR to the area of A B A
M
∆OMP is 1 : 2 2. 8 cm So, the locus of P is a circle. A
II. L is a straight line passing through A
orthocentre of ∆ ABC is a point on L. Select the correct answer using the code given In right angled ∆OMA, 10. (c) The locus of
P
III. L is a straight line passing through A centroid of below. e 2016 I OM 2 = OA 2 − AM 2 points equidistant Case II When P and Q are on the
from fixed points
∆ ABC is a point on L. (a) I and II (b) II and III (c) I and III (d) I, II and III [by pythagoras theorem] P´ opposite of AB. In this case PAQB is a
is a straight line cyclic quadrilateral.
Select the correct answer using the code given 95. Suppose chords AB and CD of a circle intersect = 52 − 42 = 25 − 16 = 25 − 16 = 9 which is the P´´
⇒ OM = 3 cm perpendicular P B
below. e 2016 I at a point P inside the circle. Two right angled
(a) I and II (b) II and III (c) I and III (d) I, II and III 2. (a) Q ∠BOC = 2∠A = 2 × 60° bisector of the
triangles A′ P ′B′ and C ′Q ′D ′ are formed as shown line joining the
A B A Q
in the figures below such that A′ P ′ = AP, (Angle subtended by an arc at the center
92. In a circle of radius 2 units, a diameter AB is double the angle subtended by it at two fixed points.
intersects a chord of length 2 units perpendi- B′ P ′ = BP, C ′Q ′ = CP, D ′Q ′ = DP and any point on the circumference of circle) Here, A , B are fixed point and P is locus.
cularly at P. If AP > BP , then AP is equal to ∠A′ P ′B′ = 90° = ∠C ′Q ′D ′. 11. (c) Given that, ⇒ ∠BOD = 50°
⇒ ∠BOC = 120° So, ∠APB + ∠AQB = 180°
e 2016 I A′
(a) (2 + 5 ) units (b) (2 + 3 ) units 3. (d) A square has four equal side. ⇒ ∠AOD = 180° − ∠BOD = 130° 15. (b) Clearly, the locus of the centre of
(c) (2 + 2 ) units (d) 3 units ∴ Each side subtends the same angle at [linear pair] circles which passes through two given
C′ the centre O. point is perpendicular bisector of the
12. (a) Here, ∠CAB = ∠BCD
93. Consider a circle with centre at C. Let OP, OQ Let angle subtended be x°.
line joining the given points.
[angles in alternate segments]
denote respectively the tangents to the circle 16. (c) As POQ is the diameter of the circle,
D C and ∠DAB = ∠CDB
drawn from a point O outside the circle. Let R be P′ B′ Q′ D′ therefore, PRQ and PSQ are two
O [angles in alternate segments]
a point on OP and S be a point on OQ such that Which of the following statements are not correct? semicircles and ∠PRQ = ∠PSQ.
⇒ ∠CAD = ∠CAB + ∠DAB
OR × SQ = OS × RP . Which of the following ∠PRQ = ∠PSQ = 90°
I. A′ P′B′ and C′Q′ D′ are similar triangles but need = ∠BCD + ∠CDB
statement(s) is/are correct? A B [angles in semicircle]
not be congruent. On adding ∠CBD both sides, we get
I. If X is the circle with centre at O and radius OR 17. (a) ∠COB = 360°− ( 110°+90° ) = 160°
II. A′ P′B′ and C′Q′ D′ are congruent triangles. So, 4x° = 360° ∠CAD + ∠CBD
and Y is the circle with centre at O and radius 1
III. A′ P′B′ and C′Q′ D′ are triangles of same area. 360° = ∠BCD + ∠CDB + ∠CBD ∴ x = ∠CAB = ∠COB
OS, then X = Y . x° = = 90°
4 2
II. ∠POC + ∠QCO = 90° IV. A′ P′B′ and C′Q′ D′ are triangles of same perimeter. = 180° [angles of a triangle] 1
4. (b) Clearly, the point of concurrency of 1 16 = × 160° = 80°
Select the correct answer using the code given Select the correct answer using the code given the perpendicular bisectors of the sides 13. (d) Here, BE = AB = cm = 8 cm 2
2 2
below. e 2016 I below. e 2016 I of a triangle is known as circumcentre. 18. (b) If S is a circle
OB = OD = 17 cm (radii) S
(a) Only I (b) Only II (a) II and III (b) I and III 5. (c) As, O is mid-point of diameter PQ with centre C and P
A E be a movable point
(c) Both I and II (d) Neither I nor II (c) I, II and IV (d) I, II, III and IV and ∠PRQ = 90° (angle in semi-circle) B
P
C
outside S, then the
So, OS || QR and OS⊥ PR O locus of P such that
PO OS the tangent from P
∴ =
ANSWERS PQ QR C F D to S are of constant length is the circle
through P with centre at C .

OS
=
1 1
⇒ OS = QR In right angled ∆OEB,
1 c 2 a 3 d 4 b 5 c 6 d 7 a 8 c 9 a 10 c
QR 2 2 OE = OB 2 − BE 2 19. (a) ∠CBF = ∠CDA
11 c 12 a 13 d 14 c 15 b 16 c 17 a 18 b 19 a 20 c (angles in alternate segments)
6. (d) PQ = RS as chords equidistant from = 172 − 82 = 289 − 64
21 d 22 b 23 c 24 d 25 c 26 b 27 d 28 a 29 b 30 c ⇒ ∠CDA = 130°
the centre are equal.
31 a 32 a 33 b 34 a 35 c 36 c 37 c 38 b 39 c 40 a = 225 = 15 cm ∠CDA + x = 180° [linear pair]
7. (a) The locus of the mid-points of all ∴OF = EF − OE = ( 23 − 15) = 8 cm x = 180° − 130° = 50°
41 b 42 d 43 b 44 b 45 a 46 c 47 b 48 d 49 d 50 b
radii of a circle is a circle. Hence, the value of x is 50°.
51 b 52 b 53 a 54 a 55 b 56 b 57 b 58 a 59 a 60 b In right angled ∆OFD,
8. (c) The perpendicular bisectors of the 20. (c) ∠ACB = ∠ADB = 32°
61 c 62 d 63 c 64 d 65 d 66 d 67 d 68 c 69 a 70 b sides of a triangle passes through the FD = OD 2 − OF 2
[angle in same segment]
71 b 72 c 73 d 74 b 75 d 76 c 77 a 78 b 79 c 80 d same point. The perpendicular bisectors = 172 − 82 = 289 − 64 ∠ACD = ∠ABD = 50°
81 c 82 a 83 d 84 d 85 c 86 b 87 b 88 a 89 c 90 c are concurrent and point is called the = 225 = 15 cm ∴ x = ∠BCD = ∠ACB + ∠ACD
91 d 92 b 93 c 94 a 95 c
circumcentre. ∴ CD = 2FD = 30 cm = 32°+50° = 82°
MATHEMATICS Circle 303 304 CDS Pathfinder

21. (d) Here, OB = OD = 3 cm and On solving, Eqs. (i) and (ii), we get The locus of the centre of the front 35. (c) Given, OA = 3 cm and AC = 3 cm 38. (b) l 42. (d) ∠PRQ = 90° as angle in a semi-circle
O ′D = O ′ A = 2 cm x and y are 40° and 25°, respectively. wheel of the cycle is a line parallel to the and OA = OC = 3 cm (radius of a circle) D B is 90°.
5
OO ′ = OD − O ′D = 3 − 2 = 1 cm path of the wheel of the bicycle at a In ∆AOC , all sides are equal.
∴ x + y = 40° + 25° = 65° 3 Since, OR is a bisector of ∠PRQ.
∴ OA = O ′ A − OO ′ = 2 − 1 = 1 cm height d / 2 cm. 7 1
B ∴ ∆ AOC , is an equilateral triangle ∴ ∠PRO = ∠ORQ = 45°
In ∆OAB, 25. (c) The locus of a point is the
30. (c) Q ∠OAC = ∠OCA
circumference of the circle with AB as ⇒ ∠AOC = 60° 8 2 T
AB = OB 2 − OA 2 diameter. [angle opposite to equal sides] B 6
4
A A
O D P C R
[by pythagoras O´ ∴ 2 ∠OAC = 80°
theorem] l is the common tangent of two circles 45°
90° [external angle of ∆OAC ] P

45°
= 3 −1 = 9−1
2 2 C
80° O Then, ∠1 = ∠4 and ∠5 = ∠8 S
∠OAC = = 40° ⇒ x = 40° 44°
= 8cm [angle in alternate segment] M
A B 2 θ
∴ Required length = BC = 2 AB But ∠4 = ∠5 [opposite angles] O
Hence, the value of x is 40°.
∴ ∠1 = ∠8
= 2 8 cm = 4 2 cm A M C Q
31. (a) Since, the direct common tangents of Similarly ∠2 = ∠7
22. (b) Here, let O , O ′ be the centres of the 1
two circles divides the line joining their Hence, ∠ABC = ∠AOC But these are alternate angles
circles. centres externally in the ratio of their 2 Also, OP = OR [radius]
26. (b) As, the tangents drawn to a circle ⇒ AD || BC ⇒ ACBD is a trapezium.
As, the centre of each lies on the from a point outside it are equal. We radii. Here, both the circles being of [Angle subtended by an arc at the ∴ ∠OPR = 45°
equal radii. Hence, their ratio is 1 : 1. centre is double the angle subtended 39. (c) We know that, two
circumference of the other, therefore the have, D R C in ∆ORS , OR = OS
two circles will have the same radius. tangents drawn from
AD = AE , BD = BF and CE = CF 32. (a) Let AB = 9 cm, BC = 7 cm, by it at any point on the circle] ⇒ ∠ORS = ∠OSR
Let the radius be r. 60° an external point to a
⇒ AD = AB + BD = AB + BF = = 30°
r AC = 6 cm circle are equal in S Q in ∆ORS , by angle sum property
∴ OC = O ′C = and AD = AC + EC = AC + CF 2
2 Let x, y and z be A length. of a triangle,
∴ 2 AD = AB + AC + ( BF + CF ) 36. (c) From figure, length of the transverse
A radii of circles with x x
∴ AP = AS …(i) A P B ∠ORS + ∠OSR + ∠ROS = 180°
2 AD = AB + AC + BC centre A , B , C . common tangent of these circles
y z
BP = BQ …(ii) 2∠ORS = 180°−44°
C x+ y =9 (distance between the centres
O O´ 27. (d) OB = 4 cm …(i) z C 2∠ORS = 136°
B y = CR = CQ …(iii)
y+z=7 … (ii) of circles)2 − (sum of radius)2 136°
DR = DS …(iv) ∴ ∠ORS = ∠OSR = = 68°
B O O´ and z + x = 6 …(iii) 2
A B = 102 − ( 45
. + 35
. )2 = 102 − 82 On adding Eqs. (i), (ii), (iii) and (iv), we ∴ ∠MRS = 68° − 45° = 23°
In right angled ∆OCA, by pythagoras On adding Eqs. (i), (ii) and (iii), we get get
theorem 2( x + y + z ) = 22 = ( 100 − 64) = 36 = 6 cm ⇒ ∠PRS = 90° + 23° = 113°
O ′ A = 2 cm, OO′ = 3 cm ( AP + BP ) + (CR + DR)
∴ AC = OA 2 − OC 2 By properties of cyclic quadrilateral,
As, OO ′ ≠ OB + O ′ A ⇒ ( x + y + z ) = 11 …(iv) A
= ( AS + DS ) + ( BQ + C Q )
r2 3 cm ∠PRS + ∠PQS = 180°
= r2 − = r So, circle does not touch each other On putting x + y = 9 in Eq. (iv), we get 5 10 cm O´ ⇒ AB + CD = AD + BC
4 2 3. ⇒ ∠PQS = 180° − 113° = 67°
externally. 9 + z = 11 ⇒ z = 2 cm O
⇒ AB = 2 AC = 3 r 4.5 cm 40. (a) If a secant to a in ∆PTQ,
Also, OO ′ ≠ OB − O ′ A On putting value of z in Eq. (ii), we get B
Common chord AB 3r B
circle intersect ∠QPT + ∠PQT + ∠PTQ = 180°
∴ = = So, circle does not touch internally, y+ 2= 7 ⇒ y =5 circle at points
Radius OA r [by angle sum property of a triangle]
hence they intersect each other at two 37. (c) A A and B and PC
Hence, ratio of common chord to the Again, putting value of y in Eq. (i), we A ⇒ ∠PTQ = 180° − 45° − 67° = 68°
distinct points. is a tangent to
get
radius is 3 : 1 . circle, then ∴ ∠RTS = ∠PTQ = 68°
x + 5 = 9 ⇒ x = 4 cm

5c
m
28. (a) Join ED, then 30° 30°
23. (c) Q ∠B + ∠D = 180° PC 2 = PA × PB P 1

5c
C 43. (b) Since, ∠ADB = ∠AOB = 50°

m
C So, radii are 4 cm, 5 cm and 2 cm.
(sum of two opposite angle in cyclic B which is equivalent to saying that area of 2
50° X
quadrilateral) 50° A 33. (b) ∠QRS = 90° [angle in semi-circle] C B rectangle with PA and PB as sides is E
5 5
⇒ ∠D = 180° − ∠B and ∠QRP = ∠QSR = y ° 2 2 equal to the area of square with PC as
= 180° − 92° = 88° E D [angle in alternate segments] sides.
Y
⇒ ∠DAE = ∠D = 88° [alternate angle] Also, ∠PRS = 90 + y ° 41. (b) Given, ∠BAD = 60° D C
In equilateral triangle, angle bisector
⇒ ∠FAD = 88° + 20° = 108° and median are same and ∠ADC = 105°
P P
S
⇒ ∠BCD = ∠FAD = 108° BC 5
O y ∴ CX = BX = = cm As ABCD is a cyclic
[angles in alternate segments] ∠DEC = ∠ACD = 50° Q 2 2 quadrilateral C O

30°
D
⇒ ∠BCD = 108° [angle in alternate segment] AX 3 AX ∴∠DCP = ∠BAD = 60° 100°
P x T In ∆CXA, cos30° = ⇒ = 105°
∠EDC = ∠BCE = 50° R AC 2 5
24. (d) As, ∠B + ∠D = 180° and∠ADC + ∠CDP A B
[angle in alternate segment] 60°
and ∠A + ∠C = 180° In ∆PRS, ⇒ AX =
3
×5 =
5 3
cm = 180° [linear line] A B
∴ ∠DEC = ∠EDC In ∆DPA, by angle sum property of a
[sum of two opposite angle in a cyclic ∠SRP + ∠RPS + ∠PSR = 180° 2 2
⇒ 105°+∠CDP = 180°
So, CD = CE AY and BC are the chord of circle. triangle
quadrilateral] [Qsum of all angles of triangle is 180°] ⇒ ∠CDP = 75° ∠DAP + ∠ADP + ∠DPA = 180°
29. (b) ∴ AX ⋅ XY = BX ⋅ XC
x + 10 + 5 y + 5 = 180° ⇒ ( 90 + y ° ) + x° + y ° = 180° In ∆CPD, ⇒ 30° + 50° + ∠DPA = 180°
5 3 5 5 5
⇒ x + 5 y = 165° …(i) ⇒ x + 2 y ° = 90° ⇒ ⋅ XY = ⋅ ⇒ XY =
2 2 2 2 3 ∠DCP + ∠CDP + ∠DPC = 180° ⇒ ∠DPA = 100°
O
Q ∠A + ∠C = 180° 34. (a) A line perpendicular to the given 5 3 5  5 3 [by angle sum property of a triangle] Also, DPB is a straight line,
line, passing through the given point is ∴ AY ⋅ AX =  +  ×
⇒ 2x + 4 + 4 y − 4 = 180° d/2  2 2 3 2 ⇒ 60° + 75° + ∠DPC = 180° ∴ ∠DPA + ∠APB = 180°
the required locus. ⇒ ∠APB = 180° − 100° = 80°
⇒ 2x + 4 y = 180° …(ii) = 25 cm 2
⇒ ∠DPC = 180° − 135° = 45°
MATHEMATICS Circle 305 306 CDS Pathfinder

44. (b) In ∆OED, 47. (b) We know that, the tangents drawn to ⇒ ∠x + ∠y = 180°− 45° = 135° …(i) 53. (a) We know that, the angles in the same 180° In right triangle OYD
⇒ k= ⇒ k = 12°
the circle from a point outside the circle ∠x = ∠AQP and ∠y = ∠BQP segment of a circle are equal. 15 OD = OY 2 + YD 2
C
are always equal. ∴ ∠XBY = ∠XAY = 45° ∴ x = 36° , y = 48° , z = 60°
2 cm [angles in alternate segment] [by pythagoras theorem]
m B In ∆BXY , by angle sum property of a
3 cm 27 c 56. (b) We know that, angle subtended on
O ∴ ∠AQB = ∠AQP + ∠BQP 2
12 +  
Q triangle the circumference is half of the angle 11 5 5
A E B = = cm
5 cm A = ∠x + ∠y = 135° subtended at the centre.  2 2
∠BXY + ∠XBY + ∠BYX = 180°

m
5 cm

38 c
1 1
S T 50. (b) We know that, the sum of opposite ⇒ 50° + 45° + ∠BYX = 180° ∠AQP = ∠AOP = 75° = 37. 5° 59. (a) In the figure,
D angles of a cyclic quadrilateral is always 2 2
R [Q ∠BXY = 50°]
[by pythagoras theorem] 180°. 57. (b)
D
M
C
A ⇒ ∠BYX = 180° − 95° = 85° P
(OD )2 = (DE )2 + ( EO )2 Q
∴ BQ = BR = 27 cm 54. (a) In right angled ∆OPB, O
S
⇒ (5)2 = (DE )2 + (3)2 R
⇒ RC = BC − BR P
R S
⇒ (DE )2 = 25 − 9 = 16 = 38 − 27 = 11 cm B 100° D M T X′
P X
⇒ DE = 4 cm ∴ RC = CM = 11 cm O
Q O
Now, DM = 25 − 11 = 14 cm Given, OM = 3 cm, OS = 5 cm. Let ∠POX = x ° , ∠ROP = y °
45. (a) Given, PO = 10 cm, OT = 6 cm and ∴ ST = DM = 14 cm C A B P C D
MS = 4 cm = RM ∠POX = ∠PQO = x
PB = 5 cm
48. (d) We know that, the sum of opposite ∴ ∠B + ∠D = 180° {Using pythagoras theorem} and ∠ROX = ∠RSO = x °+ y °
In right angled ∆OTP , [by pythagoras theorem] ⇒ RS = 8 cm
angles in a cyclic quadrilateral is always ⇒ 100 + ∠D = 180° ⇒ ∠D = 80° [angles in alternate segment]
(OP )2 = ( PT )2 + (OT )2 180°. ∴ ∠ACP = ∠PAC = 80° OB 2 = OP 2 + BP 2 We know that, TS = 1 / 3 of RT Now, in ∆SQO ,∠RSO is the exterior
A
C ⇒ ( 15)2 = ( 12)2 + BP 2 ⇒ TS = 1 / 4 of RS , if RS = 8 cm angle
[by theorem of alternate interior
B Then, TS = 2 cm ∴ ∠SQO + ∠SOQ = x ° + y °
segment] ⇒ 225 − 144 = BP 2
Also, RT × TS = PT × TQ ⇒ x + ∠SOQ = x ° + y °
P O In ∆PAC , ⇒ ( BP )2 = 81 ⇒ BP = 9 cm [When there are two intersecting ⇒ ∠SOQ = y = ∠ROP
∠P + ∠PAC + ∠PCA = 180° and in right angled ∆AOP, by chords, the product of the segments of
P 75° C
T [by angle sum property of a triangle] one chord is equal to the product of 60. (b) O
pythagoras theoram,
Q the segments of other.] C1
⇒ ( 10)2 = ( PT )2 + 62 B ⇒ ∠P + 80° + 80° = 180° (OA )2 = (OP )2 + ( AP )2 C4
30

⇒ 6 × 2 = PT × TQ C2 S
°

C3
⇒ ∠P = 180° − 160° = 20° ⇒ ( 20) = ( 12) + ( AP )
2 2 2
⇒ PT 2
= 100 − 36 ⇒ PT = 64 2
R ⇒ PT × TQ = 12
∠ACQ + ∠APQ = 180° 51. (b) Since, the length of tangents from an ⇒ ( AP )2 = 400 − 144 = 256 Now, by AM ≥ GM inequality,
⇒ PT = 64
external point to a circle are equal ⇒ AP = 16 cm
∴ PT = 8 cm ⇒ 75° + ∠APQ = 180° ( PT + TQ ) / 2 ≥ ( PT × TQ ) P T R Q
∴ CR = CS = 27 cm Hence, AB = AP − BP = 16 − 9 = 7 cm Assume that the radius of C = r and
By using theorem of circle, ⇒ ∠APQ = 180° − 75° = 105° ( PT + TQ ) / 2 ≥ 12 4
∴ BR = ( BC − CR) = 38 − 27 = 11 cm PQ = k
( PT )2 = PB × PC Since, ∠APQ + ∠BPQ = 180° 55. (b) Given,
x y z
= = =k [say] ⇒ PT + TQ ≥ 2 12 ⇒ PQ ≥ 2 12
⇒ BQ = BR = 11 cm 3 4 5 Now, PR = k / 2 = RQ = RO
[linear pair] or PQ ≥ 4 3
⇒ 82 = 5 × ( BC + PB ) join OQ, then PAQO is a rectangle. ∴ x = 3k, y = 4k and z = 5k ⇒ RS = ( k / 2) − r, RT = k / 4
∴ 105° + ∠BPQ = 180° ∴ Minimum PQ = 4 3 ST = ( k / 4) + r
⇒ 64 = 5( BC + 5) ⇒ 5BC = 39 ∴ AQ = PQ = 10 cm Since, ∠DCQ = ∠BCP = 3k
⇒ ∠BPQ = 180° − 105° = 75° 58. (a) When 2 chords AB and CD intersect Applying pythagoras theorem in
∴ BC = 78
. cm So, AB = ( AQ + BQ ) = ( 10 + 11) = 21cm [vertically opposite angle] at P then
Since, ∠ACQ is an exterior angle of triangle STR.
46. (c) Given, ∠AOB = 46°, ∆RCQ. 52. (b) Given, ∠BAX = 70° A
AP × PB = CP × PD B
2 2 2
∴ ∠BEA = 70°  k + r =  k  +  k − r
∴ ∠ACQ = ∠CRQ + ∠CQR and Hence,      
X 4   4 2 
⇒ 75° = 30° + ∠CQR [angles in alternate segment] 4 × 6 = 3 × PD, Thus, O
PD = 8 cm ⇒ r = k / 6 = PQ / 6
⇒ ∠CQR = 45° = ∠PQB E B D P
O C D A
4k 3k Now AB = AP + PB Y 61. (c)
C [vertically opposite angles] C
46° M P = 10 cm A
90° 90° In ∆BPQ,∠B + ∠P + ∠Q = 180° 5k
O B and CD = CP + PD O
[by angle sum property of a triangle] Q P Q
A B Thus, CD = 11 cm R
⇒ ∠B + 75° + 45° = 180° ⇒ ∠B = 60° In ∆DCQ, by angle sum property of a
40° Consider the circle with center O. l
1 1 49. (d) Q triangle
∴ ∠ACB = ∠AOB = × 46° = 23° C Y Drop a perpendicular from O to chord
X
2 2 A 70° ∠CDQ = 180° − (3k + 5k ) = 180° − 8k AB and CD. This will bisect the chords
45° B C
[angle subtended on the circumference By properties of cyclic quadrilateral, at X and Y . D
Since, AQBE is a cyclic quadrilateral.
∠CDQ = ∠CBA = 180° − 8k
AB In-radius of equilateral triangle of side
is half of the angle subtend on centre.]
P
y ∴ ∠AEB + ∠AQB = 180° ∴ AX = = 5 cm a
A x B 2 a= ,
In ∆MCB, ⇒ ∠AQB = 180° − 70° = 110° ⇒ ∠PBC = 8k 2 3
CD 11
In ∆ABQ, In ∆PBC , by angle sum property of and YD = = cm Diameter of larger circle =
a
∠C + ∠B + ∠BMC = 180° 2 2 3
[by angle sum property of a triangle] ∠ABQ + ∠BAQ + ∠AQB = 180° a triangle Now, PX = AX − AP = (5 − 4) = 1 cm
Q Let us say common tangent PQ touches
⇒ ∠ABQ = 180°−( 40° + 110° ) ∠P + ∠B + ∠C = 180° OY = PX = 1 cm
⇒ 23° + ∠B + 90° = 180° In ∆CAB, 45°+∠x + ∠y = 180° the two circle at R, centre of smaller
= 180°−150° = 30° ∴ 4k + 8k + 3k = 180° [Q PXOY is a rectangle] circle is I.
⇒ ∠B = 67° [by angle sum property of triangle]
MATHEMATICS Circle 307 308 CDS Pathfinder

Now, PQ is parallel to BC. AR is 64. (d) As, the tangents 66. (d) I. Circle is a locus of points which are 69. (a) In ∆OBC , we have 73. (d) Since, secants LA and BN are 77. (a) In ∆ABC , AB = BC = AC
A
perpendicular to PQ. Triangle PQR is drawn from an equidistant from a fixed point. ∠OBC + ∠OCB + ∠BOC = 180° intersecting at an exterior point M, then A
also an equilateral triangle and AORID external point to a II. Let C be a fixed circle and C be ⇒ y + y + t = 180° ⇒ t = 180° − 2 y
is a straight line. circle are equal. P R 1 2
moving circle then locus of centre of Now,z = 360°− t ⇒z = 360°− ( 180°−2 y ) r
3 a 3 a ∴ AP = AR …(i) A O
AD = a, RD = , AR = a− C is also a circle. ⇒ 2x = 180°+2 y ⇒ 2x − 2 y = 180° M • L
2
2 3 2 3 BQ = BP …(ii) B C C2 [Q z = 2x ]
N
B C
Q R
3a − 2a a 1 and CR = QC ...(iii) B
AR = = , AR = AD ⇒ x − y = 90°
2 3 2 3 3 ⇒ ∠BAC − ∠OBC = 90° Hence, ∆ABC is equilateral triangle.
On adding Eqs. (i), (ii) and (iii), we get C1 LM × AM = BM × NM
Radius of smaller circle = 1 / 3 radius of AP + BQ + CR = BP + QC + RA
O MA MN Let r be the radius of incircle and R be
70. (b) In right angled ∆OTQ , ⇒ = <1
larger circle MB LM the radius of circumcircle.
and perimeter of OT 2 = OQ 2 + TQ 2 Now, radius of incircle, r =
side AB
=
Radius of smaller circle 74. (b) The perpendicular bisector of the
1 a a ∆ABC = AB + BC + CA T 2 3 2 3
= ⋅ = III. There does not exists any point chord of a circle always passes through and radius of circumcircle,
3 2 3 6 3 = ( AP + PB ) + ( BQ + QC ) which is equidistant from three the centre. So, statement I is wrong.
side AB
+ (CR + RA ) distinct points on a line. R= =
Area of smaller circle = πr 2 12 − x The angle in a semi-circle is a right 3 3
2 = ( AP + BQ ) + ( BQ + CR) IV. If P lies on bisector of ∠B then P is 13 cm angle. So, statement II is correct. AB / 3
πa 2 R
π 
a  So, the required ratio = =
 = equidistant from AB and BC also, P x
75. (d) Here, CA = CD = CB = 25 cm
6 3 108 + (CR + AP ) lies on bisector of ∠C . A E B r AB / 2 3
2
3 2 = 2 ( AP + BQ + CR) ⇒ P is equidistant from BC and CD x = = 2: 1
Area of triangle = a P 5m Q 1
4 ∴ AP + BQ + CR = 1/ 2 ⇒ P is equidistant from AB and CD.
Hence, I, II, and IV are correct statement. O 5m 78. (b) I. AB and CD are chords when
πa 2 3 2 [perimeter of ∆ABC ] C
Ratio = : a , π : 27 3 produced meet externally at P.
67. (d) I. A c m
108 4 Hence, the statement I and II are correct. D ⇒ ( 13) = (5) + (TQ )
2 2 2
25 ∴ AP × BP = CP × DP
62. (d) I. It is true that the opposite angles 65. (d) I. Consider the possibility that M ⇒TQ 2 = 169 − 25 = 144 ⇒TQ = 12cm A B B
P 18 cm A
of a cyclic quadrilateral are could be the In right angled ∆TEB,
supplementary. centre of the D
X TB 2 = EB 2 + TE 2 P
II. It is also true that the angle circle, We know
B C
Q ( EB = BQ ) and DP = 18 cm
this could be M
subtended by an arc at the centre is So, the square is not cyclic. ⇒ ( 12 − x)2 = BQ 2 + TE 2 In right angled ∆ACP, C
double the angle subtended by it at true because M D
being the center II. A a D ⇒ 144 + x2 − 24x = x2 + (8)2 CP = CD − PD = 25 − 18 = 7cm
any point on the remaining part of Y II. Since, ABDC is a cyclic quadrilateral
point of the r ⇒ 144 + x2 − 24x = x2 + 64 Now, AC 2 = CP 2 + AP 2
the circle. ∠PAC = ∠PDB
circle would make line XM and 20 10
Hence, both statements are individually r ⇒ 24x = 80 ⇒ x = = cm [by pythagoras theorem]
and ∠PCA = ∠PBD
YM radii of the circle, which would 6 3
true, but neither statements implies to ∴ Also, ∠APC = ∠BPD [common]
mean that they are equal. B C 10 20
each other. ∴ AB = 2EB = 2x = 2 × = cm AP = AC 2
− CP 2 = ( 25 ) 2 − ( 7)2 ∴ ∆PAC ~ ∆PDB
II. The statement presents us with the Square has less area than circle. 3 3
S possiblity that 20 So, statement II is true.
63.(c) I. It is clear from the III. From the above figure it is clear that Hence, the length of AB is cm. = 625 − 49 = 576 = 24 cm
figure that points R point M lies X 79. (c) Let OA = r be the inradius of circle.
Q the diameter of circle is equal to the 3 Similarly, PB = 24 cm
Q , S and R are in a l somewhere on
diagonal of square. ⇒ 2r = a 2 Then, circumradius = OB = 2r
P the arc of XY . 71. (b) The locus obtained is the ∴ AB = AP + PB = 24 + 24 = 48 cm
straight line. M IV. circumference of the circle concentric
Well, if point A
So, statement I is true. B with the given circle and having radius 76. (c) When two chords of a circle are
M rested exactly intersect internally, then they are divided
II. Since, PQ is parallel to EF . Y equal to the distance of the chords from A r
halfway in a proportion.
the centre. •O
∴ ∠PDE = ∠DEF [alternate angle] between X and
Y Z
Y , then straight lines drawn from X 72. (c) Statements I and II are both correct, θ 2r
Also, ∠PDE = ∠EFD θ
to M and Y to M would certainly be D C because the locus of points which are B
P
P equal. So, it is possible that the sides of square equidistant from two parallel lines is a
E T X
is equal to diameter of circle. line parallel to both of them and lie mid O
III. The statement presents us with the way between them. We know that, inradius is the
D possibility that M lies somewhere Hence statement II, III and IV are perpendicular distance of centre O from
Also, it is true that the perpendicular
outside of the circle. If two tangents correct. side of triangle and circumradius OB
distances of any point on this locus line
Q
F are drawn from point M at X and Y 68. (c) We observe that the arc BC makes from two original parallel lines are equal. i.e. PX ⋅ PY = PZ ⋅ PT bisect ∠B. Again, let ∠OBA = θ
they will be equal. ∠BOC = z at the centre and ∠BAC = x further, no point outside this locus line In ∆PXZ and ∆PTY, ∠ZPX = ∠YPT In right angled ∆OAB,
[angle in the alternate segment of X at a point on the circumference. has this property. OA r
chord ED] [vertically opposite angles] sinθ = =
∴ z = 2x A C OB 2r
∴ ∠DEF = ∠DFE In ∆OBC , we have X X´ ∠PZX = ∠PYT
⇒ sin θ = = sin 30° Q sin30° = 
M 1 1
⇒ ∆DEF is an isosceles triangle. ∠OBC + ∠OCB + ∠BOC = 180° [angles in same segment] 2  2 
⇒ y + z + y = 180° Z Z´ ∠PXZ = ∠PTY ∴ θ = 30°
So, statement II is also true. Y B D [angles in same segment]
⇒ z + 2 y = 180° ⇒ 2x + 2 y = 180° Then, ∠B = 2 × θ = 2 × 30° = 60°
Hence, the statement I and II are Hence, the statement I, II and III are ⇒ x + y = 90° ∴ ∆PXZ ~ ∆PTY Hence, given triangle is an equilateral
correct. Y Y´ Hence, the both statements are correct.
correct. ⇒ ∠BAC + ∠OBC = 90° triangle.
MATHEMATICS Circle 309 310 CDS Pathfinder

80. (d) Let O be the centre of circle and AB Now, we draw a perpendicular bisector ⇒ PC =
2 2d
=
2d ⇒ y 2 + 3 y − 10 = 0 In ∆ ABC , AB = AC and AX ⊥ BC . So, I. In ∆OPQ, OP = OQ 2 − QP 2
be the chord of an arc. According to the from A to MN, which meets MN at B. 6 3 ⇒ y 2 + 5 y − 2 y − 10 = 0 AX is angle bisector of ∠A, hence it
question, passes through the incentre. Hence, all = 49 − 1 = 48 = 4 3 cm
MN 24 2 2d ⇒ y ( y + 5) − 2 ( y + 5) = 0
Then, MB = BN = = = 12 cm ∴ CD = 2PC = In ∆OMP, OP 2 = PM 2 + OM 2
2 2 3 statements are correct.
⇒ ( y + 5) ( y − 2 ) = 0
92. (b) Given, radius of a circle = 2 unit ⇒ OM = 48 − 24 = 24 cm
[Q perpendicular from centre to the 86. (b) 1 km = 1000 m, 1 dm = 10 m ⇒ y + 5 = 0 or y − 2 = 0
O chord bisects the chord] CD 2 So, OM = PM = 24 cm
Q 2 km and 26 dm ∴ y = 2 cm ∴ CP = PD = = = 1unit
z In right angled ∆ABN , = 2000 + 260 = 2260 m 2 2 ∴ ∠OPM = ∠POM = 45°
89. (c) Since, two
A B AN 2 = AB 2 + BN 2 Let radius be r m. C Now, ∠QPD = 90° + ∠OPM
y M circles touch each r O2
N ⇒ 132 = AB 2 + 122 Distance covered in revolution = 2 πr other externally if = 90°+45° = 135°
Length of chord AB = x, Radius of ⇒ 169 = AB 2 + 144 Distance covered in 452 revolution distance between a/2 Hence, statement I is correct.
A B
circle = OA = z ⇒ AB 2 = 169 − 144 = 25 their centres = A O M B O P
= 452 × 2 πr 1 II. Now, if CP = m and PD = n ,
and height of arc MN = y ⇒ AB = 5 cm Sum of their radii
Q 452 × 2 πr = 2260 a then QP × PR = CP × PD
Now, AM = MB =
AB x
= ∴ Radius of the inner circle is 5 cm. O O = r + and O M = a − r D
2260 × 7 35 1 2
2 2 ⇒ 1 × 1 = m × n ⇒ mn = 1
2 2 ∴ r= =
83. (d) Given, AD = 34 cm and AB = 30 cm 452 × 2 × 22 44 a Now, OP = OD 2 − PD 2 Also, CM = MD
[Perpendicular from the centre to a O M=
chord bisects the chord] Let C be a point on AB such that 35 35 13 1
2 = 4 − 1 = 3 units ⇒ CP + PM = PD − PM
Diameter = 2 × = =1 m ∴ AP = AO + OP = ( 2 + 3 ) units
and OM = ON − MN OC ⊥ AB 44 22 22 In right O MO ⇒ m + 24 = n − 24
1 2
=z− y [ Q ON = z (radius)] ⇒ OD = OB = 17 cm 87. (b) Distance covered in 1 revolution (O O ) 2 = (O M ) 2 + (O M ) 2 93. (c) In the given figure, OP and OQ are ⇒ n − m = 2 24
1 2 1 2
In right angled ∆OMA , CB =
AB
= 15 cm = 2 π ( 03
. ) m = 0.6 π m
2
a2
tangents to the circle with centre C .

⇒  r +  = ∴ m + n = (m − n)2 + 4mn
Also, a
2 + ( a − r )2 P
OA = OM + AM
2 2 2
Q Distance covered in 140 revolutions  2 4
[Perpendicular from centre to the
R = 96 + 4 = 10
[by pythagoras theorem] ⇒ 3r = a
2 chord bisects the chord] = 140 × 0.6 π m x So, statement II is also true
z 2 = (z − y )2 +  
x 1
⇒ ⇒ Speed the cycle ∴ r=
a C O 1
 2 ⇒ OC = OB 2 − CB 2 = 172 − 152 3 x × OP × PR
22 ar( ∆OPR)
x2 = 140 × 0. 6 × × 60 m / h III. = 2
⇒ z 2 = z 2 + y 2 − 2 yz + = 289 − 225 = 64 = 8 cm 7 S
4
90. (c) Since, OP is tangent to the circle
Q ar( ∆OMP ) 1 × OM × PM
84. (d) Since, AP = BP;DE = BD and AC =CE = 15840 m/h = 15.84 km/h OP⊥PC . 2
[Q ( a − b ) = a + b 2 − 2ab]
2 2

Q Perimeter of ∆PCD 88. (a) Let ABCD be cyclic quadrilateral. 4 P Then, OP = OQ and 4 3×1 1
x2 O = =
⇒ 2 yz − y =2
⇒ 4 ( 2 yz − y 2 ) = x2 ∠CPO = ∠CQO = 90° 24 × 24 2 3
4 = PC + PD + CD D 3 cm
⇒ 4 y ( 2z − y ) = x2 = PC + PD + CE + DE xc
C 3 I. We have, OR × SQ = OS × RP So, statement III is not true.
m ⇒ (OP − RP ) × SQ
y cm

81. (c) Given, AB and AC are the tangents = ( PC + CE ) + ( PD + DE ) 5 cm C Hence, the statement I, II are correct.
of the circle. = ( PC + CA ) + ( PD + BD ) = (OQ − SQ ) × PR 95. (c) From the given circle,
B B ⇒ (OP − RP ) × SQ
= PA + PB = 2PA = 2 × 16 = 32 cm A 2cm AP × BP = CP × PD …(i)
Here, ∆OPC is right angled triangle = (OP − SQ ) × PR
85. (c) Given, AB = d is diameter of circle in which ∠A = 60°, with ∠P = 90°
130° ⇒ SQ = PR ⇒ OR = OS
A O and CD intersect AB at P in the ratio 1 : 2.
AB = 2 cm, AD = 5 cm
(3) + ( 4) = 5 units
2 2
C OC = Hence, statement I is correct.
Since, ∠A + ∠C = 180° 3
D
∴ sin ( ∠COP ) = . II. Q ∆OPC ≅ ∆OCQ
C d [property of cyclic quadrilateral] 5 P
and OB = OC [radius of circle] P In ∆OPC , 90° + x + ∠ 1 = 180° A B
A B ⇒ 60° + ∠C = 180° 91. (d) In ∆ ABC ,
∴ ∠B = ∠C = 90° O
A ⇒ x + ∠1 = 90°
⇒ ∠C = 180° − 60° = 120° AB = AC i.e. C
[radius is perpendicular to the tangent] ⇒ ∠POC + ∠QCO = x + ∠1 = 90°
In ∆ABD, let BD be x cm. ∆ ABC is
In quadrilateral ABOC , D Hence, statement II is correct. Given that, A ′ P ′ = AP,
isosceles
∠ABO + ∠OCA + ∠BAC 22 + 52 − x 2 B ′ P ′ = BP, C ′Q ′ = CP and D ′Q ′ = DP
d 2d cos 60° = triangle. 94. (a) We have, MP = 24 cm,
+∠BOC = 360° ∴ AP = and PB = 2 × 2 ×5
3 3 If L is the locus Now,
⇒ 90° + 90° + ∠BAC + 130° = 360° OQ = OR = 7 cm
[by cosine formula] 1
⇒ ∠BAC = 180°−130° = 50°. ⇒ OA = OC = OB = d / 2 of points X B C QR 2 ar ( ∆A ′ P ′ B ′ ) × A ′ P ′× P ′ B ′
Q radius of circle = 1 diameter  1 4 + 25 − x 2 inside or on the X and QP = PR = = =1 = 2
Hence, the angle between the tangents ⇒ = ⇒ 29 − x 2 = 10 2 2
  2 2 × 2 ×5 triangle, such that BX = CX . Then, L is ar( ∆C ′Q ′D ′ ) 1 × C ′Q ′×Q ′D ′
at the ends of the radius is 50°. 2
a straight line passing through A. 2
82. (a) Given, MN = 24 cm and d d d ⇒ x 2 = 29 − 10 ⇒ x 2 = 19
In ∆OPC ,OP = OA − AP = − = As, AX is bisecting the line BC, then it AP × BP AP × BP
26 2 3 6 Again, in ∆BCD, = = = 1 [using (i)]
AN = cm = 13 cm passes through the centroid. O CP × DP AP × BP
2 Applying pythagoras theorem in ∆OPC , 32 + y 2 − x 2
cos 120° = As, AB = AC in ∆ ABC , then AX is ∴ A ′ P ′ B ′ and C ′Q ′D ′ are triangles of
OC = OP + PC
2 2 2
2 ×3 × y D
2 2 perpendicular to BC , hence it passes same area.
⇒   =   + PC 2
d d [by cosine formula] through the orthocentre. M
R So, statement III cannot be false.
 2  6 Let length of fourth side of cyclic
Q P
C Hence, only (c) is correct option.
d2 d2 quadrilateral be y cm.
A 13
cm ⇒ ( PC ) = 2

4 36 1 9 + y 2 − 19
9d 2 − d 2 8d 2 ⇒ − = [Q x 2 = 19]
M B N = = 2 2 ×3 × y
24 cm 36 36 ⇒ − 3 y = 9 + y − 192
26
311 312 CDS Pathfinder

x2 + 4x − 192 = 0 ⇒ x2 + 16x − 12x − 192 = 0 EXAMPLE 5. A lawn is in the shape of a rectangle of


⇒ x ( x + 16) − 12 ( x + 16) = 0 ⇒ ( x − 12)( x + 16) = 0 length 60 m and width 40 m. There is a footpath of
⇒ x − 12 = 0 or x + 16 = 0 uniform width 1m bordering the lawn. The area of the
∴ x = 12 [side cannot be negative] path is
Hence, side of one square = 12 cm and side of another a. 194 m2 b. 196 m2 c. 198 m2 d. 200 m2
square = 12 + 4 = 16 cm Sol. b. Given, length of the outer rectangle = 60 m
and breadth of the outer rectangle = 40 m
RECTANGLE ∴ Area of outer rectangle = 60 × 40 = 2400 m2
Let l and b be the length and breadth of a rectangle Q Width of path = 1m

AREA AND PERIMETER respectively, then


D C
∴Length of the inner rectangle = 60 − (1 + 1) = 58 m
Breadth of the inner rectangle = 40 − (1 + 1) = 38 m

Breadth(b)
Rectangle ∴ Area of the inner rectangle = 58 × 38= 2204 m2

OF PLANE FIGURES A
Length (l) B
(i) Area of rectangle = Length × Breadth = l × b
(ii) Perimeter of rectangle = 2 (Length + Breadth)
D Path

38 m
1m
C

40 m
= 2( l + b) units
(iii) Diagonal of rectangle = (Length) 2 + (Breadth) 2 A Path
B
60 m
= l 2 + b 2 units
Generally (7-9) questions have been asked from this chapter. Generally, questions are asked from Area of the path = [Area of the outer rectangle]
(iv) Area of track (Shaded region) = ( l1 b1 − l 2 b2 ) sq units
the topics related to the triangles and circles. − [Area of the inner rectangle] = ( 2400 − 2204)
D C
= 196 m2
b2 b1 Hence, the area of the path is 196 m2.
l2
w
Area of the plane figure is the amount of surface enclosed by its boundary. It is measured in square A
l1
B QUADRILATERAL
units. The perimeter of plane figure is total length of the sides enclosing the figure. Unit of Perimeter is (i) Let ABCD is a quadrilateral in which DM = h1 and
2
same as the unit of sides of a given figure. EXAMPLE 3. The area of a rectangular plot is 180 m .
If its length is 18 m. Then, its perimeter is BN = h2 are perpendiculars on diagonal AC from
a
D C
SQUARE a. 28 m b. 56 m c. 360 m d. None of these other two vertices B and D, then
1
Let each side of a square be a unit. a a Sol. b. Given, area of rectangular plot = 180 m 2 Area of the quadrilateral = × Diagonal × ( h1 + h2 )
(i) Perimeter of square = 4 (Side) = 4a units Square 2
and length of the plot = 18 m
(ii) Diagonal of square = 2 × (Side) = a 2 units Area 180 1
= × AC × ( DM + BN ) sq units
∴ Breadth = = = 10 m
1 1 Length 18 2
(iii) Area of square = Side × Side = ( a) 2 sq units = (Diagonal) 2 = d 2 sq units A B D C
2 2 Side (a) Now, Perimeter of rectangular plot
N
= 2 (Length + Breadth) = 2 (18 + 10) = 56 m
EXAMPLE 1. The perimeter of a square is 2 (2x + 4y ). Then, the area is Hence, the perimeter of rectangular plot is 56 m.
h1
h2
a. x 2 − 4 xy + 4 y 2 b. x 2 + 4y 2 c. x 2 − 4y 2 d. x 2 + 4 xy + 4 y 2 M
EXAMPLE 4. The area of the floor of a rectangular
Sol. d. Here, perimeter of a square = 2( 2 x + 4y) hall of length 40 m is 960 m 2 . Carpets of size 6 m × 4 m
A B
Perimeter of a square 2 ( 2x + 4y) are available. Then, how many carpets are required to
Q Side of a square = ∴ Side of a square = = x + 2y (ii) Let d 1 and d 2 are two diagonals and θ is angle
4 4 cover the hall?
Q Area of square = (Side)2 ∴ Area of square = ( x + 2y) 2 = x2 + 4xy + 4y 2
between them, then area of quadrilateral
a. 20 b. 30 c. 40 d. 45 1
= d 1d 2 sin θ
EXAMPLE 2. The side of a square exceeds the side of the another square by 4 cm and the sum of Sol. c. Given, area of the floor = 960m2 D 4 C 2
6
the areas of the two squares is 400cm 2 . The dimensions of the squares are Area of one carpet = 6 × 4 = 24 m2 D C
a. 8 cm and 12 cm b. 6 cm and 10 cm c. 12 cm and 16 cm d. None of these Number of carpets required d2

Sol. c. Let the side of a square be x cm. Then, side of another square = ( x + 4) cm. Area of the floor 960 θ
= = = 40 A B
Area of one carpet 24 40 m d1
Area of first square = x2 cm2 and area of second square = ( x + 4) 2 cm 2
Hence, 40 carpets are required to cover the hall.
According to the question ⇒ x2 + ( x + 4) 2 = 400 ⇒ x2 + x2 + 16 + 8x = 400 ⇒ 2x2 + 8x − 384 = 0 A B
MATHEMATICS > Area and Perimeter of Plane Figures 313 314 CDS Pathfinder

EXAMPLE 6. In a quadrilateral ABCD, diagonal EXAMPLE 8. In the parallelogram ABCD, AB = 10 cm.


AC = 44 cm and the length of the perpendicular drawn The altitude corresponding to sides AB and AD are TRAPEZIUM A

from B and D to AC are 10 cm and 20 cm, respectively. respectively 7 cm and 8 cm. Then, AD is Let the length of parallel sides of a trapezium are a and
c b
The area of the quadrilateral is a. 8.75 cm b. 8.95 cm c. 9 cm d. 9.25 cm b and distance between them is h, then h
a. 330 cm2 b. 440 cm2 c. 550 cm2 d. 660 cm2 1
Sol. a.Area of parallelogram ABCD D C Area of trapezium = (Sum of parallel sides)
Sol. d. = Base × Corresponding altitude
2
D
N × (Distance between them) B a C
∴ Area of parallelogram 8c abc
20 m ABCD = AB × DM 7 cm
m D b C (v) The radius of circumcircle =
4( Area )
44 m = 10 × 7 = 70 cm 2
…(i)
E A M B h h
A F C Also, area of parallelogram 10 cm EXAMPLE 11. The area of a triangle whose sides are
10 cm
ABCD = AD × BN = 8AD [Q BN = 8 cm] …(ii) 9 cm, 12 cm and 15 cm is
From Eqs. (i) and (ii), 8AD = 70 A a B a. 45 cm2 b. 54 cm2 c. 56 cm2 d. 64 cm2
B 70
∴ AD = = 8.75 cm 1 1 Sol. b. Here a = 9 cm , b = 12 cm and c = 15 cm
Area of quadrilateral 8 = ( AB + CD) × h = ( a + b) × h sq units
1 1 1 2 2 a + b + c 9 + 12 + 15
= AC(h1 + h2) = ( 44) ( 20 + 10) = × 44 × 30 = 660 cm 2 s= = = 18 cm
2 2 2 2 2
Hence, the area of the quadrilateral is 660 cm2.
RHOMBUS EXAMPLE 10. The difference between two parallel Area of a triangle = s ( s − a) ( s − b) ( s − c)
Let the length of each sides of a rhombus is a and
sides of a trapezium is 4 cm and the perpendicular
distance between them is 19 cm. Find the lengths [Heron’s formula]
length of both diagonals are d 1 and d 2 , then
PARALLELOGRAM C
of the parallel sides, if the area of the trapezium is = 18 (18 − 9) (18 − 12) (18 − 15)
= 18 × 9 × 6 × 3 = 54 cm 2
Let adjacent sides of a parallelogram are b and a 475 cm 2 .
a a Hence, the area of a triangle is 54 cm 2.
respectively and h is the corresponding altitude (height) d2 a. 22 cm and 18 cm b. 25 cm and 21 cm
of side a, then c. 29 cm and 25 cm d. 27 cm and 23 cm
(i) Area of the parallelogram = (Base × Height)
D O
d1
B
Sol. d. Let the length of the parallel sides of the trapezium
RIGHT ANGLED TRIANGLE
= a × h sq units a a be a cm and b cm, respectively. A figure bounded by three straight lines is called a
D Then, according to the question, ( a − b) = 4 …(i)
triangle.
C
A Let perpendicular, base and hypotenuse of a right angled
1 1
(i) Area of rhombus = × d 1 × d 2 sq units ∴ Area of trapezium = × ( a + b) × h triangle ( ∆ABC ) be p, b and h, respectively, then
Height (h) b 2
2 C
1 1
× ( a + b) × 19 = 475
A a (ii) Side of rhombus = d 12 + d 22 units ⇒ 4a 2 = d 12 + d 22 2
B 2
Base 950
(iii) Perimeter of rhombus = 4 × (Side) units ⇒ a+ b= = 50 h
(ii) Perimeter of a parallelogram = 2 (Sum of adjacent side) 19 p
= 2( a + b) units Note Diagonals of a rhombus bisect each other at right angles. ∴ a + b = 50 ...(ii)
On solving Eqs. (i) and (ii), we get a = 27 and b = 23
Note Each diagonal of a parallelogram divides it into two triangles A B
EXAMPLE 9. If the length of the diagonal of a Thus, length of parallel sides are 27 cm and 23 cm. b
of equal area.
a2 −b2 (i) Perimeter of right angled triangle = AB + BC + CA
rhombus is (a + b) and its area is sq units, then = ( b + p + h) units
EXAMPLE 7. ABCD is a parallelogram as shown in
figure, then its area is the other diagonal is
2
SCALENE TRIANGLE 1
(ii) Area of right angled triangle = × Base × Altitude
a. 12 cm2 b. 14 cm2 c. 15 cm2 d. 18 cm2 a −b a+b Let the sides of a triangle are a, b, c and h be the 2
a. a + b b. a − b c. d. 1
2 2 corresponding altitude to side a, then = × b × p sq units
Sol. a. Given, base = 3 cm and height = 4 cm
(i) Perimeter of scalene triangle, 2s = ( a + b + c ) units 2
D 3 cm C Sol. b. Let second diagonal be d units.
a2 – b2 a + b+ c EXAMPLE 12. The base of triangular field is three
90°
Given, area of rhombus = (ii) Semi-perimeter of scalene triangle, s = units
2 2 times its altitude. If the cost of cultivating the field at
1 50 per hectare be ` 675, then its base and height are
4 cm Q Area of rhombus = × d1 × d 2 (iii) Area of triangle = s ( s − a)( s − b)( s − c ) sq units
2 a. 900 m and 300 m b. 600 m and 300 m
1 a2 – b 2 1 a2 − b 2 a + b+ c
∴ × d1 × d 2 = ⇒ ( a + b) ⋅ d = where, s = [Heron’s formula] c. 500 m and 200 m d. None of these
2 2 2 2 2
90° Total cost
[Q d1 = a + b] 1 Sol. a. Area of the triangular field =
A 3 cm B or area of triangle = × a × h sq units Rate
( a − b ) ( a – b)( a + b)
2 2
2
⇒ ( a + b) d = a2 − b2 ⇒ d = = =
675
= 13.5 hec
Area of parallelogram, a+ b ( a + b) 2( Area ) 50
ABCD = Base × height = 3 × 4 = 12 cm2 ∴ d = ( a − b) [Q a2 − b2 = ( a − b)( a + b)]
(iv) The radius of incircle =
Perimeter = (13.5 ×10000) m2 = 135000 m2 ...(i)
MATHEMATICS > Area and Perimeter of Plane Figures 315 316 CDS Pathfinder

Let altitude of triangular field be x m a Sol. a. Given, radius of the wheel = 70 cm = 0.7 m
(ii) Area of isosceles triangle (v) The radius of circumcircle =
and base of field = 3x m
Circumference of the wheel = 2πr =  2 × × 0.7 = 4.4 m
3 22
1  
1
Again, area of the field = × Base × Altitude = ( s − b)( s ( s − a) = a 4b 2 − a 2 sq units A 7
2 4 66 × 1000
Distance to be covered in 1 min = = 1100 m
1 3x2 where, a = Base and b = Equal sides and 60
= × 3x × x = a a
2 2 ...(ii) s = semi− perimeter 1100
∴ Number of revolutions per minute = = 250
From Eqs. (i) and (ii), we get (iii) Area of a right isosceles triangle, in which equal sides 4.4
3x2 from a right angle is given by B a C
= 135000
2 1 Circular Ring
135000 × 2 Area = a 2 sq units EXAMPLE 15. The perimeter of an equilateral If ‘R’ and ‘r’ be outer and inner radii of a ring, then the
∴ x =
2
= 90000 ⇒ x = 300 m 2
3 triangle whose area is 4 3 cm 2 is area of ring = π ( R2 − r 2 ) sq units
C
So, base = 3x = 3 × 300 = 900 m a. 4 cm b. 3 cm c. 12 cm d. 7 cm
and altitude = x = 300 m b a 3 R
Sol. c. Area of an equilateral triangle = (Side) 2
4 r
EXAMPLE 13. The perimeter of a right triangle is
A a B 3
12 cm. The hypotenuse is 5 cm. The other two sides ⇒ (Side) 2 = 4 3, (Side) 2 = 16
and area of the triangle are (iv) If θ is the angle between two equal sides of isosceles 4
∴ Side = 4 cm EXAMPLE 18. The area of a ring whose outer and
a. 3, 4 and 6 cm2 b. 4, 3 and 12 cm2 triangle, then inner radii are respectively 20 cm and 15 cm is
Hence, the perimeter of an equilateral triangle
c. 6, 2 and 6 cm2 d. None of these 1 a. 440 cm2 b. 550 cm2 c. 565 cm2 d. 675 cm2
Area of triangle = a 2 sin θ = 3 × Side = 3 × 4 = 12 cm
Sol. a. Let other two sides of a right triangle be a cm and b cm 2 20 cm
such that a > b. A

Given, perimeter of a right angled triangle = 12 cm


CIRCLE
θ 15 cm
A a a Let the radius of a circle be r, then

5 B C
b r Sol. b. Given, radius of outer circle (R) = 20 cm
O
EXAMPLE 14. The perimeter of an isosceles triangle Radius of inner circle ( r) = 15 cm
B a C is 32 cm while equal sides together measure 20 cm. ∴ Area of the ring = [Area of outer circle
∴ a + b + 5 = 12 ⇒ a + b = 7 cm …(i) Then, area of an isosceles triangle is (i) Circumference of circle = 2πr = πD units − Area of inner circle]
Also, by pythagoras theorem, a. 48 cm2 b. 84 cm2 c. 44 cm2 d. 41 cm2 22
a2 + b2 = 25
[QD is diameter, D = 2r ] = ( πR 2 − πr 2) = π (R 2 − r 2) = ( 202 − 152)
Sol. a. Let third side of an isosceles triangle be a cm. 7
Also, ( a + b) 2 + ( a − b) 2 = 2 ( a2 + b2) (ii) Area of circle = πr 2 sq units 22 22
Given, b = 10 cm = ( 20 + 15) ( 20 − 15) = × 35 × 5 = 550 cm2
( a − b) 2 = 2 ( a2 + b2) − ( a + b) 2
∴Perimeter of isosceles triangle = a + b + b = a + 2b (iii) Distance covered by a wheel in one revolution 7 7
⇒ ( a − b) 2 = 2 ( 25) − (7) 2 = 50 − 49 = 1
∴ a− b=1 [Q a > b]..(ii) ⇒ 32 = a + 2 × 10 = a + 20 ⇒ 32 – 20 = a = Circumference of the wheel
On solving Eqs. (i) and (ii), we get ∴ a = 12 cm
Semi-circle
EXAMPLE 16. The circumference of a circle whose A diameter divides a circle into two equal parts. Each of
a = 4 cm and b = 3 cm Now, area of an isosceles triangle
1 area is 24.64 m 2 is these two arcs is called semi-circle. If r is the radius of a
∴ Area of triangle = × Base × Altitudes 1 1
= a 4b2 – a2 = × 12 4 × 102 – 122 a. 17.2 m b. 17.4 m c. 17.6 m d. 18.0 m circle, then
2 4 4 1

1
× 3 × 4 = 6 cm2 Sol. c. Let the radius of the circle be r m. (i) Area of semi-circle = πr 2 sq units
= 3 400 – 144 = 3 × 16 = 48 cm 2 2
2
∴ πr 2 = 24.64 r
22 2 7 × 24.64 (ii) Perimeter of semi-circle A B
ISOSCELES TRIANGLE EQUILATERAL TRIANGLE ⇒
7
r = 24.64 ⇒ r 2 =
22 = ( πr + 2r ) = ( π r + D) units
O

Let a be the side of an equilateral triangle, then 7 × 24.64


Let sides of an isosceles triangle are a, b and b, then ⇒ r= = 2. 8
3 22 EXAMPLE 19. If the perimeter of a semi-circular
A (i) Height (altitude) of an equilateral triangle = a 22 protractor is 36 cm, then its diameter is
2 Thus, circumference = 2πr = 2 × × 2.8 = 17.6 m
7 a. 6 cm b. 7 cm c. 7.5 cm d. 14 cm
b b 3 2
(ii) Area of an equilateral triangle = a EXAMPLE 17. The diameter of the driving wheel of Sol. d. Let the radius of the protractor be r cm, then perimeter
4
B a car is 140 cm. Then, in order to keep the speed of 36 36 r
a C
(iii) Perimeter of an equilateral triangle = 3 × Side = 3a = ( πr + 2r) = ( π + 2) r = r ⇒ 36 =
66 km/h, how many revolutions per minute must the 7 7
(i) Perimeter of isosceles triangle a wheel make? ∴ r = 7 cm
(iv) The radius of incircle =
= a + b + b = a + 2b units 2 3 a. 250 b. 275 c. 290 d. 295 Hence, diameter of the protractor = 2 r = 2 × 7 = 14 cm
MATHEMATICS > Area and Perimeter of Plane Figures 317 318 CDS Pathfinder

 π If a room of dimensions ( l × b) m is to be paved with Sol. a. Angle inscribed by the minute hand in 60 min = 360°
Quadrant of a Circle Perimeter of polygon = n × a = 2nr sin  
 n

square tiles, then
If r is the radius of a circle, then Angle inscribed by the minute hand in 30 min
1  180 °  (i) the side of the largest square tile = HCF of l and b 360°
(i) The perimeter of the quadrant Inradius of polygon, r = a cot   = × 30 = 180°
1 2  n  (ii) the least number of tiles required 60
= (Circumference of a circle) +2r l×b So, θ = 180° and r = 14 cm.
4 1  180 °  =
circumradius of polygon, R = a cosec  
1 π  2  n  (HCF of l and b) 2 Required area swept by minute hand in 30 min
= × 2πr + 2r =  + 2 r units
4 2  Area of a square inscribed in a circle of radius r is 2r 2 θ 180° 22
1 • = × πr 2 = × × 14 × 14
(i) Area of regular pentagon = 5(5 + 2 5 ) a 2 sq units 360° 360° 7
1 πr 2 4 and the side of a square inscribed in a circle of radius r
(ii) Area of the quadrant = (Area of circle) = sq units 1 22
4 4 is 2 r. = × × 14 × 14 = 308 cm2
3 3a 2 2 7
If two diameters are perpendicular to each other, (ii) Area of regular hexagon = sq units
2 • The area of the largest triangle inscribed in a semi-circle
then they divides the circle into four quadrants. of radius r is r 2 . EXAMPLE 24. Two circles touch internally. The sum
(iii) Area of regular octagon = 2 ( 2 + 1) a 2 sq units
of their areas is 116 π cm2 and distance between their
Area of Sector EXAMPLE 22. The ratio of the areas of the incircle centres is 6 cm. Then, the radii of the circles are
EXAMPLE 21. If the area of a regular hexagon is
If θ be the angle at the centre of a circle of radius r, then and circumcircle of a square are
96 3 cm 2 , then its perimeter is
2πr θ πr θ S D C
(i) Length of the arc PQ = = a. 36 cm b. 48 cm c. 54 cm d. 64 cm
360 ° 180 °
O Sol. b. Given that, area of a regular hexagon = 96 3 O O′
πr 2 θ O
(ii) Area of sector OPRQO = θ ⇒
3
3 (Side) 2 = 96 3 ⇒ (Side) 2 = 64 = (8) 2
360 ° P Q 2 A B
(iii) Area of segment PRQP R ∴ Side = 8 cm a. 4 cm and 9 cm
a. 1 : 1 b. 2 : 1 c. 1 : 2 d. 3 : 1
= (Area of sector OPRQO) − Area of ∆OPQ Hence, perimeter of a regular hexagon b. 5 cm and 10 cm
πr θ 1 2
2 = 6 × (Side) = 6 × 8 = 48 cm Sol. c. Let side AB = BC = CD = AD = x c. 4 cm and 8 cm
= − r sin θ d. 4 cm and 10 cm
360 ° 2 Q Diagonal of square = 2 x
Some Useful Results
(iv) Area of major segment QPSQ = (Area of circle) ∴Radius of circumcircle =
2x
=
x Sol. d. Let the radius of outer circle be R and radius of inner
• Angle inscribed by minute hand in 60 min = 360 ° circle = r cm
− (Area of minor segment PRQP) 2 2
• Angle inscribed by hour hand in 12 h = 360 ° x ∴ Given, πR 2 + πr 2 = 116π , R 2 + r 2 = 116
∴ Radius of incircle =
EXAMPLE 20. The arc AB of the circle with centre at O • Angle inscribed by minute hand in 1 min = 6° 2 If O and O′ be the centre of these circles, then
and radius 10 cm has length 16 cm. What is the area of • If the length of a square/rectangle is increased  π x2 π x2 
∴ Required ratio =  :  = 2 : 4 = 1: 2 OO′ = (R − r)
the sector bounded by the radii OA, OB and the arc AB? decreased by x% and the breadth is increased decreased  4 2 
2 2 2 2 by y% , the net effect on the area is given by Also, (R − r) = 6 [given]
a. 40π cm b. 40 cm c. 80 cm d. 20π cm
 ( ± x)( ± y) EXAMPLE 23. The minute hand of a clock is 14 cm So, (R + r) + (R − r) 2 = 2 (R 2 + r 2)
2
θ Net effect = ± x ± y + %.
Sol. c. Arc length = 2πr ⋅
360°  100  long. The area of the face of the clock inscribed by the ⇒ (R + r) 2 = 2 (116) − 36 = 196
A
θ θ 16 O θ minute hand in 30 min is ⇒ R + r = 196
⇒ 16 = 2πr ⋅ ⇒ =
• If the length of a square/rectangle is
360° 360° 2πr 10 16
increased/decreased by x% and the breadth is So R + r = 14 …(i)
θ B increased/decreased by y% the net effect on the area is R − r =6 …(ii)
Now, area of sector OABO = πr 2 ⋅ O 180°
360° given by On solving Eqs. (i) and (ii), we get
2 16  ( ± x) ( ± y)
= πr ⋅ = 8r = 8 × 10 = 80 cm 2 Net effect = ± x ± y + %. r = 4 cm and R = 10 cm
2πr
100 

 a. 308 cm2 b. 312 cm2 c. 412 cm2 d. 416 cm2
REGULAR POLYGON Note ( + ) Increase
( − ) Decrease
A polygon that has all sides equal and all interior angles
equal are regular polygon. • If the side of a square/rectangle/triangle is doubled the
Let, n = number of sides, a = length of side area is increased by 300%, i.e. the area becomes four
r = radius of inscribed circle (or inradius) times of itself.
R = radius of circumcircle • If the radius of a circle is decreased by x%, then, area is
1  180 °   x2 
Area of polygon = na 2 cot   decreased by  −2x +  %. And, if radius is increased
4  n   100 
 180 °  1  360 °   x2 
= nr tan 
2
 = nR sin 
2
 by x%, then area is increased by  2x +  %.
 n  2  n   100 
MATHEMATICS > Area and Perimeter of Plane Figures 319 320 CDS Pathfinder

PRACTICE EXERCISE 24. Two circles touch externally. The sum of their
areas is 130 π cm 2 and the distance between
their centres is 14 cm. The radii of the circles are
33. A garden is in the form of a
rectangle with semi-circular
endson the either side as shown 14 m
A 20 m B

(a) 11 cm, 3 cm (b) 10 cm, 4 cm in the diagram below. The


length and breadth of the C D
1. The diagonal of a square field measures 50 m. 14. The length of a rectangle is increased by 60%. By (c) 9 cm, 5 cm (d) 8 cm, 6 cm
rectangle are 20 m and 14 m, respectively. The
The area of square field is what per cent would the width have to be 25. The minute hand of a clock is 12 cm long. The cost of levelling the plot at ` 25
(a) 1250 m2 (b) 1200 m2 (c) 1205 m2 (d) 1025 m2 decreased to maintain the same area? area of the face of the clock described by the
1 (a) ` 10850 (b) ` 5425 (c) ` 8510 (d) ` 4255
2. The length of a rectangle is 2 cm more than its (a) 37 % (b) 60% (c) 75% (d) 120% minute hand in 35 min.
breadth. The perimeter is 48 cm. The area of the
2 (a) 284 cm 2 (b) 294 cm 2 (c) 274 cm 2 (d) 264 cm 2 34. Consider the adjoining D
C figure, let AB = 4 cm,
rectangle (in cm 2 ) is 15. The perimeter of a rectangular field is 240 m 26. In the given figure,
and the ratio between the length and breadth is BC = 14 cm, then the
(a) 96 (b) 128 (c) 143 (d) 144 sectors of two
5 : 3. The area of the field is m area (shaded) bounded
3. In a circle of radius 42 cm, an arc subtends an concentric circles of 3.5 by three semi-circles as
angle of 72° at the centre. The length of the arc (a) 33750 m 2 (b) 3375 m 2 (c) 3500 m2 (d) 3950 m2 radii 7 cm and 3.5 cm
shown in the adjoining
30°
are shown. The area of A B A B C
is 16. The inner circumference of a circular park is 7m figure in cm 2 , is π times
the shaded region
(a) 52.8 cm (b) 53.8 cm (c) 72.8 cm (d) 79.8 cm 440 m. The track is 14 m wide. The diameter of (a) 48 (b) 24 (c) 14 (d) 12
77 77
the outer circle of the track is (a) cm 2 (b) cm 2
4. An isosceles right angle triangle has area 4 8 35. Area of shaded portion as shown in the figure is
200 cm 2 . The length of its hypotenuse is (a) 168 m (b) 169 m (c) 144 m (d) 108 m 77
(c) cm 2 (d) None of these 5m
10 2
(a) 15 2 cm (b) cm (c) 10 2 cm (d) 20 2 cm 17. If the length and breadth of a rectangular plot
2
are increased by 50% and 20% respectively, then 27. Of the two square fields, the area of one is 1 1m
the new area is how many times the original hectare, while the other one is broader by 2%.
5. With in a rectangular garden 10 m wide and
area? The difference in their areas is
20 m long, we wish to pave a walk around the 8m
(a) 104 m 2 (b) 200 m 2 (c) 204 m 2 (d) 404 m 2
borders of uniform width so as to leave an area (a)
5
(b) 10 (c)
9
(d) None of these
of 96 m 2 for flowers. The width of the walk is 9 5 28. If the diameter of the circle is increased by
(a) 1 m (b) 2 m (c) 2.5 m (d) 2.56 m 100%, its area is increased by
18. The area of an isosceles triangle, each of whose
(a) 100% (b) 200% (c) 300% (d) 400%
6. The least number of square slabs that can be equal sides is 13 cm and whose base is 24 cm is 1m
fitted in a room 10.5 m long and 3 m wide, is (a) 60 cm 2 (b) 55 cm 2 (c) 50 cm 2 (d) 40 cm 2 29. If the two parallel sides of a trapezium are 15 cm (a) 10 m 2 (b) 12 m 2 (c) 14 m 2 (d) 16 m 2
(a) 12 (b) 13 (c) 14 (d) 15 and 25 cm respectively and the distance between
19. The difference between the sides at right angles 36. The boundary of the R
them is 7 cm, then the area of the trapezium is
7. If the side of a square be increased by 50%, the in a right angled triangle is 14 cm. The area of shaded region in the
(a) 105 cm 2 (b) 125 cm 2
percent increase in area is the triangle is 120 cm 2 . The perimeter of the adjoining diagram
(c) 140 cm 2 (d) None of these
(a) 50 (b) 100 (c) 125 (d) 150 triangle consists of three P
(a) 68 cm (b) 64 cm (c) 60 cm (d) 58 cm 30. The diagonals of a rhombus are 24 cm and 10 cm, semi-circular arc, the
8. The area of the largest circle that can be drawn respectively. The perimeter of the rhombus is C
smaller two being equal. A B
inside a square of side 14 cm in length, is 20. In a four sided field, the length of the longer (a) 50 cm (b) 52 cm (c) 60 cm (d) 68 cm If the diameter of the
(a) 84 cm 2 (b) 96 cm 2 (c) 104 cm 2 (d) 154 cm 2 diagonal is 128 m. The lengths of the
31. In the given figure OABC is a rhombus whose large one is 10 cm, then Q
9. If the radius of a circle is decreased by 50%, its perpendicular from the opposite vertices upon this the length of the
diagonal are 22.7 m and 17.3 m. The area of the three vertices A, B, C lies on the circle of radius
area will decrease by 10 cm. The area of rhombus is boundary is
(a) 25% (b) 50% (c) 75% (d) 100% field is
A (a) 31 cm (b) 10π cm (c) 20π cm (d) 19π cm
(a) 2246 m 2 (b) 2460 m 2 (c) 2540 m 2 (d) 2560 m 2
10. The area of the circle whose circumference is 37. In the adjoining figure, the larger circle with
equal to the perimeter of a square of side 11 cm 21. The adjacent sides of a parallelogram are 36 cm
and 27 cm in length. If the distance between the P
radius 4 cm is touched internally by two smaller
is B O circles which also touch each other externally at
(a) 154 cm 2 (b) 144 cm 2 (c) 134 cm 2 (d) 124 cm 2 shorter sides is 12 cm. The distance between the
longer sides is the centre O of the larger circle. The area of
11. A wire is in the form of a circle of radius 42 cm. shaded region is (in cm 2 )
(a) 9 cm (b) 10 cm (c) 11 cm (d) 12 cm
It is bent into a square. The side of the square is C
(a) 33 cm (b) 66 cm (c) 78 cm (d) 112 cm 22. The area of the quadrilateral whose sides (a) 50 3 cm 2 (b) 100 3 cm 2 (c) 75 3 cm 2 (d) 125 3 cm 2
12. A horse is tied to a pole with 28 m long string. measures are 9 cm, 40 cm, 28 cm and 15 cm and A P D
in which the angle between the first two sides is 32. In the adjoining figure, ABCD is
The area which the horse can graze is equal to a rectangle such that
a right angle is
(a) 246 m 2 (b) 2404 m 2 (c) 2464 m 2 (d) 2164 m 2 AB = 2 AD = a. P is the mid-point
(a) 206 cm 2 (b) 306 cm 2 (c) 356 cm 2 (d) 380 cm 2
13. The area of ring is 418 cm 2 . If the radius of the of AD. The area of the shaded
smaller circle is 6 cm. The radius of the bigger 23. A bicycle wheel makes 5000 revolutions in region is B C
circle is moving 11 km. The diameter of the wheel is 1 2 1 2 1 2
(a) a (b) a (c) a (d) a2 (a) π (b) 2 π (c) 3π (d) 4π
(a) 18 cm (b) 16 cm (c) 13 cm (d) 10 cm (a) 50 cm (b) 60 cm (c) 70 cm (d) 80 cm 3 2 4
MATHEMATICS > Area and Perimeter of Plane Figures 321 322 CDS Pathfinder

38. The lengths of two sides of a right angled 44. What is the area of rectangle R? Which of the statement(s) given above is/are 61. The short and long hands of a clock are 4 cm and
triangle which contain the right angle are a and I. The length of rectangle R is twice the width.
correct? e 2012 I 6 cm long, respectively. Then, the ratio of
b, respectively. Three squares are drawn on the (a) Only I (b) Only II
II. The area of rectangle R is twice the perimeter. distances travelled by tips of short hand in 2
three sides of the triangle on the outer side. (c) Both I and II (d) Neither I nor II
(a) Statement I alone is sufficient but statement II alone days and long hand in 3 days is e 2013 I
What is the total area of the triangle and the is not sufficient. 52. If the area of a circle is equal to the area of a (a) 4 : 9 (b) 2 : 9 (c) 2 : 3 (d) 1 : 27
three squares? (b) Statement II alone is sufficient but statement I alone square with side 2 π units, then what is the
(a) 2(a2 + b 2 ) + ab (b) 2(a2 + b 2 ) + 2 .5 ab is not sufficient. 62. The area of a square inscribed in a circle of
diameter of the circle? e 2012 I
(c) Both statements together are sufficient. radius 8 cm is e 2013 I
(c) 2(a2 + b 2 ) + 0.5 ab (d) 25 (a2 + b 2 ) (a) 1 unit (b) 2 units (c) 4 units (d) 8 units
(d) Neither statement I nor II is sufficient. (a) 32 cm 2 (b) 64 cm 2 (c) 128 cm 2 (d) 256 cm 2
39. A grassy field has the shape of an equilateral 53. In the ∆ABC, the base BC is trisected at D and
Directions (Q. Nos. 45-46) Answer E. The line through D, parallel to AB, meets AC
63. A circular water fountain 6.6 m in diameter is
triangle of side 6 m. A horse is tied to one of its B surrounded by a path of width 1.5 m. The area of
the questions based on the at F and the line through E parallel to AC meets
vertices with a rope of length 4.2 m. The this path (in m 2) is e 2013 I
percentage of the total area of the field which is following information. A AB at G. If EG and DF intersect at H, then
A cow is tethered at point A by a what is the ratio of the sum of the area of (a) 13.62 π . π
(b) 1315
available for grazing is best approximated by
rope. Neither the rope nor the cow is C parallelogram AGHF and the area of the ∆DHE (c) 12 .15 π (d) None of these
(a) 50% (b) 55% (c) 59% (d) 62% allowed to enter the triangle to the area of the ∆ABC ? e 2012 II 64. If an isosceles right angled triangle has area
40. From the mid-point of the side of a square of ABC . ∠BAC = 30 ° , AB = AC = 10 m. (a) 1/2 (b) 1/3 (c) 1/4 (d) 1/6 1sq unit, then what is its perimeter? e 2013 I
length 2 units, a circle of radius 2 units is (a) 3 units (b) 2 2 + 1 units
circumscribed. The area of intersection of the 45. What is the area that can be grazed by the cow if 54. If the area of a circle inscribed in an equilateral
the length of the rope is 8 m?
2
triangle is 154 cm , then what is the perimeter (c) ( 2 + 1) units (d) 2 ( 2 + 1) units
square and the circle is
1 2 176 π 2 of the triangle? e 2012 II
65. In the figure given below, the A O B
D N C (a) 134 π m (b) 121π m 2 (c) 132 π m 2 (d) m (a) 21 cm (b) 42 3 cm (c) 21 3 cm (d) 42 cm
3 3 area of rectangle ABCD is 100
E r r F 46. What is the area that can be grazed by the cow if 55. If the circumferences of two circles are in the cm 2, O is any point on AB and
θ CD = 20 cm. Then, the area of
the length of the rope is 12 m? ratio 2 : 3, then what is the ratio of their areas? D C
A B 1 2 176 π 2 e 2012 II ∆COD is e 2013 I
(a) 133 π m (b) 121π m 2 (c) 132 π m 2 (d) m (a) 2 : 3 (b) 4 : 9 (c) 1 : 3 (d) 8 : 27
M
6 3 (a) 40 cm 2 (b) 45 cm 2 (c) 50 cm 2 (d) 80 cm 2
2
56. If the circumference of a circle is equal to the 66. One side of a parallelogram is 8.06 cm and its
4 2 2
(a) π + 3 3 (b) π + 3 (c) π+ 3 (d) π+2 3 perimeter of square, then which one of the perpendicular distance from opposite side is
3 3 3 PREVIOUS YEARS’ QUESTIONS following is correct? e 2012 II 2.08 cm. What is the approximate area of the
41. In a circle with radius 2 cm, on its chord 2 2 cm (a) Area of circle = Area of square
47. If the outer and inner diameters of a stone parallelogram? e 2013 II
long taken as a diameter, another circle is (b) Area of circle ≥ Area of square
parapet around a well are 112 cm and 70 cm, (c) Area of circle > Area of square (a) 12.56 cm2 (b) 14.56 cm2 (c) 16.76 cm2 (d) 22.56 cm2
constructed. The area of part of that circle, respectively. Then, what is the area of the (d) Area of circle < Area of square
which is outside the greater circle is parapet? e 2012 I
67. The perimeter of a rectangle having area equal
(a) 1 cm 2 (b) 2 cm 2 (c) 6 cm 2 (d) 10 cm 2
(a) 264 cm 2 (b) 3003 cm 2
57. The area enclosed between the circumferences of to 144 cm 2 and sides in the ratio 4 : 9 is e 2013 II
two concentric circles is 16π cm 2 and their radii
42. If a rectangular park has length l, width w, area (c) 6006 cm 2 (d) 24024 cm 2 (a) 52 cm (b) 56 cm (c) 60 cm (d) 64 cm
are in the ratio 5 : 3. What is the area of the
a and perimeter p, which equation describing 68. What is the area between a square of side 10 cm
48. The area of a rectangle, whose one side is a is outer circle? e 2012 II
this park must be true?
2a 2. What is the area of a square having one of (a) 9π cm 2 (b) 16π cm 2 (c) 25π cm 2 (d) 36π cm 2 and two inverted semi-circular, cross-section
I. wp − 2w2 = 2a II. 2a − 2w2 − wp = 0 the diagonals of the rectangle as side? e 2012 I each of radius 5 cm inscribed in the square?
III. p − 8a = 4 (l + w )
2 2 2
(a) 2 a2 (b) 3 a2 (c) 4 a2 (d) 5 a2
58. The perimeter of a rectangle is 82 m and its area
(a) 17.5 cm 2 (b) 18.5 cm 2 e 2013 II
is 400 m 2. What is the breadth of the rectangle?
Select the correct answer using the codes given (c) 20.5 cm 2 (d) 21.5 cm 2
49. If the altitude of an equilateral triangle is e 2012 II
below (a) 18 m (b) 16 m (c) 14 m (d) 12 m 69. How many 200 mm lengths can be cut from 10 m
3 cm, then what is its perimeter? e 2012 I
(a) I and II (b) II and III of ribbon? e 2013 II
(c) I and III (d) All of these
(a) 3 cm (b) 3 3 cm (c) 6 cm (d) 6 3 cm 59. Consider the following statements
(a) 50 (b) 40 (c) 30 (d) 20
50. If the area of a rectangle whose length is 5 more I. Area of a segment of a circle is less than area of
43. A regular polygon with number of sides 12 is its corresponding sector. 70. The area of an isosceles ∆ABC with AB = AC and
than twice its width is 75 sq units. What is the
inscribed in a circle of area 9π cm 2. If r is the
perimeter of the rectangle? e 2012 I II. Distance travelled by a circular wheel of diameter altitude AD = 3 cm is 12 cm 2. What is its
radius of a circle inscribed in a polygon, then (a) 40 units (b) 30 units 2d cm in one revolution is greater than 6d cm.
consider the following statements. perimeter? e 2013 II
(c) 24 units (d) 20 units Which of the above statement(s) is/are correct? (a) 18 cm (b) 16 cm (c) 14 cm (d) 12 cm
I. Radius of circumcircle is 3 cm. (a) Only I (b) Only II e 2012 II
II. Radius of incircle is 3cos 15°.
51. A square, a circle and an equilateral triangle 71. If the area of an equilateral triangle is x and its
(c) Both I and II (d) Neither I nor II
have same perimeter.
III. Ratio of area of circumcircle to area of polygon perimeter is y, then which one of the following is
is π : 3.
Consider the following statements 60. The minute hand of a watch is 2.5 cm long. The correct? e 2013 II
I. The area of square is greater than the area of distance its extreme end transverses in 40 min is
Which of the following statement(s) are correct? (a) y4 = 432 x 2 (b) y4 = 216x 2
the triangle. (a) 10 π/3 cm (b) 3π/10 cm e 2013 I
(a) I and II (b) II and III (c) y2 = 432 x 2 (d) None of these
II. The area of circle is less than the area of triangle. (c) 10/3 cm (d) 10 cm
(c) I and III (d) All of these
MATHEMATICS > Area and Perimeter of Plane Figures 323 324 CDS Pathfinder

72. What is the area of a circle whose area is equal 81. How many circular plates of diameter d be taken 92. The sides of a triangular field are 41 m, 40 m 101. A circle of radius r is inscribed in a regular
to that of a triangle with sides 7 cm, 24 cm and out of a square plate of side 2d with minimum and 9 m. The number of rose beds that can be polygon with n sides (the circle touches all sides
25 cm? e 2013 II loss of material? e 2014 I prepared in the field if each rose bed, on an of the polygon). If the perimeter of the polygon is
(a) 80 cm 2
(b) 84 cm 2
(c) 88 cm 2
(d) 90 cm 2
(a) 8 (b) 6 (c) 4 (d) 2 average, needs 900 cm 2 space, is e 2015 I p then the area of the polygon is e 2015 II
(a) 2000 (b) 1800 (c) 900 (d) 800 pr
73. If AB and CD are two diameters of a circle of 1 1 1 (a) ( p + n) r (b) (2 p − n) r (c) (d) None of these
82. The sides of a triangle are in the ratio : : . If 2
radius r and they are mutually perpendicular, 2 3 4 93. The diameter of a wheel that makes 452
then what is the ratio of the area of the circle to its perimeter is 52 cm, then what is the length of revolutions to move 2 km and 26 dm is equal to 102. A circular path is made from two concentric
the area of the ∆ACD? e 2014 I the smallest side? e 2014 II e 2015 II circular rings in such a way that the smaller ring
π π (a) 9 cm (b) 10 cm (c) 11 cm (d) 12 cm 9 13 5 7 when allowed to roll over the circumference of the
(a) (b) π (c) (d) 2 π (a) 1 m (b) 1 m (c) 1 m (d) 1 m
2 4 22 22 11 11 bigger ring, it takes three full revolutions. If the
83. The sides of a triangle are 25 cm, 39 cm and area of the pathway is equal to n times the area
74. A rectangle of maximum area is drawn inside a 56 cm. The perpendicular from the opposite 94. From a rectangular sheet of sides 18 cm and of the smaller ring, then n is equal to e 2016 I
circle of diameter 5 cm. What is the maximum vertex on the side of 56 cm is e 2014 II 14 cm, a semi-circular portion with smaller side
(a) 4 (b) 6 (c) 8 (d) 10
area of such a rectangle? e 2014 I (a) 10 cm (b) 12 cm (c) 15 cm (d) 16 cm as diameter is taken out. Then, the area of the
(a) 25 cm 2 (b) 12.5 cm 2 84. The area of the largest triangle that can be remaining sheet will be e 2015 II 103. The number of rounds that a wheel of diameter
(c) 12 cm 2 (d) None of these (a) 98 cm 2 (b) 100 cm 2 (c) 108 cm 2 (d) 175 cm 2 7/11 m will make in traversing 4 km will be
inscribed in a semi-circle of radius r is e 2015 I
e 2016 I
75. What is the area of the larger segment of a circle (a) r 2 (b) 2 r 2 (c) 3 r 2 (d) 4 r 2 95. A square and an equilateral triangle have equal (a) 500 (b) 1000 (c) 1700 (d) 2000
formed by a chord of length 5 cm subtending an perimeter. If the diagonal of the square is
angle of 90° at the centre? e 2014 I 85. The area of a rhombus with side 13 cm and one 12 2cm, then the area of the triangle is e 2015 II 104. The base of an isosceles triangle is 300 units and
diagonal 10 cm will be e 2015 I each of its equal sides is 170 units. Then, the
25  π  25  π  (a) 24 2 cm 2 (b) 24 3 cm 2 (c) 48 3 cm 2 (d) 64 3 cm 2
 + 1 cm  − 1 cm
2 2
(a) (b) (a) 140 cm 2 (b) 130 cm 2 (c) 120 cm 2 (d) 110 cm 2 area of the triangle is e 2016 I
4 2  4 2 
96. ABCD is a square. If the sides AB and CD are (a) 9600 sq units (b) 10000 sq units
25  3π 86. Four equal-sized maximum circular plates are
(c)  + 1 cm 2 (d) None of these cut off from a square paper sheet of area 784 cm 2.
increased by 30%, sides BC and AD are increased (c) 12000 sq units (d) None of these
4  2  by 20%, then the area of the resulting rectangle
The circumference of each plate is e 2015 I
exceeds the area of the square by e 2015 II 105. Four equal disc are placed such that each one
76. Consider an equilateral triangle of side of unit (a) 11 cm (b) 22 cm (c) 33 cm (d) 44 cm touches two others. If the area of empty space
(a) 50% (b) 52% (c) 54% (d) 56%
length. A new equilateral triangle is formed by enclosed by them is 150/847 cm 2, then the radius
joining the mid-points of one, then a third 87. From a circular piece of cardboard of radius
3 cm, two sectors of 40° each have been cut off. 97. The area of a trapezium is 336 cm 2. If its of each disc is equal to e 2016 I
equilateral triangle is formed by joining the parallel sides are in the ratio 5 : 7 and the (a) 7 / 6 cm (b) 5/ 6 cm (c) 1 /2 cm (d) 5/11 cm
mid-points of second. The process is continued. The area of the remaining portion is e 2015 I
perpendicular distance between them is 14 cm,
The perimeter of all triangles, thus formed is (a) 11 cm 2 (b) 22 cm 2 (c) 33 cm 2 (d) 44 cm 2 106. AD is the diameter of a
then the smaller of the parallel sides is e 2015 II
e 2014 I circle with area 707 m2 and
88. Three equal circles each of diameter d are drawn (a) 20 cm (b) 22 cm (c) 24 cm (d) 26 cm
(a) 2 units (b) 3 units (c) 6 units (d) Infinity AB = BC = CD as shown in
on a plane in such a way that each circle touches C
98. The circumference of a circle is 100 cm. The side the figure above. All curves A B
D
77. ABC is triangle right angled at A, AB = 6 cm and the other two circles. A big circle is drawn in
of the square inscribed in the circle is e 2015 II inside the circle are semicircles
AC = 8 cm. Semi-circles drawn (outside the such a manner that it touches each of the small
100 50 2 100 2 with their diameters on AD.
triangle) on AB, AC and BC as diameteres which circles internally. The area of the big circle is (a) 50 2 cm (b) cm (c) cm (d) cm
π π π
What is the cost of levelling the
enclose areas x, y and z square units, e 2015 I
(a) πd 2 (b) πd 2 (2 – 3 )2
shaded region at the rate of ` 63 per m 2? e 2016 I
respectively. What is x + y − z equal to? e 2014 I 99. There are 437 fruit plants in an orchard planted (a) ` 29700 (b) ` 22400 (c) ` 14847 (d) None of these
(a) 48 cm 2
(b) 32 cm 2 πd 2 ( 3 + 1)2 πd 2 ( 3 + 2 )2 in rows. The distance between any two adjacent
(c) (d)
(c) 0 (d) None of these 2 12 rows is 2 m and the distance between any two 107. A rhombus is formed by joining mid-points of the
89. The angles of a triangle are in the ratio 4 : 1 : 1. adjacent plants is 2 m. Each row has the sides of a rectangle in the suitable order. If the
78. A square is inscribed in a circle of diameter 2a area of the rhombus is 2 sq units, then the area
Then, the ratio of the largest side to the same number of plants. There is 1 m clearance
and another square is circumscribing circle. The of the rectangle is e 2016 I
perimeter is e 2015 I on all sides of the orchard. What is the cost of
difference between the areas of outer and inner (a) 2 2 sq units (b) 4 sq units
2 1 3 2 fencing the area at the rate of ` 100 per metre?
squares is e 2014 I (a) (b) (c) (d) (c) 4 2 sq units (d) 8 sq units
3 2+ 3 2+ 3 1+ 3 e 2015 II
(a) a2 (b) 2 a2 (c) 3a2 (d) 4a2 (a) ` 15600 (b) ` 16800 (c) ` 18200
90. Let a, b, c be the sides of a right triangle, where 108. A circle of 3 m radius is
79. The area of a sector of a circle of radius 36 cm is (d) More information is required divided into three areas by
C
72 π cm 2 the length of the corresponding arc of c is the hypotenuse. The radius of the circle
which touches the sides of the triangle is e 2015 I 100. A square is inscribed in a right triangle with semi-circles of radii 1 m and 2 B
the sector is e 2014 I legs x and y and has common right angle with m as shown in the figure
(a) (a + b – c ) / 2 (b) (a + b + c ) / 2
(a) π cm (b) 3π cm (c) 3π cm (d) 4π cm the triangle. The perimeter of the square is above. The ratio of the three
(c) (a + 2 b + 2c ) / 2 (d) (2 a + 2 b – c ) / 2 A
given by e 2015 II areas A, B and C will be
80. What is the total area of three equilateral 91. The ratio of the outer and inner perimeters of a 2x y 4x y 2x y 4x y e 2016 I
triangles inscribed in a semi-circle of radius (a) (b) (c) (d)
circular path is 23 : 22. If the path is 5 m wide, x+ y x+ y x 2 + y2 x 2 + y2 (a) 2 : 3 : 2 (b) 1 :1 : 1 (c) 4 : 3 : 4 (d) 1 : 2 : 1
2 cm? e 2014 I
the diameter of the inner circle is e 2015 I
3 3 9 3
(a) 12 cm2 (b) cm2 (c) cm2 (d) 3 3 cm2 (a) 55 m (b) 110 m (c) 220 m (d) 230 m
4 4
MATHEMATICS Area and Perimeter of Plane Figures 325 326 CDS Pathfinder

πr 2 Decrease percent But area = 120 cm 2 [given]


New area of circle =
ANSWERS 4  5 1 
=  b − b  × × 100 % ⇒
1
x( x − 14) = 120
∴ Decrease in area   8  b  2
1 a 2 c 3 a 4 d 5 b 6 c 7 c 8 d 9 c 10 a πr 2 3 πr 2 1 ⇒ x2 − 14x − 240 = 0
= πr 2 − = = 37 % ⇒ ( x − 24)( x + 10) = 0
11 b 12 c 13 c 14 a 15 b 16 a 17 c 18 a 19 c 20 d 4 4 2
Decrease percent in area ⇒ x ≠ − 10 ∴ x = 24
21 a 22 b 23 c 24 a 25 d 26 b 27 d 28 c 29 c 30 b 15. (b) Let the length and breadth of Other side = 24 − 14 = 10 cm
 3
=  πr ×2 1
× 100 % = 75% rectangular field be 5x and 3x,
31 a 32 c 33 a 34 c 35 b 36 b 37 d 38 c 39 c 40 c 4  Hypotenuse = 242 + 102 = 676 cm
πr 2 respectively.
= 26 cm
41 b 42 c 43 d 44 c 45 d 46 c 47 c 48 d 49 c 50 a Short trick Area is decreased by l = 5x and b = 3x
∴ Perimeter = ( 24 + 10 + 26) = 60 cm
51 a 52 c 53 b 54 b 55 b 56 c 57 c 58 b 59 c 60 a  x2   (50)2  Perimeter of rectangular field = 2 ( l + b )
=  −2 x +  =  −2 × 50 +  20. (d) Here, AC = 128 m, BL = 22.7 m,
61 d 62 c 63 c 64 d 65 c 66 c 67 a 68 d 69 a 70 a  100   100  2 (5x + 3x) = 240 DM = 17.3 m
⇒ 8x = 120 ⇒ x = 15
=  −100 +
71 a 72 b 73 b 74 c 75 c 76 c 77 c 78 b 79 d 80 d 2500  D
 = −75% ∴ l = 5x = 5 × 15 = 75 m
 100 
81 c 82 d 83 c 84 a 85 c 86 d 87 b 88 d 89 c 90 a and b = 3x = 3 × 15 = 45 m
Area is decreased by 75%. Now, area of rectangle = l × b
L
91 c 92 a 93 b 94 d 95 d 96 d 97 a 98 c 99 b 100 b A C
M
101 c 102 c 103 d 104 c 105 d 106 c 107 b 108 b
10. (a) Perimeter of square = 75 × 45 = 3375 m 2
= 4 × 11 = 44 cm 16. (a) Given, inner circumference
2 πr = 44 cm [by condition] = 2 πr = 440 m B
44 ∴ Area of the field
∴ r= = 7 cm ⇒ r=
440
× 7 = 70 m
2π 2 × 22
1
= [ AC ( BL + DM )]
HINTS AND SOLUTIONS Thus, area of circle
= πr 2 =
22
× 7 × 7 = 154 cm 2
Width of track = 14 m
Q Radius of outer circle = ( 70 + 14) m
2
1
= × 128 ( 22.7 + 17.3)
2
1
1. (a) Area of square = × (Diagonal)2 5. (b) Let the width of the walk be ‘a’ m. 7. (c) Let side of a square be a. Area of 7 = 84 m
= 64 × 40 = 2560 m 2
2 20 m square = a 2 11. (b) Circumference of circle ∴ Diameter of outer circle
1 D C 22 = 2 × 84 = 168 m 21. (a) Let distance between the longer sides
= × 50 × 50 Q The side of a square be increased by = 2 πr = 2 × × 42 = 264 cm be x cm.
2 a 3a 7
50%, then new side = a + = 17. (c) Let length and breadth of rectangular
Area of parallelogram
= 1250 m 2 2 2 ∴ Length of wire = 264 cm be x and y, respectively.
Area of flowers 10 m 9a 2 = AD × LC = MC × AB
New area = Wire is bent into a square. The original area = xy
2. (c) Let length of rectangle be x cm. 4 ∴ Perimeter of square = 264 cm ∴ 36 × x = 27 × 12
New length = x + x × 50%
and breadth of rectangle = ( x − 2) cm 9a 2 5a 2 27 × 12
a Increase in area = − a2 = ⇒ 4 × Sides of square = 264 = x+ =
x 3x ⇒ x= =9
Q Perimeter of rectangle = 2( l + b ) A B 4 4 36
2 264 2 2
5a
∴ Percent increase in area = 2 × 100 ∴ Side of square = = 66 cm C
So, 2 [ x + ( x − 2)] = 48 Given, length of outer rectangle = 20 m 4 New breadth = y + y × 20% D
4a y 6y
⇒ 4x − 4 = 48 and breadth of outer rectangle = 10 m = 125% 12. (c) Given, length of string = Radius of = y+ = cm
circle = 28 m 5 5 12
52 Q Width of walk = a m Shortcut Method 3x 6 y 9 x cm 27 cm
⇒ x= = 13 Area over which the horse can graze New area = × = xy L
4 ∴ Length of inner rectangle Side of square is increased by 50%. 2 5 5
= ( 20 – a – a ) = ( 20 – 2a )m 22
∴Length = 13 cm and breadth = 11 cm ∴ Net effect on area = πr 2 = × 28 × 28 = 2464 m 2 9
7 = (original area) A M B
50 × 50 
= 50 + 50 +
and breadth of inner rectangle 5
Hence, area of rectangle % = 125% 36 cm
= ( 10 – a – a ) = ( 10 – 2 a ) m  100  13. (c) Q Area of ring = π( R2 − r 2 ) Short trick Area is increased by
= l × b = 13 × 11 ∴ Distance between the longer sides
Now, area of inner rectangle Here, r = 6 cm, R = ? 50 × 20
8. (d) Given, diameter of circle = Side of = 50 + 20 + = 80 % = 9 cm
= 143 cm 2 = ( 20 – 2a )( 10 – 2a ) 22 2 100
square = 14 cm Area of ring = 418 = ( R − 62 )
2 πrθ ∴ ( 20 − 2a )( 10 − 2a ) = 96 [given] 7 New Area = A +
80
×A= A
9 22. (b) Applying pythagoras theorem in
3. (a) Length of the arc = ∴ r = 7 cm ∆ABC , we get
⇒ 4a 2 − 60a + 104 = 0 [given] 100 5
360° D C 418 × 7 40 cm
⇒ a 2 − 15a + 26 = 0 ⇒ R2 − 36 = 18. (a) Let each equal side = a = 13 cm and
2 × 22 × 42 × 72 B C
= = 52.8 cm ⇒ ( a − 13) ( a − 2) = 0 22 base b = 24 cm
7 × 360 9 cm
But a ≠ 13, so a = 2 m ⇒ R = 133 + 36 = 169
2
∴ Area of the isosceles triangle 28 cm
4. (d) Area of an C ⇒ R= 169 = 13 cm 1
6. (c) Side of the greatest square tile = b ⋅ 4a 2 − b 2 A
isosceles right B 15 cm D
= GCM of the length and breadth of the A 4
triangle with side 14 cm 14. (a) Let the length and breadth of
92 + 402 = AC 2
=  × 24 × 4 × 169 − 24 × 24 
room 22 rectangle be l and b, respectively. 1
1 2 a Area of circle = πr 2 = ×7×7
‘a’ = a = 200 cm 2 = GCM of 10.5 and 3 is 1.5 m  4  ⇒ AC = 1681 = 41 cm
2
7 Q Area of rectangle = l × b sq units
Area of room = 10.5 × 3 m 2 = 154 cm 2 60 8l = 60 cm 2 1
∴ Side = a = 20 cm ∴ New length = l + ×l = Area of ∆ABC = × 9 × 40 cm 2
A a
B ∴ Number of tiles needed 9. (c) Let original radius of circle be r. 100 5 19. (c) Let the sides containing right 2
Hence, hypotenuse l ×b ∴ Area of circle = πr 2 New breadth = a angle be x cm and ( x − 14) cm. = 180 cm 2
=
= a2 + a2 ( H.C .F of l & b )2 Q Radius of a circle is decreased by 50%, 8l 5b
Then, lb = ×a ⇒ a= Now, area of right triangle In ∆ACD,
then
= 2a = 20 2 cm =  x × ( x − 14) cm 2
10.5 × 3 r 5 8 1 41 + 28 + 15 84
= = 14 tiles New radius = s = = = 42 cm
2.25 2 [by condition]  2  2 2
MATHEMATICS Area and Perimeter of Plane Figures 327 328 CDS Pathfinder

∆ = s ( s – a )( s – b )( s – c ) Thus, difference of area =


1
AB ( AP + PD ) 38. (c) 41. (b) Given, radius, r = 2 cm 1 2  360° 
III. Area of polygon = nR sin  
= 42( 42 – 41)( 42 – 28)( 42 – 15) = ( 10404 − 10000) m 2 2 length of chord, AB = 2 2 cm 2  n 
= 42 × 1 × 14 × 27 1
= AB × AD [Q AD = ( AP + PD )] C θ BD 1  360° 
= 404 m 2 In ∆ODB, sin = = × 12 × R sin 
2

= 14 × 3 × 3 = 126 cm 2 2
√a2 + b2 2 OB 2  12 
∴ Area of quadrilateral = Area of 28. (c) Let diameter of the circle be 2r. 1 AB 1 b
∴ Area of the circle = πr 2 = AB × = a2 = 6R2 sin 30° = 3R2
∆ABC + Area of ∆ADC 2 2 4 P
Diameter is increased by 100% Area of circumcircle πR2 π
= 180 + 126 = 306 cm 2 Q AB = a and AD = AB  A a B
D ∴ = =
New diameter = 4r A B Area of polygon 3 R2 3
23. (c) Distance covered in one revolution  2 
and new area = 4 πr 2 θ/2
11 × 1000 × 100 or π : 3
= = 220 cm Q Increase in area = 4 πr 2 − πr 2 = 3 πr 2 33. (a) Area of figure = Area of rectangle ∴ Total area 2
5000 O Hence, all three statements are
∴ Increase percentage in area + 2 (Area of semi-circular ends) = a 2 + b 2 + ( a 2 + b 2 )2 +
1
∴ The circumference of the wheel ab correct.
 3 πr 2  1 22 2
= 220 cm = × 100 % = 300% = 20 × 14 + 2 × × ×7×7 44. (c) Q From statement I
 πr
2
 2 7 = 2 ( a 2 + b 2 ) + 05
. ab
Let the diameter be ‘D’. length = 2 × width
22 Short trick Area is increased by = 280 + 154 = 434 m 2 39. (c) Suppose, a horse is tied at vertex A.
Then, πD = 220 ⇒ × D = 220  ∴Area of rectangle = 2 × width × width
x2   1002  ∴ Cost of levelling = ` ( 434 × 25) Then, area available grazing field is 2 2
7 =  2x +  % =  2 × 100 + % = 2 (width) 2
220 × 7  100   100  = ` 10850 ADE . θ 1 θ
∴ D= = 70 cm ⇒ sin = 2 = ⇒ = 45° From statement II,
A
22 = ( 200 + 100) % = 300 % 34. (c) Length of 2 2 2 2
∴ Area of rectangle = 2 × Perimeter
AC = AB + BC = 4 + 14 = 18 cm ∴ θ = 90°

m
24. (a) Let radius of given circles be x cm 29. (c) Area of trapezium = 1 / 2 60° = 2 × 2(length + width)

4.2
and ( 14 − x) cm. (Sum of parallel sides) The area (shaded) bounded by three Area of segment APBA = Area of sector
D E − Area of ∆OAB = 4 ( length + width)
∴ Sum of areas of circle × Distance between them semi-circle = Area of semi-circle with
AC as diameter − [(Area of semi-circle 1 1 From I and II,
= [ π x2 + π ( 14 − x)2 ] = 1 / 2( 25 + 15) × 7 = 140 cm 2 = . πr 2 − × OA × OB
130 π = πx2 + π( 14 − x)2 with diameter AB + Area of semi-circle 60° 60° 4 2 2 (width) 2 = 4 (2 width + width)
30. (b) Since, diagonal of rhombus bisect with diameter BC )] B C
2 (width) 2 = 12 width
[by condition] 6m 1 1
each other and perpendicular to each π r 2θ = × π × ( 2)2 − × 2 × 2
= π( 9)2 −  π ( 7)2 + π ( 2)2  ⇒ width = 6 units.
⇒ 130 = 2x2 − 28x + 196 1 1 1
other. Now, area of sector ADE = 4 2
⇒ x − 14x + 33 = 0
2
10 2  2 2  360° So, both statements together are
∴ AO = cm = 5 cm, 22 × ( 4.2) × 60°
2 = ( π − 2) cm 2
⇒ ( x − 11)( x − 3) = 0 81 1 = = 9.24 m 2 sufficient.
2 = π − [ 49 π + 4 π ] Area of shaded region = Area of
⇒ x = 11 or x = 3 24 2 2 7 × 360°
DO = cm = 12 cm semicircle with AB as diameter − Area 45. (d) The length of the rope is 8 m, then
π(81) 1 and area of equilateral
So, the radii of circles are 11 cm and 2 = − [53 π ] of segment APBA the cow will be able to graze an
3 cm. A B 2 2 3 3 area equal to the area of the circle with
π π( 28) ∆ ABC = (Side)2 = × ( 6)2 1
= × π × ( BD )2 − ( π − 2) radius = 8 m, subtracting from that the
5

25. (d) Angle inscribed by minute hand in = (81 − 53) = = 14 π 4 4


cm

2 area of the sector of the same circle with


60 min = 360°. O
2 2
= 1557
.
360° ∴ Area of shaded region
1
= × π × ( 2 )2 − ( π − 2) = 2 cm 2 angle 30°,
Angle inscribed in 35 min = × 35 9.24
60 cm ∴ Required percentage = × 100 2
12 = 14 times π 1557
.
= 210° D C 42. (c) We have, a = l × w …(i) 8
given, r = 12 cm 35. (b) Area of shaded region = Area of = 5934
. % = 59% (approx)
and p = 2 ( l + w) … (ii) 30° A
∴ AD = OA 2 + OD 2 = 52 + 122
∴ Area swept by the minute-hand in horizontal rectangle 40. (c) Given, radius of circumcircle, Putting the value of l from Eq. (i) in (ii),
35 min = 169 = 13 cm + Area of vertical rectangle r = 2 units we get
10
= Area of sector with r = 12 cm ∴ Perimeter = 4 × 13 = 52 cm wp − 2w 2 = 2a
= 5 × 1 + (8 − 1) × 1 = 5 + 7 = 12 m 2 side of square, AB = 2 units
and θ = 210° 31. (a) Here, OA = OB = OC = 10 cm p 2 − 8a = [ 2( l + w )]2 − 8lw
Since, EF = ME = MF = 2 units Now, θ
22  210  36. (b) Length of boundary = Length of arc which is equal to π(8)2 − π(8)2
= ×  12 × 12 ×  cm = 264 cm
2 2 Also, AC = 2CP = 4( l 2 + w 2 + 2lw ) − 8lw 360
7  360  ( APB + BQC + ARC ) ∴ MEF is an equilateral triangle
and CP = OC 2 − OP 2 = 102 – 52 30 176 π 2
= 4l 2 + 4w 2 + 8lw − 8lw
= π 
AB   BC  + π  AC  In ∆AEM, by pythagoras theorem = π × 64 − π × 64 = m
26. (b) Area of the shaded region = 75 = 5 3 cm  + π   
 2   2   2  AE 2 = EM 2 − AM 2 = ( 2)2 − ( 1)2 = 4 ( l 2 + w2 )
360 3
= (Area of sector with r = 7 cm,θ = 30°) As, AC = 2PC
= π   + π   + π (5)
5 5 ∴ AE = 3 units 46. (c) Since, the length of the rope is more
− (Area of sector with r = 3.5 cm, ∴ AC = 10 3 cm Hence, the statement I and III are
θ = 30°)  2  2 than that of sides AB and AC ,
∴ Area of the rhombus OABC Area of intersection region square and correct.
 22 20 π ∴ Required Area =
= 
30  1 1 circle = Area of sector MENF + Area of
×7×7×  = × d × d = × OB × AC = = 10 π cm 43. (d) I. Let R be the radius of circumcircle
  7 360  2 1 2
2 2 ∆AEM + Area of ∆FMB (area of the circle with radius 12)
Area of circumcircle = πR2
22 7 7 30   1 θ – (area of the sector of the same circle
−  × × × = × 10 × 10 3 = 50 3 cm 2 37. (d) Radius of smaller circle = 2 cm 1
 cm
2 = × πr 2 + × AE × AM ⇒ 9 π = πR2 ⇒ R = 3 cm
 7 2 2 360   2
∴ Area of shaded region 360° 2 with angle 30°)
Inradius of polygon r 30
1 = = π(12)2 − π(12)2 = 132 π m 2
=  −  cm 2 =
77 77 77 32. (c) Area of shaded region 1 + × MB × FB II.
cm 2 = [Area of larger circle – 2(Area of circumradius of polygon R 360
 6 24  8 = Area of ∆ABP + Area of ∆PDC 2 smaller circle)]
2
60° 180° 
a cot 
1 1 1 1
27. (d) Area of field = 1 hec = 10000 m 2
= × AB × AP + DC × PD 1
= [ π 42 − 2 ( π ) 22 ] = × π × (2)2 + × 3 × 1 
 n  180° 
∴ side = 10000 m = 100 m 2 2 2 360° 2 = 2 = cos   12
1 1 1 1 1  180°   n  B
= × AB × AP + AB × PD + × 3 × 1 [Q FB = EA = 3] a cosec   A

°
Q Side of other field = 102% of 100 = [ 16 π − 8 π ]

30
2 2 2 2 2  n  C
= 102 [Q AB = DC ] 4π
⇒ r = R cos 
2 180 
∴ Area of the field 1
= (8 π ) = 4 π cm 2 = + 3 = π+ 3  ° = 3 cos 15°
6 3  12 
= 102 × 102 = 10404 m 2 2
MATHEMATICS Area and Perimeter of Plane Figures 329 330 CDS Pathfinder

2 2 2
47. (c) Q Outer diameter = 112 cm Area = x ( 2x + 5) ⇒ 75 = 2x2 + 5x 3  x 3  x 3  x 57. (c) Given that, ratio of their radii = 5 : 3 So, the area of a segment of a circle is 63. (c) Area of the path = Area of circular
=   +     = 2⋅
and inner diameter = 70 cm ⇒ 2 x2 + 5x − 75 = 0 4 3 4 3 4 3 i.e. r : r = 5 :3 always less than area of its water [fountain + path]
1 2
1 ⇒ 2x 2 + 15x − 10x − 75 = 0 3  x
2 corresponding sector. − Area of circular water fountain
∴ Required area = π ( 1122 − 702 ) Now, area of ∆ HDE =  
4 ⇒ x( 2x + 15) − 5 ( 2x + 15) = 0 4 3 A II. Distance travelled by a circular wheel
⇒ ( 2x + 15) ( x − 5) = 0 3 2 r2 of diameter 2d cm in one revolution
−15 and area of ∆ABC = x ( 2d )
∴ x = 5 and 4 B = 2π = 2 × 3.14 × d = 6.28 d
70 cm 2 O r1 2 m
By given condition, 3.3
Since, width cannot be negative. which is greater than 6d cm. r=
Area of rhombus AGHF
A B ∴ Width = 5 units Hence, both statements are correct. 1.5
m
m + Area of ∆ HDE
2c and length = 2 × 5 + 5 = 15 units
11 r 5 60. (a) Q Angle described in 60 min by
∴ Perimeter of the rectangle Area of ∆ABC ⇒ 1 = ...(i) minute hand of a clock = 360° = π (33
. + 15
. )2 − π (33
. )2
r 3
= 2 ( 15 + 5) = 40 units 3  x
2
3  x
2 2 and angle described in 40 min by = [( 48
. )2 − (33
. )2 ] π
2⋅   +   Let, r = 5x and r = 3x minute hand of a clock
51. (a) Let the radius of circle is r and the
= 4 3 4 3 1 2 = ( 23.04 − 1089
. )π
1
= ( 12544 − 4900) π side of a square is a, then by given condition, 360°
3 2 Also, given that area enclosed between = × 40 = 240° = 12.15 π m 2
4 πr x the circumferences of two concentric 60°
2 πr = 4a ⇒ a = 4 2 π( 2 ⋅ 5) × 240°
1
= × 7644 ×
22 2 2 circles = 16 π cm 2 ∴ Required distance = 64. (d) Let the equal sides of isosceles right
3  x
4 7 2 3⋅   360° triangle be a cm.
πr π2 r2 ∴ π ( r 2 − r 2 ) = 16 π
1 ∴ Area of square =   = = 4 3 = 3 = 1 1 2
=
10 π Then AB = BC = a
= × 24024 = 6006 cm 2  2 4 3 2 9 3
cm
⇒ (5x)2 − (3x)2 = 16 3
4 2 2
(3.14) r 986
. r 2 x In ∆ABC , by pythagoras theorem
= = = 2.46r 2 4
48. (d) Given that, Area of rectangle = 2a 2
4 4 ⇒ 25x − 9x = 16
2 2 61. (d) Given that, length of hour hand AC 2 = AB 2 + BC 2 = 2 AB 2
54. (b) We know that, the radius of a circle = 4 cm
=l×b and area of circle = πr 2 = 3.14r 2 ⇒ 16x = 16 ⇒ x = 1
2 2
⇒ AC 2 = 2a 2 ∴ AC =
inscribed in a equilateral triangle = a / 2 3 and length of minute hand = 6 cm
2a
⇒ l × b = 2a 2 = l × a ⇒ l = 2a Again, let the side of equilateral triangle where, a be the length of the side of an ∴ x=1 A
E be x. ∴ Hour hand rotating in 1 day
equilateral triangle. ∴ r = 5 and r = 3 = 2 × 360° = 720°
Then, by given condition, 3x = 2 πr 1 2

2 πr
Given that, area of a circle inscribed in ∴Area of the outer circle = πr 2 = π(5)2 ∴ Hour hand rotating in 2 days √2
1
⇒ x= an equilateral triangle = 154 cm 2 = 2 × 720° = 1440°
a
F 3 2 = 25 π cm 2 a
∴ π 
3 2 a  Similarly,
2a ∴ Area of equilateral triangle = x  = 154 58. (b) Given that, perimeter of a rectangle
D C 4 2 3 Minute hand rotating in 1 day 90°
3 4 π2 r2 π2 2 = 82 m C
= × = r = 189. r2 2 = 24 × 360° B
154 × 7
⇒ 
a 4 9 3 3 a  a
 = = 7 × 7 =( 7)2 ∴ 2 (Length + Breadth) = 82 m
∴ Minute hand rotating in 3 days
Hence, Area of circle > Area of square 2 3 22 1
A B a ⇒ Length + Breadth = 41 m Area of ∆ABC = × a × a
> Area of equilateral triangle ⇒ = 7 ⇒ a = 14 3 cm = 72 × 360° 2
Now, in ∆ ACD, by pythagoras theorem 2 3 ⇒ l + b = 41m ...(i)
Hence, only I statements is correct. ∴ Distance travelled by hour hand ⇒
1
1 = a2 ⇒ a = 2
2
= AD + CD 2 2 ∴ Perimeter of an equilateral triangle Also, its area = 400 m 2
AC 2 π( 4) × 1440° 2
52. (c) Given that, Area of the circle = Area
= 3a = 3( 14 3 ) = 42 3 cm = = 32 π
= a 2 + 4a 2 = 5a 2 of the square = (Side) 2 ⇒ l.b = 400 ...(ii) 360° ∴ Perimeter of ∆ABC = 2a + a 2
∴ Side of square, AC = a 5 unit ∴ πr 2 = ( 2 π ) 2 ⇒ πr 2 = 4 π
55. (b) Let the radii of two circles be r and
1 Now, ( l − b )2 = ( l + b )2 − 4 lb and distance travelled by minute hand =2 2+ 2⋅ 2
Hence, area of the square = ( a 5 )2 r , respectively. 2 π( 6) × 72 × 360°
4π 2 = ( 41)2 − 4 ( 400) = = 2 ( 2 + 1) units
⇒ r2 = = 4 ∴ r = 4 = 2 units Given,
Circumference of Ist circle 2
=
= 5a 2 sq units π = 1681− 1600 =81 360
Circumference of IInd circle 3 65. (c) Given that, CD = 20 cm
49. (c) A ∴ Diameter of circle (d) = 2 ⋅ r = 2 ⋅ 2 2 ∴ l− b = 9 ...(iii) = 6 × 144 π cm
= 4 units 2 πr 2 2 rr  4 and area of rectangle ABCD = 100 cm 2
⇒ 1 = ⇒ ⇒ 1 =
1 = On solving Eqs. (i) and (iii), we get ∴ Required ratio = 32 π : 6 × 144 π
53. (b) Here, ∆ABC forms an equilateral 2 πr 3 3 r  9 A P B
r  2 = 1 : 27
a a 2 2 2l = 50 ⇒ l = 25 m and b = 16 m
triangle. ...(i) 62. (c) Let side of a square be a cm.
A 2 ∴ Required breadth (b) = 16 m
Area of Ist circle πr 2  r  4 Given that, radius of a circle = 8 cm
B C x/3 x/3 ∴ = 1 = 1 =
D
a Area of IInd circle πr 2  r  9 59. (c) I. We know that, area of segment and diameter of a circle = 16 cm
G x/3 F 2 2 (PRQP)
D Q C
Height of equilateral triangle (AD) or 4:9 = Area of sector (OPRQO) A a D
x/3 x/3 x/3 x/3
3 3 × Side 56. (c) Let the radius of a circle be r and a be − Area of ∆ OPQ ⇒ AD × CD = 100 cm 2
= × Side ⇒ 3 = H
⇒ AD × 20 = 100
2 2 the length of the side of a square. π r 2θ 1 2 a a
x/3 x/3 x/3 x/3 = − r sin θ ∴ AD = 5 cm
⇒ 2 3 = 3 × Side Given, circumference of a circle 360 2 1
S ∴ Area of ∆PDC = × PQ × CD
2 3 B C = Perimeter of a square B a C 2
∴ Side = = 2 cm x/3 D x/3 E x/3 π
3 ⇒ 2 πr = 4a ⇒ a = r = 1.57r 1
= × 5 × 20 = 5 × 10 = 50 cm 2
2 Q Length of a diagonal of a square
∴ Perimeter of an equilateral triangle 2
= 3a = 3 × 2 = 6 cm Now, area of the circle (A c ) O = Diameter of a circle [Q AD = PQ ]
where, AGHF form a rhombus and = πr = 3.14r
2 2
50. (a) Let the width of the rectangle be x ∆HDE is also an equilateral triangle. ⇒ a 2 = 16
and area of the square (As ) θ 66. (c) Area of parallelogram
∴ a = 8 2 cm
unit. ∴ Area of rhombus = (Area of ∆AGF = a 2 = 2.4649 r 2 = Base × Height
∴ Length = ( 2 x + 5) unit ∴ Area of square ABCD = a 2 = (8 2 )2
+ Area of ∆ GFH ) ∴ Area of circle > Area of square P Q = 8.06 × 2.08 = 16. 76 cm 2
According to the question, R = 64 × 2 = 128 cm 2
MATHEMATICS Area and Perimeter of Plane Figures 331 332 CDS Pathfinder

67. (a) Let b = 4x and l = 9x Now, perimeter of an isosceles triangle Now, let x and y be the length and 77. (c) In ∆ABC , by pythagoras theorem, π r 2θ 83. (c) Let the three sides of a triangle be a, b
⇒ = 72 π
∴ Area of rectangle = l × b = 2a + b = 2 × 5 + 8 breadth of the rectangle, respectively. BC 2 = AB 2 + AC 2 360° and c, respectively.
144 = 4 x × 9 x = 10 + 8 = 18 cm Now in ∆ABD, by pythagoras theorem C 72 × 360 ∴ a = 56 cm, b = 39 cm
z ⇒ θ= = 20°
144 AB 2 + AD 2 = (5)2 ⇒ x2 + y 2 = 25 36 × 36 and c = 25 cm
⇒ x2 = ⇒ x2 = 4 ∴ x = 2 71. (a) Area of equilateral triangle 10
36 3a 2 Since, they form pythagorian triplet, y 8 πr θ A
Now, b = 4 × 2 = 8 cm and = =x ...(i) Now, length of arc ACB =
4 ∴ x = 4 and y = 3 180°
l = 9 × 2 = 18 cm A 6 B 25 cm
y So, area of rectangle = 3 × 4 = 12 cm 2 π × 36 × 20 39 cm
∴ Perimeter of rectangle = 2( b + l) and perimeter = 3a = y ⇒ a = ...(ii) x
= = 4 π cm
3 75. (c) Let AO = OB = r [radius of circle] 180 c b
= 2(8 + 18) = 2 × 26 = 52 cm Now, putting the value of a from Eq. (ii) = 62 + 82 = 36 + 64 = 100 cm
∴ BC = 10 cm 80. (d) Since, ∆AOB, ∆BOC and ∆COD are
68. (d) Area between square and semi-circles in Eq. (i), we get
6 equilateral.
= Area of square − 2 (Area of semi-circle) 2
Radius of semi-circle x = = 3
3  
y D
O 2 ∴ Side of an equilateral triangle
D C 3 3 × y2 B 56 cm C
=x ⇒ x= π(3)2
4 9×4 90° Now, area of semi-circle = x = = radius of semi-circle = 2 cm a
5 2
y2 y2 B 9π Now, total area of three equilateral Perimeter of triangle, 2s = a + b + c
⇒ x= ⇒ x= A = cm 2
3 3×4 12 3 triangles
5 cm 10 cm C minor segment 2 ⇒ 2s = 25 + 39 + 56
⇒ 12 3 x = y 2 8 3
In ∆AOB, by pythagoras theorem, Radius of semi-circle, y = cm =3× (Side) 2 ⇒ s = 60 cm
2 4
On squaring both sides, we get AB = OA + OB
2 2 2 Now, area of triangle
16 π
y 4 = 432 x2 Area of semi-circle = y = cm 2 B 2 C
A B ⇒ (5 ) 2 = r 2 + r 2 [Q AB = 5 cm ] 2 = s ( s – a )( s – b )( s – c )
10 cm 72. (b) Here, a = 7 cm, b = 24 cm 25
∴ r2 = cm Radius of semi-circular,
1 22 2 10 2 2 = 60( 60 – 25)( 60 – 39)( 60 – 56)
= ( 10) − 2 × ×
2
× 5)2 = 100 − 785
. and c = 25 cm =z= = 5 cm 2 2 2
Now, area of sector 2
2 7 Semi-perimeter of triangle θ = 60 × 35 × 21 × 4 = 420 cm 2 …(i)
ACBOA = × π r2 25 π
= 21.5 cm 2
a + b + c 7 + 24 + 25 56 360° Area of semi-circle = z = cm 2 A Again area of triangle =
1
× Base
s = = = 2 2 O 2 D
Hence, the area between the square and 2 2 2 90° 25 25 π 2
semi-circular cross-section is 21.5 cm 2 . = × π× = cm 2 9 π 16 π 25 π × Altitude
= 28 cm 360° 2 8 Now, x + y − z = + − =0 3
2 2 2 =3× × 4 = 3 3 cm 2 1
69. (a) 1 m = 1000 mm According to the question, Now, area of minor segment ACBA 4 = × BC × AD ...(ii)
C 2
∴ 10 m = 10000 mm Area of circle = Area of triangle = Area of sector ACBOA − Area of triangle 78. (b) D 81. (c) D C
25 π r 2 25 π 25 From Eqs. (i) and (ii), we get
Number of 200 mm lengths that can be = s ( s − a )( s − b )( s − c ) = − = − 1
cut from 10 m of ribbon 8 2 8 4 O d/2 d/2 × BC × AD = 420
= 28( 28 − 7)( 28 − 24)( 28 − 25)  25 π − 50  2 2
=
10000
= 50 =  cm 2a
 8  ⇒
1
× 56 × AD = 420
200 = 28 × 21 × 4 × 3 = 7056 = 84 cm 2
Area of major segment = Area of circle B
2
1 A
70. (a) Area of the ∆ABC = × b × h 73. (b) Diameters of a circle intersect at the − Area of minor segment d/2 d/2 420
2 centre of circle. ⇒ AD = = 15 cm
25 π − 50  For inscribed circle, Diameter of circle
A C = πr 2 −   = Diagonal of square = 2 a
A
2d
B 28
 8  Hence, the length of AD is 15 cm.
Now, in ∆ABC by pythagoras theorem,
r 25 π ( 25 π − 50) From the figure it is clear that, 4 circular
= − AB 2 + BC 2 = AC 2 ⇒ 2 AB 2 = 4a 2 84. (a) Let the r be the radius of semi-circle.
2 8 plates of diameter d can be made from.
a
A r r B
100 π − 25 π + 50 75 π + 50 AB 2 = 2a 2 ⇒ AB = 2a In ∆ABC ,
a O = = square plate of side 2d with minimum
r Q Area of inner square AO = OB = OC = r [radius of circle]
3 cm 8 8 loss of material.
= AB 2 = ( 2a )2 = 2a 2
(h ) 25  3 π ∴ Area of largest triangle
+ 1 cm 2
25
D = (3 π + 2 ) =  For circumscribed square, 82. (d) Given, sides of a triangle are in the
8 4  2  1
= × Base × Height
Area of circle Diameter of a circle = Side of square 1 1 1
ratio = : : i.e. 6 : 4 : 3
B C Required ratio = 2
D Area of ∆ACD
76. (c)
A ∴ Area of circumscribed square 2 3 4
b/2 b/2
b πr 2 = ( 2a )2 = 4a 2 Let the sides of a triangle be 6x, 4x and
C
= = π ∴ Difference between areas of outer 3x, respectively.
1 1
⇒ 12 = × b ×3 × 2r × r E and inner squares r
2 D We know that,
2 = 4a 2 − 2a 2 = 2a 2
A B
12 × 2 74. (c) Area of rectangle inscribed in a circle Perimeter of a triangle = Sum of all sides r O r
∴ b= = 8 cm 79. (d) Given that, area of sector = 72 π cm 2
2r
3 is maximum, when B C of a triangle
b 8 F B
Here, BD = CD = = = 4 cm ⇒ 52 = 6x + 4x + 3x =
1 1
× AB × OC = × 2r × r
2 2 A B Perimeters of triangles

cm
⇒ 52 = 13x 2 2
1 1 1
In right angled ∆ABD, by pythagoras = 3 + 3 × + 3 × + 3 × + ...
theorem, 2 4 8 36
O θ
C
Q x=
52
=4
[Q AB = OA + OB ]
13 = r2
= 3  1 + + + + .... = 3 ×
D C 1 1 1 1
AB = BD 2 + AD 2
 2 4 8  1−
1 ∴ Smallest side of a triangle Hence, the area of the largest triangle
⇒ a= 42 + 32 = 16 + 9 Diameter of the circle = Diagonal of the 2 A = 3x = 3 × 4 = 12 cm is r 2 .
= 25 = 5 cm rectangle = 6 units
MATHEMATICS Area and Perimeter of Plane Figures 333 334 CDS Pathfinder

85. (c) Given, a side of rhombus ( a ) = 13 cm 87. (b) Given, radius of circle, r = 3 cm πd 2 ( 2 + 3 )2 ⇒ 2ab = ( a + b )2 – c 2 Q 452 × 2 πr = 2260 97. (a) Area of trapezium
=
and a diagonal of rhombus (d ) = 10 cm 12 ( a + b )2 – c 2 2260 × 7 35 1
1 Area of remaining portion ⇒ ab = r= = = (Sum of parallel side) × Distance
Now, in right angled ∆ABD, 2 452 × 2 × 22 44 2
= Area of circle – 2 (Area of sector Hence, the area of big circle is
ab 35 35 13 between them
E C OABQO). πd 2 ( 2 + 3 )2 Now, area of triangle = [from Eq. (ii)] Diameter = 2 × = =1 m
= . 2 44 22 22 1
2 π r 2θ ⇒ (5x + 7x ) × 14 = 336
cm

= πr 2 – 12 [( a + b ) – c ]
2 2
2
5

D 360° 40° 89. (c) Let the angles of a triangle be 4x, x [( a + b )2 – c 2 ] 94. (d) Area of remaining portion
2θ  ∆= 2 ,∆= ⇒ 12x × 7 = 336 ⇒ x = 4
= πr 2  1 –
cm

90° and x, respectively. 2 4 = Area of rectangle – Area of semi-circle


 360° 
5

O
A ∆ The length of smaller side
A B i.e. ∠A = 4x, ∴ Radius of circle, r =
D C
13 cm 2 × 40 
= πr 2  1 − = 4 × 5 = 20 cm
40° ∠B = x and s
 360°  ∠C = x
120° ( a + b – c )( a + b + c ) ( a + b − c ) 14 cm
AB 2 = AD 2 + BD 2 A Q a = = 98. (c) We have, 2 πr = 100

a

× 9 1–  a + b + c
B
4 × 
22 8 2
⇒ 13 = 5 + BD
2 2 2 = Q Sum of all 
7  36  angles of a 30° 30°  2  A B D C
⇒ BD 2 = 169 – 25 = 144 ( a + b – c)
18 cm
triangle = 180° B b C
Hence, the radius of circle is 1 22 O
× 9 1 –  =
BD = 12 cm 22 2 22 7 = 18 × 14 − × ×7×7
Q = × × 9 = 22 cm 2 ∴ ∠A + ∠B + ∠C = 180° 2
7  9  7 9 2 7
∴ EB = 2 × BD = 2 × 12 = 24 cm 91. (c) Let the radius of outer circle and = 252 − 77 = 175 cm 2 A B
Length of second diagonal ( d ) = 24 cm 88. (d) Given, diameter of each circle = d inner circle be R and r , respectively.
2 ⇒ 4x + x + x = 180° ⇒ 6x = 180°
1 ∴ Radius of each circle = d / 2 Perimeter of outer 95. (d) Let length of side of square and ∴ 2r=
100
cm
Now, area of rhombus = × d × d ⇒ x = 30° circle = 2 πR
2 1 2 F equilateral triangle be x and y, π
1 ∴ ∠A = 4x = 30 × 4 = 120°, and perimeter of respectively.
= × 10 × 24 = 120 cm 2 Let the side of square be l cm.
inner circle = 2 πr R 4x = 3 y
2 d/2 d/2 ∠B = x = 30° Then, …(i) 100 50 2
A B 2 πR 23 ∴ l 2= ⇒ l= cm
Shortcut Method and ∠C = x = 30° Q = r Given, diagonal of square = 12 2 π π
2 πr 22 5m
Here, a = 13 cm, d = 10 cm So, it is clear that, given triangle is R 23 ⇒ x 2 = 12 2 99. (b) There are 437 fruits plants.
1 d/2
isosceles triangle. Let the sides isosceles ⇒ =
We know that, D C r 22 Q x = 12 cm …(ii) Q 437 = 19 × 23
triangle be a, a and b, respectively. 4x 4
4a =2
d2 + d2 E From the figure, R = 5 + r ∴ y= = × 12 ∴ There are 19 rows and 23 trees or
1 2 ∴ Perimeter of triangle 5 + r 23 3 3
CE = AF = BF = d / 2 ∴ = ⇒ 110 + 22r = 23r there are 23 rows and 19 trees.
⇒ 4 × 132 = 102 + d 2 = a + a + b = 2a + b r 22 = 16 cm
2
Here, ∆ABC is an equilateral triangle. b a 3
Given, the distance between any two
⇒ 4 × 169 = 100 + d 2 Using sine rule, = ⇒ r = 110 m So, area of triangle = × 16 × 16 adjacent plants is 2 m and the distance
2 ∴ Altitude of equilateral triangle sin 120° sin 30° 4
So, diameter of the inner circle is 220 m. between any two adjacent rows is 2 m.
⇒ 676 – 100 = d2 =
3
(Side) =
3
( AB ) =
3

b
=
a

2b
= 2a ⇒ a =
b = 64 3 cm 2
2 d 92. (a) Let the sides of a triangular field be ∴ Length of orchard
⇒ d = 576 = 24 cm 2 2 2 3 / 2 1/ 2 3 3 96. (d) Let the side of square be x.
2 a, b and c, respectively. = [ 1 + 22 ( 2) + 1] = 46 m
1 [Q AB = AF + FB = d ] b
=
b
∴ a = 41 m, b = 40 m and c = 9 m Area of square = x 2
Now, area of rhombus = d d Now,
Breadth of orchard
2 1 2 Let D be the centroid of ∆ABC 2a + b 2 × b + b
Perimeter of triangular field = a + b + c Now, increased length
1 3 = [ 1 + 18 ( 2) + 1] = 38 m
= × 10 × 24 = 120 cm 2
= x  1 +
Q Ratio of centroid of equilateral ⇒ 2s = a + b + c = ( 41 + 40 + 9) 30  13x
2 triangle = 2 : 1 b 3  = ∴ Perimeter of orchard
= = ⇒ 2s = 90 ⇒ s = 45 m  100  10
86. (d) Let the side of a square paper sheet C 2b + 3b 2 + 3 and increased breadth = [( 46 × 2) + (38 × 2)] = 168 m
Q Area of triangular field
be a cm. 3 ∴ Cost of fencing at the rate of
= x  1 +
= s ( s – a )( s – b )( s – c ) 20  12x
Given, area of square paper sheet  =
d d 90. (a) Given, a, b and c be the sides of a = 45( 45 – 41)( 45 – 40)( 45 – 9)  100  10 = ` 100 per metre
= 784 cm 2 right triangle, where c is the hypotenuse.
D = 45 × 4 × 5 × 36 Now, new area of rectangle = 168 × 100 = ` 16800
⇒ a 2 = 784 ⇒ a = 28 cm ∴ c =a +b
2 2 2
...(i) 13x 12x 156x 2
= 9 ×5 ×5 × 4 × 9 × 4 = × = 100. (b) Let length of the side of square be l.
D C A 10 10 100
A B = 9 × 5 × 4 = 180 m 2 A
F Increased in area 90°–θ
Given, area of each rose bed = 900 cm 2
2 3 3 900 156x 2 56x 2
∴ Length of DC = × d = d = m2 = −x =2
28 cm c b
3 2 3 100 × 100 100 100
O Percentage increased in area y F D
Now, radius of big circle, ∴ Number of rose beds
r 56x 2
3 Area of triangular field = × 100
R = DE = DC + CE = d+
d
B =
28 cm a C Area of one rose bed 100x 2 l
A B 3 2
28 and perimeter of triangle 2s = a + b + c 180 × 100 × 100 = 56% l θ
Q Diameter of one circle = = 14 cm  3 1  d × 3( 2 + 3 ) = = 2000 B C
=d  + = a+ b+ c E
2 ∴ s = 900 Shortcut Method
 3 2 6 2 x
14 93. (b) 1 km = 1000 m 1 dm = 10 m l
and radius of one circle = = 7 cm ∴ Area of big circle = πR2 1 Here, x = 30% , y = 20% In ∆DEC , tan θ =
DE
=
2 Area of triangle = × BC × AC Q 2 km and 26 dm
…(i)
x −l
∴ Net effect =  x + y +

2 2 xy  EC
∴ Circumference of each plate = 2 πr 3( 2 + 3 )  = 2000 + 260 = 2260 m %
= πd ×  1  100  FD
22  6  ⇒ ∆ = ab …(ii) Let radius be r m. In ∆AFD, tan ( 90° − θ) = cot θ =
= 2π × 7 = 2 × × 7 = 44 cm 30 × 20 
= 30 + 20 +
2 Distance covered in revolution = 2 πr AF
7  d 2 × 3( 2 + 3 )2  = 56%
 100 
l
Hence, the circumference of each plate = π×  From Eq. (i), c = a + b
2 2 2
Distance covered in 452 revolution = …(ii)
 36  y−l
is 44 cm. ⇒ c 2 = ( a + b )2 – 2ab = 452 × 2 πr Hence, area is increased by 56%.
335 CDS Pathfinder

27
MATHEMATICS Area and Perimeter of Plane Figures 336

On multiplying Eqs. (i) and (ii), we get 104. (c) Here, base = 300 units and side 2 π  16 707 4 707  1
= ⋅ − ⋅ = × 707 m 2
l2 = 170 units 8  9 π 9 π  3
1=
( x − l) ( y − l) We know that, ∴ Cost of levelling the shaded portion
⇒ xy − xl − yl + l 2 = l 2 Area of an isosceles triangle 707
= × 63 = ` 14847
xy Base 3
Q l( x + y ) = xy ⇒ l= = 4 (Side) 2 − (Base) 2
x + y 4 107. (b) We know that, Area of rhombus
4xy 300 1
∴ Perimeter of square = 4l = = 4( 170)2 − (300)2 = × product of diagonals
x + y 4 2
= 75 4 × 28900 − 90000 1
= ×d ×d
101. (c) Let a be the side of regular polygon 2 1 2
a = 75 115600 − 90000 = 75 25600
then, inradius, r =
π

SURFACE AREA AND


D C
2 tan   = 75 × 160 = 12000 sq units
 n
π 105. (d) Let the radius of the circle be r. d2
⇒ 
a = 2r tan   d2
 n Then, side of square ABCD = 2 r d1
∴ Perimeter of polygon,

π
⇒ tan   =
p
π
p = na = 2nr tan  
 n

…(i)
A r r B A

From the above figure,


d1
B
VOLUME OF SOLIDS
 n 2nr Area of rectangle = d × d
1 2
π C D
Area of polygon = nr tan  
2 = 2 (Area of rhombus)
 n
= 2 × 2 = 4 sq units
p pr ∴ Area of empty space = Area of square
= nr ×
2
= [from Eq. (i)] 108. (b) In the given question, A radii of 3 m
2nr 2 − 4 [Area of quadrant] Regularly (10-12) questions have been asked from this chapter. Questions from this chapter are
is divided in such a way that the radii of
= ( 2 r ) 2 − 4  × πr 2 
102. (c) Let the radius of smaller ring be r 150 1 direct formula based, so one must be through with all the formulae and their application. It is easy
⇒ smaller semi-circle is 1 m and radii of
and the radius of bigger ring be r.
1
847  4 
bigger semi-circle is 2 m. to score section in the exam.
150 6 2
⇒ = r
847 7 C
⇒ r2 =
25 Q π = 22 
121  7  y B
x w
z
5 In this chapter, we will study how to find the surface area and volume of solid figures, like
⇒ r= cm
Since, the smaller ring takes three full 11 A parallelopiped, cube, cuboid, cylinder, cone, frustum of cone, sphere and hemisphere and the
revolutions to roll over the 106. (c) Area of circle with diameter (AD) combination of solid figures.
circumference of bigger ring.
π( AD ) 2
∴ 2 πr = 3 ( 2 πr ) ⇒ r = 3 r = Area of shaded portion A
1
Area of pathway = πr 2 − πr 2 = n( πr 2 )
1
4
= Area of semi–circle of radii 3 m − Area SOLID FIGURES
1 1 π( AD ) 2 of semi–circle of radii 2 m + Area of
⇒ πr = ( n +
2
1) πr 2 So, = 707 The objects which occupy space (i.e. they have three dimensions) are called solid figures. The solid
1 4 semi- circle of radii 1 m
r2 figures can be derived from the plane figures.
⇒ r 2 = ( n + 1) ⇒ n = 9−1=8 ⇒ AD =
707
×4 = 2
707
m
1 1 1
= π(3)2 − π ( 2)2 + π ( 1)2
9 π π 2 2 2 A solid figure has surface area and volume.
103. (d) Number of rounds that a wheel of 1
As, AB = BC = CD =
AD 2 707
= m = π ( 9 − 4 + 1) = 3 π
diameter D will make in traversing a
distance = n (πD )
3 3 π 2 Surface Area of Solid Figure
and AC = 2 AB = BD Area of portion B
Given that, distance = 4 km Surface area of a solid body is the area of all of its surfaces together. It is always measured in square units.
4 707 = 2 [Area of semi-circle of radii 2 m Surface area is also referred to as the total surface area. The area of four walls or the area excluding top
⇒ n( πD ) = 4000 m = 2CD = m − Area of semi-circle of radii 1 m]
3 π and bottom is called curved surface area or lateral surface area.
n  π ×  = 4000
7
= 2  π ( 2) 2 − π ( 1) 2  = 3 π
⇒ 1 1
 11 ∴ Area of shaded portion = 2  2 
2
[Area of semi-circle with AC as Volume of Solid Figure
n  ×  = 4000
22 7
⇒ diameter − Area of semi-circle with
Similarly, area of shaded portion C
 7 11 The measure of space occupied by a solid is called its volume. It is always measured in cubic units.
AB as diameter] = Area of portion A
⇒ n = 2000 2 π( AC ) 2 2 π( AB ) 2 Hence, the ratio of areas A, B and C is
Hence, the number of rounds is 2000.
=
8

8 1 : 1 : 1.
Cuboid
A figure which is surrounded by six rectangular surfaces, is called cuboid. It is also called rectangular
parallelopiped.
• A cuboid has 8 corners, 12 edges, 6 faces and 4 diagonals.
MATHEMATICS > Quadratic Equations and Inequalities 337 338 CDS Pathfinder

• Volume of cuboid = Area of base × height = l × b × h D 8 8 C Sol. c. Volume of cube = (side) 3 = 2197 cm3 (i) Volume of the cylinder A
B
where, l = length, b = breadth and h = height 8 8 ⇒ Side of cube = 3 volume of a cube = 13 cm = (Area of base) × Height
D C = πr 2 h cu units
32 ⇒ Surface area of cube
36 20
= 6 (side) 2 = 6 (13) 2 = 6 × 169 = 1014 cm2 (ii) Curved surface area = Circumference h
A B and length of the diagonal of the cube = 3 × side of the base × Height
• Total surface area of cuboid = 2( lb + bh + lh) 8 8 = 3 × 13 = 1. 732 × 13 = 22.516 cm = 2πrh sq units
A 8 8 B (iii) Total surface area = Curved surface + D
• Diagonal of the cuboid = l 2 + b 2 + h 2 48
EXAMPLE 7. How many 6 m cubes can be cut from C
Area of two ends
Lateral surface area or Area of four walls = 2 ( l + b) h.
D C a cuboid measuring 36 m × 15 m × 8 m?
= 2πrh + 2πr 2 = 2πr ( h + r ) sq units

8
a. 10 b. 15
EXAMPLE 1. The volume of a cuboid is 880 cm 3 , the c. 19 d. 20 EXAMPLE 9. The volume of a cylinder is 448 π cm 3
area of its base is 88 cm 2 . Then, its height is Sol. d. Volume of given cuboid = ( 36 × 15 × 8) m3
a. 10 cm b. 12 cm c. 14 cm d. 16 cm 32 and height 7 cm. Then, its lateral surface area and
20 Volume of the cube to be cut = (6 × 6 × 6) m3 total surface area are
Sol. a. Here, volume of a cuboid = 880 cm3 A B ∴ Number of cubes that can be cut from the cuboid a. 349 cm2 and 753.286 cm2
Area of the base = 88 cm2 When square of side 8 cm is cut-off from each corner Volume of the cuboid 36 × 15 × 8 b. 352 cm2 and 754.286 cm2
= = = 20
Volume of the cuboid 880 and the flaps turned up, we get an open box whose Volume of the cube 6 ×6 ×6 c. 353 cm2 and 755.286 cm2
Height = = = 10 cm
Area of the base 88 Length = 48 − (8 + 8) = 32 cm d. None of the above
Breadth = 36 − (8 + 8) = 20 cm
EXAMPLE 8. Two cubes each of 10 cm edge are Sol. b. Given, volume of the cylinder = 448 π cm3
EXAMPLE 2. The length of the longest rod that can joined end-to-end. Then, the surface area of the
be placed in a room 12 m long, 9 m broad and 8 m and height = 8 cm and height of the cylinder = 6 cm
resulting cuboid is
high is ∴ Volume of the box = l × b × h = 32 × 20 × 8 = 5120 cm3 Let radius be r, then πr 2h = 448 π
D C
a. 15 m b. 16 m 448 π 448
∴ r2 = = = 64 ⇒ r = 8 cm
c. 17 m d. 17.5 m Cube 10 hπ 7
Sol. c. Since, the length of the longest rod is the diagonal of A cuboid whose length, breadth and height are same is ∴ Lateral surface area of the cylinder = 2πrh
B
the room 22
called a cube. A = 2× × 8 × 7 = 352 cm2
10 10 10 7
= l + b + h = (12) + (9) + (8)
2 2 2 2 2 2
• A cube has 6 surfaces, 12 edges, 8 corners and 4
a. 100 cm2 b. 1000 cm2 Total surface area of the cylinder = 2π r (h + r)
diagonals.
= 144 + 81 + 64 = 289 = 17 m 22
• Cube is a special case of cuboid which has 6 square c. 2000 cm2 d. None of these = 2× × 8 (7 + 8)
7
EXAMPLE 3. The areas of three adjacent faces of a faces. Sol. b. Length of the resulting cuboid = 10 + 10 = 20 cm 22 5280
cuboid are x , y and z. If its volume is V, then which of = 2× × 8 × 15 = ≈ 754.286 cm2
• Volume of a cube = (side) 3 D C Breadth of resulting cuboid = 10 cm 7 7
the following is true? Height of resulting cuboid = 10 cm
• Total surface area of a cube
a. V = x 3 y 2 z 2 b. V 2 = xyz ∴ Surface area of resulting cuboid = 2 ( lb + bh + hl ) EXAMPLE 10. The radii of two cylinders are in the
= 6 × (side) 2
x 2y = 2[ 20 × 10 + 10 × 10 + 20 × 10] ratio 2 : 3 and their heights are in the ratio 5 : 3. Then,
c. V = 3 xyz d. V = • Diagonal of a cube = 3 × edge the ratio of their volumes is
z A B = 2[ 200 + 100 + 200] = 2 × 500 = 1000 cm2
Sol. b. Let the dimensions of a cuboid be l , b and h • Lateral surface area or area of four walls = 4 × (side) 2 a. 4 : 9 b. 16 : 25 c. 20 : 27 d. None of these
respectively, then volume of cuboid, V = lbh IMPORTANT FORMULAE Sol. c. For first cylinder,
2
Also, x = lb, y = bh and z = hl EXAMPLE 5. If the surface area of a cube is 96 cm , Volume of prism = Area of base × height
l
Let radius = r1, height = h1, volume = V1
∴ V = lbh
then, its volume is l
Lateral surface area of prism = Perimeter of base × height For second cylinder,
On squaring both sides, we get a. 16 cm3 b. 64 cm3
l
Volume of pyramid = 1 / 3 × Area of base × height
c. 12 cm3 d. 32 cm3 Let radius = r2 , height = h2 and volume = V2
V 2 = l 2b2h2 = ( lb) ⋅ ( bh) ⋅ (hl ) = xyz l
Lateral surface area = 1 / 2 × Perimeter of base × slant
Sol. b. Surface area of cube = 6 × a2 , where a = Side of a cube r1 2 h 5 2r 5h
height Then, = and 1 = ⇒ r1 = 2 and h1 = 2
EXAMPLE 4. A metallic sheet is of rectangular shape r2 3 h2 3 3 3
⇒ 6 a2 = 96 ⇒ a2 = 16 ⇒ a = 4 cm
with dimensions 48 cm × 36 cm from each of its corners Right Circular Cylinder Required ratio of their volumes be V1 : V2.
a square of 8 cm is cut-off. An open box is made of ∴ Volume of the cube = a 3 = 4 3 = 64 cm3
A right circular cylinder is a solid figure obtained by ⇒ πr12 h1 : π r22 h2 ⇒ r12h1 : r22 h2
the remaining sheet, what is the volume of the box?
EXAMPLE 6. The surface area and the length of revolving the rectangle, say ABCD about its one side,
a. 13824 cm3 b. 1728 cm3 4 2 5h2 2 20
the diagonal of the cube, if the volume of a cube is say BC. Let base radius of right circular cylinder be r ⇒ r2 : r2 h2 ⇒ : 1 ⇒ 20 : 27
c. 5120 cm3 d. None of these 9 3 27
2197 cm 3 , are and its height be h. Then,
Sol. c. Given length of metallic sheet = 48 cm
a. 1012 cm2 and 21.516 cm b. 1024 cm2 and 24.516 cm
Breadth of metallic sheet = 36 cm c. 1014 cm2 and 22.516 cm d. None of these
MATHEMATICS > Quadratic Equations and Inequalities 339 340 CDS Pathfinder

EXAMPLE 11. A cylindrical bucket of diameter 28 cm


Right Circular Cone Frustum of a Cone (ii) Surface area of sphere = 4πr 2 sq units
and height 12 cm is full of water. The water is emptied A 4
A right circular cone is a solid, If a right circular cone is cut-off by a plane parallel to (iii) Volume of a hollow sphere = π ( R3 − r 3 ) cu units
into a rectangular tub of length 66 cm and breadth 28 cm.
The height to which water rises in the tub is generated by the revolution of a the base of the cone, then the portion of the cone 3
right angled triangle about one of between the cutting plane and base of the cone is called where, r = Inner radius and R = Outer radius
a. 2 cm b. 4 cm c. 6 cm d. 8 cm
its sides containing the right l the frustum of the cone. Let R, r be the radii of base
Sol. b. Volume of water in the bucket angle. Let height of a right h and top ( R > r ) of the frustum of a cone. Let height of EXAMPLE 16. Given that the volume of a metal
22 circular cone be h, slant height be frustum of a cone be h and slant height be l, then sphere is 38808 cm3 . Then, its radius and its surface
= πr 2h = × 14 × 14 × 12 = 7392 cm3 area are
7 l and its radius be r, Then, D r C
Let h be the height to which water rises in the tub. B r
C a. 7 cm and 616 cm2 b. 21 cm and 5544 cm2
(i) The slant height of the cone, O
∴ Volume of water in the tub = 66 × 28 × h cm 3 c. 14 cm and 2464 cm2 d. None of these
l = AC = r + h units
2 2 h
Sol. b. Given, volume of the metal sphere = 38808 cm3
According to the question,
1 2
7392 (ii) Volume of cone = πr h cu units But volume of sphere =
4 3
πr
66 × 28 × h = 7392 ⇒ h = = 4 cm 3 A B
66 × 28 R 3
(iii) Curved surface area of cone = πrl sq units ∴
4 3
πr = 38808
3 7
⇒ r 3 = 38808 × × = 9261
Hence, the height to which water rises in the tub is 4 cm. (iv) Total surface area of a cone = Curved surface area (i) Slant height of frustum of right circular cone 3 4 22
+ Base area = πr ( l + r ) sq units = BC = h 2 + ( R − r ) 2 units
Hollow Cylinder ⇒ r = (9261)1/ 3 = 21cm
EXAMPLE 13. How many metres of cloth 5 m wide (ii) Volume of frustum of right circular cone
A hollow cylinder is one which ∴ Surface area of the sphere = 4 πr 2
D R C πh 2
is empty from inside and has will be required to make a conical tent, the radius of = [ R + r 2 + Rr ] cu units 22
whose base is 7 m and whose height is 24 m? 3 =4× × 21 × 21
some difference between the 7
internal and external radius. a. 100 m b. 105 m c. 109 m d. 110 m (iii) Curved (lateral) surface area of frustum of right = 5544 cm2
h
Sol. d. Given radius of base ( r) = 7 m circular cone = πl ( R + r ) sq units
Let R and r be the external and
(iv) Total surface area of frustum of right circular cone EXAMPLE 17. How many balls each of radius 2 cm
internal radii of the hollow and vertical height of tent (h) = 24 m
A B = Area of base + Area of top + Lateral surface area can be made by melting a big ball whose radius is 8 cm.
cylinder and h be its height. r
Slant height of the tent ( l ) a. 4 balls b. 16 balls c. 64 balls d. 128 balls
Then, = π [ R2 + r 2 + l ( R + r ) ] sq units
(i) Volume of hollow cylinder = π ( R − r ) h cu units
2 2 = r 2 + h2 = 242 + 72 = 625 = 25 m Sol. c. Given radius of big ball (R) = 8 cm
(v) Total surface area of bucket = π [( R + r ) l + r ] sq unit
2

22 [Q it is open at the bigger end.] 4 4


(ii) Curved surface area Now, curved surface area = πrl = × 7 × 25 = 550 m2 ∴ Volume of big ball = πR 3 = π (8) 3
= 2 π Rh + 2 π rh = 2 π ( R + r ) h sq units 7 3 3
(iii) Total surface area Q Width of cloth = 5 m EXAMPLE 15. The radii of the ends of a bucket of Radius of small ball ( r) = 2 cm
height 24 cm are 15 cm and 5 cm. Then, its capacity is 4
= 2πrh + 2πRh + 2π ( R2 − r 2 ) sq units ∴ Length required to make conical tent ∴ Volume of small balls = π ( 2) 3
a. 8000 cm3 b. 8100 cm3 3
= 2 π ( R + r ) ( h + R − r ) sq units 550
= = 110 m c. 8171.43 cm3 d. 8200.43 cm3 Volume of big ball
5 ∴ Required number of balls =
EXAMPLE 12. A hollow cylindrical tube, open at Sol. c. Given, h = 24 cm, R = 15cm and r = 5 cm Volume of small ball
both ends is made of iron 1 cm thick. The volume of 4
EXAMPLE 14. The radius and the height of a right Capacity of bucket = Volume π (8) 3 8 × 8 × 8
iron used in making the tube, if the external diameter circular cone are in the ratio 5 : 12 . If its volume is of frustum of a cone D C = 3 = = 64 balls
π ( 2) 3 2 × 2 × 2
5 cm 4
is 12 cm and the length of tube is 70 cm, is 314 m 3 , the slant height and the radius are πh 2 3
a. 2420 cm 3
b. 2520 cm 3
a. 12 m, 5 m b. 13 m, 4 m c. 1 m, 4 m d. 13 m, 5 m = [R + r 2 + Rr ]
3 24 cm
c. 2720 cm3 d. 2900 cm3 EXAMPLE 18. A copper sphere of diameter 18 cm is
Sol. d. Let radius of cone be 5x and the height of the cone 22 24
= × [(15) 2 + 52 + 15 × 5] drawn into a wire of diameter 4 mm. Then, the length
Sol. a. Here, external radius (R 1 ) = 6 cm be 12x . 7 3 A B
15 cm
1 2 22 of the wire is
and internal radius (R2) = 6 − 1 = 5 cm ∴ Volume of cone = 314 = πr h = × 8 [ 225 + 25 + 75]
3 7 a. 243 m b. 343 m c. 443 m d. 972 m
Height of hollow cylinder (h) = 70 cm 1 176 Sol. a. Let the length of wire be h cm.
⇒ × 314
. × (5 x) 2 × (12 x) = 314 ⇒ 314x3 = 314 = ( 325) = 8171.43 cm3
∴ Volume of iron used in making tube 3 7
Volume of sphere = Volume of wire [by condition]
= External volume − Internal volume ⇒ x3 = 1 ⇒ x = 1  4 π × 9 × 9 × 9 =  π × 2 × 2 × h
Sphere ⇒
3   
= πR h − πr h = πh (R − r )
2 2 2 2 Q Radius = 5 m and height = 12 m 10 10
The set of all points in space, which are
22 h
= × 70 × ( 36 − 25) ∴ Slant height = r 2 + h2 = 25 + 144 = 169 = 13 m equidistant from a fixed point, is called a r ⇒ = 972 ⇒ h = (972 × 25) cm
O 25
7 sphere. Let radius of sphere be r, then 972 × 25
Hence, the slant height and the radius are 5 m and
4 = = 243 m
= 220 × 11= 2420 cm 3
13 m. (i) Volume of sphere = πr 3 cu units 100
3 Hence, the length of the wire is 243 m.
MATHEMATICS > Quadratic Equations and Inequalities 341 342 CDS Pathfinder

container is 32400 π cm 3 , then the height h of the


Hemisphere IMPORTANT FORMULAE 12. Two solid spheres of gold having diameters 3 cm
and 4 cm are molten and then cast into one big container satisfies which one of the following ?
A plane passing through the centre cuts the sphere in l
If the side of a cube is increased by x %, then its volume sphere of gold. If the radius of this sphere is x, (a) 135 cm < h < 150 cm (b) 140 cm < h < 147 cm
two equal parts, each part is called a hemisphere.
 3  then what is the value of x3 ? (c) 145 cm < h < 148 cm (d) 139 cm < h < 145 cm
is increased by  1 +
x 
Let the radius of hemisphere be r, then O r  − 1  × 100% . (a) 125 cm 3 (b) 15.625 cm 3
  100   21. A conical flask of base radius r and height h is
2 (c) 11.375 cm 3 (d) 9.875 cm 3
(i) Volume of hemisphere = πr 3 cu units l
If the length, breadth and height of cuboid are made x , y
full of milk. The milk is now poured into a
3 13. The total surface area of a cone, whose slant cylindrical flask of radius 2r. What is the height
and z times respectively, then its volume is increased by
(ii) Curved surface area of hemisphere = 2 πr 2 sq units ( xyz − 1) × 100% . height is equal to the radius R of its base, is S. If to which the milk will rise in the flask?
h h h h
A is the area of a circle of radius 2R, then which (a) (b) (c) (d)
(iii) Total surface area = 2πr 2 + πr 2 = 3 πr 2 sq units If the length, breadth and height of a cuboid are
l

one of the following is correct? 3 6 9 12


increased by x %, y % and z % respectively, then its
2 (a) A = S (b) A = 2S (c) A = S/ 2 (d) A = 4S 22. The radius and height of a right circular cone
(iv) Volume of hemispherical shell = π( R3 − r 3 ) volume is increased by
3  xy + yz + zx xyz  14. A cylinder having base of circumference 60 cm is are in the ratio 3 : 4 and its volume is 96 π cm 3 .
where, R = outer radius, r = inner radius x + y + z + +  % . What is the lateral surface area?
 100 (100) 2  rolling without sliding at a rate of 5 rounds per
second. How much distance will the cylinder roll (a) 24 π cm2 (b) 36 π cm2 (c) 48 π cm2 (d) 60 π cm2
l
If the sides and diagonal of a cuboid are given, then
EXAMPLE 19. The volumes of two hemispheres are in 5 s?
the total surface area in terms of diagonal and sides is 23. What is the number of wax balls, each of radius
in the ratio 8 : 27. What is the ratio of their radii? given by (a) 15 m (b) 1.5 m (c) 30 m (d) 3 m 1 cm, that can be molded out of a sphere of
a. 2 : 3 b. 3 : 2 c. 1 : 2 d. 2 : 1
Total surface area = (Sum of the sides) 2 − (Diagonal) 2 . 15. 27 drops of water form a big drop of water. If the radius 8 cm?
Sol. a. Let volumes be V1 and V2. l
If the side of a cube is increased by x % , then the surface radius of each smaller drop is 0.2 cm, then what (a) 256 (b) 512 (c) 768 (d) 1024
2 3 2 3 
∴ V1 : V2 = 8 : 27 ⇒ πr1 : πr2 = 8 : 27 x2  is the radius of the bigger drop? 24. 10 cylindrical pillars of a building have to be
area is increased by  2x +  %.
3 3
 100  (a) 0.4 cm (b) 0.6 cm (c) 0.8 cm (d) 1.0 cm painted. The diameter of each pillar is 70 cm and
⇒ r13 : r23 = 8 : 27 ⇒ r1 : r2 = 2 : 3 the height is 4 m. What is the cost of painting at
16. A circus tent is made of canvas and is in the
form of a right circular cylinder and a right the rate of ` 5 per m 2 ?
circular cone above it, the diameter and height (a) ` 400 (b) ` 440 (c) ` 480 (d) ` 500
of the cylindrical part of the tent are 126 m and 25. The radii of the circular ends of a bucket of

PRACTICE EXERCISE 5 m, respectively. The total height of the tent is


21 m. Then, the cost of the canvas used for tent
at the rate of ` 12 per m 2
height 40 cm are of lengths 35 cm and 14 cm.
What is the volume of the bucket?
(a) 60060 cm3 (b) 70040 cm3
(a) ` 14850 (b) ` 168200 (c) ` 178200 (d) ` 112000 (c) 80080 cm3 (d) 80160 cm3
1. Three equal cubes are placed adjacently in a 6. How many metres of cloth 50 m wide will be 17. A solid sphere of radius 6 cm is melted into a 26. If S is the total surface area of a cube and V is its
row. The ratio of total surface area of the new required to make a conical tent, the radius of hollow cylinder of uniform thickness. If the volume, then which one of the following is correct?
cuboid to that of the sum of the surface areas of whose base is 7 m and whose height is 24 m? external radius of the base of the cylinder is 5 cm (a) V 3 = 216 S 2 (b) S 3 = 216 V 2
the three cubes is (a) 9 m (b) 11 m (c) 12 m (d) 13 m and its height is 32 cm. The uniform thickness of
(c) S 3 = 6 V 2 (d) S 2 = 36 V 3
(a) 3 : 1 (b) 6 : 5 (c) 7 : 9 (d) 6 : 7
the cylinder is
7. The radius and height of a right circular
cone are in the ratio of 5 : 12 and its volume is
(a) 1.5 cm (b) 3 cm (c) 1.2 cm (d) 1 cm 27. A cylindrical tank 7 m in diameter, contains
2. If side of a cube is increased by 12%, by how much water to a depth of 4 m. What is the total area of
percent does its volume increase? 2512 cm3 . The slant height of the cone is 18. Given a solid cylinder of radius 10 cm and length
1000 cm a cylindrical hole is made into it to wetted surface?
(a) 24 cm (b) 25 cm (c) 26 cm (d) 27 cm
(a) 35.2% (b) 40.5% (c) 45.0% (d) 42.4% obtain a cylindrical shell of uniform thickness (a) 110.5 m2 (b) 126.5 m2
8. If the height of a cone is doubled, then its (c) 131.5 m2 (d) 136.5 m2
3. The curved surface area of a cylinder is 1320 cm 2 and having volume equal to one-fourth of the
volume is increased by original volume of the original cylinder. The
and its base has diameter 21 cm, then the height 28. The radii of two cylinders are in the ratio 2 : 3
(a) 100% (b) 200% (c) 300% (d) 400% thickness of the cylindrical shell is
of the cylinder is and their curved surface areas are in the ratio
9. If the surface areas of two spheres are in the (a) 5 ( 5 − 2 ) cm (b) 7 ( 2 − 3 ) cm 5 : 3. What is the ratio of their volumes?
(a) 10 cm (b) 20 cm (c) 22 cm (d) 25 cm
ratio of 4 : 25, then the ratio of their volumes is (c) 10 cm (d) 5 2 cm (a) 20 : 27 (b) 10 : 9 (c) 9 : 10 (d) 27 : 20
4. A cylindrical vessel can hold 154 g of water. If (a) 2 : 25 (b) 4 : 75 (c) 8 : 125 (d) 16 : 125
3 19. A tent is of the shape of right circular cylinder 29. A cylindrical vessel of height 10 cm has base
the radius of its base is 3.5 cm and 1 cm of
10. A cone and a cylinder are of the same height. upto a height of 3 m and then becomes a right radius 60 cm. If d is the diameter of a spherical
water weights 1 g. The depth of the water is
Their radii of the bases are in ratio of 2 : 1. The circular cone with a maximum height of 13.5 m vessel of equal volume, then what is the value
(a) 2 cm (b) 3 cm ratio of their volumes is above the ground. The cost of painting the inner of d ?
(c) 4 cm (d) 5 cm (a) 2 : 1 (b) 3 : 2 (c) 4 : 3 (d) 1 : 3 side of the tent at the rate of ` 2 per m 2, if the (a) 30 cm (b) 60 cm (c) 90 cm (d) 120 cm
5. The curved surface area of a cylindrical pillar is radius of the base is 14 m is
2 3
11. If the height and diameter of a right circular (a) ` 2048 (b) ` 2068 (c) ` 2008 (d) ` 2088
30. A hollow sphere of internal and external
264 m and its volume is 924 m . The diameter cylinder are 32 cm and 6 cm respectively, then diameters 4 cm and 8 cm, respectively is melted
of the pillar is the radius of the sphere whose volume is equal 20. A container is in the form of a right circular into a cone of base diameter 8 cm. The height of
(a) 3 m (b) 6 m to the volume of the cylinder is cylinder surmounted by a hemisphere of the same the cone is
(c) 7 m (d) 14 m (a) 3 cm (b) 4 cm (c) 6 cm (d) 8 cm radius 15 cm as the cylinder. If the volume of the (a) 11 cm (b) 12 cm (c) 14 cm (d) 16 cm
MATHEMATICS > Quadratic Equations and Inequalities 343 344 CDS Pathfinder

31. If the diameter of a sphere is doubled, how does 41. A measuring jar of internal diameter 10 cm is 50. Water is distributed to a town of 50000 58. From a solid cylinder of height 4 cm and radius
its surface area change? partially filled with water. Four equal spherical inhabitants from a rectangular reservoir consisting 3 cm, a conical cavity of height 4 cm and of base
(a) It increases two times (b) It increases three times balls of diameter 2 cm each are dropped in it of 3 equal compartments. Each compartment has radius 3 cm is hollowed out. What is the total
(c) It increases four times (d) It increases eight times and they sink down in the water completely. The length and breadth 200 m, 100 m, respectively surface area of the remaining solid?
32. A cistern 6 m long and 4 m wide contains water change in the level of water in the jar is and 12 m depth of water in the beginning. The (a) 15π cm2 (b) 22 π cm2 (c) 33π cm2 (d) 48π cm2
to a depth of 1.25 m. What is the area of wetted 16 15 16 allowance is 20 L per head per day. For how
(a) cm (b) cm (c) cm (d) None of these 59. The curved surface of a cylinder is 1000 cm 2. A
surface? 65 16 75 many days will the supply of water hold out?
wire of diameter 5 mm is wound around it, so as
(a) 40 m2 (b) 45 m2 (c) 49 m2 (d) 73 m2 42. The height of a cone is 30 cm. A small cone is cut (a) 240 days (b) 720 days (c) 800 days (d) 900 days
to cover it completely. What is the length of the
off at the top by a plane parallel to the base. If
33. The outer and inner diameters of a circular pipe 1 51. A right circular cylinder and a right circular wire used?
are 6 cm and 4 cm, respectively. If its length is its volume be of the volume of the given cone, cone have equal bases and equal volumes. But (a) 22 m (b) 20 m
27
10 cm, then what is the total surface area in cm 2? the lateral surface area of the right circular cone (c) 18 m (d) None of these
then the height above the base where the section is 15/8 times the lateral surface area of the right
(a) 35 π (b) 110 π is made, is circular cylinder. What is the ratio of radius to 60. In order to fix an electric pole along a roadside, a
(c) 150 π (d) None of these (a) 12 cm (b) 15 cm (c) 20 cm (d) 22 cm height of the cylinder? pit with dimensions 50 cm × 50 cm is dug with
34. A toy is in the form of a cone mounted on a 43. From a wooden cylindrical block, whose diameter the help of a spade. The pit is prepared by
(a) 3 : 4 (b) 9 : 4 (c) 15 : 8 (d) 8 : 15
hemisphere such that the diameter of the base of is equal to its height, a sphere of maximum removing Earth by 250 strokes of spade. If one
the cone is equal to that of the hemisphere. If possible volume is carved out. What is the ratio 52. The volume of a cuboid whose sides are in the stroke of spade removes 500 cm3 of Earth, then
the diameter of the base of the cone is 6 cm and of the volume of the utilised wood to that of the ratio of 1 : 2 : 4 is same as that of a cube. What what is the depth of the pit?
its height is 4 cm, what is the surface area in wasted wood? is the ratio of length of diagonal of cuboid to that (a) 2 m (b) 1 m (c) 0.75 m (d) 0.5 m
of cube?
cm 2 of the toy ? (Take π = 3.14) (a) 2 : 1 (b) 1 : 2 (c) 2 : 3 (d) 3 : 2
61. A figure is formed by revolving a rectangular
(a) 1.25 (b) 175
. (c) 2 (d) 3. 5
(a) 93.62 (b) 103.62 (c) 113.62 (d) 115.50 44. The base diameter of a right circular cylinder is sheet of dimensions 7 cm × 4 cm about its length.
35. A solid cylinder of height 9 m has its curved 3 cm. There is a section making an angle of 30° 53. A field is 125 m long and 15 m wide. A tank What is the volume of the figure thus formed?
surface area equal to one-third of the total with the cross section. What is its area? 10 m × 7.5 m × 6 m was dug in it and the Earth, (a) 352 cm3 (b) 296 cm3
surface area. What is the radius of the base? 9π 3 3π 9π 9 3π thus dug out was spread equally on the (c) 176 cm3 (d) 616 cm3
(a) cm2 (b) cm2 (c) cm2 (d) cm2
4 2 8 8 remaining field. The level of the field thus raised
(a) 9 m (b) 18 m (c) 27 m (d) 30 m 62. The diagonals of the three faces of a cuboid are
45. A cone is inscribed in a hemisphere such that is equal to which one of the following?
36. The volume of a sphere is 8 times that of another (a) 15 cm (b) 20 cm (c) 25 cm (d) 30 cm x , y , z , respectively. What is the volume of the
their bases are common. If C is the volume of the
sphere. What is the ratio of their surface areas? cuboid?
cone and H that of the hemisphere, then what is 54. If C1 is a right circular cone with base radius r1 cm
(a) 8 : 1 (b) 4 : 1 (c) 2 : 1 (d) 4 : 3 x yz
the value of C : H ? (a)
and height h1 cm and C2 is a right circular 2 2
37. The dimensions of a field are 12 m × 10 m. A pit (a) 1 : 2 (b) 2 : 3 (c) 3 : 4 (d) 4 : 5
cylinder with base radius r2 cm and height h2 cm
( y2 + z2 ) ( z2 + x2 ) (x2 + y2 )
5 m long, 4 m wide and 2 m deep is dug in one 46. If the diameter of a wire is decreased by 10%, by and if r1 : r2 = 1 : n (where n is a positive integer) (b)
corner of the field and the Earth removed has how much per cent (approximately) will the and their volumes are equal, then which one of 2 2
been evenly spread over the remaining area of length be increased to keep the volume constant? the following is correct? ( y2 + z2 − x2 ) ( z2 + x2 − y2 ) (x2 + y2 − z2 )
the field. The level of the field is raised by (c)
(a) 5% (b) 17% (c) 20% (d) 23% (a) h1 = 3nh2 (b) h1 = 3n2 h2 2 2
(a) 30 cm (b) 35 cm (c) 38 cm (d) 40 cm 47. The diameter of a solid metallic right circular (d) None of the above
(c) h1 = 3h2 (d) h1 = n2 h2
38. A cube of 9 cm edge is immersed completely in a cylinder is equal to its height. After cutting out 63. The volume of a cone is equal to that of a sphere.
rectangular vessel containing water. If the the largest possible solid sphere S from this 55. A right circular cone is cut by a plane parallel to If the diameter of base of cone is equal to the
dimensions of base are 15 cm and 12 cm. Then, cylinder, the remaining material is recast to its base in such a way that the slant heights of diameter of the sphere, what is the ratio of
the rise in water level in the vessel is form a solid sphere S1. What is the ratio of the the original and the smaller cone thus obtained height of cone to the diameter of the sphere?
radius of sphere S to that of sphere S1? are in the ratio 2 : 1. If V1 and V 2 are respectively
(a) 4.05 cm (b) 4 cm (c) 3.5 cm (d) 3 cm (a) 2 : 1 (b) 1 : 2 (c) 3 : 1 (d) 4 : 1
1 1 1 1 1 1 the volumes of the original cone and of the new
39. From a solid cube of edge 3 m, a solid of largest (a) 1 : 2 3 (b) 2 3 : 1 (c) 2 3 : 3 3 (d) 3 2 : 2 2 cone, then what is V1 : V 2? 64. The length, breadth and height of a rectangular
sphere is curved out. What is the volume of solid 48. A square has its side equal to the radius of a (a) 2 : 1 (b) 3 : 1 (c) 4 : 1 (d) 8 : 1
parallelopiped are in ratio 6 : 3 : 1. If the surface
left? sphere. The square revolves round a side to area of a cube is equal to the surface area of this
2
(a) (27 − 2.25 π ) m 3 (b) (27 − 4.5 π ) m 3 generate a surface of total area S. If A is the 56. The surface area of a sphere is 616 cm . If its parallelopiped, then what is the ratio of the
surface area of the sphere, then which one of the radius is changed so that the area gets reduced volume of the cube to the volume of the
(c) 2.25 π m 3 (d) 4.5π m 3 by 75%, then the radius becomes
following is correct? parallelopiped?
40. A rectangular tank is 80 × 40 cm . Water flows
3
(a) A = 3S (b) A = 2S (c) A = S (d) A < S (a) 1.6 cm (b) 2.3 cm (c) 2.5 cm (d) 3.5 cm (a) 1 : 1 (b) 5 : 4 (c) 7 : 5 (d) 3 : 2
into it through a pipe of cross-section area 49. A swimming pool is 24 m long and 15 m broad. 57. A sphere is inscribed in a cubical box such that
40 cm 2 at the speed of 10 km/h. The rise in the
65. A hollow hemisphere is made of a sheet of a
When x number of men dive into the pool, the the sphere is tangent to all six faces of the box. metal 1 cm thick. If the outer radius is 5 cm.
1 height of the water rises by 1 cm. If the average What is the ratio of the volume of the cubical box
level of water in the tank in h is What is the weight of the hemisphere (1 cm3 of
2 amount of water displaced by one man is 0.1 m3 , to the volume of sphere? the metal weight 9 g)?
3 4 5 then what is the value of x? 4π 6
(a) cm (b) cm (c) cm (d) 6 cm (a) 6π (b) 36π (c) (d) (a) 54 π g (b) 366 π g (c) 122 π g (d) 108 π g
2 3 8 (a) 36 (b) 72 (c) 108 (d) 360 3 π
MATHEMATICS > Quadratic Equations and Inequalities 345 346 CDS Pathfinder

II. Square of volume of a spherical body is directly


66. Half of a large cylindrical tank open at the top is proportional to cube of its surface area.
79. What is the total surface area of the given frustum? 89. What is the volume of the frustum? e 2012 II
filled with water and identical heavy spherical (a) 9 π (2 10 + 5) cm2 (b) 9 π (3 10 + 5) cm2 (a) 3H(P + Q + PQ ) (b) H(P + Q + PQ )
III. A sphere occupies the biggest space but has the
balls are to be dropped into the tank without (c) H(P + Q + PQ ) / 3 (d) H(P + Q − PQ ) / 3
smallest surface area. (c) 9 π (3 10 + 4) cm 2
(d) 27 π ( 10 + 1) cm 2
spilling water out. If the radius and the height of
the tank are equal and each is four times the Which of the above statement(s) is/are correct? 90. Let the largest possible right circular cone and
radius of a ball, what is the maximum number of (a) I and II (b) I and III (c) II and III (d) All of these PREVIOUS YEARS’ QUESTIONS largest possible sphere be fitted into two cubes of
balls that can be dropped? same length. If C and S denote the volume of
73. Consider the following statements: 80. What will be the cost to plaster the inner surface cone and volume of sphere, respectively. Then,
(a) 12 (b) 24 (c) 36 (d) 48 I. The curved surface area of a right circular cone of a well 14 m deep and 4 m in diameter at the which one of the following is correct? e 2012 II
67. A cylinder is circumscribed about a hemisphere of base radius r and height h is given by rate of ` 25 per m 2? e 2012 I
(a) C = 2 S (b) S = 2 C (c) C = S (d) C = 3S
and a cone is inscribed in the cylinder so as to πr ( h + r ).
2 2 (a) ` 4000 (b) ` 4200 (c) ` 4400 (d) ` 5400
have its vertex at the centre of one end, and the 91. 10 circular plates each of thickness 3 cm, are
II. The right circular cone of base radius r and 81. What is the length of the uniform wire of placed one above the other and a hemisphere of
other end as its base. The volume of the cylinder, height h when cut opened along the slant height, diameter 0.4 cm that can be drawn from a solid
hemisphere and the cone are respectively in the radius 6 cm is placed on the top just to cover the
forms a rectangle of length πr and breadth sphere of radius 9 cm? e 2012 I
ratio cylindrical solid. What is the volume of the solid
h2 + r2 . (a) 243 m (b) 240 m (c) 60.75 m (d) 60 m so formed? e 2012 II
(a) 2 : 3 : 2 (b) 3 : 2 : 1 (c) 3 : 1 : 2 (d) 1 : 2 : 3
III. If a right circular cone is cut off by a plane 82. The total surface area of a cube is 150 cm 2. What (a) 264 π cm 3 (b) 252π cm 3
68. A conical cavity is drilled in a circular cylinder of parallel to the base of the cone, then its curved is its volume? e 2012 I (c) 236 π cm 3 (d) None of these
height 15 cm and base radius 8 cm. The height πh
surface area is [R2 + r 2 + Rr ]. (a) 64 cm3 (b) 81 cm3 (c) 125 cm3 (d) 160 cm3 92. A right circular metal cone (solid) is 8 cm high and
and the base radius of the cone are also same. 3
Then the whole surface area of the remaining IV. If a right triangle is being revolved about one of 83. If the volume of a cube is 729 cm3 , then what is the radius is 2 cm. It is melted and recast into a
solid is its edges. The surface generated by the hypotenuse the length of its diagonal? e 2012 I sphere. What is the radius of the sphere? e 2012 II
(a) 440 π cm 2 (b) 240 π cm 2 (c) 640 π cm 2 (d) 960 π cm 2 is the lateral surface of the right circular cone. (a) 9 2 cm (b) 9 3 cm (c) 18 cm (d) 18 3 cm (a) 2 cm (b) 3 cm (c) 4 cm (d) 5 cm

69. Smaller lead shots are to be prepared by Which of the above statement(s) are correct? 93. The volume of a cube is numerically equal to
(a) I and II (b) I, II and III (c) I, III and IV (d) All of these
84. The curved surface area of a right circular cone
using the material of a spherical lead shot of of radius 14 cm is 440 cm 2. What is the slant sum of its edges. What is the total surface area?
radius 1 cm. Same possibilities are listed in the height of the cone? (a) 12 sq units (b) 36 sq units e 2012 II
Directions (Q. Nos. 74-76) Read the following e 2012 I
statements given below (c) 72 sq units (d) 144 sq units
information carefully and answer the given (a) 10 cm (b) 11cm (c) 12 cm (d) 13 cm
I. The material is just sufficient to prepare 8 shots
questions that follow. 85. A large solid metallic cylinder whose radius and 94. The diameter of base of a right circular cone is
each of radius 0.5 cm.
A cone of height 10 cm and radius 5 cm is divided into height are equal to each other is to be melted and 7 cm and slant height is 10 cm, then what is its
II. A shot of radius 0.75 cm and a second shot of two parts by drawing a plane through the mid-point of its 48 identical solid balls are to be recast from the lateral surface area? e 2012 II
radius 0.8 cm can be prepared from the available axis, parallel to its base.
material. liquid metal, so formed. What is the ratio of the (a) 110 cm 2 (b) 100 cm 2 (c) 70 cm 2 (d) 49 cm 2
Which of the above statement is/are correct? 74. What is the ratio of the volume of the original radius of a ball to the radius of the cylinder? 95. What is the height of a solid cylinder of radius
(a) Only I (b) Only II cone to the volume of the frustum left? (a) 1 : 16 (b) 1 : 12 e 2012 I 5 cm and total surface area is 660 cm 2? e 2012 II
(c) Both I and II (d) Neither I nor II 4 7 8 9 (c) 1 : 8 (d) 1 : 4 (a) 10 cm (b) 12 cm (c) 15 cm (d) 16 cm
(a) (b) (c) (d)
3 8 7 8 86. What are the dimensions (length, breadth and
70. Consider the following statements: 96. If the ratio of the diameters of two spheres is
I. The volume of a cuboid is the product of the 75. What is the area of the top circle of the frustum height, respectively) of a cuboid with volume
3 : 5, then what is the ratio of their surface areas?
lengths of its coterminous edges. in square centimetre? 720 cm3 , surface area 484 cm 2 and the area of
e 2012 II
5π 25 π the base 72 cm 2? e 2012 I
II. The surface area of a cuboid is twice the sum of (a) 25π (b) (c) (d) None of these (a) 9 : 25 (b) 9 : 10 (c) 3 : 5 (d) 27 : 125
the products of lengths of its coterminous edges 4 4 (a) 9, 8 and 10 cm (b) 12, 6 and 10 cm
taken two at a time. 76. What is the ratio of the curved surface area of (c) 18, 4 and 10 cm (d) 30, 2 and 12 cm 97. What is the volume of the largest sphere that can
87. If the surface area of a sphere is 616 cm 2, then be curved out of a cube of edge 3 cm? e 2012 II
Which of the above statement(s) is/are correct? the original cone and the curved surface area of
the frustum? what is its volume? e 2012 I (a) 9π cm 3 (b) 6π cm 3 (c) 4.5 π cm 3 (d) 3π cm 3
(a) Only I (b) Only II
(c) Both I and II (d) Neither I nor II (a) 3 : 1 (b) 3 : 2 (c) 4 : 1 (d) 4: 3 (a) 4312/3 cm 3 (b) 4102/3 cm3 98. What is the quantity of cloth required to roll
71. Consider the following statements : (c) 1257 cm 3 (d) 1023 cm3 up to form a right circular tent whose base is of
The length of a side of a cube is 1 cm. Which of Directions (Q. Nos. 77-79) Read the following radius 12 m and height 5 m? e 2013 I
information carefully and answer the given Directions (Q.Nos. 88-89) Read the following
the following can be the distance between any (a) 40π m 2 (b) 60π m 2 (c) 78π m 2 (d) 156π m 2
questions that follow. information carefully and answer the given
two vertices?
Let C be a right circular cone. It is given that the two questions that follow. 99. The volume of the material of a hemispherical
I. 1 cm II. 2 cm III. 3 cm
Select the correct answer using the code given ends of a frustum of C are of radii 3 cm and 6 cm and the The areas of the ends of a frustum of a cone are P and Q, shell with outer and inner radii 9 cm and 7 cm,
below. height of the frustum is 9 cm. where P < Q and H is its thickness. respectively is approximately e 2013 I
(a) Only I (b) Only II (c) Only III (d) All of these 88. What is the difference in radii of the ends of the (a) 808 cm 3 (b) 800 cm 3 (c) 816 cm 3 (d) 824 cm 3
77. What is the slant height of the given frustum?
72. Consider the following statements. frustum? e 2012 II
(a) 3 10 cm (b) 6 10 cm (c) 12 cm (d) 15 cm 100. The ratio of surface area to diameter of a sphere
Q− P Q− P
I. When eight drops of water combine to form a (a) (b) whose volume is 36π cm3 , is e 2013 I
single drop, the surface area of all the eight drops 78. What is the height of the cone? π π
(a) 3π (b) 6π (c) 6 (d) None of these
is greater than the surface area of big drop. (a) 9 cm (b) 12 cm (c) 13.5 cm (d) 18 cm (c) Q − P (d) None of these
MATHEMATICS > Quadratic Equations and Inequalities 347 348 CDS Pathfinder

101. A cylindrical tube open at both ends is made of 111. If x is the curved surface area and y is the 119. What is the diameter of the largest circle lying on 129. Consider the following statements in respect of
metal. The internal diameter of the tube is 6 cm the surface of a sphere of surface area 616 cm 2? four spheres A, B, C and D having respective
volume of a right circular cylinder, then which
e 2014 I
and length of the tube is 10 cm. If the thickness one of the following is correct? e 2014 I radii 6, 8, 10 and 12 cm.
(a) 14 cm (b) 10.5 cm (c) 7 cm (d) 3.5 cm
of the metal used is 1 cm, then the outer curved (a) Only the ratio of the height to radius of the cylinder I. The surface area of sphere C is equal to the sum
surface area of the tube is e 2013 I is independent of x 120. The volume of a hollow cube is 216x3 . What is of surface areas of spheres A and B.
(a) 140π cm 2 (b) 146.5 π cm 2 (b) Only the ratio of height to radius of the cylinder is the surface area of the largest sphere which can
(c) 70π cm 2 independent of y II. The volume of sphere D is equal to the sum of
(d) None of these be enclosed in it? e 2014 I
(c) Either (a) or (b) (a) 18 π x2 (b) 27 π x2 (c) 36 π x2 (d) 72 π x2 volumes of spheres A, B and C. e 2014 II
102. From a solid wooden right circular cylinder, a (d) Neither (a) nor (b) Which of the above statement(s) is/are correct?
right circular cone whose radius and height are
112. A cylinder is surmounted by a cone at one end, a (a) Only I (b) Only II
same as the radius and height of the cylinder, Directions (Q. Nos. 121-122) Read the following
hemisphere at the other end. The common radius information carefully and answer the given (c) Both I and II (d) Neither I nor II
respectively is carved out. What is the ratio of
the volume of the utilised wood to that of the is 3.5 cm, the height of the cylinder is 6.5 cm questions that follow. 130. The diameter of a metallic sphere is 6 cm. The
wasted wood? e 2013 I and the total height of the structure is 12.8 cm. sphere is melted and drawn into a wire of uniform
A right angled triangle having hypotenuse 25 cm and
(a) 1 : 2 (b) 2 : 1 (c) 2 : 3 (d) 1 : 3 The volume V of the structure lies between legs in the ratio 3 : 4 is made to revolve about its circular cross-section. If the length of the wire is
e 2014 I
103. If the heights and the areas of the base of a right hypotenuse. (π = 3.14) 36 m, then what is its radius? e 2014 II
(a) 370 cm 3 and 380 cm 3 (b) 380 cm 3 and 390 cm 3
circular cone and a pyramid with square base (a) 0.1 cm (b) 0.01 cm (c) 0.001 cm (d) 1.0 cm
(c) 390 cm 3 and 400 cm 3 (d) None of these 121. What is the volume of the double cone so
are the same, then they have e 2013 I
113. The diameter of the base of a cone is 6 cm and formed? e 2014 II 131. What is the maximum distance between two
(a) same volume and same surface area
its altitude is 4 cm. What is the approximate (a) 3124 cm3 (b) 3424 cm3 (c) 3768 cm3 (d) 3924 cm3
points of a cube of side 2 cm? e 2014 II
(b) same surface area but different volumes
curved surface area of the cone? e 2014 I (a) 3 cm (b) 2 3 cm (c) 4 3 cm (d) 2 2 cm
(c) same volume but different surface areas 122. What is the surface area of the double cone so
2 2 2 2
(d) different volumes and different surface areas (a) 45 cm (b) 47 cm (c) 49 cm (d) 51 cm 132. The total outer surface area of a right circular
formed? e 2014 II
114. A drainage tile is a cylindrical shell 21 cm long. cone of height 24 cm with a hemisphere of radius
104. The volume of a right circular cone of height (a) 1101.2 cm2 (b) 11114
. cm2 (c) 1310.4 cm2 (d) 1318.8 cm2
The inside and outside diameters are 4.5 cm and 7 cm upon its base is e 2015 I
3 cm and slant height 5 cm is e 2013 I
5.1 cm, respectively. What is the volume of the 123. If the side of a cube is increased by 100%, then (a) 327 π cm 2 (b) 307 π cm 2
(a) 49.3 cm 3 (b) 50.3 cm 3
clay required for the tile? e 2014 I by what percentage is the surface area of the (c) 293 π cm 2 (d) 273 π cm 2
(c) 52 cm 3 (d) 53 cm 3
(a) 6.96 π cm 3
(b) 6.76π cm 3
cube increased? e 2014 II
105. A bucket is of a height 25 cm. Its top and bottom (c) 5.76 π cm 3 (d) None of these
(a) 150% (b) 200% (c) 300% (d) 400%
133. A sphere and a cube have same surface area.
radii are 20 cm and 10 cm, respectively. Its The ratio of square of their volumes is e 2015 I
115. A cube has each edge 2 cm and a cuboid is 1 cm 124. A cylinder circumscribes a sphere. What is the
capacity (in L) is e 2013 I (a) 6 : π (b) 5 : π (c) 3 : 5 (d) 1 : 1
long, 2 cm wide and 3 cm high. The paint in a ratio of volume of the sphere to that of the
(a) 17.5 π 3 (b) 17.5 π (c) 20 π (d) 25 π
certain container is sufficient to paint an area cylinder? e 2014 II
134. The radius of a sphere is equal to the radius of
106. The height of a cylinder is 15 cm. The lateral equal to 54 cm 2. e 2014 I the base of a right circular cone, and the volume
surface area is 660 cm 2. Its volume is e 2013 II
(a) 2 : 3 (b) 1 : 2 (c) 3 : 4 (d) 3 : 2
Which one of the following is correct? of the sphere is double the volume of the cone.
(a) 1155 cm 3 (b) 1215 cm 3 (c) 1230 cm 3 (d) 2310 cm 3 (a) Both cube and cuboid can be painted The ratio of the height of the cone to the radius
Directions (Q. Nos. 125-126) Read the following
(b) Only cube can be painted of its base is e 2015 I
107. The diameter of the Moon is approximately information carefully and answer the given
one-fourth of the diameter of the Earth. What is (c) Only cuboid can be painted questions that follow. (a) 2 : 1 (b) 1 : 2 (c) 2 : 3 (d) 3 : 2
the ratio (approximate) of their volumes? e 2013 II (d) Neither cube nor cuboid can be painted A toy is in the form of a cone mounted on the 135. If the radius of a sphere is increased by 10%,
(a) 1 : 16 (b) 1 : 64 (c) 1 : 4 (d) 1 : 128 hemisphere with the same radius. The diameter of the then the volume will be increased by e 2015 I
116. A cone of radius r cm and height h cm is divided
108. A conical cap has the base diameter 24 cm and into two parts by drawing a plane through the base of the conical portion is 12 cm and its height is 8 cm. (a) 33.1% (b) 30% (c) 50% (d) 10%
height 16 cm. What is the cost of painting the middle point of its height and parallel to the base. 125. What is the total surface area of the toy? e 2014 II 136. If three metallic spheres of radii 6 cm, 8 cm and
surface of the cap at the rate of 70 paise What is the ratio of the volume of the original (a) 132 π cm2 (b) 112 π cm2 (c) 96 π cm2 (d) 66 π cm2 10 cm are melted to form a single sphere, then
per cm 2? e 2013 II cone to the volume of the smaller cone? e 2014 I the diameter of the new sphere will be e 2015 I
(a) ` 520 (b) ` 524 (c) ` 528 (d) ` 532 (a) 4 : 1 (b) 8 : 1 (c) 2 : 1 (d) 6 : 1 126. What is the volume of the toy? e 2014 II (a) 12 cm (b) 24 cm (c) 30 cm (d) 36 cm
109. What is the whole surface area of a cone of base 117. If 64 identical small spheres are made out of big (a) 180 π cm3 (b) 240 π cm3 (c) 300 π cm3 (d) 320 π cm3
sphere of diameter 8 cm, then what is surface 137. If the height of a right circular cone is increased
radius 7 cm and height 24 cm? e 2013 II
area of each small sphere? e 2014 I 127. The areas of the three adjacent faces of a by 200% and the radius of the base is reduced by
(a) 654 cm 2 (b) 704 cm 2 (c) 724 cm 2 (d) 964 cm 2 50%, then the volume of the cone e 2015 I
(a) π cm 2 (b) 2 π cm 2 (c) 4π cm 2 (d) 8π cm 2 cuboidal box are x, 4x and 9x sq unit. What is the
110. A tent is in the form of a right circular cylinder volume of the box? e 2014 II (a) remains unaltered (b) decreases by 25%
surmounted by a cone. The diameter of the 118. The dimensions of a field are 15 m by 12 m. A (a) 6x2 cu units (b) 6x3 / 2 cu units (c) increases by 25% (d) increases by 50%
cylinder is 24 m. The height of the cylindrical pit 8 m long, 2.5 m wide and 2 m deep is dug in (c) 3x3 / 2 cu units (d) 2 x3 / 2 cu units
portion is 11 m, while the vertex of the cone is one corner of the field and the earth removed is 138. A rectangular paper of 44 cm long and 6 cm wide
16 m above the ground. What is the area of the evenly spread over the remaining area of the 128. What is the number of pairs of perpendicular is rolled to form a cylinder of height equal to
curved surface for conical portion? e 2013 II field. The level of the field is raised by e 2014 I planes in a cuboid? e 2014 II width of the paper. The radius of the base of the
(a) 3434/9 m 2
(b) 3431/8 m 2 (a) 4 (b) 8 cylinder so rolled is e 2015 I
200
(a) 15 cm (b) 20 cm (c) 25 cm (d) cm (c) 12 (d) None of these (a) 3.5 cm (b) 5 cm (c) 7 cm (d) 14 cm
(c) 3432/7 m 2 (d) 3234/7 m 2 9
MATHEMATICS > Quadratic Equations and Inequalities 349 350 CDS Pathfinder

139. The diagonals of three faces of a cuboid are 13, 149. 30 metallic cylinders of same size are melted and ANSWERS
281 and 20 linear units. Then, the total surface cast in the form of cones having the same radius
and height as those of the cylinders. 1 c 2 b 3 b 4 c 5 d 6 b 7 c 8 a 9 c 10 c
area of the cuboid is e 2015 I
(a) 650 sq units (b) 658 sq units Consider the following statements 11 c 12 c 13 b 14 a 15 b 16 c 17 d 18 a 19 b 20 a
(c) 664 sq units (d) 672 sq units I. A maximum of 90 cones will be obtained. 21 d 22 d 23 b 24 b 25 c 26 b 27 b 28 b 29 b 30 c
140. A cylindrical vessel of radius 4 cm contains II. The curved surface of the cylinder can be 31 c 32 c 33 b 34 b 35 b 36 b 37 d 38 a 39 b 40 c
water. A solid sphere of radius 3 cm is lowered flattened in the shape of a rectangle but the 41 c 42 c 43 a 44 b 45 a 46 d 47 b 48 c 49 a 50 b
into the water until it is completely immersed. curved surface of the cone when flattened has
the shape of triangle. 51 b 52 b 53 c 54 b 55 d 56 d 57 d 58 d 59 b 60 d
The water level in the vessel will rise by e 2015 I
(a) 1.5 cm (b) 2 cm (c) 2.25 cm (d) 4.5 cm Which one of the following statement(s) is/are 61 a 62 c 63 a 64 d 65 b 66 b 67 b 68 a 69 c 70 c
141. A rectangular block of wood having dimensions correct in respect of the above? e 2015 II 71 d 72 d 73 c 74 c 75 c 76 d 77 a 78 d 79 b 80 c
3m × 2m × 1.75 m has to be painted on all its (a) Both statement I and statement II are correct and 81 a 82 c 83 b 84 a 85 d 86 a 87 a 88 a 89 c 90 b
faces. The layer of paint must be 01
. mm thick. statement II is the correct explanation of statement I
91 d 92 a 93 c 94 a 95 d 96 a 97 c 98 d 99 a 100 b
Paint comes in cubical boxes having their edges (b) Both statement I and statement II are correct and
statement II is not the correct explanation of 101 d 102 a 103 c 104 b 105 a 106 d 107 b 108 c 109 b 110 c
equal to 10 cm. The minimum number of boxes
of paint to be purchased is e 2015 I
statement I 111 d 112 a 113 b 114 d 115 a 116 b 117 c 118 c 119 a 120 c
(a) 5 (b) 4 (c) 3 (d) 2 (c) statement I is correct but statement II is not correct 121 c 122 d 123 c 124 a 125 a 126 b 127 b 128 c 129 c 130 a
2 (d) statement I is not correct but statement II is correct
142. If the surface area of a cube is 13254 cm , then 131 b 132 d 133 a 134 a 135 a 136 b 137 b 138 c 139 c 140 c
the length of its diagonal is e 2015 II 150. Water is filled in a container in such a manner 141 c 142 d 143 b 144 a 145 b 146 a 147 a 148 a 149 b 150 c
(a) 44 2 cm (b) 44 3 cm (c) 47 2 cm (d) 47 3 cm that its volume doubles every 5 min. If it takes
151 c 152 c 153 d 154 c
143. A water tank, open at the top, is hemispherical 30 min for the container to be full, in how much
at the bottom and cylindrical above it. The time will it be one-fourth full ? e 2015 II

radius is 12 m and the capacity is 3312 π m3 . (a) 7.5 min (b) 15 min (c) 20 min (d) 17.5 min
The ratio of the surface areas of the spherical
and cylindrical portions, is e 2015 II
151. Consider the following statements:
I. If the height of a cylinder is doubled, the area of
HINTS AND SOLUTIONS
(a) 3 : 5 (b) 4 : 5 (c) 1 : 1 (d) 6 : 5 the curved surface is doubled. 1. (c) Let ‘a’ be the edge of three equal 3. (b) Given, diameter of the base of the 6. (b) Given, r = 7 m, h = 24 m
144. A large water tank has the shape of a cube. If II. If the radius of a hemispherical solid is doubled, cubes. Surface area of a single cube cylinder = 21 cm Slant height of cone,
3
128 m of water is pumped out, the water level its total surface area becomes fourfold. = 6 (Side)2 = 6 a 2 21
Which of the statement(s) given above is/are ∴ Radius = cm = r 2 + h2 = 242 + 72
goes down by 2m. Then, the maximum capacity ∴ Sum of the surface areas of the three 2
of the tank is e 2015 II correct? e 2015 II cubes is = 3 ( 6a 2 ) = 18a 2 = 576 + 49 = 625 = 25 m
As, curved surface area = 2 πrh = 1320
(a) Only I (b) Only II Length of resulting cuboid = 3a
(a) 512 m 3 (b) 480 m 3 (c) 324 m 3 (d) 256 m 3 22 21 Curved surface area = πrl
(c) Both I and II (d) Neither I nor II Breadth of resulting cuboid = a ⇒ 2× × × h = 1320
22
145. The areas of three mutually perpendicular faces Height of resulting cuboid = a 7 2 = × 7 × 25 = 550 m 2
of a cuboid are x, y, z. If V is the volume, then 152. From the solid gold in the form of a cube of side ∴ Total surface area of cuboid 1320 7
∴ h= = 20 cm
xyz is equal to e 2015 II length 1 cm, spherical solid balls each having = 2 ( lb + bh + hl ) 22 × 3 Width of cloth = 50 m
(a) V (b) V 2 (c) 2V (d) 2 V 2 the surface area π1/ 3 cm 2 are to be made. = 2 (3a 2 + a 2 + 3a 2 ) = 14a 2 550
Assuming that there is no loss of the material in 4. (c) 1 cm 3 of water weighs 1 g. Length required = = 11 m
146. How many right angled triangles can be formed 14a 2 7 50
process of making the balls, the maximum Required ratio = = , i.e. 7 : 9 QVolume of 154 g of water = 154 cm 3
by joining the vertices of a cuboid? e 2015 II 18a 2 9 7. (c) Let radius and height of a
(a) 24 (b) 28 (c) 32 (d) None of these
number of balls made will be e 2015 II 2. (b) Here, x = 12, ∴ Volume of cylinder = πr 2 h = 154 cm 3
right circular cone be 5x and 12x
(a) 3 (b) 4 (c) 6 (d) 9 According to the formula, percentage 22
147. Three rectangles R1, R2 and R3 have the same ⇒ (3.5)2 × h = 154 respectively.
153. A hollow cylindrical drum has internal diameter increase in volume 7
area. Their lengths x1, x2 and x3 respectively are  3
 ∴ r = 5x and h = 12x
=   1 +
x  77h
such that x1 < x2 < x3 . If V1, V 2 and V3 are the of 30 cm and a height of 1 m. What is the  − 1 × 100% ⇒ = 154
 100  1 2
maximum number of cylindrical boxes of  2 Volume of cone = πr h = 2512
volumes of the cylinders formed from the 3
diameter 10 cm and height 10 cm each that can  3
 154 × 2
⇒ h= = 4 cm
=   1 +
rectangles R1, R2 and R3 respectively by joining 12 
be packed in the drum?  − 1 × 100% 77 ⇒
1
× 3.14 × (5x)2 × 12x = 2512
the parallel sides along the breadth, then which e 2015 II
  100   3
one of the following is correct? e 2015 II (a) 60 (b) 70 (c) 80 (d) 90 5. (d) Here, 2 πrh = 264 m 2
 3
 2512
=   1 +
3 ⇒ x3 = =8
(a) V3 < V2 < V1 (b) V1 < V3 < V2 154. A pipe with square cross-section is supplying  − 1 × 100% and πr 2 h = 924 m 3 [given]
 25  314
(c) V1 < V2 < V3 (d) V3 < V1 < V2 water to a cistern which was initially empty. The 
πr 2 h 924 ⇒ x=2
area of cross-section is 4 cm 2 and the nozzle   28  3  ∴ =
148. Let V be the volume of an inverted cone with vertex =    − 1 × 100% 2 πrh 264
velocity of water is 40m/s. The dimensions of the   ∴ Radius = 10 cm, height = 24 cm
at origin and the axis of the cone is along positive  25  r 7
Y-axis. The cone is filled with water up to half of cistern 10 m × 8m × 6m. Then, the cistern will be ⇒ = ⇒r = 7m ∴ Slant height,
=
21952  2 2
full in − 1 × 100% =[ 1.405 − 1]× 100%
its height. The volume of water is e 2015 II e 2015 II  15625  l = r 2 + h2 = 102 + 242
V V V V (a) 9.5 h (b) 9 h ∴ Diameter of pillar
(a) (b) (c) (d) = 0.405 × 100% = 40.5%
8 6 3 2 (c) 8 h and 20 min (d) 8 h = r × 2 = 7 × 2 = 14 m = 676 = 26 cm
MATHEMATICS > Quadratic Equations and Inequalities 351 352 CDS Pathfinder

8. (a) Let r and H be the radius and height 15. (b) Let the radius of big drop and small 18. (c) Volume of original cylinder On dividing both sides by 225 π, we get 25. (c) Given, Outer radius of hollow sphere
of the cone respectively. drop be R and r respectively. = [ π × ( 10)2 × 1000] cm 3 ⇒ 10 + H = 144 ⇒ H = 134 cm R = 35 cm , r = 14 cm and h = 40 cm 8
( R) = = 4 cm
1 By given condition, Volume of shell ∴ Volume of the bucket 2
∴ Original volume = πr 2 h = V ∴ Height of container
1 πh 2 Volume of metal of the sphere
3 27 × Volume of smaller drops = Volume = × π × 100000 = 25000 π cm 3 = 15 + 134 = 149 cm = ( R + r 2 + Rr )
New radius = r and new height = 2h of bigger drop 4 3 4 4 4
= πR 3 − πr 3 = π ( 4 3 − 2 3 )
1 4 4 Let r be the radius of shell, then 21. (d) Since, milk is in a conical flask whose 22 × 40 3 3 3
New volume = πr 2 × 2h = 2V ∴ 27 × πr 3 = πR 3 = (352 + 142 + 35 × 14)
3 3 3 πr 2 h = 25000 π base radius and height are r and h , 7×3 4
25000 π respectively. = π × 56 cm 3
∴ Increase in volume = 2V − V = V ⇒ r2 = = 25
⇒ 27 × (0.2) = R 3 [Q r = 0.2 cm ]
3
1
=
880
( 1225 + 196 + 490)
3
Increase percentage of volume π × 1000 ∴ Volume of milk = πr 2 h
⇒ (3 × 0.2) 3 = R 3 ⇒ R = 0.6 cm 21 8
⇒ r = 5 cm 3 Radius of base of cone ( x) = = 4 cm
=  × 100 % = 100%
V Now, this milk is poured into a 880 2
126 Now, thickness = 2r = 2 × 5 = 10 cm = × 1911 = 80080 cm 3
V  16. (c) Common radius = = 63 m cylindrical flask whose base radius is 2r. 21 1 2 4
2 ⇒ R2 = 100 + 25 = 125 ∴ π 4 h = π × 56 [by condition]
9. (c) Ratio of surface area Let H be the height of cylindrical flask. 3 3
Curved surface area of cylindrical part ⇒ R = 125 26. (b) Let side of a cube be a unit.
2
∴ Volume of milk 4
4 πr 2 ∴ Thickness of cylindrical shell × 56 × 3
=   =   ⇒ ∴ Volume of cube, V = a 3
4 2 r 2 = 2 πrh
= = 3
4 πR2  25   5  R 5 = R − 10 = 125 − 10 = Volume of cylindrical flask and total surface area of cube, S = 6 ( a )2 ⇒ h= = 14 cm
16
Now, ratio of volumes = 5 5 − 5 × 2 = 5 ( 5 − 2) cm 1 2 S 3 = 63 ( a 6 ) = 216 V 2
⇒ πr h = π ( 2r )2 H Hence, the height of cone is 14 cm.
4 3 16 C 3
πr 3 3 D 19. (b) Curved surface area of cylinder 27. (b) Total area of wetted surface
=   =   = 31. (c) Surface area of sphere, S = 4 πr 2
r 2 8 21 m
= 3 22 ⇒
1 2
πr h = 4 πr 2 H ⇒ H =
h
4  R 5 125 =2× × 14 × 3 = 264 m 2 = Curved surface area of cylinder + Area 1
πR3 5 7 3 12 If radius is 2r, then surface area of
3 of base of cylinder
22. (d) Let radius and height of a cone be r sphere,
= 8 : 125 A B 10.5 m = 2 πrh + πr 2
and h. S = 4 π ( 2r )2 = 16 πr 2
63 m r 3 4r = π [2 × 3. 5 × 4 + (3.5)2 ] 2
10. (c) Let radius of cylinder = x and radius D C Q = ⇒h= …(i) ∴ S = 4S
22 13.5 cm 22 22 2 1
of cone = 2x = 2× × 63 × 5 = 1980 m 2 h 4 3 = (28 + 12. 25 ) = × 40.25 Hence, it increases four times.
7 3m 1 7 7
Height of each = h Volume of cone = πr 2 h
Volume of cone Height of cone = 21 − 5 = 16 m 3 = 126 .5 m 2 32. (c) Given, l = 6 m, b = 4 m
∴ Required ratio = Curved surface area of conical part = πrl A
Volume of cylinder
B

1
96 π = π × r 2 ×
4r 28. (b) Let height h , radius r , area S and h = 1. 25 m
22 14 m 3 3 1 1 1
1 = × 63 × ( 63)2 + ( 16)2 and volume V of first cylinder. Area of wetted surface
π 4x 2 h 7 Height of cone = 13.5 − 3 = 10.5 m 1
4 [from Eq. (i)] = 2 ( l × h + b × h) + l × b
=3 2 = = 4:3 22 Slant height of cone 96 × 3 × 3 Similarly, height h , radius r , area S
πx h 3 = × 63 × 3969 + 256 ⇒ r3 = = 216 2 2
and volume V of second cylinder.
2 = 2 ( 75. + 5) + 24
7 = (10.5)2 + (14)2 = 110.25 + 196 4 2 r = 25 + 24 = 49 m 2
11. (c) Volume of sphere = Volume of cylinder 2
22 ⇒ r = 6 cm and h = 8 cm By given condition, 1 = …(i)
4 = × 63 × 65 = 12870 m 2 = 306.65 = 17. 5 m r 3 33. (b) Given, R = 3 cm, r = 2 cm, h = 10 cm
∴ πR 3 = π × 3 × 3 × 32 2
3
7 Curved surface area of cone = πrl ∴ Lateral surface area = πrl 2π r h Total surface area
S 5 5
⇒ R3 = 3 × 3 × 3 × 8 ⇒ R = 6 cm Q Total surface area 22 [Q l = r + h ]
2 2 2 1 = ⇒
1 1
=
= × 14 × 17.5 = 770 m 2 ∴
= 2 π ( R + r) ( R + h − r)
3 = 1980 + 12870 = 14850 m 2 7 = πr r 2 + h 2 S 3 2 πr h 3
2 2 2
4 3 = 2 π (3 + 2) (3 + 10 − 2)
12. (c) Volume of first sphere = π  ∴ Total cost of canvas used Total area to be painted h 2 5 h 5
3  2 = π × 6 36 + 64 = 60 π cm 2
⇒ 1 × = ⇒ 1 = …(ii)
= 14850 × 12 = ` 178200 = 264 + 770 = 1034 m 2 = 110 π cm 2
4 h 3 3 h 2
and volume of second sphere = π ( 2) 3 23. (b) Q Volume of wax balls 2 2
34. (b) For conical part, r =
6
= 3 cm,
3 17. (d) Given, radius of cylinder = 6 cm ∴Cost of painting = 1034 × 2 = ` 2068 V π r2h r 
2
h 
4 4 2
= π ( 1)3 = π cm 3 = 1  1
1 1
∴ Volume of big sphere Volume of sphere ∴ 1 =
3 3 V π r2h r  h  h = 4cm, l = h + r 2 = 5 cm
2
4  3
3
 20. (a) Let height of the cylinder be H.  2  2
= π   + ( 2) 3  [by condition]
4 4 22 2 2 2
= πr 3 = × ×6×6×6 By given condition,
4
and volume of sphere = π (8)3 cm 3 2 Surface area of conical part = πrl
3  2  =     =
3 3 7 2 5 10
 3 or, 10 : 9 = 3.14 × 3 × 5 = 47.1 cm 2
4 3 4  27 88 × 2 × 36 Volume of hemisphere + Volume of  3   2 9
πx = π  + 8 = 3 4
⇒ cm π (8 ) 3 6
7 cylinder = Volume of container For hemispherical part, r = = 3 cm
3 3 8  ∴Required number of balls = 3 29. (b) By given condition, 2
2 3
Let r be its internal radius, R be its ∴ πr + πr 2 H = 32400 π 4
x3 = 91/ 8 = 11.375 cm 3 3 π Volume of cylindrical vessel = Volume of Surface area of hemispherical part
external radius, then material used to 3 sphere = 2 πr 2 = 2 × 3.14 × 3 × 3
13. (b) Given, l = R cast the cylinder 4
15 cm = 512 ∴ πr 2 h = π R3 = 5652
. cm 2
22
Q Total surface area of cone, = πh ( R2 − r 2 ) = × 32 ( 25 − r 2 ) 70 3
7 C 24. (b) Given, r = cm = 0.35 m ∴ Surface area of toy = 47.1 + 56.52
S = π R ( R + l ) = π R ( R + R) = 2 π R 2 D 2 4 d
3
22 ⇒ ( 60)2 × 10 =   = 103.62 cm 2
∴ Area of circle, A = π ( 2R)2 = 4 πR2 Hence, × 32 × ( 25 − r 2 ) and h = 4 m 3  2
7 35. (b) Let r and h be the radius and height
⇒ A = 2S ∴ Surface area of 10 cylinders 4 d3
88 × 2 × 36 H ⇒ ( 60)2 × 10 = × of the cylinder. Here, h = 9 m
= 3 8
14. (a) In one round, distance covered by 7 = 10 ( 2 πrh ) By given condition,
cylinder = 60 cm ⇒ d 3 = ( 60)2 × ( 60) 2 πr
88 × 2 × 36 × 7 = 10  2 × . × 4 = 88 m 2
22 2 πr × h = ( h + r)
⇒ ( 25 − r ) =
2
=9 × 035 ∴ d = 60 cm
In one second, distance covered by 22 × 32 × 7 A B  7  3
cylinder = 60 × 5 = 300 cm 15 cm 30. (c) Given, inner radius of hollow sphere ⇒
1
9 = ( 9 + r ) ⇒ 27 = 9 + r
⇒ r 2 = 25 − 9 = 16 ⇒ r = 4 cm ∴ Total cost of 10 pillars painting at
In five seconds, distance covered by 2 4 3
⇒ π × 3375 + π × 225 H = 32400 π the rate of ` 5 per m 2 ( r ) = = 2 cm
cylinder = 300 × 5 = 1500 cm = 15 m Thickness of cylinder 3 2 ∴ r = 18 m
= R − r = 5 − 4 = 1 cm ⇒ 2 π × 1125 + π × 225 H = 32400 π = 88 × 5 = ` 440
MATHEMATICS > Quadratic Equations and Inequalities 353 354 CDS Pathfinder

36. (b) Let r and r be the radii of two spheres. 41. (c) Given radius of balls = 1 cm 4 3 100 19 50. (b) Given, length of one compartment and diagonal of cube = y 3 = 2x 3
1 2 Q Volume of sphere = πr Increases in length = h−h= h
4 3 81 81 = 200 m [from Eq. (i)]
4 3 Volume of each ball = × π × 1 × 1 × 1 4
πr r 3 ∴ Volume of utilised wood = πr 3 Percentage increase =
19 / 81h
× 100%
and breadth of one compartment Hence, the required ratio
3 1 =8 ⇒ 1 = 2 3 h = 100 m
Q 4 21x 21
4 3 1
πr r 1 = π cm 3 and height of water in one compartment = = = = 175 .
2 3 and volume of wasted wood = 23.45% = 23% (approx) 2x 3 4 ×3
3 2 4 πr 2 = 12 m
∴ Volume of 4 balls 4 3 6 πr 3 − 4 πr 3 2 πr 3
∴ Ratio of their surface areas = 1 = 2 πr 3 − πr = = 47. (b) Let the height of cylinder be h. Volume of one compartment
4 πr 2 3 3 3 53. (c) Area of tank,
2 4 16 D C = 200 × 100 × 12
=   = 4 : 1
2 2 =4× π= π cm 3 4 2 = 240000 m 3 BCDE = 10 × 75 . = 75 m 2
 1 3 3 ∴ Required ratio = πr 3 : πr 3 = 2 : 1
3 3 ∴ Volume of 3 compartments G F
D C h
37. (d) Area of the field = Length × Breadth C D •
= 3 × 240000 = 720000 m 3
44. (b) Let the slant height of cone = l cm E
= 12 × 10 = 120 m 2 = 720000000 L D
h In ∆ ABC , A B 7.5 m
Area of the pit’s surface = 5 × 4 = 20 m 2 [Q 1 m 3 = 1000 L ]
h A
Area on which the Earth is to be spread Then, radius of cylinder = B 10 m C
A B A B 2 Total requirement of water 50000
= 120 − 20 = 100 cm 2 F E h inhabitants in 1 day Area of remaining field
Volume of water increased Also, radius of the sphere, r =
Volume of Earth dug out 2 = 50000 × 20 = 1000000 L ABEDFGA = 125 × 15 − 75
= 5 × 4 × 2 = 40 cm 3 = Volume of balls 2 = 1800 m 2
Q Volume of cylinder = π   h ∴ Required number of days
h
40 2 16 Volume of Earth dug = 10 × 75 . × 6
∴ Level of field raised = = m ⇒ Area of base × height = π C  2 720000000
3 = = 720 days
100 5 = 450 m 3
16 πh 3 1000000
2
= × 100 = 40 cm ⇒ π ×5×5× h = π l = By given condition, 1800 × h = 450
5 3 30° 4 51. (b) Let r and h be the radius and height
1 1
16 16 A D and volume of sphere. of the cone and r and H be the radius ⇒ h = m = × 100 cm = 25 cm
38. (a) Edge of cube = 9 cm ∴ h= = cm 3/2 B 3
πh 3 4 4
S = πr 3 = π   =
3 × 25 75 4 4 h and height of the cylinder.
For rectangular vessel,
AB 3 / 2 3 3  2 6 1 54. (b) Let r = k and r = nk
cos 30° = = [Q AC = l cm] ∴ Volume of cone = πr 2 h 1 2
Length = 15 cm and breadth = 12 cm 42. (c) Given, height of the cone = 30 cm Q Volume of remaining material 3 Since, V =V
AC l 1 2
Let the rise in the water level is x cm. ∴ Volume of original cone ( AOB ) πh 3 πh 3 πh 3 and volume of cylinder = πr 2 H 1 2
3/ 2  3 = − = ∴ πr h = πr 2 h
Volume of cube = 9 3 = 9 × 9 × 9 cm 3 1 ⇒l = = 3 cm Q cos30° = 1 3 1 1 2 2
2 
= πR2 H 4 6 12 By given condition, πr 2 h = πr 2 H
Volume of water risen in the vessel 3
3/2  ⇒
1 2
πk × h = πn2 k 2 h
∴Volume of sphere in remaining material, 3
= 15 × 12 × x cm 3 O ∴ Area of cone ACD = πrl ⇒ h = 3H …(i) 3 1 2
4
⇒ 15 × 12 × x = 9 × 9 × 9 S = πr 3 ⇒ h = 3 n2 h
3 3 3π 1
3 1 Lateral surface area of cone = πrl 1 2
9 × 9 × 9 81 C P D = π× × 3 = cm 2 55. (d) Let r be the radius, h the height and l
∴ x= = = 4.05 cm 2 2 4 3 πh 3 and lateral surface area of cylinder
15 × 12 20 By given condition, πr = the slant height of smaller cone
1
45. (a) Volume of cone, C = πR2 H 3 1 12 = 2 πrH slant height of original cone 2
39. (b) The maximum diameter of a sphere A B 3 h3 h Given, =
in a cube is of 3 m.
Q ⇒ r3 = ⇒ r = By given condition, πrl =
15
× 2 πrH slant height of smaller cone 1
1 1
16 1
2 ⋅ 21 / 3
1 = πR 3 [Q H = R] 8
4 = × πR2 × 30 = 10 πR2 cm 3 Since, ∆ ABM and ∆ ADL are similar.
∴ Volume of sphere, V = π (1.5) 3 3
Required ratio = =
r h/2
= 21 / 3 15 225 2
1
3 3 D r h / 2 ⋅ 21 / 3 ⇒ l = H ⇒ l = 2
H A
= 4.5 π m 3 A B 1 4 16
Volume of small cone OCD R R
Volume of cube, V = (3) 3 = 27 m 3 1 ⇒ r : r = 21 / 3 : 1 ⇒ r 2 + h2 =
225 2
H [Q r 2 + h 2 = l 2 ] l
2 = of volume cone AOB 1
16
and Volume of solid left = V − V 27 R 2l r
2 1 48. (c) Let side of square be x D E
225 2
= (27 − 4.5 π ) m 3 ⇒
1 2
πr h =
1
× 10 πR2 ∴ Radius of sphere = x ⇒ r2 + 9 H 2 = H [from Eq. (i)] L
3 27 C 16
40. (c) Length of water flowing in 1h = 10 km
2
Volume of hemisphere, H = πR 3 Surface area of sphere, A = 4 πx2
10 πR2 81 2 r2 81
1 3 10  R 
2
3 ⇒ r2 = H ⇒ =
∴ Length of the water flowing in h ⇒h = × ⇒h =   1 3 2
Since, square revolves round a side to 16 H 2 16
2 27 πr 2 9 r ∴C : H = πR : πR = 1 : 2
3
generate a cylinder whose height and 2r M
3 3 r 9 B C
= 5 km = 5000 m R 30 radius are x and x, respectively. ⇒ =
Q = [Q ∆ OQB ~ ∆ OPD ] H 4
Area of pipe r h 46. (d) Volume of wire = πr 2 h ∴ S = 2 π x ( x + x ) = 4 πx 2
40 1 ∴ r: H = 9: 4 Therefore, radius and height of original
= 40 cm 2 = m2 = m2 10  30 
2
10 900 Now, new radius of the wire So, it is clear that, A = S. cone will be 2r and 2h respectively.
10000 250 ∴ h=   = × 2 r × 90 9r
9  h 9 = = 52. (b) Let the sides of the cuboid be x, 2x 1
1 h 49. (a) Given, length of pool = 24 m π × ( 2r )2 × 2h
∴ Volume of water flowing in h 100 10 and 4x and the side of the cube is y. V 8r 2 h
2 ⇒ h 3 = 10 × 100 and breadth of pool = 15 m ∴ 1 = 3 = 2 =8
1
Let new length of the wire be L. . ∴ Volume of cuboid = x × 2x × 4x V 1 2 r h
= 5000 × = 20 m 3 ⇒ h = 3 1000 = 10 cm  9r
2
Rise in height of water 2 πr h
250 ∴ Volume of new wire = π   × L = 8x 3 3
∴ Required height = 1 cm = 0.01 m
 10  and volume of cube = y 3 ∴ V :V = 8 : 1
Area of bottom of tank
= H − h = 30 − 10 = 20 cm   1 2
Volume of water displaced by x men
= 80 × 40 = 3200 m 2 =
81
πr 2 L By given condition, 56. (d) By given condition,
43. (a) Let r be the radius of cylindrical = 24 × 15 × 0.01 = 3.6 m 3 Surface area of new sphere = 25% of 616
∴ Depth of water block, then height will be 2r.
100 Volume of cuboid = Volume of cube
81 But volume of water displaced by x men
=
Volume
=
20
=
1 By given condition, πr 2 h = πr 2 L ∴ 8x3 = y 3 ⇒ y = 2x …(i) 4 π r 2 = 154
m Volume of block = π ( r 2 ) ( 2r ) = 2 πr 3 100 = 0.1 x m 3 2
=   ⇒ r = 3.5 cm
Area 3200 160 154 7
A sphere of maximum possible volume 3.6 ∴ Diagonal of cuboid ⇒ r2 =
100 ∴ 0.1 x = 3.6 ⇒ x = = 36
× 4  2
1 5 ⇒ L= 22
= × 100 cm = cm is curved out whose radius will be r. h 0.1 = x2 + 4x2 + 16x2 = 21x
160 8 81 7
MATHEMATICS > Quadratic Equations and Inequalities 355 356 CDS Pathfinder

57. (d) Let side of a cube be ‘a’ unit. Hence, volume of cuboid = lbh 66. (b) Let the radius of ball be r. II. Volume of both shots h 2 + r 2 of formed rectangle, 77. (a) In ∆ABC , AC 2 = AB 2 + BC 2
a 4 4
Then, radius of sphere is unit. ( y 2 + z 2 − x 2 ) (z 2 + x 2 − y 2 ) Q Radius of base of cylinder = 4r = π (0.75) 3 + π ( 08 . )3 So, statement II is false. AC 2 = 92 + ( 6 − 3)2 = 81 + 9 = 90
2 3 3
and height of cylinder = 4r Statement III and IV are correct. ∴ AC = 3 10 cm
Volume of cube ( a) 3 6 (x + y − z )
2 2 2
4  3
3
4 
3
∴ = = = ∴ Volume of spherical ball = πr 3
4 = π    +    Solution. (74-76) 78. (d) By using properties of similar
Volume of sphere 4 π  a  3 π 2×2×2 3  4  5 
  3 As cone is divided into two parts by a triangle in ∆MPA and ∆MOC .
3  2 and volume of water 3 x
= π 
( y + z − x ) (z + x − y )
2 2 2 2 2 2 4 27 64  plane through mid-point of its axis, = ⇒ 3x + 27 = 6x
1 1 + 
58. (d) Total surface area = Curved surface = = π ( 4r )2 ( 4r ) = 32 πr 3 3  64 125  therefore it will bisect the slant height. 6 x+9
area of cylinder + Curved 2 2 ( x2 + y 2 − z 2 ) 2 ∆AOB ~ ∆ADC
 3375 + 4096  ⇒ 27 = 3x
surface area of cone + Top surface area Also, volume of remaining portion of 4
= π  A
⇒ x = 9 cm
of cylinder 63. (a) Let the radius of cone and sphere be cylinder = 32 πr 3 3  8000 

= 2 πrh + πrl + πr 2 = π [ 2rh + r 2 + rl ] r and height of cone be h . Hence, height of cone = 9 + 9
1 Let number of spherical balls be n.
= π 
4 7471 4 C
 = π (0.93) cm D
3
= π [2 × 3 × 4 + 3 + 3 3 + 4 ]2 2 2 By given condition, 4 = 18 cm
∴ 32 πr 3 = n × πr 3 3  8000  3 10
= 48 π cm 2 Volume of cone = Volume of sphere 3 79. (b) Total surface area of the frustum
h ⇒ n = 8 × 3 = 24 Hence, statement I, II are true.
59. (b) Curved surface of a cylinder
1 4
∴ π r 2 h = π ( r )3 ⇒ 1 =
2 = π[ R + r ) l + r 2 + R2 ]
3 1
3 2r 1 67. (b) From the information given in the 70. (c) I. Coterminous edges are those who
= 1000 cm 2 , 2 πrh = 1000 …(i) have same boundaries and for a 5 B
where, l = h 2 + ( R − r )2
question and the figure it is clear that O
or, 2 : 1
Length of wire used in one round Radius of the hemisphere = radius of cuboid, these may be considered as = π [( 6 + 3) 81 + 9 + 9 + 36]
= Perimeter of cylinder’s base = 2 πr 64. (d) Let the length, breadth and height of cone = height of cone = height of length, breadth and height. DC AD 1 OB = π( 9 90 + 45)
a rectangular parallelopiped be 6x, 3x ∴ = = ⇒ DC =
cylinder. Let it be r . So, given statement is true AO 2 2
∴ Number of rounds and x, respectively.
OB = 9 π(3 10 + 5) cm 2
Height of cylinder h Then, ratio of volume of cylinder, II. The surface area of a cuboid is twice ∴Radius of smaller cone will be half of
= = Also, let the side of a cube be a. hemisphere and cone. the sum of the products of lengths of 80. (c) Curved surface area of the well
Diameter of wire 0.5 original cone
its coterminous edges taken two at a 22
∴ Required length of wire
By given condition,
C = 2 πrh = 2 × × 2 × 14 = 176 m 2
D time. 74. (c) The volume of the original cone 7
h 2 πrh Surface area of a cube = Surface area of 1 250 π
= 2 πr × = Hence, both statements I and II are = × π × (5)2 × 10 = Q Expense of getting per square metre
05
. 05
. rectangular parallelopiped correct. 3 3 plastered = ` 25
=
1000
= 2000 cm or 20 m ⇒ 6 ( a )2 = 2 ( 6x × 3x + 3x × x + x × 6x) 71. (d) I. The distance between vertices The volume of the frustum. ∴ Expense of 176 m 2 = 176 × 25
2
05
. ⇒ 6a 2 = 2 ( 18x2 + 3x2 + 6x2 ) 250 π 1 = ` 4400
− × π   × 5
B and C is 1 cm. 5
=
60. (d) Let h be the depth of the pit. ⇒ 6a 2 = 54x2 A B II. The distance between A and B is 3 3  2 81. (a) Given that,
∴ Volume of Earth dug ∴ a = 3x 12 + 12 = 2 cm 250 π 125 π 875 π Radius of sphere (r) = 9 cm = 0.09 m
2 1 = − =
= πr : πr 3 : πr 3
3 F G 3 12 12 and diameter of wire (d) = 0.4 cm
= 500 × 250 cm 3 = 125000 cm 3 Now,
3 3
Volume of cube B C Thus, dividing the two volumes, we ⇒ R = 0. 2 cm = 0.002 m
But volume of pit = 50 × 50 × h 2 1 8 Given condition,
Volume of rectangular parallelopiped = 1: : =3: 2: 1 get
∴ h=
125000
= 50 cm = 0.5 m 3 3 3 3 3 7 Volume of sphere = Volume of wire
50 × 50 a (3 x ) 27x 3 4 r3
= = = = 4
⇒ π r 3 = π R2 h ⇒ h = ⋅ 2
6x × 3x × x 18x3 18x3 2 68. (a) l = 152 + 82 = 17 cm D 75. (c) Area of the top circle of the frustum
E 2 3 3 R
25 π
= π   =
5
61. (a) Since, sheet is revolved about its
or, 3 : 2 D C A 4 0.09 × 0.09 × 0.09
length, threfore a cylinder is formed with O  2 4 = × = 81 × = 243 m
3 0.002 × 0.002
65. (b) Given thickness of a metal = 1 cm l cm III. The distance between vertices B
h = 7 cm and r = 4 cm 76. (d) Let l be slant height of original cone.
15 cm 82. (c) Total surface area of cube
∴ Volume of the figure, thus formed 5 cm and D is 12 + 12 + 12 = 3 cm Curved surface area of cone
= π × 5 × l = 5 πl = 6 × (Side)2
22 4 cm Hence, the statements I and II, III
= π r2h = × 4 × 4 × 7 = 352 cm 3 ∴ 150 = 6 × (Side)2
7 are correct. Curved surface area of frustum
8 cm 5 l 15 150
62. (c) Let l, b and h be the sides of cuboid.
A B
72. (d) I. It is true that the surface area of all = 5 πl − π × × = πl ⇒ Side2 = = 25
the eight drops is greater than the 2 2 4 6
then, l 2 + b 2 = x2 …(i) 4 ∴ Side = 25 = 5 cm
Total surface area of the remaining solid surface area of big drop. Thus, the ratio of the two will be
b 2 + h2 = y 2 …(ii) 3 ∴ Volume of cube = (Side)3
∴ Inner radius = 2 πrh + πr 2 + πrl II. Let radius of spherical body be ‘a’.
and h + l =z
2 2 2
…(iii) 4 or 4 : 3. = 5 × 5 × 5 = 125 cm 3
= outer radius – thickness = 2 π × 8 × 15 + π(8)2 + π × 8 × 17 ∴ Volume of sphere, V = πa 3
Adding Eq. (i), (ii) and (iii), we get 3 Solution. (77-79)
= 5 − 1 = 4 cm = 240 π + 64π + 136π M 83. (b) Volume of cube = (Side)3
2 (l + b + h ) = x + y + z
2 2 2 2 2 2
2 ∴ Surface area of sphere, S = 4 πa 2
Volume of hemisphere = π( R3 − r 3 ) = 440 π cm 2 729 = a 3 ⇒ a = 9 cm
1 2 16 2 6
Q
⇒ l 2 + b 2 + h2 = ( x + y 2 + z2 ) 3 ⇒ V2 = π a ∝ S3 3 cm
2 2 2 69. (c) Volume of spherical lead shot x ∴ Diagonal of cube = Side × 3
= π(5 − 4 ) = π ( 125 − 64)
3 3
4 4
9 P
…(iv) 3 3 = π ( 1)3 = π cm 3 D A = 9 × 3 = 9 3 cm
3 3 III. It is true that sphere occupies biggest
From Eqs. (i), (ii), (iii) and (iv) 2
= π × 61 cm 3 space but has smallest surface area. 84. (a) Curved surface area of right circular

9 cm
3 I. Volume of 8 shots
y +z −x
2 2
z +x −y 2 2 2 2
Hence, all statements are true. cone = π r l
h= ,l = Q Weight of 1 cm 3 of metal = 9 g 4
2 2 = π (0.5) 3 × 8 22 440 × 7
∴ Weight of all metal 3 73. (c) I. It is a correct statement. B B C ∴ 440 = × 14 × l ⇒ l =
x2 + y 2 − z 2 O 6 cm 7 22 × 14
and b= = 2 / 3 π × 61 × 9 g = 366 π g
4
= π cm 3 II. The length may be πr but the
2 3 breadth is always less than Here, r = 3 cm, r = 6 cm and h = 9 cm = 10 cm
1 2
MATHEMATICS > Quadratic Equations and Inequalities 357 358 CDS Pathfinder

85. (d) Volume of cylinder = πr 2 h Given, P < Q ⇒ r =


P
and R =
Q = 1080 π + 2 π × 72 ∴ Volume of the largest sphere Thickness of the metal = 1 cm 105. (a) Given that,
1
4 3 π π = 1080 π + 144 π = 1224 π cm 3 C Height of bucket = 25 cm
Volume of sphere = πr ∴ Difference in radii of the ends of the
D C
3 2 92. (a) Given that, the height and radius of a D 10 cm
frustum = R − r
Number of spheres = 48 Q P Q− P right circular metal cone (solid) are 8 cm D C
= − = 180°
πr 2 h and 2 cm, respectively. B

Volume of cylinder
= 1 π π π
4 i.e. h = 8 cm and r = 2 cm
Volume of sphere π r3 89. (c) We know that, A 25 cm
3 2 Let the radius of the sphere be R . 3
π (Radius)3 = π  
4 4 3
π r2h πr 3
Volume of frustum 1
Then, by condition, πr 2 h = πR3
4 =
⇒ 1 = 48 ⇒ 1 = 48 πH 2 3 3 3 3  2 A B
B
4 4 3 = ( R + r 2 + Rr ) A
πr 3
πr 3 ⇒ 4 × 8 = 4R ⇒ R3 =( 2)3
3 4 . 27 9
= π = π = 4.5 π cm 3 Q Outer radius (R)
2
3 2 ∴ R=2
π  Q  P 
3 2 2 20 cm
 P Q 3 8 2
= Thickness of the metal
[Q r = h] = H   +   + .  Hence, radius of the sphere = 2 cm Radius of top ( R) = 20 cm
 π   π π π 98. (d) Given that, radius of a right circular + Internal radius
1 3
r 
3
r 
3
 and radius of bottom ( r ) = 10 cm
3  1  = 48 ⇒  1  = 48 × 4 93. (c) Let the edge of a square be x. Then, cone (r) = 12 m
⇒ = 1 + 3 = 4 cm ∴ Capacity of bucket
r  r  πH Q P PQ 
4  2  2 3 =  + +  its volume is x3 and sum of its edges is 12x. Height of a right circular cone (h) = 5 m π
3 π π π  ∴ Outer curved surface area = h ( R2 + r 2 + rh )
r r r Now, by condition, x3 = 12x ∴Lateral height of cone (l) = r 2 + h2 3
⇒ 1 = ( 64)1 / 3 ⇒ 1 = 4 ⇒ 2 = 1
H = 2 πrh = 2 × π × 4 × 10 = 80 π π
r r 1 r 4 = ( P + Q + PQ ) ⇒ x ( x − 12) = 0
2
= 144 + 25 = 169 = 13 m = × 25 ( 400 + 100 + 200) cm 3
2 2 1 3 102. (a) Let the height and radius of right 3
⇒ x = 12
2
[Q x ≠ 0] ∴ Required quantity of cloth to roll up π
Hence, the ratio of radius of ball to circular cylinder be h and r, respectively. = × 25 × 700 cm 3
90. (b) Let the side of both cubes be a, ∴ Its total surface area = 6x2 to form a right circular tent = πr l 3
cylinder is 1 : 4. Then, volume of cylinder = π r 2 h
then the height and radius of a cone, = 6 ( 12) = 72 sq units π 175 × 100 17. 5 π
a = π × (12) × (13) = 156 π m 2 = × L= L
86. (a) Volume of the cuboid = 720 cm 3 h = a and r = D C 3 1000 3
2 94. (a) Given that, 99. (a) Given that, Outer radius of
Area of base = lb = 72 Diameter of a right circular cone = 7 cm 106. (d) Given that, h = 15 cm
C C hemispherical shell (R) = 9 cm
Height of the cuboid 7 Let r be the radius of cylinder.
D ∴ Radius of a right circular cone = cm 9 cm
Volume of the cuboid D
= 2 7 cm h Lateral surface area of cylinder = 2 πrh
Base area of the cuboid and slant height of a right circular cone ⇒ 2 πrh = 660 [given]
B
B (l) = 10 cm
=
720
= 10 cm ⇒ πrh = 330
A A ∴ Lateral surface area of a cone = π r l 22 22
72 ‘S’ ⇒ × r × 15 = 330 ⇒ × r = 22
‘C’ 22 7 A
Surface area of the cuboid a = × × 10 = 11 × 10 = 110 cm 2 r B 7 7
= 2 ( lb + bh + hl ) and radius of sphere ( R) = 7 2 1 r = 7 cm
2 Volume of circular cone = π r2h Q
⇒ 484 = 2 ( 72 + b × 10 + l × 10) 1 95. (d) Let the height and radius of solid and inner radius of hemispherical shell ∴ Required ratio
3 ∴ Volume of cylinder = πr 2 h
∴ Volume of cone (C) = π r 2 h cylinder be h and r cm , respectively.
⇒ 20( l + b ) = 340 ⇒ l + b = 17 3 (r) = 7 cm Volume of utilised wood =
22
× 49 × 15
Given that, r = 5 cm = 7
2
πa 3 ∴ Volume of a hemispherical shell
= π   a ⇒ C =
So, it is obvious that length, breadth and 1 a Volume of wasted wood
...(i) and total surface area = 660 cm 2 2 = 22 × 7 × 15 = 2310 cm 3
height of the cuboid is 9 cm, 8 cm and 3  2 12 = π ( R3 − r 3 ) =
Volume of right circular cone
10 cm. 4 ⇒ 2 πrh + 2 πr = 660 2 3 107. (b) Given that, the diameter of Moon
and volume of sphere (S) = πR 3 Volume of right circular cylinder
87. (a) Surface area of the sphere ⇒ 2 πr ( h + r ) = 660 2 22 is approximately one-fourth of the
3 = × × ( 729 − 343) − Volume of right circular cone
= 4 πr 2 330 330 7 3 7 diameter of Earth.
3
πa 3 ⇒ ( h + 5) = = ×
= π   =
4 a
616 ... (ii) 5π 5 22 2 22 16984 1 2
πr h
1 2
πr h If radius of Moon = r
⇒ 616 = 4 πr ⇒ πr =
2
= 154
2 3  2 6 66 × 7 = × × 386 = = 808.76 1 Then, radius of Earth = 4r
4 ⇒ h= − 5 = 21 − 5 = 16 3 7 21 = 3 = 3 = = 1:2
From Eqs. (i) and (ii), S = 2C 1 2 2 4 3
154 × 7 22 πr 2 h − π r2 h πr h 2 πr
⇒ r =
2
= 49 91. (d) If 10 circular plates each of thickness ∴ Required height = 16 cm ≈ 808 cm (approx)
3
Volume of Moon
3 3 ∴ = 3
22 3 cm, are placed one above the other, 100. (b) Let r be the radius of the sphere. Volume of Earth 4
96. (a) Let the diameters of two spheres be 103. (c) Since, volume of cone and pyramid π ( 4r )3
Q r= 49 = 7 cm then it forms a cylinder with height
Given that, volume of sphere = 36 π 1 3
4 3 (3 × 10 = 30 cm) and a hemisphere of d and d , respectively. = × Base area × Height r3 1
∴ Volume of the sphere =
πr
1 2
4 3 = = = 1 : 64
3 radius 6 cm is placed on the top just to Q d : d = 3 :5 ⇒ π r 3 = 36 π ⇒ r 3 = 27 = (3)3 64 r 3 64
1 2
cover the cylinder that means its radius 3 Therefore, they have same volume but
4 22
= × ×7×7×7=
4312
cm 3 is also 6 cm. ∴ Ratio of their surface areas Q r = 3 cm their surface areas are not same as
108. (c) Slant height, l = h2 + r 2
3 7 3
∴ Radius of hemi-sphere (R) = 6 cm 4 πr 2 ( 2 r ) 2 d 2 ∴ Diameter of sphere nothing can be said about their slant
= 1 = 1 = 1 heights. = 162 + 122
88. (a) Given that, Radius of cylinder (r) = 6 cm 4 πr 2 ( 2 r ) 2 d 2 = 2r = 2 (3) = 6 cm
Area of first end = P = πr 2
2 2 2
and surface area of sphere = 4 πr 2 104. (b) Given that, height of cone (h) = 3 cm = 256 + 144 = 400 = 20 cm
and height of cylinder (h) = 30 cm d 
2
C
and area of second end = Q = πR2 2
= 4 π (3)2 = 36 π cm 2
=  1  =   = and slant height of cone (l) = 5 cm
3 9
∴ Volume of the solid = Volume of = 9 : 25
r  d  5 25 ∴ Required ratio
D C cylinder + Volume of hemisphere  2 Let r be the radius of cone.Then,
Surface area of sphere 36 π

16 cm
2
= πr 2 h + π R3 97. (c) From figure, it is clear that = = = 6π l = r 2 + h 2 = 5 ⇒ r 2 + h 2 = 25 l
3 Diameter of sphere 6
H 2 Diameter of a sphere = Side of the cube ⇒ r 2 = 25 − 9 = 16 ⇒ r = 4 cm
= π ( 6) × 30 + π ( 6)3
2
= 3 cm 101. (d) Given that, 1
3 ∴ Volume of cone = πr 2 h 12 cm
Radius = cm
3 Internal diameter of the tube = 6 cm 3 A B
2 Q
A
R
B = π × 36 × 30 + π × 216 2 ∴ Internal radius (r) = 3 cm 1 22 352
3 = × × 16 × 3 = = 50.3 cm 3 24 cm
Height of the tube (h) = 10 cm 3 7 7
MATHEMATICS > Quadratic Equations and Inequalities 359 360 CDS Pathfinder

Curved surface area = πrl and height of cone = h ′ In ∆AOB and ∆ACD, ∆AOB ~ ∆ACD 121. (c) Let ABC be a right angled triangle. Then, new side of cube = x + 100% of x 127. (b) Let length, breadth and height of a
=
22
× 12 × 20 cm 2 ∴ Volume of the complete structure By basic proportionality theorem, Then, hypotenuse, AC = 25 cm = x + x = 2x cuboidal box be l , b and h, respectively.
7 1 2 2 r h C
= πr h ′ + πr 2 h + πr 3 CD = , since AC = A ∴ New surface area of cube = 6( 2x ) 2
Cost of painting the surface of the cap 3 3 2 2
22 Volume of original cone
15 cm = 6 × 4x2 = 24x2 D h
h′
= × 12 × 20 × 0.70 = ` 528 = πr 2  + h + r  Now, ratio =
2
Volume of smaller cone Now, increase percentage in surface area
7 3 3  B D
1 2 E 24x2 − 6x2
109. (b) Slant height, l = h 2 + r 2 2  28 πr h
× 3.5 = × 100
. 2 B
= π (3 .5 )  + 6.5 + 3 8 6x 2
b
= ( 24)2 + ( 7)2 = 576 + 49  3 3  = 2
= 20 cm A
1  r   h 1 18 l
= 625 = 25 = π × 3.5 × 3.5 × 9. 76 = 37586
. cm 3 π    = × 100 = 300%
E 3 2    2  6 Given, areas of the three adjacent faces
C C
∴ Required ratio = 8 : 1 124. (a) Let radius of the sphere be r. are x, 4x and 9x sq units.
Let AB = 3x and BC = 4x
2.8 cm
117. (c) Volume of each small sphere Since, cylinder circumscri- D C Now, lb = x
Thus, AC 2 = AB 2 + BC 2
4 [Q area of rectangular face =
l D C π( 4) 3 [by pythagoras theorem] bes a sphere.
h Volume of bigger sphere 3 ∴ Radius of the base of h length × breadth]
3.5 cm = =
Number of small spheres 64 ⇒ ( 25)2 = (3x)2 + ( 4x)2 cylinder = r and height
12.8 cm Similarly, bh = 4x and lh = 9x
6.5 cm 4 π × 4× 4× 4 4 ⇒ ( 25)2 = 9x2 + 16x2 of cylinder = 2r
A O B = × = π cm 3 A B Now, ( lb ) ⋅ ( bh ) ⋅ ( lh ) = ( x) ⋅ ( 4x) ⋅ ( 9x)
r 3.5 cm 3 64 3 ⇒ 25 = x2 ⇒ x = 5 = Diameter of sphere
Total surface area = πr ( l + r ) ∴ AB = 15 cm and BC = 20 cm ⇒ ( lbh )2 = 36x3 ⇒ lbh = 36x3
A B Let radius of small sphere be r ′. 4
Now, volume of sphere = πr 3
=
22
× 7 ( 25 + 7) 4 4 Now, ∆ABC revolves about AC , so it 3
∴ πr ′3 = π ⇒ r ′ = 1 cm ∴ lbh = 6x3 / 2
7 3 3 forms two cones ABD and BCD. and volume of cylinder
22 Hence, volume (V ) of the structure lies Hence, volume of cuboidal box
= × 7 × 32 = 704 cm 2 Now, surface area of small sphere Since, ∆AEB and ∆ABC are similar. = πr h = πr ( 2 r ) = 2 πr
2 2 3
7 between 370 cm 3 and 380 cm 3 . BE 15 = lbh = 6x3 / 2 cu units
= 4 πr ′ 2 = 4 π cm 2 ∴
BE
=
AB
⇒ = 4 3
110. (c) Radius of cone, r =
24
= 12 cm 6 πr 128. (c) In a cuboid, 4 perpendicular face pairs
2 113. (b) Given, radius of cone = = 3 cm 118. (c) Volume of Earth dug out BC AC 20 25
∴ Required ratio = 3 3 =
4
2
= 8 × 2.5 × 2 = 40 m 3 15 × 20 2 πr 3×2 in bottom surface, 4 perpendicular face
Slant height of the cone, l = 52 + 122 ⇒ BE = = 12 cm pairs in top surface and 4 perpendicular
and height of cone = 4 cm 25 2
= 25 + 144 = 169 = 13 m D C = =2:3 face pairs in vertical surface.
Now, curved surface area = πrl So, radius of the base of cone 3
E Hence, total perpendicular pairs are 12.
where,l = r 2 + h 2 = 32 + 42 = 5 cm = BE = 12 cm 125. (a) Given, diameter of
2.5m 12 m In right angled ∆AEB, the base of the conical 129. (c) I. Surface area of sphere A
5m ∴Curved surface area = π × 3 × 5 = 15 π 12 cm
portion = 12 cm = 4 π( 6)2 = 144 π cm 2
C 15 × 22 AE = ( AB )2 − ( BE )2
D = ≈ 47 cm 2 8m ∴ Radius of conical Surface area of sphere B = 4 π(8)2
7 = ( 15)2 − ( 12)2
12 m A B portion = 6 cm 8 cm = 256 π cm 2
16 m 114. (d) Volume of clay required 15 m = 225 − 144 = 81 = 9 cm
 5.1 2 
Radius of hemisphere Surface area of sphere C = 4 π( 10)2
2
Area where Earth is spread × Field level
= π    −    × 21 So, height of cone ABD = AE = 9 cm
11 m 45
. = 6 cm and height of
raised = 400 π cm 2
12 m  2   2  ∴ Height of cone BCD = AC − AE conical portion = 8 cm
= Volume of Earth dug out Now, sum of surface area of spheres
A B = π [( 255
. )2 − ( 2. 25)2 ] × 21 = 25 − 9 = 16 cm ∴ Slant height of conical portion A and B
⇒ [( 12 × 15) − (8 × 2.5)] × h = 40 1
24 m = π ( 03
. × 48
. ) × 21 = 30. 24 π cm 2 40 40 1 Now, volume of cone ABD = πr 2 h = 62 + 82 = 36 + 64 = 144 π + 256 π = 400 π cm 2
Curved surface area for conical portion ∴ h= = = m 3
= Surface area of sphere C
115. (a) Surface area of cube which can be 180 − 20 160 4 1 [Q l = r 2 + h2 ]
= πrl =
22
× 12 × 13 =
3432 2
m painted = 6 (Side)2 = 6( 2)2 = 24 cm 2 = π( 12)2 × 9 = 432 π cm 3
100 3 = 100 = 10 cm Hence, statement I is correct.
7 7 Now, surface area of cuboid which can = cm = 25 cm and volume of cone Now, total surface area of the toy II. Q Volume of sphere D
111. (d) Curved surface area of cylinder be painted 4 1
BCD = π ( 12)2 × 16 = 768 π cm 3 4
= Curved surface area of conical portion = π( 12)3 = 2304 π cm 3
= 2 π rh = x = 2( lb + bh + lh ) 119. (a) Given, surface area of sphere 3 3
= 2( 2 + 6 + 3) = 22 cm 2 ∴ Required volume of double cone + Curved surface area of hemisphere
Volume of cylinder = πr 2 h = y = 616 cm 2 4
Volume of sphere A = π( 6)3 cm 3
2 π rh x 2y Total surface area of both cube and cuboid 616 × 7 = 432 π + 768 π = 1200 π = πrl + 2 πr 2 = π( rl + 2r 2 )
⇒ 4 πr 2 = 616 ⇒ r 2 = 3
⇒ = ⇒ r= = 22 + 24 = 46 cm 2 < 54 cm 2 4 × 22 = π ( 6 × 10 + 2 × 6 × 6) 4
π r2h y x = 1200 × 3.14 [Q π = 3.14] Volume of sphere B = π(8)3 cm 3
Hence, both cube and cuboid can be
⇒ r2 = 7 × 7 ∴ r = 7 cm = 3768 cm 3 = π ( 60 + 72) = 132 π cm 2 3
x painted.
Also, h= and volume of sphere
2 πr ∴ Diameter of the largest circle lying on 122. (d) Q Surface area of cone ABD = πrl 126. (b) Volume of the toy
116. (b) Let the cone is divided into two parts 4
x sphere = Volume of conical portion C = π( 10)3 cm 3
by a line l. = π × 12 × 15 = 180 π cm 2
∴ Required ratio = = 2 πr
h 3
A = 2 × r = 14 cm and surface area of cone + Volume of hemisphere
r 2y Now, sum of volumes of spheres
120. (c) Given, volume of cube = 216x 3 BCD = π × 12 × 20 = 240 π cm 2 =
1 2 2
πr h + πr 3 A , B and C
x h/2
x x x3 (Side) = 216x ⇒ Side = 6x
3 3 ∴Required surface area of double cone 3 3 4
= π[63 + 83 + 103 ]
= × =
= π  r 2 h + r 3 
l D = 180 π + 240 π = 420 π 1 2 3
2π ⋅
2y 2y 8 πy 2 C r/2 h Since, sphere is enclosed in hollow cube.
3 3  4
x ∴ Diameter of sphere = 6x = 420 × 3.14 = 13188
. cm 2 = π [ 216 + 512 + 1000]
= π  × ( 6)2 × 8 + × ( 6)3 
So, the ratio is not independent of x or y. 1 2 3
Now, surface area of sphere = 4 πr 2
123. (c) Let side of a cube be x.  3 
3 = 2304 π cm 3 = Volume of sphere D
112. (a) Let common radius be r cm. 2
Then, surface area of cube = 6x2
= 4 π   = 36 πx2
C O r B 6x
= π [ 96 + 144] = 240 π cm 3 Hence, statement II is also correct.
Then, height of cylinder = h  2 If the side of cube is increased by 100%.
MATHEMATICS > Quadratic Equations and Inequalities 361 362 CDS Pathfinder

130. (a) Given, diameter of a sphere, d = 6 cm 16 2 6×6×6 137. (b) Let the radius and height of the cone = 2 ( 12 × 5 + 5 × 16 + 16 × 12) volume of water tank after taking out 1 2 h
= π × Hence, volume of water = πr
d 6 9 4 × 4 × 4 × π3 be r and h, respectively. = 2 ( 60 + 80 + 192) water 3 1 2
∴ Radius of a sphere, r = = = 3 cm
2 2 6
= = 6: π ∴ Initial volume of cone = 2 × 332 = 664 sq units = l × l × ( l − 2) = l 2 ( l − 2) 1 r2 1 3V 3V V
1 ∴ l 3 − 128 = l 2 ( l − 2) = πh = πh = =
Let the radius of wire be R cm. π (V ) = πr 2 h …(i) Hence, the total surface area of cuboid is 6 4 24 πh 24 8
Hence, the ratio of square of their 3 664 sq units. ⇒ 2l 2 = 128 ⇒ l 2 = 64 ⇒ l = 8 m
Also, given the length of wire, H = 36 m volume is 6 : π. new height, H = h + h × 200% 149. (b) Let radius be r and height be h of the
140. (c) Given, radius of vessel, ( r ) = 4 cm So, volume of cubical tank
= 3600 cm = h+
200h
= 3h = 83 = 512 m 3 . cylinder.
134. (a) Let radius of sphere and cone be r. and radius of sphere ( R) = 3 cm
According to the question, 100 1
∴ Volume of sphere V = πr 3
4 145. (b) Let the sides of the cuboid be l , b Then, 30 ⋅ π r 2 h = π r 2 h × n
Volume of sphere = Volume of wire and new radius, R = r − r × 50% Let height of vessel be h cm, then volume 3
1
3 and h respectively.
50r r of vessel= πr 2 h = π( 42 )h = 16 πh
4 3 1 =r− = ⇒ n = 30 × 3 = 90 cones
⇒ πr = πR2 H and volume of cone V = πr 2 h 100 2 4 Volume of cuboid,
3 2
3 and volume of sphere = πR3 = 36 π
∴ New volume of cone (V ) = πR H
1 2 3 V = l ×b×h …(i) Statement I is correct. Statement II is
4 Now, according to the question,
⇒ × (3)3 = R2 × 3600 2
3 Given, x = lb, y = bh and z = lh also correct. But statement II is not
4 1 Let H be the rise in water level
3 V = 2V ⇒ πr 3 = 2 × πr 2 h 2
r2 ∴ xyz = lb × bh × lh correct explanation of statement of I.
= π   × 3h = π ×
1 r 1
4 × 32
1 2
3 3 × 3h ∴ Volume of water displaced = ( lbh )2 = V 2 [using Eq. (i)]
( 6)2 3  2 3 4 150. (c) At 30 min, the container was full
⇒ R2 = = ⇒ 2r = r h ⇒ 2r = h ⇒
3 2 h 2
= 1 3 3V = volume of sphere 146. (a) On each face, we can draw four right
3600 ( 60)2 r 1 = πr 2 h × = [from Eq. (i)] ∴ At 5 min before i.e. in 25 min, the
36 angled triangles by choosing one vertex
3 4 4 ⇒ 16 π H = 36 π ⇒ H = = 2.25 cm
∴ R=
6
=
1
= 0.1 cm ∴ h: r = 2: 1 3V V 16 to serve as the right angled and two container was half filled and at 20 min,
60 10 Decrease in volume = V − = container was one-fourth full.
135. (a) Let the radius of sphere be r. 4 4 adjacent vertices. Since there are six
141. (c) Volume of paint required
131. (b) Given, side of a cube = 2 cm 4 ∴ Decrease percentage in volume faces, you can form 6 × 4 = 24 triangles. 151. (c) I. Area of curved surface = 2 πrh
∴ Volume of sphere = πr 3 = 2 × (3 × 2 × 0.0001) +
∴ Maximum distance between two 3 =
Decrease in Volume
× 100% 147. (a) Let the area of each rectangle be A. Q 2 πr ( 2h ) = 2 ⋅ 2 πrh
According to the question, Initial Volume 2 × ( 2 × 1.75 × 0.0001) +
points of a cube Then, breadth of R , R and ∴ Curved surface area is also doubled.
V /4 V 2 × (3 × 1.75 × 0.0001) 1 2
= Length of diagonal If the radius of a sphere is increased by = × 100% = × 100% = 25% R are
A A
, and
A
respectively It is correct.
10%. V 4V = 0.0012 + 0.0007 + 0.00105 = 0.00295 m 3 3
x x x
= 3 × Side = 2 3 cm II. Total surface area of hemisphere
Then, new radius r ′ = r + r × 10% 138. (c) Given a rectangular paper of 44 cm Volume of cubical box = 103 .
1 2 3

r 11r
Cylinder is formed by joining parallel = 3 πr 2
132. (d) Let the slant height of cone be l cm. =r+ = long and 6 cm wide is rolled to form a = 1000 cm 3 = 0.001 m 3 side of breadth. Q 3 π ( 2r ) 2 = 3 π 4 r 2 = 4 ⋅3 ⋅ π r 2
Given, h = 24 cm and r = 7 cm 10 10 cylinder of height equal to width of the 0.00295 Here, length becomes height = 4 times the area
4 ∴ Boxes required = ≈ 3.
∴ l 2 = h2 + r 2 ∴ New volume of sphere = πr ′3 paper. 0.001 and 2 πr = breadth =
A
⇒ r=
A
It is also correct.
3 ∴Circumference of the base of cylinder x 2 πx
⇒ l 2 = 242 + 72 = 576 + 49 4 π  11r 
3
4 1331r 3 142. (d) Let the sides of a cube be l cm. 2 152. (c) Let n spherical ball be made.
= = 44 A A2
⇒ l 2 = 625 ⇒ l = 25 cm   = π× 44 44 × 7 ∴ V = πr 2 h = π 2 2 × x =
3  10 
1
3 1000 i.e. 2 πr = 44 ⇒ r = = Then, 6l 2 = 13254 4π x 4π x Then, 4 πr 2 = π 3
Total surface area = Curved surface Increased volume 2 π 2 × 22 13254 A 2
A 2
area of cone + curved surface area of ⇒ l2 = = 2209 ⇒ l = 47 Now, V , V and V are ,
4 1331 3 4 3 4 3 331 ∴ r = 7 cm 6 1 2 3
4 π x 4 πx
1 2
hemisphere = π× r − πr = πr × π3 1 −2 1 −1
3 1000 3 3 1000 Hence, the radius of the base of the ∴ Length of diagonal = l 3 = 47 3 cm
1 2
⇒ r2 = = π 3 ⇒ r2 =  π 3 
= πrl + 2 πr 2 = πr ( l + 2r ) A2 4π 4 2 
Increased percentage cylinder is 7 cm. and respectively
= π × 7( 25 + 2 × 7) = 7 π[ 25 + 14] 143. (b) Let the height of cylindrical 4 πx 1 −1
4 3 331 139. (c) Let the C portion be h. 3 ⇒ r=π 3
= 7 π × 39 = 273 π cm 2 πr × 1 1 1 2
dimensions of 2 3
πr + πr 2 h = 3312 π x <x <x ⇒ > >
= 3 1000 × 100% = 33.1% Then, Q 3
1 2 3
4 3 cuboid be l, b D h 3 x x x 4 1 −1
133. (a) Let the radius of the sphere and side πr 1 2 3 Given, n ⋅ π  π 3  = 1 × 1 × 1
πr 2  r + h  = 3312 π A2 A2 A2
and h 2 3 2 
of cube be r and a respectively. 3 ⇒ ⇒ > >
respectively. B  3  4 πx 4 πx 4 πx
∴ Total surface area of sphere 136. (b) Since, three metallic spheres are b 4 1 1
⇒ n⋅ π × × = 1 ⇒
n
=1
⇒ 12 × 12 × 12 + h  = 3312
1 2 3
A 2
( S ) = 4 πr 2 sq units melted to form a single sphere. For first face l  3  ∴ V <V <V 3 8 π 6
1 3 2 1
So, the sum of volume of three spherical l 2 + b 2 = 132 [Q l 2 + b 2 = d 2 ] 148. (a) Since, ∆OAB and ∆OCD are similar ∴ n=6
and total surface area of cube ⇒ 8 + h = 23 ⇒ h = 15m
solid sphere. ⇒ l 2 + b 2 = 169 …(i) r h 2
( S ) = 6a 2 sq units ∴ = = 153. (d) Let n cylindrical boxes be packed.
2 4 4 4 h 1
Now, according to the question, = π( 6)3 + π(8)3 + π( 10)3 For second face r
1 ⇒ n × π × 5 × 5 × 10
3 3 3 2
r2 6 b 2 + h 2 =( 281)2 1 = π × 15 × 15 × 100
S = S ⇒ 4 πr 2 = 6a 2 ⇒ = 4 We have, V = πr 2 h
= π[ 6 + 8 + 10 ]
3 3 3
⇒ b 2 + h 2 = 281. …(ii) h
1 2
a2 4 π 3 3 ∴ n = 90 boxes
…(i) For third face r
Let r be the radius of new sphere C D 154. (c) We have, area of cross-section of pipe
4
∴Volume of sphere (V ) = πr 3 cu units h + l = 20 ⇒ h + l = 400 …(iii)
2 2 2 2 2 r
1 ∴
4 3 4
πr = π( 63 + 83 + 103 ) = 4 cm 2 = 4 × 10− 4 m 2
3 On adding Eqs. (i), (ii) and (iii), we get
3 3 and 4 × 10− 4 × 40 m 3 water flows in 1 s.
and volume of cube (V ) = a cu units
3
2 ( l 2 + b 2 + h 2 ) = 850 r1
2 ⇒ r 3 = 63 + 83 + 103 ⇒ l 2 + b 2 + h 2 = 425 …(iv) Surface area of hemisphere 2 πr 2 h A
Now, Q = B Let cistern fill in t s.
2 ⇒ r 3 = 216 + 512 + 1000 Surface area of cylinder 2 πrh h
2  4 πr 3  Subtracting Eq. (i) form Eq. (iv), we get ⇒ 4 × 10− 4 × 40 × t = 10 × 8 × 6
V    16 2  r 2 
3
⇒ r 3 = 1728 r 12 4 2
 1 = 3 3  = π  2 h = 256 ⇒ h = 16 units
2
= = = 30000
V  a  ⇒ r 3 = 12 × 12 × 12 h 15 5 ⇒ t = 30000 s = h
 2  a  9 similarly, b 2 = 25 ⇒ b = 5 units O 60 × 60
  ⇒ r = 12 cm ∴ r : h = 4:5
3 and l 2 = 144 ⇒ l = 12 units 3V r r2 50
=
16 2  6 
π  ∴ Diameter of the new sphere 144. (a) Let the length of water tank be lm. ⇒ r2 = ⇒ r = ⇒ r2 = = h = 8 h and 20 min
 [from Eq. (i)] ∴ Total surface area of cuboid πh 1
2 1
4
9  4π volume of tank = l × l × l = l 3
6
= 2r = 2 × 12 = 24 cm = 2( lb + bh + lh )
28
363 364 CDS Pathfinder

1. Discrete frequency distribution Wages (in `) Number of workers


999.5-1099.5 125
A frequency distribution is called a discrete frequency
1099.5-1199.5 150
distribution, if data are presented in a way such that
exact measurements of the units are clearly shown. 1199.5-1299.5 200
1299.5-1399.5 250
Marks Number of students (frequency)
1399.5-1499.5 175
40 7
1499.5-1599.5 100
60 3

STATISTICS
80 3
100 2
Cumulative frequency
If the frequency of first class interval is added to the
Total 15
frequency of second class and this sum is added to third
class and so on, then frequencies so obtained are known
2. Continuous frequency distribution as cumulative frequency.
Exclusive form A frequency distribution in which upper
limit of each class is excluded and lower limit is EXAMPLE 1. Consider the table given below :
Usually (10-13) questions have been asked from this chapter. This chapter is very important from included, is called an exclusive form. Number of students Cumulative
Marks
e.g. In the class 0-10 of marks obtained by students, a (frequency) frequency
examination point of view and generally questions are asked from the graphical representation of data,
student who has obtained 10 marks is not included in 0-10 13 13
measures of central tendency.
this class. It will be counted in the next class i.e 10-20. 10-20 7 20

Inclusive method In this method,the classes are so 20-30 5 25


formed that the upper limit of a class is included in that 30-40 4 29
class. The following example illustrates the method. In 40-50 1 30
Statistics is the branch of Mathematics which deals with the Class Intervals Class intervals are the groups in
class 1000-1099, we include workers having wages 50-60 7 37
collection, analysis and interpretation of numerical data. In which all the observations are divided. Each class is
between ` 1000 and ` 1099. If the income of a worker 60-70 3 40
this chapter, we shall study measures of central tendency i.e. bounded by two figures (numbers) which are called
is exactly ` 1100, then it will be included in the next 70-80 4 44
mean, median and mode of ungrouped data and grouped class limits. The figure on the left side of a class, is
class 1100-1199. 80-90 5 49
data. Concept of cumulative frequency, the cumulative called its lower limit and that on the right side of a
90-100 1 50
frequency distribution how to draw cumulative frequency class, is called its upper limit. Exclusive method Inclusive method
curves (ogive) and graphical representation of data will also Total 50
Class mark It is the mid-point of the class interval. Wages (in `) Number of workers Wages (in `) Number of works
be discussed. 1000-1100 125 1000-1099 125
Lower class limit + Upper class limit Then, find the value of the following
i.e. Class mark = 1100-1200 150 1100-1199 150
Collection of Data 2
1200-1300 200 1200-1299 200
(i) frequency of class 10-20. (ii) class size.
Collection of data is the first step in statistics towards Range or a class size Difference between the upper 1300-1400 250 1300-1399 250 (iii) mid value of 60-70. (iv) total frequencies.
achieving the goal or conclusion. limit and the lower limit of a class is called its class 1400-1500 175 1400-1499 175 a. 10, 65, 50, 60 b. 7, 10, 65, 60
On the basis of collection, data are of two types size. 1500-1600 100 1500-1599 100 c. 50, 65, 10, 10 d. 7, 10, 65, 50
1. Primary data The data collected actually in the process Range = Upper limit − Lower limit Total 1000 Total 1000 Sol. d. (i) Here, frequency of class 10-20 is 7.
of investigation by the investigator is called primary data. e.g. Range of the observations 4, 7, 8, 10, 12 (ii) Class size = Upper limit − Lower limit = 30 − 20 = 10
It is original and first hand information. It is clear from the above example that both the
= 12 − 4 = 8 inclusive and exclusive methods give us the same class Upper limit + Lower limit 60 + 70
2. Secondary data The data collected by someone and used (iii) Mid value = = = 65
Frequency of an observation The number of times frequency although the class intervals are aparently 2 2
by any other person known as secondary data.
an observation occurs is called its frequency. different in the two cases. (iv) Total frequencies = 50

Presentation of Data In the above example on inclusive method, the EXAMPLE 2. The class mark of the interval 12.5-17.5
Raw or Ungrouped data When the data presented is Frequency distribution difference between the lower limit of a class and upper is
random and is not prepared according to some order, it is The tabular arrangement of data, showing the limit of the preceding class is 1. a. 5 b. 12.5 c. 15 d. 17.5
known as raw or ungrouped data. It does not give us a clear frequency of each observation is called a frequency 1 1
Therefore, we subtract from the lower limit and add Lower limit + Upper limit
picture of the class. distribution. It is a method of presenting the data in 2 2 Sol. c. Class mark =
2
a summarized form. Frequency distribution is also to upper limit of each class to make it continuous.
Grouped data When the data is arranged in any manner 12. 5 + 17. 5 30
known as frequency table. = = = 15
like ascending or descending order etc., it is called grouped The adjusted classes would be as follows: 2 2
data. It can also be presented in the form of a table called There are two types of frequency distribution which
frequency distribution table. are as follows:
MATHEMATICS Statistics 365 366 CDS Pathfinder

EXAMPLE 3. The class marks of a distribution are 1. Arithmetic mean of ungrouped or individual Shortcut Method Wages (in `) Number
d = xi − A fd
workers ( fi ) i i
54, 64, 74, 84, 94 and 104. Then, the class size is observations If x1 , x 2 , x 3 , . . . , x n are n observations, New mean = Previous mean × Constant multiplied to (xi )
a. 5 b. 10 c. 54 d. 104 each term = 18 × 3 = 54 800 7 −100 −700
then
2. Mean of grouped or continuous observations If 820 14 − 80 −1120
Sol. b. Since, class size is the difference between the x1 + x2 + . . . + xn 1 n
(i) Mean ( x ) = = ∑ xi x1 , x 2 , x 3 , . . . , x n are n observations whose 860 19 − 40 −760
class marks of two adjacent classes. n ni=1
corresponding frequencies are f1 , f 2 , f 3 , . . . , f n , then 900 25 0 0
∴ Class size = 64 − 54 = 10
It is also called direct method. 920 20 20 400
(i) Direct Method 980 10 80 800
1 n
MEASURES OF CENTRAL (ii) Mean ( x ) = A + ∑d i
ni =1 x1 f1 + x2 f2 + . . . + xn fn 1000 5 100 500
(a) Mean, x =
TENDENCY where, A = Assumed mean and d i = xi − A .
f1 + f2 + . . . + fn Total Σ f i = 100 Σ f i d i = − 880

An average or central value of a statistical series is the n Here, A = 900


It is also called shortcut method. ∑ fi xi Σf i d i −880
value of the variable which describes the charateristic i=1 Σf i x i ∴ Mean = A + = 900 + = 900 − 8.80 = 891. 2
= = Σf i 100
of the entire distribution. The following are the five EXAMPLE 4. If the heights of 5 persons are 144 cm, n Σf i
measure of central tendency ∑ fi ∴ Mean wage = `891. 2
152 cm, 150 cm, 158 cm and 155 cm, respectively. Find i=1
(i) Arithmetic mean or mean (ii) Geometric mean the mean height. n EXAMPLE 10. Find the mean of the following data.
(iii) Harmonic mean (iv) Median a. 152.5 cm b. 150 cm c. 149.8 cm d. 151.8 cm ∑ fi d i
i=1 ∑ fi d i x 15 25 35 45 55 65
(v) Mode Sum of the heights (b) Mean, x = A + =A+
Sol. d. Mean height = n
∑ fi f 4 28 15 20 17 16
Out of these measures of central tendency. Arithmetic Number of persons ∑ fi
144 + 152 + 150 + 158 + 155 759 i=1 a. 44.2 b. 42.4 c. 40.6 d. 41.6
mean, median and mode are sometimes knows as = = = 151. 8 cm
measures of location. 5 5 where, A =Assumed mean and d i = xi − A . Sol. d. Here, the values of class marks xi are given, so let
A = 35 and class width, h = difference between two
EXAMPLE 5. If the average of 6, 8, 5, 7, x and 4 is 7.  ∑ fi u i 
(ii) Step Deviation Method ( x ) = A +   ×h consecutive class marks = 10, then
Arithmetic Mean Then, the value of x is  ∑ fi  xi − a
The mean (or average) of number of observations is xi fi ui = fu
where, h = Difference between two consecutive class
i i
a. 10 b. 11 c. 12 d. 15 h
the sum of the values of all the observations divided marks. 15 − 35 −20
by the total number of observations. Sol. c. Q Sum of observation = 6 + 8 + 5 + 7 + x + 4 = 30 + x 15 4 = = −2 4 × ( −2 ) = −8
10 10
and number of observation = 6 EXAMPLE 8. The arithmetic mean of the marks from
Mean = Sum of observations/Number of observation 30 + x 30 + x 25 28
25 − 35 −10
= = −1 28 × ( −1) = −28
∴ Average = ⇒ 7= the following table is 10 10
Properties of Arithmetic Mean 6 6 35 − 35 0
Marks 0-10 10-s20 20-30 30-40 40-50 50-60 35 = ( A ) 15 = = −0 15 × 0 = 0
⇒ 42 = 30 + x ⇒ x = 42 − 30 = 12 10 10
1. If every observation is increased by a constant, Number of students 12 18 27 20 17 6
Hence, the value of x is 12. 45 − 35 10
then the mean of the observations, so obtained 45 20 = =1 20 × 1 = 20
a. 20 b. 28 c. 2800 d. 100 10 10
also increases by the same constant. EXAMPLE 6. The average of 27 observations is 35. If 5 is 55 − 35 20
Sol. b. We have, 55 17 = =2 17 × 2 = 34
2. If every observation is decreased by a constant, added to each observation, what will be the new mean? 10 10
then the mean of the observation, so obtained also a. 10 b. 20 c. 30 d. 40 Marks Class mark, (x) f fx 65 − 35 30
65 16 = =3 16 × 3 = 48
decreases by the same constant. 0-10 5 12 60 10 10
Sol. d. Given, x = 35 and n = 27 Σ fi = 100 Σ fi u i = 66
3. If each observation is multiplied by a constant, 10-20 15 18 270 Total
then the mean of the resulting observations can be Sum of observation = nx = 27 × 35 = 945 20-30 25 27 675
Thus, we have Σf iui = 66, Σf i = 100, h = 10
obtained by multiplying the mean by the same and new total of observation = 945 + 27 × 5 = 1080 30-40 35 20 700
1080  Σf u  66
constant. ∴ New mean = = 40 40-50 45 17 765 ∴ x = A + h i i  = 35 + 10 ×
27  Σf i  100
4. If each observation is divided by a constant, then 50-60 55 6 330
Shortcut Method 660
the mean of the resulting observation can be Total Σf = 100 Σfx = 2800 = 35 + = 35 + 6.6 ⇒ x = 416
.
obtained by dividing the mean by the same New mean = Previous mean + Number added to each term 100
= 35 + 5 = 40 Σfx 2800
constant. ∴ x= = = 28
Σf 100 Weighted Arithmetic Mean
5. The average of natural numbers from 1 to n is EXAMPLE 7. The mean of 53 observations is 18. If each
( n + 1) If corresponding weight of x1 , x 2 , . . . , x n are
. observation is multiplied by 3. What will be the new mean? EXAMPLE 9. Find the mean wage from the data
w1 , w 2 , . . . , w n respectively, then
2 a. 18 b. 36 c. 53 d. 54 given below :
6. The average of odd numbers from 1 to n is Wages (in `) 800 820 860 900 920 980 1000 Weighted arithmetic mean
Sol. d. Given, x = 18 and n = 53 n
Last odd number + 1 Number of workers 7 14 19 25 20 10 5
∑ w i xi
  and the average of even So, sum of observation = nx = 53 × 18 = 954
2 a. 890 b. 890.5 c. 891.2 d. 100 w1 x1 + w 2 x 2 + . . . + w n x n i=1
New total of observation = 954 × 3 = 2842 = =
Last even number + 2 Sol. c. Let the assumed mean be A = 900. w1 + w 2 + . . . + w n n

 ⋅ ∑ wi
numbers from 1 to n is  2862
∴ New mean = = 54
 2 53 The given data can be written as under : i=1
MATHEMATICS Statistics 367 368 CDS Pathfinder

EXAMPLE 11. The weighted arithmetic mean of the EXAMPLE 13. What is the geometrical mean of the f = Frequency of median class Sol. a. Arranging the data in the form of a frequency table, we
first n natural numbers, the weights being the variate which takes values 210, 201, 202, 20, 12, 10, 2, h = Size of median class have
corresponding numbers, is 1 and 0? c = Cumulative frequency of class before Value Frequency
n+1 n+ 2 2n + 1 a. 10 b. 9 c. 8 d. 0 median class 2.2 4
a. b. c. d. None of these 2.3 2
2 2 3
Sol. d. The given variates are 210, 201, 202, 20, 12, 10, 2, EXAMPLE 14. From the data given, the median 2.4 1
Sol. c. First n natural numbers are 1, 2, 3, . . . , n ; whose 1 and 0. of the average deposit balance of saving for the 2.5 2
corresponding weights are 1, 2, 3, . . . , n , respectively. ∴ Geometric mean of given variates branch during March 1982 is 2.6 1
1× 1+ 2 × 2 + ... + n × n = 9 210 × 201× 202 × 20 × 10 × 2 × 1× 0 = 9 0 = 0
∴ Weight arithmetic mean = Average deposit balance Number of deposit 2.7 2
1+ 2 + ... + n (in `)
1 + 2 + ... + n
2 2
n(n + 1)( 2n + 1) 2n + 1
2 Harmonic Mean 0 – 100 26
We see that, the value 2.2 has the maximum frequency 4. So,
the mode for the given series is 2.2.
= = =
1+ 2 + ... + n 6n(n + 1) 3 The harmonic mean of n positive numbers a1 , a 2 , . . . , a n is 100-200 68
2. Mode of a discrete frequency series In this case, mode is
2 1 11 1 1 1 200-300 145 the value of the variate corresponding to the maximum
n(n + 1)( 2n + 1)  =  + + +... + 
 H n  a1 a 2 a 3 an 
Q Σn =2 300-400 242 frequency.
 6 
 n(n + 1)  where, H denotes the harmonic mean. 400-500 188
 Σn =  EXAMPLE 16. Compute the modal value for the following
500-600 65
 2  frequency distribution.
Median 600-700 16
x 95 105 115 125 135 145 155 165 175
Combined Arithmetic Mean After arranging the given data in ascending or
a. 356 b. 300 c. 56.2 d. 356.2 f 4 2 18 22 21 19 10 3 2
If two sets of observations are given, then the combined descending order of magnitude, the value of the middle
most observation is called the median of the data. Sol. d. a. 115 b. 125 c. 22 d. 120
mean of both the sets can be calculated by the following Average deposit Cumulative
f
balance (in `) frequency (c f ) Sol. b. From the given table, it is clear that 125 has the highest
formula. 1. Median of an individual series If number of
n x + n2 x 2 observations is n. Then, arrange the observations in Less than 100 26 26 frequency i.e. 22. Hence, modal value of the given frequency
x= 1 1 distribution is 125.
n1 + n2 ascending or descending order. 100-200 68 94
200-300 145 239 3. Mode of a continuous series The class which has
where, x = Mean of sets of observations (i) If n is an odd number, then
maximum frequency is called modal class or group. The
x1 = Mean of first set of observations  n + 1 300-400 242 481
Median = Value of the   th observation mode is given by the formula,
n1 = Number of observations in first set  2  400-500 188 669
f1 − f 0
x 2 = Mean of second set of observations 500-600 65 734 Mode = l + ×h
n2 = Number of observations in second set
(ii) If n is an even number, then 2 f1 − f 0 − f 2
600-700 16 750
 n where, l = Lower limit of modal class
Value of the   th observation
EXAMPLE 12. The average salary of male employees  2 N 750
= = 375 h = Size of class interval
in a firm was ` 5200 and that of females was ` 4200. 2 2
n  f1 = Frequency of modal class
The mean salary of all the employees was ` 5000. The + Value of  + 1 th observation The frequency just greater than 375 is 481.
2  f 0 = Frequency of the class preceding the
percentage of male and female employees are, Median = ∴ Median class is 300-400.
2 modal class
respectively.  N − c
2. Median of a discrete frequency series First arrange   375 − 239 f 2 = Frequency of the class succeeding the
a. 80 and 20 b. 20 and 80 c. 60 and 40 d. 52 and 48 Median = l + 2 × h = 300 + × 100 modal class
the data in ascending or descending order and find f 242
Sol. a. Let x1 = 5200, x2 = 4200 and x = 5000
N = 300 + 56. 2 = 356. 2 EXAMPLE 17. The mode of the following distribution is
n1x1 + n2 x2 cumulative frequency. Now, find , where N = Σf i .
We know that, x= 2
n1 + n2 N
See the cumulative frequency just greater than .
Mode Class interval Frequency

n1 4 0-10 5
⇒ 5000 (n1 + n2) = 5200n1 + 4200n2 ⇒ = 2 The data value which occurs maximum number of
n2 1 times in a data distribution is called mode. 10-20 8
The corresponding value of x is median.
∴ The percentage of male employees in the firm 20-30 7
3. Median of a continuous series In this case, the 1. Mode of individual series The value which is
4 repeated maximum number of times is called 30-40 12
= × 100 = 80 class corresponding to the cumulative frequency just
4+1 mode of the series. 40-50 28
greater than N / 2 is called the median class and the
and the percentage of female employees in the firm value of median is obtained by the following 50-60 20
1 EXAMPLE 15. Find the mode for the following
= × 100 = 20 formula. 60-70 10
4+1 series 2.5, 2.3, 2.2, 2.2 2.4, 2.7, 2.7, 2.5, 2.3, 2.2,
N  70-80 10
 − c 2.6 and 2.2.
Geometric Mean Median = l +  2  ×h a. 2.2 b. 2.3 a. 46 b. 6.66
 f  c. 2.7 d. 2.6 c. 46.67 d. None of these
If a, G and b are in GP, then the geometric mean  
between a and b is, G = ab. where, l = Lower limit of median class
MATHEMATICS Statistics 369 370 CDS Pathfinder

Sol. c. Here, maximum frequency is 28. Thus, the class A frequency polygon for a grouped data can also be EXAMPLE 20. In statistics, a suitable graph
40-50 is the modal class. Here, f1 = 28, f 0 = 12, f 2 = 20,
Graphical Representation of Data drawn independently by plotting the mid-points of the for representing the partitioning of total into subpart
l = 40 and h = 10 There are many ways of picturing a frequency is
all classes along X-axis and frequencies along Y-axis and
 f1 − f 0  10 ( 28 − 12) distribution of continuous type which are discussed
∴ Mode = l +   × h = 40 + joining the plotted points by straight line. a. an ogive
 2f1 − f 0 − f 2  ( 2 × 28 − 12 − 20) below b. a pictograph
10 × 16 85 c. a histogram
= 40 +
24
= 40 + 6.666 = 46.67 (approx) 1. Bar Graph d. a pie chart
In a bar graph, bars of uniform width are drawn with 80

Temperature
Relation between Mean, Median and Mode various heights. The heights of a bar represents the
Sol. d. In statistics, a suitable graph for representing the
75 partitioning of total into sub parts is a pie chart.
The empirical relationship between the three measures frequency of observation.
of central tendency is given by EXAMPLE 21. The following diagram show the
70
Mode = 3 (Median) − 2 (Mean) expenditure of a family on various items A, B, C, D and E.
50
In case, mean = median = mode, then distribution is said 65
40 A

Frequency
to be symmetric distribution. B 30%
0 9:00 10:00 11:00 12:00 13:00 14:00 15:00
30 Time 25%
EXAMPLE 18. If in a frequency distribution, the mean E 10%
and median are 20 and 21 respectively, then its mode 20
C
is approximate by 10 4. Ogive (Cumulative Frequency Curve) D
20% 15%
a. 24 b. 23 c. 25 d. None of these When we plot the upper or lower class limits along
Sol. b. Here, mean = 20 and median = 21 10 20 30 40 50 X-axis and cumulative frequencies along Y-axis, And on Study the diagram carefully and answer the following
Mode = 3 × Median − 2 × Mean = 63 − 40 = 23
Number joining them we get a curve called an ogive. questions.
(i) The angle of pie diagram showing the expenditure
We have two types of ogive curves
Some Useful Formulae 2. Histogram incurred on item A is
(a) less than ogive (i.e. the rising curve) a. 30° b. 35°
If x1 , x 2 , …, x n are n observations with their mean M, A histogram is the graphical representation of a
then deviation d i is given by d i = | x i − M | . frequency distribution in the form of the rectangles with It is the graph between upper limits and cumulative c. 108° d. None of these
Σ |d i | class intervals as bases, and the corresponding frequencies of a distribution.
• Mean deviation for individual series is given by . (ii) Which two expenditures together will form an angle
n frequencies as height. There being no gap between any (b) more than ogive (i.e. a falling curve) of 90° at the centre of pie diagram?
Σf i | d i | two consecutive rectangles. It is the graph between lower limits and cumulative a. B and O b. C and A
• Mean deviation for discrete series is given by .
Σ fi frequencies of a distribution. c. D and E d. None of these
• Standard deviation (σ ) for ungrouped data is given by 40
Frequency

Note n
If we have both ogive (i.e. less than type and more (iii) If the income of the family is ` 3000 per month, then
Σ | d i |2 30
than type), then these two ogives intersect each other at a
σ= . 20 point.
expenditure of item C will be
n a. ` 400 b. ` 500 c. ` 600 d. ` 800
10 n
From this point, draw a perpendicular on X-axis, the point
• Standard variation for grouped data is given by at which it cuts the X-axis gives the median.
Sol. (i) c. Angle for 100% = 360°
Σf i | d i | 2 i.e. the x-coordinate of intersection point gives the median.
σ= ∴ Angle for expenditure of A = 30% of 360°
n Pie-Diagram (or Pie chart) 30
POINTS TO BE REMEMBERED = × 360 = 108°
• Variance = σ 2 A type of graph in which a circle is divided into sectors 100
A histogram consists of a set of adjacent rectangles,
Standard deviation l

such that each sector represents a proportion of the (ii) c. Q 100% = 360°
• Coefficient of variation = × 100 whose bases are equal to class sizes and height are equal
Mean to class frequencies. whole. ∴ An angle of 90° at the centre of pie diagram in
Mean deviation The total area of the histogram = Sum of areas of all E
• Coefficient of mean deviation = × 100 l
Thus, in a pie chart 90 × 100
Mean rectangles. If the class intervals are of same size (width), 30° percentage is = = 25%
then the area of histogram = NK, • Data is represented by sectors of a D A 360
EXAMPLE 19. The mean deviation of the data, 3, 5, 6, where K = Size of a class and N = Sum of frequences of circle. 120° 90°
Expenditure of items D and E makes upto
7, 8, 10, 11, 14 is all classes. • Each part of data makes a certain B (15 + 10) = 25%.
a. 4 b. 3.25 c. 2.75 d. 2.4 If the frequency distribution is discontinuous (inclusive), C 80°
l
central angle. So, expenditure of D and E together will form
change it to continuous (exclusive) and then construct a 40°
3 + 5 + 6 + 7 + 8 + 10 + 11+ 14 64 • Sum of all the angles of sector is
Sol. c.Mean = = =8 histogram. an angle of 90° at the centre of pie diagram.
8 8 360°.
° (iii) c. Expenditure on item C
∴ Σ δ = 3−8 + 5−8 + 6 −8 + 7 −8 + 8 −8  Frequency × 360 
3. Frequency polygon • Central angle =   20
+| 10 − 8 | + 11− 8 + 14 − 8 = 22  Total frequency  = 20% of ` 3000 = × 3000 = ` 600
A frequency polygon can be drawn by joining the 100
Σδ 22 mid-points of the respective tops of the rectangles of a
∴ Mean deviation = = = 2. 75
n 8 histogram in case of equal class intervals.
MATHEMATICS Statistics 371 372 CDS Pathfinder

PRACTICE EXERCISE 17. X


Frequency
0
4
1
f
2
9
3
g
4
4
23. If the mean and median of a set of numbers are
8.9 and 9 respectively, then the mode will be
(a) 7.2 (b) 8.2 (c) 9.2 (d) 10.2
The table above gives the frequency distribution
of a discrete variable X with two missing 24. If every number of a finite set is increased by
1. Frequency polygon can be drawn after drawing 11. The following pie chart shows the marks frequencies. If the total frequency is 25 and the any number k, then the measure of central
(a) ogive (b) bar chart obtained by a student in an examination who arithmetic mean of X is 2, then what is the value tendency should also increase by k. Which one of
(c) histogram (d) None of these scored 540 marks in all. The subject in which the of the missing frequency f? the following measures of central tendency does
student scored 108 marks is (a) 4 (b) 5 (c) 6 (d) 7 not have this property?
2. An ogive is used to determine
(a) Arithmetic mean
(a) mean (b) median (c) GM (d) HM 18. The yield of paddy per plot of one acre were
Mathematics obtained from a number of plots from two (b) Median
3. The mid-value of a class interval is 42. If the 90° (c) Mid-range i.e. the arithmetic mean of the largest
English different districts in a state and are summarized
class size is 10, then the upper and lower limits and the smallest numbers
63° in the following table.
of the class are (d) Geometric mean
Hindi
(a) 37.5 and 47.5 (b) 47 and 37 Social Yield of paddy per Number of plots in Number of plots in
60°
Science plot in quintals district A district B 25. If the median of the distribution (arranged in
(c) 37 and 47 (d) 47.5 and 37.5
Science 72° ascending order) 1, 3, 5, 7, 9, x, 15, 17, 19, 21 is
38.0-41.0 25 14
4. The actual lower class limits of the following 75° 10, what is the value of x?
41.0-44.0 36 29
classes 10-19, 20-29, 30-39 and 40-49 are (a) 11 (b) 13 (c) 9 < x < 15 (d) 9 ≤ x ≤ 15
(a) 9.5, 19, 29 and 39.5 (b) 10, 20, 30 and 40 44.0-47.0 59 35
(a) Science (b) Hindi 47.0-50.0 30 54
26. If the width of each of the ten classes in a
(c) 9.5, 19.5, 29.5 and 39.5 (d) 18.5, 28.5, 38.5 and 48.5
(c) English (d) Social Science frequency distribution is 2.5 and the lower class
50.0-53.0 25 41
5. If the mean of five observations x , x + 2, boundary of the lowest class is 5.1, then the
x + 4, x + 6 and x + 8 is 11, then the mean of first 12. A distribution consists of three components with Which one of the following statements is correct? upper class boundary of the highest class is
three observations is frequencies 45, 40 and 15 having their means 2, (a) 30.1 (b) 30 (c) 31.1 (d) 27.56
(a) The mode for district A is higher than the mode for
(a) 9 (b) 11
2.5 and 2 respectively. The mean of the combined district B
distribution is
27. The average value of the median of 2, 8, 3, 7, 4,
(c) 13 (d) None of these (b) The mode for district B is higher than the mode for 6, 7 and the mode of 2, 9, 3, 4, 9, 6, 9 is
(a) 2.1 (b) 2.2 (c) 2.3 (d) 2.4 district A
6. The combined mean of three groups is 12 and (a) 9 (b) 8 (c) 7.5 (d) 6
1 1 1 1 1 (c) Both the distributions are symmetric
the combined mean of first two groups is 3. If 13. If the values 1, , , , , . . . , occur at (d) Both the distributions have the same mean 28. When 10 is subtracted from each of the given
the first, second and third groups have 2, 3 2 3 4 5 n
observations, the mean is reduced to 60%. If 5 is
and 5 items respectively, then mean of third frequencies 1, 2, 3, 4, 5, 6,…, n respectively, in a 19. Average score of 50 students in a class is 44. added to all the given observations, the mean
group is frequency distribution, then the mean is Later on it was found that the score 23 was will be
(a) 10 (b) 21 1 2 incorrectly recorded as 73. The correct average
(a) 1 (b) n (c) (d) (a) 25 (b) 30 (c) 60 (d) 65
(c) 12 (d) 18 n n+1 score is
(a) 42 (b) 43 (c) 45 (d) 46 29. A data has highest value 120 and the lowest
7. 10 is the mean of a set of 7 observations and 5 is 14. If the geometric mean of three observations 40, value 71. A frequency distribution in descending
the mean of a set of 3 observations. The mean of 50 and x is 10, then the value of x is 20. In the “less than” type of ogive the cumulative order with seven classes is to be constructed.
the combined set is given by 1 frequency is plotted against
(a) (b) 4 (c) 6 (d) 2 The limits of the second class interval shall be
(a) 15 (b) 10 (c) 8.5 (d) 7.5 2 (a) the lower limit of the concerned class interval
(a) 77 and 78 (b) 78 and 85
8. In a class of 50 students, 10 have failed and (b) the upper limit of the concerned class interval
15. Suppose, X is some statistical variable with (c) 85 and 113 (d) 113 and 120
(c) the mid value of the concerned class interval
their average marks are 28. The total marks mean µ. Let x1 , x2 , . . . , xn be its deviations from 1
obtained by the entire class are 2800. The (d) any value of the concerned class interval 30. If mean of y and is M, then what is the mean
mean with the respective frequencies f1 , f2 , . . . , fn .
average marks of those who have passed are y
What is the value of the sum 21. A, B and C are three sets of values of x 1
(a) 43 (b) 53 (c) 63 (d) 70 x1 f1 + x2 f2 + . . . + xn fn ? A : 2, 3, 7, 1, 3, 2, 3 of y3 and ?
y3
9. Which of the following statements about the (a) 0 (b) 1 (c) µ (d) µ + 1 B : 7, 5, 9, 12, 5, 3, 8
M(M − 3)
2
median is true? C : 4, 4, 11, 7, 2, 3, 4 (a) (b) M 3
16. Which one of the following statements is not 3
(a) It is not affected by extreme values Select the correct statement from among the
correct with reference to a histogram? (c) M 3 − 3 (d) M (4 M 2 − 3)
(b) It can be found even, if some items are not known following
(a) Frequency curve is obtained by joining the
(c) It is useful when the data cannot be measured mid-points of the top of the adjacent rectangles with (a) Mean of A is equal to mode of C 31. For the following frequency distribution
quantitatively (b) Mean of C is equal to median of B
smooth curves 0-5 5-10 10-15 15-20 20-25 25-30
(d) All of the above Class interval
(b) Histogram is drawn for continuous data (c) Median of B is equal to mode of A
Frequency 10 15 30 80 40 20
10. The middle item of the series arranged in (c) The height of the bar is proportional to the (d) Median, mean and mode of A are same
ascending or descending order is called frequency of that class 22. A student obtains 75%, 80% and 85% marks in If m is the value of mode, then which one of the
(a) mean (b) median (d) Mode of the distribution can be obtained from the three subjects. If the marks of any other subject following is correct?
(c) mode (d) standard deviation histogram are added, then their average cannot be less than (a) 5 < m < 10 (b) 10 < m < 15
(a) 60% (b) 65% (c) 70% (d) 80% (c) 15 < m < 20 (d) 20 < m < 25
MATHEMATICS Statistics 373 374 CDS Pathfinder

Directions (Q. Nos. 44-45) The number of goals


32. Square diagrams are drawn to represent the 39. Consider the following statements in respect of 50. What is the sum of the squares of all the six
scored by a soccer team in a season were as follows:
following data histogram numbers?
I. Histogram is an equivalent graphical Number of goals Number of matches (a) 40 (b) 42 (c) 45 (d) 48
Country Pakistan India Myanmar China
representation of the frequency distribution. 0 3
Labour Production
36 81 25 100 1 6
51. What is the value of c ?
(in `) II. Histogram is suitable for continuous random
(a) 1 (b) 2 (c) 3 (d) 4
variables, where the total frequency of an 2 2
Using the scale 1 cm 2 = ` 25 what is the length interval is evenly distributed over the interval. 3 x 52. What is the mode?
of the representative square for India? Which of the statements given above is/are 4 5 (a) 1 (b) 2
(a) 1.8 cm (b) 1.2 cm (c) 1 cm (d) 2 cm correct? (c) 1, 2 and 4 (d) None of these
(a) Only I (b) Only II
44. If the mode is 1, the largest possible value of x is
33. The standard deviation of 7, 9, 11, 13, 15 is (c) Both I and II (d) Neither I nor II (a) 2 (b) 5 (c) 3 (d) 6 Directions (Q. Nos. 53-55) Study the following pie
(a) 2.4 (b) 2.5 (c) 2.7 (d) 2.82 45. If the median is 2, the smallest possible value of diagram and answer the questions.
40. Examples of data are given below
34. The total number of cellphones sold for Motorola, x is
I. Information on households collected by an Wheat
Samsung and Sony was 45664. The number of (a) 2 (b) 5 (c) 3 (d) 6
investigator by door-to-door visits.
cellphones sold for these companies were in the
II. Data on the percentage of literates, sex-wise, for
ratio 3:5:8 respectively. If these data were the different districts of a state collected from
Directions (Q. Nos. 46-47) The itemwise expenditure Rice
70° Sugar
40°
shown on a pie chart, calculate the angle records of the census of India. of a non-government organisation for the year 110°
represented by the number of cellphones sold by 2008-2009 is given below. 140°
III. General information about families, collected by
Motorola. Others
telephonic interviews. Item Expenditure (in ` lakh)
(a) 75° (b) 67.5° (c) 70° (d) 74.5°
Which one of the following in respect of the above Salary of employees 6
35. If the population figures are given for each State is correct? Social welfare activities 7
of India, then the data can be classified as (a) I and II are primary data (b) I and III are primary data Office contingency 3 53. The percentage of wheat production is
(a) qualitative (b) quantitative (c) II and III are primary data (d) I, II and III are primary data Vehicle maint evance 4 (a) 35% (b) 70% (c) 19.44% (d) 20%
(c) chronological (d) geographical Rent and hire charges 2.5
41. Consider the following types of data 54. If the sugar production is 9000 kg, then the
36. Which one of the following represents statistical I. Marks of students who appeared for a test of 100 Miscellaneous expenses 1.5 wheat production is
data? marks. (a) 20000 kg (b) 24000 kg
(a) The names of all owners of shops located in a The above data are represented by a pie diagram. (c) 15750 kg (d) None of these
II. Collar sizes of 200 shirts sold in a week.
shopping complex
(b) A list giving the names of all States of India
III. Monthly incomes of 250 employees of a factory. 46. What is the sectorial angle of the largest sector? 55. If the total production is 180000 kg, the
For which of the above data, mode is a suitable (a) 120° (b) 105° (c) 90° (d) 85° difference in sugar and wheat production is
(c) A list of all European countries and their respective
capital cities measure of central tendency? (a) 10000 (b) 15000 (c) 20000 (d) None of these
47. What is the difference in the sectorial angles of
(d) The volume of a rainfall in certain geographical area, (a) I and II (b) Only II (c) I and III (d) All of these the largest and the smallest sectors?
recorded every month for 24 consecutive months Directions (Q. Nos. 56-58) Consider the following
42. The cumulative frequency curve of a frequency (a) 90° (b) 85° (c) 82.5° (d) 77.5° frequency distribution
37. Prime numbers are the numbers which comes in distribution with 6 classes and total frequency
the table of 1 and itself only. 60 is a straight line. Consider the following Directions (Q. Nos. 48-49) The following table Class interval 1-4 4-7 7-10 10-13 13-16 16-19
I. The mean of first seven prime numbers is statements gives the frequency distribution of life length in Frequency 6 30 40 16 4 4
greater than their median. I. The first and the last classes have a frequency of hours of 100 electric bulbs having median life 20 h.
II. Mean is always greater than median. 10 each. Life of bulbs (in h) Number of bulbs
56. What is the median of the data?
II. Both the middle classes have a total frequency of 8-13 7 (a) 9.4 (b) 7.68 (c) 8.05 (d) 8.32
Select the correct option from the options given
30. 13-18 x 57. What is modal size of the given data?
below
III. The frequency distribution does not have a mode.
(a) Only I (b) Only II 18-23 40 (a) 8.14 (b) 7.88 (c) 7.62 (d) 8.48
(c) Both I and II (d) Neither I nor II Which of the statements given above are correct? 23-28 y
(a) I and II (b) I and III (c) II and III (d) All of these
58. The value of (Mean + Mode −2 Median) is equal to
28-33 10
38. Consider the following statements in respect of a (a) 0 (b) 0.2 (c) 0.4 (d) 0.1
43. State which of the following variables are 33-38 2
histogram
I. The histogram consists of vertical rectangular discrete? 48. What is the missing frequency of x?
bars with a common base such that there is no I. Number of children in a family. (a) 31 (b) 27 (c) 24 (d) 14
PREVIOUS YEARS’ QUESTIONS
gap between consecutive bars. II. Wages of workers.
II. The height of the rectangle is determined by the
49. What is the missing frequency of y? 59. Let the observations at hand be arranged in
III. The ages of students. (a) 27 (b) 24 (c) 14 (d) 11 increasing order. Which one of the following
frequency of the class it represents. IV. Weights of a set of a students. measures will not be affected when the smallest
Which of the statements given above is/are Select the correct answer using the codes given and the largest observations are removed?
Directions (Q. Nos. 50-52) The arithmetic mean,
correct? below e 2012 I
geometric mean and median of 6 positive numbers
(a) Only I (b) Only II (a) I and II (b) I, II and III (a) Mean (b) Median
7
(c) Both I and II (d) Neither I nor II (c) All of these (d) None of these a, a, b, b, c , c where a < b < c are , 2, 2 respectively. (c) Mode (d) Standard deviation
3
MATHEMATICS Statistics 375 376 CDS Pathfinder

60. Consider the following statements : 68. Consider the following statements : 77. The average of u, v, w, x, y and z is 10. What is What is the modal class? e 2014 II

I. The data collected by the investigator to be used I. A frequency distribution condenses the data and the average of u + 10, v + 20, w + 30, x + 40, y + 50 (a) 250-500 (b) 500-750
by himself are called primary data. reveals its important features. and z + 60 ? e 2013 II (c) 750-1000 (d) None of these
II. The data obtained from government agencies are II. A frequency distribution is an equivalent (a) 30 (b) 35 (c) 40 (d) 45 83. The geometric mean of ( x1 , x2 , x3 , ... , xn ) is X and
called secondary data. representation of original data.
78. The mean of the following distribution is 18. the geometric mean of ( y1 , y2 , y3 , ... , yn ) is Y .
Which of the statement(s) given above is/are Which of the statement(s) given above is/are Which of the following statement is/are correct?
Class interval Frequency
correct? e 2012 I correct? e 2013 I
(a) Only I (b) Only II 11-13 3 I. The geometric mean of
(a) Only I (b) Only II
(c) Both I and II (d) Neither I nor II 13-15 6 (x1y1 , x2y2, x3 y3 , ... , xn yn ) is XY .
(c) Both I and II (d) Neither I nor II
15-17 9 x x x x  X
61. Which one among the following statements is II. The geometric mean of  1 , 2 , 3 , ..., n  is .
correct? e 2012 I Directions (Q. Nos. 69-71) Read the following 17-19 13  y1 y2 y3 yn  Y
(a) Simple bar diagrams are those diagrams which information carefully to answer the questions that 19-21 f
Select the correct answer using the codes given
show two characteristics of the data follow. e 2013 I 21-23 5
below
(b) In pie diagrams, all the items are converted into In a frequency distribution having class intervals 0-10, 23-25 4
angles (a) Only I (b) Only II e 2014 II
10-20, 20-30 and 30-40 the respective frequencies are x,
What is the value of f ? e 2014 II (c) Both I and II (d) Neither I nor II
(c) A bar diagram is one in which data are shown in x + 8, x − 2 and x − 4 and the arithmetic mean of the
terms of bars (a) 8 (b) 9 (c) 10 (d) 11
distribution is 17.8. 84. Consider the following statements in respect of a
(d) Bar diagrams present data through length and breadth 79. Consider the following statements pertaining to discrete set of numbers.
69. The value of x is a frequency polygon of a frequency distribution
62. Consider the following distribution : I. The arithmetic mean uses all the data and is
(a) 3 (b) 6 (c) 8 (d) 12 of a continuous variable having seven class
Value of the variable 1 2 3 4 5 always uniquely defined.
70. The median lies in which one of the following intervals of equal width.
Frequency 3 f 6 5 3
class intervals? I. The original frequency distribution can be II. The median uses only one or two numbers from
reconstructed from the frequency polygon. the data and may not be unique.
For what value of f, is the arithmetic mean of (a) 0-10 (b) 10-20 (c) 20-30 (d) 30-40
the above distribution 3.1? e 2012 I II. The frequency polygon touches the X-axis in its Which of the statement(s) given above is/are
(a) 2 (b) 3 (c) 4 (d) 5
71. The modal class is correct? e 2014 II
extreme right and extreme left.
(a) 0-10 (b) 10-20 (c) 20-30 (d) 30-40
Which of the statement(s) given above is/are (a) Only I (b) Only II
63. Which of the following pair(s) is/are correctly
matched?
72. The mean of 7 observations is 7. If each observation correct? e 2014 II (c) Both I and II (d) Neither I nor II
is increased by 2, then the new mean is e 2013 I (a) Only I (b) Only II
I. Weight of a person : Continuous variable (a) 12 (b) 10 (c) 9 (d) 8 85. If xi ’s are the mid-points of the class intervals of
(c) Both I and II (d) Neither I nor II
II. Educational qualification of the person : Attribute grouped data, fi ’s are the corresponding
(a) Only I (b) Only II e 2012 II
73. There are 45 male and 15 female employees in 80.Consider the following in respect of variate which frequencies and x is the mean, then what is
(c) Both I and II (d) Neither I nor II an office. If the mean salary of the 60 employees takes values 2, 2, 2, 2, 7, 7, 7 and 7. Σfi ( xi − x ) equal to? e 2014 II
is ` 4800 and the mean salary of the male
64. The mean of 100 values is 45. If 15 is added to I. The median is equal to mean. (a) 0 (b) − 1 (c) 1 (d) 2
employees is ` 5000, then the mean salary of the
each of the first forty values and 5 is subtracted II. The mode is both 2 and 7.
female employees is e 2013 I 86. When we take class intervals on the X-axis and
from each of the remaining sixty values, then the Which of the statement(s) given above is/are corresponding frequencies on the Y-axis and
(a) ` 4200 (b) ` 4500 (c) ` 5600 (d) ` 6000
new mean becomes e 2012 II correct? e 2014 II
draw rectangles with the areas proportional to
(a) 45 (b) 48 (c) 51 (d) 55 (a) Only I (b) Only II
Directions (Q. Nos. 74-75) Read the following the frequencies of the respective class intervals,
(c) Both I and II (d) Neither I nor II
65. Which one of the following relations for the information carefully and answer the questions the graph so obtained is called e 2014 II

numbers 10, 7, 8, 5, 6, 8, 5, 8 and 6 is correct? given below. e 2013 II 81. Which of the following items of information is a (a) bar diagram (b) frequency curve
(a) Mean = Median (b) Mean = Mode e 2012 II The median of the following distribution is 14.4 and the good example of statistical data? e 2014 II (c) ogive (d) None of these
(c) Mean > Median (d) Mean > Mode (a) A table of logarithms of numbers
total frequency is 20. 87. Consider the following data:
(b) A list of names of 120 students of a class
66. In histogram the width of the bars is Class interval 0-6 6-12 12-18 18-24 24-30
(c) A list of annual incomes of the members of a club I. Number of complaints lodged due to road
proportional to e 2012 II Frequency 4 x 5 y 1 accidents in a state within a year for 5
(d) Holiday list of the offices of Government of India in
(a) Frequency (b) Number of classes the year 2013 consecutive years.
(c) Class interval (d) None of these 74. What is x equal to? II. Budgetary allocation of the total available funds
(a) 4 (b) 5 (c) 6 (d) 7 82. The following table gives ‘less than’ type to the various items of expenditure.
67. Which one of the following statements is correct? frequency distribution of income per day.
e 2013 I 75. What is the relation between x and y? Which of the above data is/are suitable for
Income (in `) less than Number of persons
(a) A frequency polygon is obtained by connecting the (a) 2 x = 3 y (b) 3x = 2 y (c) x = y (d) 2 x = y representation of a pie diagram? e 2014 II
corner points of the rectangles in a histogram 1500 100
(a) Only I (b) Only II
(b) A frequency polygon is obtained by connecting the
76. If m is the mean of p, q, r, s, t, u and v, then what 1250 80
is ( p − m ) + ( q − m ) + (r − m ) + ( s − m ) + ( t − m ) (c) Both I and II (d) Neither I nor II
mid-points of the tops of the rectangles in a 1000 70
histogram + ( u − m ) + ( v − m ) equal to? e 2013 II
750 55
88. Consider the following statements related to
(c) A frequency polygon is obtained by connecting the (a) 0 (b) s cumulative frequency polygon of a frequency
( p + v) 500 32
corner points of the class intervals in a histogram
(c) (d) None of these
distribution, the frequencies being cumulated
(d) None of the above 2 250 12 from the lower end of the range
MATHEMATICS Statistics 377 378 CDS Pathfinder

I. The cumulative frequency polygon gives an


equivalent representation of frequency
96. If a variable takes discrete values a + 4, a − 3. 5, ANSWERS
distribution table. a − 2.5, a − 3, a − 2, a + 0.5, a + 5 and a − 0. 5,
where a > 0, then the median of the data set is 1 c 2 b 3 b 4 c 5 a 6 b 7 c 8 c 9 d 10 b
II. The cumulative frequency polygon is a closed
e 2015 II 11 d 12 b 13 d 14 a 15 a 16 c 17 a 18 b 19 b 20 b
polygon with one horizontal and one vertical
(a) a − 2.5 (b) a − 1.25 (c) a − 1. 5 (d) a − 0 . 75 21 d 22 a 23 c 24 d 25 a 26 a 27 c 28 b 29 b 30 d
side. The other sides have non-negative slope.
Which of the statement(s) given above is/are
97. If each of n numbers xi = i ( i = 1, 2, 3, K , n ) is 31 c 32 a 33 d 34 b 35 b 36 d 37 c 38 c 39 c 40 b
correct? e 2014 II replaced by ( i + 1)xi , then the new mean is 41 b 42 b 43 b 44 b 45 c 46 b 47 c 48 b 49 c 50 b
(a) Only I (b) Only II e 2015 II 51 d 52 d 53 c 54 c 55 b 56 c 57 b 58 d 59 b 60 c
(c) Both I and II (d) Neither I nor II n+ 3 n (n + 1)
(a) (b) 61 b 62 b 63 a 64 b 65 a 66 c 67 b 68 c 69 d 70 b
2 2
89. The class which has maximum frequency is (n + 1) (n + 2 ) (n + 1) (n + 2 ) 71 b 72 c 73 a 74 a 75 b 76 a 77 d 78 a 79 a 80 c
(c) (d)
known as e 2014 II 3n 3 81. c 82 b 83 c 84 c 85 a 86 d 87 c 88 a 89 c 90 c
(a) median class (b) mean class
98. The weighted arithmetic mean of first 10 natural 91 a 92 b 93 c 94 d 95 b 96 b 97 d 98 d 99 b 100 a
(c) modal class (d) None of these
numbers whose weights are equal to the 101 b 102 c 103 d
90. Ten observations 6, 14, 15, 17, x + 1, 2x − 13, 30, corresponding numbers is equal to e 2015 II

32, 34 and 43 are written in ascending order. (a) 7 (b) 14 (c) 35 (d) 38 . 5
The median of the data is 24. What is the value 99. The election result in which six parties contested
of x? e 2014 II was depicted by a pie chart. Party A had an
(a) 15 (b) 18 (c) 20 (d) 24 angle 135° on this pie chart. If it secured 21960
votes, then how many valid votes in total were
HINTS AND SOLUTIONS
Directions (Q. Nos. 91-94) Consider the following cast? e 2016 I 1. (c) Frequency polygon can be drawn by Hence, the mean of first three 10. (b) Median is the middle item of the
frequency distribution. e 2015 I (a) 51240 (b) 58560 (c) 78320 (d) 87840 joining the mid-points of the respective observation is 9. series arranged in ascending or
tops in histogram. 6. (b) Total sum of items descending order.
Class Frequency 100. The mean and median of 5 observations are 9
and 8, respectively. If 1 is subtracted from each 2. (b) An ogive is used to determine the = 2 × 12 + 3 × 12 + 5 × 12 11. (d) Central angle for 540 marks = 360°.
0-10 4 = 24 + 36 + 60 = 120
observation, then the new mean and the new median. ∴ Central angle for 108 marks
10-20 5 Total sum of items of first two group
median will respectively be e 2016 I 3. (b) Let the lower limit be x. Then, the =
108
× 360° = 72°
20-30 7 (a) 8 and 7 (b) 9 and 7 upper limit of class interval = x + 10 = ( 2 + 3) × 3 = 15
540
30-40 10 (c) 8 and 9 Total sum of 5 items of group third
Q mid-value = 42 Here, the student scored 108 marks in
= 120 − 15 = 105
40-50 12 (d) Cannot be determined due to insufficient data x + ( x + 10) Social Science.
∴ = 42 ⇒ 2x + 10 = 84 105
50-60 8 2 ∴ Mean of third group = = 21
101. The age distribution of 40 children are as follows 5 12. (b) Given, n = 45,x = 2, n = 40
60-70 4 ⇒ 2x = 74 ⇒ x = 37 1 1 2
Age (in years) 5-6 6-7 7-8 8-9 9-10 10-11 ∴ Lower limit = 37 Hence, the mean of third group is 21. x = 25
. and n = 15, x = 2.
2 3 3
Number of children 4 7 9 12 6 2 Upper limit = 37 + 10 = 47 7. (c) Given, n = 7, x = 10 ∴ Required mean
91. What is the mean of the distribution? 1 1
4. (c) Lower class limits are obtained by and n = 3, x = 5 nx +nx +nx
(a) 37.2 (b) 38.1 (c) 39.2 (d) 40.1 Consider the following statements in respect of 2 2 = 1 1 2 2 3 3
subtracting 0.5 from the lower limit, so nx + nx n +n +n
92. What is the median class? the above frequency distribution clearly 9.5, 19.5, 29.5 and 39.5 are the Q Combined mean =
1 1 2 2 1 2 3

I. The median of the age distribution is 7 yr. actual lower class limits. n +n 45 × 2 + 40 × 2.5 + 15 × 2
(a) 20-30 (b) 30-40 1 2 =
(c) 40-50 (d) 50-60 II. 70% of the children are in the age group 6-9 yr. 5. (a) Here, sum of 5 observations 7 × 10 + 3 × 5 85 45 + 40 + 15
= = = 8 .5
7+3 10 90 + 100 + 30 220
93. What is the median of the distribution? III. The modal age of the children is 8 yr. = x + ( x + 2) + ( x + 4) + ( x + 6) = = = 2. 2
+ ( x + 8) Hence, the mean of combined set is 8.5. 45 + 40 + 15 100
(a) 37 (b) 38 (c) 39 (d) 40 Which of the above statements are correct?
e 2016 I = 5x + 20 8. (c) Given, total number of students = 50 Hence, the mean of the combined
94. What is the mode of the distribution? (a) I and II (b) II and III (c) I and III (d) I, II and III and total number of observation = 5 distribution is 2.2.
Total number of students passed
(a) 38.33 (b) 40.66
102. Suppose xi = λ i for 0 ≤ i ≤ 10, where λ > 1. Which ∴ Mean of total observations = 50 − 10 = 40 13. (d) Here,
(c) 42.66 (d) 43.33 n ( n + 1)
Sum of 5 observation Σf = ( 1 + 2 + 3+ ...+ n) =
one of the following is correct? e 2016 I = Total marks obtained by failed students i 2
95. There are five parties A, B, C , D and E in an (a) AM < Median (b) GM < Median Total number of observation
= 28 × 10 = 280  1
5x + 20 Σf × x =  1 × 1 + 2 ×
election. Out of total 100000 votes cast, 36000 (c) GM = Median (d) AM = Median = = 11 ⇒ 5x + 20 = 55 ∴ Total marks of 40 students
i i  2
+ 3 × + . . . + n × 
were cast to party A, 24000 to party B, 18000 5 1 1
103. Suppose xi =
1
for i = 1, 2, 3, ... , 11. Which one of = 2800 − 280 = 2520
to party C, 7000 to party D and rest to party E. ⇒ 5x = 55 − 20 = 35 ⇒ x = 7 3 n
i ∴ Required average of passed students
What angle will be allocated for party E in the ∴ Mean of first three = ( 1 + 1 + 1 + ... + n times) = n
pie chart? the following is not correct? e 2016 I 2520
e 2015 I x + ( x + 2) + ( x + 4) 3x + 6 = = 63 Σf x 2
(a) AM > 1 / 6 (b) GM >1 / 6 = = 40 ∴Mean = i i =
n
=
(a) 15° (b) 54° (c) 60° (d) 72° 3 3
(c) HM>1 / 6 (d) Median = HM Σf 1
n ( n + 1) n + 1
3 × 7 + 6 27 9. (d) Clearly, all the statements are true i
= = =9 (Q x = 7) 2
about median.
3 3
MATHEMATICS Statistics 379 380 CDS Pathfinder

2 + 3 + 7 + 1 + 3 + 2 + 3 21 1
14. (a) Geometric mean of 40, 50 and x = ( 40 × 50 × x)1 / 3 21. (d) Mean of A = = =3 y+ 38. (c) Since, the Since, median is 2 and the
7 7 y histogram 50 corresponding CF is 11.
⇒ ( 40 × 50 × x)1 / 3 = 10 (given) 30. (d) Here, mean, M = …(i)
1000 1 Mode of A = 3, having maximum frequency. 2 consists of 40 N

Frequency
⇒ 40 × 50 × x = 10 3
⇒ x= = So, must be greater than 9.
New mean vertical 30 2
40 × 50 2 Arrange data in ascending order, we have 1, 2, 2, 3, 3, 3, 7. 3 rectangular
7 + 1 1  1  1 16 + x
1
and median of A =  y3 + 3 y +  −3 y +  bars with a
20 ⇒ > 9 ⇒ x > ( 18 − 16) ⇒ x > 2
Hence, the value of x is .  th value = 4th  y  y 2
2  2  y common base 10
= = So, smallest possible value of x = 3.
15. (a) We know that the sum of deviation from their mean is zero. value = 3 2 2 such that 0
10 20 30 40 50 46. (b) Here, total expenditure
there is no
1   
2
∴ x f + x f + L + xn f n = 0 Here, mean, median and mode are equal.  1 Number
1 1 2 2  y +    y +  − 3 gap between = 6 + 7 + 3 + 4 + 25 . + 1.5
16. (c) The height of the bar is not proportional to the frequency of 22. (a) Total marks in 3 subjects = ( 75 + 80 + 85) = 240 out of 300.  y  

y  consecutive bars and also the height of = ` 24 lakh
the class. = [from Eq. (i)] the rectangle is determined by the
In any other subject the marks are atleast 0. 2 Sectorial angle of largest sector
frequency of the class it represents. 360°
17. (a) x f f×x Average of 4 subjects =
240 = M [( 2M ) − 3] = M( 4M 2 − 3)
2
= × 7 = 105°
= 60% Hence, both statements are correct in
0 4 0 4 31. (c) Since, maximum frequency is 80, 24
respect of a histogram.
1 f f ∴ Average cannot be less than 60%. hence mode will be between 15 to 20. 47. (c) Difference in the sectorial angles of
39. (c) We know that, histogram is an
2 9 18 23. (c) Given, mean = 8.9 and median = 9 1 equivalent graphical representation of the largest and the smallest sectors
32. (a) ` 25 = 1 cm 2 ⇒ ` 1 = cm 2 360°
3 g 3g Q Mode = (3 × Median) − (2 × Mean) 25 the frequency distribution and is suitable = 105° − × 1.5
81 for continuous random variables where 24
4 4 16 = (3 × 9 − 2 × 8.9) = 27 − 178
. = 9. 2 ∴ ` 81 = cm 2 = Area of square
25 the total frequency of an interval is = 105° − 225 . ° = 825 . °
Total Σf = 17 + f + g Σfx = 34 + f + 3g 24. (d) If every number of a finite set is increased by any number k, evenly distributed over the interval. N
Side of square =
81 9
= = 1.8 cm 48. (b) Here, l = 18, = 50 , c = 7 + x,
then measure of central tendency should also increase by k. 2
Σfx 34 + f + 3 g 34 + f + 3 g 25 5 Hence, both the given statements are
Q Mean = = ⇒2= (Qmean = 2) But geometric mean does not have this property. correct. f = 40 and h = 5
Σf 17 + f + g 17 + f + g 33. (d) Given, n = 5
25. (a) Given distribution is 1, 3, 5, 7, 9, x, 15, 17, 19, 21. 40. (b) Primary data is original research that  N − c
7 + 9 + 11 + 13 + 15 55  
⇒ 34 + 2 f + 2 g = 34 + f + 3 g ⇒ f − g = 0 Number of terms = 10 (even) Mean = = = 11  
…(i)
5 5
is obtained through first hand ∴ Median = l + 2 ×h
th term + Value of  + 1 th term
Given, Σf = 25 10 10 investigation. So, statements I and III f
Value of Σ | d | 2 = |7 − 11|2 + |9 − 11|2 + |11 − 11|2
2  2  are primary data. (50 − ( 7 + x))
∴ 17 + f + g = 25 ⇒ f + g = 25 − 17 ∴ Median = i
= 18 + ×5
2 + |13 − 11|2 + |15 − 11|2 = 40 41. (b) Collar sizes of 200 shirts sold in a
⇒ f + g =8 …(ii) 40
Value of 5th term + Value of 6th term Σd 2 40 week, here mode is a suitable measure of 5
On adding Eqs. (i) and (ii), we get 2 f = 8 ⇒ f = 4 10 = ∴σ = i = = 2 2 = 2 × 1. 41 ∴ 20 = 18 + ( 43 − x)
2 central tendency. 40
n 5
On putting f = 4 Eq. (i), we get 4 − g = 0 ⇒ g = 4 9+ x 42. (b) Since, the frequency in a straight line, [Q median = 20]
⇒ 10 = ⇒ 20 = 9 + x ⇒ x = 11 = 2.82
18. (b) For district A, as the class 44.0-47.0 has maximum frequency, 2 so we take all classes have equal ⇒ 800 = 720 + 215 − 5x
so it is the modal class. 34. (b) Number of cellphones sold by frequency, i.e. 10.
26. (a) Required class boundary = Lower class boundary of lowest 3 ⇒ 5x = 935 − 800
Motorola = × 45664 I. It is true that first and last class have
∴ l = 44, f = 59, f = 36, f = 30, and h = 3 class + Width of class × Number of class 16 135
1 0 2 10 frequency. ⇒ 5x = 135 ⇒ x = ⇒ x = 27
f − f 59 − 36 = 5.1 + 25
. × 10 = 30.1 5
∴ Mode = l + 1 0 × h = 44 + ×3 Central angle of Motorola II. Both the middle classes have
2f − f − f 2 × 59 − 36 − 30 27. (c) Arranging the observation in ascending order 2, 3, 4, 6, 7, 7, 8  3 × 45664  frequency, 10 + 10 = 20 Hence, the value of x is 27.
1 0 2
 
∴ Number of items = 7 × 360°  =  × 360° 
23 × 3 3 49. (c) We have,
= 44 + = 44 + 1.33 = 4533
. =  16 III. Since, all have equal frequency, so we
52 ∴ Median = 
n + 1  7 + 1 = 4th value = 6  45664   16  cannot determined the mode. 7 + x + 40 + y + 10 + 2 = 100
 th value =    
(Q
 2   2  43. (b) A discrete frequency distribution is x = 27)
For district B, as the class 47.0-50.0 has the maximum frequency,
So, median = 6 and mode of 2, 9, 3, 4, 9, 6, 9 is 9. = 67.5° such a distribution in which data are
so it is the modal class. ⇒86 + y = 100 ⇒ y = 100 − 86
1 presented in a way that exact
∴ l = 47, f = 54, f = 35, f = 41 and h = 3 ∴ Average of median and mode = ( 6 + 9) = 7.5 35. (b) If the population figures are given for
⇒ y = 14
1 0 2 2 each State of India, then data can be measurements of the units are clearly
f − f 54 − 35 classified as quantitative. shown. Clearly weights of a set of a Hence, the value of y is 14.
∴ Mode = l + × h = 47 +
1 0 ×3 28. (b) Let the mean of observations x , x , x , . . . , xn be x.
1 2 3 students is continuous, while other three
2f − f − f 2 × 54 − 35 − 41 On subtracting 10 from each, the mean would be ( x − 10).
36. (d) The volume of rainfall in certain Solutions (Q. Nos. 50-52) Since, a < b < c
1 0 2
geographical area, recorded every month are discrete.
19 × 3 Therefore, series is in increasing order
( x − 10) = 60% of x for 24 consecutive months. 44. (b) Since, modal size of given data is 1
= 47 + = 47 + 1.78 = 48.78 i.e. a , a , b , b , c , c
32 ⇒ ( x − 10) =
60 3
x = x ⇒  1 − 3  x = 10 37. (c) I. First seven prime numbers are 2, 3, and its corresponding frequency is 6.
  ∴ Median
Thus, the mode for district B is higher than the mode for 100 5  5 5, 7, 11, 13, 17. So, x cannot be equal to or more than 6.
2 + 3 + 5 + 7 + 11 + 13 + 17  6  th term +  6 + 1 th term
2 5 × 10    
district A. ⇒ x = 10 ⇒ x = = 25 Mean = Hence, the maximum value of x is 5.   2 
5 2 7 = 2
19. (b) Given, average score of 50 students = 44 58 45. (c) 2
Now, 5 is added to all the given observations, then new mean = = 8. 28 Number of goals Frequency CF 3rd term + 4th term
Total score = 44 × 50 = 2200 7 =
becomes = x + 5 = 25 + 5 = 30 0 3 3 2
7 + 1
Correct score of 50 students = ( 2200 − 73 + 23) = 2150
29. (b) Range of the data = 120 − 71 = 49 Median = Value of   th term ⇒ 2=
b+b
⇒b = 2
2150  2  1 6 9
∴ Correct average score = = 43 Range 49 2
50 ∴ Class size = = =7 = Value of 4th term = 7 2 2 11 Also, arithmetic mean
Number of classes 7
Since, prime numbers are always 3 x 11 + x a+ a+ b+ b+ c+ c
20. (b) In the less than type of ogive, the cumulative frequency is The class are 71-78, 78-85, 85-92, 92-97 etc. =
increasing, therefore here mean is always 4 5 16 + x 6
plotted against the upper limit of the concerned class interval.
So, limits of second class interval are 78 and 85. greater than median. 7 a+ b+ c
Total N = 16 + x ⇒ = ⇒ a+ c+ 2= 7
Hence, both statements are correct. 3 3
MATHEMATICS Statistics 381 382 CDS Pathfinder

⇒a + c= 7− 2=5 …(i) 60. (c) Primary data are those data which is collected by investigator 67. (b) A frequency polygon can be drawn joining the mid-points of 6−x 36 − 6x
⇒ 14. 4 − 12 = × 6 ⇒ 2.4 =
and geometric mean = ( a 2 × b 2 × c 2 )1 / 6 himself, while secondary data are those which is collected by other the respective tops of the rectangle of a histogram in the case of 5 5
8 persons. equal class intervals. ⇒ 12 = 36 − 6x ⇒ 6x = 24 ⇒ x = 4
⇒ 2 = ( abc )1 / 3 ⇒ abc = 8 ⇒ ac = = 4 [Q b = 2]…(ii) Hence, both statements are correct.
2 68. (c) A frequency distribution is a comprehensive way to classify raw 75. (b) We have, 4 + x + 5 + y + 1 = 20 ⇒ x + y = 10
∴ From Eqs. (i) and (ii), we get, a = 1, c = 4 and b = 2 61. (b) In pie diagrams, all the items are converted into angles. data of a quantitative variable. It shows how different values of a
⇒ 4 + y = 10 (Q x = 4) ⇒ y = 6
variable are distributed in different classes along with their
50. (b) Required sum = 2( a )2 + 2( b )2 + 2( c )2 62. (b) Given that, the arithmetic mean of the above distribution =3.1 x 4 2
corresponding class frequencies. It is an equivalent representation Now, = = ⇒ 3x = 2 y
= 2( 1)2 + 2( 2)2 + 2( 4)2 = 2 + 8 + 32 = 42 5 y 6 3
51. (d) The value of c is 4. ∑ xi f i x f +x f +x f +x f +x f
of original data.
Sum of the observations
i =1 1 1 2 2 3 3 4 4 69. (d)
f×x 76. (a) Q Mean =
52. (d) Q Mode = 3 (median) − 2 (mean) ⇒ = 3.1 ⇒ 5 5
= 3.1 CI x f Number of observations
5
f + f + f + f + f
= 3( 2) − 2   = 6 −
7 14 18 − 14 4
= = ∑ fi 1 2 3 4 5
0-10 5 x 5x

p+ q+ r+ s + t+u+ v
=m
3 3 3 3 i =1 10-20 15 x+ 8 15 ( x + 8) 7
70° ( 1)(3) + ( 2)( f ) + (3)( 6) + ( 4)(5) + (5)(3) 20-30 25 x−2 25 ( x − 2 ) ⇒ p + q + r + s + t + u + v = 7m …(i)
53. (c) Percentage of wheat production = × 100% = 19.44% ⇒ = 3.1
360° 3+ f + 6+ 5+ 3 30-40 35 x−4 35 ( x − 4) ∴ ( p − m) + ( q − m) + ( r − m) + ( s − m) + ( t − m)
Hence,the percentage of wheat production is 19.44%. 3 + 2 f + 18 + 20 + 15 Total Σf = 4x + 2 Σfx = 80x − 70
⇒ = 3.1 ⇒ 2 f + 56 = 3.1 f + 52.7 + (u − m ) + ( v − m )
9000 × 70 17 + f
54. (c) If 40° = 9000 kg ⇒ 70° = = 15750 kg Σfx 80x − 70 = ( p + q + r + s + t + u + v ) − 7m [from Eq. (i)]
40 Q Mean = . =
⇒ 178
⇒ 1. 1 f = 3.3 ⇒ f =3 Σf 4x + 2 = 7m − 7m = 0
Hence, the wheat production is 15750 kg. Sum of the observations
Hence, the value of f is 3. ⇒ . ( 4x + 2) = 80x − 70 ⇒ 712
178 . x + 35.6 = 80x − 70 77. (d) Q Average =
55. (b) 360° = 180000 kg Number of observations
63. (a) The weight of a person is continuous variable because the weight 105.6
Difference in Sugar and wheat angle = ( 70° − 40° ) = 30° ⇒ . x = 105.6
88 ⇒ x= = 12 u + v + w+ x+ y +z
of the persons can be divide into different but certain range of .
88 ⇒ = 10 (Q average = 10)
180000 × 30 6
∴ Required production = = 15000 kg intervals. But educational qualification of the person does not show
360 70. (b) Class interval (ci) 0-10 10-20 20-30 30-40 ⇒ u + v + w + x + y + z = 10 × 6 = 60 ...(i)
the attributes. Hence, statement I is true and statement II is false.
Hence, the difference in sugar and wheat production is 15000 kg. Frequencies ( f ) 12 20 10 8
64. (b) Let 100 value be x , x , x ,..., x Now, sum of new observations
1 2 3 100 12 32 42 50
Solutions (Q. Nos. 56-58) Cumulative frequency (cf ) u + 10 + v + 20 + w + 30 + x + 40 + y + 50 + z + 60
Sum of values x + x + x + L+ x
1 2 3 100
QMean = , 45 = N = (u + v + w + x + y + z ) + 10 + 20 + 30 + 40 + 50 + 60
Class interval fi cf xi fi xi Total number of values 100 Here, N = 50 ⇒ = 25
1-4 6 6 2.5 15.0 2 = 60 + 10 + 20 + 30 + 40 + 50 + 60 = 270 [from Eq. (i)]
⇒ x + x + x + L + x = 4500 …(i)
4-7 30 36 5.5 165.0 1 2 3 100 The frequency just greater than 25 is 32. Now, average of u + 10, v + 20, w + 30, x + 40, y + 50
Now, according to the question
7-10 40 76 8.5 340.0 So, median class is 10-20. 270
∴ Sum of new value and z + 60 = = 45
10-13 16 92 11.5 184.0 71. (b) Here, modal class is 10- 20, because it has maximum frequency 6
= ( x + 15 + x + 15 + L + x + 5) + x − 5 + x
13-16 4 96 14.5 58.0 1 2 40 41 42 i.e. 20. 78. (a) Class interval xi fi xi fi
−5 + L + x −5
16-19 4 100 17.5 70.0 100 72. (c) Given that, mean of 7 observations = 7 11-13 12 3 36
Sum of new value
Total N = 100 832.0 ∴ New mean = ⇒ New mean = Previous mean + Number added to each term 13-15 14 6 84
Number of values
N 100 ( x + x + x + L + x ) + 40 × 15 = 7+ 2 = 9 15-17 16 9 144
56. (c) We have, = = 50, so cf just greater than 50 is 76 which 1 2 3 40
17-19 18 13 234
2 2 73. (a) Given that, number of male employees (M) = 45
+ (x +x +L + x ) − 5 × 60 19-21 20 f 20 f
lies in class 7-10. =
41 42 100 Number of female employees (F) = 15
∴ Median class = 7 − 10, l = 7, f = 40, c = 36, h = 3 100 Mean salary of male employees (x ) = ` 5000 21-23 22 5 110
 N − c ( x + x + L + x ) + 600 − 300 M
1 2 100 4500 + 300 Total number of employees = ( M + F ) = 45 + 15 = 60
23-25 24 4 96
  = = [from Eq. (i)]
Median = l +  2  50 − 36  × 3 = 7 + 42 = 8.05 Total Σfi = 40 + f Σxi fi = 704 + 20 f
 ×h = 7+   100 100 Mean salary of total employees ( x ) = ` 4800
 f   40  40 4800 Here, Σf = 40 + f and Σxf = 704 + 20 f and mean, x = 18
MF
  = = 48 Let mean salary of female employees be x F . i i
100 Mx + F x 45 × 5000 + 15 × x Σx f 704 + 20 f
57. (b) The maximum frequency is 40, which lies in the class = 7 − 10 Hence, the new mean is 48. x = M F
⇒ 4800 = F ∴ Mean, x = i i ⇒ 18 =
(M + F ) 60 Σf 40 + f
∴ Modal class = 7 − 10 ⇒ l = 7, f = 40, f = 30, f = 16, h = 3 MF
i
1 0 2 65. (a) Given numbers are 10, 7, 8, 5, 6, 8, 5, 8 and 6.
 f − f  ⇒ 704 + 20 f = 720 + 18 f
Mode = l +  1 0  ×h First we arrange these numbers in ascending order. ⇒ 4800 × 60 − 45 × 5000 = 15 × x
2f − f − f  F
⇒ 20 f − 18 f = 720 − 704 ⇒ 2 f = 16 ⇒ f = 8
 1 0 2 5, 5, 6, 6, 7, 8, 8, 8, 10 ∴ x = 4800 × 4 − 3 × 5000 = 300 ( 16 × 4 − 50)
Total term, n = 9 Hence, the value of f is 8.
F
40 − 30 (odd) = 300 × 14 = ` 4200
= 7 +   × 3 = 7 + 30 = 7 + 088
 . = 788
. Now,
 2 × 40 − 30 − 16  34 Hence, the mean salary of female employees is ` 4200. 79. (a) Frequency polygon is formed by joining the mid-points of
5 + 5 + 6 + 6 + 7 + 8 + 8 + 8 + 10 63
(i) Mean = = =7 histogram. The original frequency distribution can be
Σf x 832 9 9  n − cf 
58. (d) Mean = i i = = 8.32   reconstructed from frequency polygon. Frequency polygon does
n + 1  9 + 1 th term = 5th term = 7 74. (a) Median = l +  2  ×h
Σf 100 (ii) Median =   th term =   not touch the X-axis in its extreme right and extreme left.
i  2   2   f 
∴ Mean + Mode − 2 Median = 832
. + 788
. − 2 × 8.05   Hence, only statement I is true.
= 16.2 − 16.1 = 0.1 (iii) Mode =8 (higher frequency term)  20 − ( 4 + x)  2 × 4 + 7 × 4 8 + 28
∴ Mean = Median 80. (c) I. Mean of all observations = = = 4 .5
  10 − 4 − x
59. (b) In an increasing order arrangements of observations, the ⇒ 14.4 = 12 +  2  × 6 = 12 + ×6 8 8
66. (c) In histogram the width of the bars is proportional to class 5 5 For median, first we arrange the given observations in
median will not be affected when the smallest and the largest  
observations are removed. interval.   ascending order = 2, 2, 2, 2, 7, 7, 7, 7, n=8 (even)
MATHEMATICS Statistics 383 384 CDS Pathfinder

th term +  + 1 th term Here, n = 10 96. (b) We have, discrete values a + 4 , a − 35,
. a − 25,
n n (even) N
. I. Here, N = 40 ⇒ = 20
2  2
∴ Median = 2 Value of   th term + Value of  + 1 th term a − 3, a − 2, a + 0.5, a + 5 and a − 0.5
n n
2  2 2  Thus, 7 -8 is the median class.
∴ Median = Now, ascending order is a − 35,
. a − 3, a − 25,
. a − 2,
th term +  + 1 th term
8 8 2 N
2  4th + 5th 2 + 7 a − 05,
. a + 05,. a + 4, a + 5. ∴ l = 7, f = 9 , C = 11, = 20 and h = 1
Value of   th term + Value of  + 1 th term
= 2 = = = 4 .5 10 10
a − 2 + a − 0.5 2a − 25
. 2
2 2 2  2  2  ∴ Median = = = a − 1.25
= 2 2 N
−C
II. Mode is both 2 and 7, since frequency of occurrence is same i.e. 2
Value of 5th term + Value of 6th term 97. (d) We have, x = i Median = l + 2 ×h
maximum frequency. = i f
81. (c) A list of annual incomes of the members of a club is a good 2 2x + 3x + 4x + K+ ( n + 1)x n
x + 1 + 2x − 13 3x − 12 Mean = 1 2 3 20 − 11
example of statistical data. = = = 7+ ×1
n 9
2 2
82. (b) 2 × 1 + 3 × 2 + 4 × 3+ K+ ( n + 1) × n
Income Class Number of
Frequency 3x − 12 = = 7 + 1 = 8 yr
less than interval persons But given, median = 24 ⇒ = 24 n
2 Hence, statement I is incorrect.
1500 1250-1500 100 20 n ( n + 1)( 2n + 1) n ( n + 1)
⇒ 3x − 12 = 24 × 2 = 48 ⇒ 3x = 48 + 12 +
Σn ( n + 1) Σn2 + Σn 6 2 II. Total number of children (N) = 40
1250 1000-1250 80 10
⇒ 3x = 60 ⇒ x = 20 = = =
1000 750-1000 70 15 n n n Number of children in the age group 6-9 = 7 + 9 + 12 = 28
Hence, the value of x is 20. n ( n + 1)  2n + 1  ( n + 1)( n + 2)
750 500-750 55 23 = + 1 = 28
Solutions (Q. Nos. 91-94)  3  ∴ Required percentage = × 100% = 70%
500 250-500 32 20 2n 3 40
1 × 1 + 2 × 2 + 3 × 3+ K+10 × 10
250 0-250 12 12 Class Frequency Class mark f(x ) cf 98. (d) Weighted AM = Hence, statement II is correct.
interval (f) (x) 10
III. Q Modal group = 8 - 9,
Here, maximum frequency is 23. 0-10 4 5 20 4 10 × 11 × 21
So, the modal class is 500-750. 6 21 × 11 77 ∴ Modal age = 8 yr
10-20 5 15 75 9 = = = = 38.5
83. (c) Geometric mean of 20-30 7 25 175 16 10 6 2 Since, 9 is not included in this group.
( x , x , x , L , xn ) = ( x ⋅ x L xn )1 / n = X 30-40 10 35 350 26 99. (b) Let the total votes be x. Hence, statement III is correct.
1 2 3 1 2
and geometric mean of ( y , y , y , ⋅ ⋅ ⋅ , y n ) 40-50 12 45 540 38 360°
1 2 3 Then, central angle of party A = × 21960 102. (c) We have 11 observations as follow
50-60 8 55 440 46 x
= ( y ⋅ y ⋅ ⋅ ⋅ yn ) 1/n
=Y 1, λ , λ2 , λ3 , λ4 , λ5 , λ6 , λ7 , λ8 , λ9 , λ10
1 2 360° 360° × 21960
∴ Geometric mean of ( x y , x y , K , xn y n )
60-70 4 65 260 50 ⇒ 135° = × 21960 ⇒ x = = 58560
1 1 2 2 x 135° Here, number of observations = 11 (odd)
Total 50 1860
11 +
Median = 
= ( x y ⋅ x y ⋅ ⋅ ⋅ xn y n ) 1/n
Sum of 5 observations 1
 th term = 6th term = λ
5
1 1 2 2 100. (a) Q = 9 …(i)
Σfx 1860 Number of observations  2 
= ( x ⋅ x ⋅ ⋅ ⋅ xn )1 / n × ( y ⋅ y ⋅ ⋅ ⋅ y n )1 / n = XY 91. (a) Mean = = = 37. 2
1 2 1 2
1/n Σf 50 ⇒ Sum of 5 observations = 9 × 5 = 45 1 + λ + λ2 +…+ λ10  λ11 − 1 
 x1 x x   x 1 x2 x  Hence, the value of mean is 37.2. ∴ AM = = 
Geometric mean of  , 2 , K, n  =  ⋅ ⋅ ⋅⋅ n  If 1 is subtracted from each observation, then 11  11( λ − 1) 
y y y n   y y y n  N 50 Sum of 5 observations − 5
 1 2 1 2 92. (b) Here, N = 50, Now, = = 25 New mean of 5 observations = 1
2 2
( x ⋅ x ⋅ ⋅ ⋅ xn )1 / n X 5 and GM = ( 1⋅ λ ⋅ λ2 ⋅ λ3 … λ10 ) 11 = ( λ1 + 2 + 3 + … + 10 )1 / 11
=
1 2
= which lies in the cumulative frequency corresponding class 45 − 5
( y ⋅ y ⋅ ⋅ ⋅ y n )1 / n Y interval for cf 26 is 30-40. = =8
1 2 5 = ( λ )55 /11 = λ5 …(ii)
From the table, l = 30, f = 10, C = 16 and h = 10 5 + 1
Median of 5 observations = 
93. (c)
84. (c) Arithmetic mean uses all the data and is always uniquely  th term = 3rd term = 8 From Eqs. (i) and (ii), we get GM = Median
 N −C   2 
defined. Median uses only one or two numbers from the data and  
n + 1   103. (d) We have, 11 observations as follows
may not be uniques.   th term for odd n and half the sum of ∴ Median = l + 2 ×h If 1 is subtracted from each observation, then
1 1 1 1 1
 2  f New median = 8 − 1 = 7 1, , , , , …,
 n + 1 th and nth terms for even n. ( 25 − 16) 2 3 4 5 11 (odd)
  = 30 + × 10 = 30 + 9 = 39 Hence, the new mean and median are 8 and 7, respectively.
2  11 + 1
∴Median = 
2 10 1
101. (b)  th term = 6th term = …(i)
85. (a) If xi ’s are the mid-points of the class intervals of grouped 94. (d) Modal class of the given data is 40-50, because it has largest Age (in years)
Number of Cumulative  2  6
children frequency
data, f ’s are the corresponding frequencies and x is the mean, frequency among the given classes of the data i.e. 12. 1 1 1 1 1 1 1 
i 5-6 4 4 and =  + + + +…+ 
then Σ f ( x − x ) = 0 Here, l = 40, f = 12 , f = 10, f = 8 and h = 10 HM 11  1 1 / 2 1 / 3 1 / 4 1 / 11
i i 1 0 2 6-7 7 11
86. (d) When we take class intervals on the X-axis and corresponding
 f − f  12 − 10 1 1
∴ Mode = l +    × 10 7-8 9 20 ⇒ = (1 + 2 + 3 + … +11 )
 × h = 40 + 
1 0
frequencies on the Y-axis and draw rectangles with the areas  HM 11
proportional to the frequencies of the respective class intervals, the  2 f 1 − f 0 − f 2   2 × 12 − 10 − 8  8-9 12 32
1 66 1
graph so obtained is called histogram. 2 × 10 20 9-10 6 38 ⇒ = ⇒ HM =
= 40 + = 40 + = 40 + 333
. = 43.33 HM 11 6
87. (c) Both statements I and II are suitable for representation of a pie 24 − 18 6 10-11 2 40
diagram. Total N = 40 ∴ HM = Median [from Eq. (i)]
Hence, the mode of given data is 43.33.
88. (a) Here, statement I is correct but statement II is not correct. 95. (b) Given, total number of votes = 100000
89. (c) The class which has maximum frequency is known as modal Party E get votes = 100000 − (36000 + 24000 + 18000 + 7000)
class. = 100000 − 85000 = 15000
90. (c) Given observations in ascending order are 15000
∴ Angle allocated for party E = × 360° = 54°
6, 14, 15, 17, x + 1, 2x − 13, 30, 32, 34 and 43. 100000

You might also like